Hadzic's Textbook of Regional Anesthesia and Acute Pain Management: Self-Assessment and Review [1 ed.] 9781260142716, 126014271X, 9781260142709, 1260142701

Questions and answers for thorough understanding of the latest advances in regional anesthesia and pain management Reg

2,576 401 35MB

English Pages [449] Year 2019

Report DMCA / Copyright

DOWNLOAD FILE

Polecaj historie

Hadzic's Textbook of Regional Anesthesia and Acute Pain Management: Self-Assessment and Review [1 ed.]
 9781260142716, 126014271X, 9781260142709, 1260142701

  • Commentary
  • TRUE PDF

Table of contents :
Cover
Title Page
Copyright Page
Dedication
Contents
Contributors
Preface
Acknowledgments
PART 1 History
1 The History of Local Anesthesia
PART 2 Foundations of Local and Regional Anesthesia
SECTION 1 ANATOMY AND HISTOLOGY OF PERIPHERAL NERVOUS SYSTEM AND NEURAXIS
2 Functional Regional Anesthesia Anatomy
3 Histology of the Peripheral Nerves and Light Microscopy
4 Connective Tissues of Peripheral Nerves
5 Ultrastructural Anatomy of the Spinal Meninges and Related Structures
SECTION 2 PHARMACOLOGY
6 Clinical Pharmacology of Local Anesthetics
7 Controlled-Release Local Anesthetics
8 Analgesic Adjuvants in the Peripheral Nervous System
9 Local Anesthetic Mixtures for Peripheral Nerve Blocks
10 Continuous Peripheral Nerve Blocks: Local Anesthetic Solutions and Infusion Strategies
SECTION 3 EQUIPMENT FOR PERIPHERAL NERVE BLOCKS
11 Equipment for Regional Anesthesia
12 Equipment for Continuous Peripheral Nerve Blocks
13 Electrical Nerve Stimulators and Localization of Peripheral Nerves
SECTION 4 PATIENT MANAGEMENT CONSIDERATIONS
14 Developing Regional Anesthesia Pathways
15 Infection Control in Regional Anesthesia
16 Local Anesthetics, Regional Anesthesia, and Cancer Recurrence
17 Perioperative Regional Anesthesia and Analgesia: Effects on Cancer Recurrence and Survival After Oncological Surgery
PART 3 Clinical Practice of Regional Anesthesia
PART 3A Local and Infiltrational Anesthesia
18 Intra-articular and Periarticular Infiltration of Local Anesthetics
19 Regional and Topical Anesthesia for Awake Endotracheal Intubation
PART 3B Intravenous Regional Block for Upper and Lower Extremity
20 Intravenous Regional Block for Upper and Lower Extremity Surgery
PART 3C Neuraxial Anesthesia
SECTION 1 SPINAL ANESTHESIA
21 Neuraxial Anatomy (Anatomy Relevant to Neuraxial Anesthesia)
22 Spinal Anesthesia
22A Mechanisms and Management of Failed Spinal Anesthesia
SECTION 2 EPIDURAL ANESTHESIA
23 Epidural Anesthesia and Analgesia
SECTION 3 CAUDAL ANESTHESIA
24 Caudal Anesthesia
SECTION 4 COMBINED SPINAL AND EPIDURAL ANESTHESIA
25 Combined Spinal-Epidural Anesthesia
SECTION 5 POSTDURAL PUNCTURE HEADACHE
26 Postdural Puncture Headache
PART 3D Ultrasound-Guided Nerve Blocks
SECTION 1 FUNDAMENTALS OF ULTRASOUND-GUIDED REGIONAL ANESTHESIA
27 Physics of Ultrasound
28 Optimizing an Ultrasound Image
29 Introduction to Ultrasound-Guided Regional Anesthesia
SECTION 2 ULTRASOUND-GUIDED HEAD AND NECK NERVE BLOCKS
30 Nerve Blocks of the Face
SECTION 3 ULTRASOUND-GUIDED NERVE BLOCKS FOR THE UPPER EXTREMITY
31A Ultrasound-Guided Cervical Plexus Block
31B Ultrasound-Guided Interscalene Brachial Plexus Block
31C Ultrasound-Guided Supraclavicular Brachial Plexus Block
31D Ultrasound-Guided Infraclavicular Brachial Plexus Block
31E Ultrasound-Guided Axillary Brachial Plexus Block
31F Ultrasound-Guided Blocks at the Elbow
31G Ultrasound-Guided Wrist Block
SECTION 4 ULTRASOUND-GUIDED NERVE BLOCKS FOR THE LOWER EXTREMITY
32A Ultrasound-Guided Femoral Nerve Block
32B Ultrasound-Guided Fascia Iliaca Block
32C Ultrasound-Guided Lateral Femoral Cutaneous Nerve Block
32D Ultrasound-Guided Obturator Nerve Block
32E Ultrasound-Guided Saphenous (Subsartorius/Adductor Canal) Nerve Block
32F Ultrasound-Guided Sciatic Nerve Block
32G Ultrasound-Guided Popliteal Sciatic Block
32H Ultrasound-Guided Ankle Block
SECTION 5 ULTRASOUND-GUIDED NERVE BLOCKS FOR ABDOMINAL AND THORACIC WALL
33 Ultrasound-Guided Transversus Abdominis Plane and Quadratus Lumborum Blocks
34 Pectoralis and Serratus Plane Blocks
PART 3E Local and Regional Anesthesia for Oral and Maxillofacial Surgery
35 Oral and Maxillofacial Regional Anesthesia
PART 3F Local and Regional Anesthesia for the Eye
36 Local and Regional Anesthesia for Ophthalmic Surgery
PART 4 Ultrasound Imaging of Neuraxial and Perivertebral Space
37 Sonography of the Lumbar Paravertebral Space and Considerations for Ultrasound-Guided Lumbar Plexus Block
38 Lumbar Paravertebral Sonography and Considerations for Ultrasound-Guided Lumbar Plexus Block
39 Spinal Sonography and Applications of Ultrasound for Central Neuraxial Blocks
PART 5 Obstetric Anesthesia
40 Obstetric Regional Anesthesia
PART 6 Pediatric Anesthesia
41 Regional Anesthesia in Pediatric Patients: General Considerations
42 Pediatric Epidural and Spinal Anesthesia and Analgesia
43 Peripheral Nerve Blocks for Children
44 Acute and Chronic Pain Management in Children
PART 7 Anesthesia in Patients with Specific Considerations
45 Perioperative Regional Anesthesia in the Elderly
46 Regional Anesthesia and Cardiovascular Disease
47 Regional Anesthesia and Systemic Disease
48 Regional Anesthesia in the Patient with Preexisting Neurologic Disease
49 Acute Compartment Syndrome of the Limb: Implications for Regional Anesthesia
50 Peripheral Nerve Blocks for Outpatient Surgery
51 Neuraxial Anesthesia and Peripheral Nerve Blocks in Patients on Anticoagulants
52 Regional Analgesia in the Critically Ill
53 Acute Pain Management in the Opioid-Dependent Patient
54 Regional Anesthesia in Patients with Trauma
55 Regional Anesthesia for Cardiac and Thoracic Anesthesia
56 Regional Anesthesia in Austere Environment Medicine
57 Anesthesia for Humanitarian Relief Operations
PART 8 Regional Anesthesia in the Emergency Department
58 Regional Anesthesia and Acute Pain Management in the Emergency Department
PART 9 Complications of Local and Regional Anesthesia
59 Complications and Prevention of Neurologic Injury with Peripheral Nerve Blocks
60 Assessment of Neurologic Complications of Regional Anesthesia
61 Perioperative Nerve Injury Unrelated to Nerve Blockade
62 Monitoring, Documentation, and Consent for Regional Anesthesia Procedures
63 Diagnosis and Management of Spinal and Peripheral Nerve Hematoma
PART 10 LAST: Local Anesthetic Systemic Toxicity
64 Local Anesthetic Systemic Toxicity
PART 11 Perioperative Outcome and Economics of Regional Anesthesia
65 Regional Anesthesia, Cost, Operating Room, and Personnel Management
66 Regional Anesthesia and Perioperative Outcome
67 The Effects of Regional Anesthesia on Functional Outcome After Surgery
PART 12 Acute Pain Management
68 Intravenous Patient-Controlled Analgesia
69 Continuous Peripheral Nerve Blocks
70 Organization of an Acute Pain Management Service Incorporating Regional Anesthesia Techniques
71 Multimodal Analgesia: Pharmacologic Interventions and Prevention of Persistent Postoperative Pain
72 The Role of Nonopioid Analgesic Infusions in the Management of Postoperative Pain
PART 13 Education in Regional Anesthesia
73 Teaching Regional Anesthesia
PART 14 Statistics and Principles of Research Design in Regional Anesthesia and Acute Pain Medicine
74 Principles of Statistical Methods for Research in Regional Anesthesia
Index
A
B
C
D
E
F
G
H
I
J
K
L
M
N
O
P
Q
R
S
T
U
V
W

Citation preview

Hadzic’s Textbook of Regional Anesthesia and Acute Pain Management Self-Assessment and Review

Hadzic_FM_p00i-pxx.indd 1

22/04/19 6:40 PM

NOTICE Medicine is an ever-changing science. As new research and clinical experience broaden our knowledge, changes in treatment and drug therapy are required. The authors and the publisher of this work have checked with sources believed to be reliable in their efforts to provide information that is complete and generally in accord with the standards accepted at the time of publication. However, in view of the possibility of human error or changes in medical sciences, neither the authors nor the publisher nor any other party who has been involved in the preparation or publication of this work warrants that the information contained herein is in every respect accurate or complete, and they disclaim all responsibility for any errors or omissions or for the results obtained from use of the information contained in this work. Readers are encouraged to confirm the information contained herein with other sources. For example and in particular, readers are advised to check the product information sheet included in the package of each drug they plan to administer to be certain that the information contained in this work is accurate and that changes have not been made in the recommended dose or in the contraindications for administration. This recommendation is of particular importance in connection with new or infrequently used drugs.

Hadzic_FM_p00i-pxx.indd 2

22/04/19 6:40 PM

Hadzic’s Textbook of Regional Anesthesia and Acute Pain Management Self-Assessment and Review Editor

Admir Hadzic, MD, PhD Professor of Anesthesiology Consultant, Anesthesiology, Intensive Care, Emergency Medicine and Pain Therapy Ziekenhuis Oost-Limburg Genk, Belgium Director, NYSORA, The New York School of Regional Anesthesia New York, New York Assistant Editor

Angela Lucia Balocco, MD Anesthesiologist Research Fellow NYSORA, The New York School of Regional Anesthesia New York, New York

New York Chicago San Francisco Athens London Madrid Mexico City Milan New Delhi Singapore Sydney Toronto

Hadzic_FM_p00i-pxx.indd 3

22/04/19 6:40 PM

Copyright © 2019 by McGraw-Hill Education. All rights reserved. Except as permitted under the United States Copyright Act of 1976, no part of this publication may be reproduced or distributed in any form or by any means, or stored in a database or retrieval system, without the prior written permission of the publisher. ISBN: 978-1-26-014271-6 MHID: 1-26-014271-X The material in this eBook also appears in the print version of this title: ISBN: 978-1-26-014270-9, MHID: 1-26-014270-1. eBook conversion by codeMantra Version 1.0 All trademarks are trademarks of their respective owners. Rather than put a trademark symbol after every occurrence of a trademarked name, we use names in an editorial fashion only, and to the benefit of the trademark owner, with no intention of infringement of the trademark. Where such designations appear in this book, they have been printed with initial caps. McGraw-Hill Education eBooks are available at special quantity discounts to use as premiums and sales promotions or for use in corporate training programs. To contact a representative, please visit the Contact Us page at www.mhprofessional.com. TERMS OF USE This is a copyrighted work and McGraw-Hill Education and its licensors reserve all rights in and to the work. Use of this work is subject to these terms. Except as permitted under the Copyright Act of 1976 and the right to store and retrieve one copy of the work, you may not decompile, disassemble, reverse engineer, reproduce, modify, create derivative works based upon, transmit, distribute, disseminate, sell, publish or sublicense the work or any part of it without McGraw-Hill Education’s prior consent. You may use the work for your own noncommercial and personal use; any other use of the work is strictly prohibited. Your right to use the work may be terminated if you fail to comply with these terms. THE WORK IS PROVIDED “AS IS.” McGRAW-HILL EDUCATION AND ITS LICENSORS MAKE NO GUARANTEES OR WARRANTIES AS TO THE ACCURACY, ADEQUACY OR COMPLETENESS OF OR RESULTS TO BE OBTAINED FROM USING THE WORK, INCLUDING ANY INFORMATION THAT CAN BE ACCESSED THROUGH THE WORK VIA HYPERLINK OR OTHERWISE, AND EXPRESSLY DISCLAIM ANY WARRANTY, EXPRESS OR IMPLIED, INCLUDING BUT NOT LIMITED TO IMPLIED WARRANTIES OF MERCHANTABILITY OR FITNESS FOR A PARTICULAR PURPOSE. McGraw-Hill Education and its licensors do not warrant or guarantee that the functions contained in the work will meet your requirements or that its operation will be uninterrupted or error free. Neither McGraw-Hill Education nor its licensors shall be liable to you or anyone else for any inaccuracy, error or omission, regardless of cause, in the work or for any damages resulting therefrom. McGraw-Hill Education has no responsibility for the content of any information accessed through the work. Under no circumstances shall McGraw-Hill Education and/or its licensors be liable for any indirect, incidental, special, punitive, consequential or similar damages that result from the use of or inability to use the work, even if any of them has been advised of the possibility of such damages. This limitation of liability shall apply to any claim or cause whatsoever whether such claim or cause arises in contract, tort or otherwise.

This book is dedicated to all students of anesthesiology and regional anesthesia and acute pain medicine.

Hadzic_FM_p00i-pxx.indd 5

22/04/19 6:40 PM

This page intentionally left blank

Hadzic_FM_p00i-pxx.indd 6

22/04/19 6:40 PM

Contents Contributors............................................................................................ xi Preface...................................................................................................xvii Acknowledgments.............................................................................. xix

PART 1

History

1

1 The History of Local Anesthesia.................................... 3

PART 2 Foundations of Local and Regional Anesthesia

SECTION 1 A NATOMY AND HISTOLOGY OF PERIPHERAL NERVOUS SYSTEM AND NEURAXIS

5

5

2 Functional Regional Anesthesia Anatomy.................. 7 3 Histology of the Peripheral Nerves and Light Microscopy............................................................11 4 Connective Tissues of Peripheral Nerves...................15 5 Ultrastructural Anatomy of the Spinal Meninges and Related Structures...............................17

SECTION 2 PHARMACOLOGY 21 6 Clinical Pharmacology of Local Anesthetics.............23 7 Controlled-Release Local Anesthetics .......................27 8 Analgesic Adjuvants in the Peripheral Nervous System..............................................................31 9 Local Anesthetic Mixtures for Peripheral Nerve Blocks....................................................................35 10 Continuous Peripheral Nerve Blocks: Local Anesthetic Solutions and Infusion Strategies...........37

SECTION 3 E QUIPMENT FOR PERIPHERAL NERVE BLOCKS

PART 3 Clinical Practice of Regional Anesthesia

75

PART 3A Local and Infiltrational Anesthesia

75

18 Intra-articular and Periarticular Infiltration of Local Anesthetics ......................................................77 19 Regional and Topical Anesthesia for Awake Endotracheal Intubation .................................81

PART 3B Intravenous Regional Block for Upper and Lower Extremity 85 20 Intravenous Regional Block for Upper and Lower Extremity Surgery .....................................87

PART 3C Neuraxial Anesthesia

SECTION 1 SPINAL ANESTHESIA

41

57

91

91

21 Neuraxial Anatomy (Anatomy Relevant to Neuraxial Anesthesia) ..............................................93 22 Spinal Anesthesia ..........................................................99 22A Mechanisms and Management of Failed Spinal Anesthesia ............................................103

SECTION 2 EPIDURAL ANESTHESIA

11 Equipment for Regional Anesthesia ..........................43 12 Equipment for Continuous Peripheral Nerve Blocks ...................................................................49 13 Electrical Nerve Stimulators and Localization of Peripheral Nerves .....................................................53

SECTION 4 PATIENT MANAGEMENT CONSIDERATIONS

16 Local Anesthetics, Regional Anesthesia, and Cancer Recurrence .................................................65 17 Perioperative Regional Anesthesia and Analgesia: Effects on Cancer Recurrence and Survival After Oncological Surgery ............................71

107

23 Epidural Anesthesia and Analgesia .........................109

SECTION 3 CAUDAL ANESTHESIA 24

121

Caudal Anesthesia .......................................................123

SECTION 4 C OMBINED SPINAL AND EPIDURAL ANESTHESIA 127 25 Combined Spinal-Epidural Anesthesia....................129

14 Developing Regional Anesthesia Pathways .............59 15 Infection Control in Regional Anesthesia .................63

Hadzic_FM_p00i-pxx.indd 7

22/04/19 6:41 PM

viii

Contents

SECTION 5 POSTDURAL PUNCTURE HEADACHE 135 26 Postdural Puncture Headache ..................................137

PART 3D Ultrasound-Guided Nerve Blocks

35 Oral and Maxillofacial Regional Anesthesia ...........233

141

SECTION 1 F UNDAMENTALS OF ULTRASOUNDGUIDED REGIONAL ANESTHESIA 141 27 Physics of Ultrasound .................................................143 28 Optimizing an Ultrasound Image .............................147 29 Introduction to Ultrasound-Guided Regional Anesthesia ...................................................151

SECTION 2 ULTRASOUND-GUIDED HEAD AND NECK NERVE BLOCKS

155

30 Nerve Blocks of the Face ............................................157

SECTION 3 U LTRASOUND-GUIDED NERVE BLOCKS FOR THE UPPER EXTREMITY 161 1A Ultrasound-Guided Cervical Plexus Block ..............163 3 31B Ultrasound-Guided Interscalene Brachial Plexus Block...................................................167 31C Ultrasound-Guided Supraclavicular Brachial Plexus Block ..................................................169 31D Ultrasound-Guided Infraclavicular Brachial Plexus Block ..................................................173 31E Ultrasound-Guided Axillary Brachial Plexus Block ..................................................................177 31F Ultrasound-Guided Blocks at the Elbow .................181 31G Ultrasound-Guided Wrist Block ................................185

SECTION 4 ULTRASOUND-GUIDED NERVE BLOCKS FOR THE LOWER EXTREMITY 187 2A Ultrasound-Guided Femoral Nerve Block ...............189 3 32B Ultrasound-Guided Fascia Iliaca Block ....................195 32C Ultrasound-Guided Lateral Femoral Cutaneous Nerve Block ..............................................201 32D Ultrasound-Guided Obturator Nerve Block ...........203 32E Ultrasound-Guided Saphenous (Subsartorius/Adductor Canal) Nerve Block ..........205 32F Ultrasound-Guided Sciatic Nerve Block ..................207 32G Ultrasound-Guided Popliteal Sciatic Block .............213 32H Ultrasound-Guided Ankle Block ...............................215

SECTION 5 ULTRASOUND-GUIDED NERVE BLOCKS FOR ABDOMINAL AND THORACIC WALL

217

33 Ultrasound-Guided Transversus Abdominis Plane and Quadratus Lumborum Blocks ................219 34 Pectoralis and Serratus Plane Blocks .......................225

Hadzic_FM_p00i-pxx.indd 8

PART 3E Local and Regional Anesthesia for Oral and Maxillofacial Surgery 231 PART 3F Local and Regional Anesthesia for the Eye 237 36 Local and Regional Anesthesia for Ophthalmic Surgery ....................................................239

PART 4 Ultrasound Imaging of Neuraxial and Perivertebral Space 243 37 Sonography of the Lumbar Paravertebral Space and Considerations for Ultrasound-Guided Lumbar Plexus Block ...................................................245 38 Lumbar Paravertebral Sonography and Considerations for Ultrasound-Guided Lumbar Plexus Block ...................................................249 39 Spinal Sonography and Applications of Ultrasound for Central Neuraxial Blocks .................255

PART 5

Obstetric Anesthesia

261

40 Obstetric Regional Anesthesia ..................................263

PART 6

Pediatric Anesthesia

271

41 Regional Anesthesia in Pediatric Patients: General Considerations ..............................................273 42 Pediatric Epidural and Spinal Anesthesia and Analgesia ...............................................................277 43 Peripheral Nerve Blocks for Children .......................283 44 Acute and Chronic Pain Management in Children ....................................................................285

PART 7

Anesthesia in Patients with Specific Considerations

287

45 Perioperative Regional Anesthesia in the Elderly ................................................................289 46 Regional Anesthesia and Cardiovascular Disease ..........................................................................295 47 Regional Anesthesia and Systemic Disease ...........299 48 Regional Anesthesia in the Patient with Preexisting Neurologic Disease .......................303 49 Acute Compartment Syndrome of the Limb: Implications for Regional Anesthesia ......................307 50 Peripheral Nerve Blocks for Outpatient Surgery .....................................................309 51 Neuraxial Anesthesia and Peripheral Nerve Blocks in Patients on Anticoagulants ..........313 52 Regional Analgesia in the Critically Ill .....................317 53 Acute Pain Management in the Opioid-Dependent Patient ........................................319 54 Regional Anesthesia in Patients with Trauma ..................................................................325

22/04/19 6:41 PM

Contents

ix

55 Regional Anesthesia for Cardiac and Thoracic Anesthesia ....................................................329 56 Regional Anesthesia in Austere Environment Medicine ...............................................333 57 Anesthesia for Humanitarian Relief Operations .........................................................337

66 Regional Anesthesia and Perioperative Outcome ........................................................................381 67 The Effects of Regional Anesthesia on Functional Outcome After Surgery ..........................383

PART 8

68 Intravenous Patient-Controlled Analgesia .............387 69 Continuous Peripheral Nerve Blocks .......................389 70 Organization of an Acute Pain Management Service Incorporating Regional Anesthesia Techniques ...............................................391 71 Multimodal Analgesia: Pharmacologic Interventions and Prevention of Persistent Postoperative Pain ......................................................393 72 The Role of Nonopioid Analgesic Infusions in the Management of Postoperative Pain .............397

Regional Anesthesia in the Emergency Department

341

58 Regional Anesthesia and Acute Pain Management in the Emergency Department ........343

PART 9 Complications of Local and Regional Anesthesia

347

59 Complications and Prevention of Neurologic Injury with Peripheral Nerve Blocks ........................349 60 Assessment of Neurologic Complications of Regional Anesthesia ...............................................355 61 Perioperative Nerve Injury Unrelated to Nerve Blockade ............................................................359 62 Monitoring, Documentation, and Consent for Regional Anesthesia Procedures ........................363 63 Diagnosis and Management of Spinal and Peripheral Nerve Hematoma .............................367

PART 12 Acute Pain Management

PART 13 Education in Regional Anesthesia

385

401

73 Teaching Regional Anesthesia ..................................403

PART 14 Statistics and Principles of Research Design in Regional Anesthesia and Acute Pain Medicine 407

PART 10 LAST: Local Anesthetic Systemic Toxicity 371

74 Principles of Statistical Methods for Research in Regional Anesthesia .............................409

64 Local Anesthetic Systemic Toxicity ...........................373

Index.........................................................................................413

PART 11 Perioperative Outcome and Economics of Regional Anesthesia

377

65 Regional Anesthesia, Cost, Operating Room, and Personnel Management ........................379

Hadzic_FM_p00i-pxx.indd 9

22/04/19 6:41 PM

This page intentionally left blank

Hadzic_FM_p00i-pxx.indd 6

22/04/19 6:40 PM

Contributors Sherif Abbas, MD Anesthesiologist UZ Leuven, Catholic University of Leuven Leuven, Vlaams-Brabant, Belgium Michael Akerman, MD Assistant Professor of Anesthesiology Regional Anesthesia and Acute Pain Medicine Weill Cornell Hospital New York, New York Arthur Atchabahian, MD, FASA Professor of Clinical Anesthesiology Director, Regional Anesthesia Fellowship NYU School of Medicine New York, New York Angela Lucia Balocco, MD Anesthesiologist Research Fellow NYSORA, The New York School of Regional Anesthesia New York, New York Vikram Bansal, MD Assistant Professor of Anesthesiology Vanderbilt University Medical Center Nashville, Tennessee Michael J. Barrington, PhD Professor, Centre for Integrated Critical Care | Department of Medicine & Radiology | Melbourne Medical School Faculty of Medicine, Dentistry and Health Sciences The University of Melbourne, Victoria 3010 Australia Senior Staff Anaesthetist St.Vincent’s Hospital Melbourne Melbourne, Australia Thomas Fichtner Bendtsen, MD, PhD Professor of Anesthesiology Aarhus University Hospital Aarhus, Denmark Siska Bjørn, BSc PhD Fellow Aarhus University Hospital Aarhus, Denmark

Hadzic_FM_p00i-pxx.indd 11

Jan Boublik, MD, PhD Clinical Assistant Professor Stanford University Stanford Medical School Department of Anesthesiology, Perioperative and Pain Medicine Stanford, California Astrid De Bruyn, MD Resident Anesthesiology Jessa Hospital Hasselt, Belgium Donal J. Buggy, MD, FRCPI, FCAI, FRCA Full Professor, Anaesthesiology & Perioperative Medicine & Consultant in Anaesthesiology Mater University Hospital, School of Medicine, University College Dublin, Ireland Christiana Burt, MA (Cantab), FRCA Consultant Anaesthetist Royal College of Anaesthetists College Tutor Royal Papworth Foundation Hospital Trust Cambridge, Cambridgeshire Asokumar Buvanendran, MD William Gottschalk Professor of Anesthesiology Rush University Medical Center Chicago, Illinois Kenneth D. Candido, MD Chairman Advocate Illinois Masonic Medical Center Professor of Anesthesiology and Surgery University of Illinois Chicago, Illinois Kathleen Chan, MD Fellow in the Division of Acute and Perioperative Pain Medicine University of Florida College of Medicine Gainesville, Florida Franklin Chiao, MD, LAc Director of Acute Pain Management Attending Physician Westchester Medical Center Ardsley, New York

22/04/19 6:41 PM

xii

Contributors

Ki Jinn Chin, MBBS(Hons), MMed, FRCPC Associate Professor, Department of Anesthesia Toronto Western Hospital, University of Toronto Toronto, Ontario, Canada

Matthias Desmet, MD, PhD Consultant Anesthesist AZ Groeninge Kortrijk, Belgium

Jason Choi, MD Attending Anesthesiologist White Plains Hospital White Plains, New York

Hesham Elsharkawy, MD, MBA, MSc, FASA Associate Professor of Anesthesiology Case Western Reserve University Staff, Departments of General Anesthesiology and Outcomes Research Anesthesiology Institute Cleveland Clinic Cleveland, Ohio

Stephen Choi, BSc, MD, FRCPC, MSc Staff Anesthesiologist, Sunnybrook Health Sciences Centre Associate Professor, Department of Anesthesia, University of Toronto Toronto, Ontario, Canada Alwin Chuan, MBBS, PhD, FANZCA Conjoint Associate Professor, University of New South Wales Director, Regional Anaesthesia Fellowship Liverpool Hospital Sydney, Australia Cara Connolly, MB, BCh, BAO, LRCP & SI (Hons), MSc, FCAI Consultant Anaesthetist Mater Misericordiae University Hospital Dublin, Ireland Steve Coppens, MD Head of Clinic Anesthesiology Fellowshipdirector Regional Anesthesia University Hospitals Leuven Leuven, Belgium Jennifer L. Cowell, MD Assistant Professor of Anesthesiology and Perioperative Medicine Rutgers Robert Wood Johnson Medical School New Brunswick, New Jersey Pieter Vander Cruyssen, MD, FIPP Anesthesiologist, Department of Anesthesiology and Pain Management AZ Maria Middelares Gent, Belgium Seppe Dehaene, MD Anesthesiologist OLV van Lourdesziekenhuis Waregem, Belgium Lejla Dervišević, MD Senior Teaching Assistant of Human Anatomy Department of Human Anatomy Medical Faculty University of Sarajevo Sarajevo, Bosnia and Herzegovina

Hadzic_FM_p00i-pxx.indd 12

Paul Fettes, MBChB, BSc Consultant Anaesthetist and Honorary Senior Lecturer Department of Anaesthesia Ninewells Hospital & Medical School Dundee, United Kingdom Jeff Gadsden, MD, FRCPC, FANZCA Associate Professor of Anesthesiology Chief, Division of Orthopedic, Plastic and Regional Anesthesiology Regional Anesthesiology and Acute Pain Medicine Fellowship Director Duke University Medical Center Durham, North Carolina Tong J. Gan, MD, MBA, MHS, FRCA Professor and Chairman Department of Anesthesiology Stony Brook University Stony Brook, New York Will Gauntlett, MBBCh, FRCA Consultant Anaesthetists Alder Hey Children’s Hospital Liverpool, Cheshire Philippe Gautier, MD Head of Department Department of Anesthesiology Clinique Ste Anne-St Remi, CHIREC Brussels, Belgium Liane Germond, MD Director Obstetric Anesthesia Ochsner Health System New Orleans, Louisiana Leen Govaers, MD Medical Doctor in Anesthesiology Fellow in Regional Anesthesia Universitair Ziekenhuis Leuven Leuven, Belgium

22/04/19 6:41 PM

Contributors

Admir Hadzic, MD, PhD Professor of Anesthesiology Consultant, Anesthesiology, Intensive Care, Emergency Medicine and Pain Therapy Ziekenhuis Oost-Limburg Genk, Belgium Director, NYSORA, The New York School of Regional Anesthesia New York, New York Thomas M. Halaszynski, DMD, MD, MBA Professor of Anesthesiology Senior Director of Regional Anesthesia/Acute Pain Medicine Yale University School of Medicine New Haven, Connecticut Brian E. Harrington, MD Staff Anesthesiologist Billings Clinic Hospital Billings, Montana Ilvana Hasanbegovic, MD Associate Professor of Anatomy Department of Anatomy Faculty of Medicine University of Sarajevo Sarajevo, Bosnia and Herzegovina Daryl Steven Henshaw, MD Associate Professor of Anesthesiology Medical Director Section of Regional Anesthesia and Acute Pain Management Wake Forest School of Medicine Winston Salem, North Carolina Jacob Hutchins, MD, MHA Director of the Division of Regional Anesthesia, Acute Pain, and Ambulatory Anesthesia University of Minnesota Hospital Minneapolis, Minnesota Barys Ihnatsenka, MD Associate Professor of Anesthesiology College of Medicine, University of Florida Gainesville, Florida Vivian H. Y. Ip, MBChB, FRCA Associate Clinical Professor University of Alberta Hospital Edmonton, Canada J. Douglas Jaffe, DO, FASA Fellowship Director: Regional Anesthesiology and Acute Pain Medicine Associate Professor of Anesthesiology Wake Forest School of Medicine Wake Forest Baptist Hospital Winston Salem, North Carolina

Hadzic_FM_p00i-pxx.indd 13

xiii

Hassanin Jalil, MD Anesthesiologist Intensive Care Specialist Regional Anesthesia, NYSORA Hasselt, Jessa Hospital Hasselt, Belgium Hari Kalagara, MD, FCARCSI, EDRA Assistant Professor of Anesthesiology The University of Alabama at Birmingham (UAB) Birmingham, Alabama Sowmya Kantamneni, MD Fellow in the Division of Acute and Perioperative Pain Medicine University of Florida College of Medicine Gainesville, Florida Gary Kao, MD Interventional Pain Physician Tricity Pain Associates Corpus Christi, Texas Manoj K. Karmakar, MD, FRCA, DA (UK), FHKCA, FHKAM Director of Paediatric Anaesthesia Department of Anaesthesia and Intensive Care, Faculty of Medicine, The Chinese University of Hong Kong Hong Kong, SAR, China Brendan Keen, MD Anesthesiologist US Anesthesia Partners Colorado Denver, Colorado James K. Kim, MD Assistant Professor University of Pennsylvania Health System Philadelphia, Pennsylvania Jung H. Kim, MD Assistant Professor Icahn School of Medicine at Mt. Sinai St. Luke’s and Mt. Sinai West Hospitals New York, New York Nebojsa Nick Knezevic, MD, PhD Vice Chair for Research and Education Advocate Illinois Masonic Medical Center Associate Professor of Anesthesiology and Surgery University of Illinois Chicago, Illinois Sree Kolli, MD, EDRA Staff Anesthesiologist Associate Director Acute Pain/Regional Anesthesia Cleveland Clinic Cleveland, Ohio

22/04/19 6:41 PM

xiv

Contributors

Samantha Kransingh, MD Anesthesiologist Fellow in Regional Anesthesia Department of Anesthesiology, Intensive Care, Emergency Medicine and Pain Therapy Ziekenhuis Oost-Limburg Genk, Belgium Alison Krishna, MD Assistant Professor of Anesthesiology Mount Sinai St. Luke’s and Mount Sinai West Department of Anesthesiology New York, New York Lisa Kumar, MD Anesthesiologist Baptist Hospital of Miami Miami, Florida Maxine M. Kuroda, PhD, MPH Epidemiologist/Biostatistician NYSORA New York, New York M. Kwesi Kwofie, MD, FRCPC Director of Regional Anesthesia and Acute Pain Assistant Professor Department of Anesthesia, Pain Management and Perioperative Medicine Dalhousie University Halifax, Nova Scotia, Canada Malikah Latmore, MD Assistant Professor of Anesthesiology Mount Sinai St. Luke’s and West Hospitals New York, New York Chad Lee, MD Interventional Pain Physician Georgia Pain and Wellness Center Atlanta, Georgia Ine Leunen, MD Anesthesiologist Intensive Care Medicine AZ Turnhout Turnhout, Belgium Matt Levine, MBChB, FANZCA Specialist Anaesthetist Capital and Coast District Health Board Wellington, New Zealand Ana M. Lopez, MD, PhD, DESA Visiting Professor, KU Leuven Consultant Anesthesiology Ziekenhuis Oost-Limburg Genk, Belgium

Hadzic_FM_p00i-pxx.indd 14

Sofie Louage, MD Anesthesiologist-Intensivist AZ Glorieux Ronse, Belgium Belen De Jose Maria, MD, PhD, ECFMG Consultant in Pediatric Anesthesia Hospital Sant Joan de Deu, University of Barcelona Barcelona, Spain Colleen Mccally, DO Assistant professor of Anesthesiology Assistant Director of East Hills ASC St Francis Hospital Roslyn, New York Colin J. L. McCartney, MBChB, PhD, FRCA, FRCPC Professor and Chair of Anesthesiology and Pain Medicine University of Ottawa Ottawa, Ontario, Canada Shaun De Meirsman, MD President Belgian Anesthesia Trainees University Hospitals Leuven Leuven, Belgium Justin Morello, MD Department of Anesthesiology Ochsner Clinic Foundation New Orleans, Louisiana Hiroaki Murata, MD, PhD Associate Professor Department of Anesthesiology Nagasaki University Graduate School of Biomedical Sciences Nagasaki, Japan Tatsuo Nakamoto, MD, PhD Professor of Anesthesiology Director Regioal Anesthesia/Pain Medicine Kansai Medical University Hospital Hirakata, Osaka, Japan Kristof Nijs, MD Anesthesiology Resident Jessa Hospital Hasselt, Belgium John-Paul J. Pozek, MD Assistant Professor of Anesthesiology Residency Research Coordinator The University of Kansas Health System Kansas City, Kansas Stavros Prineas BSc(Med), MBBS, FRCA, FANZCA Specialist Anaesthetist Nepean Hospital Sydney, Australia

22/04/19 6:41 PM

xv

Contributors

John Rae, FRCA, FFICM Specialty Registrar in Anaesthesia Ninewells Hospital Dundee, United Kingdom Pascal A. Ramsodit, MSc, MD Anesthesiologist and Chronic Pain Specialist. Dijklander Ziekenhuis Hoorn, The Netherlands Kasra Razmjou, MD Assistant Professor of Anesthesiology Medical Director, Acute Pain Service MedStar Georgetown University Hospital Washington, DC Bernard Roach, MBBS, PGClinUS, FANZCA Specialist Anaesthetist Liverpool and Wollongong Hospitals New South Wales, Australia Christopher B. Robards, MD Assistant Professor of Anesthesiology Mayo Clinic Florida Jacksonville, Florida Steve Roberts, MBChB, FRCA Consultant Anaesthetist Alder Hey Children’s NHS Foundation Trust Liverpool, United Kingdom Meg A. Rosenblatt, MD, FASA Professor of Anesthesiology and Orthopedics Icahn School of Medicine at Mount Sinai Chair, Department of Anesthesiology, Perioperative and Pain Medicine Mount Sinai St. Luke’s and West Hospitals New York, New York Siddharth Sata, DO Assistant Professor of Anesthesiology Duke University School of Medicine Durham, North Carolina Sebastian Schulz-Stübner, MD PhD Privatdozent in Anesthesia Chief Physician German Consulting Center for Infection Control and Prevention (BZH GmbH) Freiburg, Germany Ali Shariat, MD Assistant Professor of Anesthesiology Mount Sinai West and St. Luke’s Hospitals New York, New York Uma Shastri, MD, FRCPC Assistant Professor of Anesthesiology Vanderbilt University Nashville, Tennessee

Hadzic_FM_p00i-pxx.indd 15

Yanxia Sun, MD, PhD Chairman Department of Anesthesiology Bejing Lu Dao Pei Hospital Staff Anesthesiologist Department of Anesthesiology Beijing TongRen Hospital Capital Medical University Fengtai, China Evan Sutton, MD Anesthesiologist Bend Anesthesiology Group Bend, Oregon Tiffany Tedore, MD Associate Professor of Anesthesiology Co-Director, Regional Anesthesiology and Acute Pain Medicine New York Presbyterian Hospital Weill Cornell Medical College New York, New York Antony R. Tharian, MD Program Director Advocate Illinois Masonic Medical Center Assistant Professor of Anesthesiology University of Illinois Chicago, Illinois Luc Tielens, MD Paediatric Anesthesiologist President of the Dutch Association for Regional Anesthesia (DARA) Radboudumc Nijmegen, The Netherlands Ban C.H. Tsui, Dip Eng, BSc(Math), B.Pharm, MSc, MD, FRCP(C), PG Dip Echo Professor Director, Stanford University Pediatric Regional Anesthesia (SUPRA) Director of Research, Division of Adult Regional Anesthesia Department of Anesthesiology, Perioperative and Pain Medicine, Stanford University Stanford, Calofornia Vishal Uppal, MBBS, DA, EDRA, FRCA Assistant Professor & Staff Anesthesiologist Director, Regional Anesthesia Fellowship Program Department of Anesthesia, Pain Management & Perioperative Medicine Dalhousie University, Halifax Halifax, Nova Scotia, Canada Sam Van Boxstael, MD Consultant in Emergency Medicine, Anesthesiology and ICU Ziekenhuis Oost-Limburg Genk, Belgium

22/04/19 6:41 PM

xvi

Contributors

Catherine Vandepitte, MD, PhD Consultant Anaesthesiologist Kritieke Diensten Ziekenhuis Oost-Limburg Genk, Belgium Cedric Van Dijck, MD Dept. of Anesthesiology, Emergency Medicine & Critical Care Ziekenhuis Oost-Limburg Genk, Belgium Pascal Vanelderen, MD, PhD Head of the Emergency Department Ziekenhuis Oost-Limburg Genk, Belgium Professor at the Faculty of Medicine and Life Sciences Hasselt University Diepenbeek, Belgium Astrid Van Lantschoot, MD Staff member anesthesiology ZOL Genk Genk, Belgium Thibaut Vanneste, MD Anesthesiologist Hospital Oost-Limburg Genk, Belgium André Van Zundert, MD, PhD, FRCA, EDRA, FANZCA Professor & Chairman Discipline of Anesthesiology The University of Queensland—Faculty of Medicine & Biomedical Sciences Chair, University of Queensland, Burns, Trauma & Critical Care Research Centre Chair, RBWH/University of Queensland, Centre for Excellence & Innovation in Anaesthesia, Department of Anaesthesia & Perioperative Medicine Queensland, Australian

Hadzic_FM_p00i-pxx.indd 16

Tom C. Van Zundert, MD, PhD, EDRA, FANZCA Udayana University, Bali, Indonesia Ziekenhuis Oost Limburg Genk, Belgium Alexandru Visan, MD, MBA CEO, Executive Cortex Consulting Miami, Florida Alexander Vloka, MD Internal Medicine Resident Boise VA Medical Center Boise, Idaho Philippe Volders, MD Department of Anesthesia and Critical Care Regional Anesthesia Algemeen Ziekenhuis Diest Diest, Belgium Christopher Wahal, MD Assistant Professor of Anesthesiology Department of Anesthesiology Sidney Kimmel Medical College at Thomas Jefferson University Philadelphia, Pennsylvania Takayuki Yoshida, MD, PhD, EDRA Assistant Professor Department of Anesthesiology Kansai Medical University Hospital Hirakata, Osaka, Japan Adam C. Young, MD Assistant Professor of Anesthesiology & Pain Medicine Co-Director, Acute Pain Service Assistant Professor Anesthesiology & Interventional Pain Medicine Rush University Medical Center Chicago, Illinois

22/04/19 6:41 PM

Preface Regional anesthesia and acute pain medicine protocols are rapidly changing. Introduction of ultrasound in interventional pain management and regional anesthesia has led to substantial changes in practice management, protocols, techniques, and applications, and their effects on patient safety and efficacy. Nearly all anesthesiology journals now incorporate a section on regional anesthesia and acute pain medicine. This evolution of the practice and expansion of new knowledge mandates frequent updates through continuous medical education. While the didactic knowledge of regional anesthesia and acute pain medicine is available in anesthesiology textbooks, a compendium of information for the purpose of knowledge assessment in the subspecialty does not exist. Hence, NYSORA’s Textbook of Regional Anesthesia and Acute Pain Management aims to fill this gap by providing a comprehensive databank of questions that can be used to test students’ knowledge and clinical reasoning regarding new developments in the field. In making this book, we have selected a team of opinion leaders throughout the world and paired them with students of anesthesiology in order to prepare the questions and logical answers. The questions are

Hadzic_FM_p00i-pxx.indd 17

organized in specific sections, whereas the answers are provided from NYSORA’s textbooks and relevant additional literature citations. To our knowledge, this is the first question book that focuses on the rapidly developing subspecialty of regional anesthesia and acute pain management and point-of-care ultrasound-guided interventional analgesia and anesthesia. With this volume we primarily aim at students of anesthesiology, but the question bank can also be used to assess knowledge acquisition of fellows in regional anesthesia and acute pain medicine, and/or to test the knowledge of applicants for the diploma in regional anesthesia (eg, EDRA, European Diploma of Regional Anesthesia, administered by ESRA, the European Society for Regional Anesthesia). We hope that this question book will be useful in assessing knowledge acquisition. We invite comments and suggestions for future editions and also look forward to developing this question book into a global knowledge assessment test. Sincerely, Prof. Admir Hadzic

22/04/19 6:41 PM

This page intentionally left blank

Hadzic_FM_p00i-pxx.indd 6

22/04/19 6:40 PM

Acknowledgments Writing a book is always a large undertaking that is difficult to accomplish without collaboration and support. I would like to thank all NYSORA team members who have donated their time, knowledge, and wisdom to this volume. I would also like to acknowledge the outstanding students of medicine, anesthesiology residents of the Catholic University of Leuven and Katholieke Universiteit Leuven (KUL), and NYSORA Europe fellows in regional anesthesia at Ziekenhuis Oost-Limburg (ZOL), Genk, Belgium. Several talented and resourceful anesthesiologists are richly deserving of specific mention: Drs. Angela Lucia Balocco, Ana Lopez, and Catherine Vandepitte. Special thanks to NYSORA’s research team: Drs. Ingrid Meex PhD, Gülhan Özyürek, and Marijke Cipers. Likewise, a big THANK YOU to Professor Marc Vandevelde, and Dr. Steven

Hadzic_FM_p00i-pxx.indd 19

Coppens at KUL, as well as René Heylen and the leadership of ZOL, Genk, Belgium. Your wisdom and vision have created a platform to make scholarly endeavors, such as completing this book writing, possible. I would also like to thank the entire Department of Anesthesiology, Intensive Care Emergency Medicine and Pain Therapy at ZOL—your dedication to clinical care and teaching clinical medicine is inspiring. Finally, much appreciation to Professor Dr. Jan Van Zundert for his advice, wisdom, and coaching me to join this inspiring group of anesthesiologists in bettering education and clinical care in perioperative medicine and for an opportunity to develop the orthopedic anesthesia and research unit at ZOL, in Limburg, Belgium. Prof. Admir Hadzic

22/04/19 6:41 PM

This page intentionally left blank

Hadzic_FM_p00i-pxx.indd 6

22/04/19 6:40 PM

PART 1 History

Chapter 1

Hadzic_Ch01_p001-004.indd 1

The History of Local Anesthesia  3

18/04/19 2:31 PM

This page intentionally left blank

Hadzic_FM_p00i-pxx.indd 6

22/04/19 6:40 PM

1 The History of Local Anesthesia Alwin Chuan

QUESTIONS DIRECTIONS: Choose the one best response to each question. 1.   In 1653, the first person to describe the anesthetic effects of coca was: A. Blas Valera B. Bernabé Cobo C. Pedro Pizarro D. Paolo Mantegazza 2.  The local anesthetic properties of cocaine were first appreciated, in 1884, for the treatment of: A. Peripheral neuromas B. Cutaneous lesions C. Morphine addiction D. Glaucoma 3.  An important advancement in local anesthetic drugs was due to the introduction of: A. Prilocaine, as it is metabolized to end products that are nontoxic B. Lidocaine, as it is an amide local anesthetic with lower side effects C. Ropivacaine, as the racemic mixture reduces the risk of cardiac toxicity D. Procaine, as there are fewer allergic reactions than tetracaine 4.  Tumescent anesthesia refers to: A. Intraneural injections of local anesthetics into peripheral nerves causing swelling and conduction block B. Use of large volumes of dilute local anesthetic solutions for cutaneous procedures C. Cutaneous infiltration of physiologic sterile water and local anesthetic solutions to effect conduction block D. Large volumes of a dilute solution of 2% cocaine with physiologic sterile saline for cutaneous procedures

5.   The first successful and reliable method to extend the duration of effect of local anesthetic drugs was: A. Adding epinephrine to cocaine, by Heinrich Braun B. Using multiple tourniquets to stop redistribution of procaine, by August Bier C. Pressing proximally to the site of cocaine injections, by J. Leonard Corning D. Doubling the concentration of injected cocaine, by William Halsted 6.  The basis for the modern method of spinal anesthesia was: A. Heinrich Quincke injecting cocaine via a paramedian approach to treat hydrocephalus B. J. Leonard Corning injecting cocaine into the subarachnoid space of dogs C. August Bier and Rudolph Matas injecting morphine into the subarachnoid space of each other D. August Bier and August Hildebrandt injecting cocaine into the subarachnoid space of each other 7.  A 28-year-old primigravid patient is to undergo spinal anesthesia for an elective lower segment cesarean delivery. Which of the following is not a potential complication of spinal anesthesia? A. Inadvertent change in the baricity of the solution when tetracaine is used B. Paralysis of the abdominal and thoracic muscles that are involved in respiration C. Sympathetic blockade resulting in vasodilation and hypotension D. Painful spastic paresis 8.  Important steps in the evolution of continuous regional anesthesia included which of the following? A. Description of prolonged subarachnoid anesthesia blockade by Leonard Corning B. Identification of the epidural space by Edward Tuohy C. Using a catheter for labor epidural analgesia by Manuel Curbelo D. Fixation and tunnelling of indwelling catheters by Lincoln Sise

3

Hadzic_Ch01_p001-004.indd 3

18/04/19 2:31 PM

4

PART 1

History

9.  Our understanding of pain has progressed over time, through evolution of several influential theories. Which of the following was not instrumental in arriving at our current concepts of nerve conduction and pain management? A. Specificity theory B. Choleric theory C. Spinal gate theory D. Intensive pain theory

ANSWERS AND EXPLANATIONS 1.  B is correct. Bernabé Cobo, who spent his life bringing Christianity to the Incas, was the first to describe the anesthetic effects of coca. 2.  D is correct. Carl Koller performed the first ophthalmologic surgical procedure using local anesthesia on a patient with glaucoma. 3.  B is correct. Lidocaine is an amino amide derivative, a stable compound not influenced by exposure to high temperatures, and, most importantly, one that does not have the allergic potential of the ester-type local anesthetics. The metabolite of prilocaine is implicated in methemoglobinemia. Ropivacaine is an S-enantiomer formulation. Procaine has the same allergic potential as tetracaine; both are ester anesthetics. 4.  B is correct. Karl Ludwig Schleich’s approach still seems to be relevant, particularly with the recent European enthusiasm for tumescent anesthesia, in which sometimes-huge volumes of very dilute local anesthetic are used for surface surgery. 5.  A is correct. Corning’s successes with prolonging the action of local anesthetic with a physical tourniquet inspired Heinrich F. W. Braun to substitute epinephrine, a “chemical tourniquet,” for the Esmarch tourniquet.

Hadzic_Ch01_p001-004.indd 4

6.  D is correct. Bier was able to demonstrate that small amounts of local anesthetic (cocaine) injected into the subarachnoid space could provide surgical anesthesia for over 67% of the body. Bier concluded that Corning’s injection was extradural, and that he (Bier) deserved to be acknowledged for introducing spinal anesthesia. 7.  A is correct. Inadvertent change in the baricity of the solution when tetracaine is used is not a potential complication of spinal anesthesia. Lincoln Sise began using tetracaine because of its longer duration of action but was concerned about controlling the height of the block. Following Arthur Barker’s recommendations regarding hyperbaric solutions, Sise added 10% glucose with success. Options B, C, and D are known complications. 8.  C is correct. In 1947, Manuel Martinez Curbelo of Cuba is credited with using the Tuohy needle and a small ureteral catheter to provide continuous lumbar epidural analgesia. Corning inadvertently described extradural anesthesia. The epidural space was first described by Achille Dogliotti. Sise experimented with adding glucose to tetracaine to increase the baricity to control the block height after subarachnoid blocks. 9.  B is correct. The Choleric theory was not instrumental in arriving at our current concepts of nerve conduction and pain management. The Choleric theory is part of “The four temperament theory” described by Hippocrates. Options A, C, and D were instrumental theories in arriving in our current concepts of nerve conduction and pain management. Suggested Reading Hadzic A.The history of local anesthesia. In: Chuan A, HarropGriffiths W, eds. Textbook of Regional Anesthesia and Acute Pain Management. 2nd ed. New York, NY: McGraw-Hill Publishing; 2017:chap 1.

18/04/19 2:31 PM

PART 2 Foundations of Local and Regional Anesthesia

Section 1

Anatomy and Histology of Peripheral Nervous System and Neuraxis Chapter 2

Functional Regional Anesthesia Anatomy  7

Chapter 3

Histology of the Peripheral Nerves and Light Microscopy  11

Chapter 4

Connective Tissues of Peripheral Nerves  15

Chapter 5

Ultrastructural Anatomy of the Spinal Meninges and Related Structures  17

Hadzic_Ch02_p005-010.indd 5

18/04/19 2:29 PM

This page intentionally left blank

Hadzic_FM_p00i-pxx.indd 6

22/04/19 6:40 PM

2 Functional Regional Anesthesia Anatomy Ana M. Lopez

QUESTIONS DIRECTIONS: Choose the one best response to each question. 1. Which of the following descriptions is correct about the structure of the neuron? A. A typical neuron consists of a cell body, which is also called the soma, with a small or absent nucleus. B. Many dendrites branch out from the cell body; their main function is to spread out outgoing messages. C. A typical neuron consists of several axons, which carry outgoing messages. They can vary in length and are also called nerve fibers. D. Most neurons are incapable of dividing under normal circumstances and have limited ability to repair themselves after injury. 2. The structure of a typical peripheral nerve is as follows: A. The endoneurium is a resistant layer of connective tissue surrounding each axon; its main function is to deliver strength to the peripheral nerve. B. The nerve fascicle is surrounded by the perineurium. It protects the neural tissue from the surrounding tissue, functioning as a blood–nerve barrier. C. Fascicular bundles are continuous throughout the peripheral nerve. Because of this, axons starting in one quadrant of the nerve keep the same position distally. D. The peripheral nerve is composed of three parts: (1) somatosensory or efferent neurons, (2) motor or afferent neurons, and (3) autonomic neurons. 3. Which of the following anatomical descriptions of the structure of a peripheral nerve is correct? A. The interfascicular epineurium consists of elastic connective tissue fibers binding and maintaining the fascicles in a consistent disposition along the nerve. B. The perineurium is the loose connective tissue that connects the nerve to the surrounding tissues, also known as paraneurium.

C. From inside to outside the nerve, the layers enveloping the fibers are: endoneurium, epineurium, perineurium, and paraneurium. D. The fascicles divide and merge with adjacent bundles redistributing the fibers alongside the nerves. 4. With respect to spinal nerve anatomy, it is true that: A. The anterior rami of cervical and lumbosacral spinal nerves coalesce to form plexuses. B. The dorsal rami of the spinal nerves carry only sensory fibers and innervate the skin of the back and limbs. C. The motor fibers arise from neurons in the posterior horn of the spinal cord. D. The sensory innervation of the bones (osteotomes) closely follows that of the overlying myotomes and dermatomes. E. The thoracic spinal nerves exit above their corresponding vertebra, in contrast to lumbar spinal nerves which exit below the corresponding vertebra. 5. The cervical plexus: A. Is divided into the deep and superficial cervical plexus, the superficial anastomoses with the brachial plexus B. Innervates relevant structures for respiration such as the interscalene muscles, the diaphragm, and infrahyoid muscles C. Forms the phrenic nerve by junction of fibers from C3 to C5 and travels caudally and anterior deep to the anterior scalene muscle D. Forms cutaneous sensory nerves that innervate the neck, face, scalp, and upper thorax 6. The architecture of the brachial plexus is as follows: A. The roots of C5 and C6 form the upper trunk, C7 and C8 the middle trunk, and T1 and T2 the lower trunk. B. The anterior divisions of the upper and middle trunk join to form the medial cord. C. The posterior cord is formed out of all the posterior divisions. D. The distal nerves of the brachial plexus arise distally to the clavicle except the suprascapular nerve that leaves the upper trunk at the supraclavicular level.

7

Hadzic_Ch02_p005-010.indd 7

18/04/19 2:29 PM

8

PART 2

Foundations of Local and Regional Anesthesia

7. The median nerve innervates muscles that cause: A. Abduction of the shoulder B. Flexion of the elbow and wrist C. Pronation of the forearm D. Adduction of the thumb 8. The radial nerve: A. Passes from the axilla to the posterior compartment of the arm through the quadrangular space of Velpeau where it gives a branch to the teres minor muscle B. Receives fibers from C5 to T1 C. Descends along the shaft of the humerus in the spiral groove innervating the triceps and brachialis muscles D. At the elbow divides into a posterior and lateral cutaneous branch, which innervates the lateral aspect of the forearm overlying the radius 9. The ulnar nerve: A. Receives fibers from the upper and middle trunk B. Along the arm, descends superficial to the fascia of the triceps muscle and crosses the elbow posterior to the medial epicondyle C. Proximal to the wrist, sends a branch to innervate the adductor pollicis muscle and the skin of the thenar eminence D. Innervates all the interosseous muscles in the hand 10. The cutaneous innervation of the lower limb is as follows: A. The anterolateral aspect of the thigh is innervated by the lateral femorocutaneous and the genitofemoral nerves, which branch off from the femoral nerve proximal to the inguinal ligament. B. The anterior branch of the obturator nerve contributes to the innervation of the medial aspect of the thigh. C. The posterior aspect of the thigh is supplied by the sural nerve, a branch of the sciatic nerve. D. The anterior aspect of the leg is innervated by the saphenous nerve, the distal sensory branch of the femoral nerve. 11. The sciatic nerve: A. Is made up of two distinct nerves, which travel together in the same tissue sheath from the onset down to the popliteal fossa B. Exits the pelvis through the greater sciatic foramen superior to the piriformis muscle C. In the posterior thigh descends in between the semimembranosus and semitendinosus muscles, medial to the long head of biceps femoris D. At the level of the popliteal fossa gives off its four terminal branches: tibial nerve, deep peroneal, superficial peroneal, and sural nerves. 12. Which description of joint innervation is complete? A. The shoulder is mainly innervated by the suprascapular and axillary nerves, which branch off the upper and middle trunks of the brachial plexus.

Hadzic_Ch02_p005-010.indd 8

B. The elbow is mainly innervated by the radial nerve on the posterior side and the median nerve on the anteromedial side. C. The hip is mainly innervated by branches of the femoral and obturator nerves. D. The knee is mainly innervated by branches of the femoral and tibial nerves.

ANSWERS AND EXPLANATIONS 1. D is correct. In adults, most neurons are incapable of dividing under normal circumstances and have limited ability to repair themselves after injury. A is incorrect. The cell body of the typical neuron has a large nucleus. B is incorrect. The function of dendrites is to receive incoming messages. C is incorrect. A typical neuron has only one axon. 2. B is correct. The nerve fascicle is surrounded by the perineurium, which imparts mechanical strength to the peripheral nerve. In addition to its mechanical strength, the perineurium functions as a diffusion barrier to the fascicle, isolating the endoneural space around the axon from the surrounding tissue. This barrier helps to preserve the ionic milieu of the axon and functions as a blood–nerve barrier. A is incorrect. The endoneurium is a thin and delicate layer of loose connective tissue surrounding each fiber. C is incorrect. The fascicular bundles are not continuous throughout the peripheral nerve. They divide and anastomose with one another as frequently as every few millimeters. D is incorrect. The peripheral nerve is composed of three parts: (1) somatosensory or afferent neurons, (2) motor or efferent neurons, and (3) autonomic neurons. 3. D is correct. The fascicles continuously divide and merge with adjacent bundles redistributing the fibers alongside the nerve. A is incorrect. The interfascicular epineurium contains adipose tissue, fibroblasts, mastocytes, blood vessels (with small nerve fibers innervating these vessels), and lymphatics. B is incorrect. The perineurium surrounds the fascicles and imparts mechanical strength to the peripheral nerve. It also function as a blood–nerve barrier. The paraneurium is the loose connective tissue that connects the nerve to surrounding tissues. C is incorrect. The correct order is endoneurium, perineurium, epineurium, and paraneurium.

18/04/19 2:30 PM

CHAPTER 2

4. A is correct. The anterior rami of cervical and lumbosacral spinal nerves coalesce to form the cervical, brachial and lumbosacral plexuses, respectively. B is incorrect. The dorsal rami carry both motor and sensory fibers and innervate muscle, bones, joints, and the skin of the back. C is incorrect. Motor fibers arise from neurons on the anterior horn of the spinal cord. D is incorrect. The innervation of the osteotomes, myotomes, and dermatomes does not always follow the same segmental pattern. E is incorrect. The cervical spinal nerves exit cranially to the corresponding vertebra, in contrast to the thoracic and lumbar spinal nerves that exit caudally to the vertebra. 5. B is correct. The cervical plexus has important implication in normal respiratory function. In addition to the diaphragm, it innervates the scalene muscles, which promote inspiration by elevating the first rib, and the infrahyoid muscles, which open the laryngeal aditus to facilitate inspiration. A is incorrect. There is no division of the cervical plexus, although the plexus can be blocked at a deep, intermediate, or superficial level. C4 may contribute to the brachial plexus and C5 to the cervical plexus, but it cannot be considered anastomosis. C is incorrect. The phrenic nerve travels caudally and anterior superficial to the fascia of the anterior scalene. D is incorrect. The cervical plexus innervates the neck, scalp, and upper thorax, but it is not involved in the innervation of the face. 6. C is correct. The posterior divisions of all three trunks join to form the posterior cord. A is incorrect. The upper trunk is formed by C5-C6, the middle trunk is the continuation of C7, and the lower trunk is formed by C8-T1. B is incorrect. The lateral cord is formed mainly by the anterior divisions of upper and middle trunks, although it may receive some fibers from the anterior division of the lower trunk. D is incorrect. Several terminal branches arise at the supraclavicular level within the posterior cervical triangle, such as the dorsal scapular, long thoracic, and the thoracodorsal. 7. C is correct. The median nerve innervates the pronator teres and quadratus responsible for pronation of the wrist. A is incorrect. Abduction of the shoulder is provided by the axillary nerve. B is incorrect. Flexion of the elbow is provided by the musculocutaneous nerve. D is incorrect. Adduction of the thumb is provided by the ulnar nerve.

Hadzic_Ch02_p005-010.indd 9

Functional Regional Anesthesia Anatomy

9

8. B is correct. The radial nerve arises from the posterior cord and receives fibers from all posterior divisions (C5-T1). A is incorrect. The axillary nerve, not the radial nerve, passes through the quadrangular space of Velpeau and innervates the teres minor. C is incorrect. The musculocutaneous nerve innervates the brachialis muscle. D is incorrect. The sensory branch of the musculocutaneous nerve innervates the lateral aspect of the forearm. 9. D is correct. The ulnar nerve sends fibers to all interosseous muscles in the hand and to the lumbrical muscles affecting the ring and little fingers. The ulnar nerve ends by innervating the deep head of the flexor pollicis brevis muscle. A is incorrect. The ulnar nerve is formed mainly from fibers of the lower trunk, C8-T1. B is incorrect. At the axilla the ulnar nerve crosses the medial intermuscular septum and descends deep to the fascia of the triceps. C is incorrect. Proximal to the wrist, the ulnar nerve sends a cutaneous branch to the hypothenar eminence. In the hand it gives a deep branch that innervates the adductor pollicis muscle. 10. B is correct. The anterior branch of the obturator nerve passes superficial to the obturator externus muscle, descends the thigh in the muscle plane between the adductor brevis and adductor longus, and terminates in the gracilis muscle. En route, it innervates all of these muscles and the skin covering the medial side. A is incorrect. The femorocutaneous branch and the genitofemoral nerves are branches of the lumbar plexus independent of the femoral nerve. C is incorrect. The sural nerve branches out the tibial and common peroneal nerves at the popliteal fossa and innervates the posterior aspect of the leg. The posterior femoral cutaneous nerve supplies the posterior aspect of the thigh. D is incorrect. The saphenous nerve innervates the skin on the medial side of the leg, ankle, and foot. The anterolateral aspect of the leg is supplied by the peroneal nerve. 11. A is correct. The sciatic nerve is formed by the junction of the tibial and common peroneal nerves. These two branches are distinct from the onset and travel together enveloped in the same tissue sheath. B is incorrect. The sciatic nerve exits the pelvis inferior to the piriformis muscle. C is incorrect. In the posterior thigh, the sciatic nerve passes between the adductor magnus and the long head of the biceps femoris. D is incorrect. At the level of the popliteal fossa, the sciatic nerve divides into the tibial and common peroneal branches.

18/04/19 2:30 PM

10

PART 2

Foundations of Local and Regional Anesthesia

12. A is correct. Innervation to the shoulder joints stems mostly from the axillary and suprascapular nerves (C5-C7). B is incorrect. Nerve supply to the elbow joint includes branches of all major nerves of the brachial plexus that cross the joint: musculocutaneous, radial, median, and ulnar nerves. C is incorrect. Nerves to the hip arise from the femoral, obturator, and sciatic nerves.

Hadzic_Ch02_p005-010.indd 10

D is incorrect. Knee innervation is obtained from branches from the femoral, obturator, and sciatic nerves.

Suggested Reading Hadzic A.Functional regional anesthesia anatomy. In: Carrera A, Lopez AM, Sala-Blanch X, Kapur E, Hasanbegovic I, Hadzic A, eds. Textbook of Regional Anesthesia and Acute Pain Management. 2nd ed. New York, NY: McGraw-Hill Publishing; 2017:chap 3.

18/04/19 2:30 PM

3 Histology of the Peripheral Nerves and Light Microscopy Ilvana Hasanbegovic, Lejla Dervišević, and Alexander Vloka

QUESTIONS DIRECTIONS: Choose the one best response to each question. 1. Which of the following nerve fibers would be the first to be blocked by local anesthetics? A. Aβ fibers B. Aα fibers C. Aγ fibers D. C fibers 2. Which of the following local anesthetics would you choose for effective sciatic nerve block using Labat’s posterior approach? A. 30 mL of 1% mepivacaine B. 20 mL of 1.5% mepivacaine C. 20 mL of 0.75% bupivacaine D. 30 mL of 0.2% ropivacaine 3. Peripheral nerve lesions can be classified in terms of their degree of functional disruption. Which of the following nerve lesions commonly occurs during peripheral nerve blocks? A. Neuropraxia B. Axonotmesis C. Neurotmesis D. All of the above 4. Does intraneural injection cause irreversible nerve injury? A. Yes, always B. No, never C. Only with long-bevel needles D. Infrequently , but possible 5. A good knowledge of anatomy is crucial for efficient execution of peripheral nerve blocks. There are several methods used for nerve identification and preservation

of nerve integrity during the procedure. Which of the following methods is used for proper identification of peripheral nerves during block procedure? A. Nerve stimulator B. Ultrasound C. Injection pressure monitoring D. All of the above 6. During the performance of a peripheral nerve block, the risk of intrafascicular injection differs from site to site in the peripheral nervous system. What peripheral nerve block is associated with the highest incidence of nerve injury? A. Interscalene block B. Supraclavicular block C. Infraclavicular block D. Wrist block 7. High injection pressure and consequent nerve damage during peripheral nerve block indicate: A. Intrafascicular injection B. Extrafascicular injection C. Perineural injection D. Intraneural injection 8. Which of the nerve connective tissue layers is the most important in preserving the integrity of the peripheral nerve? A. Inner epineurium B. External epineurium C. Mesoneurium D. Perineurium 9. The mechanism responsible for nerve injury following intraneural injection of local anesthetics is: A. Needle trauma B. Neurotoxicity C. Ischemic injury D. Multifactorial

11

Hadzic_Ch03_p011-014.indd 11

18/04/19 2:35 PM

12

PART 2

Foundations of Local and Regional Anesthesia

10. An 80-year-old man with diabetes mellitus was scheduled for a surgical procedure on his lower extremity with the use of peripheral nerve block. The resolution of the ultrasound scan was poor and the anesthesiologist experienced difficulties performing the block. What could be a possible reason for the poor resolution on the ultrasound image? A. Age-related changes B. Sex differences C. Some morphological variations D. Preexisting pathology

ANSWERS AND EXPLANATIONS 1. C is correct. Aγ myelinated fibers supply the muscle spindles. They have a smaller diameter than other myelinated fibers. Myelinated nerve fibers with a smaller diameter are more rapidly blocked because the “critical length” of the smaller nerve fiber contains a larger number of Ranvier nodes than a larger myelinated nerve fiber. A and B are incorrect. Proprioceptive afferent (Aα) and motor efferent (Aβ) nerve fibers have the same diameter and are therefore equally sensitive to local anesthetics. The sensitivity of nerve fibers to local anesthetics is not determined by whether they are sensory or motor, but rather by their diameter. D is incorrect. C fibers have the smallest diameter. However, they are unmyelinated. Because myelinated fibers are more easily blocked than unmyelinated fibers, C fibers are more resistant to the action of local anesthetics. 2. B is correct. The dose and concentration of local anesthetic should be optimized for different nerve blocks. The larger the nerve, the more concentrated the local anesthetic must be to obtain effective neural blockade. A is incorrect. A high volume with a low concentration of local anesthetic solution is associated with a lower success rate and a delayed onset time compared to a low volume with a high concentration of the same local anesthetic. C is incorrect. Bupivacaine provides longer duration of nerve blockade compared to other commonly used local anesthetics. However, it also has the worst cardiotoxic profile. Bupivacaine carries a significantly higher risk of cardiac arrest and difficulties in resuscitation. This is especially important because currently there is no monitoring that could prevent systemic toxicity of local anesthetics. Bupivacaine has fallen out of favor in many centers due not only to its potential for serious toxicity, but also the availability of ropivacaine, a local anesthetic characterized by a slightly decreased duration of action and an improved safety profile. D is incorrect. Ropivacaine 0.2% is usually sufficient to provide excellent sensory analgesia but spare any motor blockade.

Hadzic_Ch03_p011-014.indd 12

3. D is correct. Injuries to nerves during peripheral nerve blockade are usually of mixed type. Neuropraxia refers to a mild nerve insult in which the axons and connective tissue structures supporting them remain intact. This type of injury is often associated with focal demyelination and is generally reversible over the course of weeks to several months. Axonal interruption with conservation of the neural connective tissues is termed axonotmesis. Neurotmesis represents complete fascicular interruption, including the axons and the connective tissue. Because the nerve is severed, recovery depends on the surgical reapproximation of the two stumps. Even with prompt surgical intervention, recovery is often poor. 4. D is correct. A needle placed intraneurally can be in one of two locations: within the loose epineurial sheath that surrounds the fascicles or inside the fascicle itself. Intraneural needle placement with resultant injection within the epineurium does not lead to an imminent neurological injury. The reason why nerve injury is infrequent is that the vast majority of these injections do not occur within the fascicle. A is incorrect. It is well established that injection of even a very small amount of local anesthetic within the fascicle (intrafascicular) can lead to widespread axonal degeneration and permanent neural damage in animals, whereas extrafascicular injection does not disrupt the normal neural architecture. B is incorrect. Direct intrafascicular injection into the peripheral nerve can result in nerve injury. Therefore it is necessary to be aware of injection pressure in order to avoid nerve injuries. C is incorrect. Long-bevel needles are more likely to puncture and enter the neural fascicle compared with shortbevel needles. However, if the nerve fascicle becomes accidentally impaled during a nerve block procedure, the lesions induced by short-bevel needles tend to be more severe and take longer to repair than those induced by long-bevel needles. 5. D is correct. Nerve stimulators, ultrasound, and injection pressure monitoring each have their own distinct set of advantages and limitations. For this reason, these three technologies are best used in a complementary fashion to minimize the potential for nerve injury, rather than just relying on the information provided by one monitor alone. The combination of all three monitors is likely to produce the safest possible environment in which to perform a peripheral nerve block. Peripheral nerve stimulators are used to localize nerves in order to perform nerve blockade. However, it has been documented that the needle tip can be already in the nerve but may not elicit a motor response even at customarily used low stimulating currents. Use of ultrasound to visualize the needle position and injection pressure monitoring increases the safety of the peripheral nerve block.

18/04/19 2:35 PM

CHAPTER 3

Ultrasound guidance may not always be a sufficiently effective means of preventing nerve injury. The reliability of ultrasound to keep the needle tip extraneural depends largely on the skill of the operator and the imaging characteristics of the needle and the tissue. Furthermore, at the present time the resolution of the sonographic image is such that it is impossible to tell if the needle tip is within the intrafascicular or extrafascicular space, which is the critical anatomic differentiation to avoid nerve injury. Finally, by the time the nerve can be seen swelling on the image, the damage may have already been done if the injection is made with the needle tip inside the fascicle. One shortcoming of injection pressure monitoring is that although it is highly sensitive, it lacks specificity. In other words, the absence of high injection pressure appears to effectively rule out an intrafascicular injection. However, the high injection pressure also can be caused by peripheral nerve block needle obstruction, attempted injection into a tendon, or tissue compression caused by the ultrasound transducer. A “syringe-hand-feel” is a subjective technique and is not reliable. Injection pressure should be objectively monitored. 6. A is correct. The risk of intrafascicular injury correlates with the cross-sectional fascicle to epineurium ratio. The brachial plexus at the level of the trunks contains much more neural than connective tissue. For this reason a needle entering the nerve at this point is more likely to encounter a fascicle on its trajectory. This may contribute to the disproportionately higher rate of postoperative neuropathy following interscalene block. The incidence of nerve injury and neurologic symptoms from interscalene block is 3%. B is incorrect. As peripheral nerves move away from the spinal cord, the ratio of connective tissue to neural tissue within the nerves tends to increase. The brachial plexus elements below the clavicle have a ratio of connective tissue to neural tissue of approximately 2:1, whereas the more proximal trunks and divisions have a 1:1 ratio. The incidence of nerve injury and neurologic symptoms for supraclavicular block is 0.03%. C is incorrect. In the interscalene and supraclavicular regions of the brachial plexus, the nerves are more densely packed and oligofascicular, while more distally, they are polifascicular with a large amount of stromal tissue. D is incorrect. The wrist block technique involves advancing the needle toward the three nerves that supply the hand: the median, ulnar, and radial nerves. In the vicinity of joints, the fascicles are usually thinner and more numerous and tend to be surrounded by a greater amount of connective tissue, which reduces the risk of intrafascicular injection and nerve injury. 7. A is correct. The injection pressure rises abruptly during the intrafascicular injection of local anesthetic and can remain higher than the capillary perfusion pressure beyond the duration of the injection itself predisposing to neuronal ischemia and inflammation. Furthermore, pressure curves derived from intrafascicular versus

Hadzic_Ch03_p011-014.indd 13

Histology of the Peripheral Nerves and Light Microscopy

13

extrafascicular injection in canine sciatic nerves show that a pattern of very high initial injection pressure followed by a sharp drop to baseline is associated with poor outcome and severe neuronal histological damage and may suggest fascicular rupture. B is incorrect. High force (pressure) is required for the intrafascicular injection of local anesthetic compared to extrafascicular injection. C is incorrect. In contrast to the intrafascicular injection, extrafascicular injection is associated with a minimal rise in pressure, which can be explained by its loose and accommodating stromal architecture. D is incorrect. The intraneural injection is not always associated with nerve injury. Extrafascicular intraneural injections usually present with low injection pressures indicating safe neuronal blockade. 8. D is correct. The perineurium is a sheath of connective tissue consisting of several layers of perineural cells, which surrounds each individual fascicle. It acts as a physical and chemical barrier. Injection into that compartment will disrupt the perineurium and result in neural injury. A is incorrect. The epineurium is a condensation of loose areolar connective tissue that surrounds a peripheral nerve and binds its fascicles in a common bundle. Epineurium that extends between the fascicles is called the interfascicular or inner epineurium. B is incorrect. The epineurium that surrounds the entire nerve trunk is the epifascicular or external epineurium. C is incorrect. The mesoneurium is a loose areolar tissue covering the outside of the nerve, which extends from the epineurium to the surrounding tissue. Mesoneurium gives protection against nerve trauma and is a conduit for nerve gliding during movement. It can accommodate the injected volume of local anesthetic during nerve blockade. 9. D is correct. Most peripheral nerve injuries that are associated with peripheral nerve blocks have a multifactorial etiology. It is difficult to differentiate the relative magnitude of the contributing factors. Once the perineurium is breached, the spectrum of the subsequent injury is wide and multifactorial. A is incorrect. Needle-related nerve injuries may result from forceful needle nerve contact or intrafascicular injection. It has been postulated that an intraneural injection may cause sustained high intraneural pressure, which may lead to nerve ischemia and potential injury. One of the main causes of block-related peripheral nerve injury is injection of local anesthetic into the fascicle, causing rupture of the perineurium and loss of the protective environment within the fascicle with consequent myelin and axonal degeneration. B is incorrect. All local anesthetics are potentially neurotoxic. The site of local anesthetic injection may be the primary determinant of whether neurotoxicity will occur, especially if the concentration is high and duration of exposure prolonged. Most chemical substances, including all local anesthetics, injected intrafascicularly lead

18/04/19 2:35 PM

14

PART 2

Foundations of Local and Regional Anesthesia

to severe fascicular injury, whereas the same substances injected intraneurally but extrafascicularly cause less injury or no detectible injury at all. C is incorrect. Damage to the nerve vasculature during nerve blocks can result in local or diffuse ischemia and occurs when there is direct vascular injury, acute occlusion of the arteries from which the vasa nervorum are derived, or from hemorrhage within the nerve sheath. Local anesthetics and adjuncts can also potentially reduce neuronal blood flow. 10. A is correct. Age-related changes in the peripheral nerves result from the cumulative, lifelong effect of various pathogenic factors modified by genetic determinants and by the gradual decrease in the regenerative capacity of peripheral nerves. Age-related changes of nerves and surrounding tissues may be responsible for the typically poorer ultrasonographic images of the peripheral nerves and surrounding tissues in the elderly as compared to younger subjects. B is incorrect. There are few reports about sex difference concerning the morphology of the human peripheral nerves. All of them underlying that there is no statistically significant difference in the total number, average transverse area, or average circularity ratio of myelinated axons between the females and males. C is incorrect. In the case of suspected variations on one side, the anesthesiologist should check the patient’s other side and exclude existing variations. D is incorrect. Patients with underlying nerve pathology are more susceptible to peripheral nerve complications, including prolonged duration of block and increased neurotoxicity from local anesthetics. But in this case, preexisting pathology is probably not the reason for the poor ultrasound picture.

Hadzic_Ch03_p011-014.indd 14

Suggested Readings Brull R, McCartney CJ, Chan VW, El-Beheiry H. Neurological complications after regional anesthesia: contemporary estimates of risk. Anesth Analg. 2007;104(4):965-974. Gentili F, Hudson AR, Hunter D, Kline DG. Nerve injection injury with local anesthetic agents: a light and electron microscopic, fluorescent microscopic, and horseradish peroxidase study. Neurosurgery. 1980;6(3):263-272. Hadzic A. Histology of the peripheral nerves and light microscopy. In: Cvetko E, Meznarič M, Pintaric TS, eds. Textbook of Regional Anesthesia and Acute Pain Management. 2nd ed. New York, NY: McGraw-Hill Publishing; 2017:chap 4. Hadzic A, Dilberovic F, Shah S, et al. Combination of intraneural injection and high injection pressure leads to fascicular injury and neurologic deficits in dogs. Reg Anesth Pain Med. 2004;29(5):417-423. Moriyama H, Hayashi S, Inoue Y, Itoh M, Otsuka N. Sex differences in morphometric aspects of the peripheral nerves and related diseases. NeuroRehabilitation. 2016;39(3):413-422. Selander D, Sjöstrand J. Longitudinal spread of intraneurally injected local anesthetics. An experimental study of the initial neural distribution following intraneural injections. Acta Anaesthesiol Scand. 1978;22(6):622-634. Taboada MM, Rodriguez J, Bermudez M, et al. Low volume and high concentration of local anesthetic is more efficacious than high volume and low concentration in Labat’s sciatic nerve block: a prospective, randomized comparison. Anesth Analg. 2008;107(6):2085-2088. New York School of Regional Anesthesia. https://www.nysora.com.

18/04/19 2:35 PM

4 Connective Tissues of Peripheral Nerves Ana M. Lopez

QUESTIONS DIRECTIONS: Choose the one best response to each question. 1.  Which of the following descriptions is correct about the structure of the nerve? A. Along the nerve, the same axon can contribute to different fascicles. B. The location of nerve fascicles inside the nerve is consistent along the nerve, with little variation. C. The proportion of connective tissue in the nerve decreases as it travels distally. D. In the vicinity of a joint, the nerve has fewer fascicles of larger size to offer more resistance to stretching. 2.  The connective tissue layers of a typical peripheral nerve are as follows from inside to outside: A. Endoneurium - paraneurium - epineurium perineurium B. Perineurium - endoneurium - paraneurium epineurium C. Endoneurium - perineurium - epineurium paraneurium D. Perineurium - endoneurium - epineurium paraneurium 3.  Which of the following anatomical descriptions of the structure of a peripheral nerve is correct? A. The interfascicular epineurium consists of elastic connective tissue fibers binding and maintaining the fascicles in a consistent disposition along the nerve. B. The perineurium is the loose connective tissue that connects the nerve to the surrounding tissues, also known as paraneurium. C. The endoneurium consists of concentric layers of connective tissue with tight cellular junctions that form a blood–nerve barrier. D. The fascicles divide and merge with adjacent bundles redistributing the fibers alongside the nerves. 4.  The outermost connective tissue layer enveloping the nerve: A. Is the perineurium B. Is absent in multifascicular nerves

C. Contains adipocytes, mast cells, lymphatics, and blood vessels D. Is the epineurium that can be distinctly identified by ultrasound before injection 5.  Which statement is true regarding peripheral nerve blocks? A. The local anesthetic reaches the axons through the intraneural capillary network and is independent of the size of the fibers. B. The architecture of connective tissue layers in the peripheral nerve influences the spread of local anesthetic and therefore, the onset of the block. C. The proportion of local anesthetic that traverses the connective tissue sheaths and reaches the axons is about 50% of the injectate. D. Despite substantial variability in the characteristics of the connective tissue layers, the dynamics and quality of neural blockade are consistent and reliable in different patient populations.

ANSWERS AND EXPLANATIONS 1.  A is correct. Inside each nerve, the axons form an intraneural plexus in such a fashion that one axon can contribute to different fascicles along the nerve length. B is incorrect. The number, size, and locations of fascicles in peripheral nerves are also variable even within a single nerve and can vary as much as 23 times along a 4- to 5-cm length of nerve. C is incorrect. The proportion of neural tissue is higher at the origin of the nerve. Distally, the proportion of connective tissue increases and can reach up to 75% of the nerve cross section. D is incorrect. In the proximity of joints, the fascicles are thinner, more numerous, and have a thicker perineurium, which may confer better protection against pressure and stretching.

15

Hadzic_Ch04_p015-016.indd 15

18/04/19 2:36 PM

16

PART 2

Foundations of Local and Regional Anesthesia

2.  C is correct. The endoneurium is a thin, delicate layer of loose connective tissue surrounding each fiber. A group of fibers forms the nerve fascicle, which is surrounded by the perineurium. The epineurium is the outer layer of connective tissue. The paraneurium is the connective tissue that binds the nerve to the surrounding tissues or more than one nerve together. A, B, and D are incorrect. These options don’t follow the correct order for connective tissue layers from inside to outside.

B is incorrect. The outermost connective tissue layer is present in all multifascicular nerves, although it may be absent in small monofascicular terminal branches. D is incorrect. Ultrasound equipment currently used to perform peripheral nerve blocks is not able to distinguish the epineurium. However, the epineurium can be identified after unintended subepineurial injection, which may not be uncommon. For that reason, the routine use of additional monitoring, such as with nerve stimulators or pressure indicators, is highly recommended.

3.  D is correct. The fascicles divide and merge with adjacent bundles redistributing the fibers alongside the nerves. A is incorrect. The interfascicular epineurium contains adipose tissue, fibroblasts, mastocytes, blood vessels (with small nerve fibers innervating these vessels), and lymphatics. B is incorrect. The perineurium surrounds the fascicles and imparts mechanical strength to the peripheral nerve. It also functions as a blood–nerve barrier. The paraneurium is the loose connective tissue that connects the nerve to surrounding tissues.

5.  B is correct. Diffusion of anesthetic into the axons is influenced by the presence and characteristics of the connective tissue sheaths (eg, perineurium, myelin) and the size and location of the axons inside fascicles. The speed and the amount of local anesthetic that comes into contact with the axons determine the onset of the blockade. A is incorrect. The local anesthetic injected perineurally must traverse the epineurium, perineurium, and endoneurium. Local anesthetic injected intravenously for a Bier block most likely reaches the nerve endings through the intraneural capillary network.

C is incorrect. The above description corresponds to the perineurium: It consists of concentric layers of flattened cells separated by layers of collagen. Tight junctions in the inner layers of the perineurium and tight junctions in endoneurial capillaries form a blood–nerve barrier structure. 4.  C is correct. The epineurium contains adipocytes, fibroblasts, connective tissue fibers, mast cells, small lymphatics, as well as blood vessels and small nerve fibers innervating the vessels. A is incorrect. The outermost connective tissue sheath of peripheral nerves is the epineurium.

Hadzic_Ch04_p015-016.indd 16

C is incorrect. Only a small proportion of the injected local anesthetic comes in direct contact with the axons. In experimental studies the amount of local anesthetic inside the nerve when the block was complete was 1.6% of the injected dose. D is incorrect. Due to substantial variability in the characteristics of the connective tissue layers, the dynamics and quality of neural blockade are inconsistent.

Suggested Reading Hadzic A. Connective tissues of peripheral nerves. In: Reina MA, Sala-Blanch X, Machés F, Arriazu R, Prats-Galino A, eds. Textbook of Regional Anesthesia and Acute Pain Management. 2nd ed. New York, NY: McGraw-Hill Publishing; 2017:chap 5.

18/04/19 2:36 PM

5 Ultrastructural Anatomy of the Spinal Meninges and Related Structures Siddharth Sata

QUESTIONS DIRECTIONS: Choose the one best response to each question. 1. Which of the following statements is most accurate regarding dural puncture? A. The size of the lesion created by a 24-gauge spinal needle is similar with both the pencil-point and cutting-needle designs. B. Cutting needles produce a greater and rougherappearing injury to dural fibers as compared to pencil-point needles. C. Sagittal bevel orientation of a cutting spinal needle is less likely to produce postdural puncture headache (PDPH) as compared to a transverse orientation. D. The morphology of dural lesions is consistent between different spinal needle designs. 2. A 23-year-old primigravid patient for cesarean delivery receives a spinal anesthetic with a 24-gauge cutting needle. Which of the following mechanical factors is true regarding her spinal puncture? A. Passing a needle into the dural sac creates a lesion in the dura mater while sparing the arachnoid layer. B. The dural layer will take at least 3 to 5 days to completely close. C. Both the arachnoid and dural layers are violated; however, the arachnoid violation contributes more significantly to cerebrospinal fluid leakage. D. The U-shaped dural flap created by a cutting needle is likely to lead to significant cerebrospinal fluid leakage. 3. Which of the following statements regarding the anatomical and equipment-related factors of postdural puncture headache (PDPH) is most accurate?

A. Conclusive evidence exists to demonstrate that needlebone contact causes deformation of cutting needles, which leads to increased incidence of PDPH. B. Pencil-point needles lead to less traumatic violations of the dural sac, leading to decreased incidence of PDPH. C. Cutting needles cause a “burst”-type lesion of the dural sac with unpredictable damage of the dura and arachnoid layers leading to increased incidence of PDPH. D. Increased tearing and trauma created by pencil-point needles lead to inflammation of the dural sac, which may be protective against PDPH. 4. Transient root irritation syndrome and cauda equina syndrome: A. Likely have a greater risk of incidence with spinal microcatheter infusion versus single-shot spinal injection B. Are very unlikely to occur without direct needle or catheter trauma to spinal nerve roots C. Are likely to be caused by local anesthetic injection into the dural sac of spinal nerves D. Will likely not occur if the patient does not experience a paresthesia or dysesthesia during spinal needle or catheter placement 5. After placing an epidural catheter at the L3–4 interspace, you dose it with 15 mL of 2% lidocaine in three divided doses to achieve surgical anesthesia. After 20 minutes have passed, the patient has a 10 cm circular patch of skin that retains temperature and sharp pain sensation. Which of the following anatomical features is the most likely cause? A. Anterior meningo-vertebral ligament B. Lateral meningo-vertebral ligament C. Ligamentum flavum D. Posterior meningo-vertebral ligament

17

Hadzic_Ch05_p017-020.indd 17

18/04/19 2:38 PM

18

PART 2

Foundations of Local and Regional Anesthesia

6. Which of the following statements is true regarding epidural fat? A. Below L4–5 epidural fat is the main component surrounding nerve roots within the dural sleeves. B. Epidural fat adheres to nerve roots in the dural sleeves, which limits movement of the dura during flexion and extension. C. Patients with spinal stenosis have increased quantities of epidural fat leading to compression of the nerve roots and spinal cord. D. The quantity of posterior epidural fat decreases as you travel caudally from the cervical region to the lumbar region. 7. You inject an 8 mL bolus of 0.25% bupivacaine via a Tuohy needle at the T12–L1 interspace prior to epidural catheter placement. Within 5 minutes you notice the patient rapidly develops upper extremity weakness and apnea with sparing of abdominal dermatomal anesthesia. Why do you suspect subdural spread of the injectate? A. A large volume of greater than 5 mL is required to cause subdural spread with subsequent high blockade. B. Iatrogenic creation of the subdural space leads to highly unpredictable neuraxial anesthesia with unexpected high-level blockade. C. The spinal cord is still present at this spinal level, which increases the likelihood of subdural injection. D. The subdural space is a compliant, nonadherent, and well-circumscribed layer between the dura and arachnoid mater in most patients. 8. Which of the following spinal needle designs is least likely to cause fiber tearing and damage with less subsequent inflammation in the dura and arachnoid, conferring an increased risk of postdural puncture headache? A. Quincke B. Sprotte C. Greene (noncutting) D. Whitacre 9. The trabecular arachnoid sheath: A. Is an integral component of all nerve roots in the cauda equina B. Is greatly adherent to spinal nerve roots, making it impossible to insert a needle between the two C. Is responsible for minimizing the movement of nerve roots within the dural sac D. Provides significant mechanical protection to spinal nerve roots 10. Which of the following is true regarding epidural fat? A. Epidural fat lies within a continuous circumferential plane in the epidural space at the lumbar levels. B. Excessive fat deposits, such as those seen in epidural lipomatosis, are a generally harmless finding without clinical sequelae. C. The uneven distribution of epidural fat in the lumbar area can lead to differential diffusion of substances through the epidural space, potentially altering drug kinetics.

Hadzic_Ch05_p017-020.indd 18

D. Patients with kyphoscoliosis are likely to have decreased epidural fat, particularly in the concave areas of curvature.

ANSWERS AND EXPLANATIONS 1. A is correct. The size of the lesion created by a 24-gauge spinal needle is similar with the pencil-point and cuttingneedle designs. B is incorrect. Pencil-point needles produce a greater and rougher-appearing injury to dural fibers as compared to cutting needles. C is incorrect. Bevel orientation does not affect the size or morphology of lesions when using cutting needles. D is incorrect. The morphology of dural lesions is dependent on the design of needle tip. 2. C is correct. The size and morphology of arachnoid lesions seem to be more important for laminar sealing and cerebrospinal leakage than the size and morphology of dural lacerations. A is incorrect. Passing a needle into the dural sac creates a lesion in both the dura mater and arachnoid layer. B is incorrect. The dural defect created by a cutting spinal needle is almost completely occluded after approximately 15 minutes. D is incorrect. After needle withdrawal, the U-shaped flap created by a cutting needle returns to the original position due to CSF pressure and the elastic properties of the dura. 3. D is correct. The increased fiber tearing produced by pencil-point needles may promote greater inflammatory response at the edges of the lesion that paradoxically results in earlier occlusion and lower incidence of PDPH. A is incorrect. The definitive impact of spinal needle deformation from needle-bone contact on the incidence of PDPH is difficult to study and remains hypothetical. B is incorrect. Pencil-point needles lead to more traumatic violations of the dural sac with ensuing inflammation that decreases the incidence of PDPH. C is incorrect. Cutting needles create a cleaner defect in the dural sac with less inflammation, which leads to prolonged closure of the defect, possibly leading to increased PDPH. 4. A is correct. Injections of local anesthetic through a microcatheter into these arachnoid sheaths could be more devastating than a single injection. This is because the injection of a single large volume would eventually be diluted by leakage outside the sheath, whereas repeated doses of small volumes may be more likely to lead to neurotoxicity due to the continuous or repeated exposure to a high concentration of local anesthetics. B is incorrect. Injection of local anesthetic within an arachnoid sheath of a spinal nerve root can lead to prolonged exposure to high concentrations of local anesthetic

18/04/19 2:38 PM

CHAPTER 5

Ultrastructural Anatomy of the Spinal Meninges and Related Structures

without dilution from CSF, leading to a nerve lesion without direct needle trauma. C is incorrect. Injection into the arachnoid sheath, not the dural sac surrounding spinal nerve roots, leads to prolonged exposure to high concentrations of local anesthetic. D is incorrect. A patient may not feel any symptoms during arachnoid sheath injection of local anesthetic, and, as explained above, needle-nerve contact is not necessary. 5. A is correct. The anterior meningo-vertebral ligament, which connects the dural sac with the posterior longitudinal ligament of the spine, is more compact. In some patients, fibrous flaps that fix the dural sac to the posterior longitudinal ligament may incompletely divide the anterior epidural space. B is incorrect. The lateral meningo-vertebral ligament is thinner than the anterior and does not influence the spread of fluids in the epidural space. C is incorrect. Ligamentum flavum is superficial to the dura and has not been identified as a structure that commonly creates septae in the epidural space. D is incorrect. The posterior meningo-vertebral ligament is thinner than the anterior and does not influence the spread of fluids in the epidural space. 6. A is correct. Below L4–5 epidural fat is the main component surrounding nerve roots within the dural sleeves. B is incorrect. Epidural fat within the dural sleeves allows for displacement of the dura within the vertebral canal during flexion and extension. C is incorrect. Spinal stenosis leads to decreased epidural fat and is often absent at the stenosed levels. D is incorrect. Posterior epidural fat increases as you travel caudally and is most prominent around L3–4 and L4–5. 7. B is correct. Subdural anesthetic blockade, caused by inadvertent injection of local anesthetic partially or entirely between the dura and arachnoid, results in highly unpredictable spinal or epidural anesthesia and complications due to an unanticipated high-level blockade. A is incorrect. Even small subdural injections of a few milliliters can lead to subdural spread with high-level neuraxial blockade. C is incorrect. The presence of the spinal cord at the level of subdural injection does not necessarily have an impact on the risk of iatrogenic creation of a subdural space.

Hadzic_Ch05_p017-020.indd 19

19

D is incorrect. Contrary to classic teaching, the subdural space is an iatrogenic artifact created by tearing of weak cohesive forces between neurothelial cells, leading to fissures between the dura and arachnoid. 8. A is correct. Quincke is least likely to cause fiber tearing and damage with less subsequent inflammation in the dura and arachnoid, conferring an increased risk of PDPH. B, C, and D are incorrect. Sprotte, Greene, and Whitacre are all non-cutting pencil-point type needles that will not create a clean cut in the dura and arachnoid leading to some level of inflammation. 9. C is correct. During movement, these sheaths stabilize and prevent excessive movements of the nerve roots within the dural sac. However, the sheaths confer little mechanical protection against trauma. A is incorrect. Trabecular arachnoid sheaths variably envelope nerve roots, and in some cases are completely absent on nerve roots. B is incorrect. Needles and microcatheters can and have been placed between nerve roots and the surrounding arachnoid sheath. D is incorrect. Arachnoid sheaths confer little mechanical protection to nerve roots. 10. C is correct. The distribution of epidural fat in the lumbar vertebral canal is uneven, being more abundant in the dorsal region than in the ventral and lateral regions. The total amount, distribution, and morphology of fat in the epidural space and nerve root cuffs affect the diffusion of substances across these compartments. A is incorrect. At the lumbar level epidural fat is separated into anterior and posterior spaces. B is incorrect. Epidural lipomatosis can lead to spinal cord and nerve root compression with significant neurologic symptoms. D is incorrect. Kyphoscoliotic patients have asymmetrically distributed epidural fat with a greater quantity concentrated in the concave portions of the epidural space.

Suggested Reading Hadzic A. Ultrastructural anatomy of the spinal meninges and related structures. In: Reina MA, Franco CD, Prats-Galino A, Machés F, López A, de Andrés JA, eds. Textbook of Regional Anesthesia and Acute Pain Management. 2nd ed. New York, NY: McGraw-Hill Publishing; 2017:chap 6.

18/04/19 2:38 PM

This page intentionally left blank

Hadzic_FM_p00i-pxx.indd 6

22/04/19 6:40 PM

Section 2

Pharmacology Chapter 6

Clinical Pharmacology of Local Anesthetics  23

Chapter 7

Controlled-Release Local Anesthetics  27

Chapter 8

Analgesic Adjuvants in the Peripheral Nervous System  31

Chapter 9

Local Anesthetic Mixtures for Peripheral Nerve Blocks  35

Chapter 10

Continuous Peripheral Nerve Blocks: Local Anesthetic Solutions and Infusion Strategies  37

Hadzic_Ch06_p021-026.indd 21

19/04/19 10:04 AM

This page intentionally left blank

Hadzic_FM_p00i-pxx.indd 6

22/04/19 6:40 PM

6 Clinical Pharmacology of Local Anesthetics Shaun De Meirsman

QUESTIONS DIRECTIONS: Choose the one best response to each question. 1.  Which of the following is true regarding the history of local anesthetics (LAs)? A. Cocaine was the first applicable product used by Carl Koller as a LA. B. Ten years after the first introduction of cocaine as a LA, the first successful brachial plexus block was performed. C. Leonard Corning was the first to perform a spinal anesthesia with cocaine. D. Esters occur naturally. The first synthetic local anesthetic was an amide like lidocaine, which was introduced almost 100 years later. 2.  Voltage-gated sodium (Na) channels have a specific structure. Which of the following statements is true? A. They consist of two large α-subunits and two smaller β-subunits. The β-subunit is the place of LA binding, causing a blockade in neural excitability and impulse transmission between neural fibers. B. They consist of six transmembranous segments. A shift in these segments causes inadequate transmission of neurotransmitters causing an inexcitable neuron. C. They consist of one α-subunit and two β-subunits. The α-subunit is the site of ion conduction and of LA binding. Sodium channels have at least three native configurations. D. The distribution between sodium channels in myelinated and unmyelinated fibers is the same. The difference in conduction mechanism causes LA to be more potent in the unmyelinated nerve fibers. 3.  Potency of local anesthetics (LAs) increases with: A. Decreased molecular weight, increased lipid solubility, more protein bound, and decreased duration of action B. Increased molecular weight, increased lipid solubility, less protein bound, and increased duration of action

C. Increased molecular weight, increased lipid solubility, more protein bound, and increased duration of action D. Decreased molecular weight, decreased lipid solubility, less protein bound, and decreased duration of action 4.  Speed of onset of local anesthetic increases with: A. Decreased lipid solubility, high pKa value, more aqueous solubility, and increased molecular weight B. Increased lipid solubility, intermediate pKa value, less aqueous solubility, and decreased molecular weight C. Increased lipid solubility, high pKa value, less aqueous solubility, and increased molecular weight D. Decreased lipid solubility, intermediate pKa value, more aqueous solubility, and decreased molecular weight 5.  Many factors can influence the effectiveness of local anesthetic, such as: A. Site of administration, temperature, pregnancy, additives, and dose B. Site of administration, temperature, pregnancy, gender, and dose C. Site of administration, temperature, gender, age, and dose D. Pregnancy, additives, age, dose, and molecular weight 6.  During pregnancy local anesthetics (LAs) can be used safely. Which of the following statements is true? A. Pregnant women are less neurally susceptible to local anesthetics. Esters undergo a more rapid metabolization through the liver during pregnancy. B. Pregnant women are more neurally susceptible to local anesthetics, so more LA is necessary to have the same clinical effect. C. Spinal spread of LA neuraxially will increase because of the decrease in thoracolumbar cerebrospinal fluid volume. D. LA toxicity is more common in pregnant women because protein binding and protein concentration are increased during pregnancy.

23

Hadzic_Ch06_p021-026.indd 23

19/04/19 10:04 AM

24

PART 2

Foundations of Local and Regional Anesthesia

7.  Which of the following statements is true, concerning local anesthetic (LA) allergic reactions? A. True immunologic responses to LA are rare. Esters are more often the cause of an anaphylaxis compared to amides. B. When injecting a LA intravenously it causes more anaphylaxis when compared to locally. Amide LAs are more often the cause of an anaphylaxis compared to esters. C. Preservatives are the cause of the allergic reactions, not LA. True immunologic responses to LA are frequent. D. LA skin testing has a high positive predictive value in regards to allergic reactions. There are no cross reactions between different LAs. 8.  Neurotoxicity and local anesthetic (LA) are a cause of concern. Which of the following statements is true? A. Signs and symptoms of early LA systemic toxicity consist of tremor, blurry vision, vertigo, nausea, vomitus, tinnitus, convulsions, and hypotension. B. In laboratory settings, the convulsive dose of LA compared to the lethal dose is around five times higher. C. Lidocaine is more cardiotoxic than bupivacaine due to the more avidly and longer binding of lidocaine to cardiac Na channels. D. All of the LAs cause vasodilation of vascular smooth muscle cells except for cocaine—it produces local vasoconstriction. 9.  Treatment of local anesthetic (LA) toxicity depends on the severity of clinical symptoms. Which of the following statements is true? A. Minor reactions such as tinnitus and metallic taste in the mouth should be treated by lipid infusions to create a shift in LA concentration from intracellular to extracellular. B. Seizures due to LA should be treated with anticonvulsive therapy such as carbamazepine or valproic acid. C. When toxicity from LA progresses to a level of myocardial failure and rhythm disturbances, intubation and resuscitation measures including lipid emulsion therapy should be put into motion. D. LA toxicity is common, due to the fact that local anesthetics are lipophilic and can pass easily through the blood–brain barrier.

ANSWERS AND EXPLANATIONS 1.  A is correct. Carl Koller and Joseph Gartner used cocaine to produce topical anesthesia of the conjunctiva. The birth of local and regional anesthesia dates from 1884, when Koller and Gartner reported their success at producing topical cocaine anesthesia of the eye in the frog, rabbit, dog, and human. B is incorrect. Only one year after the first introduction of cocaine as a LA, the first successful brachial plexus block was performed.

Hadzic_Ch06_p021-026.indd 24

C is incorrect. August Bier was the first person to use cocaine for spinal anesthesia. D is incorrect. Only cocaine is a natural occurring local anesthetic ester. Others are synthetic variants. The first introduction of an amide was in 1948, around 70 years later. 2.  C is correct. They consist of one α-subunit and two β-subunits. The α-subunit is the site of ion conduction and of LA binding. Sodium channels have at least three native configurations. A is incorrect. Sodium (Na) channels consist of one large α-subunit and two smaller β-subunits in humans. The α-subunit is the place of local anesthetic binding. B is incorrect. The α-subunit has four homologous domains, each with six transmembranous segments. D is incorrect. The distribution between Na channels in myelinated and unmyelinated is not the same. Myelinated fibers have nodes of Ranvier, which are dense and packed with Na channels. Unmyelinated fibers have more diffuse dispersal of Na channels, which makes them difficult targets for LA. 3.  C is correct. The rate of diffusion across the nerve sheath is determined by the concentration of the drug, its degree of ionization (ionized LA diffuses more slowly), its hydrophobicity, and the physical characteristics of the tissue surrounding the nerve. More lipid-soluble LAs are relatively water insoluble, highly protein bound in blood, less readily removed by the bloodstream from nerve membranes, and more slowly “washed out” from isolated nerves in vitro. Thus, increased lipid solubility is associated with increased protein binding in blood, increased potency, and longer duration of action. A, B, and D are incorrect. Nerve-blocking potency of LAs increases with increasing molecular weight and increasing lipid solubility. The increased lipid solubility is associated with increased protein binding in blood, increased potency, and longer duration of action. 4.  A is correct. The pKa value generally correlates with the speed of onset of action of most amide LA drugs; the closer the pKa value to the body pH, the faster the onset. LA rate of onset is associated with the aqueous diffusion rate, which declines with increasing molecular weight. The relationship between concentration and block onset is logarithmic, not linear; in other words, doubling the concentration of LA will only marginally speed up the onset of the block although it will block the fibers more effectively and prolong the duration. B, C, and D are incorrect. See explanation for answer A. 5.  A is correct. Many factors influence the ability of a given LA to produce adequate regional anesthesia, including the dose, site of administration, additives, temperature, and pregnancy. B, C, and D are incorrect. See explanation for answer A.

19/04/19 10:04 AM

CHAPTER 6

6.  C is correct. Spread of neuraxial anesthesia likely increases during pregnancy due to decrease in thoracolumbar cerebrospinal fluid volume. A is incorrect. Pregnant women are more susceptible to LAs on a neural level. Esters undergo rapid hydrolysis in the blood by nonspecific esterase. B is incorrect. Pregnant women will need less LA to achieve the same clinical response. D is incorrect. Protein binding and protein concentration decline during pregnancy. This has an impact on the potency of LA. 7.  A is correct. True immunologic reactions to LAs are rare. Accidental intravenous injections of LAs are sometimes misdiagnosed as allergic reactions. True anaphylaxis appears more common with ester LAs that are metabolized directly to PABA than to other LAs. B is incorrect. There is no difference between allergic reaction when injecting intravenously; however, the chance of adverse LA systemic toxicity symptoms is often misdiagnosed in the beginning. Esters are more associated with allergic reactions. C is incorrect. Sometimes preservatives can be the cause of an allergic reaction. D is incorrect. LA skin testing has a high negative predictive value in regards to allergic reactions. Cross reactions between esters and amides can occur occasionally. 8.  D is correct. The LAs produce dilation of vascular smooth muscle at clinical concentrations. Cocaine is the only LA that consistently produces local vasoconstriction.

Hadzic_Ch06_p021-026.indd 25

Clinical Pharmacology of Local Anesthetics

25

A is incorrect. Signs and symptoms of LA systemic toxicity do not include blurry vision, nausea, and vomitus. In a later stage, there will be convulsions and hypotension, and even death can occur. B is incorrect. In laboratory settings the LA concentration to cause seizures will be around three times less than the lethal dose. C is incorrect. Bupivacaine is more cardiotoxic than lidocaine because of the more avidly and longer binding to cardiac Na channels. 9.  C is correct. When toxicity from LA progresses to a level of myocardial failure and rhythm disturbances, intubation and resuscitation measures including lipid emulsion therapy should be put into motion. A is incorrect. Minor reactions should be carefully observed and monitored; they usually dissipate over time. B is incorrect. Seizures induced by LA should be managed by maintaining a patent airway and by providing oxygen. Seizures may be terminated with intravenous midazolam (0.05–0.10 mg/kg) or propofol (0.5–1.5 mg/kg) followed by ventilation with bag and mask or intubation. D is incorrect. LA toxicity is not common; it is rare. This is because of safer LA, safer regional anesthesia practices, and improved treatments.

Suggested Reading Hadzic A. Clinical pharmacology of local anesthetics. In: Butterworth IV J, ed. Textbook of Regional Anesthesia and Acute Pain Management. 2nd ed. New York, NY: McGraw-Hill Publishing; 2017:chap 7.

19/04/19 10:04 AM

This page intentionally left blank

Hadzic_FM_p00i-pxx.indd 6

22/04/19 6:40 PM

7 Controlled-Release Local Anesthetics John-Paul J. Pozek

QUESTIONS DIRECTIONS: Choose the one best response to each question. 1. Liposomal bupivacaine: A. Frequently leads to near-toxic doses of local anesthetic B. Has a structure consisting of a multilamellar liposome C. Has a structure consisting of a multivesicular liposome D. Has a structure consisting of a unilamellar liposome 2. What are some advantages of encapsulating local anesthetics (LAs) in liposomes? A. Has been shown to be nontoxic and biodegradable B. Has been shown to consistently release medication over 36 hours C. LAs are the only medication to be encapsulated in liposomes D. Stable due to inability to oxidize the lipid bilayer 3. A 56-year-old woman with hypertension, coronary artery disease, diabetes, end stage renal disease, and alcoholic cirrhosis presents for a hemorrhoidectomy. Which of the following clinical considerations is true? A. Avoid use of liposomal bupivacaine in a patient with previous cardiac disease due to risk of local anesthetic systemic toxicity (LAST). B. Caution is advised when using liposomal bupivacaine in patients with severe hepatic dysfunction. C. Caution is advised when using liposomal bupivacaine in patients with severe renal dysfunction. D. Diabetic nephropathy can lead to a 1.5-fold increase in maximum plasma concentration of liposomal bupivacaine.

4. Which solution is the safest to use in diluting liposomal bupivacaine? A. Bupivacaine HCl B. Lidocaine 2% C. Sterile saline D. Sterile water 5. During which procedure has the use of liposomal bupivacaine resulted in patients with significantly less pain and less opioid requirements when compared with traditional bupivacaine? A. Inguinal hernia repair B. Hemorrhoidectomy C. Second-metatarsal osteotomy D. Total hip arthroplasty E. Total knee arthroplasty 6. A 72-year-old patient with hypertension, diabetes, and lower back pain with opioid tolerance had a bunionectomy and first-metatarsal osteotomy 12 hours ago. At that time, tissue infiltration with liposomal bupivacaine occurred. The patient reports increasing pain and the surgeon asks if a repeat injection of liposomal bupivacaine is possible at this time. How would you proceed? A. Repeat tissue infiltration with lidocaine B. Repeat tissue infiltration with bupivacaine HCl C. Repeat tissue infiltration with liposomal bupivacaine D. Wait 12 hours before repeat local anesthetic injection E. Wait 24 hours before repeat local anesthetic injection 7. Which statement is true about the pharmacokinetics of liposomal bupivacaine? A. After injection, about 10% of local anesthetic is present in the free form. B. Liposomal bupivacaine has a peak plasma concentration within the first hour after injection. C. Liposomal bupivacaine has two peaks in plasma concentration. D. Systemic absorption of local anesthetic depends solely on total dose of the medication injected.

27

Hadzic_Ch07_p027-030.indd 27

19/04/19 10:05 AM

28

PART 2

Foundations of Local and Regional Anesthesia

8. An 80-year-old man with hypertension, diabetes, and hyperlipidemia presents with a small bowel obstruction for exploratory laparoscopy, possible open laparotomy. Which of the following is true about use of liposomal bupivacaine for postoperative analgesia? A. Epidural injection of 266 mg of liposomal bupivacaine causes a higher incidence of motor blockade than bupivacaine 50 mg. B. Injection of liposomal bupivacaine should occur at least 20 minutes after wound infiltration of lidocaine. C. Liposomal bupivacaine has been extensively studied and is FDA approved for usage in lumbar epidurals. D. Transversus abdominis plane block with liposomal bupivacaine has been shown to significantly decrease pain and opioid usage compared to bupivacaine HCl. E. Transversus abdominis plane block with liposomal bupivacaine has been shown to significantly decrease pain and opioid usage compared to ropivacaine. 9. A 34-year-old man with a right rotator cuff tear presents for repair. The patient is enrolled in a study with SABER-bupivacaine used for tissue infiltration as the postoperative analgesic plan. Which of the following statements is true regarding Tmax of the medication? A. A large review of 11 trials demonstrated two peaks of Tmax at 1 hour and 12 hours. B. Tmax has been demonstrated at 72 hours in multiple trials. C. Tmax depends on ability of the delivery system to dissolve in situ. D. Tmax would be longer in abdominal surgery due to a larger surgical area. 10. Bupivacaine-collagen implant: A. Has been approved by the FDA for surgical site implantation B. Exhibits a monophasic peak of increased concentration C. Consists of a biodegradable and fully resorbable collagen matrix D. Has demonstrated significantly decreased pain scores at 24 and 48 hours when compared with bupivacaine

ANSWERS AND EXPLANATIONS 1. C is correct. Has a structure consisting of a multivesicular liposome. Each chamber in the multivesicular liposome is separated from adjacent chambers by lipid membranes. A is incorrect. The DepoFoam delivery system can reduce systemic exposure and toxicity by reducing peak serum levels of a drug. B and D are incorrect. Multivesicular liposome is distinguished structurally from unilamellar and multilamellar liposomes.

Hadzic_Ch07_p027-030.indd 28

2. A is correct. Has been shown to be nontoxic and biodegradable as the liposomes are composed of naturally occurring substances. B is incorrect. LA in liposomes has not been shown to consistently release medication over 36 hours. C is incorrect. Multiple hydrophobic and hydrophilic materials have been formulated in liposomes. DepoFoam has been used with cytarabine as DepoCyt(e). D is incorrect. Liposomes are unstable colloidal systems, due to both their larger physical size and the fact that they are prone to oxidation. 3. B is correct. Caution is advised when using liposomal bupivacaine in patients with severe hepatic dysfunction. A 1.5-fold increase in the maximum plasma concentration was seen in patients with moderate hepatic impairment following a single 300 mg infiltration of liposomal bupivacaine. A is incorrect. Liposomal bupivacaine does not increase the risk of LAST in patients with previous cardiac disease. C is incorrect. Approximately 6% of bupivacaine is excreted unchanged in urine; therefore no caution is advised in using liposomal bupivacaine in patients with severe renal function. D is incorrect. Diabetic nephropathy has not been shown to increase the maximum plasma concentration following a single 300 mg infiltration of liposomal bupivacaine. 4. C is correct. Sterile saline. Dilution with isobaric solution is recommended up to a maximum total volume of 300 mL. A is incorrect. Dilution with bupivacaine may cause the disruption of the carrier, accelerating release of bound bupivacaine and toxicity. B is incorrect. Dilution with lidocaine may cause the disruption of the carrier, accelerating release of bound bupivacaine and toxicity. D is incorrect. Hypobaric solutions such as sterile water may disrupt the liposomal carrier, leading to loss of sustained efficacy and high system drug levels. 5. B is correct. Hemorrhoidectomy. In a double-blind, randomized, controlled study by Erol Onel and colleagues, patients were found to have statistically significant less pain and opioid usage over the first 72 hours with liposomal bupivacaine when compared with bupivacaine HCl. A is incorrect. A study compared liposomal bupivacaine with bupivacaine HCL in hernia repairs. While there was no significant difference in pain scores between the two groups, the liposomal bupivacaine group demonstrated benefits for secondary endpoints. C is incorrect. In patients receiving a bunionectomy with first-metatarsal osteotomy, liposomal bupivacaine showed a statistically significant reduction in pain at 24 and 36 hours when compared with placebo.

19/04/19 10:05 AM

CHAPTER 7

D is incorrect. No studies have shown significantly less pain and opioid usage with liposomal bupivacaine in total hip arthroplasty. E is incorrect. Multiple studies have investigated analgesic efficacy of liposomal bupivacaine in wound infiltration after total knee arthroplasty. No study has shown a statistically significant difference in pain scores or opioid usage. 6. D is correct. Wait 12 hours before repeat local anesthetic injection. Repeat LA administration is not recommended within 24 hours following administration. A and B are incorrect. Additional LA of any kind is not recommended for 24 hours following liposomal bupivacaine administration. C is incorrect. Repeat tissue infiltration with liposomal bupivacaine is not recommended within 72 hours following initial dose. E is incorrect. Repeat LA injection can happen sooner than 36 hours. 7. C is correct. Liposomal bupivacaine has two peaks in plasma concentration. This is due to the fact that only 3% of the LA in liposomal bupivacaine is present in the free form. The first Tmax occurs within the first hour followed by a second within 12 hours. A is incorrect. After injection 3% is present in the free form. B is incorrect. Liposomal bupivacaine exhibits two peaks in plasma concentration. D is incorrect. Systemic absorption is due to total dose of drug administered, administration route, and vascularity of the administration site. 8. B is correct. Injection of liposomal bupivacaine should occur at least 20 minutes after wound infiltration of lidocaine. This is done to avoid possible toxic levels of lidocaine and bupivacaine. A is incorrect. A study by Eugene R. Viscusi et al. demonstrated that the incidence of motor blockade is less in patients receiving liposomal bupivacaine 266 mg via epidural versus bupivacaine HCl 50 mg via epidural. C is incorrect. Liposomal bupivacaine has not been approved by the FDA for usage in epidurals. D and E are incorrect. No study shows liposomal bupivacaine significantly decreases pain and opioid usage when compared to bupivacaine HCl and ropivacaine in transversus abdominis plane blocks. 9. D is correct. Tmax would be longer in abdominal surgery due to a larger surgical area. Studies have shown differing Tmax with surgical procedure, with Tmax being shorter in shoulder versus abdominal surgeries. A and B are incorrect. A large review of 11 clinical trials with both healthy subjects and those undergoing varied surgical procedures demonstrated a varied Tmax at 24–48 hours.

Hadzic_Ch07_p027-030.indd 29

Controlled-Release Local Anesthetics

29

C is incorrect. Dissolution of the SABER-bupivacaine eliminates the need for removal. 10. C is correct. Consists of a biodegradable and fully resorbable collagen matrix. Bupivacaine-collagen implant is a medication being developed for implantation in sites of surgical trauma to provide postsurgical analgesia. A is incorrect. Bupivacaine-collagen implant has not been approved by the FDA for surgical site implantation. B is incorrect. Bupivacaine-collagen implant, like liposomal bupivacaine, has demonstrated a biphasic peak of increased concentration. D is incorrect. No study has shown decreased pain scores at 24 and 48 hours with bupivacaine-collagen implant when compared with bupivacaine.

Suggested Readings Angst MS, Drover DR. Pharmacology of drugs formulated with DepoFoam: a sustained drug delivery system for parenteral administration using multivesicular liposome technology. Clin Pharmacokinet. 2006;45(12):1153-1176. Clinical trial no SKY0402-C-110. An open-label, phase I study to assess the pharmacokinetics and safety of SKY0402 in subjects with impaired hepatic function. Pacira Pharmaceuticals Inc., 2011. Exparel (Bupivacaine Liposome Extended-Release Injectable Suspension) [prescribing information]. Pacira Pharmaceuticals Inc., 2011. Gan T, et al. SABER-bupivacaine reduced pain intensity for 72 hours following abdominal surgery relative to bupivacaine HCl. Presented at 2014 Annual Meeting of the American Society of Anesthesiologists, October 15, 2014, New Orleans, LA. Golf M, et al. A phase 3, randomized, placebo-controlled trial of DepoFoam bupivacaine (extended release bupivacaine local analgesic) in bunionectomy. Adv Ther. 2011;28(9): 776-788. Hadzic A. Abikhaled JA, Harmon WJ. Impact of volume expansion on the efficacy and pharmacokinetics of liposome bupivacaine. Local Reg Anesth. 2015;8:105-111. Hadzic A. Controlled-release local anesthetics. In: Pozek J-P J, Beausang D, Segna KG, Viscusi ER, eds. Textbook of Regional Anesthesia and Acute Pain Management. 2nd ed. New York, NY: McGraw-Hill Publishing; 2017:chap 8. Howell SB. Clinical applications of a novel sustained-release injectable drug delivery system: DepoFoam technology. Cancer J. 2001;7(3):219-227. Hu D, et al. Pharmacokinetic profile of liposome bupivacaine injection following a single administration at the surgical site. Clin Drug Investig. 2013;33:109-115. Kulkarni PR, et al. Liposomes: a novel drug delivery system. Int J Curr Pharm Res. 2011;3(2):10-18. Langford RM, et al. A single administration of depobupivacaine intraoperatively results in prolonged detectable plasma bupivacaine and analgesia in patients undergoing inguinal hernia repair. Presented at 62nd Postgraduate Assembly in Anesthesiology, December 12-16, 2008, New York, Poster 9088.

19/04/19 10:05 AM

30

PART 2

Foundations of Local and Regional Anesthesia

Marcaine (Bupivacaine HCl) [US prescribing information]. Hospira Inc., 2009. Onel E, et al. Exparel, a liposomal bupivacaine local analgesic, extends pain relief and decreases opioid use. Presented at Annual Meeting of the American Society of Anesthesiologists, October 16–20, 2010, San Diego, CA. Sekar M, et al. Drug delivery of biologics: a controlled release strategy. Presented at IBV’s 17th Annual TIDES Conference, May 3-6, 2015, San Diego, CA.

Hadzic_Ch07_p027-030.indd 30

Shah J, et al. The PK profile of SABER-bupivacaine in humans across surgical models demonstrates sustained 72-hour drug delivery. Presented at 2014 Annual Meeting of the American Society of Anesthesiologists, October 15, 2014, New Orleans, LA. Viscusi ER, Candiotti KA, Onel E, Morren M, Ludbrook GL. The pharmacokinetics and pharmacodynamics of liposome bupivacaine administered via a single epidural injection to healthy volunteers. Reg Anesth Pain Med. 2012;37(6):616-622.

19/04/19 10:05 AM

8 Analgesic Adjuvants in the Peripheral Nervous System Colin J. L. McCartney and Stephen Choi

QUESTIONS DIRECTIONS: Choose the one best response to each question. 1.  Which of the following is not a chemical mediator of tissue damage/inflammation causing nociceptive pain in peripheral nerves? A. Bradykinin B. Somatostatin C. Serotonin D. Glutamate 2.  The suppression of which currents cause attenuation of substance P release? A. Mg2+ B. PO43C. K+ D. Ca2+ and Na+ 3.  The perineural administration of which opioid in conjunction with local anesthetics have demonstrated analgesic efficacy and increase in duration of analgesia? A. Buprenorphine B. Morphine C. Fentanyl D. Sufentanil 4.  Which perineural adjuvant (in conjunction with local anesthetic) is approved for perineural use in peripheral nerve blocks?

A. Dexamethasone B. Clonidine C. Epinephrine D. Buprenorphine 5.  Which of the following is not a postulated mechanism for the action of perineural dexamethasone? A. Reducing inflammatory mediators B. Reducing ectopic neuronal discharge C. Suppressing K+ nociceptive C-fiber activity D. Suppressing Na+ mediated action potentials 6.  Which of the following is a side effect of perineural clonidine administration? A. Hyperglycemia B. Agitation C. Hypotension D. Respiratory depression

ANSWERS AND EXPLANATIONS 1.  B is correct. Somatostatine is not a chemical mediator. Chemical mediators in a wide array are produced in the peripheral nervous system and have both excitatory and inhibitory influences on peripheral sensory nerve transmission. Figure 8–1 includes a list of the mediators released by tissue injury and inflammation and a variety of agents acting on neuroreceptors. Bradykinin, serotonin, and glutamate have an excitatory influence while somatostatin has an inhibitory influence.

31

Hadzic_Ch08_p031-034.indd 31

22/04/19 1:30 PM

32

PART 2

Foundations of Local and Regional Anesthesia Immune cells

Inhibitory Influences: Opioids(μ, δ, κ) α2-Adrenoceptor(α2c) Adenosine (A1) Cannabinoids (CB1, CB2)

GABA (GABAB) Orphinan (ORL1) Somatostatin

Mast cell

Platelets

Sensory neuron Tissue Injury Inflammation

Dorsal root ganglion Blood vessel

Spinal cord Excitatory Influences: Prostanoids (EP, IP) Bradykinin (B1, B2) Histamine (H1) Serotonin (5-HT1, 5-HT2, 5-HT3, 5-HT4) ATP (P2X3)

α2-Adrenoceptor(α2A) Glutamate (NMDA, AMPA, KA) Acetylcholine (N) Adenosine (A2A, A3) Tachykinins (NK1, NK2) Nerve growth factor (TrkA)

Sympathetic nerves

FIGURE 8–1  Excitatory and inhibitory influences on peripheral nerve activity by mediators released by tissue injury and inflammation and by a variety of agents acting on neuroreceptors. AMPA = α-amino-3-hydroxy-5-methylisoxazole-4-propionic acid; KA = kainic acid; NMDA = N-methyl-d-aspartate; NK = neurokinin; TrkA = Tropomyosin receptor kinase A.

2.  D is correct. Opioid receptors and neuropeptides (eg, substance P) are synthesized in the dorsal root ganglion and transported along intra-axonal microtubules into central and peripheral processes of the primary afferent neuron. At the terminals, opioid receptors are incorporated into the neuronal membrane and become functional receptors. On activation by exogenous or endogenous opioids, opioid receptors couple to inhibitory G proteins. This leads to direct or indirect suppression of Ca2+ or Na+ currents and subsequent attenuation of substance P release. 3.  A is correct. The two opioid agonists that have demonstrated analgesic efficacy when administered perineuronally are buprenorphine and tramadol. Buprenorphine is a partial μ-receptor agonist with a very high receptor affinity compared with fentanyl (24-fold) or morphine (50-fold). In addition, it has intermediate lipid solubility, which allows it to cross the neural membrane.1,2 Candido and colleagues3 added 0.3 mg buprenorphine (a partial opioid agonist) to a combination of mepivacaine and tetracaine in axillary block and found an almost 100% increase in the duration of analgesia compared with the administration of axillary block plus the same dose of intramuscular buprenorphine with no significant increase in adverse

Hadzic_Ch08_p031-034.indd 32

effects. This supports the peripheral analgesic effect of buprenorphine and the earlier findings of two studies that examined buprenorphine without a systemic control group.4,5 4.  C is correct. Perineural adjuvants are all off-label use of medications except for epinephrine. 5.  D is correct. Suppressing Na+ mediated action potentials is not a postulated mechanism for the action of perineural dexamethasone. Perineural corticosteroids are thought to exert their effect by several mechanisms, including attenuating the release of inflammatory mediators, reducing ectopic neuronal discharge, and inhibiting potassium channel–mediated discharge of nociceptive C fibers.6-8 6.  C is correct. More recently, a meta-analysis by Popping and colleagues estimated that clonidine prolonged postoperative analgesia, sensory block, and motor block by 122, 74, and 141 minutes, respectively.9 Clonidine, however, also increased the probability of hypotension (odds ratio [OR] 3.61), fainting (OR 5.07), sedation (OR 2.28), and bradycardia (OR 3.09). There was no observed dose response between a range of 30 and 300 μg, with the majority receiving 150 μg.

22/04/19 1:30 PM

CHAPTER 8

References 1. Gutstein H, Akil H. Opioid analgesics. In: Hardman J, Limbird L, eds. Goodman & Gilman’s The Pharmacologic Basis of Therapeutics. 10th ed. McGraw-Hill; 2001:601. 2. Lanz E, Simko G, Theiss D, et al. Epidural buprenorphine—A double-blind study of postoperative analgesia and side effects. Anesth Analg. 1984;63:593-598. 3. Candido KD, Winnie AP, Ghaleb AH, et al. Buprenorphine added to the local anesthetic for axillary brachial plexus block prolongs postoperative analgesia. Reg Anesth Pain Med. 2002;27: 162-167. 4. Candido KD, Franco CD, Khan MA, et al. Buprenorphine added to the local anesthetic for brachial plexus block to provide postoperative analgesia in outpatients. Reg Anesth Pain Med. 2001;26:352-356. 5. Viel EJ, Eledjam JJ, De La Coussaye JE, et al. Brachial plexus block with opioids for postoperative pain relief: comparison between buprenorphine and morphine. Reg Anesth. 1989;14: 274-278.

Hadzic_Ch08_p031-034.indd 33

33

Analgesic Adjuvants in the Peripheral Nervous System

6. Attardi B, Takimoto K, Gealy R, Severns C, Levitan ES. Glucocorticoid induced up-regulation of a pituitary K+ channel mRNA in vitro and in vivo. Receptors Channels. 1993;1:287-293. 7. Eker HE, Cok OY, Aribogan A, Arslan G. Management of neuropathic pain with methylprednisolone at the site of nerve injury. Pain Med. 2012;13:443-451. 8. Johansson A, Hao J, Sjolund B. Local corticosteroid application blocks transmission in normal nociceptive C-fibres. Acta Anaesthesiol Scand. 1990;34:335-338. 9. Popping DM, Elia N, Marret E, Wenk M, Tramer MR. Clonidine as an adjuvant to local anesthetics for peripheral nerve and plexus blocks: a meta-analysis of randomized trials. Anesthesiology. 2009;111(2):406-415.

Suggested Reading Hadzic A. Analgesic adjuvants in the peripheral nervous system. In: McCartney CJL, Choi S, eds. Textbook of Regional Anesthesia and Acute Pain Management. 2nd ed. New York, NY: McGraw-Hill Publishing; 2017:chap 9.

22/04/19 1:30 PM

This page intentionally left blank

Hadzic_FM_p00i-pxx.indd 6

22/04/19 6:40 PM

9 Local Anesthetic Mixtures for Peripheral Nerve Blocks Jason Choi

QUESTIONS DIRECTIONS: Choose the one best response to each question. 1.  In general, local anesthetics (LAs) with the following property tend to have shorter latency of action: A. High lipid solubility B. Low pKa C. High ratio of ionized state D. Large molecular weight 2.  Which property of local anesthetics (LAs) most significantly prolongs duration of action? A. Protein binding B. High lipid solubility C. High molecular weight D. Greater vascularity at site of injection 3.  Previous studies of landmark-based techniques of peripheral nerve blocks (PNBs) examining mixtures of local anesthetics (LAs) have shown the following: A. Mixing of LAs was shown to be ideal with the benefits of faster onset of short-acting LAs and the prolonged duration of long-acting LAs. B. Use of bupivacaine for PNB was shown to have consistent and predictable onset. C. Differences in onset of long-acting LAs versus mixtures of LAs were shown to be not significant. D. General conclusions are difficult to reach due to the small number of studies and inconsistent methodologies. 4.  Recent studies of mixing local anesthetics (LAs) for ultrasound-guided peripheral nerve blocks (PNBs) show: A. Compared to landmark-based techniques, there is an overall decrease in latency, even for long-acting LAs. B. Duration of block of mixed LA is similar to the duration of the long-acting LA.

C. There is clinically significant difference in onset time between short-acting versus mixed agents or long-acting. D. The sequence of injecting short-acting or long-acting LA was significant in affecting onset or duration. 5.  With local anesthetic systemic toxicity and the mixing of local anesthetics (LAs), the following is true: A. Larger doses can be administered than would be with single agents. B. Toxicity is additive and fractions must be individually calculated. C. Mixtures of LAs work synergistically and overall result in lower total equivalent doses to reach toxic levels. D. Ultrasound-based techniques do not change risk of toxicity.

ANSWERS AND EXPLANATIONS 1.  B is correct. Low pKa. LAs with lower pKa tend to have faster onset of action. Because local anesthetics are weak bases, a lower pKa will favor the nonionized form at physiologic pH. The more nonionized drug present at the lipid membrane, the more the drug can permeate the sodium channel of the nerve. A is incorrect. Highly lipid soluble LAs tend to have slower latency (eg, bupivacaine). An exception to the rule is etidocaine. C is incorrect. The greater the ratio of ionized state of drug, the more difficult it is to cross lipid membranes and inhibit the sodium channel of the nerve axons. D is incorrect. While molecular weight inversely affects the aqueous diffusion rate and thus onset of action, most LAs are similar in size.

35

Hadzic_Ch09_p035-036.indd 35

18/04/19 3:34 PM

36

PART 2

Foundations of Local and Regional Anesthesia

2.  B is correct. High lipid solubility. Block duration is mostly determined by the extent that the LA remains in the vicinity of the nerve. High lipid solubility allows the drug to remain in or near lipid membranes rather than be absorbed to continue its action on the membrane sodium channels. A is incorrect. Degree of protein binding has little clinically relevant effect on block duration. This is due to the fact that dissociation times of LAs from sodium channels are measured in seconds. C is incorrect. Molecular weight does not factor into duration of action. D is incorrect. Greater vascularity of block site increases systemic absorption of the LA and shortens its duration. Decreasing vascularity or vascular uptake through the addition of vasoconstrictors such as epinephrine will prolong the drug’s presence at the site and increase its duration. 3.  D is correct. Surprisingly, conclusions regarding mixtures of LAs are difficult to reach due to relatively small number of studies, hetergeneous methodologies, and mixed conclusions. Often techniques varied and controls were lacking. A is incorrect. While mixing of LAs conferred the benefits of rapid onset and extended duration, often the duration of PNB was compromised versus using long-acting agents alone.1,2 B is incorrect. Latency with bupivacaine was often shown to be less predictable and varied.3 Often mixtures of LAs or use of short-acting LAs had significantly shorter latency.2 C is incorrect. Although methodologies were inconsistent and block site heterogeneous, the onset of block using bupivacaine was often significantly slower than mixtures with lidocaine or chloroprocaine.1,2 4.  A is correct. Recent studies of ultrasound-guided injections of LAs show an overall decrease in latency times for long-acting anesthetics such as bupivacaine or ropivacaine. The majority of studies show decreased time to block onset. The precision of block placement afforded with ultrasound guidance has resulted in faster onset and lower minimum volumes of drug.4 B is incorrect. Recent studies of LA mixtures have demonstrated high variability and low predictability in duration of sensory and motor block. C is incorrect. While there is a statistically significant difference in latency of mixtures of LA versus long-acting LA only, the difference in clinical terms could be considered minimal (12 minutes versus 6 minutes).5 Of note, the downside to this few-minute advantage in onset is about a 50% reduction in duration versus long-acting drug alone.

Hadzic_Ch09_p035-036.indd 36

D is incorrect. Gadsden et al in a study comparing sequencing of mepivacaine and bupivacaine showed identical onset times and duration for either sequence groups. Sequencing does not seem to have any significant role in affecting PNB characteristics. 5. B is correct. Toxicity of LAs, when using mixtures, is additive and individual fractions must be calculated. A is incorrect. Compounding LAs does not decrease toxicity. The toxicity of each individual drug is additive and must be calculated. However, the calculation of each individual component to reach toxicity is unclear. C is incorrect. There is no strong evidence that compounding LAs work synergistically and may lower overall toxic threshold.6,7 D is incorrect. Ultrasound-based techniques have decreased the risk of LA systemic toxicity due to the increased precision and lower doses needed for clinical efficacy.8

References 1. Bromage PR, Gertel M. Improved brachial plexus blockade with bupivacaine hydrochloride and carbonated lidocaine. Anesthesiology. 1972;36:479-487. 2. Cuvillon P, Nouvellon E, Ripart J, et al. A comparison of the pharmacodynamics and pharmacokinetics of bupivacaine, ropivacaine (with epinephrine) and their equal volume mixtures with lidocaine used for femoral and sciatic nerve blocks: a double-blind randomized study. Anesth Analg. 2009;108:641-649. 3. Cunningham NL, Kaplan JA. A rapid-onset, long-acting regional anesthetic technique. Anesthesiology. 1974;41:509-511. 4. Salinas FV, Hanson NA. Evidence-based medicine for ultrasound-guided regional anesthesia. Anesthesiol Clin. 2014;32:771-787. 5. Laur JJ, Bayman EO, Foldes PJ, Rosenquist RW. Triple-blind randomized clinical trial of time until sensory change using 1.5% mepivacaine with epinephrine, 0.5% bupivacaine, or an equal mixture of both for infraclavicular block. Reg Anesth Pain Med. 2012;37:28-33. 6. Daos FG, Lopez L, Virtue RW. Local anesthetic toxicity modified by oxygen and by combination of agents. Anesthesiology. 1962;23:755-761. 7. Spiegel DA, Dexter F, Warner DS, Baker MT, Todd MM. Central nervous system toxicity of local anesthetic mixtures in the rat. Anesth Analg. 1992;75:922-928. 8. Barrington MJ, Kluger R. Ultrasound guidance reduces the risk of local anesthetic systemic toxicity following peripheral nerve blockade. Reg Anesth Pain Med. 2013;38:289-297.

Suggested Reading Hadzic A. Local anesthetic mixtures for peripheral nerve blocks. In: Choi J, Gadsden J, eds. Textbook of Regional Anesthesia and Acute Pain Management. 2nd ed. New York, NY: McGraw-Hill Publishing; 2017:chap 10.

18/04/19 3:34 PM

10 Continuous Peripheral Nerve Blocks: Local Anesthetic Solutions and Infusion Strategies Colleen Mccally

QUESTIONS DIRECTIONS: Choose the one best response to each question. 1. When making a decision for your patient regarding the effects of a continuous femoral nerve block, which has the greater influence on continuous nerve blocks? A. Calculating the total dose delivered B. Selecting the appropriate concentration C. Choosing an additive to extend its duration D. Choosing a long-acting local anesthetic 2. Local anesthetics (LAs) are delivered continuously. When selecting a delivery regimen, what is your goal? A. Decrease the total amount of LA consumption B. Decrease additional opioid requirements C. Decrease the disturbance of daily function such as sleep D. All of the above 3. A patient is having a total knee replacement. What has been shown to decrease the basal infusion rate of a continuous LA? A. Adding opioids preoperatively B. Giving the patient intravenous Decadron intraoperatively C. Adding patient-controlled boluses of LA D. Administering non-opioid-based medication preoperatively 4. A patient who is scheduled for an arthroscopic rotator cuff repair will be discharged home with a continuous interscalene catheter with patient-controlled boluses. Explain the benefit of the catheter with patientcontrolled boluses to this patient. A. Allows the patient to be in control of his or her pain management B. Decreases the incidence of an insensate extremity

C. Decreases local anesthetic consumption D. Allows for longer infusion duration in an ambulatory setting E. All of the above 5. Select if the statement is true or false: A. At this time, data to supports adding additives (adjuvants) to local anesthetic for perineural infusions. B. Analgesia is optimized with an infusion pump that delivers both an adjustable basal rate and patientcontrolled bolus doses. C. The “optimal” concentration of local anesthetic for infusion is ropivacaine 0.5%. D. Studies have suggested that sensory-and-motor block regresses faster with ropivacaine than with bupivacaine 6. Which regimen of lower extremity continuous nerve blocks has been proven to be most beneficial to patients? A. Findings of RCTs at the femoral or fascia iliaca location have not demonstrated an overwhelming preference for a specific regimen. B. A superior analgesic for femoral and fascia iliaca continuous nerve block has been proven with a continuous basal infusion. C. A superior analgesic for femoral and fascia iliaca continuous nerve block has been proven for scheduled hourly boluses. D. None of the above 7. What is the ideal goal of adding adjuvant to infusate? A. Improve analgesic quality B. Spare LA consumption C. Minimize motor block D. Decrease the amount of postoperative phone calls E. Answers A, B, and C

37

Hadzic_Ch10_p037-040.indd 37

22/04/19 1:33 PM

38

PART 2

Foundations of Local and Regional Anesthesia

8. You are an anesthesiologist. A pharmaceutical company representative asks you for the most important goals for a device pump to deliver LA. What is your answer? A. Accurate delivery and dose B. Reliability C. Portable device D. All of the above 9. What are the national pharmacologic guidelines for preparing a regional anesthesia pump? A. The pump must be filled within 24 hours of use. B. The pump reservoir must be filled in an “ISO class 5” environment. C. The person filling a regional anesthetic pump must have a pharmacist degree. D. The person filling a regional anesthetic pump must wear sterile gloves. 10. As the anesthesiologist in charge of your local ambulatory surgery center, your supply manager asks which of the following are true when selecting a regional anesthetic pump for your patient. What do you answer? A. Elastomeric pumps will infuse 5% of the set basal rate in the first 3–8 hours. B. Electronic pumps are reported to be the most accurate within 5% of the programmed basal rate. C. In elastomeric pumps, the physics of the internal reservoir allow for the ability to refill the pump without difficulty. D. Elastomeric pumps tend to emit noises.

ANSWERS AND EXPLANATIONS 1. A is correct. Calculating the total dose has the greatest effect on femoral nerve block. B is incorrect. Evidence suggests that for infusions involving the femoral nerve block, LA concentration is of minimal importance compared with the total dose. C is incorrect. Additives have proven to extend the duration of the peripheral nerve block; however, the total dose most influences the peripheral nerve block. D is incorrect. When choosing a long-acting agent, it is desirable for sensory and motor block to resolve quickly and predictably at the termination of infusion. It remains unknown if there is an “optimal” concentration of local anesthetic. For femoral nerve block evidence suggest total dose being of more importance. 2. D is correct. Adding a patient-controlled bolus, to a basal infusion, usually decreases the required basal infusion rate, the incidence of an insensate extremity, and local anesthetic consumption, the last allowing for a longer infusion duration in the ambulatory setting. At the infraclavicular location utilizing a basal infusion with bolus also has been found to decreased sleep disturbances.

Hadzic_Ch10_p037-040.indd 38

3. C is correct. Adding patient-controlled boluses of LAs has been shown to decrease the basal infusion rate of continuous LA. A is incorrect. Adding opioids preoperatively, preemptive analgesic has been proven to decrease postoperative pain. B is incorrect. Administering intravenous Decadron intraoperatively has been shown to have similar effects on duration as being added to the peripheral nerve block LAs. D is incorrect. Administering non-opioid-based medication preoperatively has been studied extensively and has been shown to decrease opioid requirements. 4. A, B, C, and D are all true, so option E is correct. An infusion regimen that includes basal infusion and patientcontrolled bolus minimizes breakthrough pain, supplemental analgesic requirements, decreases the required basal infusion rate, the incidence of an insensate extremity, and local anesthetic consumption. The last allows for a longer infusion duration in the ambulatory setting. 5. A is false. At this time, there are few data to support adding additives (adjuvants) to local anesthetic for perineural infusions. B is true. Analgesia is optimized with an infusion pump that delivers both an adjustable basal rate and patientcontrolled bolus doses. C is false. At this time it remains unknown if there is an “optimal” concentration of local anesthetic. Commonly described concentrations include ropivacaine 0.1%–0.4%. D is true. Studies have suggested that sensory-andmotor block regresses faster with ropivacaine than with bupivacaine. 6. A is correct. Findings of the RCTs at the lower extremity have not been able to demonstrate a preference for a specific regimen. B, C, and D are incorrect. Sensory and motor effects are similar when comparing repeated, scheduled hourly bolus doses to a continuous basal infusion of the same hourly volume and dose. Nearly all studies reported the total local anesthetic dose reduced with bolus-only dosing. 7. E is correct. Improving the quality of a block enhances having an insensate extremity. Decreasing LA consumption is extremely important to decrease the incidence of LA toxicity, and minimizing a motor block is key especially when approaching postoperative pain. Patients are recovering faster and expected to ambulate especially when a motor block is not part of postoperative care. D is incorrect. Decreasing postoperative phone calls is part of policy planning when running a postoperative pain service. However, it is not part of the indication for an additive for a postoperative pain pump.

22/04/19 1:33 PM

CHAPTER 10

Continuous Peripheral Nerve Blocks: Local Anesthetic Solutions and Infusion Strategies

8. D is correct. Creating a device that is an accurate delivery system is extremely important. If the delivery system was not consistent or reliable, you could potentially be delivering an unsafe amount of LA or not delivering enough to provide an acceptable amount of anesthetic. Having a nonbulky and easily portable carrying device for LA delivery is of equal importance. 9. B is correct. National pharmacologic guidelines require the pump reservoir to be filled in an “ISO class 5” environment. This is for the patient’s safety to prevent contamination of the LA delivery system. A is incorrect. The pump reservoir should be filled according to the duration of the infusion. Infusions greater than 48 hours have been associated with increased infection risk. C is incorrect. Although commonly filled by pharmacists, it is not necessary to be a pharmacist to prefill a LA pump. D is incorrect. Wearing sterile gloves is not part of the national pharmacologic guidelines.

Hadzic_Ch10_p037-040.indd 39

39

10. B is correct. Electronic pumps are more accurate when setting the basal rate and more consistent over a scheduled period of time. A is incorrect. Elastomeric pumps will actually infuse to a less accurate degree 110%–130% of the set basal rate in the first 3–8 hours. Although commonly used, this should be considered when sending a patient home or to the hospital floor with a LA pump. C and D are incorrect. Due to the physics of the creation of an elastomeric pump, the internal reservoir is not easily refilled. Based on their construction, they do not emit noises. Electronic pumps, on the other hand, tend to emit noises that can be a nuisance when patients are trying to sleep.

Suggested Reading Hadzic A. Continuous peripheral nerve blocks: local anesthetic solutions and infusion strategies. In: Monahan AM, Ilfeld BM, eds. Textbook of Regional Anesthesia and Acute Pain Management. 2nd ed. New York, NY: McGraw-Hill Publishing; 2017:chap 11.

22/04/19 1:33 PM

This page intentionally left blank

Hadzic_FM_p00i-pxx.indd 6

22/04/19 6:40 PM

Section 3

Equipment for Peripheral Nerve Blocks Chapter 11

Equipment for Regional Anesthesia  43

Chapter 12

Equipment for Continuous Peripheral Nerve Blocks  49

Chapter 13

Electrical Nerve Stimulators and Localization of Peripheral Nerves  53

Hadzic_Ch11_p041-048.indd 41

22/04/19 1:35 PM

This page intentionally left blank

Hadzic_FM_p00i-pxx.indd 6

22/04/19 6:40 PM

11 Equipment for Regional Anesthesia Vivian H. Y. Ip and Ban C. H. Tsui

QUESTIONS DIRECTIONS: Choose the one best response to each question. 1. A 30-year-old pregnant woman is scheduled for an elective caesarian delivery. You have decided to perform a spinal anesthetic for her surgery. What type of spinal needle causes less risk of postdural puncture headache? A. Labat B. Pitkin C. Quincke D. Whitacre 2. A 50-year-old man weighing 120 kg was scheduled for a debridement of an infected wrist. A supraclavicular block was performed under ultrasound guidance and nerve stimulator with a threshold of 0.2 mA. Thirty mL of 0.5% ropivacaine was injected after negative aspiration for blood. Ten minutes later, the patient started getting a little restless and began having jerky movement. He also complained of tingling in the mouth. Blood pressure and heart rate remain stable. After managing airway, breathing, and circulation, what do you reach for from the nerve block cart to get ready to administer? A. Epinephrine B. Intralipid 20% C. Midazolam D. Propofol 3. A 79-year-old man consented to have a supraclavicular block with sedation for his upcoming hand surgery for flexor tendon repair. He is a vague historian and does not understand English very well. He has been giving inconsistent responses while doing a sensory testing with ice. What is the most objective method to assess his surgical block that does not require patient feedback? A. Current perception threshold B. Pain assessment C. Sensory testing with graded filament D. Temperature/infrared recording

4. When performing peripheral nerve block with ultrasound guidance, what is the role of peripheral nerve stimulators? A. For additional monitoring of needle position B. No role for peripheral nerve stimulators because they are not accurate C. No role for peripheral nerve stimulators because ultrasound can provide a clear image of the nerve(s) D. None of the above 5. For a 60-year-old woman of Asian descent who speaks limited English, which pain assessment scale should be used to assess how the pain affects her everyday living to better define pain severity? A. Defense and Veterans Pain Rating Scale (DVPRS) B. Face Pain Scale (FPS) C. Numeric Rating Scale (NRS) D. Pain Assessment Checklist for Seniors with Limited Ability to Communicate (PACSLAC) 6. During injection, what is the injection opening pressure dependent upon? A. Needle size B. Needle type C. Injection speed D. Syringe size E. None of the above 7. A 68-year-old woman is scheduled for a small bowel resection. She speaks limited English, but with the help of her daughter for translation consent to an epidural for postoperative pain relief was obtained. An epidural catheter was sited successfully and you would like to test to see whether the epidural is positioned correctly. You cannot contact the daughter or a translator. What is the best and most objective option to assess the effectiveness of her epidural? A. I cannot check the block preoperatively so I can use the epidural intraoperatively as if it is placed correctly and see if the patient expresses pain in recovery. B. Miller Fisher test C. Test for sensory block with a sharp pin 20 minutes after local anesthetic (LA) is injected via the epidural and look for any grimacing. D. Epidural stimulation (Tsui) test

43

Hadzic_Ch11_p041-048.indd 43

22/04/19 1:35 PM

44

PART 2

Foundations of Local and Regional Anesthesia

8. When performing peripheral nerve block(s), which are the reliable techniques for measuring opening injection pressure? A. In-line pressure manometer, compressed air injection technique (CAIT), and syringe feel B. In-line pressure manometer and compressed air injection technique (CAIT) C. In-line pressure manometer and syringe feel D. Compressed air injection technique (CAIT) and syringe feel 9. When performing an interscalene nerve block with ultrasound guidance and concurrent use of a nerve stimulator, what is the nerve stimulation threshold, less than which may suggest intraneural needle-tip location or needle-nerve contact? A. 0.2 mA with a pulse width of 0.1 ms B. 0.5 mA with a pulse width of 0.1 ms C. 1.0 mA with a pulse width of 0.1ms D. 2.0 mA with a pulse width of 0.1 ms 10. Why is it important to measure injection pressure when performing nerve block(s)? A. To ensure the opening pressure is kept below 15 psi B. To help distinguish needle-tip location in the perineural tissue versus needle-nerve contact or intrafascicular needle placement C. To reduce the likelihood of undesired neuraxial spread during regional blocks close to the neuraxis D. All of the above 11. When performing peripheral nerve block using the compressed air injection technique (CAIT), how much air (above the injectate) is compressed during fluid injection for the injection pressure to be within the safe limit of less than 760 mm Hg? A. 50% B. 75% C. 100% D. None of the above 12. During a popliteal nerve block, the BSmartTM (B-Braun Medical, Melsungen, Germany) pressure monitor is placed between the nondistensible tubing, proximal to the needle, and the syringe. Which of the following is true regarding the injection pressure? A. If the manometer is in the white zone of < 15 psi, the injection pressure is too low for a successful block. B. If the manometer is in the yellow zone of 15–20 psi, the injection pressure suggests forceful needle-nerve contact. Injection may result in inflammatory changes. Therefore, slight withdrawal of the needle is warranted. C. If the manometer is in the yellow zone of 15–20 psi, the injection pressure is good for injection of local anesthetic. D. If the manometer is in the green zone of > 20 psi, the injection pressure is at the correct range and injection of local anesthetic should continue. 13. What statement is true regarding the difference between the catheter-over-needle design and traditional perineural catheters?

Hadzic_Ch11_p041-048.indd 44

A. The catheter-over-needle design results in more leakage compared to a traditional catheter. B. The catheter-over-needle design results in less leakage versus a traditional catheter. C. For a catheter-through-needle assembly, the needle is within the catheter, which provides a snug-fitting catheter to the skin even when the needle is removed. D. The catheter tip is much more predictable in traditional perineural catheters compared to the catheter-over-needle design. E. None of the above 14. You are performing a popliteal nerve block catheter in a patient scheduled for an ankle fusion surgery. A catheter tray was set up and a 51 mm catheter-overneedle assembly was opened. When you look with the ultrasound, you find that the sciatic nerve is 5 cm deep. What would you do? A. Change the catheter for a 75 mm or 83 mm catheterover-needle set. B. Change my technique to a single-shot nerve block rather than a catheter approach. C. Use the catheter that is opened because I can still reach the target, but push onto the skin when performing the nerve block. D. None of the above 15. What is the range of current necessary for the epidural stimulation test? A. 0 to 1.5 mA B. 0 to 5 mA C. 1 to 5 mA D. 1 to 10 mA or higher 16. What polarity should the stimulation electrode (ie, needle tip) be when performing peripheral nerve blocks with a nerve stimulator? A. Anode (red) should be selected because it is more effective than cathode (black) at depolarizing the nerve membrane. B. Cathode (black) should be selected because it is more effective than anode (red) at depolarizing the nerve membrane. C. It makes no difference because both anode and cathode are equally effective at depolarizing the nerve membrane.

ANSWERS AND EXPLANATIONS 1. D is correct. Atraumatic needles such as pencil-point tip Whitacre or Sprotte needles are associated with less risk of postdural puncture headache.1,2 A is incorrect. Gaston Labat was one of the pioneers of the widespread acceptance of spinal anesthesia in the 1920s in both Europe and the United States.3 B is incorrect. It was a medium-gauge, short, sharpbeveled cutting spinal needle with a stylet. C is incorrect. The Quincke spinal needle is a cutting needle.

22/04/19 1:35 PM

CHAPTER 11

2. B is correct. The symptoms of this patient suggest LA toxicity. Supraclavicular block is a vascular area with potential for LA absorption. Patients who are moribund with infections may have metabolic acidosis, and in turn be predisposed to LA toxicity. The correct management once airway is established is the administration of Intralipid 20% intravenous bolus dose followed by an infusion. Therefore, Intralipid should always be available in the cart where nerve blocks are performed. A is incorrect. Should there be cardiac or hemodynamic compromise, basic/advanced life support should be initiated and epinephrine will then be appropriate. C is incorrect. If the symptoms progress to seizure, benzodiazepine should be administered. D is incorrect. Propofol might be used for seizure suppression but should not be used if there is cardiovascular instability. 3. D is correct. Infrared thermal imaging has been tested as a means to monitor block progression.4,5 This test is based on the skin temperature in the digits increasing following brachial plexus block. Studies have demonstrated that infrared thermography of the digits had a high positive predictive value for block success after brachial plexus block. This test does not require feedback from patients. A is incorrect. The current perception threshold is to test sensory level by applying electrical current via a percutaneous electrode connected to a specialized current generator. The reproducibility of this method has been tested in volunteers with acceptable results using a common peripheral nerve stimulator.6 If the current required to elicit a sensory response was greater over time than the baseline (preblock and unblocked region) current, this was an indicator of block progression. However, this method does need feedback from the patient. B is incorrect. Pain assessment will be useful if the patient had pain prior to the block such that comparison can be made. However, it is still a subjective measure. There are several types and some include facial cartoons, such as the Defense and Veterans Pain Rating Scale (DVPRS).7 There is also a scale for elderly patients known as the Pain Assessment Checklist for Seniors with Limited Ability to Communicate (PACSLAC),8 which can be used to assess pain in individuals with dementia or cognitive impairment, and who have communication problems. C is incorrect. Sensory testing with grade filament still requires patient cooperation and feedback. Therefore, this method has the same problem that occurs with sensory testing using ice. 4. A is correct. The combined use of ultrasound and nerve stimulation creates a more objective method of achieving accurate and safe blocks while allowing monitoring and visualization of the block needle and targets in real time. With the introduction of ultrasound, the role of nerve stimulators has changed from nerve seeking to monitoring for needle-nerve contact or intraneural needle tip placement.

Hadzic_Ch11_p041-048.indd 45

45

Equipment for Regional Anesthesia

B is incorrect. There are several reasons for a peripheral nerve stimulator to malfunction or be inaccurate. If noninsulated needles are used, they transmit current throughout their entire shaft; the current density at the tip is therefore much lower than with insulated needles and a higher current is required for nerve stimulation. The use of injectates can also affect the current density at the needle tip. The use of nonelectrolyte/nonconducting injectates, for example, dextrose 5% in water (D5W), reduces the conductive area and increases the current density at the needle tip with resulting maintenance or augmentation of the motor response at a low current. However, the use of conducting injectates, such as normal saline or local anesthetic, increases the conductive area at the needle tip and increases the threshold current requirement.9 C is incorrect. Ultrasound should not be used as the sole guidance device, but, rather, should be used in conjunction with other monitoring modalities to minimize the risk of intraneural injection.10,11 This is because nerve swelling as a result of intraneural injection can be difficult to note in real time with ultrasound. By the time nerve swelling is noticed, it may be too late to prevent nerve injury as it takes only a miniscule volume of local anesthetic to rupture the perineurium when the needle is placed intrafascicularly. Furthermore, the resolution of the ultrasound machine used for peripheral nerve block is not high enough to recognize intrafascicular injection. Therefore, nerve injury with peripheral nerve blocks despite the use of ultrasound continues to be reported.12-14 The rate of residual paresthesia and numbness after ultrasound-guided peripheral nerve block is estimated to be 0.18% up to 16%.15-18 D is incorrect. See explanation for answer A. 5. A is correct. The Defense and Veterans Pain Rating Scale (DVPRS) feature notes on how the pain affects everyday living that can be used to more precisely define the severity of the pain. It also features facial cartoons that can be used to obtain feedback on pain severity from individuals with limited communication ability.7 B is incorrect. The face pain scale is commonly used in pediatric populations. There has been modification of the scale for use in older adults to assess pain intensity.19 However, it does not assess the effect of pain on daily living. C is incorrect. The numeric rating scale (NRS) and the visual analog scale (VAS)20 are the most popular pain scales 0–10 where 0 indicates “no pain at all,” and 10 indicates “worst pain ever.” However, these scales do not take into account how pain affects daily living. D is incorrect. The Pain Assessment Checklist for Seniors with Limited Ability to Communicate (PACSLAC) can be used to assess pain in individuals with dementia or cognitive impairment and who have trouble communicating.8 6. E is correct. Opening injection pressure is not dependent on needle size, needle type, injection speed, or syringe size. Therefore, if the opening injection pressure is detected high, it is advisable to withdraw the needle slightly and change the approach before injecting. One

22/04/19 1:35 PM

46

PART 2

Foundations of Local and Regional Anesthesia

should note that the opening injection pressure will be high if the needle tip is against a fascia.27 7. D is correct. The Tsui test is the use of nerve stimulation to confirm and guide in placing epidural catheters using electrical epidural stimulation. A is incorrect. This is not the best approach because a nonfunctioning epidural requires intraoperative opioid. This may cloud the clinical scenario once the patient is in recovery. If it is noted in recovery that the epidural is ineffective, the patient may have extensive pain making it more difficult to perform the epidural. Once the epidural is resited, the pain response is abolished and the patient may become more sedated from the opioid. B is incorrect. The Miller Fisher test is used in diagnosing normal pressure hydrocephalus by withdrawing CSF during lumbar puncture that results in improvement in cognitive function. C is incorrect. Eliciting a pain response with a sharp pin may not cause grimace in a patient who may not understand what is happening. This approach may leave multiple pin marks on the patient. 8. B is correct. An in-line pressure manometer continuously displays the pressure during injection, allowing clinicians to quantify the injection pressure. The manometer is color coded such that when the pressure is 20 psi or greater, the indicator will be red to warn the operator. The compressed air injection technique (CAIT) is the use of air above the injectate. Study has shown that at 50% air compression, the injection pressure was 760 mm Hg or less, which was well below the threshold of less than 25 psi (1293 mm Hg).21 A, C, and D are incorrect. Syringe feel is traditionally in clinical practice. However, it is a subjective feel to assess resistance to injection of local anesthetic. This approach is operator-dependent22,23 and it can be affected by needle lengths, diameter, and syringe types. 9. A is correct. A nerve stimulation threshold of less than 0.2 mA may suggest intraneural needle-tip location or needle-nerve contact.24,25 Observation of a motor response signifies needle-nerve proximity with a low threshold current; however, a lack of motor response at a 0.2 mA or less threshold with a pulse width of 0.1 ms does not always rule out intraneural needle placement. B, C, and D are incorrect. See explanation for answer A. 10. D is correct. Monitoring injection pressure can help distinguish needle-tip location in the perineural tissue versus needle-nerve contact or intrafascicular needle placement.26,27 Using canine models, it was shown that high injection pressure (> 20 psi) can result in persistent neurological damage indicative of intrafascicular injection.28 High injection pressure can cause undesired neuraxial spread during regional blocks such as lumbar plexus or brachial plexus blocks.29,30 11. A is correct. The compressed air injection technique (CAIT) is the clinical application of Boyle’s law. A volume of air is aspirated above the volume of injectate within a

Hadzic_Ch11_p041-048.indd 46

syringe. During injection, the volume of air is compressed and maintained at a certain percentage. At 50% air compression, the injection pressure is 760 mm Hg or less, well below the threshold of less than 25 psi (1293 mm Hg).21 B, C, and D are incorrect. See explanation for answer A. 12. B is correct. When the BSmartTM (B-Braun Medical, Melsungen, Germany) device is showing the critical injection pressure in the yellow zone of 15–20 psi, it suggests that the needle is too close to the nerve causing forceful needle-nerve contact. Injection at this point could result in intraneural inflammatory changes.31 A is incorrect. The white indicator on the BSmartTM device suggests perineural placement of the needle and injection at this point is associated with low opening injection pressure.27 C is incorrect. See explanation for answer B. D is incorrect. First of all, the pressure >20 psi is displayed as the red zone on the BSmartTM device. It is in the unsafe zone and injection should be halted with slight withdrawal of the needle. High injection pressures at the onset of injection may indicate an intraneural needle placement and lead to severe fascicular injury and persistent neurologic deficit.28 13. B is correct. The catheter-over-needle design has the needle within the catheter for initial needle insertion. Therefore, upon removing the needle, the catheter remains snug in relation to the skin. This can potentially reduce leakage32 and result in less disruption of the surrounding dressing, which in turn, reduces catheter migration. There are also fewer steps and less risk of catheter dislodgement.33 For the E-cath (Pajunk MEDIZINTECHNOLOGIE GmbH, Geisingen, Germany) kit with a “catheter-within-catheter” design, the inner catheter literally replaces the needle, and the inner catheter tip is essentially at the exact location where the needle tip was before needle withdrawal. A is incorrect. One of the advantages of the catheterover-needle design includes less leakage from the catheter insertion site. C is incorrect. In the traditional catheter-through-needle assembly the needle is surrounding the catheter. D is incorrect. The catheter-over-needle design literally replaces the needle, and the inner catheter tip is essentially at the exact location where the needle tip was before needle withdrawal, making the catheter tip much more predictable than the traditional perineural catheters. 14. A is correct. If the target is deeper than the needle, a longer needle should be used. Ideally, a preliminary scan is performed to ascertain the length of the needle/catheter that is required to prevent any wastage. If the needle is too short for the target, especially for catheter-over-needle assemblies, the catheter tip will not reach the target. With pressure on the skin, the needle may reach the target, but the catheter will invariably bounce away from the target resulting in a failed block or maintenance of the nerve block. However, too long a needle/catheter is difficult to handle and manipulate. Longer needles can also bend more easily

22/04/19 1:35 PM

CHAPTER 11

and be difficult to steer. Therefore, choose an appropriate size needle/catheter that ideally is the shortest possible. B is incorrect. If the catheter length does not reach the target, unless a catheter of longer length is not available, it is not a reason to change to a single-shot block. The patient will not benefit from a longer duration of analgesia from a continuous nerve block. C is incorrect. As mentioned above, the needle is too short to reach the target. Even with some pressure on the skin to enable the needle to reach the target, the catheter will not stay in place because the subcutaneous tissue will bounce back once the pressure is released. D is incorrect. See explanation for answer A. 15. D is correct. The lower current range is used primarily to alert for potential intraneural needle placement, whereas the higher range is mainly used for the epidural stimulation test (1 to 15 mA).34 A, B, and C are incorrect. These are low current ranges, used primarily to alert for potential intraneural needle placement. 16. B is correct. The polarity of the needle will affect its ability to stimulate the nerve at a given current and should be clearly marked. The cathode (black) is selected as the stimulation electrode because it is three to four times more effective than the anode (red) at depolarizing the nerve membrane. Nowadays, most nerve stimulators have different connections for anode and cathode, and the cathode has a special cylindrical connection for the needle. A is incorrect. The cathode (black) is more effective than the anode. C is incorrect. Cathode and anode are not equally effective at depolarizing the nerve membrane.

References 1. Castrillo A, Tabernero C, Garcia-Olmos LM, et al. Postdural puncture headache: impact of needle type, a randomized trial. Spine J. 2015;15:1571-1576. 2. Hammond ER, Wang Z, Bhulani N, et al. Needle type and the risk of post-lumbar puncture headache in the outpatient neurology clinic. J Neurol Sci. 2011;306(1-2):24-28. 3. Labat G. Local, regional and spinal anesthesia. Ann Surg. 1921;74:673-683. 4. Asghar S, Bjerregaard LS, Lundstrom LH, et al. Distal infrared thermography and skin temperature after ultrasound-guided interscalene brachial plexus block: a prospective observational study. Eur J Anaesthesiol. 2014;31:626-634. 5. Asghar S, Lundstrom LH, Bjerregaard LS, Lange KH. Ultrasoundguided lateral infraclavicular block evaluated by infrared thermography and skin temperature. Acta Anaesthesiol Scand. 2014;58:867-874. 6. Tsui BC, Shakespeare TJ, Leung DH, et al. Reproducibility of current perception threshold with the Neurometer® vs the Stimpod NMS450 peripheral nerve stimulator in healthy volunteers: an observational study. Can J Anaesht. 2013;60:753-760. 7. Buckenmaier CC III, Galloway KT, Polomano RC, et al. Preliminary validation of the Defense and Veterans Pain Rating Scale (DVPRS) in a military population. Pain Med. 2013;14:110-123.

Hadzic_Ch11_p041-048.indd 47

47

Equipment for Regional Anesthesia

8. Fuchs-Lacelle S, Hadjistavropoulos T. Development and preliminary validation of the pain assessment checklist for seniors with limited ability to communicate (PACSLAC). Pain Manag Nurs. 2004;5:37-49. 9. Tsui B. Electrical nerve stimulation. In: Atlas of Ultrasound and Nerve Stimulation-Guided Regional Anesthesia. Germany: Springer; 2007:13-14. 10. Brull R, Hadzic A, Teina MA, Barrington MJ. Pathophysiology and etiology of nerve injury following peripheral nerve blockade. Reg Anesth Pain Med. 2015;40(5):479-490. 11. Neal JM, Barrington MJ, Brull R, et al. The second ASRA practice advisory on neurologic complications associated with regional anesthesia and pain medicine. Executive Summary 2015. Reg Anesth Pain Med. 2015;40(5):401-430. 12. Cohen JM, Gray AT. Functional deficits after intraneural injection during interscalene block. Reg Anesth Pain Med. 2010;35: 397-399. 13. Reiss W, Kurapati S, Shariat A, Hadzic A. Nerve injury complicating ultrasound electrostimulation-guided supraclavicular brachial plexus block. Reg Anesth Pain Med. 2010;35:400-401. 14. Hara K, Sakura S, Yokokawa N, Tadenuma S. Incidence and effects of unintentional intraneural injection during ultrasound guided subgluteal sciatic nerve block. Reg Anesth Pain Med. 2012;37: 289-293. 15. Sites BD, Taenzer AH, Herrick MD, et al. Incidence of local aesthetic systemic toxicity and postoperative neurologic symptoms associated with 12,668 ultrasound-guided nerve blocks: an analysis from a prospective clinical registry. Reg Anesth Pain Med. 2012;37:478-482. 16. Liu SS, YaDeau JT, Shaw PM, et al. Incidence of unintentional intraneural injection and postoperative neurological complications with ultrasound-guided interscalene and supraclavicular nerve blocks. Anaesthesia. 2011;66:168-174. 17. Widmer B, Lustig S, Scholes CJ, et al. Incidence and severity of complications due to femoral nerve blocks performed for knee surgery. Knee. 2013;20:181-185. 18. Bilbao Ares A, Sabate A, Porteiro L, et al. Neurological complications associated with ultrasound-guided interscalene and supraclavicular block in elective surgery of the shoulder and arm. Prospective observational study in a university hospital. Rev Esp Anestesiol Reanim. 2013;60:384-391. 19. Kim EJ, Buschmann MT. Reliability and validity of the Faces Pain Scale with older adults. Int J Nurs Stud. 2006;43(4):447-456. 20. Huskisson EC. Visual analogue scales. In: Melczak R (ed): Pain Measurement and Assessment. Raven Press; 1983:33-37. 21. Tsui BC, Li LX, Pillay JJ. Compressed air injection technique to standardize block injection pressures. Can J Anaesth. 2006;53:1098-1102. 22. Claudio R, Hadzic A, Shih H, et al. Injection pressures by anesthesiologists during simulated peripheral nerve block. Reg Anesth Pain Med. 2004;29(3):201-205. 23. Theron PSI, Mackay Z, Gonzalez JG, Donaldson N, Blanco R. An animal model of “syringe feel” during peripheral nerve block. Reg Anesth Pain Med. 2009;34(4):330-332. 24. Bigeleisen PE, Moayeri N, Groen GJ. Extraneural versus intraneural stimulation threshold during ultrasound-guided supraclavicular block. Anesthesiology. 2009;110:1235-1243. 25. Robards C, Hadzic A, Somasundaram L, et al. Intraneural injection with low-current stimulation during popliteal sciatic nerve block. Anesth Analg. 2009;109:673-677.

22/04/19 1:35 PM

48

PART 2

Foundations of Local and Regional Anesthesia

26. Gadsden J, Latmore M, Levine DM, Robinson A. High opening injection pressure is associated with needle-nerve and needlefascia contact during femoral nerve block. Reg Anesth Pain Med. 2016;41(1):50-55. 27. Gadsden JC, Choi JJ, Lin E, Robinson A. Opening injection pressure consistently detects needle-nerve contact during ultrasound-guided interscalene brachial plexus block. Anesthesiology. 2014;120(5):1246-1253. 28. Hadzic A, Dilberovic F, Shah S, et al. Combination of intraneural injection and high injection pressure leads to fascicular injury and neurologic deficits in dogs. Reg Anesth Pain Med. 2004;29:417-423. 29. Orebaugh SL, Mukalel JJ, Krediet AC, et al. Brachial plexus root injection in a human cadaver model: injectate distribution and effects on the neuraxis. Reg Anesth Pain Med. 2012;37:525-529. 30. Gadsden JC, Lindenmuth DM, Hadzic A, et al. Lumbar plexus block using high-pressure injection leads to contralateral and epidural spread. Anesthesiology. 2008;109:683-688.

Hadzic_Ch11_p041-048.indd 48

31. Steinfeldt T, Poerschi S, Nimphius W, et al. Forced needle advancement during needle-nerve contact in a porcine model: histological outcome. Anesth Analg. 2011;113:417-420. 32. Ip V, Bouliane M, Tsui B. Potential contamination of the surgical site caused by leakage from an interscalene catheter with the patient in a seated position: a case report. Can J Anaesth. 2012;59:1125-1129. 33. Tsui BC, Ip VH. Catheter-over-needle method reduces risk of perineural catheter dislocation. Br J Anaesth. 2014;112:759-760. 34. Tsui B. Epidural stimulation test. Paediatr Anaesth. 2004;14(12): 1031-1032.

Suggested Reading Hadzic A. Equipment for regional anesthesia. In: Ip VHY, Tsui BCH, eds. Textbook of Regional Anesthesia and Acute Pain Management. 2nd ed. New York, NY: McGraw-Hill Publishing; 2017:chap 12.

22/04/19 1:35 PM

12 Equipment for Continuous Peripheral Nerve Blocks Alexandru Visan

QUESTIONS DIRECTIONS: Choose the one best response to each question. 1. The correct dosage for Intralipid 20% is: A. 1.0 mL/kg (lean body mass) bolus over 1 minute and every 3–5 minutes up to 2 mL/kg, 0.5 mL/kg/min infusion, maximum total 8–10 mL/kg for the first 30 minutes B. 1.5 mL/kg (actual body weight) bolus over 1 minute and every 3–5 minutes up to 3 mL/kg, 0.25 mL/kg/min infusion, maximum total 8–10 mL/kg for the first 30 minutes C. 1.5 mL/kg (lean body mass) bolus over 1 minute and every 3–5 minutes up to 3 mL/kg, 0.25 mL/kg/min infusion, maximum total 8–10 mL/kg for the first 30 minutes D. 0.25 mL/kg (actual body weight) bolus over 1 minute and every 3–5 minutes up to 3 mL/kg, 1.5 mL/kg/min infusion, maximum total 8–10 mL/kg for the first 30 minutes 2. The benefits of continuous peripheral nerve blockade (CPNB) include: a) CPNB provides extended analgesia for acute pain, chronic pain, and palliative care. b) Sympathectomy associated with CPNB provides benefits in addition to superior analgesia in microvascular, reimplantation, and free-flap surgery. c) CPNB is associated with less respiratory depression, lower opioid requirements, and early rehabilitation. d) CPNB is associated with better sleep pattern and less cognitive dysfunction postoperatively. A. c B. c, d C. a, c D. a, b, c, d

3. Necessary equipment in a typical block cart for safe and efficient continuous peripheral nerve blockade (CPNB) placement includes: a) Nerve stimulator and electrodes b) Ultrasound machine with probes of appropriate frequency, size, and shape c)  Selection of block needles and catheter sets of appropriate diameter and length d)  Oxygen source, oxygen masks, suction, and cardioverter/defibrillator A. c B. b, c C. a, b, c D. a, b, c, d 4. Necessary medications in a typical block cart for safe practice of regional anesthesia include: a)  Short-acting local anesthetics (LAs) of various concentrations, such as lidocaine and/or mepivacaine b)  L ong-acting LAs of various concentrations, such as ropivacaine and/or bupivacaine c)  Advanced cardiac life support drugs such as epinephrine, atropine, and vasopressin d) Intralipid 20% 1 liter A. c B. b, c C. a, b, c D. a, b, c, d 5. It is advantageous to add epinephrine to the local anesthetic solution because epinephrine: A. Enhances systemic absorption of perineurally placed local anesthetic B. Causes local vasodilation C. Decreases the length of block D. Acts as a marker of intravascular injection

49

Hadzic_Ch12_p049-052.indd 49

4/18/19 2:27 PM

50

PART 2

Foundations of Local and Regional Anesthesia

6. Which statement is correct? A. A stimulating catheter is associated with lower local anesthetic consumption and reduced requirement for additional opioid analgesia. B. When compared to insulated short-bevel needles, uninsulated needles provide more focused current output and therefore facilitate more accurate nerve localization. C. The closer the stimulating block needle or catheter to the nerve, the stronger the motor response. D. Continuous peripheral nerve blockade was originally developed around year 1946, therefore has a longer history and wider clinical application than that of the single-injection nerve block technique. 7. Which statement is true? A. Peripheral nerve catheters are radiolucent. B. Single-orifice catheters provide better local anesthetic spread than multiorifice catheters. C. Peripheral nerve catheters are flexible and frictionless. D. Multiorifice catheters have an open distal tip at the end of the catheter. 8. Which needle localization technique requires direct contact into the plexus? A. Fascial click B. Paresthesia technique C. Fluoroscopy D. Ultrasound 9. Choose from the following options to correctly identify the two device sets below.

FIGURE A  Used with permission from Pajunk (Geisengen, Germany).

FIGURE B  Used with permission from Holly Evans, MD.

Hadzic_Ch12_p049-052.indd 50

A. Figure A: catheter-through-needle device; Figure B: cannula-over-needle device B. Figure A: cannula-over-needle device; Figure B: catheter-through-needle device C. Figure A: thread-assist catheter device; Figure B: cannula-over-needle device D. Figure A: catheter-through-needle device; Figure B: thread-assist catheter device 10. Infection risks during placement and maintenance of continuous peripheral nerve blocks (PNBs) can be reduced by which practice? A. Prepare the local anesthetic solution for continuous infusion in a clean environment, such as in a pharmacy using a laminar flow workbench. B. Place an opaque, occlusive dressing over the catheter site. C. Cover the ultrasound probe for PNB placement with Tegaderm. D. Disinfect the potential catheter site with alcohol swabs.

ANSWERS AND EXPLANATIONS 1. C is correct. The correct dosage for Intralipid 20% is 1.5 mL/kg bolus (lean body mass) over 1 minute and every 3–5 minutes up to 3 mL/kg, 0.25 mL/kg/min infusion. Total maximum 8 mL/kg for the first 30 minutes based on this chapter and 10 mL/kg based on ASRA recommendations. A is incorrect. It is correct that Intralipid 20% dosing is calculated based on lean body mass, but the infusion rate is wrong. B is incorrect. The dosage for Intralipid 20% looks right, but it is calculated based on the actual body weight but not the lean body mass recommended by ASRA; therefore it is still incorrect. D is incorrect. This answer confused the bolus dosage and continuous infusion rate. The American Society of Regional Anesthesia and Pain Medicine recommends an upper limit or maximum dose of 10 mL/kg of the lipid emulsion over the first 30 minutes. 2. D is correct. Answer D includes benefits a, b, c, and d. CPNB has multiple advantages as compared to singleinjection blockade as well as opioids. All four options below are among the many benefits of CPNB: a) When compared with single-injection technique, CPNB has additional benefits such as extended analgesia and can be utilized in acute pain, chronic pain management, as well as palliative care. b) Prolonged sympathectomy associated with CPNB is routinely utilized in microvascular, reimplantation, and free-flap surgery to promote local circulation, wound healing, and improve surgical outcome, in addition to providing superior and longer analgesia.

4/18/19 2:27 PM

CHAPTER 12

c) The prolonged opioid-sparing effects associated with CPNB not only decrease opioid consumption and opioid-related side effects, but also facilitate early postoperative ambulation. d) CPNB has been shown to be associated with better sleep pattern and less cognitive dysfunction postoperatively, partially due to opioid-sparing effects. A, B, and C are incorrect. See explanation for answer D. 3. D is correct. Answer D includes equipment options a, b, c, and d. A block cart containing basic supplies for nerve block placement and emergency equipment is typical and essential for a safe and efficient nerve block service. All the following should be part of the block cart: a) Nerve stimulator and electrodes remain to be essential and important for nerve location. b) The ultrasound machine is increasingly important and popular for nerve blockades. Depending on the target nerve/plexus, different ultrasound probes of various frequency, size, and shape can provide adequate resolution, penetration, and therefore better guidance on nerve block placement. c) Block needles and catheter sets of various diameter and length (2 inch, 4 inch, 6 inch, etc.) should be stocked for CPNB at different nerves/plexuses. d) Complications are rare but may still occur so necessary equipment such as an oxygen source, oxygen masks, suction, cardioverter/defibrillator need to be readily available. A, B, and C are incorrect. See explanation for answer D. 4. D is correct. Answer D includes medications a, b, c, and d. All of the following listed are necessary medications in a typical block cart for safe and efficient practice of regional anesthesia: a) Short-acting agents such as lidocaine or mepivacaine allow for rapid onset yet early recovery of the sensorimotor block. This facilitates prompt assessment of neurological function following surgery and prior to starting a continuous perineural infusion. b) Block initiation with long-acting agents such as bupivacaine or ropivacaine extends the duration of dense anesthesia and analgesia. c) Advanced cardiac life support drugs such as epinephrine, atropine, and vasopressin need to be readily available for the rare but very possible adverse events associated with nerve block placement. d) Intralipid 20% is the main treatment for local anesthetic systemic toxicity and usually at least 1 liter is needed for a typical adult patient. A, B, and C are incorrect. See explanation for answer D. 5. D is correct. Epinephrine at concentrations of 1:200,000 or 1:400,000 can be added to the local anesthetic solution to limit the systemic absorption of perineurally placed local anesthetics, as well as to serve as a marker of intravascular injection. A is incorrect. Epinephrine decreases systemic absorption of perineurally placed local anesthetic through local vasoconstriction.

Hadzic_Ch12_p049-052.indd 51

Equipment for Continuous Peripheral Nerve Blocks

51

B is incorrect. Epinephrine induces local vasoconstriction through actions on adrenoceptors, mostly alpha1 receptors on blood vessels, therefore decreasing systemic absorption. C is incorrect. Epinephrine increases rather than decreases the length of block through vasoconstriction and slower systemic absorption. D is incorrect. Epinephrine can be utilized as a marker of intravascular injection during nerve block placement as its intravascular injection induces transit tachycardia and/or hypertension through its action on adrenoceptors, mostly beta receptors. 6. A is correct. Stimulating catheters for CPNB, when compared to nonstimulating catheters, have been shown to be associated with lower local anesthetic consumption and reduced requirement for additional opioid analgesia; however, their placement may be more technically challenging under ultrasound guidance. B is incorrect. The relatively large uninsulated segment at the distal tip of the uninsulated needle produces less focused current output, therefore could decrease the accuracy of nerve identification. C is incorrect. The distance between the stimulating block needle or catheter and the nerve/plexus is not always associated with the strength of motor response. For example, studies have shown that the stimulating needle can be in close contact with or even be within the nerve yet no motor twitch was elicited at relatively low current. D is incorrect. The design of CPNB needles and catheters initially lagged behind the development of single-injection PNB equipment, thus limiting their clinical application. 7. C is correct. Originally epidural catheters were adapted for CPNB, and they are well suited for this application by being nonirritating, flexible, and generating minimal friction on passage through needles. In addition, catheters with graduated markings provide an indicator of insertion depth. A is incorrect. CPNB catheters are radiopaque, which provides an additional method to confirm catheter placement localization. B is incorrect. Multiorifice catheters provide better local anesthetic spread than single-orifice catheters with one opening at the tip. D is incorrect. A multiorifice catheter has a closed distal tip and three distal openings (0.5, 1.0, and 1.5 cm from the tip) close to the end of the catheter, whereas a single-orifice catheter has a single opening at the end of the catheter. 8. B is correct. There are several nerve localization techniques. The paresthesia technique involves advancing the nerve block needle toward a nerve/plexus with needleinduced sensory paresthesia as the end point. It therefore frequently involves direct needle and nerve/plexus contact. A is incorrect. Traditionally the fascial click technique is mostly utilized in field blocks and involves a tactile “pop” sensation as a nerve block needle penetrates fascial layers. It is associated with limited needle/nerve direct contact and low accuracy for nerve localization.

4/18/19 2:27 PM

52

PART 2

Foundations of Local and Regional Anesthesia

C is incorrect. Fluoroscopy visualizes nearby radiopaque structures such as bones, with or without contrast dye, to indirectly localize the nerve/plexus and therefore does not involve direct neural structures and needle contact. D is incorrect. With ultrasound, neural and perineural anatomy and anatomical variants can be directly visualized, precise location and depth to neural structures can be estimated/measured, and surrounding structures and vasculature can be identified and avoided. In addition, the nerve block needle, catheter, and local anesthetic spread can be seen in relation to the nerve/plexus in real time. Direct contact within the plexus is mostly avoided. 9. B is correct. With the cannula-over-needle system depicted in Figure A, once the needle is in the appropriate location close to the nerve the cannula is advanced in Seldinger fashion, similar to peripheral intravenous line placement. With the catheter-through-needle technique depicted in Figure B, the needle is advanced until the tip is in a position close to the nerve, then the catheter is advanced through the hemostatic valve. The needle is then removed while advancing the catheter to prevent dislodgement during this time. The catheter is then withdrawn to the desired depth according to the markings on the catheter, similar to epidural catheter placement. Figure C below depicts a thread-assist device, in which the catheter resides in a coiled housing and the blue portion at the tip allows for easy introduction/threading of the catheter, reminiscent of a central line guidewire.

A is incorrect. This statement confused the cannulaover-needle system and catheter-through-needle system. C is incorrect. Figure A is a cannula-over-needle system not a thread-assist device; Figure B is a catheter-through-needle system not a cannula-over-needle system. D is incorrect. This option correctly identified Figure A, but Figure B is not a thread-assist device. 10. A is correct. The local anesthetic solution should be prepared in a clean environment in a sterile fashion, ideally in a pharmacy with the capacity to prepare compounded medications and using a laminar flow workbench. B is incorrect. A transparent or clear occlusive dressing should be placed over the catheter site for easier visualization during follow-up for potential catheter dislodgement, signs of infection (eg, redness, induration, or discharge), and bleeding. C is incorrect. In contrast to single-injection nerve blockade, sterile ultrasound probe covers that shield the entire probe and most of the cable should be used in CPNB placement. D is incorrect. More effective than alcohol alone is 2% chlorhexidine gluconate with 70% isopropyl alcohol; so it is recommended to disinfect the potential catheter site.

Suggested Readings Hadzic A. Equipment for continuous peripheral nerve blocks. In: Evans H, Nielsen KC, Melton MS, Greengrass RA, Steele SM, eds. Textbook of Regional Anesthesia and Acute Pain Management. 2nd ed. New York, NY: McGraw-Hill Publishing; 2017:chap 13. Neal JM, Bernards CM, Butterworth JF, et al. American Society of Regional Anesthesia and Pain Medicine Practice Advisory on Local Anesthetic Systemic Toxicity. Reg Anesth Pain Med. 2010;35:152-161.

FIGURE C  Used with permission from Pajunk (Geisengen, Germany).

Hadzic_Ch12_p049-052.indd 52

4/18/19 2:27 PM

13 Electrical Nerve Stimulators and Localization of Peripheral Nerves Tom C. Van Zundert

QUESTIONS DIRECTIONS: Choose the one best response to each question. 1. All of the following answers are correct except one. Choose the answer that is not correct. Motor response to nerve stimulation of the femoral nerve is absent: A. Under general anesthesia with sevoflurane, rocuronium, and a continuous infusion of remifentanil B. Under spinal anesthesia C. In a patient who already received two injections of local anesthetics D. When the saphenous nerve is the target 2. Which setting (electrical repetition rate) of the peripheral nerve stimulator should be used to detect muscular twitches, when the current is set at 0.5- to 1-mA intensity and 0.1 ms duration? A. 2 Hz B. 50 Hz C. 100 Hz D. 200 Hz 3. Regarding action potential, which of the following statements is correct? A. Pain fibers have a short chronaxy. B. It takes a shorter time to depolarize the membrane of pain fibers compared to motor fibers. C. Motor fibers have a long chronaxy. D. Motor fibers have a short chronaxy (only the area of the nodes of Ranvier count). 4. What is the correct definition of rheobase? A. The pulse duration at double the threshold B. The lowest threshold current at infinitively long pulse duration C. The pulse width at half the threshold D. The electrical pulse that is most effective

5. The capacitance in a circuit varies with the frequency context of the stimulation current, ie, the impedance. Which statement is correct? A. The longer the impulse, the higher its frequency content. B. The shorter the impulse, the lower its impedance. C. The electrode-to-skin impedance is constant between individuals. D. The electrode-to-skin impedance can be influenced by the quality of the ECG electrode. 6. Once the nerve stimulation needle is connected to the nerve stimulator, which of the following settings is correct? A. For superficial blocks, select 0.1 mA as the starting current intensity. B. For superficial blocks, select 1 ms as the current duration. C. For deep blocks, select 1.5 mA as the starting current intensity. D. For deep blocks, select 1 ms of current duration. 7. Regarding nerve-mapping pens, which of the following statements is not true? A. Nerve-mapping pens can locate superficial nerves, up to a depth of 3 cm. B. Nerve-mapping pens can identify the best site for needle detection in patients with difficult anatomy. C. The electrode tip of the stimulator should have an atraumatic tip not larger than 3 mm in diameter. D. A shorter stimulus duration (eg, 0.1 ms) is needed to accomplish transcutaneous nerve stimulation. 8. An insulated needle is properly positioned, close to the nerve at around 0.3 mA and 0.1 ms pulse duration during peripheral nerve stimulation. Which of the following procedures is incorrect in regards to abolishing the motor response? A. Inject local anesthetic B. Inject dextrose 5%

53

Hadzic_Ch13_p053-056.indd 53

22/04/19 1:38 PM

54

PART 2

Foundations of Local and Regional Anesthesia

C. Inject normal saline D. Inject a large volume (> 10 mL) of local anesthetic 9. Which of the following statements is not true? A. The presence of a motor response with a stimulating current (< 0.2 mA, 0.1 ms) indicates intraneural needle placement. B. The presence of a motor response with a stimulating current (< 0.2 mA, 0.1 ms) indicates possible intrafascicular needle placement. C. The absence of a motor response does not rule out intraneural needle placement. D. The electrical impedance halves when the needle is advanced from an extraneural to an intraneural position. 10. What should be the current range of nerve stimulators? A. 0–5 mA B. 5–10 mA C. 10–20 mA D. 20–50 mA

11. A stimulus frequency of 1–3 Hz means: A. 1–3 ms per pulse B. 1–3 mA per pulse C. 1–3 pulses per second D. 1–3 J per second

ANSWERS AND EXPLANATIONS 1. B is correct. Reliability of peripheral nerve stimulation is not affected by the presence of spinal or epidural anesthesia; therefore, motor response to stimulation of the femoral nerve will be present in this case. 2. A is correct. 2 Hz is ideal to detect motor responses. See Figure 13–1. 3. D is correct. Pain fibers have a long chronaxy; It takes only a short time to depolarize the membrane of the motor

Hadz Hadz Ha dzicc - Laan ncea// NYS YSO OR RA FIGURE 13–1  An algorithm for use of nerve stimulation with ultrasound-guided nerve blocks. Please note that the nerve stimulator here is used primarily as a safety-monitoring tool, rather than a nerve localization tool. The stimulator is set at 0.5 mA (0.1 ms), and the current is rarely manipulated. Instead, a motor response is obtained; extra caution is exercised as this indicates an intimate needle-nerve relationship. Instead of adjusting the current intensity to determine at which current the motor response extinguishes, the needle is slightly withdrawn to abolish the response and distance the needle tip from the nerve. A small amount of local anesthetic is then injected to determine the needle tip location while avoiding an opening pressure greater than 15 psi. LA, local anesthetic; OIP, opening injection pressure.

Hadzic_Ch13_p053-056.indd 54

22/04/19 1:38 PM

CHAPTER 13

fiber up to the threshold level. Motor fibers have a short chronaxy because of the relatively low capacitance of their myelinated membrane (only the area of the nodes of Ranvier count); therefore, it takes only a short time to depolarize the membrane up to the threshold level. 4. B is correct. The lowest threshold current at infinitively long pulse duration is called rheobase. 5. D is correct. Some lower-priced lower-quality ECG electrodes can have too high an impedance/resistance. 6. C is correct. For deep blocks, select 1.5 mA as the starting current intensity. 7. D is not true, so option D is correct. A longer stimulus duration (1 ms) is needed to accomplish transcutaneous nerve stimulation. 8. B is correct. This procedures is incorrect regarding abolishing the motor response. An injection of 5% dextrose in water (low conductivity) does not lead to loss of the muscle twitch if the needle position is not changed.

Hadzic_Ch13_p053-056.indd 55

Electrical Nerve Stimulators and Localization of Peripheral Nerves

55

9. D is not true, so option D is correct. The electrical impedance doesn’t halves, it doubles (12.1 to 23.2 kΩ) when the needle is advanced from an extraneural to an intraneural position. 10. A is correct. The amplitude range should be 0–5 mA or even 0–1 mA in order to check the nerve endpoint. High threshold currents mean that the needle tip is too far from the nerve endpoint. Lower threshold currents (< 0.1 mA) can mean a risk of neural damage (intraneural/intrafascicular injection). 11. C is correct. 1–3 pulses per second

Suggested Reading Hadzic A. Electrical nerve stimulators and localization of peripheral nerves. In: van Zundert A, Hadzic A, eds. Textbook of Regional Anesthesia and Acute Pain Management. 2nd ed. New York, NY: McGraw-Hill Publishing; 2017:chap 14.

22/04/19 1:38 PM

This page intentionally left blank

Hadzic_FM_p00i-pxx.indd 6

22/04/19 6:40 PM

Section 4

Patient Management Considerations Chapter 14

Developing Regional Anesthesia Pathways  59

Chapter 15

Infection Control in Regional Anesthesia  63

Chapter 16

Local Anesthetics, Regional Anesthesia, and Cancer Recurrence  65

Chapter 17

 erioperative Regional Anesthesia and Analgesia: Effects on Cancer Recurrence and P Survival After Oncological Surgery  71

Hadzic_Ch14_p057-062.indd 57

4/18/19 2:37 PM

This page intentionally left blank

Hadzic_FM_p00i-pxx.indd 6

22/04/19 6:40 PM

14 Developing Regional Anesthesia Pathways Philippe Volders and Pieter Vander Cruyssen

QUESTIONS DIRECTIONS: Choose the one best response to each question. 1.  An anesthetic pathway depends on: A. The institution B. The techniques used by surgeons C. The techniques used by anesthesiologists D. All of the above 2.  Which of the following is not a goal of an anesthetic pathway? A. Brief and clear guidelines for every health-care provider involved in the pathway B. Clinician can focus on characteristics of a patient instead of characteristics of a cohort C. Reduce perioperative complications and risks D. Same consistent treatment for every patient 3.  Which of the following is an advantage for choosing local and regional anesthesia (LRA) over general anesthesia (GA)? A. LRA can give additive pain control in the postoperative period. B. LRA can give additive pain control postoperatively and can affect morbidity and mortality. C. The operation is in a daycare setting. D. LRA reduces costs and complications. 4.  Meta-analysis of clinical pathways shows that there are: A. Lower rates of complications but worse documentation B. Lower rates of complications and improved documentation C. Reduction in length of stay but increased risk of readmission D. Reduction in hospital costs but increased risk of readmission

5.  When we want to add a treatment to reduce a risk of complication of 5% in the initial pathway, it comes with extra costs (c), risks (r), and benefits (b). Which of the following is correct when we give the extra treatment to only the 10% of patients with the highest risk of the complication? A. Costs (c) and risks (r) decrease by a factor of 10, benefits (b) stay about the same. B. Costs (c) decrease by a factor of 10, risks (r) and benefits (b) stay about the same. C. Costs (c), risks (r), and benefits (b) of the extra treatment all decrease by a factor of 10. D. Costs (c), risks (r), and benefits (b) of the extra treatment all stay about the same. 6.  Which of the following cannot be an explanation when a clinically relevant complication occurs while following a pathway, although previous studies did not detect this complication? A. The effect size detected by the studies is not big enough. B. There is a type II error in the studies. C. The studies were underpowered. D. Worst-case scenario should be adjusted. 7.  Which of the following gives the correct sequence for developing a clinical pathway? A. Identify goals and treatment options. Identify and estimate risks, costs, and benefits. Develop worst- and best-case scenarios. Choose a pathway. Continue to gather new data on the pathway. B. Choose a pathway. Identify goals and treatment options. Identify and estimate risks, costs, and benefits. Develop worst- and best-case scenarios. Continue to gather new data on the pathway. C. Continue to gather new data on the pathway. Identify and estimate risks, costs, and benefits.

59

Hadzic_Ch14_p057-062.indd 59

4/18/19 2:37 PM

60

PART 2

Foundations of Local and Regional Anesthesia

Identify goals and treatment options. Develop worst- and best-case scenarios. Choose a pathway. D. Identify goals and treatment options. Choose a pathway. Continue to gather new data on the pathway. Identify and estimate risks, costs, and benefits. Develop worst- and best-case scenarios.

ANSWERS AND EXPLANATIONS 1.  D is correct. Benefits of an anesthetic pathway will of course depend on the peculiarities of the pathway itself, the institution, the surgical and anesthetic techniques, and the other health-care providers using the pathway (such as nursing care and physical therapy). A pathway designed for one institution may not be appropriate for another institution. Furthermore, the goals of an anesthetic pathway may vary; some pathways may be designed to reduce morbidity and mortality, whereas others may be focused on reducing cost while maintaining a high level of patient care. 2.  D is correct. Same consistent treatment for every patient is not a goal of an anesthetic pathway. Pathways should not be indiscriminately adhered to, as some patients will require modification to atone for specific medical conditions or patient preferences. Although developing a clinical pathway often involves lengthy analyses with uncertainties, final pathway elements should be brief and specific. Benefits of an anesthetic pathway will depend on all health-care providers using the pathway. Characteristics of an entire cohort were already examined during pathway development, so clinicians can focus on modifications of the pathway to atone for specific patient characteristics. A recent meta-analysis of clinical pathways found that they were associated with lower rates of patient complications.1 3.  B is correct. Regional anesthesia (either neuraxial or peripheral block) is often a key feature of anesthetic pathway because there is evidence that regional anesthesia may affect the mortality and morbidity of common surgeries and because pain control in the postoperative period is often challenging.2-6 4.  B is correct. A meta-analysis of clinical pathways found that they were associated with lower rates of patient complications (eg, wound infections, bleeding, and pneumonia), as well as improved documentation. Most studies also showed a reduction in length of stay and hospital costs without increased risk of readmission rates and mortality. The considerable heterogeneity of successful clinical pathways has not allowed investigators to identify features common to successful (or unsuccessful) clinical pathways.1

Hadzic_Ch14_p057-062.indd 60

5.  A is correct. When an extra treatment is only given to 10% instead of 100% of patients it speaks for itself that the cost of the extra treatment will decrease by a factor of 10. Likewise, the less we apply the extra treatment, the smaller is the chance the risks attached to the extra treatment will emerge. If the subpopulation with the highest risk of complication is accurately identified, the benefits of the extra treatment may be unaffected or minimally affected because the 5% of patients who would statistically develop the complication are still in the group of 10% who receive the extra treatment. 6.  A is correct. Using power calculators of statistics packages, we can estimate the effect size that would be detected by a study. Effect sizes lower than this will not be reliably detected. It then falls to the clinician to decide whether this effect size is small enough that smaller effect sizes are clinically irrelevant. In this case, a smaller effect size is clinically relevant, so the effect size detected by the studies is not small enough. In statistics, a type II error involves incorrectly concluding that no association exists when in fact one does. The probability of a type II error is signified by the letter β. Power = 1-β. The lower the power, the higher is β and consequently the higher is the probability of a type II error. When a study is underpowered, there is a high probability a type II error occurred. Clinical pathways should be evaluated and refined continually. The clinician should adjust the worst-case scenario taking this complication into account and reevaluate if the chosen pathway is the most likely to benefit the patient. 7.  A is correct. A process for developing clinical pathways consists of the following: 1. Identifying the goals of the pathway and different treatment options to help achieve those goals. This includes all stages of anesthesia and surgery, including postoperative care. 2. Identifying ways in which each treatment interacts with the others (eg, premedications may delay discharge due to sedation). 3. Identifying possible risks, costs, and benefits of each treatment, including mortality, morbidity, patient satisfaction, institutional capabilities, financial cost, and other factors. 4. Making numerical estimates of the risks, costs, and benefits identified in step 3. Some of this information (eg, information on cost) can be gathered and tabulated. However, most of the information cannot be known with any certainty. In many cases, the medical literature can provide estimates of the probabilities involved, as long as the physician understands the limitations of a particular study. 5. Using the estimates developed in step 4 to develop probable outcomes and worst-case and best-case scenarios of each pathway.

4/18/19 2:37 PM

CHAPTER 14

6. Choosing the pathway most likely to benefit the patient based on step 5. 7. Continually refining the estimates of the risks, costs, and benefits as new information becomes available and refining the clinical pathways. 8. Steps 4, 5, and 6 would be trivial if we could quantify the risks and benefits of each intervention as easily as we could quantify the financial costs of procedures and equipment. However, this is rarely the case. In many cases there is not adequate published data to guide the process. Nevertheless, the closest match to one’s institutional conditions can be used to estimate probabilities used in step 5. 9. Recognizing the limitations of the existing evidence for each intervention in the literature is crucial in protocol development. Important is the understanding that recently introduced drugs, procedures, or therapies will have unknown long-term risks.

References 1. Rotter T, Kinsman L, James E, et al. Clinical pathways: effects on professional practice, patient outcomes, length of stay, and hospital costs. Cochrane Database Syst Rev. 2010(3):CD006632.

Developing Regional Anesthesia Pathways

61

3. Memtsoudis SG, Sun X, Chiu YL, et al. Perioperative comparative effectiveness of anesthetic technique in orthopedic patients. Anesthesiology. 2013;118(5):1046-1058. 4. Stundner O, Chiu YL, Sun X, et al. Comparative perioperative outcomes associated with neuraxial versus general anesthesia for simultaneous bilateral total knee arthroplasty. Reg Anesth Pain Med. 2012;37(6):638-644. 5. Memtsoudis SG, Stundner O, Rasul R, et al. Sleep apnea and total joint arthroplasty under various types of anesthesia: a populationbased study of perioperative outcomes. Reg Anesth Pain Med. 2013;38(4):274-281. 6. Liu J, Ma C, Elkassabany N, Fleisher LA, Neuman MD. Neuraxial anesthesia decreases postoperative systemic infection risk compared with general anesthesia in knee arthroplasty. Anesth Analg. 2013;117(4):1010-1016.

Suggested Reading Hadzic A. Developing regional anesthesia pathways. In: Neice A, Barrington MJ, eds. Textbook of Regional Anesthesia and Acute Pain Management. 2nd ed. New York, NY: McGraw-Hill Publishing; 2017:chap 15.

2. Pugely AJ, Martin CT, Gao Y, Mendoza-Lattes S, Callaghan JJ. Differences in short-term complications between spinal and general anesthesia for primary total knee arthroplasty. J Bone Joint Surg Am. 2013;95(3):193-199.

Hadzic_Ch14_p057-062.indd 61

4/18/19 2:37 PM

This page intentionally left blank

Hadzic_FM_p00i-pxx.indd 6

22/04/19 6:40 PM

15 Infection Control in Regional Anesthesia Sebastian Schulz-Stübner

QUESTIONS DIRECTIONS: Choose the one best response to each question. 1.  What is true about the epidemiology of infections related to regional anesthesia? A. E. coli is the predominant pathogen for infections related to regional anesthesia. B. Microorganisms from the oral cavity of the patient are a predominant source of infections associated with epidural catheters. C. Epidural catheters inserted for long-term pain control become infected more frequently than those used for short periods of time. D. Younger males undergoing hip replacement surgery are at an especially high risk for epidural catheter– associated infections. E. Severe sequelae of infected neuraxial catheters are so infrequent, that case reports in the literature are missing. 2.  Which infection control measures are important? A. The patient needs to wear a mask during placement of an epidural catheter for labor and delivery. B. A sterile gown should be worn during insertion of epidural or nerve block catheters. C. A fresh unsterile glove should be used to cover an ultrasound probe used for real-time needle guidance. D. There is no valid surveillance method for regional analgesia catheter-related infections. E. Use of 1% povidone-iodine is recommended for skin disinfection before spinal anesthesia. 3.  What is important for the diagnosis and management of epidural catheter–associated infections? A. The insertion site of the epidural catheter should be inspected every other day for signs of infections. B. If an infection is suspected, leave the catheter in site until positive culture results from the insertion site confirm the diagnosis. C. Local antibiotic treatment at the insertion site is usually sufficient.

D. If an abscess is suspected or neurologic dysfunction is present, imaging studies should be performed. E. A consult with an infectious disease specialist is indicated only for methicillin-resistant S. aureus (MRSA) infection. 4.   The ASA guideline recommendations for the placement of neuraxial blocks include, except: A. Before performing a neuraxial technique, a history, physical examination, and review of relevant laboratory studies should be conducted. B. Consider administering antibiotic therapy before performing a neuraxial technique in a known or suspected bacteremic patient. C. Avoid lumbar punctures in patients with a known epidural abscess. D. Always use aseptic techniques during the preparation of equipment and placement of neuraxial needles and catheters. E. All recommendations are included in the guideline. 5.   All of the following are recommendations during continuous epidural infusions, except: A. Consider removing unwitnessed accidentally disconnected catheters. B. Disconnection and reconnection can be done repeatedly because it doesn’t relate with increased risk of infection. C. Perform a daily evaluation of patients with catheters for early signs and symptoms of infectious complications. D. If an infection is suspected, remove the catheter and consider culturing the catheter tip.

ANSWERS AND EXPLANATIONS 1.  C is correct. In general, epidural catheters inserted for long-term pain control become infected more frequently than those used for short periods of time. Du Pen and associates identified 30 superficial (9.3/10,000 catheterdays), 8 deep catheter track (2.5/10,000 catheter-days), and 15 epidural space (4.6/10,000 catheter-days) infections among 350 patients who had long-term epidural catheters.1 63

Hadzic_Ch15_p063-064.indd 63

4/18/19 2:38 PM

64









PART 2

Foundations of Local and Regional Anesthesia

Similarly, Zenz and colleagues identified two cases of meningitis among 139 patients (1.4%, or 2.1/10,000 catheter-days) treated for pain due to malignancy.2 Coombs reported that 10 of 92 (10.9%) cancer patients acquired local infections, and 2 (2.2%) acquired meningitis.3 A is incorrect. Streptococcal species, S. aureus, and Pseudomonas aeruginosa are the most common causative agents but multiresistant species also emerge as causative pathogens as their endemic impact grows within healthcare systems. B is incorrect. Microorganisms from the patient’s or anesthesia practitioner’s flora can be inoculated directly when a catheter or needle is inserted into the epidural or subarachnoid space. D is incorrect. Malignancy and reduced immunocompetence might be risk factors for catheter infection, but young age is not. E is incorrect. Case reports of infections occurring after epidural neuraxial blockade point out that complications from infection can be severe and often lead to epidural or intraspinal abscesses.

2.  B is correct. A sterile gown should be worn during insertion of epidural or nerve block catheters in order to prevent contact of the catheter with the practitioner’s bare skin during placement. A is incorrect. Wearing a mask during insertion of indwelling neuraxial or peripheral nerve catheters is suggested for the practitioner. C is incorrect. A sterile ultrasound transducer cover should be routinely used with ultrasound-guided procedures. D is incorrect. Surveillance for catheter site infections is one of the most effective methods for reducing the incidence and consequence of indwelling catheter–related infections. E is incorrect. Alcohol plus chlorhexidine has been shown to reduce the risk of catheter-associated bloodstream infections significantly compared with povidone-iodine. This has led to the recommendation to use alcoholic chlorhexidine for skin preparation despite some concerns about potential neurotoxicity. The latter might be the reason American Society of Anesthesiologists (ASA)

Hadzic_Ch15_p063-064.indd 64

members were equivocal on the issue during the consensus process, while external experts were in favor of the recommendation. 3.  D is correct. Imaging studies should be performed immediately and consultation with other appropriate specialties should be promptly obtained. A is incorrect. Daily evaluation of patients with indwelling catheters for early signs and symptoms (eg, fever, backache, headache, erythema, tenderness at the insertion site) of infectious complications should be performed throughout the patient’s stay in the facility. B is incorrect. Remove an in situ catheter and consider culturing the catheter tip if an infection is suspected. C is incorrect. Appropriate systemic antibiotic therapy should always be administered at the earliest sign or symptom of a serious neuraxial infection. E is incorrect. Consultation with a physician with expertise in the diagnosis and treatment of infectious diseases should always be considered. 4.  E is correct. All recommendations are included in the ASA guidelines for the placement of neuraxial blocks. 5.  B is correct. Disconnection and reconnection can’t be done repeatedly. It should be limited to minimize the risk of infectious complications. A, C, and D are incorrect. These statements are correct recommendations during continuous epidural infusions.

References 1. Du Pen SL, Peterson DG, Williams A, et al. Infection during chronic epidural catheterization: diagnosis and treatment. Anesthesiology. 1990;73:905-909. 2. Zenz M, Piepenbrock S, Tryba M. Epidural opiates: long-term experiences in cancer pain. Klin Wochenschr. 1985;63:225-229. 3. Coombs DW. Management of chronic pain by epidural and intrathecal opioids: newer drugs and delivery systems. Int Anesth Clin. 1986;24:59-74.

Suggested Reading Hadzic A. Infection control in regional anesthesia. In: Schulz-Stübner S, Pottinger JM, Coffin SA, Herwaldt LA, eds. Textbook of Regional Anesthesia and Acute Pain Management. 2nd ed. New York, NY: McGraw-Hill Publishing; 2017:chap 16.

4/18/19 2:38 PM

16 Local Anesthetics, Regional Anesthesia, and Cancer Recurrence Cara Connolly and Donal J. Buggy

QUESTIONS DIRECTIONS: Choose the one best response to each question.

E. Proinflammatory cytokines (IL-6, tumor necrosis factor alpha [TNF-α], IL-13) released from infiltrating leukocytes close to the primary tumor site can activate nuclear factor κB (NF-κB) and STAT3 in cancer cells and thus inhibit tumor growth and proliferation.

1. Which of the following statements is true regarding the perioperative period and immune modulation? A. Hypotension and hypovolemia at the time of surgery may influence the immune balance of cancer patients in a deleterious fashion predisposing to cancer cell metastasis. B. Hypotension and hypovolemia may influence immune modulation by increasing the Th1 (T helper 1) response. C. Th2-type immune responses are necessary for antitumor immunity. D. NK cells have a crucial role in the propagation of metastases in the perioperative period.

3. Which of the following statements is true regarding anesthetic factors that may influence cancer metastasis in the perioperative period? A. Hypothermia has been shown to increase NK cell activity in in vivo animal studies. B. Hyperglycemia has been shown in in vitro studies to predispose toward cancer cell metastasis via activation of the surgical stress response. C. Midazolam and ketamine have both been shown to impair antitumor immune responses in vivo and in vitro. D. Blood transfusion has been shown to suppress antitumor elements of the perioperative immune response.

2. Which of the following statements is true regarding the Th1 to Th2 ratio and role in the development of cancer metastasis? A. The natural killer (NK), CD4+ Th1 (T helper 1), and CD8+ CTL (cytotoxic lymphocyte) cells are the most important cells in the promotion of tumor growth and metastasis. B. Th2 (T helper 2), tumor-associated macrophages (TAM), and regulatory T (Treg) cells are among the major antitumor immune effector cells. C. NK cells have a crucial role, particularly in eliminating metastases without prior sensitization and major histocompatibility complex (MHC) restriction. D. NK cell function can be upregulated by prostaglandins, especially prostaglandin E2 (PGE2), shifting the cytokine balance toward Th2 dominance and promoting tumor angiogenesis.

4. Which of the following statements is true regarding anesthetic agents that may influence cancer metastasis in the perioperative period? A. Halothane, isoflurane, and sevoflurane enhance NK cell activity. B. Propofol at clinical concentrations has significant effects on the propagation of NK cells. C. Propofol has inhibiting activity against cyclooxygenase (COX) 2. D. A comparison between propofol and isoflurane showed that propofol, but not isoflurane, increased the Th1/ Th2 ratio, which alters the immune system to favor cancer metastasis.

65

Hadzic_Ch16_p065-070.indd 65

4/18/19 2:39 PM

66

PART 2

Foundations of Local and Regional Anesthesia

5. Which of the following statements is true regarding opioids and cancer metastasis? A. All opioids exert similar, detrimental effects on immune function during the perioperative period. B. Morphine-induced immunosuppression is mediated by its binding to a member of the seven-transmembrane G-protein-coupled μ-opioid receptors (MORs) on immune cells. C. Epidemiologic studies suggested that patients who receive general anesthesia with opioids rather than local anesthetics (LAs) or regional anesthetics may have a lower rate of cancer recurrence. D. Previous studies have shown that the expression of MORs was decreased in lung cancer cells and that the silencing of MOR in those cells increased tumor growth and metastasis in mouse models. E. Previous studies have also shown that MOR1 expression has an effect on cancer progression, with underexpression of MOR1 in human non-small cell lung cancer (NSCLC) cells showing increased proliferation, migration, invasion, and transendothelial migration. 6. Which of the following statements is true regarding regional anesthesia and cancer metastasis? A. In vitro studies have demonstrated antiproliferative and cytotoxic effects of local anesthetics on cancer cells. B. The dose range for amide local anesthetics required to produce antiproliferative effects is outside that used in the clinical setting. C. Even in the absence of surgery, epidural anesthesia has been shown to significantly impact immune function. D. Animal studies have failed to demonstrate antiproliferative effects of local anesthetics on cancer cells. E. Clinical studies have demonstrated the antiproliferative effects of local anesthetics on cancer cells. 7. Which of the following statements is true regarding cancer and inflammation? A. While local anesthetics have anti-inflammatory properties, these are exerted locally—they do not affect the mediators of inflammation at a molecular level. B. Cancer and inflammation both lead to the activation of the nuclear transcription factor NF-κB as well as other transcription factors, but this is achieved by different, separate pathways—the intrinsic and extrinsic pathways. C. The intrinsic pathway is the inflammatory or infectious condition that is responsible for the risk of developing cancer at certain sites, such as colon, prostate, and pancreas. D. The extrinsic pathway is activated by genetic events such as mutations, chromosomal rearrangements, and so on that affect oncogenes. Cells that are transformed by these events produce inflammatory mediators, thereby generating an inflammatory microenviroment in the tumor area. E. The role of cell adhesion molecules (CAMs) such as ICAM-1, VCAM-1, E-selectin, and P-selectin has been studied extensively in the process of inflammation. CAMs have been implicated in tumor progression.

Hadzic_Ch16_p065-070.indd 66

8. Which of the following statements is true regarding the role of ICAM-1 on inflammation and metastasis? A. ICAM-1 expression in serum of patients with malignant melanoma was associated with an increased disease-free interval and survival. B. Significantly lower levels of serum ICAM-1 were detected in patients with liver metastasis, and its levels were decreased in serial blood samples obtained from patients with progressing disease. C. Decreased expression of ICAM-1 influenced tumor progression in colorectal cancer, with low expression of ICAM-1 in tumors an indicator of metastatic potential and poor prognosis. D. ICAM-1 is associated with a variety of cancer types and has a role in cancer metastasis. It could, therefore, be useful as a biomarker for tumor prognosis as well as a target for therapeutic interventions. 9. Which of the following is true regarding the role of Src protein tyrosine kinase (PTK) in cancer metastasis? A. The Src family of nonreceptor PTKs plays a critical role in a variety of cellular signal transduction pathways, including those important for the proliferation of the tumor, disruption of cell/cell contacts, migration, invasiveness, and resistance to apoptosis. B. Activation of Src family kinases is common in a variety of human cancers, occurring via a single, common pathway and is frequently a critical event in tumor progression. C. Once tumor cells leave their primary site, they enter the blood vessels and again they extravasate to form satellite lesions. Previous studies in mice have shown that endothelial barrier protection by VEGF-mediated Src activation protected from tumor extravasation and metastasis. D. Previous studies have shown that mice genetically deficient in the c-Src have an increased risk for tumor cell metastasis. 10. Which of the following is true regarding local anesthetics and cancer metastasis? A. A recent study showed that lidocaine, but not ropivacaine, may inhibit inflammatory cytokine signaling, proliferation, and migration of lung adenocarcinoma cells. B. Amide local anesthetics have been shown to block TNF-α-induced Src activation and ICAM-1 phosphorylation in vitro. Both of these processes may favor the extravasation of tumor cancer cells and metastasis. C. Amide local anesthetics have been shown to significantly inhibit Src protein tyrosine kinase, which lies both upstream and downstream of activated ICAM-1, functioning as a regulator of endothelial permeability— an important component in the prevention of tumor metastasis. D. Amide local anesthetics have been shown to significantly inhibit Src protein tyrosine kinase, which is involved in signalling epithelial-to-mesenchymal transformation and extravasation of cancer cells, a process necessary for the prevention of tumor metastasis.

4/18/19 2:39 PM

CHAPTER 16

ANSWERS AND EXPLANATIONS 1. A is correct. Hypotension and hypovolemia at the time of surgery may influence the immune balance of cancer patients in a deleterious fashion predisposing to cancer cell metastasis. B is incorrect. Hypotension and hypovolemia may influence immune modulation in the perioperative period by suppressing the Th1 response, thereby shifting the balance in favor of Th2. C is incorrect. Th1 response is the immune response necessary for antitumor immunity. D is incorrect. NK cells have a crucial role in antitumor immunity. 2. C is correct. NK cells have a crucial role, particularly in eliminating metastases without prior sensitization and major histocompatibility complex (MHC) restriction. A is incorrect. The natural killer (NK), CD4+ Th1 (T helper 1), and CD8+ CTL (cytotoxic lymphocyte) cells are the major antitumor immune effector cells. B is incorrect. Th2 (T helper 2), tumor-associated macrophages (TAM), and regulatory T (Treg) cells are among the most important cells in the promotion of tumor growth and metastasis. D is incorrect. NK cell function can be downregulated by prostaglandins, especially prostaglandin E2 (PGE2) by shifting cytokine balance toward Th2 dominance and promoting tumor angiogenesis. E is incorrect. Proinflammatory cytokines (IL-6, tumor necrosis factor alpha [TNF-α], IL-13) released from infiltrating leukocytes close to the primary tumor site can activate nuclear factor κB (NF-κB) and STAT3 in cancer cells and thus contribute to cancer proliferation and survival. 3. C is correct. Midazolam and ketamine have both been shown to impair antitumor immune responses in vivo and in vitro. This is done via impaired dendritic cell induction of the Th1-type immune response. A is incorrect. Hypothermia has been shown to suppress NK cell activity. B is incorrect. While hyperglycemia does lead to the activation of the surgical stress response, no data have shown that perioperative hyperglycemia is associated with tumor spread or metastasis. D is incorrect. Allogenic blood transfusion has been shown to have a diverse immunomodulatory effect. Also, while some suppression of the antitumor elements of the immune system has been demonstrated, this is confounded by the fact that sicker patients tend to receive blood transfusion and this finding may be a result of this. Furthermore, anemia may also have deleterious effects on perioperative immunity. Therefore, this patient subset requires a balanced approach to blood transfusion.

Hadzic_Ch16_p065-070.indd 67

Local Anesthetics, Regional Anesthesia, and Cancer Recurrence

67

4. C is correct. Propofol has inhibiting activity against cyclooxygenase (COX) 2. This is beneficial because in most human cancer cells COX-2 is overexpressed. A is incorrect. Halothane, isoflurane, and sevoflurane have been shown to suppress NK cell activity (an important component of the antitumor immune response). The exact mechanism by which these agents interact with the immune system is unclear. B is incorrect. Propofol at clinical concentrations has not been shown to have an effect on NK cells. D is incorrect. While propofol (not isoflurane) increases the Th1/Th2 ratio, this is beneficial to cancer patients by attenuating the metastatic process. 5. B is correct. Morphine-induced immunosuppression is mediated by its binding to a member of the seventransmembrane G-protein-coupled μ-opioid receptors (MORs) on immune cells. A is incorrect. All opioids exert different effects on the immune system, and this makes their assessment (as to their effect on cancer cells) as a group complex and challenging. For example, morphine administration reduces CTL and NK cell functions and IL-2 and IFN-γ expression in T cells. Tramadol has been shown to increase NK cell activity. Fentanyl was shown to have a suppressive effect on NK activity in nonsurgical individuals but to have a positive effect in operative subjects. In contrast, remifentanil did not impair NK activity. C is incorrect. Epidemiologic studies suggested that patients who receive general anesthesia with opioids rather than local anesthetics (LAs) or regional anesthetics may have a higher rate of cancer recurrence. D is incorrect. Previous studies have shown that the expression of MORs was increased in lung cancer cells and that the silencing of MOR in those cells decreased tumor growth and metastasis in mouse models. E is incorrect. Previous studies have also shown that MOR1 expression has an effect on cancer progression, with overexpression of MOR1 in human non-small cell lung cancer (NSCLC) cells showing increased proliferation, migration, invasion, and transendothelial migration. 6. A is correct. In vitro studies have demonstrated antiproliferative and cytotoxic effects of amide local anesthetics on cancer cells. B is incorrect. The dose range for amide local anesthetics required to produce antiproliferative effects is within that used in the clinical setting. For example, Martinsson et al demonstrated that ropivacaine suppressed growth and proliferation of human colon adenocarcinoma cells in vitro in a dose-dependent manner, with the effective concentrations within therapeutic range. C is incorrect. In the absence of surgery, epidural anesthesia has only minor effects on immune cell functions. However, several preliminary studies in humans have indicated the beneficial effects of epidural anesthesia on immune cell functions in the setting of surgery-induced stress response.

4/18/19 2:39 PM

68

PART 2

Foundations of Local and Regional Anesthesia

D is incorrect. Animal studies have demonstrated antiproliferative effects of amide local anesthetics on cancer cells. For example, Wada et al demonstrated that spinal block attenuated the prometastatic immune suppression demonstrated in the rat model undergoing laparotomy. Bar-Yosef et al showed that spinal block decreased lung metastasis in a rat model undergoing lung resection surgery for lung cancer—the group without spinal blockade showed a 17-fold increase in lung metastasis. E is incorrect. Conflicting data have been produced so far in terms of clinical studies. Thus far, studies investigating associations between local anesthetics and metastasis after cancer surgery have been limited by many factors, including small numbers, retrospective nature, limited availability of medical records (thus limited information on type of anesthetic, amount, and so on), lack of control selection, and the presence of multiple confounding factors impacting on cancer recurrence (including surgical technique). 7. E is correct. The role of cell adhesion molecules (CAMs) such as ICAM-1, VCAM-1, E-selectin, and P-selectin has been studied extensively in the process of inflammation. CAMs have been implicated in tumor progression. A is incorrect. Local anesthetics have anti-inflammatory properties and do affect the mediators of inflammation at the molecular level. There is also evidence that inflammatory mechanisms may play a role in cancer metastasis. B is incorrect. Cancer and inflammation are connected by two pathways: an intrinsic one and an extrinsic one. The two pathways are linked and lead to a common one and follow the activation of the nuclear transcription factor NF-κB and other transcription factors. C is incorrect. The intrinsic pathway is activated by genetic events such as mutations, chromosomal rearrangements, and so on that affect oncogenes. Cells that are transformed by these events produce inflammatory mediators, thereby generating an inflammatory microenvironment in the tumor area. D is incorrect. The extrinsic pathway is the inflammatory or infectious condition that is responsible for the risk of developing cancer at certain sites, such as colon, prostate, and pancreas. 8. D is correct. ICAM-1 is associated with a variety of cancer types and has a role in cancer metastasis. It could, therefore, be useful as a biomarker for tumor prognosis as well as a target for therapeutic interventions. A is incorrect. ICAM-1 expression in serum of patients with malignant melanoma was associated with a reduction in the disease-free interval and survival. B is incorrect. Significantly higher levels of serum ICAM-1 were detected in patients with liver metastasis, and its levels were increased in serial blood samples obtained from patients with progressing disease. C is incorrect. Overexpression of ICAM-1 influenced tumor progression in colorectal cancer, with high expression of ICAM-1 in tumors an indicator of metastatic potential and poor prognosis.

Hadzic_Ch16_p065-070.indd 68

9. A is correct. The Src family of nonreceptor protein tyrosine kinases plays a critical role in a variety of cellular signal transduction pathways, including those important for the proliferation of the tumor, disruption of cell/ cell contacts, migration, invasiveness, and resistance to apoptosis. B is incorrect. Activation of Src family kinases is common in a variety of human cancers, may occur through different mechanisms, and is frequently a critical event in tumor progression. C is incorrect. Once tumor cells leave their primary site, they enter the blood vessels and again they extravasate to form satellite lesions. Previous studies in mice have shown that endothelial barrier disruption by VEGF-mediated Src activation potentiated tumor cell extravasation and metastasis. D is incorrect. Previous studies have shown that mice genetically deficient in the c-Src are resistant to tumor cell metastasis. 10. B is correct. Amide local anesthetics have been shown to block TNF-α-induced Src activation and ICAM-1 phosphorylation in vitro. Both of these processes may favor the extravasation of tumor cancer cells and metastasis. A is incorrect. A recent study showed that lidocaine and ropivacaine may inhibit inflammatory cytokine signalling, proliferation, and migration of lung adenocarcinoma cells. C is incorrect. Amide local anesthetics have been shown to significantly inhibit Src protein tyrosine kinase, which lies both upstream and downstream of activated ICAM-1, functioning as a regulator of endothelial permeability—an important component for tumor metastasis. D is incorrect. Amide local anesthetics have been shown to significantly inhibit Src protein tyrosine kinase, which is involved in signalling epithelial-to-mesenchymal transformation and extravasation of cancer cells, a process necessary for tumor metastasis.

Suggested Readings Al-Hasani R, Bruchas MR. Molecular mechanisms of opioid receptor-dependent signaling and behavior. Anesthesiology. 2011;115:1363-1381. Eltzschig HK, Carmeliet P. Hypoxia and inflammation. N Engl J Med. 2011;364:656-665. Hadzic A. Local anesthetics, regional anesthesia, and cancer recurrence. In: Borgeat A, Aguirre J, Votta-Velis EG, eds. Textbook of Regional Anesthesia and Acute Pain Management. 2nd ed. New York, NY: McGraw-Hill Publishing; 2017:chap 17. Hayes SH, Seigel GM. Immunoreactivity of ICAM-1 in human tumors, metastases and normal tissues. Int J Clin Exp Pathol. 2009;2:553-560. Huang WC, Chan ST, Yang TL, Tzeng CC, Chen CC. Inhibition of ICAM-1 gene expression, monocyte adhesion and cancer cell invasion by targeting IKK complex: molecular and functional study of novel alpha-methylene-gamma-butyrolactone derivatives. Carcinogenesis. 2004;25:1925-1934. Inada T, Kubo K, Shingu K. Possible link between cyclooxygenaseinhibiting and antitumor properties of propofol. J Anesth. 2011;25:569-575.

4/18/19 2:39 PM

CHAPTER 16 Johnson JP. Cell adhesion molecules in the development and progression of malignant melanoma. Cancer Metastasis Rev. 1999;18:345-357. Kennedy R, Celis E. Multiple roles for CD4+ T cells in anti-tumor immune responses. Immunol Rev. 2008;222:129-144. Kim I, Moon SO, Kim SH, Kim HJ, Koh YS, Koh GY. Vascular endothelial growth factor expression of intercellular adhesion molecule 1 (ICAM-1), vascular cell adhesion molecule 1 (VCAM-1), and E-selectin through nuclear factor-kappa B activation in endothelial cells. J Biol Chem. 2001;276:7614-7620. Maruo Y, Gochi A, Kaihara A, et al. ICAM-1 expression and the soluble ICAM-1 level for evaluating the metastatic potential of gastric cancer. Int J Cancer. 2002;100:486-490. Ohta N, Ohashi Y, Fujino Y. Ketamine inhibits maturation of bone marrow-derived dendritic cells and priming of the Th1-type immune response. Anesth Analg. 2009;109:793-800. Ohta N, Ohashi Y, Takayama C, Mashimo T, Fujino Y. Midazolam suppresses maturation of murine dendritic cells and priming of lipopolysaccharide-induced t helper 1-type immune response. Anesthesiology. 2011;114:355-362.

Hadzic_Ch16_p065-070.indd 69

69

Local Anesthetics, Regional Anesthesia, and Cancer Recurrence Rosette C, Roth RB, Oeth P, et al. Role of ICAM1 in invasion of human breast cancer cells. Carcinogenesis. 2005;26:943-950.

Son KA, Kang JH, Yang MP. Ketamine inhibits the phagocytic responses of canine peripheral blood polymorphonuclear cells through the upregulation of prostaglandin E2 in peripheral blood mononuclear cells in vitro. Res Vet Sci. 2009;87:41-46. Stojanovic A, Cerwenka A. Natural killer cells and solid tumors. J Innate Immun. 2011;3:355-364. Summy JM, Gallick GE. Src family kinases in tumor progression and metastasis. Cancer Metastasis Rev. 2003;22:337-358. van Meurs M, Wulfert FM, Jongman RM, et al. Hemorrhagic shock-induced endothelial cell activation in a spontaneous breathing and a mechanical ventilation hemorrhagic shock model is induced by a proinflammatory response and not by hypoxia. Anesthesiology. 2011;115:474-482. Welters ID, Menzebach A, Goumon Y, et al. Morphine suppresses complement receptor expression, phagocytosis, and respiratory burst in neutrophils by a nitric oxide and mu(3) opiate receptor-dependent mechanism. J Neuroimmunol. 2000;111:139-145.

4/18/19 2:39 PM

This page intentionally left blank

Hadzic_FM_p00i-pxx.indd 6

22/04/19 6:40 PM

17 Perioperative Regional Anesthesia and Analgesia: Effects on Cancer Recurrence and Survival After Oncological Surgery Tong J. Gan and Yanxia Sun

QUESTIONS DIRECTIONS: Choose the one best response to each question. 1. Which of the following is the main reason for cancerrelated death? A. Primary cancer B. Metastatic recurrence C. Comorbidity D. Cancer treatment 2. Which of the following is not a recognized factor that contributes to metastatic spread? A. Surgery B. Regional anesthesia C. General anesthesia D. Opioid administration 3. Which of the following comments is true based on the current evidence? A. Regional anesthesia may improve overall survival rate and reduce cancer recurrence following oncology surgery. B. Regional anesthesia may improve overall survival but did not reduce cancer recurrence after oncologic surgery. C. Regional anesthesia reduced cancer recurrence but did not affect overall survival rate after oncology surgery. D. Regional anesthesia did not reduce either overall survival rate or cancer recurrence after oncology surgery. 4. The use of opioids during surgery and pain management after surgery may contribute to metastatic spread due to: A. Inhibition of immune function and stimulation of angiogenesis B. Reducing postoperative pain C. Increasing growth factors that cause malignant cells to proliferate

5. Which of the following is not a plausible explanation for the beneficial effect of regional anesthesia on cancer recurrence after oncology surgery? A. Regional anesthesia could potentially attenuate the stress response related to surgery. B. Regional anesthesia could preserve immune function. C. Regional anesthesia decreases the intraoperative use of volatile anesthetics and perioperative opioid. D. Regional anesthesia could inhibit the function of NK cells. 6. The discrepancy of the effect of regional anesthesia (RA) on overall and recurrence-free survival may be due to the following potential explanations: A. RA may not significantly influence recurrence but might slow down its progression, resulting in a prolonged survival. B. RA may increase survival independently of underlying disease through better rehabilitation, less chronic pain, and anti-inflammatory properties. C. Lack of statistical power could be a limitation to demonstrate the true association. D. All of the above 7. Tumor biology varies considerably from organ to organ, and the effects of regional anesthesia (RA), if any, are likely to vary from site to site. Based on the current evidence, which of the following cancer types sees more benefits from RA? A. Colorectal cancer B. Ovarian caner C. Prostate cancer D. Not known 8. Which of the following is a significant concern for anesthesiologists managing oncology patients undergoing surgery? A. Anesthesia technique B. Fluid therapy C. Pain management D. Airway management 71

Hadzic_Ch17_p071-074.indd 71

22/04/19 1:39 PM

72

PART 2

Foundations of Local and Regional Anesthesia

9. The best practice for perioperative care of oncology surgical patients should be: A. Good analgesia, amelioration of the stress response, and reduction of inflammation B. Regional anesthesia C. Opioid treatment D. General anesthesia 10. Which of the following is not true? A. Cancer-associated pain, whether acute or chronic, requires treatment. B. While there is ongoing research on the potential effect of anesthesia and analgesia on recurrence or metastasis, there is currently no evidence to justify altering anesthetic techniques in cancer patients. C. Cancer patients should continue to receive best practice anesthetic techniques in accordance with their own decisions and comorbidities, as discussed with their individual anesthesiologist. D. Regional anesthesia and analgesia should be used for oncology surgery because it improves cancer recurrence-free survival.

ANSWERS AND EXPLANATIONS 1. B is correct. Metastatic recurrence, rather than the primary cancer, accounts for approximately 90% of cancer-related deaths. 2. B is correct. Regional anesthesia is not a recognized factor that contributes to metastatic spread. The evolution of cancer cells in local and metastatic recurrence depends largely on the balance between the host defenses and the tumor’s ability to seed, proliferate, and attract new blood vessels. However, at least three perioperative factors may influence this balance and contribute to metastatic spread: surgery, general anesthesia, and opioid administration. Regional anesthesia could preserve immune function and attenuate the stress response related to surgery and decreases the intraoperative use of volatile anesthetics and perioperative opioid. This combination of effects might allow enhanced preservation of perioperative immune function and potentially reduces the incidence of cancer recurrence. 3. B is correct. A recently published systematic review and meta-analysis included studies that compared the effects of regional anesthesia on cancer recurrence and mortality with those of general anesthesia. Results showed that perioperative regional anesthesia may improve overall survival following oncologic surgery (HR = 0.84, 95% CI 0.75 to 0.94, I2 = 41%). However, there was no positive association found between regional anesthesia and reduced cancer recurrence (HR = 0.91, 95% CI 0.70 to 1.18, I2 = 83%). 4. A is correct. A majority of cancer patients suffer from severe pain leading to poor quality of life. Opioids remain the mainstay for treating cancer patients with chronic pain and during pre-, intra- and postoperative pain

Hadzic_Ch17_p071-074.indd 72

management. However, opioids could promotes tumor growth and metastasis via induction of angiogenesis and lymphangiogenesis. Opioids can also have pro-cancer progression effect via dysregulation of apoptosis and inhibition of natural killer (NK) cells leading to the reduction in immune response. Opioid-induced mast cell activation can lead to enhanced tumor growth via action of substance P and other mediators on endothelial cells and tumor cells. 5. D is correct. Regional anesthesia could preserve immune function and attenuate the stress response related to surgery and decreases the intraoperative use of volatile anesthetics and perioperative opioid. This combination of effects might allow enhanced preservation of perioperative immune function and potentially reduces the incidence of cancer recurrence. Surgical stress, not regional anesthesia, could inhibit the function of NK cells. 6. D is correct. All statements (A, B, and C) are correct potential explanations for the discrepancy of the effect of regional anesthesia (RA) on overall and recurrence-free survival. 7. D is correct. Although the effects of RA on cancer recurrence may be specific to certain tumors, subgroup analyses of current meta-analysis indicated that the absence of an effect was consistent for different types of cancer (colorectal, prostate, and ovarian cancer). It appears that RA does not differentially affect recurrence or overall survival for various cancer types. 8. A is correct. The importance of cellular immunity in long-term outcome after cancer surgery has been well demonstrated. Animal models and human studies both point to NK cell activity in the perioperative period as being a critical factor in determining outcome after potentially curative surgery. Other components of host immunity play important roles. The possible interaction between factors under the control of the anesthetist, such as anesthetic technique, and cellular immunity is becoming increasingly clear. For instance, there are multiple reports of specific drugs effecting NK cell activity. Acute pain suppresses NK cell activity. Optimizing postoperative pain management may attenuate the post-surgical inhibition of host anti-tumor defense mechanisms, including of NK cells. This has been demonstrated in a rat model. The potentially deleterious effects of acute pain are difficult to separate from the effects of opioids. One conclusion is that opioids improve in vivo cancer resistance only in the setting of postoperative pain and that opiates given under basal conditions can be immunosuppressive and pro-metastatic. Therefore, the anesthetic technique is crucial for pain management after oncological surgery. 9. A is correct. Three perioperative factors may contribute to metastatic spread: surgery, general anesthesia, and opioid administration. Surgery itself may depress cellmediated immunity, natural killer (NK) cells activity in particular, reduce antiangiogenic factors, as well as increase proangiogenic factors and growth factors responsible for the proliferation of malignant tissue. In addition,

22/04/19 1:39 PM

CHAPTER 17

Perioperative Regional Anesthesia and Analgesia: Effects on Cancer Recurrence and Survival After Oncological Surgery

general anesthesia with drugs such as thiopental or halogenated volatile agents may also modulate host immunity by inhibiting NK cell, dendritic cell, macrophages and neutrophils function. Finally, the use of opioids during surgery and pain management after surgery have been shown to inhibit immune function and stimulates angiogenesis. 10. D is not true, so option D is correct. Based on retrospective data, perioperative Regional Anesthesia (RA) may improve overall survival after oncologic surgery. However,

Hadzic_Ch17_p071-074.indd 73

73

current evidence does not support positive association between RA and cancer recurrence-free survival. Further prospective clinical trials are needed to clarify this important issue.

Suggested Reading Hadzic A. Perioperative regional anesthesia and analgesia: effects on cancer recurrence and survival after oncological surgery. In: Gan ZS, Sun Y, Gan TG, eds. Textbook of Regional Anesthesia and Acute Pain Management. 2nd ed. New York, NY: McGraw-Hill Publishing; 2017:chap 18.

22/04/19 1:39 PM

This page intentionally left blank

Hadzic_FM_p00i-pxx.indd 6

22/04/19 6:40 PM

PART 3 Clinical Practice of Regional Anesthesia

PART 3A

Local and Infiltrational Anesthesia Chapter 18

Intra-articular and Periarticular Infiltration of Local Anesthetics  77

Chapter 19

Regional and Topical Anesthesia for Awake Endotracheal Intubation  81

Hadzic_Ch18_p075-080.indd 75

22/04/19 1:41 PM

This page intentionally left blank

Hadzic_FM_p00i-pxx.indd 6

22/04/19 6:40 PM

18 Intra-articular and Periarticular Infiltration of Local Anesthetics Meg A. Rosenblatt and Alison Krishna

QUESTIONS DIRECTIONS: Choose the one best response to each question. 1. Intra-articular injection for postoperative analgesia is best comprised of what type of local anesthetic mixture? A. High volume, high concentration B. High volume, low concentration C. Low volume, high concentration D. Low volume, low concentration 2. All of the following characteristics of local infiltration analgesia contribute to the safety of the technique except: A. Use of large volume of local anesthetic B. Use of dilute local anesthetic concentrations C. Absence of major blood vessels in articular structures D. Use of vasoconstricting adjuvants 3. Why is the chondrotoxic effect of bupivacaine not a concern when using it for local infiltration analgesia after a total knee replacement? A. Toxicity is seen only after prolonged infusions. B. Adding epinephrine to the bupivacaine mitigates the toxicity. C. Injecting with a short-bevel needle prevents the associated cartilage damage. D. All cartilage is removed from the joint during surgery. 4. Which of the following is an advantage of local infiltration analgesia (LIA) over peripheral nerve block for total knee or total hip arthroplasty? A. Single injection location B. Potential for continuous delivery of analgesic medication C. Ability to add nonlocal anesthetic adjuvants D. Lack of motor impairment

5. Which of the following is an advantage of local infiltration analgesia over epidural analgesia for total knee or total hip arthroplasty? A. Potential for continuous delivery of analgesic medication B. Ability to use technique in anticoagulated patients C. Avoidance of infectious complications D. Decreased post-op nausea 6. Regarding local infiltration analgesia for total knee arthroplasty, which of the following is true? A. Analgesia is most effective when medications are injected only in the posterior joint capsule. B. No additional benefit has been shown with anesthetic volumes > 50 mL. C. Reduced opioid consumption has been shown with LIA over placebo. D. Continuous LIA catheters should not be used > 24 hours due to the risk of infection. 7. Which of the following is true regarding the use of local infiltration analgesia via a catheter-based infusion after total knee arthroplasty? A. The catheter should be positioned in the subcutaneous plane to avoid contamination of the sterile prosthesis. B. Catheter placement should be considered in a patient who may be unable to receive multimodal analgesia postoperatively. C. Top-up injections via the catheter should begin 24 hours postoperatively. D. Top-up injections via the catheter should be carried out only by trained anesthesia personnel. 8. Compared to a single-shot LIA, the use of an LIA catheter is associated with which of the following? A. Increased risk of infection B. Improved quality of analgesia C. Decreased narcotic usage D. Longer duration of analgesia

77

Hadzic_Ch18_p075-080.indd 77

22/04/19 1:41 PM

78

PART 3

Clinical Practice of Regional Anesthesia

9. Regarding LIA for total hip arthroplasty, which of the following is true? A. Direct injection of medications into muscle should be avoided. B. Adjuvant medications have a local effect without systemic absorption. C. The use of LIA may decrease opioid consumption up to 2 weeks postoperatively. D. Continuous infusion technique is recommended due to larger incision as compared to total knee arthroplasty. 10. During a total hip arthroplasty the femoral component is surgically prepared. Which of the following muscles must be injected when providing analgesia for this portion of the procedure? A. Obturator B. Piriformis C. Pectineus D. Sartorius

ANSWERS AND EXPLANATIONS 1. B is correct. A high volume of injectate is necessary due to the many structures and layers that must be infiltrated. This necessitates a low concentration of local anesthetic to avoid administration of a toxic dose of local anesthetic. A, C, and D are incorrect. All of the other answer choices involve administration of toxic doses of local anesthetic (A) or inadequate coverage of structures (C, D). 2. A is correct. The use of large volume of local anesthetic does not contribute to the safety of the technique. One of the problems with local infiltration analgesia is that many structures and layers must be infiltrated, requiring a large volume of local anesthetic, putting the patient at potential risk for local anesthetic toxicity. B is incorrect. Use of dilute local anesthetic allows for a large volume of local to be infiltrated without risking delivering a toxic dose of local anesthetic. C is incorrect. Absence of major blood vessels in articular structures prevents systemic uptake of local anesthetic that can occur with peripheral blockade of nerves that often run with vascular structures. D is incorrect. Use of vasoconstricting adjuvants, specifically epinephrine, is often added to large volume infiltration solutions to prolong their local action and prevent systemic uptake. 3. D is correct. There are several theoretical concerns with infiltration of local anesthetics and adjuvant medications for joint replacement surgery. Local anesthetic may be neurotoxic to small nerves, bupivacaine has been shown to be chondrotoxic, and NSAIDs/glucocorticoids may slow/ impair wound healing. In joint replacement surgery, the native cartilage is removed and thus the chondrotoxicity of bupivacaine is not of clinical significance.

Hadzic_Ch18_p075-080.indd 78

A is incorrect. While neurotoxicity has been shown with repeated doses of local anesthetic, dilute ropivacaine has been proven safe for 2-3 days of continuous infusion. Bupivacaine is chondrotoxic in both single doses and continuous infusion; however, this is not clinically relevant for the reasons above. B and C are incorrect. Neither adding epinephrine nor using a specific needle mitigates the chondrotoxicity of bupivacaine. 4. D is correct. A major benefit of LIA is the lack of motor impairment often seen with alternative techniques of nerve blocks, such as femoral or sciatic nerve blocks, or epidural analgesia. A is incorrect. While LIA is a simple infiltrative technique, multiple injections must be made into several tissue planes and structures as compared with peripheral nerve blockade in which individual injections are made around specific nerves. B is incorrect. Catheter-based techniques with continuous analgesia can be employed in both LIA and peripheral nerve blockade. C is incorrect. Nonlocal anesthetic adjuvants can be added for multimodal analgesia and prolongation of blockade in both peripheral nerve blockade and LIA. 5. B is correct. Unlike epidural analgesia, which requires practitioners to follow the ASRA guidelines regarding anticoagulants and neuraxial blockade, infiltrative analgesia can be performed safely in patients with abnormal coagulation status. A is incorrect. Catheter-based techniques with continuous analgesia can be employed in both LIA and epidural analgesia. C is incorrect. Infectious complications are a concern in both epidural analgesia and LIA and aseptic technique should be employed when delivering a top-up for both modalities. D is incorrect. While one study has shown similar side effects between LIA and placebo, most studies have been focused on pain scores and post-op opioid consumption. Further studies are needed to compare the difference in side effects between the techniques. 6. C is correct. Multiple studies comparing both single-shot and catheter-based LIA to femoral, epidural, and placebo have shown reduced opioid consumption when LIA is used. A is incorrect. For optimal analgesia, the LIA solution should be infiltrated into the posterior capsule by the surgeon, circularly around the prosthesis, and into the fascia and subcutis. B is incorrect. Early studies published described a volume of local anesthetic and adjuvants less than 50 mL; however, later studies have shown increased benefit with a highvolume solution > 100 mL.

22/04/19 1:41 PM

CHAPTER 18

D is incorrect. The technique described in this book involves aseptic top-ups at 8, 16, and 24 hours postoperatively. There is a risk of infection with any indwelling catheter; however, the duration of use is not limited to 24 hours and should be determined by the individual practitioner. 7. B is correct. While studies have indicated no additional benefit to an LIA catheter in addition to optimal multimodal therapy, they have indicated some benefit if not all components of a multimodal regimen are used and should be considered in patients who are not able to tolerate a PO multimodal regimen. A is incorrect. The use of catheters in major joint replacement is controversial due to the risk of contamination of the prosthetic implant. However, for optimal analgesia, the catheter should be positioned lateral to the knee joint. C is incorrect. Local anesthetics used for infiltration have a limited duration of action. If a catheter-based approach is chosen, top-ups should begin at 8 hours postoperatively. D is incorrect. One of the benefits of local infiltration analgesia is its simplicity, with aseptic top-ups able to be done easily by a nurse in the recovery room or on the hospital floor. 8. A is correct. Catheter placement after total joint replacement has been considered a theoretical risk for infection and a recent report has shown an increased rate of infections with catheters. B, C, and D are incorrect. While multiple studies have proven the superior analgesia of LIA compared to placebo or IV analgesics, no studies have proven catheter-based LIA to be superior to single shot in quality of analgesia, narcotic usage, or duration of analgesia. 9. C is correct. Multiple studies comparing LIA to placebo and IV opioids for total hip arthroplasty have shown improved pain scores and decreased opioid consumption. In addition, one study by Anderson et al demonstrated less pain and opioid consumption up to 2 weeks postoperatively. A is incorrect. Although the chondrotoxic and neurotoxic effects of local anesthetics are known, direct muscle injection has been shown to be safe and the LIA technique for total hip arthroplasty includes injection into the adductor, external rotator, and gluteus muscles. B is incorrect. While it is known that a portion of the infiltrated medications is systemically absorbed, it is unknown whether the analgesic effect of adjuvant medications is due to local effect or systemic absorption. Studies have shown that morphine and ketorolac have a local effect; however, further studies are needed to fully elucidate the mechanism. D is incorrect. Unlike total knee arthroplasty, where studies have shown that catheter placement may be of benefit, there is strong evidence that a catheter-based approach

Hadzic_Ch18_p075-080.indd 79

79

Intra-articular and Periarticular Infiltration of Local Anesthetics

does not confer any additional benefit after total hip arthroplasty. 10. A is correct. Unlike LIA for total knee arthroplasty, analgesia for total hip arthroplasty involves direct injection into several muscle groups including the adductor muscles, gluteus medius, and external rotators (quadratus femoris, obturator, and the tendon of the gluteus maximus). B, C, and D are incorrect. Injection into the other muscles listed is not necessary for adequate analgesia after total hip arthroplasty.

Suggested Readings Hadzic A. Textbook of Regional Anesthesia and Acute Pain Management. 2nd ed. New York, NY: McGraw-Hill Publishing; 2017. Andersen KV, Bak M, Christensen BV, et al. A randomized, controlled trial comparing local infiltration analgesia with epidural infusion for total knee arthroplasty. Acta Orthop. 2010;81:606-610. Andersen KV, Nikolajsen L, Haraldsted V, et al. Local infiltration analgesia for total knee arthroplasty: should ketorolac be added? Br J Anaesth. 2013;111:242-248. Andersen LO, Husted H, Kristensen BB, et al. Analgesic efficacy of subcutaneous local anaesthetic wound infiltration in bilateral knee arthroplasty: a randomised, placebo-controlled, doubleblind trial. Acta Anaesthesiol Scand. 2010;54:543-548. Andersen LO, Husted H, Otte KS, et al. A compression bandage improves local infiltration analgesia in total knee arthroplasty. Acta Orthop. 2008;79:806-811. Andersen LO, Husted H, Otte KS, et al. High-volume infiltration analgesia in total knee arthroplasty: a randomized, doubleblind, placebo-controlled trial. Acta Anaesthesiol Scand. 2008;52:1331-1335. Banks A. Innovations in postoperative pain management: continuous infusion of local anesthetics. AORN J. 2007;85:904-914. Busch CA, Shore BJ, Bhandari R, et al. Efficacy of periarticular multimodal drug injection in total knee arthroplasty. A randomized trial. J Bone Joint Surg Am. 2006;88:959-963. Carli F, Clemente A, Asenjo JF, et al. Analgesia and functional outcome after total knee arthroplasty: periarticular infiltration vs continuous femoral nerve block. Br J Anaesth. 2010;105:185-195. Essving P, Axelsson K, Kjellberg J, et al. Reduced morphine consumption and pain intensity with local infiltration analgesia (LIA) following total knee arthroplasty. Acta Orthop. 2010;81:354-360. Gomez-Cardero P, Rodriguez-Merchan EC. Postoperative analgesia in TKA: ropivacaine continuous intraarticular infusion. Clin Orthop Relat Res. 2010;468:1242-1247. Hadzic A. Intra-articular and periarticular infiltration of local anesthetics. In: Raeder J, Spreng UJ, eds. Textbook of Regional Anesthesia and Acute Pain Management. 2nd ed. New York, NY: McGraw-Hill Publishing; 2017:chap 19. Kazak BZ, Aysu SE, Darcin K, et al. Intraarticular levobupivacaine or bupivacaine administration decreases pain scores and provides a better recovery after total knee arthroplasty. J Anesth. 2010;24:694-699. Murphy TP, Byrne DP, Curtin P, et al. Can a periarticular levobupivacaine injection reduce postoperative opiate consumption during primary hip arthroplasty? Clin Orthop Relat Res. 2012;470:1151-1157.

22/04/19 1:41 PM

80

PART 3

Clinical Practice of Regional Anesthesia

Piper SL, Kramer JD, Kim HT, Feeley BT. Effects of local anesthetics on articular cartilage. Am J Sports Med. 2011;39:2245-2253. Spreng UJ, Dahl V, Hjall A, et al. High-volume local infiltration analgesia combined with intravenous or local ketorolac + morphine compared with epidural analgesia after total knee arthroplasty. Br J Anaesth. 2010;105:675-682.

Hadzic_Ch18_p075-080.indd 80

Toftdahl K, Nikolajsen L, Haraldsted V, et al. Comparison of periand intraarticular analgesia with femoral nerve block after total knee arthroplasty: a randomized clinical trial. Acta Orthop. 2007;78:172-179. Vendittoli PA, Makinen P, Drolet P, et al. A multimodal analgesia protocol for total knee arthroplasty. A randomized, controlled study. J Bone Joint Surg Am. 2006;88:282-289.

22/04/19 1:41 PM

19 Regional and Topical Anesthesia for Awake Endotracheal Intubation Michael Akerman and Tiffany Tedore

QUESTIONS DIRECTIONS: Choose the one best response to each question. 1. A 56-year-old woman with a history of neck radiation and difficult intubation is scheduled for a laparoscopic cholecystectomy. An awake oropharyngeal intubation with nerve blocks is planned. Blockade of which cranial nerve would best anesthetize the vocal cords? A. Glossopharyngeal nerve B. Hypoglossal nerve C. Trigeminal nerve D. Vagus nerve 2. What are the relevant landmarks for a superior laryngeal nerve block? A. The palatoglossal and palatopharyngeal arches B. The hyoid bone and thyroid cartilage C. The thyroid cartilage and cricoid cartilage D. The angle of the jaw and the mastoid process 3. During ultrasound-guided superior laryngeal nerve block a vessel is visualized just below the greater cornu of the hyoid bone. This vessel is most likely to be the: A. Internal carotid artery B. Superior laryngeal artery C. Vertebral artery D. Inferior thyroid artery 4. An injection of lidocaine just deep to the cricothyroid membrane blocks which nerve distribution? A. The pharyngeal nerve B. The internal laryngeal nerve C. The recurrent laryngeal nerve D. The external laryngeal nerve

A. Vertebral artery B. Internal carotid artery C. Superior laryngeal artery D. Inferior thyroid artery 6. A 43-year-old man with no significant past medical history is having an awake fiber-optic intubation performed due to an inhalational injury. Topicalization of the oropharynx is being performed with 20% benzocaine. After successful intubation of the larynx, the SpO2 remains at 86%. You have confirmed correct position with the fiber optic. In addition to providing 100% FiO2, what other treatment should be started? A. Replace the endotracheal tube. B. Administer 1.5 mL/kg 20% intralipid IV. C. Administer 100 mg methylene blue IV. D. Place a surgical airway. E. Place patient in hyperbaric therapy. 7. After receiving topical anesthesia of the nasopharynx and oropharynx, a patient continues to elicit a gag reflex during an awake nasal intubation. Which nerve(s) is responsible for the afferent limb of this response? A. Trigeminal nerve B. Pterygopalatine ganglion C. Vagus nerve D. Glossopharyngeal nerve E. Recurrent laryngeal nerve 8. While performing an awake endotracheal intubation, you decide to use remifentanil for sedation. Which of the following is a disadvantage of this medication? A. Tachycardia B. Spontaneous ventilation C. Recall D. Hypertension E. Uncooperative

5. During the performance of an external (peristyloid) glossopharyngeal nerve block, blood is aspirated prior to the injection of local anesthetic. The needle tip is most likely in which vessel? 81

Hadzic_Ch19_p081-084.indd 81

4/18/19 3:05 PM

82

PART 3

Clinical Practice of Regional Anesthesia

9. Which of the following local anesthetics also has vasoconstrictor properties? A. Cocaine 10% B. Lidocaine 2% C. Bupivacaine 0.5% D. Mepivacaine 1.5% E. Chloroprocaine 2%

ANSWERS AND EXPLANATIONS

10. While preparing to perform an awake fiberoptic intubation, you decide to utilize regional anesthesia techniques to facilitate the procedure. Which of the following is the correct anatomical landmark for the external approach for the glossopharyngeal nerve block? A. Styloid process B. Palatopharyngeal arch C. Cricoid cartilage D. Thyroid cartilage E. Cornu of the hyoid

1. D is correct. The larynx is innervated by the vagus nerve (Figure 19–1). Above the vocal cords (base of tongue, posterior epiglottis, aryepiglottic folds, and arytenoids), the internal branch of the superior laryngeal nerve (a branch of the vagus nerve) supplies innervation. For the vocal cords and below the vocal cords, the recurrent laryngeal nerve (a branch of the vagus nerve) is the supplier. A is incorrect. The glossopharyngeal nerve provides innervation to the pharynx, posterior third of the tongue, fauces, tonsils, vallecular, and anterior surface of the epiglottis. B is incorrect. The hypoglossal nerve does not innervate the pharynx or larynx. C is incorrect. A branch of the trigeminal nerve (the lingual nerve) innervates the anterior two-thirds of the tongue.

Cervical sympathetic ganglion Inferior ganglion of vagus nerve Pharyngeal nerve Superior laryngeal nerve External laryngeal branch Internal laryngeal nerve

Ansa of Galen

Internal laryngeal branch (recurrent laryngeal nerve) Vagus nerve Recurrent laryngeal nerve

Hadzic ic - Laan nce ea/ a/ NYSSO OR RA

FIGURE 19–1  Innervation of the larynx. (Images from NYSORA Continuing Medical Education.)

Hadzic_Ch19_p081-084.indd 82

4/18/19 3:05 PM

CHAPTER 19

Regional and Topical Anesthesia for Awake Endotracheal Intubation

2. B is correct. To perform the block using the external approach, the patient is placed in the supine position and will need a degree of neck extension to facilitate identification of the hyoid bone. Once identified, the hyoid bone is gently displaced to the side where the block is to be performed and a 25-gauge needle is inserted from the lateral side of the neck, aiming toward the greater cornu. Once contact has been made, the needle is walked off the bone inferiorly, and injecting 2 mL of 2% lidocaine here will block both the internal and the external branches of the superior laryngeal nerve (Figure 19–2). If the needle is advanced a few millimeters, it will pierce the thyrohyoid membrane, and a “give” is felt. Injecting local anesthetic here will result in only the internal branch of the superior laryngeal nerve being blocked. As with all blocks, careful aspiration must be performed prior to injection, especially as the carotid artery is in close proximity. If it is difficult to identify the hyoid bone, the superior cornu of the thyroid cartilage can be identified instead. This is located by identifying the thyroid notch, tracing the upper edge posteriorly until the superior cornu can be palpated as a small round structure. This lies just inferior to the greater cornu of the hyoid bone. A, C, and D are incorrect. The palatoglossal and palatopharyngeal arches are the landmarks for the intraoral glossopharyngeal nerve block, while the angle of the jaw and the mastoid process are the landmarks for the external glossopharyngeal nerve block. The thyroid cartilage and cricoid cartilage are landmarks for the translaryngeal block of the recurrent laryngeal nerve.

83

A, C, and D are incorrect. The internal carotid artery, vertebral artery, and inferior thyroid artery are not found in this location. 4. C is correct. To perform this [recurrent laryngeal nerve block], the patient should be supine, with the neck extended be identified in the midline, then the palpating finger should be moved in a caudad direction until the cricoid cartilage is palpated. The cricothyroid membrane lies between these two structures, immediately above the cricoid cartilage. The thumb and the third digit of one hand should stabilize the trachea at the level of the thyroid cartilage, then a 22 or 20 gauge needle should be inserted perpendicular to the skin with the aim to penetrate the cricothyroid membrane (above the cricoid cartilage) (Figure 19–3). A, B, and D are incorrect. The pharyngeal nerve innervates the pharynx. The superior laryngeal nerve lies inferior to the greater cornu of the hyoid bone, where it splits into the internal and external branches. The internal laryngeal nerve penetrates the thyrohyoid membrane, continuing submucosally in the piriform recess. The external laryngeal nerve does not penetrate the thyrohyoid membrane; it descends on the larynx deep to the sternothyroid muscle.

Had Ha dzzicc - Laan nce ea// NY YSSO OR RA FIGURE 19–3  Translaryngeal block. (Images from NYSORA Continuing Medical Education.)

Had Ha Hadz dzzic ic - Lan ance ceaa// NYSO YSO YS OR RA FIGURE 19–2  Superior laryngeal nerve block. (Images from NYSORA Continuing Medical Education.)

3. B is correct. The superior laryngeal nerve can be seen superficial to the thyrohyoid membrane when the medial aspect of the probe is rotated cephalad. The internal branch of the superior laryngeal nerve runs along with the superior laryngeal artery, just below the greater cornu of the hyoid bone.

Hadzic_Ch19_p081-084.indd 83

5. B is correct. The peristyloid approach aims to infiltrate local anesthetic just posterior to the styloid process where the glossopharyngeal nerve lies. In close proximity to this is the internal carotid artery, so care must be taken when using this approach. A, C, and D are incorrect. The vertebral artery, superior laryngeal artery, and inferior artery are not found in this location. 6. C is correct. Methemoglobinemia is a known side effect of topical anesthesia. This results from the oxidation of iron atoms in hemoglobin. Once methemoglobin is formed, this molecule is no longer able to bind oxygen. Cyanosis develops when methemoglobin levels approach 10%.

4/18/19 3:05 PM

84









PART 3

Clinical Practice of Regional Anesthesia

When levels rise to 30% patients may complain of dyspnea, headache, lightheadedness, confusion, and lethargy. Levels > 70% are often fatal. Treatment is with early diagnosis and IV methylene blue. A is incorrect. The endotracheal tube has already been confirmed to be in the correct location. Replacing will not address the underlying problem. B is incorrect. Intralipid is used for the treatment of local anesthetic systemic toxicity (LAST). This patient does not display signs or symptoms of LAST. D is incorrect. This patient has a correctly placed endotracheal tube. Placing a surgical airway will not address the underlying problem. E is incorrect. Hyperbaric therapy is used for carbon monoxide poisoning and has no role in the treatment of methemoglobinemia.

7. D is correct. The gag reflex is triggered by stimulation of areas that are innervated by the glossopharyngeal nerve, posterior third of the tongue, valeculla, and anterior surface of the epiglottis. The efferent arc is provided by the vagus nerve. A is incorrect. The trigeminal nerve is responsible for much of the sensory component of the nose. B is incorrect. The palatine nerves are relayed through the pterygopalatine ganglion and provide part of the sensory innervation to the nose. C is incorrect. The vagus nerve provides sensory innervation to the larynx. Above the vocal cords is innervated by the internal branch of the superior laryngeal nerve. Below the vocal cords is by the recurrent laryngeal nerve. The vagus nerve also acts as the motor limb of the gag reflex. E is incorrect. The recurrent laryngeal nerve is responsible for sensory innervation below the vocal cords. 8. C is correct. Remifentanil has been shown to provide good intubating conditions but results in a higher incidence of recall. A is incorrect. Remifentanil is likely to cause bradycardia, not tachycardia. B is incorrect. Remifentanil may depress the respiratory drive. Even if spontaneous ventilation is maintained, this is not a disadvantage. D is incorrect. Remifentanil is likely to cause hypotension, not hypertension. E is incorrect. Remifentanil use is associated with high patient satisfaction scores and will not cause a patient to be uncooperative.

Hadzic_Ch19_p081-084.indd 84

9. A is correct. Cocaine is the only local anesthetic with vasoconstrictor properties; therefore, it is particularly useful for topical anesthesia of the nasopharynx, which is highly vascular. B, C, D, and E are incorrect. Lidocaine, bupivacaine, and mepivacaine are amide local anesthetics that do not have vasoconstrictor properties. Chloroprocaine is an ester local anesthetic, often used for its quick onset; however, it does not have vasoconstrictor properties. 10. A is correct. The peristyloid approach aims to infiltrate local anesthetic just posterior to the styloid process. B is incorrect. The palatopharyngeal arch is the oral cavity at the base of the tonsillar pillars. This landmark is used for the glossopharyngeal block via an internal approach. C and D are incorrect. The cricothyroid membrane lies between the cricoid and thyroid structure. It is at this point where the recurrent laryngeal nerve is anesthetized. A needle is inserted above the cricoid cartilage. This is done with continuous aspiration and the appearance of bubbles indicates the needle tip in the trachea. At this point the local anesthetic is injected. E is incorrect. The cornu of the hyoid is used as an anatomical landmark for anesthetizing the internal branch of the superior laryngeal nerve.

Suggested Readings Basra et al. Methemoglobinemia after fiberoptic intubation in a patient with an unstable cervical fracture. J Spinal Disorder Tech. 2006;19(4):302-304. Furlan JC. Anatomical study applied to anesthetic block technique of the superior laryngeal nerve. Acta Anaesthesiol Scand. 2002;46:199-202. Hadzic A. Regional and topical anesthesia for awake endotracheal intubation. In: Ahmad I, ed. Textbook of Regional Anesthesia and Acute Pain Management. 2nd ed. New York, NY: McGraw-Hill Publishing; 2017:chap 20. Lallo A, Billard V, Bourgain JL. A comparison of propofol and remifentanil target-controlled infusions to facilitate fiberoptic nasotracheal intubation. Anesthesia & Analgesia. 2009;108(3): 852-857. 3. Pani et al. Regional and topical anaesthesia of upper airways. Indian J Anaesth. 2009;53(6):641-648. Puchner W, Egger P, Puhringer F, Lockinger A, Obwegeser J, Gombotz H. Evaluation of remifentanil as a single drug for awake fiberoptic intubation. Anaesthesiologica Scandanavia. 2002;46(4):350-354.

4/18/19 3:05 PM

PART 3B

Intravenous Regional Block for Upper and Lower Extremity Chapter 20

Hadzic_Ch20_p085-090.indd 85

Intravenous Regional Block for Upper and Lower Extremity Surgery  87

22/04/19 6:04 PM

This page intentionally left blank

Hadzic_FM_p00i-pxx.indd 6

22/04/19 6:40 PM

20 Intravenous Regional Block for Upper and Lower Extremity Surgery Antony R. Tharian, Nebojsa Nick Knezevic, and Kenneth D. Candido

QUESTIONS DIRECTIONS: Choose the one best response to each question. 1. Which of the following statements is not true regarding intravenous regional anesthesia (IVRA) (Bier block)? A. Produces analgesia by the local anesthetic bathing bare nerve endings in the tissue B. Produces analgesia by conduction block in the substance of the nerve C. Produces analgesia by reducing neurogenic inflammation D. Produces analgesia by vasoconstriction of the vasa nervorum 2. For which of the following conditions is a Bier block not an option for providing analgesia? A. Excision of a ganglion cyst in the dorsum of the wrist B. Complex regional pain syndrome affecting the hand C. Raynaud’s disease affecting the fingertips of the upper extremity D. Neuroma excision of the big toe 3. For which of the following conditions can an intravenous regional anesthesia (IVRA) (Bier block) technique be safely used for providing analgesia of the upper extremity? A. Compound fracture of the wrist B. Sickle cell disease C. Closed reduction of distal radius fracture in a 10-yearold boy D. Preexisting arteriovenous fistula in the right upper extremity 4. Which of the following adjunct techniques has been shown to speed up the onset time of intravenous regional anesthesia (IVRA) using lidocaine?

A. Using an equipotent dose of chloroprocaine instead of lidocaine B. Addition of bicarbonate to lidocaine C. Addition of dexmedetomidine to lidocaine D. Addition of acetaminophen to lidocaine 5. A 43-year-old woman underwent carpal tunnel release surgery under intravenous regional anesthesia (IVRA) (Bier block) using prilocaine. She developed headaches, confusion, and dyspnea after release of the tourniquet. She was noted to have cyanosis of the fingertips and the perioral area. Which of the following medications is the drug of choice in the treatment of this condition? A. Lipid emulsion B. Sodium bicarbonate C. Methylene blue D. Nitroglycerine 6. An otherwise healthy 23-year-old man is undergoing excision of a ganglion cyst of the right wrist under intravenous regional anesthesia (IVRA) (Bier block) using lidocaine. He becomes hypotensive soon after the injection of the local anesthetic. Which of the following conditions is the most likely cause of the hypotension? A. Allergic reaction to lidocaine B. Uncontrolled bleeding C. Faulty tourniquet D. Myocardial infarction 7. Which of the following statements regarding intravenous regional anesthesia (IVRA) (Bier block) is true? A. The risk of local anesthetic toxicity is greater with lower extremity procedures than upper extremity procedures. B. Can be safely used for procedures lasting more than 2 hours C. Risk of local anesthetic toxicity is eliminated if the tourniquet is released after at least 30 minutes from the time of injection of local anesthetic. D. 40 mL of 0.5% lidocaine provides longer duration of analgesia than 12 mL of 2% lidocaine.

87

Hadzic_Ch20_p085-090.indd 87

22/04/19 6:04 PM

88

PART 3

Clinical Practice of Regional Anesthesia

8. Which of the following drugs is least useful in the treatment of methemoglobinemia? A. Ascorbic acid B. Methylene blue C. Dextrose D. Normal saline 9. Which of the following techniques has been shown to decrease postoperative pain scores in patients undergoing upper extremity surgery under intravenous regional anesthesia (IVRA) using lidocaine? A. Addition of fentanyl to the local anesthetic B. Addition of clonidine to the local anesthetic C. Addition of dexketoprofen to the local anesthetic D. Addition of acetaminophen to the local anesthetic 10. Which of the following situations increases the chances of local anesthetic toxicity during intravenous regional anesthesia (IVRA)? A. Exercise of the extremity after deflation of the tourniquet B. Keeping the tourniquet on for at least 30 minutes after injection of the local anesthetic C. Placing the tourniquet as close as possible to the site of injection of the local anesthetic D. Inadequate exsanguination of the extremity prior to injection of the local anesthetic

ANSWERS AND EXPLANATIONS 1. D is not true, so option D is correct. Bier block does not produce analgesia by vasoconstriction of the vasa nervorum. While the indirect anesthesia described by Bier is thought to occur due to the transport of local anesthetic to the substance of the nerve through the vasa nervorum, there is no evidence that it causes vasoconstriction of the vasa nervorum. A is true, so option A is incorrect. Cadaver dissections of the upper extremity after injecting methylene blue have shown that direct anesthesia during IVRA occurs as a result of the local anesthetic bathing bare nerve endings in the tissue. B is true, so option B is incorrect. The indirect (delayed onset) anesthesia is thought to occur due to conduction block in the substance of the nerve as a result of the transport of local anesthetic via the vasa nervorum to the substance of the nerve. C is true, so option C is incorrect. In cases of complex regional pain syndrome, Bier block has been shown to decrease neurogenic inflammation, especially when mepivacaine is the local anesthetic chosen for the block. 2. C is correct. Bier block is not an option for providing analgesia in patients with Raynaud’s disease. In patients with severe peripheral vascular disease or preexisting vascular arteriovenous shunts, intravenous regional anesthesia (IVRA) is contraindicated. A is incorrect. Bier Block is an option for this condition. IVRA is most suited for surgery and manipulation of the extremities requiring anesthesia of up to 1 hour duration, and also for peripheral, soft tissue operations such as ganglionectomies.

Hadzic_Ch20_p085-090.indd 88

B is incorrect. Bier Block is an option for this condition. IVRA is effective as a treatment adjunct for complex regional pain syndrome (CRPS), because of its ability to decrease neurogenic inflammation, especially when mepivacaine is the local anesthetic used. D is incorrect. Bier Block is an option for this condition. The only significant difference in IVRA for the upper and lower extremities is that the latter requires relatively larger volumes of local anesthetic, hence it can be safely used for surgical procedures in the lower extremity that take 1 hour or less. 3. C is correct. A retrospective study comparing IVRA and conscious sedation for the reduction of pediatric forearm fractures found IVRA to be a safe, efficient, and costeffective method of reducing pediatric forearm fractures.1 A is incorrect. Compound fractures are a contraindication for IVRA, because of the obvious difficulty in achieving successful exsanguination of the extremity and the possibility of the development of compartment syndrome with the injection of large volumes of local anesthetic. B is incorrect. Sickle cell disease is a contraindication for IVRA because of the risk of regional hypoperfusion and venous stasis that develops with tourniquet inflation, which can precipitate sickling. D is incorrect. Preexisting vascular malformations and/or A-V shunts are a contraindication for IVRA, because of the difficulty in applying a tourniquet successfully over the shunt. 4. C is correct. Adding dexmedetomidine (0.5 μg/kg) to 0.5% lidocaine has been shown to increase the onset speed of sensory and motor block when compared to 0.5% lidocaine alone.2 A is incorrect. When lidocaine was compared with alkalinized and nonalkalinized chloroprocaine, both used as 0.5% concentrations, and used exclusively for hand surgery, alkalinized chloroprocaine behaved similarly to lidocaine, but plain chloroprocaine offered no benefit and produced more side effects than seen with lidocaine.3 B is incorrect. A study comparing alkalinized 0.5% lidocaine using 1.4% sodium bicarbonate with plain 0.5% lidocaine found no clinical advantage in the alkalinized group with respect to the onset of sensory block, motor block, or the appearance of postoperative pain.4 D is incorrect. Addition of acetaminophen (300 mg) to lidocaine 3 mg/kg, diluted to a total volume of 40 mL, had no effect on the onset of sensory block, when compared to lidocaine 3 mg/kg alone, diluted to 40 mL. However, the duration of sensory block was prolonged with the addition of acetaminophen.5 5. C is correct. Methylene blue is the drug of choice in the treatment of severe methemoglobinemia. It provides an artificial electron transporter for the reduction of methemoglobin (MetHb) via the NADPH-dependent pathway. The recommended dose of MB is 1–2 mg/kg IV over 5 minutes. A is incorrect. Prilocaine is metabolized to orthotoluidine, an oxidizing compound capable of converting hemoglobin to MetHb. High MetHb levels can result in the clinical

22/04/19 6:04 PM

CHAPTER 20

Intravenous Regional Block for Upper and Lower Extremity Surgery

picture described in the question. Lipid emulsion is of no use in the treatment of this condition. B is incorrect. Sodium bicarbonate can be used as an adjunct for the management of acidosis in severe methemoglobinemia. D is incorrect. Nitroglycerine is of no use in the treatment of methemoglobinemia. 6. C is correct. A faulty tourniquet can result in the injected lidocaine entering the systemic circulation causing excessive plasma concentrations of lidocaine. This can lead to peripheral vasodilation and diminished cardiac contractility, which is usually clinically seen as hypotension. A is incorrect. Anaphylaxis and immediate allergic reaction, occurring within an hour of administration is exceedingly rare and there are only a handful of cases reported in the medical literature. Delayed type hypersensitivity reactions such as contact dermatitis and localized swelling are more frequently reported. It is very unlikely that an allergic reaction is the most probable cause of hypotension in the above scenario. B is incorrect. Ganglion cyst excision is not usually associated with bleeding and the presence of a tourniquet minimizes the chances of bleeding. D is incorrect. Myocardial infarction is extremely unlikely in this otherwise healthy young patient. 7. A is correct. The obvious size disparity between upper and lower extremities necessitates relatively larger volumes of local anesthetic be used, in order to fill the larger vascular compartment of the lower extremity from the distally placed intravenous cannula to the proximal tourniquet (100 mL vs 50 mL). Inadvertent or unintentional cuff deflation, cuff failure, and an intact interosseous circulation may all result in the release of large doses of local anesthetic into the systemic circulation. B is incorrect. IVRA is most suited for surgery and manipulation of the extremities requiring anesthesia of up to 1 hour duration. C is incorrect. Seizures occurring after tourniquet deflation were reported with tourniquet times as long as 60 minutes and with a delay of up to 10 minutes after tourniquet deflation. D is incorrect. A study comparing low-concentration/ high-volume lidocaine (0.5% of 30–50 mL lidocaine) and higher-concentration/lower-volume lidocaine (2% of 12–15 mL lidocaine) showed a faster onset and delayed regression of sensory block in the higher-concentration/ lower-volume group. 8. D is correct. Normal saline does not offer any potential benefits in the treatment of methemoglobinemia. A is incorrect. Methemoglobinemia can occur when prilocaine is used for intravenous regional anesthesia. Orthotoluidine, a metabolite of prilocaine, is an oxidizing compound capable of converting hemoglobin to methemoglobin. While methylene blue is the drug of choice in the treatment of this condition, ascorbic acid also provides an electron transporter for the reduction of MetHb via the NADPH-dependent pathway.

Hadzic_Ch20_p085-090.indd 89

89

B is incorrect. Methylene blue also provides an artificial electron transporter for the reduction of MetHb via the NADPH-dependent pathway. The dose is 1–2 mg/kg IV over 5 minutes, with the total dose not to exceed 7–8 mg/kg. C is incorrect. Dextrose should be given in methemoglobinemia because the major source of NADH (reduced nicotinamide adenine dinucleotide) in the red blood cells is the catabolism of sugar through glycolysis. Dextrose is also necessary to form NADPH (reduced NAD phosphate) through the hexose monophosphate shunt, which is necessary for methylene blue to be effective. 9. C is correct. Addition of dexketoprofen to lidocaine for IVRA resulted in more rapid onset of sensory and motor block, longer recovery time, decreased intraoperative and postoperative pain scores and decreased postoperative analgesic requirements. A is incorrect. Multiple studies looking at the addition of fentanyl to lidocaine found no benefit in terms of tourniquet pain in IVRA. B is incorrect. Clonidine was found to provide no measurable benefits when added to lidocaine for IVRA in patients undergoing carpal tunnel release. D is incorrect. There was no demonstrable decrease in postoperative pain scores under IVRA when acetaminophen was added to lidocaine. 10. A is correct. Immediately after performing IVRA it is important to maintain the previously anesthetized limb quiescent after deflation of the tourniquet. More than the usual amount of local anesthetic from the isolated limb is released into the systemic circulation if the limb is inadvertently exercised in the immediate postdeflation period, which in turn can increase the chances of development of local anesthetic toxicity. B is incorrect. Usually, while performing IVRA, there are no signs or symptoms of cardiovascular or central nervous system toxicity if the tourniquet is deflated at least 30 minutes after the local anesthetic is injected into the vein. However, there is a case report of seizures occurring in the postdeflation period, even after a tourniquet time of 60 minutes. C is incorrect. Multiple studies have compared the effects of placing the tourniquet in the forearm versus upper arm, in the performance of hand surgeries using IVRA. The placement of tourniquet in the forearm provided excellent surgical anesthesia, with half the dose of the local anesthetic used when the tourniquet was placed in the upper arm. There were no significant differences in the quality of surgical anesthesia, onset, and regression of sensory blocks between groups. The reduction of the amount of local anesthetic in the forearm group would, at least theoretically, be associated with less local anesthetic toxicity. D is incorrect. Inadequate exsanguination of the extremity while performing IVRA would result in a failed block. Failure to exsanguinate adequately does not necessarily increase the chances of local anesthetic toxicity.

22/04/19 6:04 PM

90

PART 3

Clinical Practice of Regional Anesthesia

References 1. Aarons CE, Fernandez MD, Willsey M, Peterson B, Key C, Fabregas J. Bier block regional anesthesia and casting for forearm fractures: safety in the pediatric emergency department setting. J Pediatr Orthop. 2014;34:45-49. 2. Memis D, Turan A, Karamanlioglu B, et al. Adding dexmedetomidine to lidocaine for intravenous regional anesthesia. Anesth Analg. 2004;98:835-840. 3. Lavin P, Henderson C, Vaghadia H. Non-alkalinized and alkalinized 2-chlorprocaine versus lidocaine for intravenous regional anesthesia during outpatient hand surgery. Can J Anaesth. 1999;46:939-945.

Hadzic_Ch20_p085-090.indd 90

4. Benlabed M, Jullien P, Guelmi K, et al. Alkanization of 0.5% lidocaine for intravenous regional anesthesia. Reg Anesth. 1990;15:59-60. 5. Sen H, Kulachi Y, Bicerer E, Ozkan S, Dagli G, Turan A. The analgesic effect of paracetamol when added to lidocaine for intravenous regional anesthesia. Anesth Analg. 2009;109(4):1327-1330.

Suggested Reading Hadzic A. Intravenous regional block for upper and lower extremity surgery. In: Candido KD, Tharian AR, Winnie AP, eds. Textbook of Regional Anesthesia and Acute Pain Management. 2nd ed. New York, NY: McGraw-Hill Publishing; 2017:chap 21.

22/04/19 6:04 PM

PART 3C

Neuraxial Anesthesia

Section 1

Spinal Anesthesia Chapter 21

Neuraxial Anatomy (Anatomy Relevant to Neuraxial Anesthesia)  93

Chapter 22

Spinal Anesthesia  99

Chapter 22A Mechanisms and Management of Failed Spinal Anesthesia  103

Hadzic_Ch21_p091-098.indd 91

22/04/19 6:24 PM

This page intentionally left blank

Hadzic_FM_p00i-pxx.indd 6

22/04/19 6:40 PM

21 Neuraxial Anatomy (Anatomy Relevant to Neuraxial Anesthesia) Barys Ihnatsenka, Kathleen Chan, and Sowmya Kantamneni

QUESTIONS DIRECTIONS: Choose the one best response to each question. 1. In most humans, what level do the dural sac and spinal cord terminate? A. In infants, dural sac at S4 and spinal cord at L3. In adults, dural sac at S2 and spinal cord at L1. B. In adults and infants, dural sac at S2 and spinal cord at L1. C. In infants, the spinal cord is tethered to the dural sac and they both terminate at S4. Shortly after birth, the filum terminale ablates and in adulthood, dural sac terminates at S4 and spinal cord at L1. D. In infants and adults, dural sac terminates at S4. In infants, spinal cord is at L1 and grows down to L3 as an adult. 2. What is the correct palpable landmark and its corresponding vertebral level? A. Root of scapular spine with T4 B. Iliac crest with L4–L5 disc C. Vertebra prominens to T1 D. Inferior angle of the scapula to T10 3. What is the difference between the composition of supraspinous ligament and ligamentum flavum? A. The major composition of supraspinous is type I and type III collagen whereas ligamentum flavum is predominantly type II and type X collagen. B. The major composition of supraspinous is elastin whereas ligamentum flavum is predominantly collagen. C. The major composition of supraspinous is collagen whereas ligamentum flavum is predominantly elastin. D. The major composition of supraspinous is type II and X collagen whereas ligamentum flavum is predominantly type I and type III collagen.

4. How is ligamentum flavum different in the cervical region compared to the lumbar region? A. In the cervical region, ligamentum flavum is thicker to stabilize the connecting lamina of the cervical spine in order to support the weight of the head. B. Ligamentum flavum does not ascend beyond C3, leaving part of cervical spine without ligamentum flavum support whereas it spans the entire lumbar spine. C. There is no difference between ligamentum flavum in the cervical and lumbar region. D. There is a higher likelihood of midline ligamentum flavum gaps in the cervical region than the lumbar region. 5. Which of the following statements is true regarding the epidural space? A. The ligamentum flavum forms the posterior border of the epidural space. B. It surrounds the dura mater circumferentially and extends from the foramen magnum to the sacrococcygeal ligament. C. It contains adipose tissue, blood vessels, nerve roots, and loose connective tissue in a nonuniform distribution. D. All statements are true. 6. Which of the following statements is false regarding the subdural space? A. It is a meningeal plane that lies between the dura and the arachnoid mater. B. It is an acquired space that only becomes real after tearing off neurothelial cells within the space. C. Subdural injection is a very common occurrence, with an estimated incidence of 5% of epidural injections. D. Injection of a small dose of local anesthetic into the area can have profound hemodynamic and sympatholytic effects.

93

Hadzic_Ch21_p091-098.indd 93

22/04/19 6:24 PM

94

PART 3

Clinical Practice of Regional Anesthesia

7. In a scoliotic spine, which needle approach during a neuraxial procedure takes advantage of the less obstructed needle path and largest area in interlaminar space? A. Midline approach B. Paramedian approach from the concave side of the curve C. Paramedian approach from the convex side of the curve D. It makes no difference which approach is taken. 8. Which of the following statements regarding dermatomes is false? A. Every spinal nerve has a sensory dermatomal area defined for it from C1–S5. B. Rootlets that contribute to a root innervate serially overlapping parts of the cutaneous surface cranially to caudally. C. Each cutaneous point is innervated by fibers from as many as five adjacent spinal nerves. D. The dermatome for pain and temperature exceeds the dimensions of the dermatome for touch for the same nerve. 9. Which of the following statements is false regarding cerebrospinal fluid (CSF)? A. There is about 100 mL of CSF within the spinal canal. B. The CSF concentration of glucose is the same as the concentration found within serum. C. CSF is dynamic and flows freely throughout the dural sac and cranially. D. CSF pressure normally ranges between 10–20 mm H2O when patient is on the side. 10. Which layer of the peripheral nerve is continuous with the pia mater? A. Perineurium B. Endoneurium C. Epineurium D. The pia mater is not continuous with the peripheral nerve. 11. The cauda equina nerve roots: A. Have relatively stable specific positions relative to one another within the dural sac B. Are anchored in place by intrathecal ligaments at various regions of the dural sac but slightly change with body position C. Are much more crowded in the upper levels compared to lower levels D. All of the above are true statements about the cauda equina nerve roots. 12. Which of the following is an intrathecal ligament? A. Interspinous ligament B. Filum terminale C. Ligamentum flavum D. Anterior longitudinal ligament

Hadzic_Ch21_p091-098.indd 94

ANSWERS AND EXPLANATIONS 1. A is correct. In the embryo, the spinal cord terminates in the sacral area. During further development, the vertebral column grows faster than the spinal cord leading to the variation in its position as the individual ages. At birth, the conus medullaris terminates at L3 and dura mater terminates at S4. The conus medullaris settles in its final position by 2 months of age as the vertebral column outpaces the spinal cord in growth. From 2 months into adulthood, the conus medullaris finds its terminal position at L1 when the individual is in an upright and erect position and the dural sac terminates distally at S2. Because the dural sac has a more distal termination, inadvertent intrathecal puncture when attempting caudal epidurals in premature infants is more likely than in older children or adults. B is incorrect. After the infant is born, the neuraxiom continues to grow and mature. The bony structures of the spine grow at a different rate than the contents of the spinal column. As a result, the levels of the dural sac and spinal cord do not remain at the same level from birth into adulthood. C is incorrect. A tethered spinal cord is a pathological disorder. This can occur when the filum terminale is thickened and attached to the sacrum or a lipoma growth on the filum terminale tethers the spinal cord to the surrounding sacrum. This syndrome is closely associated with spina bifida. D is incorrect. Because the vertebral column grows faster than the spinal cord, the spinal cord terminates lower at birth, L3, and higher in adulthood, L1. Studies corroborate the majority of spinal cords terminate at the level of L1, L1–L2 disc, and upper L2. There are variations of spinal cord termination as high as T12 and as low as upper L3 in adults. 2. B is correct. The classical teaching iliac crests (Tuffier’s line) cross midline at the level of the L4–L5 disc (Figure 21–1). A is incorrect. The root of the scapular spine is adjacent to T3 spinous process. C is incorrect. The most prominent spinous process in the lower neck when neck is flexed is the spinous process of C7. It may be difficult to differentiate this one occasionally from C6 or T1. D is incorrect. The inferior angle of the scapula corresponds to T7 spinous process. Unsurprisingly, there is anatomical variation observed and different body habitus further decreases the accuracy of the palpable landmark technique. Most commonly, Tuffier’s line corresponds with L4–L5 disc; however, the line can correspond with a space as high as L3–L4 disc and as low as L5–S1 disc. Without radiologic or sonographic imaging, the palpable landmark technique is estimated to be about 50% accurate.

22/04/19 6:24 PM

CHAPTER 21

3. C is correct. Unlike other ligaments, ligamentum flavum is largely composed of elastin fibers arranged parallel in cephalocaudal direction. When passing a needle through midline of the interspinous space, the needle traverses supraspinous ligament and interspinous ligament before it reaches ligamentum flavum. The composition difference between ligaments gives the tactile increased resistance when ligamentum flavum is encountered. Exiting ligamentum flavum also gives the tactile sensation of loss of resistance. A, B, and D are incorrect. See explanation for answer C.

Vertebra prominens C7

Root of spine of scapula T3

4. D is correct. Embryologically, ligamentum flavum is the fusion of a left and right structure at midline around 11 weeks of gestation. Ligamentum flavum inconsistently fuses at midline, leaving midline gaps. This is more common in the cervical region and less common beyond the thoracic region. Midline gaps in the cervical region have been characterized with an incidence of 50%–87%. The clinical implications of the midline gap are advancement of a needle until inadvertent dural violation and increased risk of spinal cord injury. In a study published in 2003, caudal gaps, continuous gaps, and caudal widening gaps were characterized as variations of defects observed in the ligamentum flavum (Figure 21–2).

Inferior angle of scapula T7

Rib margin 10cm from midline L1 Superior aspect of iliac crest L4

A and C are incorrect. Ligamentum flavum is also thinner at the cervical and high thoracic region when compared to the lumbar region. B is incorrect. Ligamentum flavum attaches at the base of the skull and spans the entire cervical, thoracic, and lumbar spine.

Posterior superior iliac spine S2

5. D is correct. A, B, and C are true statements regarding the epidural space.

FIGURE 21–1  Skeletal landmarks used to determine the level of neuraxial anesthesia placement.

A

95

Neuraxial Anatomy (Anatomy Relevant to Neuraxial Anesthesia)

B Vertebral arch

Vertebral arch

Ligamentum flavum

Gap Vertebral arch

Vertebral arch C

D Vertebral arch

Gap Vertebral arch

Vertebral arch

Gap Vertebral arch

FIGURE 21–2  Variations in ligamentum flavum morphology. The view from anteriorly depicts the upper and lower vertebral arches. The ligamentum flavum, which may fuse in the mid-line (A), or may feature a caudal gap for the passage of vessels (B) or a continuous gap (C), which may widen toward the caudal end (D). (Reproduced with permission from Lirk P, Kolbitsch C, Putz G, et al: Cervical and high thoracic ligamentum flavum frequently fails to fuse in the midline, Anesthesiology. 2003 Dec;99(6):1387-1390.)

Hadzic_Ch21_p091-098.indd 95

22/04/19 6:24 PM

96

PART 3

Clinical Practice of Regional Anesthesia

6. C is false, so option C is correct. Subdural injection is a relatively rare occurrence, with an estimated incidence of 0.1%-0.82% of epidural injections. A, B, and D are incorrect. All of these are true statements regarding the subdural space. 7. C is correct. Made from a diagnosis of a Cobb angle greater than 10 degrees in the coronal plane, scoliosis has been found in up to 68% of asymptomatic volunteers over the age of 60 years. Scoliosis causes the vertebral bodies to rotate toward the convexity of the curve. This, in turn, rotates the spinous processes so that they rotate toward the concavity of the curve. In other words, spinous processes that are palpable or visible are more medial (on concave side) than interlaminar space (on convex side). This makes a paramedian approach from the convex side of the curve most advantageous for neuraxial procedures. A is incorrect. While a midline approach may be used when attempting to access neuraxial spaces, it should be noted that the curvature of the spine causes rotation of the vertebrae to varying degrees, making interlaminar spaces harder to predict. B is incorrect. The Cobb angle is calculated by drawing a parallel line to the superior endplate of the vertebra above the deformity and a line drawn parallel to the inferior endplate of the vertebra one level below the deformity. As the angle increases, the rotation of the vertebral body toward the convexity of the curve increases and the compensatory rotation of the spinous process toward the concavity of the spine increases. This causes the paramedian approach from the concave side to have the smallest interlaminar space and space between spinous process and transverse process on the concave side smaller with less room for the needle. D is incorrect. Because scoliosis causes rotation and curvature of the spine, it also changes the interlaminar spaces from different angles. For this reason, examining prior imaging of the spine as well as having an ultrasound to examine the spine prior to neuraxial procedures may help determine angulation of the spine, orientation of the spinous processes and degree of their rotation, and depth of the lamina. 8. A is false, so option A is correct. C1 does not have a sensory area identified for it, although stimulation of the posterior root causes orbital and forehead pain. B is true, so option B is incorrect. Each dermatome is the cutaneous area supplied by a particular spinal nerve. With the exception of C1 each spinal nerve has a nerve rootlet contributing to a serially overlapping section of the body, creating dermatomal maps. C is true, so option C is incorrect. The dermatomal distributions and their nerve coverage are not as simple as initially described. There is a certain level of overlap noted between dermatomal sections and it is believed blocking a single nerve does not produce complete loss of sensation in a discrete dermatomal pattern. This is thought to occur because of divergence of afferent fibers upon entrance into the tract of Lissauer in the spinal cord. Every dermatome

Hadzic_Ch21_p091-098.indd 96

is innervated by fibers from not only its spinal nerve root level, but the two nerves above and below it as well. As a result, sensory dermatomes are innervated and receive contribution from up to five spinal nerves. D is true, so option D is incorrect. The dermatomal distribution for pain and temperature exceeds the dimensions of the dermatome for touch for the same nerve because the sensory fibers travel to even more levels of the spinal cord. 9. B is false, so option B is correct. While CSF is somewhat similar in composition to serum, there are some notable differences. CSF has a lower pH at about 7.32, glucose concentration is about two-thirds of the serum value, potassium tends to be lower in the CSF at 2.9 mEq/L, higher CSF concentration of pCO2 at 47 mm Hg and chloride at 125 mEq/L. Protein is also found in much lower concentrations in the CSF when compared to serum, although the protein concentration varies at different levels of the spinal cord with a higher concentration found in the lumbar CSF versus the ventricular CSF. A is true, so option A is incorrect. While about 500 mL of CSF may be produced daily, there is only about 100 mL in the spinal canal at any given time. Of that about 30–80 mL can be found below the T11–T12 region. The difference in CSF volume may affect the spread and density of subarachnoid block in different patients despite otherwise identical technique and dose of LA. C is true, so option C is incorrect. CSF is certainly dynamic and free flowing. In addition to it being constantly produced and cleared, oscillations of the CSF along with pulsations of the arteries in the skull are thought to contribute to CSF mixing. As much as 9 mm of movement within the cervical CSF and 4 mm of movement within the thoracolumbar CSF have been noted. Movements of hydrophilic drugs such as morphine cranially in CSF after intrathecal injection may be responsible for delayed respiratory depression hours after the injection. D is true, so option D is incorrect. The arachnoid granulations responsible for absorbing CSF have the ability to sense pressure and tend to maintain CSF pressure at about 10–20 cm H2O in the lateral position. (Sitting patient up for spinal block does increase the pressure in the dural sac in the lumbar area.) 10. A is correct. The pial and arachnoid meninges of the nerve root are continuous with the perineurium of the peripheral nerve, which define a fascicle. Each fascicle contains multiple axons. Therefore, an injection within the peripheral nerve fascicle may transmit solution into the subpial space of roots and then into the CSF or spinal cord. This transmission tends to be a very slow process and may take hours to reach the subpial roots. B is incorrect. The endoneurium surrounds each axon and is the innermost connective tissue covering of a peripheral nerve. C is incorrect. The epineurium is the outermost layer of the peripheral nerve and is continuous with the dura mater of the nerve root just as the pia is associated with

22/04/19 6:24 PM

CHAPTER 21

the perineurium. There are three connective tissue layers that surround the nerve. From innermost to outermost, it is endoneurium, perineurium, and epineurium. D is incorrect. The pia mater is indeed continuous with the peripheral nerve, consecutive with the perineurium. 11. D is correct. A, B, and C are true statements regarding the cauda equina nerve roots. Based on topograms created from MRI studies and cadavers, it has been shown that neighboring nerve roots share a predictable position relative to one another and along their expected trajectories throughout the dural sac. At the lumbar levels, as the cauda equine is formed, the nerve roots are somewhat mobile within the dural sac, but movement is limited by subarachnoid ligaments such as the arachnoid trabecular ligament. 12. B is correct. Along with the filum terminale, intrathecal ligaments include the denticulate ligament, the dorsal arachnoid septum, and the dorsolateral arachnoid septum. Intrathecal ligaments, also known as subarachnoid ligaments, function to anchor the spinal cord and nerve roots within the subarachnoid space. The filum terminale is an extension of the pia mater and attaches the conus medullaris to the coccyx. A is incorrect. Interspinous ligament is a relatively narrow band that attaches between spinous processes. Anteriorly, it is fused to the ligamentum flavum, while it is fused with the supraspinous ligament posteriorly. C is incorrect. The ligamentum flavum is a thick ligament that connects the lamina of adjacent lamina to one

Hadzic_Ch21_p091-098.indd 97

97

Neuraxial Anatomy (Anatomy Relevant to Neuraxial Anesthesia)

another. The increase in resistance noted when advancing a needle toward the epidural space is the resistance of the ligamentum flavum and the loss of resistance noted when the epidural space is found is the change felt upon exiting the ligamentum flavum. The ligamentum flavum is actually two halves that join in the middle and fuse together. However, there are areas of the spine where this fusion may not be complete, more often from C3–T2, although it can occur below these levels as well. The implication of this is that it is possible to not appreciate a loss of resistance at such a level upon entering the epidural space from the midline and accidentally puncturing dura. D is incorrect. The anterior and posterior longitudinal ligaments run along the anterior and posterior aspects of the vertebral bodies, respectively. They serve to help stabilize and reinforce the vertebral column, just as the other intervertebral ligaments do.

Suggested Readings Anatomy of the neuraxis. In: Cousins J, ed. Cousins and Bridenbaugh’s neural blockade in clinical anesthesia and management of pain. 4th ed. Philadelphia: Lippincott Williams & Wilkins; 2009. Hadzic A. Neuraxial anatomy (anatomy relevant to neuraxial anesthesia). In: Orebaugh SL, Eng HC, eds. Textbook of Regional Anesthesia and Acute Pain Management. 2nd ed. New York, NY: McGraw-Hill Publishing; 2017:chap 22. Reina MA, DeAndres JA, Hadzic A, Prats-Galino A, Sala-Blanch X, vanZundert AAJ, eds. Atlas of functional anatomy for regional anesthesia and pain medicine. Springer International PU; 2015.

22/04/19 6:24 PM

This page intentionally left blank

Hadzic_FM_p00i-pxx.indd 6

22/04/19 6:40 PM

22 Spinal Anesthesia André Van Zundert

QUESTIONS DIRECTIONS: Choose the one best response to each question. 1. Who was the first to describe spinal anesthesia? A. William Halsted B. James Corning C. Carl Koller D. Augustus Bier 2. Intraoperative nausea and vomiting is reduced by adding what to the spinal solution? A. Morphine B. Clonidine C. Neostigmine D. Fentanyl 3. An increased incidence of hypotension after spinal anesthesia is seen when: A. A combined spinal-epidural is used B. Prophylactic glycopyrrolate is administered C. Prophylactic ondansetron is administered D. An intensive motor blockade is avoided 4. Which statement is false regarding hearing loss after spinal anesthesia? A. It is due to loss of outer hair cell function. B. It disappears spontaneously. C. It occurs irrespective of the technique used. D. It occurs more often in the obstetric population. 5. Postoperative urinary retention following spinal anesthesia is due to all except: A. Use of opioids B. Use of vasopressors C. Use of > 1 L fluids D. Use of anticholinergic drugs 6. Shivering following spinal anesthesia: A. Does not occur as fast (within 30 minutes) as with epidural anesthesia B. Is less intense compared to epidural anesthesia C. Is due to an increase shivering threshold D. Can be treated with IV clonidine 30–150 μg

7. Pruritus following spinal anesthesia with an opioidlocal anesthetic solution is: A. Distributed around the nose and face B. Reduced by use of 5-HT3 receptor antagonists C. Less than after epidural administration D. Proportionate to the dose of the local anesthetic used 8. Postdural puncture headache (PDPH) following spinal anesthesia is caused by a slow healing process of: A. Ligamentum flavum B. Dura mater C. Arachnoid mater D. Pia mater 9. What statement is false regarding when meningitis can occur after spinal anesthesia? A. Minimum circulating E. coli count is > 500 CFU/mL B. Spinal equipment is contaminated with chemicals/ detergents C. Oral flora of the anesthesiologist is not protected with a face mask D. Patient is on Muromonab—CD3 therapy 10. Which statement is correct regarding the vascular system of the spinal cord? A. The anterior cord is relatively protected from ischemia by abundant anastomoses. B. The central area of the anterior spinal cord is less prone to ischemia. C. The superficial arterial system of the spinal cord consists of three longitudinal arteries and a pial plexus. D. The anterior cord is protected by two anterior spinal arteries. 11. Which of the following has the least chance to lead to a cauda equina syndrome? A. Use of continuous spinal microcatheters B. Use of hyperbaric 5% lidocaine for spinal anesthesia C. An operation performed in the lithotomy position D. Use of aspiration of cerebrospinal fluid (CSF) before and after local anesthetic injection

99

Hadzic_Ch22_p099-102.indd 99

22/04/19 1:48 PM

100

PART 3

Clinical Practice of Regional Anesthesia

12. Which statement is correct regarding arachnoiditis that can occur after spinal injection? A. It occurs more often after the use of short-acting than after long-acting local anesthetics. B. It occurs more often after oil-based dye vs water-based dye myelograms. C. Intrathecal corticosteroids are the treatment of choice. D. A traumatic puncture of the dura mater may result in postdural puncture headache but not in arachnoiditis. 13. Accidental total spinal anesthesia may develop after an epidural attempt at cervical level. Which statement is false? A. Before the loss of consciousness, the patient will always alert you about weakness of the upper limbs. B. Resuscitation includes cardiorespiratory support. C. Patient may show pupil dilatation, loss of consciousness, and ventilator arrest. D. Removal of cerebrospinal fluid (CSF) (10 mL aliquots) and replacement by an equal volume of NaCl 0.9% will decrease the duration of the total spinal block.

19. Which of the following is not a mechanism for uptake of local anesthetics in spinal anesthesia? A. Diffusion from cerebrospinal fluid to pia mater B. Diffusion into the central portion of the spinal cord C. Diffusion into the spaces of Virchow-Robin D. Diffusion into the vascular system 20. Which is the most important factor in determining the spread of local anesthetics in the subarachnoid space? A. Dose B. Volume C. Concentration D. Patient positioning 21. Which of the following statements is false? A. Cerebrospinal fluid (CSF) is produced in the brain at the rate of 0.35 mL/min. B. The adult volume of CSF is 150 mL. C. CSF is equally distributed to the cranium and the spinal canal. D. The adult CSF volume is constant.

14. Which statement regarding the application of spinal anesthesia is false? A. Spinal anesthesia provides excellent operating conditions for surgery below the umbilicus only. B. Spinal anesthesia is the technique of choice for cesarean delivery because of safety, reliability, and patient expectation. C. The stress response to cardiac surgery is reduced by intrathecal bupivacaine. D. Blood loss has been found to be lower with spinal compared with general anesthesia.

22. Dilution of epinephrine with local anesthetic is a potential source of drug error. Choose the correct concentration of epinephrine: A. Adding 0.1 mL of epinephrine to 10 mL of local anesthetic yields a concentration of 1:10,000. B. Adding 0.1 mL of epinephrine to 20 mL of local anesthetic yields a concentration of 1:50,000. C. Adding 1 mL of epinephrine to 10 mL of local anesthetic yields a concentration of 1:100,000. D. Adding 0.1 mL of epinephrine to 20 mL of local anesthetic yields a concentration of 1:200,000.

15. When performing spinal anesthesia using the paramedian approach, which layer of anatomy is traversed? A. Ligamentum supraspinosum B. Ligamentum interspinosum C. Ligamentum nuchae D. Ligamentum paraspinosum

23. To prevent rostral spread of the local anesthetic in spinal anesthesia, which of the following is the best option, if one intends to obtain analgesia of the perineum? A. Combine the sitting position with an isobaric solution B. Combine the Trendelenburg position with a hyperbaric solution C. Combine the jackknife position with a hypobaric solution D. Flexion of the supine patient’s hips and knees

16. Which statement is false regarding where the given anatomical structure ends in an adult patient? A. Dural sac ends at S2. B. Arachnoid mater ends at S2. C. Pia mater ends at filum terminale. D. Spinal cord ends at S2. 17. Which dermatome matches the incorrect surface anatomy? A. T10 = umbilicus B. T6 = xyphoid C. T4 = nipples D. T12 = inguinal area 18. The sensory block (dermatomal level) of spinal anesthesia is required to cover T4 to perform: A. Intestinal surgery B. Upper abdominal surgery C. Gynecological surgery D. Vaginal delivery of a fetus

Hadzic_Ch22_p099-102.indd 100

24. Which of the following options provides the best results regarding spinal anesthesia resulting in a “saddle block,” keeping the patient in that position for 5 minutes before surgery starts? A. Sitting position for low lumbar or sacral anesthesia in a lean body patient using pethidine only B. Sitting position for morbidly obese patients using 0.5% bupivacaine C. Lateral decubitus position for obstetric patient using 2% lignocaine with glucose D. Prone position for patient using 2% lignocaine with adrenaline 25. Characteristics of a subdural block are: A. Fast onset of high sensory level block with intense motor blockade B. Fast onset of high sensory level block with motor and sympathetic sparing

22/04/19 1:48 PM

CHAPTER 22

C. Fast onset compared with subarachnoid block D. Fast onset of nerve block that resolves quickly (< 1 hour) 26. Which statement is false regarding spinal anesthesia in the obstetric patient? A. Neuraxial blockade carries a higher risk of postdural puncture headache in the obstetric vs the nonobstetric population. B. Neuraxial blockade carries a higher risk of major permanent complications in the obstetric vs the nonobstetric population. C. Performing spinal anesthesia may be more difficult in the obstetric population due to the pronounced lumbar flexion and the gravid uterus. D. Aortocaval compression may worsen spinal-induced hypotension more in the obstetric population. 27. Which statement is false regarding spinal anesthesia in the obstetric patient? A. A T4-sensory block is required for a cesarean delivery. B. After spinal anesthesia, pregnant patients need to be positioned with a wedge under the right hip. C. Prior to spinal anesthesia (similar to general anesthesia), pregnant woman should receive 30 mL of 0.3 M sodium citrate to increase her stomach acidity. D. Sensory loss in the upper limb should warn the clinical regarding impending diaphragmatic paralysis.

101

Spinal Anesthesia

9. A is correct. It’s false that the minimum circulating E. coli count to develop meningitis is > 500 CFU/mL. Carp and Bailey performed lumbar puncture in bacteremic rats, and only those with a circulating Escherichia coli count greater than 50 CFU/mL at the time of lumbar puncture developed meningitis. Although meningitis after a lumbar puncture has also been described in bacteremic children; the incidence of meningitis after diagnostic lumbar puncture is not significantly different in bacteremic patients compared with spontaneous incidence of meningitis. 10. C is correct. The superficial arterial system of the spinal cord consists of three longitudinal arteries (the anterior spinal artery and two posterior spinal arteries) and a pial plexus. 11. D is correct. The use of aspiration doesn’t lead to a cauda equina syndrome; aspiration of cerebrospinal fluid before and after a local anesthetic injection has been described as a way to prevent CES after spinal anesthesia. 12. B is correct. Cauda equina syndrome occurs more often in myelograms from oil-based dyes. 13. A is correct. It is false that the patient will always alert you about weakness of the upper extremities. The patient may or may not show signs and symptoms of numbness and weakness of the upper limb.

ANSWERS AND EXPLANATIONS

14. A is correct. Spinal anesthesia not only provides good operating conditions for surgeries below the umbilicus, it can be also used in surgery above the umbilicus too (eg, laparoscopic surgery).

1. B is correct. James Leonard Corning, a neurologist in New York, in 1885 described the use of cocaine for spinal anesthesia.

15. D is correct. Spinal anesthesia using the midline approach traverses the supraspinous ligament, whereas in the paramedian approach the paraspinous muscle is traversed.

2. D is correct. Intrathecal fentanyl reduces intraoperative nausea and vomiting by improving block quality, decreasing supplemental opioids, or decreasing hypotension. 3. B is correct. Prophylactic glycopyrrolate can increase hypotension after spinal anesthesia. 4. D is correct. It’s false that hearing impairment occurs more often in the obstetric population; it is less common. 5. B is correct. The use of vasopressors is not related with urinary retention. 6. D is correct. Treatment of shivering is either IV meperidine 50 mg, tramadol 0.25–1 mg/kg, or clonidine 30–150 μg. 7. A is correct. Pruritus associated with neuraxial opioids (and IV opioids) is often distributed around the nose and face (huge vascularized area). 8. C is correct. It is the arachnoid, not the dural, damage that determines PDPH characteristics. Dural mater fiber has viscoelastic properties, in contrast to the arachnoid mater.

Hadzic_Ch22_p099-102.indd 101

16. D is correct. The spinal cord doesn’t end at S2, it ends at L3 (at birth) and L1 (in adults). Variation can be found in adults between T12 and L3. 17. D is correct. The T12 dermatome doesn’t correspond to the inguinal area; L1 is the right dermatome for this area. 18. B is correct. Upper abdominal procedures require analgesia of the surface anatomy region covered by dermatomes up to T4. 19. B is correct. Diffusion into the central portion of the spinal cord is NOT a mechanism for uptake of local anesthetics in spinal anesthesia. Only the most superficial portion of the spinal cord is affected by diffusion of local anesthetics. 20. A is correct. Dose = volume × concentration. 21. D is correct. CSF volume is not constant in adults. There is wide variability in CSF volume in adults; hence it is difficult to predict the level of spinal blockade.

22/04/19 1:48 PM

102

PART 3

Clinical Practice of Regional Anesthesia

22. D is correct. Adding 0.1 mL of epinephrine to 10 mL of local anesthetic yields a 1:100,000 concentration of epinephrine. 23. A is correct. Perineum analgesia can be obtained best with the sitting position and a hyperbaric solution; second best with an isobaric local anesthetic solution. 24. A is correct. Pethidine in the subarachnoid space results in a circumscript caudal block without hypotension. 25. B is correct. Subdural block is characterized by a high sensory level with motor and sympathetic sparing.

Hadzic_Ch22_p099-102.indd 102

26. B is correct. A neuraxial blockade doesn’t carry a higher risk of major permanent complications in obstetric patients. In this population, spinal anesthesia carries lower risks of major permanent complications compared to the nonobstetric group. 27. C is correct. 30 mL of 0.3 M sodium citrate doesn’t increase stomach acidity; this is used to decrease stomach acidity.

Suggested Reading Hadzic A. Spinal anesthesia. In: Chin A, van Zundert A, eds. Textbook of Regional Anesthesia and Acute Pain Management. 2nd ed. New York, NY: McGraw-Hill Publishing; 2017:chap 23.

22/04/19 1:48 PM

22A Mechanisms and Management of Failed Spinal Anesthesia Paul Fettes and John Rae

QUESTIONS DIRECTIONS: Choose the one best response to each question. 1. The most likely reason for lack of cerebrospinal fluid (CSF) flow despite correct needle placement in the subarachnoid space is: A. Absence of CSF (a “dry tap”) B. Low CSF pressure C. Manufacturing problems leading to a blocked needle lumen D. Obstruction of the needle lumen by clot or tissue 2. Regarding the use of ultrasound in neuraxial blockade: A. It has not been shown to improve success rates. B. Real-time scanning of needle placement is a technically simple skill. C. It can be used to aid the identification of the midline and lumbar level in patients with difficult anatomy. D. It is recommended as best practice for neuraxial block in all patients. 3. Aspiration of cerebrospinal fluid (CSF) through the spinal needle: A. Decreases the height of the resultant block B. Has been shown to decrease the failure rate of central neuraxial block C. Provides confirmation of satisfactory needle position prior to injection D. Will never affect the position of the needle within the subarachnoid space 4. Accidental subdural injection of local anesthetic solution: A. Is more common with spinal than epidural anesthesia B. Is easily identified by the operator by an obvious “give” C. May result from the dura bridging the aperture of a pencil-point needle D. Results in a dense block allowing surgery to proceed

5. Which of the following may result in a lower than desired level of neuraxial blockade in a patient with normal positioning? A. Increased lumbosacral cerebrospinal fluid (CSF) volume B. Small patient stature C. Use of amide local anesthetic drugs D. Use of hyperbaric anesthetic solution 6. Hyperbaric or “heavy” local anesthetic solutions: A. Cause less cardiovascular instability than an equal dose of isobaric solution B. Contain 8% saline to achieve a density higher than that of CSF C. Result in a slower onset, lower block than an equal dose of isobaric solution D. Spread along the concavity of the thoracic spine giving a predictable block height 7. The use of intrathecal catheters: A. Is not associated with increased infection risk B. Is part of the standard combined spinal-epidural (CSE) technique C. May result in arachnoiditis resulting from the effects of concentrated local anesthetic D. Provides a safe and effective alternative to epidural analgesia for laboring women 8. In the case of a patchy spinal block of inadequate density for surgery: A. Faulty local anesthetic is the most likely cause B. General anesthesia is likely to be required C. Repeat injection is usually the recommended solution D. Sodium channel mutations causing resistance to local anesthetics are a common cause 9. A spinal block of inadequate duration for surgery: A. Is most likely due to “faulty” local anesthetic B. Is more likely with levobupivacaine than with lidocaine C. Can rarely be appropriately managed with judicious doses of analgesia and sedation D. May result from a “syringe swap” 103

Hadzic_Ch22A_p103-106.indd 103

22/04/19 1:50 PM

104

PART 3

Clinical Practice of Regional Anesthesia

10. A spinal block of insufficient height for surgery: A. May be improved by placing the patient in reverse Trendelenburg position B. May occur if lumbar puncture was at too low a lumbar interspace C. Calls for repeating the procedure D. Can be manipulated by altering the patient’s position when using isobaric solutions 11. Regarding failed spinal anesthesia: A. Inadequate anesthesia and failed spinal anesthesia are interchangeable terms. B. Failure rates are variable but are expected to be less than 1% in experienced hands. C. Failure rates have always been reported to be less than 10%. D. Errors of judgment are not important as a cause of failure.

ANSWERS AND EXPLANATIONS 1. D is correct. Obstruction of the needle lumen by clot or tissue. Fine-bore spinal needles may become easily obstructed by a plug of tissue or clot, particularly if advanced without the stylet properly placed. In the case of any doubt, it is recommended to flush the needle or replace with fresh equipment. A is incorrect. A true “dry tap” is rare but has been reported in cases with very low CSF volumes or when the subarachnoid space is obliterated—by a compressive lesion in the cauda equina or lumbar arachnoiditis.1 B is incorrect. Low CSF pressure—for example in patients with a CSF leak following neurosurgery or accidental dural puncture during an epidural procedure—may result in very slow flow of CSF through the needle. C is incorrect. Such manufacturing issues are rare and should be easily identifiable by use of the stylet. 2. C is correct. It can be used to aid the identification of the midline and lumbar level in patients with difficult anatomy. Ultrasound has been shown to be superior to palpation of anatomical landmarks in identifying the vertebral level land midline.2 A is incorrect. Data from randomized controlled trials have shown an improvement in success rates of both epidural and spinal procedures using ultrasound guidance. B is incorrect. Real-time ultrasound-guided neuraxial blockade is an advanced technique requiring an experienced operator. D is incorrect. For this reason its use is often reserved for certain subgroups of “difficult” patients. 3. C is correct. Provides confirmation of satisfactory needle position prior to injection. The ability to easily aspirate CSF at the beginning and end of the injection provides

Hadzic_Ch22A_p103-106.indd 104

reassurance for the clinician that the needle tip is still within the subarachnoid space. A is incorrect. Aspiration of CSF should have no effect on the resultant block, provided all the contents of the syringe are returned to the subarachnoid space. B is incorrect. Provided that the needle tip is not moved, aspiration of CSF will not alter the success of central neuraxial block. D is incorrect. A disadvantage of this practice is that subtle posterior movement of the needle tip may occur on aspiration, thus dislodging it from the subarachnoid space. 4. C is correct. May result from the dura bridging the aperture of a pencil-point needle. If the pencil-point needle is not far enough into the subarachnoid space, the dura may bridge across the aperture. With the positive pressure of injection, the dura moves forward and a portion of the solution may flow into the epidural or subdural space. A is incorrect. Subdural block is more commonly recognized with epidural anesthesia but has been reported with spinal anesthesia. B is incorrect. The operator will feel the usual “give” as the tip of the pencil-point needle advances into the subarachnoid space. However as the aperture is some distance from the needle tip, the injectate may emerge outside of this space. D is incorrect. Subdural block is unpredictable and patchy and is unlikely to prove sufficient for surgery. 5. A is correct. Increased lumbosacral CSF volume. The volume of CSF within the subarachnoid space has been shown to be an important determinant of block height, with increasing volumes resulting in a lower block. Increased CSF volumes may be seen in patients with connective tissue disorders such as Marfan syndrome who develop dural ectasia. B is incorrect. Small patient stature would be expected to result in a relatively higher level of block for the same dose of local anesthetic. C is incorrect. Most commonly used local anesthetics (eg, bupivacaine, ropivacaine) are amide drugs and would have no implication on block height. D is incorrect. Use of hyperbaric solution gives a predictable block height by its spread along the thoracic spinal curvature, which can be manipulated with postural maneuvers. 6. D is correct. Spread along the concavity of the thoracic spine giving a predictable block height. Following the injection of a hyperbaric solution it travels predominantly by bulk flow under the influence of gravity, “downward” along the curvature of the thoracic spine. This predictable spread allows block height to be manipulated by postural maneuvers. A is incorrect. Plain levobupivacaine (essentially isobaric) gives a block of slower onset and less predictable height than hyperbaric bupivacaine. Its slow onset results in less cardiovascular instability than the equivalent hyperbaric dose.

22/04/19 1:50 PM

CHAPTER 22A

B is incorrect. Dextrose, not saline, is added to achieve increased density. C is incorrect. As in A, hyperbaric solutions result in a quicker onset block of more predictable height. 7. C is correct. May result in arachnoiditis resulting from the effects of concentrated local anesthetic. Local anesthetic is in itself neurotoxic at high dose. There are case reports of arachnoiditis resulting from the concentrated effect of continuously infused local anesthetic on nerve roots. A is incorrect. Leaving a catheter in the subarachnoid space is associated with increased potential for introduction of infection and meningitis. B is incorrect. The standard combined spinal-epidural (CSE) technique involves a single shot spinal and placement of a catheter into the epidural space. D is incorrect. In the case of inadvertent dural puncture during attempted epidural placement, an intrathecal catheter can be placed. This can be used for analgesia and may reduce the rate of resultant postdural puncture headache (PDPH). However great care must be taken to label the catheter as intrathecal and ensure that only the anesthesiologist administers top-up doses. In addition to the potential for the administration of inappropriately large doses of local anesthetic, there may also be an increased risk of infection, although this would be difficult to prove or quantify. 8. B is correct. General anesthesia is likely to be required. If a block is of insufficient quality, the therapeutic options are intravenous analgesia or sedation, or general anesthesia. In anything but the most minor procedures, general anesthesia will usually be required. A is incorrect. Local anesthetics are chemically and heatstable compounds. Drug failures with modern qualitycontrol procedures are rare but have been reported. If being considered, corroboration with colleagues or pharmacy may help to establish whether others have had similar issues. C is incorrect. Repeat injection carries a significant risk of producing a high block or even a “total spinal.” In

L4 L5

L3

L2

Mechanisms and Management of Failed Spinal Anesthesia

105

addition, if the block is patchy for anatomical reasons then repeat injection will not improve the situation. D is incorrect. Sodium channelopathies are rare, have not been demonstrated to cause local anesthetic resistance, and are not found in asymptomatic individuals. 9. D is correct. May result from a “syringe swap.” This refers to the unintentional injection of the wrong agent due to the presence of multiple syringes of similar size and appearance on the procedure tray. It is a well-recognized cause of failed spinal, although easily preventable with syringe labels and diligent procedure. A is incorrect. Local anesthetics are chemically and heatstable compounds. Drug failures with modern qualitycontrol procedures are rare but have been reported. If being considered, corroboration with colleagues or pharmacy may help to establish whether others have had similar issues. B is incorrect. Levobupivacaine has a longer duration of action than lidocaine and so would be unlikely to cause a block of shorter duration. C is incorrect. A patient who has a block of inadequate duration for surgery can often be managed appropriately with judicious doses of analgesia and sedation. Propofol and remifentanil infusions can be used at low plasma concentrations to good effect. 10. B is correct. May occur if lumbar puncture was at too low a lumbar interspace. If local anesthetic is injected at too low a level, hyperbaric solution may spread predominantly in a caudal direction below the lumbar lordosis, resulting in a block of lower than desired height. See Figure 22A–1. A is incorrect. Reverse Trendelenburg position involves elevating the head of the bed and thus will not improve block height. C is incorrect. Calculating a dose for any repeat injection is fraught with difficulty and carries a significant risk of producing a high block or even a “total spinal.” D is incorrect. Isobaric solutions cannot be manipulated by changes in posture. This is the main rationale for the use of hyperbaric solutions.

L1

Hadzzic Ha ic - Lancea/ NYSORA FIGURE 22A–1  Injection at the fourth interspace or lower reduces the risk of cord damage, but it may result in predominantly caudal spread of the drug and an inadequate block for surgery.

Hadzic_Ch22A_p103-106.indd 105

22/04/19 1:50 PM

106

PART 3

Clinical Practice of Regional Anesthesia

11. B is correct. Failure rates are reported as variable, but are normally less than 1% in experienced hands. A is incorrect. Inadequate anesthesia and failed spinal anesthesia are not interchangeable terms, because appropriate management of a patient with an inadequate block from a spinal anesthetic may result in a satisfactory outcome for surgery. C is incorrect. Failure rates as high as 17% have been reported. D is incorrect. Errors of judgment were found to be the main causative factor of failure in one reported series.

Hadzic_Ch22A_p103-106.indd 106

References 1. Ropper, A. and Samuels, M. Adams and Victors Principles of Neuroanatomy. 9th ed. New York, NY: McGraw-Hill; 2009. 2. Ghosh SM, Frca M, Madjdpour C, Chin KJ, Mmed M, Frcpc F. Ultrasound-guided lumbar central neuraxial block. BJA Education 2016;16:213–20.

Suggested Reading Hadzic A. Mechanisms and management of failed spinal anesthesia. In: Rae JD, Fettes PDW, eds. Textbook of Regional Anesthesia and Acute Pain Management. 2nd ed. New York, NY: McGraw-Hill Publishing; 2017:chap 23A.

22/04/19 1:50 PM

Section 2

Epidural Anesthesia Chapter 23

Hadzic_Ch23_p107-120.indd 107

Epidural Anesthesia and Analgesia  109

22/04/19 6:26 PM

This page intentionally left blank

Hadzic_FM_p00i-pxx.indd 6

22/04/19 6:40 PM

23 Epidural Anesthesia and Analgesia Cedric Van Dijck

QUESTIONS DIRECTIONS: Choose the one best response to each question. 1. Which is not a potential benefit of epidural anesthesia in patients undergoing total hip arthroplasty? A. Reduction in operative time B. Reduced deep vein thrombosis (DVT) C. Shorter hospital stay D. Less postoperative cognitive dysfunction E. Less perioperative hypotension 2. Which statement is correct regarding epidural analgesia and anesthesia for lower limb vascular surgery? A. It should not be used due to the need for perioperative anticoagulation. B. Only mixtures with epinephrine may be used. C. It should not be used due to the frequent comorbidities of peripheral arterial occlusive disease (PAOD) patients. D. It improves graft patency. E. General anesthesia (GA) is superior in most large studies. 3. A patient undergoing cystectomy requires a block up to which sensory level to attain adequate analgesia? A. T4 B. T6 C. T8 D. T10 4. Which is a potential disadvantage of epidural anesthesia in hysteroscopic procedures? A. Decubitus wounds B. Increased glycine absorption compared to general anesthesia (GA) C. Inadequate analgesia due to too low sensory block D. More postoperative mechanical voiding dysfunction 5. Which is not a potential benefit of a thoracic epidural anesthesia (TEA) for abdominal surgery? A. Decreased incidence of postoperative ileus B. Decreased duration of mechanical ventilation

C. Superior analgesia D. Increased survival E. Decreased postoperative catabolism, leading to less muscle wasting 6. In myasthenia gravis: A. Epidural is contraindicated due to prolonged muscle weakness B. Ester local anesthetics (LAs) are preferred C. Larger LA doses are often needed D. Epidural eliminates the need for perioperative neuromuscular blocking drugs (NMBDs) 7. In malignant hyperthermia: A. Both ester and amide local anesthetics (LAs) are safe B. Neither ester, nor amide LAs are safe C. Only ester LAs are considered safe D. Only amide LAs are considered safe 8. Which of the following is not a documented effect of epidural anesthesia and analgesia? A. Less breast cancer recurrence B. Less prostate cancer recurrence C. Protective role in sepsis D. All of the above are potential beneficial effects. E. None of the above is a potential beneficial effect. 9. Your thoracic surgeon asks you to assess a patient for the possibility of receiving patient-controlled epidural analgesia for a thoracoscopic lobectomy. The patient is a 68-year-old man with a known intracranial metastasis from a primary lung tumor. A CT performed in the last 24 hours shows a large intracranial space-occupying lesion without any mass effect. What is your next step to assess reasonability of performing neuraxial anesthesia? A. Perform MRI. B. Rule out hydrocephalus. C. No additional studies are necessary, neuraxial anesthesia is safe in this patient. D. No additional studies are necessary, neuraxial anesthesia is unsafe in this patient.

109

Hadzic_Ch23_p107-120.indd 109

22/04/19 6:26 PM

110

PART 3

Clinical Practice of Regional Anesthesia

10. The placement of an epidural catheter is considered safe in which of the following cases? A. 12 hours after therapeutic low molecular weight heparin (LMWH) B. 5 days after clopidogrel intake C. A patient with immune thrombocytopenic purpura (ITP) and a platelet count of 70,000 D. A patient who discontinued warfarin 5 days ago 11. The placement of an epidural catheter is considered unsafe in which of the following cases? A. Gestation thrombocytopenia with a platelet count of 70.000 B. A patient with a platelet count of 80.000 and clinical bleeding C. A patient who took his daily aspirin 80 mg until the morning of the case D. A hemophilia A asymptomatic carrier 12. Epidural catheter placement is considered unsafe in: A. Human immunodeficiency virus (HIV) primary infection B. Human immunodeficiency virus (HIV) seropositive state C. Herpes simplex virus (HSV) secondary infection

D. Varicella zoster virus (VZV) with a lesion at the desired site of epidural placement E. Erysipelas of the left lower leg 13. Epidural anesthesia and analgesia in patients with moderate to severe aortic stenosis (AS) is: A. Safe B. A better choice than single-shot spinal (SSS) anesthesia C. An absolute contraindication D. No data on the safety of epidural in AS are available. 14. The vertebra prominens is: A. always C6 B. always C7 C. C6 in one-third of the cases D. T1 in one-third of the cases 15. Following the Figure 23–1, the spine of scapula (point 1) and the inferior angle of the scapula (point 2) correspond to which vertebrae level? A. point 1: T2; point 2: T12 B. point 1: T3; point 2: T7 C. point 1: T6; point 2: T12 D. point 1: C7; point 2: T6

FIGURE 23–1  Skeletal landmarks used to determine the level of epidural placement. (Images from NYSORA Continuing Medical Education.)

Hadzic_Ch23_p107-120.indd 110

22/04/19 6:26 PM

CHAPTER 23

16. Which is true regarding anatomic landmarks in epidural placement? A. The vertebra prominens is usually either one of C6, C7, and T1. B. Anesthesiologists identify the correct level of epidural placement in only +/−30% of cases. C. Anesthesiologists puncture up to two levels higher than believed in +/−15% of cases. D. The level of epidural placement is misidentified in 60% of cases. E. All of the above F. None of the above 17. Despite interindividual variation, on average, the ligamentum flavum is: A. Thinner in pregnancy B. Thickest at the cervical level C. Thickest at the lumbar level D. Thinnest at the thoracic level 18. The ligamentum flavum consists of: A. One ligament B. Two ligaments, continuously fused C. Two ligaments, with incomplete fusion most often at lumbar level D. Two ligaments, with incomplete fusion most often at cervical level 19. The thumb corresponds to dermatome: A. C5 B. C6 C. C7 D. C8 20. The inguinal ligament corresponds to dermatome: A. T10 B. T11 C. T12 D. L1 21. Blood supply to the spinal cord: A. Is supplied by two anterior and two posterior spinal arteries B. Is drained by a venous plexus draining into one anterior and two posterior spinal veins C. Has most limited collateral circulation at the thoracic level D. Has most limited collateral circulation at the lumbar level E. Is drained by a venous plexus that is most prominent posteriorly 22. Epidural fat: A. Increases with age B. Speeds up onset of local anesthetic (LA) block C. Is organized with midline strands resulting in unequal spread and unequal block D. Can lead to false loss-of-resistance (LOR) sensation

Hadzic_Ch23_p107-120.indd 111

Epidural Anesthesia and Analgesia

111

23. Differential blockade takes place after an initial dose of local anesthetic (LA) in the epidural space in the following order: A. Motor, temperature sensation, touch sensation, pinprick sensation B. Temperature sensation, pinprick sensation, touch sensation, motor C. Temperature sensation, pinprick sensation, motor, touch sensation D. Pinprick sensation, temperature sensation, touch sensation, motor 24. Cardiovascular effects of epidural anesthesia are due to: A. Splanchnic capacitance vessels B. Sympathectomy induced by local anesthetic (LA) C. Decreased catecholamine release D. Arteriolar dilation more than venous dilation E. Sympathectomy and decreased catecholamine release 25. Hypotension in epidural anesthesia: A. Is more common in lower blocks due to more capacitance of the lower extremity veins B. Is more common in younger patients due to more elastic vasculature C. Is less common in combined general/epidural anesthesia D. Occurs more in spinal than in epidural anesthesia E. Is less common in patients with a history of cardiovascular disease due to less elastic vasculature 26. In abdominal surgery, thoracic epidural anesthesia (TEA) has beneficial effects, including: A. Increased vital capacity B. Increased tidal volume C. Increased cough strength D. Faster recuperation from paralytic ileus E. Less nausea and vomiting 27. A patient undergoes total knee arthroplasty under epidural anesthesia with mild sedation. Temperature measured at the axilla is 34°C. This is most likely due to: A. Decreased heat production B. Peripheral vasodilation C. Impaired thermoregulatory control D. All of the above 28. Which is the correct order of longest to shortest duration of action of the following local anesthetics (LAs), when injected into the epidural space? A. Lidocaine > levobupivacaine > ropivacaine > chloroprocaine B. Bupivacaine> lidocaine > chloroprocaine > ropivacaine C. Chloroprocaine> ropivacaine > bupivacaine > lidocaine D. Levobupivacaine > ropivacaine > lidocaine > chloroprocaine E. Ropivacaine > bupivacaine > chloroprocaine > lidocaine F. Levobupivacaine > ropivacaine > chloroprocaine > lidocaine

22/04/19 6:26 PM

112

PART 3

Clinical Practice of Regional Anesthesia

29. The most efficient strategy to speed up the onset of an epidural block is: A. Add sodium bicarbonate to the local anesthetic (LA) mixture of bupivacaine B. Add sodium bicarbonate to the LA mixture of chloroprocaine C. Add procaine to the mixture of bupivacaine D. Add adrenaline 1/200.000 to the mixture of LA 30. Which is a potential beneficial effect of the addition of clonidine to the local anesthetic (LA) mixture? A. Faster onset of the block B. Less nausea and vomiting C. Less paralytic ileus D. Augmentation of LA effect 31. The local anesthetic (LA) with the shortest two-segment regression is: A. Lidocaine B. 2-chloroprocaine C. Ropivacaine D. Bupivacaine 32. With advancing age: A. Epidural local anesthetic (LA) dose requirement increases B. Epidural fat increases, leading to longer blockade C. Spread of epidural injectate decreases D. More hypotension is observed when LA is administered epidurally 33. Which type of epidural catheter had the highest success rate for labor analgesia? A. Single end-port catheter B. Multiport catheter C. Wire-reinforced catheter D. Flexible-tip nylon catheter E. No significant difference has been proven. 34. Which is not a potential benefit of epidural catheter placement in the sitting position? A. It is easier to identify the midline. B. Skin-to-epidural space distance is shorter. C. Greater cephalad spread of hypobaric solutions D. Lower incidence of inadvertent epidural vein cannulation 35. Which is not a potential benefit of epidural catheter placement in the lateral decubitus position? A. Decreased incidence of epidural vein cannulation B. Attenuation of vagal reflexes C. Hemodynamic changes are better tolerated D. Shorter epidural-to-skin distance 36. Which is a symptom of pneumocephalus? A. Severe headache B. Nausea C. Convulsions D. Hemiparesis E. All of the above F. None of the above

Hadzic_Ch23_p107-120.indd 112

37. Loss of resistance to air is not associated with: A. Venous air embolism B. Pneumocephalus C. Patchy block D. Faster onset of postdural puncture headache (PDPH) E. Decreased risk of accidental dural puncture 38. Hanging drop technique: A. Cannot be performed with all needle types B. Occurs due to tenting of the dura C. Occurs due to subatmospheric pressure in the dural space D. All of the above 39. During advancement of the epidural needle, structures are crossed in the following order: A. Interspinous ligament – supraspinous ligament – flavum – epidural space B. Interspinous ligament – flavum – epidural space – supraspinous ligament C. Supraspinous ligament – interspinous ligament – flavum – epidural space D. Supraspinous ligament – flavum – epidural space – interspinous ligament 40. During paramedian approach of the epidural space, structures are met in the following order: A. Supraspinous ligament – flavum – epidural space – interspinous ligament B. Supraspinous ligament – interspinous ligament – flavum – epidural space C. Paraspinous tissue – flavum – epidural space D. Paraspinous tissue – lateral epidural ligament – flavum – epidural space 41. During caudal approach to epidural blockade, structures are encountered in the following order: A. Sacrococcygeal ligament – sacral canal – epidural space B. Ischiococcygeal ligament – sacral canal – epidural space C. Sacrococcygeal ligament – ischiococcygeal ligament – sacral canal – epidural space D. Sacrococcygeal ligament – iliococcygeal ligament – sacral canal – epidural space 42. An epidural test dose is considered positive for intravascular placement if: A. An increase in heart rate is observed within 1 minute. B. The change of heart rate is 20% in either direction. C. The heart rate does not change. D. An increase in heart rate of 10–25 bpm is observed within 1 minute E. An increase of BP of 20% within 15 minutes is observed. 43. An epidural test dose is considered positive for intrathecal placement if: A. Significant motor block occurs within 5 minutes. B. Hypotension occurs within 5 minutes. C. Paresthesia occurs within 5 minutes. D. All of the above

22/04/19 6:26 PM

CHAPTER 23

44. The addition of sodium bicarbonate to a local anesthesia (LA) mixture containing ______ increases block duration. A. Mepivacaine B. Ropivacaine C. Bupivacaine D. Levobupivacaine E. All of the above 45. A patient in her second stage of labor for a first pregnancy is still experiencing pain. The most efficient strategy to relieve this patient’s pain is: A. Top up dose with high dose/high volume local anesthetic (LA) mixture B. Top up dose with high dose/low volume LA mixture C. Top up dose with low dose/high volume LA mixture D. Have the patient sit upright to block lower segments E. None of the above 46. In case of difficult catheter threading, it is advisable to: A. Withdraw needle and catheter in all cases B. Infuse 10 mL of saline and retry C. Have the patient take a deep breath D. Withdraw the catheter and reinsert and remove the stylet and retry 47. Which of the following increases the risk of a bloody tap? A. Placing the catheter in lateral decubitus B. Infusing 10 mL of saline before catheter threading C. Midline approach D. Threading the catheter > 5 cm into the epidural space E. Using a wire-reinforced catheter 48. A 32-year-old primigravid patient received an epidural at L3–L4 for labor analgesia. In the last 7 hours, she received three top-up doses for inadequate pain relief. Currently, she experiences a patchy block in both legs and abdomen. The best strategy is to: A. Replace the catheter with another one at a different interspace B. Administer a top-up dose with clonidine 75 μg C. Withdraw the catheter 2 cm D. Administer a top-up dose of 20 mL lidocaine 2% E. Administer a top-up dose of 10 mL saline 49. When encountering difficulties removing an epidural catheter, it is best to: A. Forcefully remove the catheter B. Insert an epidural needle as a guide over the catheter to remove it C. Consult neurosurgery for surgical removal D. Remove the catheter in lateral decubitus 50. When a patient exhibits symptoms of severe local anesthetic systemic toxicity (LAST), the first step is to: A. Proceed to intubation and sedation B. Administer large doses of propofol C. Start lipid emulsion therapy

Hadzic_Ch23_p107-120.indd 113

Epidural Anesthesia and Analgesia

113

D. Administer benzodiazepines to prevent or treat seizures E. Call for help F. Alert the nearest cardiopulmonary bypass facility 51. A 65-year-old ASA status 2 male patient comes in for a preoperative consultation. He is scheduled for a total knee replacement in 2 weeks time. He reports an allergic reaction at the dentist’s office during a root canal treatment 1 year ago. Which statement is true? A. Cross-reactivity between the local anesthetic (LA) from the dentist and the LA you plan to use is likely. B. You should order serum tryptase to confirm the allergic status. C. It was most likely a true allergic reaction. D. You should draw blood and order an IgE panel. E. All of the above F. None of the above 52. The best study to confirm a diagnosis of chemical arachnoiditis postepidural anesthesia is: A. White count and C-reactive protein (CRP) in standard blood analysis B. Lumbar puncture C. Plain lumbar x-ray D. CT of the spine E. MRI of the spine 53. Which is an appropriate strategy to avoid back pain postepidural placement? A. Bed rest after placement and removal of the catheter B. Epidural administration of nonsteroidal antiinflammatory drugs (NSAIDs) C. Epidural administration of dexamethasone D. Epidural administration of chloroprocaine 54. Which is not a characteristic of postdural puncture headache (PDPH)? A. Positional headache B. Neck pain C. Neck stiffness D. Tinnitus E. Diplopia F. Loss of consciousness (LOC) 55. Which patient is at elevated risk of postdural puncture headache (PDPH)? A. A patient undergoing a single-shot spinal with a 27 Gauge Sprotte needle B. An 85-year-old man C. A man with a BMI of 42 D. A laboring woman 56. Epidural blood patch (EBP): A. Offers relief in > 90% of cases B. Relief may be transient C. Decreases duration of hospital stay D. All of the above are true. E. None of the above is true.

22/04/19 6:26 PM

114

PART 3

Clinical Practice of Regional Anesthesia

57. Spinal epidural abscess (SEA): A. Occurs almost exclusively in patients with predisposing factors B. Occurs due to direct inoculation of bacteria on the epidural needle C. Neurological sequelae are rare D. Occurs exclusively in patients who underwent epidural instrumentation 58. The ER physician consults you for a 28-year-old patient who gave birth 2 days ago and received epidural analgesia for labor. The patient presents with fever, stiff neck, headache, confusion, and back pain. The most likely diagnosis is: A. Postdural puncture headache (PDPH) B. Meningitis C. Spinal epidural abscess (SEA) D. Uterine infection E. Transient fever associated with labor analgesia 59. The most common side effect or complication of epidural analgesia is: A. Epidural hematoma (EH) B. Nerve injury C. Total spinal anesthesia (TSA) D. Postdural puncture headache (PDPH) E. All of the above occur equally commonly. 60. When an epidural hematoma is suspected: A. The diagnosis can be made only after residual block wears off. B. Urgent MRI confirms or rules out the diagnosis. C. Intervention is indicated only in cases of bowel/bladder dysfunction. D. Surgical intervention must take place within 8 hours. 61. An effective treatment for a patient with pruritus caused by epidural opioids is: A. Promethazine 25 mg B. Cetirizine 5 mg C. Droperidol 0.625 mg D. Nalbuphine 5 mg

ANSWERS AND EXPLANATIONS 1. E is correct. Hypotension is a common side effect of epidural anesthesia due to sympathectomy. A, B, C, and D are incorrect. Studies have proven turnover and operative time is faster when the patient receives neuraxial anesthesia (option A). Other benefits of epidural anesthesia for total hip arthroplasty include a lower incidence of postoperative DVT (option B), a shorter hospital stay (option C), and less postoperative cognitive dysfunction (option D). 2. D is correct. Evidence suggests epidural anesthesia promotes graft patency.

Hadzic_Ch23_p107-120.indd 114

A is incorrect. Perioperative anticoagulation is needed in many vascular procedures; however, the perioperative dosing regimen usually does not augment the risk of epidural hematoma. B is incorrect. Epinephrine is used to prolong the block, uptake occurs but is unlikely to result in clinical significant vasopressive or inotropic effect. C is incorrect. Avoiding GA is beneficial in the majority of patients with comorbidities such as coronary artery disease. E is incorrect. Conflicting evidence exists on this matter; however, most studies suggest a better outcome with neuraxial anesthesia. 3. A is correct. T4. The bladder is innervated by T9–T10, but the incision and traction on the viscera requires a sensory level block until level T4. 4. B is correct. There appears to be increased glycine absorption from irrigation fluids in epidural anesthesia vs GA. A is incorrect. Positioning is the same for the procedure regardless of anesthetic technique. Incidence of decubitus is lower in awake patients, as only part of the body is anesthetized. C is incorrect. This is unlikely if the epidural is placed at the appropriate interspace. D is incorrect. An awake patient can cooperate in the perioperative cough test, theoretically resulting in a lower incidence of postoperative mechanical voiding dysfunction. Studies have not confirmed this effect so far. 5. D is correct. Increased survival is not a potential benefit of thoracic epidural anesthesia (TEA) for abdominal surgery. Evidence for increased survival has been only documented in multiple rib fractures. A, B, C, and E are incorrect. Studies have found potential benefits of TEA for abdominal surgery to be a lower incidence of postoperative paralytic ileus, decreased duration of mechanical ventilation, superior analgesia, and less pronounced muscle wasting. 6. D is correct. When placed at the correct interspace and with appropriately dosed LAs, motor block is achieved in the surgical field, eliminating the need for NMBDs, avoiding problems with residual neuromuscular block upon conclusion of the procedure. A is incorrect. Epidural anesthesia is preferred due to the diminished needs for perioperative opioids and sedatives, decreasing the chance of drug-induced respiratory depression. B is incorrect. Ester metabolism may be prolonged in patients on cholinesterase inhibitors, making amide LAs the preferred choice. C is incorrect. Reduced doses of LAs are often appropriate in patients with myasthenia gravis. 7. A is correct. Blocks are recommended and both amide and ester LAs are considered safe.

22/04/19 6:26 PM

CHAPTER 23

8. D is correct. Recent evidence suggests an association between epidural analgesia and cancer recurrence, likely due to less humoral stress response and decreased opioid requirements during the operation. A protective role in sepsis has also been postulated. 9. B is correct. Hydrocephalus must be ruled out if a proven intracranial mass exists. A is incorrect. MRI does not provide additional information on intracranial pressure when a recent CT is available. C is incorrect. Neuraxial anesthesia can only be considered if the risk of herniation with inadvertent dural puncture is low compared to the potential benefits. D is incorrect. Neuraxial anesthesia may be reasonable in some cases. 10. C is correct. ITP does not affect platelet function, and 70,000 is an acceptable threshold to place an epidural catheter. A is incorrect. Therapeutic LMWH must be stopped for 24 hours before epidural catheter insertion. B is incorrect. Clopidogrel must be stopped 7 days prior to epidural catheter insertion. D is incorrect. Warfarin must be stopped 10 days prior to epidural catheter insertion, and a safe INR must be documented before puncture. 11. B is correct. Clinical bleeding with low platelet count should raise concern for qualitative and/or quantitative platelet disorder. Epidural instrumentation may be associated with an elevated risk of epidural hematoma in such cases. A is incorrect. Platelet function is normal in gestational thrombocytopenia and 70.000 is considered sufficient for epidural catheter insertion. C is incorrect. Aspirin 80 mg/day is not a contraindication for epidural catheter insertion. D is incorrect. Asymptomatic carriers are not at elevated risk of epidural hematoma after epidural catheter insertion. 12. D is correct. VZV with an active infection at the desired site of epidural placement is considered a contraindication, as it may lead to inoculation of the virus in the epidural space, resulting in hematogenic spread of the virus and subsequent complications. A is incorrect. The central nervous system is infected early in the course of an HIV infection, and there is no evidence that neuraxial anesthesia contributes to the development of neurological symptoms of an HIV infection. B is incorrect. Neurologic complications of HIV are not uncommon, but there is no evidence neuraxial anesthesia causes these. It is advisable to always document existing neurologic deficits prior to the procedure. C is incorrect. There are no documented cases of septic or neurologic complications following neuraxial procedures in patients with secondary HSV infection.

Hadzic_Ch23_p107-120.indd 115

Epidural Anesthesia and Analgesia

115

E is incorrect. Localized skin infection at the site of epidural catheter insertion is a relative contraindication to the procedure; however, an infection at a site away from the puncture site is not a contraindication. 13. B is correct. Gradual onset of sympathectomy and hypotension is impossible in SSS. Therefore, epidural is a better choice provided that local anesthetic (LA) is titrated carefully in incremental doses. A is incorrect. Epidural anesthesia may result in hypotension. A decreased afterload can critically diminish coronary artery perfusion in aortic stenosis, leading to cardiac ischemia. C is incorrect. Epidural anesthesia may be considered in AS on a case-to-case basis. D is incorrect. Data suggest an elevated risk of complications; however, a risk-benefit analysis must be made in all cases. 14. D is correct. The spinous process of T1 may be equally or more prominent than C7 in one-third of patients. A and B are incorrect. The most prominent spinous process may be C6, C7, or T1. C is incorrect. C6 may be equally prominent to C7 in up to half of the cases. 15. B is correct. Most commonly, the base of the spine of the scapula corresponds to T3 and the inferior angle to T7. A, C, and D are incorrect. T6 is higher than the point of the scapula, which corresponds to T7. T12 can be identified as the vertebra that articulates with the lowest rib. 16. E is correct. All statements are true. A is true, so Answer E (all of the above) is correct. The most prominent spinous process can be either C6 or T1 in half of the cases. B is true, so Answer E (all of the above) is correct. Broadbent et al recently documented the interspace identified by the anesthesiologist corresponds to the correct level in only a third of cases. C is true, so Answer E (all of the above) is correct. Lirk et al found trained anesthesiologists to place the epidural needle more cranially than intended. Up to 15% placed the epidural needle two levels higher than intended. D is true, so Answer E (all of the above) is correct. Van Gessel et al found the level of puncture is misidentified in almost two-thirds of cases. F is incorrect. 17. C is correct. See Table 23–1. Flavum measures 5-6 mm on average in the lumbar region, A is incorrect. Flavum is thicker in pregnancy, presumably due to edema. It can be up to 10 mm thick in the lumbar region. B is incorrect. It is thinnest at the cervical level, averaging 1.5–3 mm. D is incorrect. Flavum measures an average of 3–5 mm thick in the thoracic region.

22/04/19 6:26 PM

116

PART 3

Clinical Practice of Regional Anesthesia

TABLE 23–1  Thickness of the ligamentum flavum at different vertebral levels. Vertebral Level

Thickness (mm)

Cervical

1.5–3.0

Thoracic

3.0–5.0

Lumbar

5.0–6.0

Caudal

2.0–6.0

18. D is correct. Flavum is built up by two ligaments, which fuse incompletely at the midline. Midline gaps in the ligamentum flavum are most prevalent at the cervical level. See Table 23–2.

TABLE 23–2  Surface landmark correlation to dermatomal level. Level of Blockade

Anatomic Landmark

C6

Thumb

C8

Fifth finger

T1

Inner aspect of arm

T4

Nipple

T6

Xiphoid process

T10

Umbilicus

T12

Inguinal ligament

S1

Lateral aspect of foot

S2–S4

Perineum

19. B is correct. The thumb is innervated by fibers of the root C6. A is incorrect. C5 innervates the lateral part of the upper arm. C is incorrect. C7 innervates the middle, or third finger, and occasionally parts of the 2nd and 4th finger. D is incorrect. C8 innervates the fifth finger, and a variable portion of the 4th finger. 20. C is correct. The inguinal ligament is innervated by fibers of the root T12. A is incorrect. T10 innervates the dermatome that includes the umbilicus. B is incorrect. T11 innervates the dermatome that sits in between the umbilicus and the inguinal ligament. D is incorrect. L1 innervates the portion of the upper thigh that sits distal to the inguinal ligament. 21. C is correct. Collateral circulation is most limited at the thoracic level, making it more prone to ischemic injury during vascular procedures. A is incorrect. The spinal cord is supplied by one anterior and two posterior spinal arteries.

Hadzic_Ch23_p107-120.indd 116

B is incorrect. The venous plexus drains into two anterior and two posterior epidural veins. D is incorrect. Collateral circulation is most limited at the thoracic level. E is incorrect. The epidural venous plexus is more prominent anteriorly. 22. D is correct. Epidural fat can lead to false LOR sensation when it sits in clefts of incompletely fused flavum. A is incorrect. Epidural fat decreases with age, which is one of the main causative factors of decreased dose requirements with age. B is incorrect. Epidural fat takes up LA, slowing onset of LA block. C is incorrect. Midline strands are seen in the midline epidural fat pad. They have no significant effect on spread of LA. 23. B is correct. Sensory block occurs at lower LA concentration than motor block, because motor fibers are thicker and better insulated by myelin sheaths. Sensory functions are blocked in the following order: temperature first, followed by pinprick, and finally touch. Sympathetic nerve fibers are blocked at different rates and to a different extent. 24. E is correct. Epidural block of autonomous nerve fibers leads to direct inhibition of sympathetic outflow as well as decreased catecholamine release from the adrenal glands. This effect depends on LA dose and level of epidural block. A is incorrect. Splanchnic venous dilation is caused by inhibition of sympathetic nerve fibers and is a contributing factor to hypotension, but is not the sole mechanism. B is incorrect. Sympathectomy is one of two predominant mechanisms of hypotension. C is incorrect. Decreased catecholamine release is one of two predominant mechanisms of hypotension. D is incorrect. Due to the amount of blood in the venous system, venodilation contributes to a greater extent to hypotension after epidural block. 25. D is correct. Spinal anesthesia does not offer the potential benefit of slowly incrementing the dose of local anesthetic (LA). Onset of anesthesia and autonomous fiber block is sudden and deep, leading to more profound and less controllable hypotension. A is incorrect. Hypotension is more common in thoracic and cervical epidural blockade, due to anesthetic effect of vasomotor fibers. B is incorrect. Hypotension is more common in elderly patients. C is incorrect. Hypotension is more common when general anesthesia is combined with epidural anesthesia, due to the additional effect of hypnotic drugs on vasomotor outflow. E is incorrect. Hypotension is more common in patients with a history of cardiovascular disease such as hypertension.

22/04/19 6:26 PM

CHAPTER 23

26. D is correct. TEA is associated with a faster recovery from paralytic ileus, due to less need for opioid analgesics and blocking of autonomous nerve fibers. This leads to unopposed vagal tone, increasing motility of the gastrointestinal tract. A is incorrect. Vital capacity may be reduced due to the decrease in expiratory reserve volume that occurs as accessory muscles involved in expiration are blocked. B is incorrect. No change in tidal volume is observed with TEA. C is incorrect. Cough strength is decreased in TEA. Studies have proven this does not adversely affect outcome. E is incorrect. TEA can cause nausea and vomiting due to opioid absorption from the local anesthetic (LA) mixture as well as by inducing hypotension. 27. D is correct. Blocking of autonomous nerve fibers leads to peripheral vasodilation in the blocked dermatomes, increasing heat loss. Afferent fibers are blocked as well, leading to impaired central thermoregulation. Heat production is also decreased due to decreased metabolism.

Epidural Anesthesia and Analgesia

117

C is incorrect. Ropivacaine has a two-segment regression of 180–260 minutes. D is incorrect. Bupivacaine has a two-segment regression of 180–260 minutes. 32. D is correct. With advancing age, requirement for LA for epidural blockade decreases, most likely due to decreased epidural fat, leading to increased epidural space and less uptake of LA by adipose cells, eventually increasing LA effect. A is incorrect. LA dose requirement decreases with age. B is incorrect. Epidural fat decreases with age, which most contributes to the diminishing LA requirements with advancing age. C is incorrect. With advancing age, the neuroforamina become narrower and the compliance of the epidural space changes, allowing LA to spread more. 33. E is correct. Data are inconclusive and recent studies have failed to demonstrate a significant difference in efficacy in catheter types.

28. D is correct. Levobupivacaine has an average duration of 150–225 minutes to recovery from sensory block, ropivacaine 140–220 minutes, lidocaine 60–120 minutes, and chloroprocaine an average duration of blockade of 30–60 minutes. Other options: Bupivacaine has an average duration of 160–220 minutes and has therefore similar duration of action to ropivacaine.

34. D is correct. Lower incidence of inadvertent epidural vein cannulation is not a potential benefit of epidural catheter placement in the sitting position. This is a benefit of cannulation in the lateral decubitus position. A, B, and C are incorrect. These options are potential benefits of performing neuraxial procedures in the sitting position.

29. B is correct. The addition of sodium bicarbonate to the LA mixture of chloroprocaine makes more nonionized or lipid form of LA molecules available for rapid uptake into the axon, where it can exert its action. A is incorrect. Ropivacaine and bupivacaine precipitate with the addition of sodium bicarbonate, making the mixture inactive. C is incorrect. The addition of chloroprocaine to a bupivacaine LA mixture shortens duration and effectiveness of bupivacaine. D is incorrect. Adrenaline contributes to a longer lasting effect, but not to a faster onset.

35. D is correct. Shorter epidural-to-skin distance is a potential benefit of cannulation in the sitting position. A, B, and C are incorrect answers because these options are potential benefits of performing neuraxial procedures in the sitting position.

30. D is correct. Clonidine potentiates the effect and duration of LAs without increasing the risk of hypotension. It also works synergistically with opioids in the LA mixture. A is incorrect. Clonidine does not speed up onset of epidural blockade; it does prolong the block. B is incorrect. This is not a known effect of epidural clonidine. C is incorrect. This is an effect of all thoracic epidurals for abdominal surgery. 31. B is correct. 2-Chloroprocaine has a two-segment regression of 45–75 minutes. A is incorrect. Lidocaine has a two-segment regression of 60–140 minutes.

Hadzic_Ch23_p107-120.indd 117

36. E is correct. All are symptoms of significant air entry into the subarachnoid space. 37. E is correct. Loss of resistance to air (LOR) is not associated with a decreased risk of accidental dural puncture. Air is compressible and may result in a false LOR and, relatedly, an increased incidence of accidental dural puncture. A, B, C, and D are incorrect. These options are associated with loss of resistance to air. 38. A, B, and C are all true, so Answer D (all of the above) is the correct choice. This technique requires a needle with wings. Subatmospheric pressure and tenting of the dura leading to further negativation of dural pressure are responsible for the retraction of the drop into the epidural needle. 39. C is correct. The supraspinous ligament connects the ends of the spinous processes, the interspinous ligament sits in between the spinous processes, and flavum delineates the posterior aspect of the epidural space. A, B, and D incorrect. These options represent an incorrect order of crossing of the spinous ligaments.

22/04/19 6:26 PM

118

PART 3

Clinical Practice of Regional Anesthesia

40. C is correct. After puncture of the skin, the needle is advanced through paraspinous tissue, consisting of muscle and fibrous tissue, to penetrate the ligamentum flavum, entering the epidural space. A is incorrect. This is an incorrect order. B is incorrect. This is the correct order during a midline approach. D is incorrect. There is no lateral epidural ligament in human anatomy. 41. A is correct. After puncture of the skin, the needle is advanced through the sacrococcygeal ligament at the midline, entering the sacral canal and subsequently the epidural space within the canal. B, C, and D are incorrect. Ischiococcygeal and iliococcygeal ligaments are not midline structures and should not be encountered in the caudal approach. 42. D is correct. Test dose is considered positive for intravascular placement if an increase in heart rate occurs of 10–25 bpm or at least 20% of baseline, within 1 minute. Limitations to this exist (eg, a laboring patient experiencing a contraction) so anesthesiologists administering test dose should be aware the test dose is not failsafe. A is incorrect. Not every increase in heart rate should be regarded positive. Only an increase in heart rate greater than 10-25 bpm, or more than 20% from baseline should be regarded as a positive result. B is incorrect. An decrease in heart rate cannot be achieved with an intravascular test dose. C is incorrect. No change in heart rate is regarded as negative for intravascular catheter placement. E is incorrect. Changes in blood pressure are not routinely used to judge the effect of the test dose. An increase in blood pressure greater than 20 mmHg has been proposed to diagnose inadvertent intravascular catheter placement in patients on beta adrenergic blocking drug therapy, as increases in heart rate may not be observed in this population. 43. A is correct. Only significant motor block after the correct test dose of 45 mg lidocaine (plus adrenaline to rule out intravascular placement) is considered diagnostic for intrathecal placement of the catheter. B is incorrect. Hypotension may follow intrathecal administration of the test dose, but only significant motor block is considered a positive result. C is incorrect. Paresthesia may follow intrathecal administration of the test dose, but only significant motor block, not sensory block alone, is considered a positive result. D is incorrect. Hypotension and paresthesia are not considered positive results for the test dose. 44. A is correct. Addition of sodium bicarbonate to mepivacaine, lidocaine, and chloroprocaine mixtures speeds up onset, prolongs duration of action, and improves the quality of the block.

Hadzic_Ch23_p107-120.indd 118

B, C, D, and E are incorrect. Addition of sodium bicarbonate to bupivacaine and ropivacaine mixtures leads to precipitation and unsuccessful block. 45. B is correct. When a denser block is required (eg, for surgical delivery or for second stage of labor), the addition of a higher dosed, but lower volume LA mixture is most effective. A and C are incorrect (see above). D is incorrect. This is not an appropriate technique to relieve pain. E is incorrect. 46. D is correct. Reinsertion of the stylet may clear debris from the needle, facilitating catheter insertion. A is incorrect. Care must be taken to never withdraw a catheter through the needle when resistance is felt. However, if the catheter is easily removed, the needle may stay in the epidural position. B is incorrect. Large volumes of saline may interfere with quality of the block. C is incorrect. Breathing does not significantly affect catheter placement. 47. D is correct. Threading the catheter deep into the epidural space increases the risk of inadvertent venous cannulation. A, B, C, and E are incorrect. All of the other strategies (A, B, C, E) decrease the risk of a bloody tap. 48. A is correct. When multiple top-up doses are insufficient, it is best to replace the catheter. B is incorrect. Clonidine should have been considered at an earlier point. After multiple top-ups, it is best to replace the catheter. C is incorrect. Catheter withdrawal should have been considered at an earlier point. After multiple top-ups, it is best to replace the catheter. D is incorrect. This is a high-dose, high-volume, top-up dose, which will likely enhance side effects such as hypotension, leading to decreased placental perfusion. E is incorrect. Epidural volume expansion is successful in combined spinal-epidural (CSE), after administration of the subarachnoid component. In epidural it will likely result in decreased block quality. 49. D is correct. Lateral decubitus will allow most patients to flex the spine more, resulting in a larger intervertebral space and easier catheter removal. A is incorrect. The catheter should never be removed with force, as it is fragile and could break. B is incorrect. Inserting an epidural needle over the catheter as a guide could result in transection of the catheter or infection. C is incorrect. Surgical removal is rarely indicated. Even in the case of retained fragments, removal is rarely necessary.

22/04/19 6:26 PM

CHAPTER 23

50. E is correct. It is always the first step in any resuscitation setting to call in additional help, no matter the skills of the first responder. A is incorrect. This is only indicated in the case of compromised airway or inadequate protective airway reflexes. B is incorrect. Propofol is dissolved in lipid emulsion; however, toxic doses would have to be administered to achieve the correct dose of lipid emulsion. C, D, and F are incorrect. These are all steps in the treatment algorithm, but they are not the primary step. 51. F is correct. A is incorrect. Cross-reaction between esters and amides is unlikely, and cross-reaction within drug classes is not 100%. B is incorrect. Serum tryptase is elevated in the first 24 hours after an allergic reaction, as a marker of mast cell degranulation. It is not diagnostic for allergy after 1 year. C is incorrect. Most “allergic reactions” described by patients are side effects, aspecific symptoms, or effects of adrenaline in the mixture. It is always necessary to characterize the complaint to determine the likelihood of a true allergic reaction. D is incorrect. An IgE panel will test only for common household allergens, not for LAs. E is incorrect. 52. E is correct. MRI is the most likely to confirm the diagnosis. A negative MRI does not always rule out chemical arachnoiditis. A, B, C, and D are incorrect. A CT or an LP (option B) may provide useful information but are less likely to confirm diagnosis. Plain x-ray (option C) and lab (option A) are not likely to contribute to the diagnostic process. Bloodwork may be useful when other diagnoses must be ruled out (eg, infectious diseases). 53. C is correct. Evidence suggests that epidural administration of dexamethasone results in a lower incidence of back pain after epidural instrumentation for anesthesia and analgesia. A is incorrect. Bed rest increases the risk of back pain. B is incorrect. NSAIDs may reduce the risk of back pain when administered with the LA mixture used for skin numbing during the procedure. D is incorrect. The incidence is not related to type of LA. However, some chloroprocaine mixtures may contain preservatives that increase the risk of back pain. 54. F is correct. LOC is not a characteristic of PDPH and should prompt further investigation to rule out more serious pathology, such as infectious meningitis. A, B, C, D, and E are incorrect. All these answers are possible symptoms of PDPH.

Hadzic_Ch23_p107-120.indd 119

Epidural Anesthesia and Analgesia

119

55. D is correct. Epidural analgesia for labor has a higher incidence of PDPH. A is incorrect. The use of a cutting needle is associated with a higher risk of PDPH. The Sprotte needle, however, is a pencil point needle, associated with a lower incidence of PDPH. Moreover, a smaller gauge needle, such as the 27 Gauge Sprotte needle, is associated with a lower risk of PDPH. B is incorrect. PDPH is rarely observed in the elderly. Younger age, however, is a risk factor for PDPH. C is incorrect. A low Body Mass Index is a risk factor for PDPH, a high Body Mass Index is not. 56. D is correct. EBP offers relief in the majority of cases, with studies reporting success rates of 90% and above. A, B, and C are true, so Answer D (All of the above are true.) is the correct choice. Relief may be transient in some cases. Studies report a shorter hospital stay and fewer ER visits. E is incorrect. 57. A is correct. SEA occurs almost exclusively in patients with predisposing factors such as immunocompromised status or diabetes. B is incorrect. Hematogenous inoculation is most likely in SEA. C is incorrect. Neurological sequelae are common, as 33%–47% of patients suffer long-term morbidity. D is incorrect. An estimated 5% of SEAs occur after epidural catheter placement. 58. B is correct. Confusion and stiff neck are not symptoms of the other listed diagnoses. Stiff neck and neurological symptoms should always arouse suspicion of meningitis, especially after neuraxial procedures. A is incorrect. PDPH presents as positional headache, commonly 1–2 days after neuraxial techniques or lumbar puncture. C is incorrect. SEA does not cause stiff neck. D is incorrect. Uterine infection should always be considered postpartum, but does not present with stiff neck. E is incorrect. This occurs in the first hours after catheter placement, not after 2 days, and is not associated with neurological symptoms or stiff neck. 59. D is correct. PDPH occurs approximately in 80% of cases where the dura was unintentionally punctured. Inadvertent dural puncture occurs in approximately 1% of cases. A is incorrect. The incidence of EH is estimated at 1:150,000. B is incorrect. The incidence of nerve injury is estimated at 1:1000. C is incorrect. TSA occurs in 1:1400. E is incorrect.

22/04/19 6:26 PM

120

PART 3

Clinical Practice of Regional Anesthesia

60. D is correct. Surgical intervention must occur within 8 hours of onset of symptoms to avoid permanent neural damage. A is incorrect. Waiting until block wears off may delay diagnosis and possibly results in permanent neural damage. B is incorrect. Urgent MRI confirms the diagnosis in the majority of cases, but an inexperienced radiologist may miss the diagnosis. C is incorrect. Surgical intervention must not be delayed when the diagnosis is confirmed. Intervention is indicated in all cases.

Hadzic_Ch23_p107-120.indd 120

61. D is correct. Nalbuphine 5 mg is the most effective treatment. A second dose of 5 mg may be administered if the first dose is unsuccessful. A and B are incorrect. Antihistamines are ineffective against opioid-induced pruritus. C is incorrect. Droperidol has antihistamine properties, but is not effective in the treatment of opioid-induced pruritus.

Suggested Reading Hadzic A. Epidural anesthesia and analgesia. In: Toledano RD, Van de Velde M, eds. Textbook of Regional Anesthesia and Acute Pain Management. 2nd ed. New York, NY: McGraw-Hill Publishing; 2017:chap 24.

22/04/19 6:26 PM

Section 3

Caudal Anesthesia Chapter 24

Hadzic_Ch24_p121-126.indd 121

Caudal Anesthesia  123

4/18/19 6:07 PM

This page intentionally left blank

Hadzic_FM_p00i-pxx.indd 6

22/04/19 6:40 PM

24 Caudal Anesthesia Antony R. Tharian, Nebojsa Nick Knezevic, and Kenneth D. Candido

QUESTIONS DIRECTIONS: Choose the one best response to each question. 1. Which of the following anatomic structures is not part of the sacral canal? A. Filum terminale B. Conus medullaris C. Spinal meninges D. Cauda equine 2. Which of the following statements is true with regards to caudal anesthesia? A. Success rates are higher in adults when compared to children. B. It is easier to access the caudal canal of men when compared to women. C. The cephalad flow of local anesthetic (LA) is less limited in lordotic patients with exaggerated lumbosacral angles, when compared to patients with more flattened lumbosacral angles. D. LA injected into the caudal canal can easily egress through the anterior sacral foramina.

using 1 mL/kg of 0.15% ropivacaine in a 3-year-old child weighing 14 kg? A. Relaxation of the anal sphincter B. Vasodilation of the lower extremities C. Urinary retention D. Sensory blockade of L5 dermatome 6. Which of the following local anesthetics (LAs) used in caudal epidural block provides the longest duration of analgesia in adults? A. 300 mg of lidocaine 2% B. 400 mg of 2% chloroprocaine C. 150 mg of 1% ropivacaine D. 100 mg of 0.5% bupivacaine 7. Following caudal block in children undergoing genitourinary surgery, the serum levels of which of the following hormones is not lowered? A. Thyroxine (T4) B. Prolactin C. Insulin D. Cortisol

3. Which of the following conditions is not an indication for a caudal epidural block? A. Postdural puncture headache (PDPH) B. Lumbar radiculopathy C. Postherpetic neuralgia affecting the T8 dermatome D. Lumbar postlaminectomy pain syndrome

8. Which of the following findings is the earliest indicator of intravascular injection following the administration of an epinephrine-containing local anesthetic (LA) test dose while performing caudal epidural block in a pediatric patient? A. Elevation in heart rate by 10 beats/minute B. Increase in systolic blood pressure by 15 mm Hg C. T-wave changes in the ECG D. Circumoral pallor

4. Which of the following statements is true regarding caudal epidural anesthesia? A. In adults, it is usually done with the patient in the lateral position. B. In pediatric patients, it is usually done with the patient awake. C. The epidural needle can be safely advanced to the S3 or S4 level in pediatric patients. D. The “whoosh test” can be used to identify correct needle placement in the caudal canal.

9. A 4-year-old boy is undergoing circumcision under general anesthesia. After induction of anesthesia, he received a caudal block for postoperative analgesia. A decrease in which of the following clinical parameters is most helpful in confirming that the caudal block is successful? A. Blood pressure B. Anal sphincter tone C. Heart rate D. Lower extremity temperature

5. Which of the following clinical findings is least likely to be found after a successful caudal epidural blockade 123

Hadzic_Ch24_p121-126.indd 123

4/18/19 6:07 PM

124

PART 3

Clinical Practice of Regional Anesthesia

10. Which of the following medications is unsafe to use for percutaneous epidural neuroplasty in a patient with lumbar postlaminectomy pain syndrome with lumbar radiculopathy refractory to treatment with conventional analgesic therapies? A. Hypertonic saline B. Triamcinolone C. Betamethasone D. Preservative-free 2% lidocaine

ANSWERS AND EXPLANATIONS 1. B is correct. The conus medullaris is not part of the sacral canal, it is the tapered, lower end of the spinal cord, and it is found at lumbar vertebral levels 1 (L1) and/or 2 (L2). While it can occur at a lower level, it is not usually located within the sacral canal. A is incorrect. The filum terminale is a delicate strand of fibrous tissue about 20 cm in length, proceeding downward from the apex of the conus medullaris. It runs along the sacral canal and its lowest margin emerges at the sacral hiatus and traverses the dorsal surface of the fifth sacral vertebra and sacrococcygeal joint to reach the coccyx, where it is referred to as the coccygeal ligament. C is incorrect. Spinal meninges are part of the sacral canal. Near the midlevel of the sacral canal (typically the middle one-third of S2), the subarachnoid and subdural spaces cease to exist, and the lower sacral spinal roots and filum terminale pierce the arachnoid and dura maters. D is incorrect. Cauda equina, including filum terminale, are part of the sacral canal. 2. D is correct. The sacral foramina afford anatomical passages that permit the spread of injected solutions, such as LAs and adjuvants. The posterior sacral foramina are essentially sealed by the multifidus and sacrospinalis muscles, but the anterior foramina are unobstructed by muscles and ligaments, permitting ready egress of solutions through them. A is incorrect. The adipose tissue content of the caudal canal is subject to an age-related decrease in its density. This change is thought to be responsible for the transition from the predictable spread of LAs administered for caudal anesthesia in children to the limited and unpredictable segmental spread seen in adults. B is incorrect. The level of maximum curvature of the sacrum is at the level of S3 in almost 70% of patients. This curvature is more severe in men than in women. Therefore, a noncurving epidural needle will more likely pass easily into the caudal canal of women than men. C is incorrect. The angle between the axis of the lumbar canal and the sacral canal varies between 7 degrees and 70 degrees in subjects with marked lordosis. The cephalad flow of caudally injected solutions may be more limited in

Hadzic_Ch24_p121-126.indd 124

lordotic patients with exaggerated lumbosacral angles than in those with more flattened lumbosacral angles. 3. C is correct. Postherpetic neuralgia affecting the T8 dermatome is not an indication for a caudal epidural block. The unpredictable cephalad spread of anesthetics administered into the caudal canal limits the use of this technique in situations where it is essential to provide lower thoracic and upper abdominal neuraxial blockade. A is incorrect. There have been case reports of successful treatment of PDPH by performing an epidural blood patch through a caudally placed needle. B is incorrect. Lumbar radiculopathy is an indication for a caudal epidural block. D is incorrect. Caudal anesthesia has been suggested to be a better choice than traditional lumbar peridural block in patients with lumbar postlaminectomy pain syndrome. Attempting to place a needle through a spinal surgery scar may increase the likelihood of dural tear and increase the possibility of hematoma formation over the cauda equina from entrapment of blood between layers of the scar and connective tissue, resulting in cauda equina syndrome. 4. D is correct. The “whoosh test” has been described for identifying correct needle placement in the caudal canal. This characteristic sound has been noted during auscultation of the thoracolumbar region during the injection of 2–3 mL of air into the caudal epidural space. A is incorrect. The lateral position is often preferred in pediatric patients because it permits easy access to the airway when general anesthesia or heavy sedation has been administered prior to performing the block. In adults, it is usually performed in the prone position although the lateral position or the knee-chest position can also be used. B is incorrect. In pediatric patients, the block is often performed with the patient fully anesthetized, which is not recommended in older children and adults. C is incorrect. The dural sac extends lower in children than in adults, hence the epidural needle should not be advanced deeper than the S3 or S4 level to avoid puncturing the dura. 5. B is correct. Vasodilation of the lower extremities is least likely to be found after a successful caudal epidural blockade. Even though sympathetic block occurs following caudal block administration, it is not as extensive as with lumbar or thoracic epidural blockade. This is due to the fact that the sympathetic outflow from the preganglionic sympathetic fibers of the spinal cord ends at the L2 level. Hence, caudal block should not routinely result in peripheral vasodilation of the lower extremities to the degree witnessed with lumbar epidural blockade. A is incorrect. Caudal anesthesia causes a quick onset of full muscle relaxation of the anal area due to blockade of the sacral contribution of the parasympathetic nervous system; hence, relaxation of the anal sphincter can be used to predict the success of a caudal epidural blockade.

4/18/19 6:07 PM

CHAPTER 24

C is incorrect. The blockade of the sacral contribution of the parasympathetic nervous system can result in urinary retention following caudal epidural blockade. D is incorrect. The segmental spread of analgesia following caudal epidural block is more predictable and more extensive in children up to 12 years of age when compared to adults because anatomical barriers at the lumbosacral junction, which can develop with age, have not yet developed in this younger age group. Hence, sensory blockade of the L5 dermatome would be an expected finding following a successful caudal epidural blockade. 6. D is correct. 100 mg of 0.5% bupivacaine provides about 180–270 minutes prior to two-segment regression. A is incorrect. 300 mg of lidocaine 2% provides about 90–150 minutes prior to two-segment regression. B is incorrect. 400 mg of 2% chloroprocaine provides about 45–80 minutes prior to two-segment regression. C is incorrect. 150 mg of 1% ropivacaine provides about 120–210 minutes prior to two-segment regression. 7. A is correct. Caudal block lowers the metabolic and endocrine responses to stress. However, there is no evidence to show that serum thyroxine levels are lowered following administration of caudal epidural blockade. B is incorrect. Mean serum prolactin levels are lowered following a successful caudal epidural blockade. C is incorrect. Mean serum insulin levels are lowered following a successful caudal epidural blockade. D is incorrect. Mean serum cortisol concentrations are lowered following a successful caudal epidural blockade. 8. C is correct. T-wave changes on ECG are the earliest indicator of intravascular injection following the administration of epinephrine-containing test dose while performing caudal epidural blockade in children. This is followed by increases in heart rate and finally by blood pressure changes. A is incorrect. While elevation of the heart rate by 10 beats/minute is indicative of intravascular injection, it tends to occur after the development of T-wave changes on ECG. B is incorrect. Increase in systolic blood pressure by 15 mm Hg is also indicative of intravascular injection; however, it tends to occur after the development of T-wave changes and heart rate changes. D is incorrect. Circumoral pallor is not an indicator of intravascular injection following the administration of epinephrine-containing test dose while performing caudal epidural blockade in children.

Hadzic_Ch24_p121-126.indd 125

Caudal Anesthesia

125

9. B is correct. Relaxation of the anal sphincter following caudal epidural blockade occurs as a result of sacral parasympathetic blockade. This finding can be used to predict the success of caudal block in providing postoperative analgesia. A is incorrect. Caudal epidural block in children may induce significant changes in descending aortic blood flow while maintaining blood pressure and heart rate. C is incorrect. Multiple studies with transesophageal Doppler probes in pediatric patients undergoing caudal blocks have shown that mean heart rate and blood pressure remains unchanged, although significant changes such as decreases in aortic vascular resistance and increases in aortic ejection volume in the descending aorta were noted. D is incorrect. An increase in lower extremity temperature may occur due to vasodilation from sympathetic blockade of the lower extremities following successful caudal epidural block in pediatric patients. 10. B is correct. Triamcinolone is not recommended for epidural neuroplasty, because the particulate steroids may occlude the catheter and can also possibly cause spinal ischemia from unintentional intravascular injection. A is incorrect. Percutaneous epidural neuroplasty is a technique in which a caudal epidural catheter is left in place for up to 3 days for the purpose of injecting hypertonic saline into the epidural space to treat radiculopathy with low back pain and associated epidural scarring, typically from previous lumbar spinal surgery. Hypertonic saline is used to prolong pain relief due to its local anesthetic effect and its ability to reduce edema in previously scarred or inflamed nerve roots. C is incorrect. Betamethasone, being a non-particulate steroid, can be safely used for epidural neuroplasty. D is incorrect. Preservative-free 2% lidocaine can be safely used for epidural neuroplasty, although multiple studies have shown that it is more efficacious when mixed with hypertonic saline and betamethasone than when mixed with normal saline in the treatment of chronic lumbar radicular pain from lumbar postlaminectomy pain syndrome.

Suggested Reading Hadzic A. Caudal anesthesia. In: Candido KD, Tharian AR, Winnie AP, eds. Textbook of Regional Anesthesia and Acute Pain Management. 2nd ed. New York, NY: McGraw-Hill Publishing; 2017:chap 25.

4/18/19 6:07 PM

This page intentionally left blank

Hadzic_FM_p00i-pxx.indd 6

22/04/19 6:40 PM

Section 4

Combined Spinal and Epidural Anesthesia Chapter 25

Hadzic_Ch25_p127-134.indd 127

Combined Spinal-Epidural Anesthesia  129

22/04/19 1:52 PM

This page intentionally left blank

Hadzic_FM_p00i-pxx.indd 6

22/04/19 6:40 PM

25 Combined Spinal-Epidural Anesthesia Cedric Van Dijck

QUESTIONS DIRECTIONS: Choose the one best response to each question. 1. Which of the following is an advantageous reason to choose combined spinal-epidural (CSE) over conventional epidural analgesia for labor? A. Conventional epidural analgesia lengthens the first stage of labor. B. CSE shortens the total duration of labor. C. CSE causes less motor block than conventional epidural analgesia. D. CSE is the only technique that allows the parturient to ambulate during labor analgesia. E. Local anesthetic (LA) requirement is lower in CSE. 2. A 43-year-old woman with hypertension and hypercholesterolemia is scheduled to undergo anterior cruciate ligament reconstructive surgery in an ambulatory surgery setting. Which of the following is not a benefit of combined spinal-epidural (CSE) compared to single-shot spinal (SSS) or conventional epidural? A. CSE allows for a quicker discharge due to lower anesthetic dosing. B. CSE provides less motor block. C. There is a lower potential for anesthetic toxicity compared to conventional epidural. D. Sequential CSE allows for a wider safety margin regarding hemodynamics. E. None of the above is a known benefit of CSE. 3. Epidural volume extension (EVE): A. Compresses the epidural space, facilitating local anesthetic (LA) caudad spread, which theoretically causes a higher sensory block B. Is more efficient in labor analgesia than for surgery C. Is influenced by volume and patient position during injection, but not baricity D. Should not be used in an ambulatory surgery setting 4. The ligamentum flavum is thickest at: A. Cervical level B. Thoracic level

C. Lumbar level D. There is no difference in thickness across the vertebral column. E. Thickness of the ligament differs in all patients and therefore no reliable trend can be seen in flavum thickness. 5. Which is a potential benefit of the needle-throughneedle (NTN) technique? A. It allows for a test dose to be given upon epidural catheter placement. B. It has higher success rates than separate needle technique (SNT). C. It is known for a lower conversion rate to general anesthesia in cesarean delivery. D. It is less time-consuming compared to SNT. 6. Which is not a potential benefit of separate needle technique (SNT)? A. It has a higher success rate than needle-through-needle (NTN) technique. B. Dural puncture is more comfortable than with NTN. C. Patients experience it as more comfortable than NTN. D. It allows for a test dose to be given. E. None of the above is a known benefit of SNT. 7. Correct position of the epidural catheter in combined spinal-epidural (CSE) using needle-through-needle (NTN) technique may be confirmed by: A. Ultrasound-guided puncture B. Nerve stimulation C. Test dose D. None of the above 8. Which (combination of) drug(s) has the best profile (pain relief and side effects) for intrathecal administration in combined spinal-epidural (CSE) for labor analgesia? A. Sufentanil 10 mcg B. Bupivacaine 2 mg + fentanyl 5 mcg C. Bupivacaine 2 mg D. Bupivacaine 2 mg + fentanyl 25 mcg

129

Hadzic_Ch25_p127-134.indd 129

22/04/19 1:52 PM

130

PART 3

Clinical Practice of Regional Anesthesia

9. Which opioid has the fewest side effects when admin­ istered for the intrathecal component of combined spinal-epidural (CSE) for labor analgesia? A. Sufentanil 5–10 mcg B. Meperidine 10 mg C. Morphine 2 mg D. Fentanyl 50 mcg 10. Which of the following is a possible cause of a failed spinal component in combined spinal-epidural (CSE)? A. The spinal needle is too short to puncture the dura. B. The spinal needle is too small of a caliber to puncture the dura. C. The needle diverts from the midline. D. The needle diameter is too small. E. All of the above are possible causes of failed spinal component. 11. When the spinal component of combined spinal-epidural (CSE) is compared to single-shot spinal (SSS): A. CSE is associated with a lower level of motor block. B. CSE is associated with the same recovery time as SSS. C. SSS is associated with a higher use of vasopressors. D. Recovery from CSE is faster. E. CSE is associated with a higher sensory block than SSS. 12. A patient in the PACU experiences painful dysesthesia in the left leg after combined spinal-epidural (CSE) L4–L5 for total knee arthroplasty in the right knee. The most likely cause of this is: A. Tissue coring B. Metal toxicity from the needles C. Trauma from the spinal needle D. Inadvertent spinal cord puncture E. Infectious meningitis 13. Which of the following strategies helps avoid postdural puncture headache (PDPH) in combined spinalepidural (CSE)? A. Intrathecal injection of opioids B. Injection of local anesthetic (LA) mixture through the epidural catheter C. Both A and B are correct. D. None of the above 14. Which of the following is not a measure resulting in a lowered complication or increased success rate of combined spinal-epidural (CSE) technique? A. Rotating the needle 180° before introduction of the epidural catheter B. Protrusion of the spinal needle 13 mm past the epidural needle orifice C. External fixation devices holding the spinal needle in place before administration of intrathecal drugs D. Using an epidural needle with a back hole and tilting the spinal needle 10° to pass it through the correct opening at the needle tip 15. After the subarachnoid block wears off in a parturient who received combined spinal-epidural (CSE) for labor

Hadzic_Ch25_p127-134.indd 130

analgesia, she is transferred to the OR semi-urgently for a cesarean delivery for nonreassuring fetal heart rate. Which statement is true? A. A test dose should be administered containing local anesthetic (LA) and epinephrine 1:200.000 mixture. B. Epinephrine should not be administered in the test dose. C. Administration of LA mixture for the surgery will serve as a means of detecting intrathecal or intravascular placement of the epidural catheter. D. Dilute and short-acting LA will discriminate between an epidural, intrathecal, or intravascular catheter.

ANSWERS AND EXPLANATIONS 1. C is correct. CSE has been proven to cause less motor weakness than does conventional epidural analgesia. Even though this is true for analgesia for labor, CSE is known to cause a more profound motor block compared to conventional epidural when used for surgical intervention (eg, knee or hip arthroplasty). A is incorrect. CSE has been proven to shorten the first stage of labor, but conventional epidural analgesia has not been proven to lengthen this stage. B is incorrect. Studies found that total duration of labor was within the same reference range for both CSE and conventional epidural analgesia. D is incorrect. Though patients who received CSE can ambulate during labor depending on the dose and drug they received, other techniques may also allow the parturient to ambulate. E is incorrect. Although LA requirement is lower for the initial dose in CSE, Patel et al reported that the epidural minimal local anesthetic concentration (MLAC) of bupivacaine increased by a factor 1.45 when intrathecal medication (CSE) was administered. It seems that the dosage-reduction advantage of CSE does not extend past the initial epidural dose.1 2. B is correct. Less motor block is not a benefit of combined spinal-epidural (CSE). Numerous studies have proven CSE to cause a more profound motor block during surgery. The differences in dosing and drugs used for surgical block allow for more motor blockade in CSE compared to an epidural, without severely compromising hemodynamics. Bear in mind that the opposite is true for CSE during labor analgesia. A is incorrect. This is true when the anesthesiologist managing the case uses a lower dose for the spinal component. The “safety net” of having an epidural for a top-up dose should the subarachnoid block wear off before the end of the procedure allows for a lower dose of intrathecal local anesthetics (LAs) and/or opioids, allowing for a faster postoperative recovery and discharge in an ambulatory surgery setting.

22/04/19 1:52 PM

CHAPTER 25

C is incorrect. Because a subarachnoid block will cover most of this procedure, the dose of LA administered will be a lot lower compared to the epidural loading dose. LA concentrations in the systemic circulation will thus be much lower, providing a larger safety margin regarding local anesthetic systemic toxicity (LAST). D is incorrect. With the use of sequential CSE (low-dose spinal followed by incremental top-up doses using the epidural catheter), the anesthesiologist can titrate LA more carefully to find a lower dose that achieves successful surgical block. Theoretically, this slow loading has the advantage of managing hemodynamics better than with SSS or conventional epidural. The disadvantage is that this technique takes more time (CSE takes longer to place and titration of LA is time-consuming). Case reports have described this technique as successful in high-risk patients. The patient in this case is not considered high-risk. E is incorrect. This option is incorrect because answer choices A, C, and D are benefits of combined spinal-epidural (CSE). 3. B is correct. Even though evidence on EVE is still sparse, current literature suggests that the benefit of EVE with regards to block height is more pronounced in patients receiving combined spinal-epidural (CSE) with EVE for labor analgesia than for patients undergoing surgery. A is incorrect. In EVE, normal saline is injected in the epidural space, thus compressing the subarachnoid space, not the epidural space itself. Compression of the subarachnoid space by epidural injection improves caudad spread of the subarachnoid injectate as confirmed by myelography studies. C is incorrect. Studies have suggested that EVE for CSE is influenced by position of the patient: epidural injection of saline in the sitting position was less likely to result in extension of sensory block when compared to injection in the supine position with a left-lateral tilt of 15°. Other researchers have reported effects of volume: less efficient with smaller volumes and no effect with a volume lesser than 5 mL. Baricity may also influence the results of this technique: EVE resulted in a higher sensory block with isobaric bupivacaine but not with hyperbaric bupivacaine. D is incorrect. EVE with CSE holds the theoretical advantage of using a lower dose of intrathecal LA, as it improves spread of the injectate. This could be useful in ambulatory surgery as a lower dose of LA allows the patient to recover faster from the neural blockade, making a faster discharge plausible. 4. C is correct. Lumbar level. Flavum is thickest at lumbar level (5–6 mm).2 A is incorrect. Flavum is thinnest at the cervical level, at about 1.5–2 mm. B is incorrect. Flavum is thicker at the thoracic level, at about 3–5 mm. D is incorrect. Flavum increases in thickness from rostral to caudal. E is incorrect. Even though interindividual variability does occur, this trend is seen in the majority of cases.

Hadzic_Ch25_p127-134.indd 131

Combined Spinal-Epidural Anesthesia

131

5. D is correct. In SNT, at least two separate punctures with approach of the dural space are needed, which takes more time. A is incorrect. In NTN technique, test dose can be given only after placement of the subarachnoid component of the block, rendering it less reliable. B is incorrect. Success rate is reported to be higher in SNT, as well as a lower conversion rate to general anesthesia in multiple reports.3-7 C is incorrect. Data from elective cesarean deliveries performed under SNT reported a lower rate of conversion to general anesthesia due to failed neuraxial block. For this reason, it is not a potential benefit of the NTN technique. 6. C is correct. This is not a potential benefit of a SNT technique. Patients experience NTN as more comfortable, as only one puncture of the skin and approach of the dura is needed in NTN. A is incorrect. An SNT is associated with a higher success rate than NTN and with a lower conversion rate to general anesthesia with cesarean delivery.3-7 B is incorrect. Dural puncture itself seems to be experienced as more comfortable with SNT; the mechanism for this is not clear. D is incorrect. In SNT, a test dose can be given as with a conventional epidural before administration of the spinal component of the technique. E is incorrect. This option is incorrect because answer choices A, B, and D are potential benefits of SNT. 7. B is correct. The application of a low current (1 to 10 mA) electrical stimulation could be used to confirm the correct placement of the epidural catheter. A positive motor response (truncal or limb) could indicate that the catheter is in the epidural space.8 A is incorrect. Ultrasound-guided puncture may improve success rate of CSE, but it cannot confirm correct position of the catheter. C is incorrect. Test dose is a reliable means of detecting intravascular or intrathecal placement, but cannot confirm or suggest correct epidural position. D is incorrect. This option is incorrect because distractor B is the correct answer. 8. B is correct. The minimum local anesthetic dose (MLAD, comparable to the ED50) for bupivacaine plus fentanyl is 1.99 mg plus 5 mcg of fentanyl. Increasing the dose to 15 or 25 mcg of fentanyl did not increase block or pain relief but was associated with more side effects (option D). A is incorrect. Sufentanil alone is insufficient to provide adequate analgesia in the second stage of labor, as are all of the other opioids. C is incorrect. Bupivacaine alone works for a shorter period of time when compared to bupivacaine with lowdose opioid. D is incorrect. See explanation for B above.

22/04/19 1:52 PM

132

PART 3

Clinical Practice of Regional Anesthesia

9. A is correct. Sufentanil 5–10 mcg approaches the ED95 of intrathecal sufentanil, which is 8.9 mcg. B is incorrect. Meperidine is associated with higher incidence of nausea, vomiting, and hypotension, despite its intrinsic local anesthetic (LA) properties. C is incorrect. Morphine has a 60-minute delay in between intrathecal injection and effect, is associated with more side effects (eg, pruritus, nausea, vomiting), and holds the potential for delayed respiratory depression. D is incorrect. Fentanyl 50 mcg is inadequately dosed for intrathecal administration, as the ED50 (5.5 mcg) and the ED95 (17.4 mcg) are far lower. 10. E is correct. A, B, C, and D are all possible cause of failed spinal epidural (CSE). The dura is a very tough membrane that can be difficult to puncture when the needle is too short to tent the dura enough to puncture it eventually. The dura is a very tough membrane that can be difficult to puncture when the needle is too floppy to apply enough force. The dura can be missed laterally when the needle trajectory diverts from the midline. A too small of a caliber needle may delay cerebrospinal fluid (CSF) drainage through the needle, so that the needle is advanced into the anterior epidural space before CSF is seen. 11. E is correct. Motor and sensory block is higher in CSE when administered with 4 mL loss of resistance (LOR) air (rendering option A incorrect). B is incorrect. Recovery time from the spinal component of CSE takes longer (rendering option D incorrect). C is incorrect. Vasopressor use and hypotension occur more often in CSE. 12. C is correct. Trauma from the spinal needle occurs in up to 37% of spinal punctures in a CSE using a needlethrough-needle (NTN) technique. A minority of patients have symptoms that last beyond the duration of the subarachnoid block, and symptoms spontaneously resolve in almost all cases within a short period of time. A is incorrect. Tissue coring is frequent (up to 88% in CSE) but is unlikely to result in symptoms, especially this quickly after the administration of the block. B is incorrect. Metal toxicity has been proven to be of little relevance by many studies. D is incorrect. Even though up to 50% of anesthesiologists incorrectly identify the level of puncture, puncture of the cord at L4–L5 is very unlikely, as only 19% of the population has a spinal cord that extends beyond L1. E is incorrect. Infectious meningitis is a rare complication of CSE and would present later and with different symptoms. It is however important to always consider this diagnosis, as this condition can deteriorate rapidly when the diagnosis is missed.

Hadzic_Ch25_p127-134.indd 132

13. C is correct. Both intrathecal injection of opioids (option A) and injection of LA volume (option B) into the epidural space help decrease the incidence of PDPH in CSE. The mechanisms of this effect remain speculative. D is incorrect. See explanation for answer C. 14. A is correct. This maneuver doesn’t result in lowered complication or increased success rate. Rotating the needle 180° theoretically decreases the risk of passing the catheter through the site of the dural puncture. Studies, however, have proven this technique has a higher incidence of “wet tap.” B is incorrect. Protrusion of the needle 13 mm past the needle orifice of the epidural needle results in a 0% failure rate, whereas advancement of only 10 mm has a 15% failure rate. C is incorrect. External fixation devices help fixate the needle and are known to avoid needle movement resulting in loss of the correct position and inadverdent epidural injection of the spinal drug mixture. D is incorrect. This technique was described by McCaroll and Joshi and increased success rate of passing the spinal needle through the back hole of the epidural needle.9 15. A is correct. A conventional test dose containing LA and dilute epinephrine is less reliable than in a standard surgical patient due to heart rate variability in the laboring patient, but it can still suggest inadverdent catheter placement. B is incorrect. There is a theoretical concern of compromising placental circulation with the administration of epinephrine in case of an intravacular catheter. However intravascular placement should be ruled out prior to the administration of high doses of LA needed to achieve a surgical block. C is incorrect. The doses of LA administered to achieve surgical blockade are too toxic and intravascular or intrathecal placement of the catheter should be ruled out first by aspiration and administration of a test dose. D is incorrect. A dilute and short-acting mixture of LA such as chloroprocaine may serve as an alternative to a conventional test dose; however, in this setting it would be too time-consuming.

References 1. Patel NP, Armstrong SL, Fernando R, et al. Combined spinal epidural vs epidural labour analgesia: does initial intrathecal analgesia reduce the subsequent minimum local analgesic concentration of epidural bupivacaine? Anesthesia. 2012;67:584-593. 2. Lirk P, Moriggl B, Colvin J, et al. The incidence of lumbar ligamentum flavum midline gaps. Anesth Analg. 2004;98:1178-1180. 3. McAndrew CR, Harms P. Paraesthesiae during needle-throughneedle combined spinal epidural versus single-shot spinal for elective caesarean section. Anaesth Intensive Care. 2003;31:514-517. 4. Lyons G, Macdonald R, Mikl B. Combined epidural spinal anaesthesia for Caesarean section. Through the needle or in separate spaces? Anaesthesia. 1992;55:199-201.

22/04/19 1:52 PM

CHAPTER 25 5. Casati A, D’ambrosio A, De Negri P, Fanelli G, Tageriello V, Tarantino F. A clinical comparison between needle-throughneedle and double segment techniques for combined spinal and epidural anesthesia. Reg Anesth Pain Med. 1998;55:3904. 6. Rawal N, Van Zundert A, Holmström B, Crowhurst JA. Combined spinal epidural technique. Reg Anesth. 1997;55:40623. 7. Backe SK, Sheikh Z, Wilson R, Lyons GR. Combined epidural/ spinal anaesthesia: Needle-through-needle or separate spaces? Eur J Anaesthesiol. 2004;21:854-857.

Combined Spinal-Epidural Anesthesia

133

9. Joshi GP, MaCarroll SM: Combined spinal-epidural anesthesia using needle-through-needle technique [Letter]. Anesthesiology 1993;78: 406-407.

Suggested Reading Hadzic A. Combined spinal-epidural anesthesia. In: Ranasinghe JS, Davidson E, Birnbach DJ, eds. Textbook of Regional Anesthesia and Acute Pain Management. 2nd ed. New York, NY: McGraw-Hill Publishing; 2017:chap 26.

8. Tsui BC, Gupta S, Finucane B. Determination of epidural catheter placement using nerve stimulation in obstetric patients. Reg Anesth. 1999;24:17-23.

Hadzic_Ch25_p127-134.indd 133

22/04/19 1:52 PM

This page intentionally left blank

Hadzic_FM_p00i-pxx.indd 6

22/04/19 6:40 PM

Section 5

Postdural Puncture Headache Chapter 26

Hadzic_Ch26_p135-140.indd 135

Postdural Puncture Headache  137

22/04/19 6:27 PM

This page intentionally left blank

Hadzic_FM_p00i-pxx.indd 6

22/04/19 6:40 PM

26 Postdural Puncture Headache Brian E. Harrington

QUESTIONS DIRECTIONS: Choose the one best response to each question. 1. In addition to headache, patients with postdural puncture headache (PDPH): A. Are seldom nauseated B. May develop unilateral cranial nerve palsy C. Rarely experience associated symptoms D. Usually experience fever 2. The risk of developing postdural puncture headache (PDPH) is decreased with: A. Cutting (eg, Quincke) needles B. Female gender C. Large-gauge needles D. Older age 3. The meninges consist of the dura mater, arachnoid mater, and pia mater. Which of the following statements is true regarding the meninges? A. The dura mater contains frequent tight junctions. B. The arachnoid mater is acellular. C. The dura mater is thicker than the arachnoid mater. D. The alignment of dural fibers is parallel to the vertebral column. 4. Cerebrospinal fluid (CSF) in adults: A. Is produced at a rate of 0.35 mL/minute B. Is mostly intracranial C. Has a total volume of 10 mL/kg D. Is primarily produced by the arachnoid villa 5. Classic features of postdural puncture headache (PDPH) include: A. Bilateral, symmetric headache B. Changes in mental status (eg, sedation) C. Improvement in symptoms upon assuming an upright position D. Onset within 6 hours of meningeal puncture

6. Which of the following statements regarding an epidural blood patch (EBP) is true? A. Rate of success is directly related to the blood volume used. B. Performance of an EBP should be delayed for 24 hours following the onset of symptoms. C. Permanent resolution of headache exceeds 90% with an EBP. D. Backache is a common symptom following EBP. 7. Which measure has been shown to shorten the duration of postdural puncture headache (PDPH)? A. Intravenous hydration B. Prophylactic epidural blood patch (EBP) C. Oral methylxanthenes (eg, caffeine) D. Epidural saline infusion 8. Which of the following statements is true regarding the diagnosis of postdural puncture headache (PDPH)? A. Postpartum headaches after spinal anesthesia for cesarean delivery are usually PDPH. B. Imaging of the head (eg, MRI) is often useful in establishing a diagnosis of PDPH. C. Benign headaches in the perioperative setting are usually less severe than PDPH. D. It is not necessary to perform a physical exam to establish a diagnosis of PDPH. 9. Effective measures that can be employed to reduce the risk of postdural puncture headache (PDPH) include: A. Epidural loss-of-resistance using saline instead of air B. Replacing the stylet after diagnostic lumbar puncture prior to needle withdrawal C. Bedrest for a period of at least 6 hours following spinal anesthesia D. Aggressive oral hydration (particularly with caffeinecontaining fluids) 10. Pain associated with postdural puncture headache (PDPH): A. Is associated with cerebral vasoconstriction B. Is mediated by the abducens nerve (CN VI) C. Usually has a temporal distribution D. May result from traction on bridging vessels 137

Hadzic_Ch26_p135-140.indd 137

22/04/19 6:27 PM

138

PART 3

Clinical Practice of Regional Anesthesia

ANSWERS AND EXPLANATIONS 1. B is correct. In contrast to most bilateral and symmetric symptoms associated with PDPH, episodes of diplopia usually involve unilateral cranial nerve palsies.1 A is incorrect. Most patients with PDPH experience nausea (the most common associated symptom). C is incorrect. The IHS criteria for PDPH require the presence of at least one associated symptom. D is incorrect. Fever is not associated with PDPH and should prompt investigation of other (eg, infectious) diagnoses. 2. D is correct. For reasons that are incompletely understood, older patients have a lower risk of PDPH than younger patients.2 A is incorrect. The risk for PDPH is decreased with noncutting (eg, pencil-point) needles (Figure 26–1). B is incorrect. Nonpregnant female patients have approximately double the risk for PDPH compared with agematched male patients. C is incorrect. Needle gauge is a major risk factor for PDPH, with meningeal puncture with large-gauge needles being associated with a greater chance of developing PDPH, more severe symptoms, and higher likelihood of needing an epidural blood patch. 3. C is correct. The dura mater is ten times thicker than the arachnoid mater. The acellular dura mater, however, is relatively permeable to water and the arachnoid mater appears to be vital in preventing the loss of cerebrospinal fluid (CSF) (Figure 26–2).3 A is incorrect. Tight junctions and occluding junctions are characteristic of the arachnoid mater.

B is incorrect. The arachnoid mater is cellular. (The dura mater is acellular.) D is incorrect. While dural fibers form concentric layers, they have a random orientation. 4. A is correct. CSF is produced at a rate of approximately 0.35 mL/minute (21 mL/hour). This is significant, because there is evidence to suggest that total CSF loss in the setting of postdural puncture headache (PDPH) may be limited to around 20 mL. CSF production may thereby reconstitute normal CSF volume within 1 hour of halting further CSF loss with an epidural blood patch.4 B is incorrect. CSF is equally distributed between the intracranial and extracranial compartments. C is incorrect. The total volume of CSF in adults is around 150 mL (~2 mL/kg). Experimentally, PDPH symptoms can generally be produced with a loss of approximately 10% of total CSF volume (~15 mL). D is incorrect. CSF is produced in the choroid plexus and reabsorbed through the arachnoid villa. 5. A is correct. The cardinal symptom of PDPH is a bilateral, symmetric headache having a characteristic postural nature. Unilateral headache should prompt an investigation into other types of headache.5 B is incorrect. Changes in mental status such as sedation or confusion are inconsistent with PDPH and should be aggressively investigated as they may indicate more serious causes of headache. C is incorrect. PDPH symptoms should worsen upon assuming an upright position and improve with assuming a recumbent position. D is incorrect. Symptoms of PDPH are generally delayed, beginning 12 to 24 hours following meningeal puncture. 6. D is correct. Backache is the most common side effect of the EBP and is seen in approximately 25% of patients. Before undergoing a blood patch procedure, patients should be advised that backache is relatively common, usually mild, and self-limited. A is incorrect. There is little association between volume of blood used for EBP and success. Notably, patients are frequently unable to tolerate volumes greater than 20 mL.6 B is incorrect. While many practitioners wait for a period of time before performing an EBP, there appears to be little justification for delaying the procedure.7 C is incorrect. Although greater than 90% of patients receiving an EBP initially experience relief, the rate of complete relief is realistically around 75%.8

FIGURE 26–1  Spinal needles of different manufacturers with same external diameter. Non-cutting: A: Whitacre type. B: Spinal type. C: Sprotte type. Cutting: D and E: Quincke type. Scanning electron microscopy. Magnification X40. (Reproduced with permission from van Kooten F, Oedit R, Bakker SL, et al: Epidural blood patch in post dural puncture headache: a randomised, observer-blind, controlled clinical trial, Neurol Neurosurg Psychiatry. 2008 May;79(5):553-558.)

Hadzic_Ch26_p135-140.indd 138

7. B is correct. While the prophylactic EBP does not reduce the incidence of PDPH after accidental dural puncture, it does appear to significantly shorten the duration of symptoms if PDPH occurs.9 A is incorrect. IV hydration may provide some symptomatic relief but does not influence the duration of symptoms.10

22/04/19 6:27 PM

CHAPTER 26

Postdural Puncture Headache

139

A. Dura mater

Dural fibers

B. Arachnoid layer Desmosomes

Tight junctions

Arachnoid cells

FIGURE 26–2  A: Human spinal dura mater. Collagen fibers in random direction. Scanning electron microscopy. Magnification ×6500. (Reproduced with permission from Dittmann M, Reina MA, López García A: New results in the visualization of the spinal dura mater with scanning electron microscopy, Anaesthesist. 1998 May;47(5):409-413.)

B: Human spinal arachnoid layer. Arachnoid cells. Transmission electron microscopy. Magnification ×150,000.

(Reproduced with permission from Reina MA1, Prats-Galino A, Sola RG, et al: Structure of the arachnoid layer of the human spinal meninges: a barrier that regulates dural sac permeability, Rev Esp Anestesiol Reanim. 2010 Oct;57(8):486-492.)

C is incorrect. Oral methylxanthenes may provide some symptomatic relief but do not influence the duration of headache symptoms. D is incorrect. Epidural saline infusion has never been demonstrated to decrease the duration of PDPH symptoms. 8. C is correct. Benign headaches are very common in the perioperative setting. Etiologies include dehydration, hypoglycemia, anxiety, and caffeine withdrawal. These headaches are generally less severe than PDPH, underscoring the importance of determining the severity of symptoms when trying to establish a diagnosis of PDPH.11,12 A is incorrect. Most headaches following obstetric anesthesia interventions are benign. Even severe headaches have been found to more commonly be benign (eg, tensiontype headaches) than PDPH.13 B is incorrect. Imaging is generally unnecessary and unremarkable in the setting of PDPH. It is most useful in diagnosing serious non-PDPH pathology.

Hadzic_Ch26_p135-140.indd 139

D is incorrect. Although the value of the physical exam is limited in the diagnosis of PDPH, it is essential in ruling out some serious causes of headache. A directed physical exam should include vital signs (patients with PDPH should be afebrile and relatively normotensive) and basic neurologic exam (which—with the possible exception of ocular movement—should be nonfocal). 9. B is correct. Strupp M et al demonstrated that replacement of the stylet significantly reduced the development of PDPH after lumbar puncture. It seems advisable to perform this quick, simple, and risk-free maneuver whenever possible.14 A is incorrect. The medium used for epidural loss-ofresistance techniques does not appear to consistently influence the incidence of accidental dural puncture (ADP) or PDPH. C is incorrect. Enforced bedrest does not reduce the incidence of PDPH.

22/04/19 6:27 PM

140

PART 3

Clinical Practice of Regional Anesthesia

D is incorrect. While patients are encouraged to avoid dehydration, over-hydration does not reduce the risk of developing PDPH. 10. D is correct. Postural headache symptoms in the setting of intracranial hypotension are thought to be due to a combination of cerebral vasodilation and traction on bridging vessels (between the dura and cranium). A is incorrect. Because intracranial volume remains constant (the Monro-Kellie hypothesis), the loss of intracranial cerebrospinal fluid results in a compensatory vasodilation. Because veins are much more compliant than arteries, this is primarily manifest as venodilation. B is incorrect. The abducens nerve is an efferent motor (not sensory) nerve. Through pressure or traction, intracranial hypotension can compromise function of the abducens nerve, resulting in paresis of the lateral rectus muscle causing diplopia.15 C is incorrect. Classically, headache symptoms of PDPH are bilateral and symmetric. Rather than temporal, symptoms are usually frontal or occipital (or both).

References 1. Nishio I, Williams BA, Williams JP. Diplopia: a complication of dural puncture. Anesthesiology. 2004;100:158-164. 2. Lybecker H, Møller JT, May O, Nielsen HK. Incidence and prediction of postdural puncture headache: a prospective study of 1021 spinal anesthesias. Anesth Analg. 1990;70;389-394. 3. Van Zundert A, Reina MA, Lee RA. Prevention of post-dural puncture headache (PDPH) in parturients. Contributions from experimental research. Acta Anaesthiol Scand. 2013;57:947-949. 4. Levine D, Rapalino O. The pathophysiology of lumbar puncture headache. J Neurol Sci. 2001;192:1-8. 5. Lybecker H, Djernes M, Schmidt JF. Postdural puncture headache (PDPH): onset, duration, severity, and associated symptoms. An analysis of 75 consecutive patients with PDPH. Acta Anaesthesiol Scand. 1995;39(5):605-612. 6. Paech M, Doherty DA, Christmas T, Wong CA; Epidural Blood Patch Trial Group. The volume of blood for epidural blood patch

Hadzic_Ch26_p135-140.indd 140

in obstetrics: A randomized, blinded clinical trial. Anesth Analg. 2011;113:126-133. 7. Sandesc D, Lupei MI, Sirbu C, Plavat C, Bedreag O, Vernic C. Conventional treatment or epidural blood patch for the treatment of different etiologies of post dural puncture headache. Acta Anaesthesiol Belg. 2005;56:265-269. 8. Safa-Tisseront V, Thormann F, Malassine P, et al. Effectiveness of the epidural blood patch in the management of post-dural puncture headache. Anesthesiology. 2001;95:334-339. 9. Scavone BM, Wong CA, Sullivan JT, Yaghmour E, Sherwani SS, McCarthy RJ. Efficacy of a prophylactic epidural blood patch in preventing postdural puncture headache in parturients after inadvertent dural puncture. Anesthesiology. 2004;101:1422-1427. 10. Warwick WI, Neal JM. Beyond spinal headache: prophylaxis and treatment of low-pressure headache syndromes. Reg Anesth Pain Med. 2007;32:455-461. 11. Goldszmidt E, Kern R, Chaput A, Macarthur A. The incidence and etiology of postpartum headaches: a prospective cohort study. Can J Anesth. 2005;52:971-977. 12. Santeanen U, Rautoma P, Luurila H, Erkola O, Pere P. Comparison of 27-gauge (0.41-mm) Whitacre and Quincke spinal needles with respect to post-dural puncture headache and non-dural puncture headache. Acta Anaesthesiol Scand. 2004;48:474-479. 13. Stella C, Jodicke CD, How HY, Harkness UF, Sibal BM. Postpartum headache: is your workup complete? Am J Obstet Gynecol. 2007;196:318.e1-7. 14. Strupp M, Brandt T, Muller A. Incidence of post-lumbar puncture syndrome reduced by reinserting the stylet: a randomized prospective study of 600 patients. J Neurol. 1998;245:589-592. 15. Larrier D, Lee A. Anatomy of headache and facial pain. Otolaryngol Clin N Am. 2003;36:1041-1053.

Suggested Reading Hadzic A. Postdural puncture headache. In: Harrington BE, Reina MA, eds. Textbook of Regional Anesthesia and Acute Pain Management. 2nd ed. New York, NY: McGraw-Hill Publishing; 2017:chap 27.

22/04/19 6:27 PM

PART 3D

Ultrasound-Guided Nerve Blocks

Section 1

Fundamentals of Ultrasound-Guided Regional Anesthesia Chapter 27

Physics of Ultrasound  143

Chapter 28

Optimizing an Ultrasound Image  147

Chapter 29

Introduction to Ultrasound-Guided Regional Anesthesia  151

Hadzic_Ch27_p141-146.indd 141

19/04/19 10:42 AM

This page intentionally left blank

Hadzic_FM_p00i-pxx.indd 6

22/04/19 6:40 PM

27 Physics of Ultrasound Jacob Hutchins

QUESTIONS DIRECTIONS: Choose the one best response to each question. 1. Ultrasound is defined as sound that has a frequency greater than ___ cycles per second. A. 1000 B. 10,000 C. 20,000 D. 5000 2. What type of ultrasound image mode results in a two-dimensional image of an area that is also scanned by a linear array of 100–300 piezoelectric elements? A. A-mode B. B-mode C. Doppler mode D. M-mode 3. Which of the following is increased by an increase in either the propagation speed or density of the medium? A. Frequency B. Axial resolution C. Acoustic velocity D. Acoustic impedance 4. A 43-year-old obese man is having a below-knee amputation. You decide to perform an ultrasound-guided subgluteal sciatic nerve block. You place the probe in the correct position but are having a difficult time viewing the sciatic nerve due to the increased depth required to view the nerve. Which of the following can be adjusted to improve your view of the sciatic nerve at this increased depth? A. Increase the dynamic range above 100 dB B. Increase the frequency C. Decrease the frequency D. Change the ultrasound machine to A-mode 5. Nearly complete reflection occurs when: A. The media impedances are equal. B. Sound is redirected by rough surfaces.

C. There is a significant difference between media impedance. D. Sound energy is converted into heat. 6. The use of ultrasound for nerve blocks can result in a bioeffect that is characterized by: A. Decreased heat within the tissues B. Increased heat within the tissues C. Hazardous conditions only when the thermal index is below 1.0 D. No effect on tissue temperature 7. What does the use of a coupling medium such as ultrasound gel help to eliminate or minimize? A. Reflection B. Refraction C. Scattering D. Absorption 8. Which of the following statements is true regarding the use of color flow Doppler? A. Red color denotes flow away from the transducer and blue color denotes flow toward the transducer. B. Color flow Doppler is a color-coded map superimposed on an M-mode image. C. Blue color denotes flow away from the transducer and red color denotes flow toward the transducer. D. Color flow Doppler is five times more sensitive than power Doppler in detecting blood flow. 9. When you adjust the focus, what are you improving on the plane of interest? A. Time-gain compensation B. Frequency C. Spatial resolution D. Acoustic impedance 10. Which of the following adjustments will result in improved resolution? A. Decreasing the frame rate B. Decreasing the frequency C. Increasing the beam width D. Increasing the wavelength

143

Hadzic_Ch27_p141-146.indd 143

19/04/19 10:42 AM

144

PART 3

Clinical Practice of Regional Anesthesia

ANSWERS AND EXPLANATIONS 1. C is correct. Ultrasound is high-frequency sound and refers to mechanical vibrations above 20 kHz. Ultrasound frequencies commonly used for medical diagnosis are between 2 and 15 MHz. A, B, and D are incorrect. These frequencies are not related to the definition of ultrasound. 2. B is correct. This produces a two-dimensional image by scanning 100–300 piezoelectric elements as opposed to only one element that occurs in a mode. In B-mode the horizontal and vertical directions represent real distances in tissue, whereas the intensity of the grayscale indicates echo strength. A is incorrect. A-mode is when the transducer sends a single pulse of ultrasound into the medium and creates only a one-dimensional image. C is incorrect. Color Doppler produces a color-coded map of Doppler shifts superimposed onto a B-mode ultrasound image. Blood flow direction depends on whether the motion is toward or away from the transducer. D is incorrect. This occurs when a single beam in an ultrasound scan can be used to produce a picture with a motion signal, where movement of a structure such as a heart valve can be depicted in a wavelike manner. 3. D is correct. Acoustic impedance is the degree of difficulty demonstrated by a sound wave being transmitted through a medium; it is equal to density ρ multiplied by acoustic velocity c (z = ρc). It increases if the propagation speed or the density of the medium is increased. A is incorrect. Frequency is the number of cycles repeated per second and measured in hertz (Hz). B is incorrect. Axial resolution is the minimum separation of above-below planes along the beam axis. It is determined by spatial pulse length. C is incorrect. Acoustic velocity is the speed at which a sound wave travels through a medium. It is equal to the frequency times the wavelength. 4. C is correct. If the ultrasound penetration is not sufficient to visualize the structures of interest, a lower frequency is selected to increase the penetration. However, the use of longer wavelengths (lower frequency) results in lower resolution because the resolution of ultrasound imaging is proportional to the wavelength of the imaging wave. A is incorrect. Dynamic range is the range of amplitudes from largest to the smallest echo signals that an ultrasound system can detect. Dynamic range less than 50 dB or greater than 100 dB is probably too low or too high in terms of visualization of peripheral nerve. B is incorrect. Higher frequencies are absorbed at a greater rate than lower frequencies. Therefore, they are not good for improved scanning of deeper tissues. However, a higher scanning frequency gives better axial resolution.

Hadzic_Ch27_p141-146.indd 144

D is incorrect. A-mode provides little information on the spatial relationship of imaged structures. Therefore, A-mode ultrasound is not applicable in regional anesthesia. 5. C is correct. The strength of the reflection from an interface is variable and depends on the difference of impedances between two affinitive media and the incident angle at the boundary. If there is a significant difference between media impedances, there will be nearly complete reflection. A is incorrect. If the media impedances are equal, there is no reflection (no echo). B is incorrect. Scattering is the redirection of sound in any direction by rough surfaces or by heterogeneous media. D is incorrect. Absorption is the direct conversion of sound energy into heat. 6. B is correct. The generation of heat increases as ultrasound intensity or frequency is increased. For similar exposure conditions, the expected temperature increase in bone is significantly greater than in soft tissues. A is incorrect. Ultrasound results in generation of heat, not decrease of heat. C is incorrect. The thermal index is defined as the transducer acoustic output power divided by the estimated power required to raise tissue temperature by one degree Celsius. A thermal index greater than 1.0 is hazardous. D is incorrect. Ultrasound increases tissue temperature. 7. A is correct. A coupling medium must be used between the transducer and the skin to displace air from the transducer-skin interface. In clinical scanning, even a very thin layer of air between the transducer and skin may reflect virtually all the ultrasound, hindering any penetration into the tissue. Therefore, a coupling medium, usually an aqueous gel, is applied between surfaces of the transducer and skin to eliminate the air layer and eliminate or minimize reflection. B is incorrect. Refraction is a change in sound direction when crossing the boundary between two media. C is incorrect. Scattering is the redirection of sound in any direction by rough surfaces or by heterogeneous media. D is incorrect. Absorption is the direct conversion of the sound energy into heat. 8. C is correct. Blood flow direction depends on whether the motion is toward or away from the transducer. Selected by convention, red and blue colors provide information about the direction and velocity of the blood flow. The red color on the top of the bar denotes the flow coming toward the ultrasound probe, and the blue color on the bottom of the bar indicates the flow away from the probe. A is incorrect. Selected by convention, red and blue colors provide information about the direction and velocity of the blood flow. The red color on the top of the bar denotes the flow coming toward the ultrasound probe, and the blue color on the bottom of the bar indicates the flow away from the probe.

19/04/19 10:42 AM

CHAPTER 27

B is incorrect. Color flow Doppler is a color-coded map superimposed on a B-mode image. D is incorrect. Power Doppler is five times more sensitive than color flow Doppler in detecting blood flow but it does not provide any information on the direction and speed of blood flow. 9. C is correct. Adjusting focus improves the spatial resolution on the plane of interest because the beam width is converged. However, the reduction in beam width at the selected depth is achieved at the expense of degradation in beam width at other depths, resulting in poorer images below the focal zone. A is incorrect. Gain is the ratio of output to input electric power; it controls the brightness of the image. The gain is usually measured in decibels (dB). Increasing the gain amplifies not only the returning signals, but also the background noise within the system in the same manner. Time-gain compensation (TGC) is time-dependent amplification. TGC function can be used to increase the amplitude of incoming signals from various tissue depths. B is incorrect. Frequency is the number of cycles repeated per second and measured in hertz (Hz).

Hadzic_Ch27_p141-146.indd 145

Physics of Ultrasound

145

D is incorrect. Acoustic impedance z is the degree of difficulty demonstrated by a sound wave being transmitted through a medium; it is equal to density ρ multiplied by acoustic velocity c (z = ρc). It increases if the propagation speed or the density of the medium is increased. 10. A is correct. Imaging resolution will be compromised by increasing the frame rate. Optimizing the ratio of resolution to the frame rate is essential for providing the best possible image. B is incorrect. Higher frequencies have narrower focus and provide better axial and lateral resolution; thus, decreasing the frequency decreases the resolution. C is incorrect. Lateral resolution can also be improved by adjusting focus to reduce the beam width. D is incorrect. This is effectively decreasing the frequency and as such results in decreased resolution.

Suggested Reading Hadzic A. Physics of ultrasound. In: Xu D, ed. Textbook of Regional Anesthesia and Acute Pain Management. 2nd ed. New York, NY: McGraw-Hill Publishing; 2017:chap 28.

19/04/19 10:42 AM

This page intentionally left blank

Hadzic_FM_p00i-pxx.indd 6

22/04/19 6:40 PM

28 Optimizing an Ultrasound Image Shaun De Meirsman

QUESTIONS DIRECTIONS: Choose the one best response to each question. 1.  There are different sonographic imaging modes for medical diagnostics, such as conventional imaging, compound imaging, and tissue harmonic imaging. What statement is true regarding conventional imaging? A. Implemented by acquiring several (usually three to nine) overlapping frames from different frequencies or from different angles B. Generated from a single-element angle beam at a primary frequency designated by the transducer C. Acquires the information from harmonic frequencies (multiples of the primary frequency) generated by an ultrasound beam transmission through tissue D. Used in obese patients because the contrast resolution can be increased for adipose tissue 2.  Functional imaging components are useful in optimizing an ultrasound image and consist of the following five adjustments: A. Doppler, depth, gain, focus, frequency B. Doppler, depth, gain, angle, frequency C. depth, gain, frequency, focus, ambiance D. Doppler, gain, depth, ambiance, frequency 3.  What should you do when the spread of local anesthetic (LA) is not seen during the injection process? A. The operator should stop the injection, tilt the transducer, and inject a tiny amount of LA (or air) to locate the needle tip and spread of injectate. B. If you are sure of the target structure, keep injecting. C. Check the ultrasound machine and the injectate to ensure it is all in working order. Keep injecting; the spread will show up sooner or later.

D. Tell the nurse to push harder so the velocity and change in volume make a bigger appearance on the ultrasound machine. 4.  Catheters are often not visualized on ultrasound because: A. They are not visible because the material is not echogenic. B. The needle, nerve, and catheter are never in the same plane of the ultrasound beam, therefore becoming challenging to image. They also coil within the tissue sheaths. C. Catheters have only one echo dot; only the catheter tip is visible. D. This depends on the expertise level of the operator. 5.  When you encounter a shadowing artifact, you can resolve the problem by: A. Changing the scanning plane to find the best acoustic window B. Not doing anything. Shadowing points to a beneficial diagnostic value to point out the different nerve structures C. Changing the transducer, which helps to optimize the ultrasound image D. Switching to an in-plane/out-of-plane view. This can change the appearance of the ultrasound image; when you have located the target structure you can switch back to the original view. 6.  You are preparing to perform an ankle block. Ideally the transducer selected should be: A. A linear transducer B. A curved (phased-array) transducer C. A hockey stick transducer D. I use the one that is available at my clinic.

147

Hadzic_Ch28_p147-150.indd 147

19/04/19 10:44 AM

148

PART 3

Clinical Practice of Regional Anesthesia

7.  There are a lot of ultrasound artifacts. The accompanying figure is an example of: Needle

(Reproduced with permission from Hadzic A: Hadzic’s Peripheral Nerve Blocks and Anatomy for Ultrasound-Guided Regional Anesthesia, 2ed. New York, NY: McGraw-Hill Education; 2011.)

A. Shadowing artifact B. Velocity error artifact C. Enhancement artifact D. Reverberation artifact 8.  Which of the following statements gives the best advice regarding aiming for good patient position? A. Position the patient in such a way that the dominant hand is most optimally placed to handle the probe. The ultrasound machine should be in perpendicular view. B. Arrange the patient, the anesthesiologist, the ultrasound machine, and the sterile block tray ergonomically to allow for time-efficient performance of the procedure. C. Position the patient in such a way that the block is easy to perform for the operator; this leads to greater block success. The ultrasound machine should be plugged in to allow for quick access. The sterile tray should always be located next to the ultrasound machine. D. Explain the procedure to the patient and give sedation, then look for the natural course of the patient’s body and fixate this position using tape. Place the ultrasound and sterile tray ergonomically and position yourself next to the ultrasound machine. Make sure you can rotate your head to look at the display. 9.  When preparing to scan a patient, the operator can use the following acronym to help gain a good ultrasound image: A. SCANNING B. BART C. BATS D. FAST

D is incorrect. Tissue harmonic imaging can be useful in adipose tissue because the contrast resolution can be increased resulting in a better image of the target structure. 2.  A is correct. Doppler, depth, gain, focus, frequency. B is incorrect. Angle is used in spatial compounding. It combines multiple frames from different angles. C is incorrect. Doppler is useful to differentiate target structures from vessels. D is incorrect. Ambiance is set to help increase an ultrasound image. This is part of your preparation before scanning. 3. 

A is correct. The operator should stop the injection, tilt the transducer, and inject a tiny amount of LA (or air) to locate the needle tip and spread of injectate. B is incorrect. The target structure should always be visualized to reduce the risk of nerve injury and to assure block success. C is incorrect. Machine failure can be an option. Also, the freeze function could be activated by accident. LA spread is visible at the needle tip; make sure it is seen on the ultrasound screen. D is incorrect. Telling the nurse to stop injecting would be the better approach. You can inject a little bit of LA to help visualize the needle tip. If not, you should try to optimize the needle image.

4. 

B is correct. The needle, nerve, and catheter are never in the same plane of the ultrasound beam. Therefore the catheter can become challenging to image. They also coil within the tissue sheaths making them less visible. A is incorrect. Catheter materials are in a limited fashion echogenic. C is incorrect. Catheters have different marking systems. The catheter tip is often visible because the needle catheter plane is the same. When the catheter is advanced, it can be visualized if the transducer is brought more distally. The catheter tip can also be visualized by observing the spread of an injectate. D is incorrect. Level of expertise may be beneficial in visualization of different structures; however, a catheter in general can be difficult to see. Injecting a small amount of air can be beneficial to view the catheter tip.

5. 

ANSWERS AND EXPLANATIONS 1. 

B is correct. Conventional imaging is generated from a single-element angle beam at a primary frequency designated by the transducer. A is incorrect. This is the definition of compound imaging. C is incorrect. This is the definition of tissue harmonic imaging.

Hadzic_Ch28_p147-150.indd 148

A is correct. Changing the scanning plane to find the best acoustic window. B is incorrect. Acoustic shadowing may have a beneficial diagnostic value; however, the shadowing can interfere with nerve visualization in regional anesthesia. C is incorrect. Changing the transducer might help if the selected procedure demands a different probe. The shadowing problems are not resolved because the shadow is cast by a significant attenuation of ultrasound signal deep to tissues and structures that absorb or reflect most of the ultrasound waves, as bones, calcifications, or air. This is

19/04/19 10:44 AM

CHAPTER 28

manifested by a weak or absent echo area, which appears as a shadow on the imaging behind a bright, hyperechoic interface. D is incorrect. Switching back to the original view will bring back the shadowing issues. 6. 

C is correct. A hockey stick ultrasound transducer is an ideal choice for vascular access or a superficial block with limited space. A is incorrect. A linear transducer is best for scanning superficial anatomic structures. B is incorrect. A curved (phased-array) transducer displays a sector image and is typically better for deeperpositioned structures. D is incorrect. Your clinic should have different transducers that best fit the scheduled procedure.

7 .  D is correct. Reverberation displays in the form of parallel, equally spaced bright linear echoes behind the reflectors in the near field of the image. The multiple echoes occur when the ultrasound beam bounces repeatedly between the interfaces of the transducer and a strong reflector, especially when these two interfaces are parallel to each other. A is incorrect. Shadowing is an attenuation of ultrasound signal deep to tissues and structures that absorb or reflect most of the ultrasound waves. B is incorrect. Velocity error is the displacement of the interface, which is caused by the difference of actual velocity of ultrasound in human soft tissue, compared with the calibrated speed, which is assumed to be a constant velocity of 1540 m/s set by the ultrasound system. C is incorrect. Enhancement manifests as overly intense echogenicity behind an object. 8. 

B is correct. The patient should be positioned in such a way that the patient, the anesthesiologist, the ultrasound machine, and the sterile block tray are all arranged

Hadzic_Ch28_p147-150.indd 149

Optimizing an Ultrasound Image

149

ergonomically to allow for time-efficient performance of the procedure. A is incorrect. Ideally there is no dominant hand positioning. The ultrasound machine should be set up on the opposite side of the patient with the screen at the operator’s eye level. C is incorrect. There is no literature evidence stating this fact. Patient discomfort is a common cause of block failure. D is incorrect. Always make sure that, as an operator you are comfortable and relaxed; this will enhance the procedure. Place the ultrasound opposite of your location and make sure needle eye alignment and ultrasound display is in one line. 9. 

A is correct. SCANNING. S: Supplies, C: Comfortable positioning, A: Ambiance, N: Name and procedure, N: Nominate transducer, I: Infection control, N: Note lateral/ medial/superior/inferior orientation on screen, G: Gain depth. B is incorrect. BART (Blue Away, Red Toward). Derived from the color Doppler technique, it is a mnemonic to understand the colors associated with the flow of blood. C is incorrect. BATS (Better Anesthesia True Sonography). A principle that explains the way to use sonography to help enhance a safer anesthesia practice. D is incorrect. FAST (Focused Assessment with Sonography for Trauma). Most commonly used in the emergency department.

Suggested Reading Hadzic A. Optimizing an ultrasound image. In: Xu D, De Meirsman S, Schreurs R, eds. Textbook of Regional Anesthesia and Acute Pain Management. 2nd ed. New York, NY: McGraw-Hill Publishing; 2017:chap 29.

19/04/19 10:44 AM

This page intentionally left blank

Hadzic_FM_p00i-pxx.indd 6

22/04/19 6:40 PM

29 Introduction to Ultrasound-Guided Regional Anesthesia Uma Shastri and Vikram Bansal

QUESTIONS DIRECTIONS: Choose the one best response to each question. 1. Benefits of ultrasound-guided nerve blocks include: A. Visualization of the anatomy in the area B. Visualization of the needle tip C. Visualization of real-time local anesthetic spread D. All of the above 2. The mnemonic PART is used to describe the components that optimize an ultrasound image. “P” refers to: A. Placing the transducer in a position over the extremity at which the underlying nerve is expected to be in the field of view B. Minimizing the distance to the target and compressing the underlying subcutaneous adipose tissue C. Fine-tuning of the view of the target structure D. Bringing the face of the probe into a perpendicular arrangement with the underlying target to maximize the number of returning echoes 3. In performing a brachial plexus block, at which of the following locations will the nerves appear most hypoechoic? A. Interscalene B. Supraclavicular C. Infraclavicular D. Axillary 4. Performing nerve blocks using the short-axis, in-plane approach provides all of the following advantages except: A. Visualization of the entire needle B. Increased safety while teaching nerve blocks C. Better block success D. Allows monitoring depth of needle placement

5. The concept of heeling improves ultrasound image by: A. Ascertaining needle tip location B. Moving insertion site further away from the probe C. Greater scattering of ultrasound beams D. Pressing in on the edge of the transducer opposite the insertion site 6. When approaching an ultrasound-guided interscalene nerve block, the nerve should be approached tangentially when passing through fascial planes because it: A. Prevents contact with the nerve B. Makes it easier to pass through the fascial layers C. Keeps the needle in-plane D. Prevents forward motion of the needle 7. For what is the technique of hydrodissection used? A. Advancing the needle B. Improving image clarity C. Preventing paresthesia if local anesthetic (LA) is injected D. Speeding the onset of the block 8. According to one study, despite visualizing the needle tip indenting the surface of the nerve, and a nerve stimulator set to deliver a current of 0.5 mA or less, motor stimulation was not elicited how much of the time? A. 0% B. 10% C. 25% D. 50% 9. Which of the following is a benefit of ultrasound-guided peripheral nerve blockade? A. Eliminates local anesthetic systemic toxicity (LAST) B. Eliminates intravascular injections due to vessel visualization C. Eliminates paresthesia and nerve injury D. Allows real-time assessment of local anesthetic (LA) placement

151

Hadzic_Ch29_p151-154.indd 151

19/04/19 10:45 AM

152

PART 3

Clinical Practice of Regional Anesthesia

10. Which of the following techniques helps optimize local anesthetic (LA) deposition? A. Deposition of LA in muscle B. Combining ultrasound and nerve stimulator techniques for peripheral nerve blockade C. Observing LA beyond the needle tip D. Injecting LA in small aliquots

ANSWERS AND EXPLANATIONS 1. D is correct. There are many practical advantages of ultrasound for nerve blockade. Ultrasound allows visualization of the anatomy of the region of interest. This allows visualization of the needle pathway to the target while avoiding structures that might be damaged by the needle. Ultrasound also allows visualization of the needle tip as it is passed through the tissues, confirming alignment with the intended path, reducing the likelihood of inadvertent needle trauma to unintended structures. Real-time ultrasound imaging also permits continual visualization of local anesthetic (LA) solution delivery to ensure proper distribution. 2. B is correct. The mnemonic PART stands for Pressure, Alignment, Rotation, and Tilting. All of these maneuvers can improve ultrasound image by different mechanisms. “P” refers to pressure, which involves applying force on the probe to minimize the distance to the target via compression of superficial structures, including subcutaneous adipose tissue. A is incorrect. Refers to alignment, which involves placing the transducer in a position over the extremity/trunk at which underlying nerve is expected to be in the field of view. C is incorrect. Refers to rotation, which involves turning the probe slightly to fine-tune the target structure. D is incorrect. Refers to tilting the probe, which improves the image by bringing the face of the probe into a perpendicular arrangement with the underlying target to maximize the number of returning echoes, thus providing the best image. 3. A is correct. Nerves in short axis have an appearance that is to some extent determined by their proximity to the neuraxis. In close association with the spine, nerves and nerve roots are comprised primarily of neural tissue, with minimal connective tissue. Because neural tissue appears hypoechoic on ultrasound, while the connective tissue between fascicles is hyperechoic, nerves near the neuraxis appear as dark nodules, or hypoechoic. For this reason, the interscalene location, being the most proximal location, will appear the most hypoechoic. B, C, and D are incorrect. Neither of these options is the closest approach to the neuraxis. 4. C is correct. Neither the short-axis in-plane or out-ofplane has been shown to be superior in terms of block success. This makes C the correct answer. Nerve imaging may be performed in either short-axis or long-axis position.

Hadzic_Ch29_p151-154.indd 152

The short-axis provides round, often hyperechoic nerve structures, whereas the long-axis provides the longitudinal view of the nerve. Most people are more used to the round, often hyperechoic neural elements seen with the short-axis view. A and B are incorrect. Visualization of the entire needle is a benefit of the short-axis, in-plane view. Because the entire needle is visualized with the short-axis, in-plane approach, increased safety while teaching is also an advantage, thus making both A and B incorrect. D is incorrect. Since in-plane needle advancement can visualize the shaft of the needle, monitoring of the depth of the needle insertion is easier. 5. D is correct. Heeling also results in a more parallel alignment of the probe face with the needle. Heeling is accomplished by pressing in on the edge of the transducer opposite the insertion site. Answers A, B, and C improve ultrasound image in different ways. A is incorrect. Hydrolocation helps ascertain needle tip location by injecting small volumes of dextrose or local anesthetic (LA). B is incorrect. Moving the insertion site further away from the probe is another mechanism that is used to accomplish a more parallel alignment. A more parallel alignment of the needle to the face of the probe allows for more echoes to be transmitted back to the transducer, thus resulting in a superior image. C is incorrect. Greater scattering of ultrasound beams improves image. This is accomplished by the use of echogenic needles. The etching on the surfaces of these needle shafts causes the increased scattering, thus more echogenic image of the needle. 6. A is correct. Commonly, fascial planes lie just superficial or adjacent to the nerve target. The motion of advancing the needle without approaching tangentially may actually result in the needle thrusting forward and encountering the nerve if the sudden give of the fascial plane is not anticipated. B is incorrect. Temporarily steepening the needle angle may permit an easier and more controlled passage through fascial layers. C is incorrect. Passing through tough fascial layers often makes the needle out-of-plane. D is incorrect. Approaching the nerve tangentially does not prevent forward motion of the needle. 7. B is correct. Hydrodissection is deliberate injection of fluid into tissue planes that can be utilized to separate structures allowing better clarity in imaging. A is incorrect. Hydrodissection does not advance the needle but creates a fluid pocket into which one can advance the needle. C is incorrect. Hydrodissection does not prevent a paresthesia as the needle can still come into contact with the nerve, but it does improve visualization.

19/04/19 10:45 AM

CHAPTER 29

D is incorrect. Hydrodissection does not speed the onset of a nerve block. Only the type of LA, dose, and concentration can do so. 8. C is correct. A motor response was not elicited 25% of the time, despite visualization of the ultrasound needle, limiting the usefulness of a combined ultrasound and nerve stimulator technique at times for peripheral nerve blockade. A, B, and D are incorrect. See explanation for answer C. 9. D is correct. Real-time imaging of LA injection in small aliquots permits assessment of correct disposition of the LA around specific nerves, increasing the chances of a successful blockade. A is incorrect. LAST is less likely but still possible if high doses of concentration of LAs are placed or placement of drug is incorrect. B is incorrect. Small vessels are not always identified with Doppler or ultrasound, and aspiration before injection is a must to prevent intravascular injection despite the use of ultrasound.

Hadzic_Ch29_p151-154.indd 153

Introduction to Ultrasound-Guided Regional Anesthesia

153

C is incorrect. Errant needle placement can occur, which does not prevent paresthesia or nerve injury. 10. D is correct. Injecting LA in small aliquots allows the user to identify proper placement of LA, make adjustments, and monitor for toxicity. A is incorrect. LA in muscle or fascial layers above or below the desired nerve or plane sequester LA and lead to failed blocks. B is incorrect. Ultrasound and nerve stimulator techniques improve safety of blocks but do not improve nerve localization. C is incorrect. LA should be observed at the tip to ensure proper placement of LA.

Suggested Reading Hadzic A. Introduction to ultrasound-guided regional anesthesia. In: Orebaugh SL, Kirkham KR, eds. Textbook of Regional Anesthesia and Acute Pain Management. 2nd ed. New York, NY: McGraw-Hill Publishing; 2017:chap 30.

19/04/19 10:45 AM

This page intentionally left blank

Hadzic_FM_p00i-pxx.indd 6

22/04/19 6:40 PM

Section 2

Ultrasound-Guided Head and Neck Nerve Blocks Chapter 30

Hadzic_Ch30_p155-160.indd 155

Nerve Blocks of the Face  157

19/04/19 10:47 AM

This page intentionally left blank

Hadzic_FM_p00i-pxx.indd 6

22/04/19 6:40 PM

30 Nerve Blocks of the Face Jan Boublik

QUESTIONS DIRECTIONS: Choose the one best response to each question. 1. What of the following statements is true regarding trigeminal nerve block? A. It is a purely sensory block. B. It is the primary therapy for patients with trigeminal neuralgia. C. It can be performed distally and proximally. D. It carries preganglionic and postganglionic fibers. 2. What statement is true regarding landmark-based superficial trigeminal nerve blocks? A. The ophthalmic nerve (V1) enters the orbit and divides into its branches prior to exiting the orbit. B. 3–5 mL of local anesthetic (LA) should be injected per nerve. C. The terminal branch of V2, the infraorbital nerve, can be blocked both intra- and extra-orally. D. The most common complication of the superficial trigeminal nerve block is penetration of the foramen. 3. Which of the following statements is true about ultrasound-guided (USG) superficial trigeminal nerve block? A. Doppler imaging is not helpful. B. Probe orientation is always sagittal. C. It offers no advantages compared to landmark-based techniques. D. Bilateral injections of the frontal nerve may be necessary because of the cross-innervation of the forehead areas. 4. Complications of superficial nerve blocks of the face include: A. Hematoma B. Facial artery vasospasm C. Infections D. Nerve damage

5. Which statement is true regarding regional blocks of the nose? A. Complete analgesia and anesthesia can be achieved. B. Bilateral blocks are required in most cases. C. They involve blockade of all branches of the trigeminal nerve. D. They are virtually free of complications and side effects. 6. Regional blockade of the ear: A. Is solely provided by blocking terminal nerves of the trigeminal nerve B. Can achieve total analgesia of the external ear by blockade of the auriculotemporal, greater auricular, and lesser auricular nerve C. Is a useful analgesic option for ear and tympanomastoid surgery D. Can cure hiccups 7. Greater occipital nerve (GON) block: A. Is a versatile option for both chronic pain management and perioperative regional anesthesia B. Has the same success rate with landmark-based and ultrasound-guided techniques C. Is performed without the need for monitoring for vascular structure D. Is easy because of the constant location of the GON at the horizontal level of the greater occipital protuberance and mastoid 8. Which of the following statements is true regarding the scalp block? A. Requires blockade of five nerves from the trigeminal nerve and cervical rami B. Can be performed both as infiltrative anesthesia and with targeted nerve blocks C. Should only be performed in adults given the potential of local anesthetic (LA) toxicity D. Epinephrine should not be used given the risk of ischemia.

157

Hadzic_Ch30_p155-160.indd 157

19/04/19 10:47 AM

158

PART 3

Clinical Practice of Regional Anesthesia

9. Which of the following statements is true regarding the block of the maxillary nerve? A. All branches arise from the pterygopalatine fossa. B. It is equivalent for the ultrasound-guided and classic landmark techniques. C. Nerve stimulation is of little help. D. It is a versatile tool for perioperative analgesia in adult and pediatric anesthesia. 10. The mandibular nerve block: A. Blocks the smallest branch of the trigeminal nerve B. Provides sensory and motor anesthesia to both the anterior and posterior branches of the nerve C. Can lead of transient facial paralysis D. Using the classic landmark technique involves placement of the needle as low as possible in the space between the zygomatic arch and the center of the mandibular notch

ANSWERS AND EXPLANATIONS 1. C is correct. The trigeminal nerve block can be performed either distally at the respective exit points from the facial bones (V1–V3) or proximally at their emergence from the cranium (V2–V3). A is incorrect. The trigeminal nerve is a mixed sensory and motor nerve. While it is predominantly sensory, it does also carry the motor fiber for the muscles of mastication in its mandibular branch (V3). B is incorrect. Blockade of the trigeminal ganglion is a treatment for patients with trigeminal neuralgia who failed pharmacological therapy. A blockade with local anesthetic (LA) is typically used as a diagnostic test prior to neurolysis. D is incorrect. The trigeminal nerve is comprised of purely postganglionic fibers. Blockade of the trigeminal ganglion achieves blockade of preganglionic and postganglionic fibers. 2. C is correct. In the extra-oral approach, the needle is inserted perpendicular to the skin while palpating the infraorbital foramen until bony resistance is encountered. The needle is then advanced in a caudal and medial direction while keeping a finger on the foramen to prevent cephalad advancement. For the intraoral approach, the target landmark is the intersection of a vertical line through the papilla and a horizontal line through the alae of the nose. The buccal mucosa is entered between the canine and first molar and the needle advanced in an upward and outward direction. A is incorrect. The ophthalmic nerve divides into its three branches (lacrimal, frontal, and nasociliary) prior to entering the orbit. B is incorrect. Only 0.5 mL are necessary to achieve a successful block; 3–5 mL carry a risk of generalized seizures and/or epidural and subarachnoid anesthesia given the proximity of blood vessels.

Hadzic_Ch30_p155-160.indd 158

D is incorrect. Penetration of the foramen is relatively rare; persistent paresthesia and hematoma formation are far more common. 3. D is correct. The forehead has a rich network of crossinnervation from both sides. Thus, a supplemental block of the contralateral side is often required. A is incorrect. Doppler imaging may be helpful in detecting small vessels next to the nerve. Further, it may help delineate the spread of local anesthetic (LA) in real time. B is incorrect. While all foramina are located about 2.5 cm from the midline in a line running vertically roughly through the pupil, several different transducer positions and orientations are needed. For the supraorbital notch, a transverse orientation and a cephalad to caudal direction are needed. The infraorbital notch requires scanning in a sagittal plane from medial to lateral along the lower orbital margin. Lastly, the mental foramen, a transverse plane, and scanning from caudal in a cephalad direction yields the best success. C is incorrect. There are several potential advantages to using the USG superficial trigeminal nerve technique.1 Real-time guidance, ability to use Doppler to avoid blood vessels, and less needling likely lead to lower incidences of hematomas and fewer nerve injuries. 4. A is correct. Ecchymosis and hematoma occur given the vascularity of the face. Application of direct pressure and use of small-bore (25/27G) needles decrease the likelihood and severity. B is incorrect. Facial artery vasospasm with skin blanching and mucosal ischemia can be due to two mechanisms: a) the epinephrine vasoconstrictor effect on the alpha-adrenergic receptors of the blood vessels that irrigate the skin and mucosa, and b) a sympathetic effect from the impact of the needle on sympathetic fibers of the tunica adventitia of the artery, causing a vasospasm in the sympathetic plexus around the internal carotid artery. It is very rare though.2 C is incorrect. With the use of sterile and clean technique, the incidence is very rare. D is incorrect. Nerve damage is actually pretty rare as most superficial nerves of the face are small and contain a relatively large amount of connective tissue compared to neural structures. If present, they are in most instances transient with complete resolution. 5. B is correct. Bilateral nerve blocks are required in most cases given the complex innervation and high degree of procedures close to the midline involving nasal surgery. A is incorrect. Frequent supplementation by topical application or surgical infiltration is necessary given the complex innervation of the nasal cavity even in skilled hands. C is incorrect. Only ophthalmic (V1) and maxillary nerve (V2) branches such as the nasociliary, supratrochlear (V1), infraorbital, and nasal and nasopalatine branches of the maxillary nerve (V2) are needed.

19/04/19 10:47 AM

CHAPTER 30

D is incorrect. While generally very safe when performed correctly, transient minor complications such as ecchymosis and hematoma at the puncture site, and diplopia from blockade of the superior oblique muscle of the eye still occur and should be discussed with patients. 6. C is correct. Blockade of all these nerves can achieve opioid-free analgesia or at least significant decrease, leading to less postoperative nausea and vomiting3 and be useful in a variety of surgeries in and around the ear. A is incorrect. Blockade of the ear requires blockade both of branches of the trigeminal nerve (auriculotemporal branch of V2, upper-anterior two-thirds ear), cervical plexus (greater auricular nerve-anterior skin angle mandible, lobe and lower posterior auricle), and lesser occipital nerve (anterior roots C2 and 3—upper earlobe and lateral occiput). B is incorrect. In addition to the nerve outlined in A, blockade of the auricular branch of the vagus nerve is needed for complete analgesia.4 Blockade of all these nerves can achieve opioid-free analgesia or at least significant decrease, leading to less postoperative nausea and vomiting.3 D is incorrect. Phrenic nerve block is a treatment option for intractable hiccups AND a possible complication of peripheral nerve blockade. However, ultrasound allows for precise targeting of the phrenic nerve. 7. A is correct. The GON block is a great option in a variety of settings including neurosurgery,5 obstetrics,6 and chronic pain.7 B is incorrect. In a cadaveric study by Greher et al,8 a novel sonographic approach at the C2 level superficial to the obliquus capitis muscle, yielded a 100% success rate compared to 80% with the traditional landmark-based technique. C is incorrect. The GON block is very safe given its superficial location. However, intravascular injection is possible given the proximity of the occipital and vertebral artery. Thus, careful aspiration prior to injection and ideally imaging with ultrasound are recommended. D is incorrect. There is considerable variation, 1.5–7.5 cm from the midline for the GON at the level of the occipital protuberance and mastoid process. It may be part of the explanation of the higher success rate in the study by Greher.8 8. B is correct. Infiltrative techniques are effective given the superficial nature of the nerves innervating the scalp. The only “downside” is that larger volumes around 30 mL are needed to achieve a complete ring block of the scalp. Alternatively, more targeted selective regional technique of the nerves outlined in answer A can be performed. A is incorrect. The following seven branches need to be blocked for a complete scalp block: • Cervical spinal nerves: greater and lesser occipital nerve and great auricular nerve

Hadzic_Ch30_p155-160.indd 159

Nerve Blocks of the Face

159

• Trigeminal nerve: ophthalmic nerve (frontal, supratrochlear, and supraorbital nerves), auriculotemporal, and zygomaticotemporal nerves C is incorrect. Scalp block is an effective analgesic and anesthetic technique for both pediatric and adult patients. It is extremely versatile ranging from anesthesia for scalp laceration, foreign body removal, neurosurgical analgesia and anesthesia,9,10 and chronic pain management. D is incorrect. Epinephrine should be included in LA given the vascularity of the scalp to avoid LA toxicity, particularly with infiltrative and landmark-based techniques. 9. D is correct. The maxillary nerve block is of great use for cleft lip repairs and major surgery of the maxilla, ethmoid sinus, and pterygopalatine or infratemporal fossa.11 A is incorrect. The branches exit the cranium at the pterygopalatine fossa: pterygopalatine nerve, zygomatic nerve, posterior superior alveolar nerve, and infraorbital nerve. The middle meningeal nerve, innervating the dura mater, however, does not. B is incorrect. While both techniques have been used with good clinical success, the ultrasound-guided technique allows for direct visualization of the internal maxillary artery, needle tip, and monitoring of the spread of the LA.12 C is incorrect. Nerve stimulation can be of help as the disappearance of the temporal muscle contraction signifies that one has entered the pterygopalatine fossa. 10. C is correct. Transient facial paralysis13 can occur either immediately or delayed (up to 24 hours), lasting for weeks or months. Exclusion of facial paralysis of central origin is mandatory, and a referral to a neurologist to rule out pathologic entities causing facial paralysis (eg, Ramsay Hunt syndrome, Lyme disease, Guillain-Barré syndrome, sarcoidosis) is recommended. Reported therapy is anti-inflammatory medication (eg, prednisolone), reassurance, and therapies aimed at prevention of complications (mainly ophthalmic) such as eye patches if the patient is unable to close his or her eye. A is incorrect. The maxillary nerve is the largest branch of the trigeminal nerve. B is incorrect. The anterior branch provides motor innervation to the masseter, temporalis, mylohyoid, pterygoid, tensor tympani, and palate muscles as well as some sensory coverage via the buccal nerve. The larger, posterior branch is purely sensory and comprised of the lingual, auriculotemporal, and inferior alveolar/mental nerve. D is incorrect. In the traditional, landmark-based study, the needle is to be placed as high as possible in the space between the zygomatic arch and the center of the mandibular notch to avoid puncture.

References 1. Tsui BC. Ultrasound imaging to localize foramina for superficial trigeminal nerve block. Can J Anaesth. 2009 Sep;56(9):704-706.

19/04/19 10:47 AM

160

PART 3

Clinical Practice of Regional Anesthesia

2. Kronman JH, Giunta JL. Reflex vasoconstriction following dental injections. Oral Surg Oral Med Oral Pathol. 1987;63: 542-544. 3. Suresh S, Barcelona SL, Young NM, Seligman I, Heffner CL, Coté CJ. Postoperative pain relief in children undergoing tympanomastoid surgery: is a regional block better than opioids? Anesth Analg. 2002 Apr;94(4):859-862. 4. Giles WC, Iverson KC, King JD, Hill FC, Woody EA, Bouknight AL. Incision and drainage followed by mattress suture repair of auricular hematoma. Laryngoscope. 2007 Dec;117(12):2097-2099. PubMed PMID: 17921905. 5. King MR, Anderson TA. Ultrasound-guided peripheral nerve blocks for ventricular shunt revision in children. A&A Case Rep. 2014 Dec 15;3(12):157-159. 6. Akyol F, Binici O, Çakır M. Ultrasound-guided bilateral greater occipital nerve block for the treatment of postdural puncture headache. Turk J Anaesthesiol Reanim. 2014 Feb;42(1):40-42. doi: 10.5152/TJAR.2013.59. Epub 2013 Aug 29. PubMed PMID: 27366386; PubMed Central PMCID: PMC4894104. 7. Vanderhoek MD, Hoang HT, Goff B. Ultrasound-guided greater occipital nerve blocks and pulsed radiofrequency ablation for diagnosis and treatment of occipital neuralgia. Anesth Pain Med. 2013 Sep;3(2):256-259. 8. Greher M, Moriggl B, Curatolo M, Kirchmair L, Eichenberger U. Sonographic visualization and ultrasound-guided blockade of the greater occipital nerve: a comparison of two selective

Hadzic_Ch30_p155-160.indd 160

techniques confirmed by anatomical dissection. Br J Anaesth. 2010 May;104(5):637-642. 9. Nguyen A, Girard F, Boudreault D, et al. Scalp nerve blocks decrease the severity of pain after craniotomy. Anesth Analg. 2001 Nov;93(5):1272-1276. 10. Pinosky ML, Fishman RL, Reeves ST, et al. The effect of bupivacaine skull block on the hemodynamic response to craniotomy. Anesth Analg. 1996 Dec;83(6):1256-1261. 11. Mesnil M, Dadure C, Captier G, et al. A new approach for perioperative analgesia of cleft palate repair in infants: the bilateral suprazygomatic maxillary nerve block. Paediatr Anaesth. 2010 Apr;20(4):343-349. 12. Sola C, Raux O, Savath L, Macq C, Capdevila X, Dadure C. Ultrasound guidance characteristics and efficiency of suprazygomatic maxillary nerve blocks in infants: a descriptive prospective study. Paediatr Anaesth. 2012 Sep;22(9):841-846. 13. Tzermpos FH, Cocos A, Kleftogiannis M, Zarakas M, Iatrou I. Transient delayed facial nerve palsy after inferior alveolar nerve block anesthesia. Anesthesia Progress. 2012;59(1):22-27.

Suggested Reading Hadzic A. Nerve blocks of the face. In: Sola C, Dadure C, Choquet O, Capdevila X, eds. Textbook of Regional Anesthesia and Acute Pain Management. 2nd ed. New York, NY: McGraw-Hill Publishing; 2017:chap 31.

19/04/19 10:47 AM

Section 3

Ultrasound-Guided Nerve Blocks for the Upper Extremity Chapter 31A

Ultrasound-Guided Cervical Plexus Block  163

Chapter 31B

Ultrasound-Guided Interscalene Brachial Plexus Block  167

Chapter 31C

Ultrasound-Guided Supraclavicular Brachial Plexus Block  169

Chapter 31D

Ultrasound-Guided Infraclavicular Brachial Plexus Block  173

Chapter 31E

Ultrasound-Guided Axillary Brachial Plexus Block  177

Chapter 31F

Ultrasound-Guided Blocks at the Elbow  181

Chapter 31G

Ultrasound-Guided Wrist Block  185

Hadzic_Ch31A_p161-166.indd 161

22/04/19 1:55 PM

This page intentionally left blank

Hadzic_FM_p00i-pxx.indd 6

22/04/19 6:40 PM

31A Ultrasound-Guided Cervical Plexus Block Sherif Abbas

QUESTIONS DIRECTIONS: Choose the one best response to each question. 1. In which location is local anesthetic injected for a superficial (intermediate) cervical plexus block? A. Superficially to the deep cervical fascia and not deep to the prevertebral fascia B. Superficially to the deep cervical fascia and not between the investing layer of the deep cervical fascia and the prevertebral fascia C. Between the investing layer of the deep cervical fascia and the prevertebral fascia, and not deep to the prevertebral fascia D. Medial to the sternocleidomastoid muscle (SCM) and deep to the prevertebral fascia 2. The best possible target location for depositing local anesthetic during superficial cervical plexus technique is: A. Medial to the carotid artery B. Lateral to the brachial plexus C. In the vicinity of nerve roots C2–C4 D. Superficial to the carotid artery and internal jugular vein within the sternocleidomastoid muscle (SCM) 3. Figure 31A–1 shows the anatomy of the deep cervical plexus and its main branches and anastomoses. Identify the supraclavicularis nerve. A. A B. B C. C D. D

A B C D

Haad H dzziicc - Lancea/ NYSORA FIGURE 31A–1  Anatomy of the deep cervical plexus and its main branches and anastomoses.

4. In Figure 31A–2, what structure is marked with the number 5? A. Suprascapular nerve B. Greater auricular nerve C. Phrenic nerve D. Long thoracic nerve

163

Hadzic_Ch31A_p161-166.indd 163

22/04/19 1:55 PM

164

PART 3

Clinical Practice of Regional Anesthesia

6. What is the target location for local anesthetic injection during superficial cervical plexus block? A. Under the investing layer of the deep cervical fascia B. Medial to the internal jugular C. Above the deep cervical fascia D. Lateral to the middle scalene muscle 7. In Figure 31A–4, identify the location of the superficial cervical plexus. A. A B. B C. B D. D

GE Le

D B Middle scalene

FIGURE 31A–2  Anatomy of the cervical plexus. The cervical plexus is seen emerging behind the posterior border of the sternocleidomastoid muscle at the intersection of the muscle with the external jugular vein.

5. When blocking the cervical plexus, the expected sensory distribution is marked by which locations shown in Figure 31A–3? A. 1 + 2 B. 2 + 3 + 4 C. 1 + 3 + 4 D. 2 + 3

Sternocleidomastoid

A Nerve

C Anterior scalene

Nerve Nerve 2

3

FIGURE 31A–4  Sonographic image of the brachial plexus providing a general understanding of overall regional sono-anatomy.

8. Which of the following is not an indication for a superficial cervical plexus block? A. Superficial neck surgery B. Carotid endarterectomy C. Tennis elbow surgery D. Clavicle fracture

1

3

2

4

9. In ultrasound-guided technique, the sono-anatomy landmarks needed to identify the best level to target the cervical plexus are: A. Mastoid process, sternocleidomastoid muscle, C4 transverse process B. Mastoid process, sternocleidomastoid muscle, C6 transverse process C. Carotid, internal jugular, sternocleidomastoid muscle D. Carotid, internal jugular, sternocleidomastoid muscle, brachial plexus between the anterior and middle scalene muscles

FIGURE 31A–3  Sensory distribution of the head and neck.

Hadzic_Ch31A_p161-166.indd 164

22/04/19 1:55 PM

CHAPTER 31A

10. In Figure 31A–5, which lettered circle describes the posterior border of the sternocleidomastoid muscle and the site of skin infiltration? A. A B. B C. C D. D

Ultrasound-Guided Cervical Plexus Block

165

2. C is incorrect. The goal of the ultrasound (US)-guided technique of superficial cervical plexus block is to deposit local anesthetic within the vicinity of the sensory branches of the nerve roots C2, C3, and C4. A is incorrect. This will be medial to the carotid artery and closer to the trachea and possible part of the thyroid gland. B is incorrect. This will be too far from the location of the cervical plexus in relation to the brachial plexus. D is incorrect. The plexus can be visualized as a small collection of hypoechoic nodules (honeycomb appearance or hypoechoic [dark] oval structures) immediately deep or lateral to the posterior border of the SCM and not within it. 3. D is correct. This is the supraclavicularis nerve. A is incorrect. This is the great auricular. B is incorrect. This is the lesser occipital. C is incorrect. This is the transverse cervical.

A B

C

D

4. B is correct. Number 5 is the greater auricular nerve. A, C, and D are incorrect. Numbers 1, 2, 3, and 4 are sternocleidomastoid muscle, mastoid process, clavicle, and external jugular vein, respectively. 5. B is correct. The expected distribution of the cervical plexus is covered by the following branches: transverse cervical nerves, cutaneous cervical nerve and the supraclavicularis nerves. A is incorrect. Region 1 is innervated by the minor occipital nerve which will not be covered by blocking the cervical plexus. Region 2 is partly correct but not complete. C is incorrect. Region 1 is not covered by cervical plexus.

FIGURE 31A–5  Anatomy of the nerve supply to head, the muscles of the neck and proximal shoulder.

D is incorrect. Region 3 and 4 do not show a complete expected distribution of the cervical plexus block.

11. The length and the gauge of the needle best used during the superficial cervical plexus block is: A. 4 cm, 18-gauge needle B. 5 cm, 18-gauge needle C. 8 cm, 22- to 25-gauge needle D. 5 cm, 22- to 25-gauge needle

6. A is correct. To perform a cervical plexus block the needle is inserted through the skin, platysma, and investing layer of the deep cervical fascia and local anesthetic is injected under it. Figure 31A–6 is the sonographic example that shows the best spread of the local anesthetic.

ANSWERS AND EXPLANATIONS 1. C is correct. For the superficial (intermediate) cervical plexus block, the injection is made between the investing layer of the deep cervical fascia and the prevertebral fascia, whereas for the deep cervical plexus block, local anesthetic is deposited deep to the prevertebral fascia. A, B, and D are incorrect. These options are not correct locations of local anesthetic injection to perform an intermediate cervical plexus block.

Hadzic_Ch31A_p161-166.indd 165

Cephalad

12. Performing ultrasound-guided superficial plexus block is best done by: A. High-frequency linear transducer B. Low-frequency convex transducer C. Low-frequency sector transducer D. Low-frequency linear transducer

FIGURE 31A–6  Cervical plexus (longitudinal view): desired spread of local anesthetic under the deep cervical fascia to block the cervical plexus (CP).

22/04/19 1:55 PM

166

PART 3

Clinical Practice of Regional Anesthesia

B is incorrect. Medial to internal jugular will be too close to the carotid and far from the cervical plexus.

B is incorrect. Mastoid process is not seen during the ultrasound scan of the cervical plexus.

C is incorrect. Above the deep cervical plexus will be mostly subcutaneous and not close enough to the nerves of cervical plexus.

C is incorrect. All mentioned are correct but missing an important sono-anatomical land mark (the roots of the brachial plexus).

D is incorrect. This will be too far lateral from the correct location of the cervical plexus.

10. C is correct. As seen in Figure 31A–5, targeting location C will cover the transverse cervical nerve, accessory nerve, supraclavicular nerve, and nerve supply to clavicle.

7. A is correct. Location A shows best target endpoint of the cervical plexus, which is located superficially to the deep cervical fascia and underneath the lateral border of the sternocleidomastoid muscle (SCM). B is incorrect. This will be lateral to the middle scalene muscle and too far from the cervical plexus. C is incorrect. This will be too medial and far from the cervical plexus. D is incorrect. This is mostly subcutaneous and not lateral or deep enough to reach the cervical plexus. 8. C is correct. This is not an indication for a superficial cervical plexus block. Nerve supply to the elbow joint includes branches of all major nerves of the brachial plexus that cross the joint: musculocutaneous, radial, median, and ulnar nerves. A, B, and D are incorrect. All these are indications for a superficial cervical plexus block. 9. D is correct. During an ultrasound-guided cervical plexus block, to target the best level for injection, an effort must be made to identify the following ultrasound landmarks: Carotid artery, internal jugular vein, sternocleidomastoid muscle, brachial plexus between the anterior and middle scalene muscles. Figure 31A–7 shows the relationship between all of these structures.

Lateral

A is incorrect. Mastoid process and C4 transverse process are not seen during dynamic scan of the cervical plexus.

A is incorrect. Injection targeting region A will block the lesser occipital nerve. B is incorrect, injection targeting region B will block the greater auricular nerve. D is incorrect. Injection targeting region D will not provide complete anesthesia of the cervical plexus.. 11. D is correct. The equipment needed for a cervical plexus block includes the following: • Ultrasound machine with a linear transducer (8–18 MHz), sterile sleeve, and gel • Standard nerve block tray • A 10-mL syringe containing local anesthetic • A 5 cm, 22- to 25-gauge needle attached to low-volume extension tubing • Sterile gloves A is incorrect. 4 cm length might be used but 18G needle would be too thick. B is incorrect. Length is correct but 18G needle would be too thick. C is incorrect. Needle gauge is correct but needle would be too long. 12. A is correct. The equipment needed for a cervical plexus block includes ultrasound machine with a linear transducer. Most newer transducers provide a larger range of frequencies, eg, 8–18 MHz, however, understanding the location of the cervical plexus requires good knowledge of ultrasound and that selecting a high frequency transducer means that choosing the higher range in that probe is necessary to have a better view of the plexus, therefore, if the ultrasound machine allows the manual selection of the frequency, the range of 10–15 MHz or superficial setting, should be utilized to better view the cervical plexus. B is incorrect. Convex transducer is not used for any superficial nerve scan. C is incorrect. Sector probes are used in cardiac and some abdominal scans. D is incorrect. Using a low frequency transducer or a lower range of frequencies in any given linear probe will not provide good imaging of superficial structures.

FIGURE 31A–7  Superficial cervical plexus (transverse view): needle path (1) and position to block the cervical plexus (CP). The needle is seen positioned underneath the lateral border of the sternocleidomastoid muscle (SCM) and superficial to the prevertebral fascia with the transducer in a transverse position. ASM, anterior scalene muscle; CA, carotid artery; MSM, middle scalene muscle.

Hadzic_Ch31A_p161-166.indd 166

Suggested Reading Hadzic A. Ultrasound-guided cervical plexus block. In: Bendtsen TF, Abbas S, Chan V, eds. Textbook of Regional Anesthesia and Acute Pain Management. 2nd ed. New York, NY: McGraw-Hill Publishing; 2017:chap 32A.

22/04/19 1:55 PM

31B Ultrasound-Guided Interscalene Brachial Plexus Block Philippe Gautier

QUESTIONS DIRECTIONS: Choose the one best response to each question. 1.  Which of the following is true regarding interscalene block? A. Interscalene block is indicated for surgery of the shoulder, elbow, and wrist. B. Interscalene block is indicated for shoulder and upper arm procedures. C. Interscalene block concerns the whole brachial plexus. D. The ulnar nerve is blocked with an interscalene block.

4.  Which of the following is true regarding the phrenic nerve? A. An interscalene block with 10 mL will almost never result in distribution of local anesthetic out of the interscalene groove. B. Lowering the volume and performing a low interscalene block are the two most common propositions to minimize the incidence of phrenic palsy. C. An interscalene brachial plexus block can be considered in patients with respiratory insufficiency. D. A medial-to-lateral approach is more suitable to avoid a direct trauma of the phrenic nerve. E. The phrenic function is assesed using the B-mode on the medioclavicular line.

2.  When an in-plane approach is used, why should the needle be positioned between the two scalene muscles at the level of C5 and C6? A. This approach is the best way to avoid a puncture of the vertebral artery. B. The C5-C6 nerve roots are the only contributions targeted by this block. C. An in-plane approach definitely avoids direct trauma of collateral terminal branches. D. The lateral-to-medial approach still presents a risk of puncturing the phrenic nerve.

5.  Selective suprascapular and axillary nerve blocks: A. Are branches of the brachial plexus (upper trunk). B. Selectively block the shoulder. C. The axillary nerve is located close to the humeral artery. D. Do not require general anesthesia.

3.  The following statements regarding the interscalene brachial plexus block are true, except: A. The brachial plexus is typically visualized at a depth of 1-3 cm. B. Provides reliable anesthesia and analgesia for shoulder and upper arm surgery. C. The brachial plexus at the interscalene level is seen medial to the carotid artery and internal jugular vein, between the anterior and middle scalene muscles. D. External landmarks for this block are: The clavicle, sternocleidomastoid muscle, and external jugular vein. E. Needle should be inserted in-plain from a lateralto-medial direction.

1.  B is correct. The interscalene block results in a spread of local anesthesia around C5-C7 and superior and middle trunk. This will cover all the peripheral nerves innervating the shoulder. A is incorrect. Elbow and wrist are innervated by the lower trunk (C8-T1) of the brachial plexus which is not covered by an interscalene block. C is incorrect. C8-T1 for the inferior trunk are typically missed. D is incorrect. The ulnar nerve originates from the C8-T1 nerve roots that are typically missed with an interscalene block.

ANSWERS AND EXPLANATIONS

167

Hadzic_Ch31B_p167-168.indd 167

22/04/19 1:58 PM

168

PART 3

Clinical Practice of Regional Anesthesia

2.  A is correct. The vertebral arteries arise from the subclavian arteries and enter deep to the transverse process at the level of C6 or occasionally (in 7.5% of cases) at the level of C7. The vertebral arteries run upward and backward between the Longus colli and the scalenus anterior. This position at the posterior aspect of the anterior scalene muscle is essential: a needle located in the middle of the interscalene groove is away from the vertebral artery. The Doppler should be used to differentiate the vertebral artery from C7, with C7 being lateral to the vertebral artery. B is incorrect. Depending of the level of punction, the needle is positioned between C5 and C6, between C6 and C7, or between the superior and the middle trunk. The local anesthetic solution will spread around these structures. C is incorrect. If an in-plane approach is chosen, the needle will cross the middle scalene muscle. Two nerves are usually running in this muscle: the long thoracic nerve and the dorsal scapular nerve. A trauma of the long thoracic nerve will affect the serratus anterior and result in a winged scapula. A trauma of the dorsal scapular nerve will affect the rhomboid muscles, which pull the scapula toward the spine and levator scapulae muscle, which elevates the scapula. D is incorrect. The phrenic nerve runs over the anterior scalene muscle. 3.  C is not true, so option C is correct. The brachial plexus at the interscalene level is seen lateral to the carotid artery, and internal jugular vein. A is true, so option A is incorrect. At the interscalene level, the depth at which you can typically identify the brachial plexus is 1–3 cm. B is true, so option B is incorrect. The interscalene approach to brachial plexus supplies the sensory and motor innervation for the shoulder and upper arm. The supraclavicular branches of the cervical plexus that supply the skin over the acromion and clavicle are generally also blocked due to the proximal and superficial spread of the local anesthetic. D is true, so option D is incorrect. The knowledge of external landmarks substantially facilitates and shortens the time to obtain the view necessary for block performance. The transducer is positioned behind the clavicular head of the sternocleidomastoid muscle (SCM) and over the external jugular vein. E is true, so option E is incorrect. The lateral-to-medial insertion is often chosen to prevent injury to the phrenic

Hadzic_Ch31B_p167-168.indd 168

nerve, which is typically located anteriorly to the anterior scalene muscle, although one should be aware that the dorsal scapular nerve and the long thoracic nerve usually runs through the middle scalene muscle and could potentially be injured as well. 4.  B is correct. When a low interscalene block is performed, the phrenic nerve has already turned around the anterior scalene muscle. The distance between the needle and the phrenic nerve is progressively increasing, so that the risk to reach the phrenic nerve is decreasing. A reduction of the LA volume helps decrease this side effect. A is incorrect. An interscalene block with 10 mL will always result in distribution of local anesthetic out of the interscalene groove. A phrenic nerve paralysis should always be considered. C is incorrect. A phrenic paresia must always be considered. An interscalene block should be avoided. D is incorrect. The phrenic nerve runs at the anterior aspect of the anterior scalene, so that a medial-to-lateral approach represents a risk of direct trauma of the nerve. E is incorrect. The diaphragmatic function is better observed on the anterior axillary line. 5.  B is correct. These nerves selectively block the shoulder. The suprascapular nerve supplies most of the posterior aspect of the shoulder joint. The subscapular and axillary nerves, the lateral pectoral nerve and occasionally the musculocutaneous nerve supply the anterior aspect of the joint. A is incorrect. The suprascapular nerve arises from the upper trunk and the axillary nerve from the posterior cord of the brachial plexus. C is incorrect. The axillary nerve runs along the posterior circumflex humeral artery. D is incorrect. These blocks must be associated with general anesthesia.

Suggested Readings Hadzic A. Ultrasound-guided interscalene brachial plexus block. In: Gautier PE, Vandepitte C, Gadsden J, eds. Textbook of Regional Anesthesia and Acute Pain Management. 2nd ed. New York, NY: McGraw-Hill Publishing; 2017:chap 32B. Verenna A-M, Alexandru D, Karimi A., et al. Dorsal scapular artery variations and relationship to the brachial plexus, and a related thoracic outlet syndrome case. J Brachial Plex Peripher Nerve Inj. 2016;11(1):e21-e28. Published online 2016 May 10. doi: 10.1055/s-0036-1583756

22/04/19 1:58 PM

31C Ultrasound-Guided Supraclavicular Brachial Plexus Block Seppe Dehaene and Catherine Vandepitte

QUESTIONS DIRECTIONS: Choose the one best response to each question. 1. Regarding anatomy, which of the following statements is correct? A. The block is performed where divisions are formed from the trunks. B. The dorsal scapular artery crosses posterior to the middle scalene muscle. C. The subclavian artery passes anterior to the anterior scalene muscle. D. The superficial and lateral branches come from C8 and T1. 2. Regarding ultrasound appearance, which of the following statements is correct? A. The brachial plexus is seen as a hypoechoic structure deep to the artery. B. The brachial plexus is typically visualized at a 1–3 cm depth. C. The first rib is seen as a hyperechoic line with acoustic enhancement. D. The pleura is seen as a hyperechoic static line. 3. Regarding distribution, which part of the arm is never anesthetised by a supraclavicular block? A. Carpal bones B. Shoulder C. Skin of the proximal part of the medial side of the arm D. Ulnar part of the forearm 4. What type of equipment is typically used during a supraclavicular block? A. Curved transducer, 20–25 mL local anesthetic, 8 cm needle B. Linear transducer, 20–25 mL local anesthetic, 5 cm needle C. Linear transducer, 10–15 mL local anesthetic, 8 cm needle D. Linear transducer, 10–15 mL local anesthetic, 5 cm needle

Hadzic_Ch31C_p169-172.indd 169

5. For a supraclavicular brachial plexus block, ideally the patient is positioned: A. In lateral position with the patient’s head in neutral position B. In semi-sitting position with patient’s head turned to the ipsilateral side C. With the head turned to the contralateral side and the ipsilateral arm abducted D. With the head turned to the contralateral side and reaching to knee on the ipsilateral side 6. For a supraclavicular brachial plexus block, after the ultrasound probe is placed in the transverse plane immediately proximal to the clavicle, what statement is true? A. Clockwise rotation can help visualize the sheath containing the plexus. B. Color Doppler should be routinely used prior to needle insertion. C. Tilting the transducer caudally can help to obtain a cross-sectional view of the subclavian artery. D. All three statements are true. 7. Which of the follow is true? Multiple injections may: A. Allow for reduction in the required volume of local anesthesia B. Increase the speed of onset and the success rate C. Carry out a higher risk of nerve injury D. All three statements are true. 8. Which statement is true regarding dyspnea after supraclavicular block? A. A pneumothorax generally presents within 1 hour after the block is performed. B. A pneumothorax is seen in about 1% of the cases. C. Symptomatic hemidiaphragmatic paresis is seen in about 1% of the cases. D. The incidence of hemidiaphragmatic paresis in continuous supraclavicular block is as high as in continuous interscalene block.

169

19/04/19 11:24 AM

170

PART 3

Clinical Practice of Regional Anesthesia

9. A 52-year-old man with no significant medical history is planned for osteosynthesis of the humerus after a complex fracture. After ultrasound-guided needle placement, no motor response is seen. What are your next steps? A. Carefully advance the needle until motor response of the arm, forearm, or hand is seen (at 0.5 mA, 0.1 msec) before aspiration and slow injection of 1–2 mL of local anesthetic. B. Carefully aspirate and inject 1–2 mL of local anesthetic. When injection pressure is high, carefully advance the needle because you are most likely injecting in the fascia. C. Carefully aspirate and inject 1–2 mL of local anesthetic. When the injection displaces the plexus, give a total amount of 20–25 mL of local anesthetic. D. Carefully aspirate and inject 1–2 mL of local anesthetic. When the injection separates the trunks posterior to the subclavian artery, give a total amount of 20–25 mL of local anesthetic. 10. An obese woman with chronic obstructive pulmonary disease Gold III and history of acute myocardial infarction is scheduled for elbow surgery. Which of the following anesthetic approaches would you follow? A. Perform an ultrasound-guided supraclavicular block with double injection (between upper and middle trunk and below lower trunk) and > 25 mL of local anesthetic to avoid rescue general anesthesia. B. Perform an ultrasound-guided supraclavicular block with single injection between the upper and middle trunk to prevent placement near the pleura and a possible pneumothorax. Give a dose of > 25 mL so the local anesthetic will diffuse toward the lower trunk. C. Perform an ultrasound-guided supraclavicular catheter placement. Give an initial dose of 10 mL of local anesthetic. After 20–30 minutes, test the anesthesia of the arm and give small increments of local anesthetic if necessary. D. Perform an ultrasound-guided interscalene catheter placement. Give an initial dose of 15–20 mL of local anesthetic and start PCA at 5 mL/hour with an ondemand bolus of 5 mL every hour.

ANSWERS AND EXPLANATIONS 1. A is correct. Because the trunks and divisions of the brachial plexus are relatively close as they pass over the first rib, the extension and quality of anesthesia are favorable. B is incorrect. The dorsal scapular artery is often seen within the plexus C is incorrect. The subclavian artery passes between anterior and middle scalene muscle. D is incorrect. The superficial and lateral branches come from C5 to C7. 2. B is correct. See Figure 31C–1. A is incorrect. The brachial plexus is seen as a hypoechoic structure posterior and superficial to the artery.

Hadzic_Ch31C_p169-172.indd 170

Haadz H dzic zicc - Laan nce cea/ a/ NY YSSOR ORA FIGURE 31C–1  Supraclavicular brachial plexus block: transducer position just proximal the clavicle and needle insertion. The brachial plexus is very shallow at this location, typically 1–3 cm; therefore, inclination of the needle should be equally shallow. The image also shows the caudal tilt that is useful in obtaining best image of the plexus. (Images from NYSORA Continuing Medical Education.)

C is incorrect. The rib casts an acoustic shadow. Acoustic enhancement can be seen deep to the vascular structures. D is incorrect. Typical for the pleura is the dynamic sliding motion of the visceral pleura in synchrony with the patient’s respiration. 3. C is correct. The medial side of the forearm is innervated by the intercostobrachial nerve. Most frequently formed by (the posterior branch of) the lateral cutaneous branch of the second intercostal nerve. Sometimes in association with the first or third intercostal nerve. A is incorrect. The carpal bones are innervated by the radial nerve. B is incorrect. Spread is seen travelling cranially between the anterior and middle interscalene muscle, blocking the branches from C5-C7. D is incorrect. The ulnar part of the forearm is innervated by the medial cutaneous nerve of the forearm. 4. B is correct. A frequency of 10–12 MHz is ideal to visualize structures 2–4 cm deep. The brachial plexus is typically at 1–3 cm depth in the supraclavicular region. The shortest possible needle is most easy to manipulate. A is incorrect. The brachial plexus is typically at 1–3 cm depth. A curved transducer is used to identify deeper structures. A 5 cm needle is long enough and easier to manipulate. C and D are incorrect. 20–25 mL local anesthetic is typically used. It has been suggested that lower volumes can be used in older1 and in obese patients.2 5. D is correct. This will depress the clavicle slightly and allow better access to the structures in the anterolateral neck. A is incorrect. A slight lateral position can be used with the patients head turned to the contralateral side. B is incorrect. Turning the head to the contralateral side will give better access to the structures of the anterolateral neck. C is incorrect. Abduction of the ipsilateral arm will elevate the clavicle and shoulder and will block the access.

19/04/19 11:24 AM

CHAPTER 31C

6. A, B, and C are all true, so option D is correct With the patient in the proper position, the skin is disinfected and the transducer is positioned in the transverse plane immediately proximal to the clavicle, slightly posterior to its midpoint. Color Doppler should be routinely used prior to needle insertion to rule out the passage of large vessels (ie, dorsal scapular artery, transverse cervical artery, suprascapular artery) in the anticipated trajectory of the needle. The transducer is tilted caudally, as if to image the chest contents, to obtain a cross-sectional view of the subclavian artery.3 7. A, B, and C are all true, so option D is correct. Tran et al concluded that there were no significant advantages of a double injection over a single injection. Although the onset time was shorter, there was no significant difference in total anesthesia-related time because the performance time was longer.4 Arab et al on the other hand found a significantly higher success rate in sensory block after a tripleinjection technique than a single injection at 10, 15, and 20 minutes after injection.5 After 30 minutes, no significant difference was found. Tran found a significantly higher amount of needle passages in the double-injection group, which can lead to a higher risk of neural injury. 8. C is correct. Symptomatic hemidiaphragmatic paresis is seen in about 1% of the cases.6 A is incorrect. A pneumothorax typically has a delayed rather than immediate presentation. B is incorrect. The incidence of a pneumothorax is estimated at 0.04%.7 D is incorrect. Continuous interscalene block is more likely to provoke complete or partial hemidiaphragmatic paresis at 1 hour after admission to the post-anesthetic care unit and 24 hours after the surgery volume than continuous supraclavicular block.8,9 9. D is correct. See Figure 31C–2.

171

Ultrasound-Guided Supraclavicular Brachial Plexus Block

A is incorrect. Despite accurate needle placement, motor response may be absent. B is incorrect. The inability to initiate injection at an opening pressure of less than 15 psi may signal an intrafascicular injection. C is incorrect. An additional advancement may be required to see the spread within the connective tissue; often a small “pop” is felt. 10. C is correct. For pulmonary reasons, spread towards the phrenic nerve should be avoided. For cardiac reasons, an insufficient block and rescue general anesthesia should be avoided. Slow titration is best way to ensure this. The supraclavicular block reduces the risk of spread towards the phrenic nerve compared to an interscalene block. A and B are incorrect. The risk of phrenic nerve palsy is lower than with the interscalene block but cannot be reliably avoided. A higher volume of local anesthetic may be more likely to provoke phrenic paresis as suggested by Mogahed et al and Song et al.10,11 D is incorrect. Continuous interscalene block is more likely to provoke complete or partial hemidiaphragmatic paresis at 1 hour after admission to the post-anesthetic care unit and 24 hours after the surgery volume than continuous supraclavicular block.8,9

References 1. Pavičić Šarić J, Vidjak V, Tomulić K, Zenko J. Effects of age on minimum effective volume of local anesthetic for ultrasoundguided supraclavicular brachial plexus block. Acta Anaesthesiol Scand. 2013;57:761-766. 2. Gupta PK, Pace NL, Hopkins PM. Effect of body mass index on the ED50 volume of bupivacaine 0.5% for supraclavicular brachial plexus block. Br J Anaesth. 2010 Apr;104(4):490-495. doi:10.1093/bja/aeq017. 3. Murata H, Sakai A, Hadzic A, Sumikawa K. The presence of transverse cervical and dorsal scapular arteries at three ultrasound probe positions commonly used in supraclavicular brachial plexus blockade. Anesth Analg. 2012;115:470-473. 4. Tran DQ, Munoz L, Zaouter C, Russo G, Finlayson RJ. A prospective, randomized comparison between single- and double-injection, ultrasound-guided supraclavicular brachial plexus block. Reg Anesth Pain Med. 2009;34:420-424.

SA 1st Rib

Anterior

BP

MSM

Pleura Pleura

FIGURE 31C–2  The desired spread of local anesthetic (blueshaded areas) in two different needle positions to accomplish brachial plexus (BP) block. Local anesthetic should spread within the connective tissue sheath resulting in separation of the brachial plexus trunks posterior to the subclavian artery (SCA).

Hadzic_Ch31C_p169-172.indd 171

5. Arab SA, Alharbi MK, Nada EM, Alrefai DA, Mowafi HA. Ultrasound-guided supraclavicular brachial plexus block: single versus triple injection technique for upper limb arteriovenous access surgery. Anesth Analgesia. 2014;118(5):1120-1125. 6. Perlas A, Lobo G, Lo N, Brull R, Chan VW, Karkhanis R. Ultrasound-guided supraclavicular block: outcome of 510 consecutive cases. Reg Anesth Pain Med. 2009;34:171-176. 7. Abell DJ, Barrington MJ. Pneumothorax after ultrasound-guided supraclavicular block: presenting features, risk, and related training. Reg Anesth Pain Med. 2014 Mar-Apr;39(2):164-167. 8. Wiesmann T, Feldmann C, Müller HH, et al. Phrenic palsy and analgesic quality of continuous supraclavicular vs. interscalene plexus blocks after shoulder surgery. Acta Anaesthesiol Scand. 2016 Sep;60(8):1142-1151. doi:10.1111/aas.12732.

19/04/19 11:24 AM

172

PART 3

Clinical Practice of Regional Anesthesia

9. Koh WU, Kim HJ, Park HS, Choi WJ, Yang HS, Ro YJ. A randomised controlled trial comparing continuous supraclavicular and interscalene brachial plexus blockade for open rotator cuff surgery. Anaesthesia. 2016;71(6): 692-699.

11. Song JG, Jeon DG, Kang BJ, Park KK. Minimum effective volume of mepivacaine for ultrasound-guided supraclavicular block. Korean J Anesthesiol. 2013;65:37-41.

10. Mogahed MM, Abd El Ghafar MS. Ultrasound guided supraclavicular brachial plexus block for arterio-venous shunt surgery in chronic renal failure, comparative study between two volumes of bupivacaine. J Anesth Clin Res. 2017;8(744).

Hadzic A. Ultrasound-guided supraclavicular brachial plexus block. In: Bendtsen TF, Lopez AM, Vandepitte C, eds. Textbook of Regional Anesthesia and Acute Pain Management. 2nd ed. New York, NY: McGraw-Hill Publishing; 2017:chap 32C.

Hadzic_Ch31C_p169-172.indd 172

Suggested Reading

19/04/19 11:24 AM

31D Ultrasound-Guided Infraclavicular Brachial Plexus Block Jennifer L. Cowell and Arthur Atchabahian

QUESTIONS DIRECTIONS: Choose the one best response to each question. 1.  Compared to other brachial plexus nerve blockade approaches, the infraclavicular approach: A. Has an increased risk of respiratory complaints by the patient if performed correctly without complications B. Is ideal for placement of a continuous infusion catheter due to the relative depth of the plexus at this level C. Is well suited for shoulder surgery as the nerves are still proximal to the location of the surgical incision D. Provides the clearest view of the nerves when visualized with the ultrasound 2.  A 68-year-old man with end-stage renal disease, hypertension, history of coronary artery bypass graft (×4), peripheral artery disease, and chronic obstructive pulmonary disease is scheduled for insertion of a brachiobasilic prosthetic graft for future hemodialysis access. If planning a regional technique as the primary form of anesthetic in this patient, A. It is appropriate to perform an axillary block on either the right or the left arm with a supplemental musculocutaneous block. B. It is appropriate to perform an interscalene block to completely block sensation to the arm if the surgery is on the right arm. C. It is appropriate to use an infraclavicular block on either the right or left arm with supplemental intercostobrachial block. D. It is important to know what the patient’s cardiopulmonary functional status is before deciding on an axillary block. 3.  When performing an infraclavicular block, it is necessary to: A. Inject 20–30 mL of local anesthetic under the pectoralis minor muscle and above the axillary artery at the level of the cords B. Surround the axillary artery in a U-shaped fashion to ensure that all three cords have been sufficiently covered

C. Use the nerve stimulator to obtain a motor response of wrist extension before injecting local anesthetic D. Visualize all three cords individually as they surround the artery in order to be able to surround them completely 4.  With the use of a nerve stimulator during an ultrasoundguided infraclavicular block: A. A motor response of elbow flexion indicates the ideal location for injecting. B. A posterior cord motor response will be the first motor response seen. C. The motor response to stimulation of the medial cord is finger flexion. D. Wrist extension with stimulation indicates the ideal location for injecting. 5.  When performing an infraclavicular block, after obtaining an adequate view via ultrasound: A. Advance the needle in plane to just above the axillary artery, between the medial and lateral cords, and inject local anesthetic. B. Advance the needle to just posterior to the lateral cord and inject 20–30 mL of local anesthetic to surround the lateral cord. C. Advance the needle to just posterior to the lateral cord and inject local anesthetic, moving the needle as needed to achieve a U-shaped spread. D. Advance the needle out of plane toward the medial cord and inject local anesthetic, moving the needle as needed to achieve a U-shaped spread. 6.  On which of the following patients would be most appropriate to use an infraclavicular block as the primary mode of anesthesia? A. A patient undergoing a biceps tenodesis B. A patient undergoing an open reduction and internal fixation (ORIF) of the elbow the day of injury C. A patient who returns to the hospital seven days after injury for ORIF of the elbow D. A patient with a torn rotator cuff having elective arthroscopic repair 173

Hadzic_Ch31D_p173-176.indd 173

19/04/19 11:23 AM

174

PART 3

Clinical Practice of Regional Anesthesia

7.  In the image below, the arrow is pointing to which structure?

A. At the level of the branches in the axillary space B. At the level of the cords in the infraclavicular space C. At the level of the cords in the supraclavicular space D. At the level of the divisions in the supraclavicular space 9.  The infraclavicular block: A. Blocks the brachial plexus at the level of divisions B. Can be used for shoulder surgery C. Is a preferred location for placing a catheter for continuous blockade D. Is a motor sparing nerve block

ANSWERS AND EXPLANATIONS 1.  B is correct. At the level of the infraclavicular block, the neurovascular bundle is relatively deep to the skin as compared to the supraclavicular and interscalene blocks. Because of the fact that it lies under the pectoral muscles, a continuous infusion catheter would have a decent amount of space to be tunneled through to prevent dislodgement.

(Reproduced with permission from Hadzic A: Hadzic’s Textbook of Regional Anesthesia and Acute Pain Management, 2nd ed. New York, NY: McGraw-Hill Education; 2017.)

A. Axillary artery B. Lateral cord C. Medial cord D. Posterior cord 8.  The following image shows the brachial plexus at which level?

(Reproduced with permission from Hadzic A: Hadzic’s Textbook of Regional Anesthesia and Acute Pain Management, 2nd ed. New York, NY: McGraw-Hill Education; 2017.)

Hadzic_Ch31D_p173-176.indd 174

  A is incorrect. Below the clavicle, it is unlikely that there will be phrenic nerve impairment as a side effect of this block. Although there is a risk of pneumothorax, if there are no complications during the procedure and it was well visualized, the patient should have less of risk of complaining of respiratory symptoms. C is incorrect. At this level, the cords of the plexus are being blocked and it is below the clavicle. Although they are still proximal to the shoulder, there are small branches of the plexus that have already split off before the level of the infraclavicular block. Most notable is the suprascapular nerve, which branches at the level of the trunks and provides much of the sensation to the shoulder. D is incorrect. As mentioned, this block is deeper than other approaches to the brachial plexus and as such, the image is not as clear. The nerves surround the axillary artery with a high amount of anatomic variation. Thankfully, they are almost always still located within very close proximity to the artery so visualization of spread around the artery will be sufficient. 2.  C is correct. An infraclavicular block could be used safely on either arm in this patient with a supplemental block of the intercostobrachial nerve, which provides sensation to the medial arm. This is done subcutaneously over the medial aspect to the arm just distal to the axilla. A is incorrect. An axillary block would not be appropriate in this patient as the incision is of the anteromedial upper arm at approximately the same level as the block and it is unlikely to provide adequate anesthesia. B is incorrect. A patient who had coronary artery bypass graft (×4) could have left-sided phrenic nerve damage from harvesting of the left internal mammary artery (LIMA) and therefore a right-sided interscalene block would be inappropriate as it could result in complete diaphragmatic paralysis.

19/04/19 11:23 AM

CHAPTER 31D

D is incorrect. In a patient with poor cardiopulmonary function at baseline, regional anesthesia is good choice because it prevents these patients from being subjected to general anesthesia and mechanical ventilation. While it is important to know every patient’s cardiopulmonary status ahead of time (if possible), this information can change your anesthetic management within the choice of brachial plexus block. The plan for an axillary block would not be changed based on the patient’s cardiopulmonary status as it is distal enough to not cause any changes outside of the arm. 3. B is correct. An infraclavicular block is performed at the level of the cords of the brachial plexus just below the clavicle and medial to the coracoid process. It is deep to both of the pectoralis muscles and as such, the visualization on ultrasound is not as clear as a more superficial block. A is incorrect. If the anesthetic were to be injected above the artery, below the pectoralis muscle, although some people may have one of their cords in that position, the anesthetic would spread along the fascial plane and would not anesthetize all of the cords. C is incorrect. Some people have areas of tissue that prevent the spread in a full U shape without repositioning the needle to pop through those layers. A nerve stimulator may be useful as the nerves themselves are hard to visualize and it will confirm needle tip location. However, wrist extension is caused by the stimulation of the posterior cord and this may be the first one that is encountered. If this is the only motor response obtained, there is no guarantee that the other cords are being anesthetized due to the possibility of tissue planes compartmentalizing the individual cords as mentioned above. D is incorrect. There is a large amount of anatomical variation in terms of the positions of the cords around the axillary artery and this is made more variable by the position of the upper extremity. Luckily, they are all within a sheath and therefore it is not necessary to visualize them individually, but rather to inject local anesthetic in such a way that it creates a U around the caudad, cephalad, and posterior sides of the artery. It will appear as a properly oriented U shape on the ultrasound screen if done properly. 4.  D is correct. When performing an infraclavicular block with ultrasound guidance, the use of nerve stimulation can be helpful for confirming placement of the needle for injection of local anesthetic. The goal of the infraclavicular block is to form a U shape around the axillary artery with local anesthetic. A is incorrect. This is not the ideal location to inject because there is a significant chance that the local anesthetic will not spread to the other cords due to fascial connections. B is incorrect. The first motor response seen will typically be from the lateral cord, which will be seen as elbow or finger flexion. C is incorrect. Continuing to advance the needle until a response of finger or wrist extension indicates that the needle is located near the posterior cord and at the proper location for injecting. There is no need to attempt

Hadzic_Ch31D_p173-176.indd 175

Ultrasound-Guided Infraclavicular Brachial Plexus Block

175

stimulation of the medial cord, which causes thumb flexion and opposition as well as ulnar deviation of the wrist, because due to its location, there is a risk of puncturing the artery if attempted. 5. C is correct. After obtaining an adequate view of the axillary artery with the indication of cords around it (it is not always possible to clearly visualize the cords), the needle should be advanced in plane towards the posterior cord on the lateral side of the artery. If on injecting, it is seen that the local anesthetic is not spreading around the artery adequately, there may be small tissue connections causing compartments and the needle should be repositioned to puncture these connections. A and B are incorrect. The goal of the block is to surround the posterior side of the artery with a U-shaped spread of local anesthetic, ensuring that all three cords will be covered with anesthetic. D is incorrect. Injecting out of plane or toward the medial cord is not advised due to the risk of puncturing the artery or causing a pneumothorax. 6. C is correct. An infraclavicular block provides reliable sensory and motor blockade to the elbow and forearm, as well as the lateral arm. Regional anesthesia is not recommended for acute fractures due to the possibility of masking compartment syndrome, therefore this block would be best for a patient who is having a scheduled procedure. A is incorrect. The medial aspect of the arm is innervated by the intercostobrachial nerve. Therefore any incision made on the medial aspect of the arm would need an additional block, such as for a biceps tenodesis. B is incorrect. A patient undergoing ORIF of an elbow would have adequate coverage from an infraclavicular block, but regional anesthesia is not recommended for acute fractures due to the possibility of masking compartment syndrome. D is incorrect. This block will cover the lateral arm as well as part of the shoulder; however, the shoulder is immediately adjacent to the location of the injection of the local anesthetic so this block will not provide an adequate block for shoulder arthroscopy due to the branching of nerves supplying the shoulder prior to the level of the cords of the brachial plexus. 7.  C is correct. In this parasagittal view of the brachial plexus, one can see the axillary artery and vein as well as the three cords of the brachial plexus, the medial, the lateral, and the posterior. The arrow points to the medial cord, which is often located at approximately 5 o’clock compared to the axillary artery. Refer to Figure 31D–1. A is incorrect. It should be noted that there is substantial anatomical variation between different people and a clear picture is not always visible of one or more of the cords. Thankfully surrounding the axillary artery is sufficient for an adequate block. B and D are incorrect. The posterior is approximately at 7 o’clock and the lateral at 9 o’clock.

19/04/19 11:23 AM

176

PART 3

Clinical Practice of Regional Anesthesia

A is incorrect. It is more proximal than the typical image of the infraclavicular block where the cords are surrounding the axillary artery. C and D are incorrect. At the level of the costoclavicular space, the probe is still inferior to the clavicle and the brachial plexus is still at the level of the cords, but they are clustered together in a fashion more reminiscent of a supraclavicular image. Above the clavicle though, in the supraclavicular space, the brachial plexus is at the level of the divisions, not the cords.

Pectoralis major muscle

Pectoralis minor muscle

MC

LC

AA

AV

PC

Pleura

FIGURE 31D–1  In this probe position, the pleura lies closer to the brachial plexus.

8.  B is correct. The image depicted is an image in the costoclavicular space, a variation of the infraclavicular block. Refer to Figure 31D–2.

Pectoralis major

9. C is correct. The infraclavicular block is a useful block for procedures of the forearm, elbow, and hand. Compared to other blocks of the brachial plexus, it is relatively deep being under the pectoral muscles as opposed to being just under the skin and platysma. Additionally, it is in an area that is relatively less mobile as compared to brachial plexus blocks above the clavicle. Because of these factors, it is the preferred location for a continuous infusion catheter as it is less likely to become dislodged resulting in an inadequate blockade. A is incorrect. The infraclavicular block is performed at the level of the cords of the brachial plexus. B is incorrect. Despite the injection being proximal to the shoulder, it would be inadequate for a shoulder surgery. This is due to the fact that the shoulder is innervated in part by the suprascapular nerve, which branches off of the plexus at the level of the transition from roots to trunks. It is also innervated by the axillary nerve, which would be covered by the infraclavicular block as it branches from the posterior cord. D is incorrect. The axillary nerve additionally provides motor innervation to the teres minor, deltoid, and long head of the triceps brachii.

Suggested Reading

PC

LC

AA

AV

MC

Hadzic A. Ultrasound-guided infraclavicular brachial plexus block. In: Atchabahian A, Vandepitte C, Lopez AM, eds. Textbook of Regional Anesthesia and Acute Pain Management. 2nd ed. New York, NY: McGraw-Hill Publishing; 2017:chap 32D.

Pleura

FIGURE 31D–2  Ultrasound view of the brachial plexus at the costoclavicular space. The lateral (LC), medial (MC) and posterior (PC) cords are clustered together lateral to the axillary artery lying more superficial. At this level, the pectoralis minor is not seen deep to the pectoralis major.

Hadzic_Ch31D_p173-176.indd 176

19/04/19 11:23 AM

31E Ultrasound-Guided Axillary Brachial Plexus Block Hassanin Jalil, Kristof Nijs, and Astrid De Bruyn

QUESTIONS DIRECTIONS: Choose the one best response to each question. 1. Which of the following is not an indication for an axillary brachial plexus block? A. Shoulder arthroscopy B. Carpal tunnel release C. Elbow arthroscopy D. Osteosynthesis of the radius 2. Which of the following statements is true regarding ultrasound-guided axillary brachial plexus block? A. The identification of all the individual nerves is absolutely necessary in performing an axillary brachial plexus block. B. Three muscles surround the neurovascular axillary bundle: the biceps, the coracobrachialis, and the conjoined tendon of the teres major and latissimus dorsi muscle. C. The axillary brachial plexus block gets its name from the main nerve that gets anesthetized. D. A perivascular approach alone is insufficient to get a good surgical axillary brachial plexus block. 3. Which of the following statements is true regarding ultrasound-guided axillary brachial plexus block? A. Arm abduction over 90 degrees may be useful in identifying anatomical structures. B. The ultrasound probe is placed on the long axis of the arm, at the level at the junction of the teres major and the biceps brachii. C. A curved transducer probe is preferable in the axillary brachial plexus block. D. A high-frequency ultrasound transducer is used. 4. Which of the following statements is true regarding ultrasound-guided axillary brachial plexus block? A. The ulnar nerve lies lateral to the axillary artery. B. The radial nerve lies lateral to the axillary artery.

C. The median nerve lies lateral to the axillary artery. D. All needle positions occur through three separate needle insertion points. 5. Which of the following statements is false? A. The brachial plexus gives rise to the axillary nerve. B. The ulnar nerve lies outside the axillary sheath and is more difficult to visualize on ultrasound. C. The musculocutaneous nerve supplies the sensation to the lateral forearm and the motor branches innervate the biceps. D. The median nerve is more superficial than the radial nerve. 6. Which of the following statements is true concerning ultrasound-guided axillary brachial plexus block? A. The multiple-injection technique gives the best result. B. The musculocutaneous and the radial nerve are located inferior to the axillary artery. C. An adequate block results in anesthesia of the entire arm. D. The medial skin of the upper arm cannot be blocked by an axillary brachial plexus block. 7. Regarding which nerves provide cutaneous supply to different parts of the arm, which of the following is false? A. The ulnar nerve supplies sensation to both surfaces of the medial one-and-a-half fingers. B. The median nerve supplies the sensation of the medial aspect of the palm, the palmar surface of the first threeand-a-half fingers, and the dorsal area of their tips. C. The musculocutaneous nerve supplies sensation to the lateral antebrachial cutaneous nerve (one of its branches) and supplies sensation to the lateral forearm. D. The radial nerve supplies the sensation of the posterior area of the arm and forearm. Distally the radial nerve supplies the dorsum of the hand, without the lateral three-and-a-half fingertips.

177

Hadzic_Ch31E_p177-180.indd 177

18/04/19 6:04 PM

178

PART 3

Clinical Practice of Regional Anesthesia

8. Which of the following statements is true regarding ultrasound-guided axillary brachial plexus block? A. An extra injection is usually required for the musculocutaneous nerve. B. Local anesthetic should first be deposited anterior to the axillary artery. C. A continuous axillary block is not a useful technique for analgesia. D. The needle is inserted in-plane, starting medial in a lateral direction. 9. Which of the following is false regarding ultrasoundguided axillary brachial plexus block? A. Frequent aspiration and slow administration of local anesthetic are critical for decreasing the risk of intravascular injection. B. The musculocutaneous nerve is located a few centimeters away from the axillary artery between the biceps and the coracobrachialis muscle. C. Abduction of the arm to 90 degrees is necessary to allow for transducer placement and needle advancement. D. Surrounding the axillary artery, four of the principal brachial plexus branches can be seen. 10. Which of the following is true regarding ultrasound-guided axillary brachial plexus block? A. Only forearm and hand surgery are good indications for the axillary brachial plexus block. B. The goal is to deposit local anesthetic around the brachial artery. C. The median nerve is more superficial than the radial nerve. D. The ulnar nerve lies outside the axillary sheath and is more difficult to visualize on ultrasound.

ANSWERS AND EXPLANATIONS 1. A is correct. The axillary nerve is not blocked because it departs from the posterior cord more proximally in the axilla. Therefore the skin over the deltoid muscle is not anesthetized and a shoulder arthroscopy is not a good indication for an axillary brachial plexus block. B is incorrect. For a carpal tunnel release the medial and ulnar nerves need to be anesthetized; these are blocked in the axillary brachial plexus block. However a forearm block may be more appropriate when only a carpal tunnel release is intended. C is incorrect. The radial and musculocutaneous nerves must be blocked for an elbow arthroscopy. The axillary brachial plexus block is an appropriate locoregional technique. D is incorrect. The radial and musculocutaneous nerves are blocked by the axillary brachial plexus block, which is appropriate for the osteosynthesis of the radius.

Hadzic_Ch31E_p177-180.indd 178

2. B is correct. The bicep muscles form the anterior and superficial edge, the wedge-shaped coracobrachialis forms the anterior and deep edge, and the conjoined tendon of the teres major and the latissimus dorsi forms the medial and posterior edge of the compartment. A is incorrect. Although individual nerves can usually be identified, this is not absolutely necessary because the deposition of local anesthetic around the axillary artery is sufficient for an effective block. C is incorrect. The block gets its name from the approach and not from the axillary nerve, which itself is not blocked because it departs from the posterior cord more proximally in the axilla. D is incorrect. The perivascular approach of the axillary brachial plexus block is a simple injection of local anesthetic deep to the artery, at the 6 o’clock position, instead of targeting the three nerves individually. This technique may shorten the duration of the block procedure, but also delay the onset time, resulting in no difference in total time from skin puncture to onset of the surgical block. An extra injection is required for the musculocutaneous nerve in most individuals because the nerve lies outside the neurovascular bundle. 3. D is correct. The structures of interest are superficial (1–3 cm below the skin), and the axillary artery is readily identified within a centimeter of the skin surface on the medial aspect of the proximal arm. High-frequency transducers have good echographic visibility on superficial anatomical structures (8–14 MHz). A is incorrect. Abduction of the arm to 90 degrees is necessary to allow for transducer placement and needle advancement. Care should be taken not to over-abduct the arm, as this may cause patient discomfort as well as traction on the brachial plexus, making it theoretically more vulnerable to injury by needle or injection. Also over-abduction of the arm may obscure the axillary artery and veins. B is incorrect. The ultrasound probe is placed on the short axis of the arm, at the level of the junction of the teres major and biceps brachii. The pectoralis major muscle is palpated as it inserts onto the humerus, and the transducer is placed on the skin immediately distal to that point. C is incorrect. The linear ultrasound probe is designed for superficial imaging. 4. C is correct. The median nerve lies lateral to the axillary artery. A is incorrect. The ulnar nerve is indeed located medial to the axillary artery. B is incorrect. The radial nerve can be located with ultrasound lying medial to the axillary artery, superficial to the conjoined tendon of the teres major and the latissimus dorsi muscles. D is incorrect. All needle redirections are done through the same needle insertion site for the radial, medial, and

18/04/19 6:04 PM

CHAPTER 31E

ulnar nerves. The perivascular approach is also a possibility by one injection of local anesthetic deep to the artery, at the 6 o’clock positioning, instead of targeting the three nerves individually. An extra injection is required for the musculocutaneous nerve in most individuals because the nerve lies outside the neurovascular bundle. 5. B is false, so option B is correct. The musculocutaneous nerve lies outside the axillary sheath. It is located in the fascial layers between the biceps and coracobrachialis muscles, though its location is variable and can be seen within either muscle. The ulnar nerve lies in the axillary sheath. A is true, so option A is incorrect. The axillary nerve departs from the posterior cord of the brachial plexus more proximally in the axilla. C is true, so option C is incorrect. The nerve supplies sensation to the lateral forearm and has motor branches to the coracobrachialis, the biceps brachii, and the brachial muscles. D is true, so option D is incorrect. The median nerve is more superficial than the radial nerve. 6. D is correct. The medial skin of the upper arm is innervated by the intercostobrachial nerve and by the medial brachial cutaneous nerve, which originates from the anterior cord of the brachial plexus. Neither nerve is targeted by an axillary brachial plexus block. However, the intercostobrachial nerve (T2) can be blocked by an additional subcutaneous injection just distal to the axilla. A is incorrect. Although targeting the three individual nerves is effective, it is not absolutely necessary. One injection of local anesthetic perivascular around the axillary artery is a simple alternative with only a short delay in onset time, resulting in no difference in total time from skin puncture to onset of the surgical block. B is incorrect. The radial nerve is located inferior to the axillary artery. The musculocutaneous nerve lies outside the axillary sheath. It is located in the fascial layers between the biceps and coracobrachialis muscles, though its location is variable and can be seen within either muscle. C is incorrect. The axillary brachial plexus block (including the musculocutaneous nerve) results in anesthesia of the upper limb from the mid-arm down to and including the hand.

Ultrasound-Guided Axillary Brachial Plexus Block

179

palmar surface of the first three-and-a-half fingers, and the dorsal area of their tips. A, C, and D are incorrect. All of these statements are true. 8. A is correct. An extra injection is required for the musculocutaneous nerve in most individuals because the nerve lies outside the neurovascular bundle. B is incorrect. Local anesthetic should be deposited posterior to the artery first, to avoid displacement of structures of interest deeper and thereby obscuring the nerves, which may occur if injections for the median or ulnar nerves are carried out first. C is incorrect. The indwelling axillary catheter is a useful technique for long-term analgesia and sympathetic block. The goal of the continuous axillary block is to place the catheter within the vicinity of the branches of the brachial plexus (ie, within the sheath of the brachial plexus). D is incorrect. The needle is inserted in-plane from the anterior aspect and directed to the posterior aspect of the axillary artery. It is important to insert the needle in-plane and not to dissect the veins in the medial area of the axillary sheath. 9. D is false, so option D is correct. Surrounding the axillary artery, three of the four principal branches of the brachial plexus can be seen: the median (superficial and lateral to the artery), the ulnar (superficial and medial to the artery), and the radial (posterior and lateral or medial to the artery) nerves. A, B, and C are incorrect. All of these statements are true. 10. C is correct. The median nerve is more superficial than the radial nerve. A is incorrect. Elbow, forearm, and hand surgery are good indications for the axillary brachial plexus block. B is incorrect. The goal is to deposit local anesthetic around the axillary artery. D is incorrect. The musculocutaneous nerve lies outside the axillary sheath. It is located in the fascial layers between the biceps and coracobrachialis muscles, though its location is variable and can be seen within either muscle. The ulnar nerve lies in the axillary sheath.

Suggested Reading Hadzic A. Ultrasound-guided axillary brachial plexus block. In: Vandepitte C, Lopez AM, Jalil H, eds. Textbook of Regional Anesthesia and Acute Pain Management. 2nd ed. New York, NY: McGraw-Hill Publishing; 2017:chap 32E.

7. B is false, so option B is correct. The median nerve supplies the sensation of the lateral aspect of the palm, the

Hadzic_Ch31E_p177-180.indd 179

18/04/19 6:04 PM

This page intentionally left blank

Hadzic_FM_p00i-pxx.indd 6

22/04/19 6:40 PM

31F Ultrasound-Guided Blocks at the Elbow Hassanin Jalil, Kristof Nijs, and Astrid De Bruyn

1. Which of the following statements is false regarding nerves that provide cutaneous supply to parts of the hand and wrist? A. The ulnar nerve supplies sensation to both surfaces of the medial one-and-a-half fingers. B. The median nerve supplies the sensation of the medial aspect of the palm, the palmar surface of the first threeand-a-half fingers, and the dorsal area of their tips. C. The musculocutaneous nerve supplies sensation to the lateral antebrachial cutaneous nerve (one of its branches) and supplies sensation to the lateral forearm and wrist. It needs to be blocked separately by a subcutaneous wheal distal to the elbow if lateral wrist surgery is planned. D. The radial nerve supplies the sensation of the posterior area of the arm and forearm. Distally the radial nerve supplies the dorsum of the hand, without the lateral three-and-a-half fingertips.

3. The peripheral nerves are best visualized in the elbow region using ultrasound at specific locations. Which one of the following statements is false? A. The radial nerve is best visualized above the lateral aspect of the elbow, lying in the interfascial plane between the brachioradialis and the brachialis muscles. The transducer is placed transversely on the anterolateral aspect of the distal arm, 3–4 cm above the elbow crease. B. The median nerve is imaged at the level of the elbow crease, as it is located superficial. The transducer is placed just above the crease and adjusted to obtain a clear view of the brachial artery, as the median nerve lies in close contact to the artery on its medial side. C. The ulnar nerve is identified at the anteromedial aspect of the elbow, a few centimeters proximally to the elbow crease. It is the hyperechoic oval structure immediately underneath the brachial fascia and superficial to the triceps muscle. D. The ulnar nerve is identified at the posteromedial aspect of the elbow, a few centimeters proximally to the elbow crease. It is the hyperechoic oval structure immediately underneath the brachial fascia and superficial to the triceps muscle.

2. Which one of the following statements is true regarding ultrasound-guided blocks at the elbow? A. The main indication for a forearm block is as a means of rescuing or supplementing an incomplete or failed proximal brachial plexus block. B. The radial, median, and ulnar nerve can be blocked at only one location in the forearm. C. The use of a tourniquet in forearm surgery is well tolerated when the radial, median, and ulnar nerve are blocked. D. It is not necessary to completely surround the entire nerve in a circumferential pattern.

4. Which one of the following statements is true regarding ultrasound-guided blocks at the elbow? A. The radial nerve divides just distal to the elbow crease into the superficial and deep branches where it can be located very easily. B. A small-gauge needle is safer to perform a superficial block like the radial, median, and ulnar nerves around the elbow. C. Both the out-of-plane and in-plane approach can be used for all three nerve blocks. D. A curved transducer probe is preferable in ultrasound-guided blocks at the elbow.

QUESTIONS DIRECTIONS: Choose the one best response to each question.

181

Hadzic_Ch31F_p181-184.indd 181

19/04/19 11:23 AM

182

PART 3

Clinical Practice of Regional Anesthesia

5. Which one of the following statements is false regarding patient positioning to perform ultrasound-guided blocks at the elbow? A. The blocks are performed with the patient in the supine position. B. During the radial nerve block, the arm is flexed at the elbow and the hand of the patient is placed on the patient’s abdomen. C. The median and ulnar nerves are blocked with the arm in abduction. D. The median and radial nerves are blocked with the arm in abduction. 6. Which one of the following operations is not an indication for an ultrasound-guided block at the elbow of the radial, median, and ulnar nerve? A. Carpal tunnel release B. Osteosynthesis of the distal radius C. Trigger finger release D. Thumb prosthesis 7. Motor movement can be expected when nerves are stimulated. Which one is of the following statements is false? A. Stimulation of the radial nerve gives wrist and finger extension, especially in the thumb. B. Stimulation of the median nerve gives finger flexion, wrist flexion, and wrist pronation. C. Stimulation of the ulnar nerve gives only thumb adduction. D. Stimulation of the ulnar nerve gives thumb adduction combined with ring and little-finger flexion. 8. Which one of the following statements is correct regarding ultrasound-guided blocks at the elbow? A. Anterior to the elbow joint the radial nerve can be visualized between the biceps and the brachioradialis. B. Injection of 10–12 mL of local anesthetic per nerve is necessary to assure an adequate block. C. At the level of the elbow joint, the median nerve is situated lateral to the brachial artery. D. At the level of the elbow joint, the radial nerve is located between the brachioradialis and brachialis muscles. 9. Which one of the following statements is false regarding ultrasound-guided blocks at the elbow? A. Distal nerve blocks of the radial, median, and ulnar nerve after a proximal brachial plexus block have to be performed with great precision. B. In the forearm, the ulnar nerve is located medial to the ulnar artery. C. The ulnar nerve is targeted at the level of the elbow joint. D. The blocking of the radial, median, and ulnar nerve at the elbow is insufficient for the surgical procedure of an extensor tendon release.

Hadzic_Ch31F_p181-184.indd 182

10. Which one of the following statements is false regarding ultrasound-guided blocks at the elbow? A. The ulnar nerve is located at the posteromedial aspect of the elbow, a few centimeters proximally to the crease, as a hyperechoic oval structure. B. It is unnecessary to completely surround the entire nerve in a circumferential pattern. C. The out-of-plane approach can also be used for the three blocks. D. The radial nerve is best visualized above the medial aspect of the elbow, lying in the interfascial plane between the biceps muscle and the brachialis muscle.

ANSWERS AND EXPLANATIONS 1. B is false, so option B is correct. The median nerve supplies the sensation of the lateral aspect of the palm, the palmar surface of the first three-and-a-half fingers, and the dorsal area of their tips. A, C, and D are incorrect. All of these statements are true. 2. D is correct. It is unnecessary to completely surround the entire nerve in a circumferential pattern, although this can enhance the speed of block onset. A is incorrect. The two main indications for a forearm block are a standalone technique for hand and/or wrist surgery and as a means of rescuing or supplementing an incomplete or failed proximal brachial plexus block. B is incorrect. There are a variety of locations where a practitioner could approach each of these nerves, most of which are similar in efficacy. C is incorrect. The use of a tourniquet, either on the arm or forearm, usually requires sedation and/or additional analgesia. 3. C is false, so option C is correct. The ulnar nerve is identified at the posteromedial aspect of the elbow, a few centimeters proximally to the elbow crease. It is the hyperechoic oval structure immediately underneath the brachial fascia and superficial to the triceps muscle. A, B, and D are incorrect. All of these statements are true. 4. C is correct. Further, the in-plane approach is safer when experience in locoregional anesthesia is limited. A is incorrect. The radial nerve divides just distal to the elbow crease into the superficial (sensory) and deep (motor) branches. These smaller divisions of the radial nerve are more challenging to identify in the forearm. A single injection above the elbow is favored because it ensures blockade of both.

19/04/19 11:23 AM

CHAPTER 31F

B is incorrect. When using a small-gauge needle we should pay meticulous attention in order to avoid an intraneural injection, which is more likely with a smallerdiameter and sharp-tip design needle. D is incorrect. The linear ultrasound probe is designed for superficial imaging. 5. D is false, so option D is correct. The median and ulnar nerves are blocked with the arm in abduction. A, B, and C are incorrect. All of these statements are true. 6. B is correct. The blocking of the radial, median, and ulnar nerve at the elbow is insufficient for osteosynthesis of the distal radius. As an alternative, an axillary brachial plexus block should be performed. A is incorrect. The blocking of the radial, median, and ulnar nerve at the elbow is sufficient to achieve an adequate block for carpal tunnel release surgery. C is incorrect. The blocking of the radial, median, and ulnar nerve at the elbow is sufficient. Depending on the digit to be released, a different nerve needs to be blocked. D is incorrect. The blocking of the radial, median, and ulnar nerve at the elbow is sufficient. In some patients the lateral antebrachial cutaneous (a branch of the musculocutaneous nerve) should also be blocked. 7. C is false, so option C is correct. Stimulation of the ulnar nerve gives thumb adduction combined with ring and little-finger flexion. A, B, and D are incorrect. All of these statements are true.

Hadzic_Ch31F_p181-184.indd 183

Ultrasound-Guided Blocks at the Elbow

183

8. D is correct. At the level of the elbow joint, the radial nerve is located between the brachioradialis and brachialis muscles. A is incorrect. Anterior to the joint the radial nerve can be visualized between the extensor carpi radialis muscles (longus and brevis) and brachioradialis. B is incorrect. Injection of 4–5 mL of local anesthetic per nerve provides an adequate nerve block after negative aspiration. C is incorrect. At the elbow the median nerve is situated medial to the brachial artery. 9. C is false, so option C is correct. The ulnar nerve should be blocked above or below the elbow joint. The ulnar block should never be performed at the level of the elbow joint because of the risk of pressure-related nerve injury in the ulnar notch. A, B, and D are incorrect. All of these statements are true. 10. D is false, so option D is correct. The radial nerve is best visualized above the lateral aspect of the elbow, lying in the interfascial plane between the brachioradialis and the brachialis muscle. A, B, and C are incorrect. All of these statements are true.

Suggested Reading Hadzic A. Ultrasound-guided blocks at the elbow. In: Lin J-A, Bendtsen TF, Lopez AM, Jalil H, eds. Textbook of Regional Anesthesia and Acute Pain Management. 2nd ed. New York, NY: McGraw-Hill Publishing; 2017:chap 32F.

19/04/19 11:23 AM

This page intentionally left blank

Hadzic_FM_p00i-pxx.indd 6

22/04/19 6:40 PM

31G Ultrasound-Guided Wrist Block Sofie Louage

QUESTIONS DIRECTIONS: Choose the one best response to each question. 1.  A wrist block can best be performed: A. At the wrist crease. At this level, the median nerve is easier to access as it is located superficial to the tendons, just below the retinaculum. B. 5–10 cm proximally to the wrist crease, to ensure the blockade of the palmar branches of median and ulnar nerves C. At the elbow crease, to cover the deep motor branch of the radial nerve D. In-plane so the nerves can be blocked with just one puncture 2.  The easiest way to recognize the median nerve is: A. By using neurostimulation, searching for flexion of the digits B. By searching at the lateral side of the brachial artery in the elbow crease C. By sliding the transducer up and down the arm and tilting the transducer slightly D. By increasing the depth to at least 5 cm 3.  Distribution of the wrist block can be tested as follows: A. Absence of thumb adduction for the ulnar nerve and thumb opposition for the median nerve B. Cold sensation of the fifth digit for the ulnar nerve and cold sensation of the dorsal side of the thumb for the median nerve C. All nail beds are innervated by the radial nerve D. By surgical incision in the wrist crease for carpal tunnel surgery 4.  A wrist block: A. Its indicated for hand and finger surgery B. Its considered a fascial plane technique C. Blocks the deep branch of the radial nerve D. It is always performed at the level of the wrist crease

5.  Which of the following statements is true regarding wrist block? A. A wrist block is most easily performed with the patient in supine position to expose the volar surface of the wrist. B. To block the median nerve transducer is placed in a transverse orientation proximally to the wrist crease, with a slight tilt distally (toward the hand). C. To block the ulnar nerve it’s useful to identify the ulnar artery; the ulnar nerve will be imaged as a triangular or oval hyper-echoic structure medial to the artery. D. Ultrasound can be used at the forearm to image and block the superficial branch of the radial nerve. E. All statements are true.

ANSWERS AND EXPLANATIONS 1. B is correct. An ultrasound-guided “wrist” block is often performed 5–10 cm proximally to the wrist crease. This location ensures the blockade of the palmar branches of the median and ulnar nerves, which take off a few centimeters proximally to the wrist crease. A is incorrect. At the wrist crease, it is easy to confuse the tendons for the nerve and vice versa; for this reason, it is recommended to slide the transducer 5–10 cm proximally to the volar side of the forearm, to confirm the location of the nerve. C is incorrect. The radial nerve divides just above the elbow crease into the superficial (sensory) and deep (motor) branches. Only the superficial branch needs to be blocked for anesthesia of the hand. D is incorrect. For each of the blocks, the needle may be inserted either in-plane or out-of-plane. Ergonomics often dictates which is more effective. 2. C is correct. As the median nerve exhibits pronounced anisotropy, tilting the transducer slightly will make the nerve appear alternately brighter or darker with respect to the background. A is incorrect. Neurostimulation of the median nerve will give flexion of the digits. This can be used when there is doubt about the median nerve. Nevertheless we prefer to avoid placing the needle tip that close to the nerve. 185

Hadzic_Ch31G_p185-186.indd 185

22/04/19 1:59 PM

186

PART 3

Clinical Practice of Regional Anesthesia

B is incorrect. The median nerve crosses the elbow medial to the brachial artery. D is incorrect. The median nerve assumes an increasingly superficial position in the forearm. Mostly it is no deeper than 3 cm. 3. A is correct. As the ulnar nerve innervates the adductor pollicis muscle, an ulnar nerve block can be tested by absence of thumb adduction. As the median nerve innervates the three thenar muscles, including the opponens pollicis muscle, a median nerve block can be tested by absence of thumb opposition. B is incorrect. The ulnar nerve gives sensory innervation of the fifth digit. The median nerve gives sensory innervation of the palmar side of the thumb. C is incorrect. Nail beds are respectively innervated by the palmar digital branches of the ulnar and the median nerve. D is incorrect. For carpal tunnel surgery an additional subcutaneous injection at the wrist crease at the incision level is necessary to anesthetize the palmaris ramus of the median nerve.1 4. A is correct. A wrist block its indicated for hand (palmar surface) and finger surgery. It results in anesthesia of the entire hand, except the territory of the deep branch of the radial nerve (dorsal surface).

Hadzic_Ch31G_p185-186.indd 186

B is incorrect. During a wrist block, specific targets for local anesthetic injection are the median, ulnar and radial nerves, while during a fascial plane block, local anesthetic is injected in a tissue plane aiming to reach various anatomic locations. C is incorrect. At the level of the elbow the radial nerve divides into deep and superficial branches. During a wrist block only the superficial branch of the nerve can be found at the level of the forearm. D is incorrect. To perform a wrist block the transducer should be placed 5–10 cm proximal to the wrist crease. 5. E is correct. A, B, C, and D are true statements regarding wrist block.

Reference Macaire P, Singelyn F, Narchi P, Paqueron X. Ultrasound- or nerve stimulation-guided wrist blocks for carpal tunnel release: a randomized prospective comparative study. Reg Anesth Pain Med. 2008;33(4):363-368.

Suggested Reading Hadzic A. Ultrasound-guided wrist block. In: Leunen I, Louage S, Jalil H, Sala-Blanch X, eds. Textbook of Regional Anesthesia and Acute Pain Management. 2nd ed. New York, NY: McGraw-Hill Publishing; 2017:chap 32G.

22/04/19 1:59 PM

Section 4

Ultrasound-Guided Nerve Blocks for the Lower Extremity Chapter 32A Ultrasound-Guided Femoral Nerve Block  189 Chapter 32B Ultrasound-Guided Fascia Iliaca Block  195 Chapter 32C Ultrasound-Guided Lateral Femoral Cutaneous Nerve Block  201 Chapter 32D Ultrasound-Guided Obturator Nerve Block  203 Chapter 32E Ultrasound-Guided Saphenous (Subsartorius/Adductor Canal) Nerve Block  205 Chapter 32F Ultrasound-Guided Sciatic Nerve Block  207 Chapter 32G Ultrasound-Guided Popliteal Sciatic Block  213 Chapter 32H Ultrasound-Guided Ankle Block  215

Hadzic_Ch32A_p187-194.indd 187

19/04/19 11:29 AM

This page intentionally left blank

Hadzic_FM_p00i-pxx.indd 6

22/04/19 6:40 PM

32A Ultrasound-Guided Femoral Nerve Block Ine Leunen, Jennifer L. Cowell, and Arthur Atchabahian

QUESTIONS DIRECTIONS: Choose the one best response to each question. 1. Topographically, what is the position of the femoral nerve regarding the fascia iliaca? A. The femoral nerve lies above the fascia iliaca. B. The femoral nerve lies underneath the fascia iliaca. C. The femoral nerve lies between two layers of the fascia iliaca. D. The femoral nerve lies between the fascia lata and the fascia iliaca. 2. Which is the best location/level to perform a femoral nerve block? A. At the level of the femoral artery B. At the level of the bifurcation of the femoral artery and the deep artery of the thigh C. Underneath the bifurcation of the deep femoral artery and the deep artery of the thigh D. The femoral artery is not visible when performing a femoral nerve block. 3. A 67-year-old woman is scheduled for a total knee replacement. She receives a femoral nerve block and multimodal analgesia is prescribed. When the patient arrives on the ward, nurses are anxiously calling because of weakness in the operated leg. What kind of weakness do you expect, so you can reassure the nurse and patient? A. Not able to lift toes (dropfoot) B. Not able to extend knee (quadriceps weakness) C. Not able to flex knee (hamstring weakness) D. Full motor block of the leg 4. Which statement is true regarding a femoral block? A. The volume of local anesthetic (LA) needed for adequate anesthesia after a femoral nerve block lies between 10–15 mL.

B. The femoral nerve always lies immediately attached to the femoral artery. C. Spread of LA for a successful block needs to be circumferential around the femoral nerve. D. An out-of-plane technique is the safest technique to perform a single-shot femoral nerve block. 5. When the femoral nerve is stimulated, what kind of twitch do you expect? A. Flexion of the knee B. Contractions of the sartorius muscle C. Contractions of the quadriceps with movement of the patella D. Abduction of the hip 6. At what depth do you expect the femoral nerve when performing a femoral nerve block? A. At a maximal depth of 2 cm B. 2 to 4 cm C. 4 to 6 cm D. Deeper than 6 cm 7. When inserting a catheter for continuous femoral nerve block, what is the best way to leave the catheter in? A. In-plane approach with the catheter advanced underneath the femoral nerve B. Out-of-plane approach with the catheter right above the nerve C. Out-of-plane approach with the catheter underneath the most lateral part of the nerve D. In-plane approach with the catheter underneath the most lateral part of the nerve 8. Which statement is true regarding a femoral block? A. A femoral nerve block does not offer full analgesia after knee arthroplasty. B. Obese patients cannot receive a femoral nerve block. C. Studies have reported that femoral nerve block has a high incidence of complications related to nerve injury. D. The femoral nerve lies within the femoral sheath.

189

Hadzic_Ch32A_p187-194.indd 189

19/04/19 11:29 AM

190

PART 3

Clinical Practice of Regional Anesthesia

9. In Figure 32A–1, which numbers indicate the dermatomes of the femoral nerve? A. 1 + 4 B. 1 + 2 + 3 C. 2 + 3 + 4 D. 4 + 5

2 1

3 B

4

5

C

FIGURE 32A–1  Sensory innervation of the femoral nerve and its cutaneous branches.

10. What is the ultrasound image of the femoral nerve?

D

A

Hadzic_Ch32A_p187-194.indd 190

11. A continuous-infusion catheter in place for a femoral nerve blockade: A. Is more commonly performed out-of-plane as there is difficulty passing the catheter in the cephalad direction when in-plane

19/04/19 11:29 AM

CHAPTER 32A

12. The use of nerve stimulation when performing a femoral nerve block: A. Demonstrates that the needle is still above the fascia iliaca if the quadriceps begin contracting when the current is set to 0.5 mA B. Is a useful confirmation of placement when performing a single injection, but is not necessary for an adequate block C. Is useful for confirming the placement of a catheter when inserting a continuous-infusion catheter D. Should cause contraction of the sartorius muscle when the stimulating needle is in the proper location

FN FA Lateral

B. Is more secure if the insertion point at the skin is more lateral as it will be more stable in the iliacus muscle than in adipose tissue C. Is most reliable when the position of the distal end of the catheter is visualized after initial location is confirmed with nerve stimulation D. Will have better analgesia if a stimulating catheter is used in conjunction with ultrasound guidance for proper placement

191

Ultrasound-Guided Femoral Nerve Block

DAT

FV

FIGURE 32A–2  Ultrasound image of the femoral nerve (FN) as seen distally.

13. Which statement is true regarding performing a femoral block after surgical closure for postoperative pain control? A. The block should be performed in the recovery room, regardless of the type of surgery. B. The block should be performed with nerve stimulation, regardless of the type of surgery. C. The patient should be fully awake to provide feedback regarding intrafascicular injection. D. The practitioner may need to alter the ultrasound settings following hip arthroscopy.

16. If an image such as Figure 32A–2 is obtained when performing an ultrasound-guided femoral nerve block, what should the practitioner do next? A. Insert the needle from the lateral side, in-plane, and inject local anesthetic. B. Insert the needle out-of-plane while hydrodissection and inject local anesthetic. C. Move the ultrasound transducer caudad until the arteries join together, then inject local anesthetic. D. Move the ultrasound transducer cephalad until the arteries join together, then inject local anesthetic.

14. For proper placement of local anesthetic while performing a femoral nerve block: A. If the femoral artery and deep artery of the thigh (profunda femoris) are both seen, the transducer should be moved proximally. B. The local anesthetic should be injected while using moderate pressure on the transducer to improve visualization of the nerve. C. The needle should be visualized directly below the femoral nerve for adequate block of the femoral nerve. D. The transducer should be placed over the inguinal ligament for proper identification of landmarks via ultrasound.

17. The ideal location for injection of local anesthetic for a femoral nerve block is: A. Between the fascia lata and fascia iliaca B. Below the fascia iliaca C. Immediately adjacent to the femoral nerve D. Within the femoral nerve bundle

15. What statement is true regarding introducing a catheter for a continuous femoral nerve block infusion? A. The in-plane short-axis approach from the lateral-tomedial direction is the most common. B. The in-plane short-axis approach puts the catheter perpendicular to the nerve; therefore it is difficult to place. C. The out-of-plane approach is preferred due to the alignment with the neurovascular sheath. D. The use of a stimulating catheter can increase the chance of a successful block as it confirms placement of the catheter tip.

Hadzic_Ch32A_p187-194.indd 191

18. A femoral nerve block as the primary form of anesthetic would be most appropriate for which of the following patients? A. A patient undergoing an anterior quadriceps tendon repair B. A patient undergoing a proximal tibia open reduction and internal fixation (ORIF) after a motor vehicle accident 6 hours prior C. A patient undergoing a total knee replacement D. A patient undergoing repair of the lateral malleolus 19. Which statement is true regarding performing an ultrasound-guided femoral nerve block? A. The sartorius muscle is an important landmark. B. A “pop” can usually be felt when passing the fascia iliaca. C. Fully surrounding the nerve with local anesthetic is imperative to obtain an effective block. D. The block is best performed at the level where the deep femoral artery (profunda femoris) can be seen distinctly from the femoral artery.

19/04/19 11:29 AM

192

PART 3

Clinical Practice of Regional Anesthesia

Fascia lata

Femoral nerve

Pectineus muscle

Fascia iliaca

Femoral sheath

Iliopsoas muscle

FIGURE 32A–3  Arrangement of the fascial sheaths at the inguinal crease. Femoral nerve is enveloped by two layers of fascia iliaca, whereas femoral vessels are contained in the vascular (femoral) sheath made up of fascia lata.

ANSWERS AND EXPLANATIONS 1. C is correct. This is mostly the position to find the femoral nerve, in between two layers of the fascia iliaca. In some patients, however, the femoral nerve lies completely underneath the fascia iliaca (Figure 32A–3). 2. A is correct. Approaching the femoral nerve lower than the bifurcation can lead to an incomplete block due to proximal branches that already separate from the femoral nerve. 3. B is correct. The femoral nerve originates from L2-3-4 and has motor branches that supply innervation of the rectus femoris, vastus medialis, vastus lateralis, and vastus intermedius. All these muscles form the quadriceps. The quadriceps is the only extensor of the knee. So, motor weakness after a femoral nerve block will be seen as an inability to extend the knee. A dropfoot is caused by weakness of the tibialis anterior, extensor hallucis longus, and extensor digitorum longus muscle. Innervation of these muscles derives from the fibular nerve, a motor branch of the sciatic nerve. Weakness of the hamstrings (semitendinosus, semimembranosus, and biceps femoris muscle) derives from a blockade of the tibial nerve. The sciatic nerve originates from L4 to S3 and holds two major branches: the tibial nerve and the fibular branch. 4. A is correct. If the spread regarding the femoral nerve is acceptable, a volume of 10 to 15 mL of LA is sufficient.

Hadzic_Ch32A_p187-194.indd 192

Sometimes the femoral nerve can lie more lateral regarding the artery. This is important to know when advancing the needle without a good view of the nerve. Tilting the probe can optimize the image for a better view of the nerve. This is also the reason for us to perform the femoral nerve block in an out-of-plane technique. Viewing a spread of LA on one side of the femoral nerve is sufficient to have a good nerve block for both surgical and postoperative analgesia. 5. C is correct. Stimulating the femoral nerve gives a muscle contraction of the quadriceps muscles. Because these are attached to the patella by patellar ligament, a patella twitch is expected when stimulating the femoral nerve. A twich of the sartorius muscle means the needle might be to anteromedial. This can lead to stimulation of a proximal branch of the femoral nerve. If so, redirect the needle more lateral. 6. B is correct. Knowing the depth before performing the block gives you the ability to adjust the ultrasound setting beforehand. This may result in better visibility. 7. A is correct. Approaching the femoral nerve from the lateral side in an in-plane way, gives you a clear view of the nerve. If you see spread of local anesthesia (LA) underneath the nerve, you can advance the catheter and inject LA through the catheter to confirm its position. 8. A is correct. Full analgesia after knee arthroplasty requires involvement of the obturator to give full analgesia after total knee replacement.

19/04/19 11:29 AM

CHAPTER 32A

In obese patients, the nerve can be better visualized by placing a wedge underneath the hip. Also, taping the excessive amount of adipose tissue may result in a better ultrasound image and an easier approach. The incidence of nerve damage after a femoral nerve block is relatively rare, around 0.3%. The femoral sheath contains only the femoral artery and vein; the femoral nerve lies outside the femoral sheath. 9. D is correct. Region 4 is the skin territory innervated by branches of the femoral nerve, and Region 5 by the saphenous nerve, itself a branch of the femoral nerve. 10. A is correct as it is the ultrasound image of the femoral nerve. B is the saphenous nerve, C is radial nerve, and D is sciatic nerve. 11. B is correct. The more lateral the starting point for needle insertion for continuous femoral nerve block, the longer the catheter would be within the iliacus muscle, which may help prevent dislodgment because muscle tends to stabilize a catheter better than adipose tissue. A continuous-infusion catheter should be inserted in the same fashion as injecting a single shot for a femoral nerve block, and then the catheter should be firmly secured after positioning. A is incorrect. When placing the catheter, the in-plane technique is most commonly used to prevent accidental puncture of the femoral nerve when inserting directly above it. C is incorrect. Ultrasound guidance of a nonstimulating catheter should be used. When confirming position with the ultrasound, visualization of spread of local anesthetic within the proper compartment is preferred for a reliable block. D is incorrect. Stimulating catheters do not improve analgesia, but prolong the insertion time and cause more trauma due to increased likelihood of multiple manipulations. 12. B is correct. When doing an ultrasound-guided femoral nerve block, the use of nerve stimulation is not needed, but can help confirm positioning if there is questionable anatomy or poor visualization. A is incorrect. The nerve stimulator will cause a contraction of the quadriceps when the needle has passed through the fascia iliaca and is near the femoral nerve. With very high current, this can happen outside of the fascia iliaca, but it is unlikely when the current is at 0.5 mA or less. C is incorrect. When inserting a continuous-infusion catheter, nerve stimulation may actually lead to excessive manipulation and attempts without added benefit. D is incorrect. If sartorius muscle contraction is demonstrated, the needle should be redirected until the quadriceps muscles contract. 13. D is correct. Following hip arthroscopy, the irrigation fluid used for visualization of the surgical field can become infiltrated into the tissue locally due to the high

Hadzic_Ch32A_p187-194.indd 193

Ultrasound-Guided Femoral Nerve Block

193

pressure used. This may cause changes to the anatomy as visualized by ultrasound; therefore the depth of the field may have to be increased. A is incorrect. Postoperative blocks can be performed in any setting where the patient is being cardiovascularly monitored. However, given the surgical dressings, it may be inappropriate to perform the block in the recovery room as the site of injection may be covered with bandages. B is incorrect. The bandages or splint material may obscure any of the responses caused by the nerve stimulator, and therefore the nerve stimulator may just cause the patient pain without being helpful to the practitioner. C is incorrect. Postoperative blocks are performed at the end of a general anesthetic. Generally, patients are sedated for comfort while the blocks are performed. With ultrasound guidance and avoidance of injecting into a location with high resistance, intrafascicular injection is unlikely. While the feedback from a fully awake patient may also help this, it is not a reliable sign of intrafascicular injection. 14. A is correct. The femoral block should be performed at the level before the bifurcation of the femoral artery and the deep artery of the thigh to ensure local anesthetic is injected into the proper sheath. B is incorrect. The transducer can be used with moderate pressure for optimal view of the artery; however, the pressure should be minimal when injecting so the femoral vein is in view and not compressed and to prevent compression of the compartment that would limit spread of the local anesthetic. C is incorrect. To perform this block, the transducer should be placed in the femoral crease and moved laterally and medially until there is visualization of the femoral artery, then cephalad if needed to ensure only one artery is visualized. D is incorrect. Once an adequate view is seen, the needle could be inserted in any location around the femoral artery for an adequate block; however, the medial side would be impractical given the location of the artery. 15. A is correct. When inserting a continuous infusion catheter for a femoral nerve block, the most common approach is with the introducer being inserted in-plane. Like a single injection, the in-plane approach increases safety and prevents intraneural injection or manipulation by the introducer. This is due to the relative ease of hitting the nerve when approaching it from directly above. B is incorrect. With the in-plane approach, the introducer is coming toward the nerve at an angle. However, it does not prevent the anesthesia provider from advancing the catheter. A catheter will follow the path of least resistance, which will be through the recently injected local anesthetic. C is incorrect. After the provider visualizes the location of the spread of the local anesthetic on the ultrasound monitor, the catheter should be secured. This visualization of spread is satisfactory for positioning.

19/04/19 11:29 AM

194

PART 3

Clinical Practice of Regional Anesthesia

D is incorrect. Stimulating catheters have not been shown to improve the quality of the anesthetic block and have also been shown to increase the length of time needed to perform the block as well as increasing the amount of manipulation of the needle within the tissue. 16. D is correct. When performing a femoral nerve block, the ideal location is cephalad to the branch point of the profunda femoris artery, also known as the deep artery of the thigh. Cephalad to this branch point, the artery is located within the femoral sheath and the nerve adjacent is within the fascia iliaca. A is incorrect. After finding the ideal location, the needle should be inserted in-plane and local anesthetic should be injected into the sheath. B is incorrect. While it is possible to perform this block out-of-plane, there is increased risk of intraneural injection due to the angle of approach and the fact that the needle is inserted directly above the nerve instead of approaching it from the side. C is incorrect. To ensure a complete block of the nerve, injection should be done cephalad to this arterial branch point, not caudal, to avoid any significant branch points of the nerve as well. 17. C is correct. Local anesthetic should be immediately adjacent to the femoral nerve when performing a femoral nerve block. It should be within the pocket formed by the two layers of the fascia iliaca at the level of the femoral artery before the branch point of the deep artery of the thigh (profunda femoris). A is incorrect. It is important to locate the different fascial layers in this area because injection between the fascia lata and the fascia iliaca is unlikely to be an adequate blockage. B is incorrect. Below the fascia iliaca, if spread is toward the nerve, it may produce some degree of blockade, but this is best described as a fascia iliaca compartment block. D is incorrect. Although the local anesthetic should be adjacent to the nerve, injecting directly into the nerve bundle may cause long-term nerve damage as well as immediate paresthesias and thus should be avoided. 18. A is correct. The femoral nerve block provides sensory blockade of the anterior thigh, the knee joint, part of the hip joint, and the medial aspect of the lower leg. Without additional blockade of the sciatic nerve, proximal or distal, the femoral nerve block would be sufficient for surgery to the anterior thigh only, such as quadriceps tendon repair.

Hadzic_Ch32A_p187-194.indd 194

B is incorrect. A trauma patient undergoing ORIF of the proximal tibia would have a similar pain to a knee replacement; however, it is not advised to perform a nerve block immediately following an injury due to the risk of masking a compartment syndrome. C is incorrect. Although the incision for a total knee replacement is likely to be covered by the femoral nerve block, the substantial manipulation of the structures posterior to the knee joint, even though accessed anteriorly, would mean that the sciatic nerve would also have to be anesthetized. D is incorrect. As mentioned above, the medial aspect of the lower leg has sensory innervation that originates from the femoral nerve, but the lateral malleolus would be innervated from branches of the sciatic nerve. 19. B is correct. When performing an ultrasound-guided femoral nerve block, the needle must pass through the fascia iliaca in order to reach the femoral nerve. This will be felt as a pop by the anesthetic provider. A is incorrect. When performing a femoral nerve block, the femoral crease should be identified as a starting landmark and the femoral pulse can be palpated medially to the nerve as guidance for the ultrasound transducer. However, other than the femoral crease, no surface landmarks are needed for this block. When visualizing the femoral nerve by ultrasound, the sartorius may be seen as a triangular structure lateral and superficial to the nerve. C is incorrect. The spread of the local anesthetic will not necessarily be circumferential due to the vascular structures as well as the strength of the surrounding fascia and sheaths; however, a circumferential spread is not imperative for a complete and reliable block. D is incorrect. After identification of the femoral nerve and artery, the needle is inserted with the goal of injecting local anesthetic immediately adjacent to the nerve. The needle should be inserted when the visualized structures show only a single artery—the femoral artery. If the image is too distal, it will show the femoral artery as well as the deep artery of the thigh (profunda femoris). At this point, it is possible that branches of the femoral nerve have already split off and the injection should be made more proximal.

Suggested Reading Hadzic A. Ultrasound-guided femoral nerve block. In: Atchabahian A, Leunen I, Vandepitte C, Lopez AM, eds. Textbook of Regional Anesthesia and Acute Pain Management. 2nd ed. New York, NY: McGraw-Hill Publishing; 2017:chap 33A.

19/04/19 11:29 AM

32B Ultrasound-Guided Fascia Iliaca Block Matthias Desmet, Arthur Atchabahian, and Jennifer L. Cowell

QUESTIONS DIRECTIONS: Choose the one best response to each question. 1. Which of the following statements regarding the anatomy of the fascia iliaca is true? A. The deep circumflex artery is covered by the fascia iliaca. B. The fascia iliaca is bound superiorly to the inguinal ligament. C. The fascia iliaca merges with the fascia overlying the psoas muscle. D. The sartorius muscle is covered by the fascia iliaca. 2. Which of the following statements regarding the use of a fascia iliaca compartment block (FICB) in clinical practice is true? A. An FICB significantly reduces morphine consumption after total hip arthroplasty. B. An FICB significantly reduces morphine consumption in patients with a hip fracture. C. An FICB facilitates early mobilization after lower limb surgery. D. To avoid contractions of the adductor muscles during transurethral bladder resections, an FICB can be used. 3. Which of the following statements regarding the technique of a fascia iliaca compartment block (FICB) is true? A. Injection close to the femoral nerve will result in cranial spread toward the lumbar plexus. B. One can reduce the volume of local anesthetics and still preserve the efficacy of an FICB if a suprainguinal approach is used. C. Spread observed medial to the femoral artery guarantees a block of the common branch of the obturator nerve. D. To ensure adequate spread under the fascia iliaca, one should release the pressure exerted with the ultrasound probe.

4. Which of the following statements is true? A. If a high opening injection pressure injection is detected during a fascia iliaca compartment block (FICB), the needle should be withdrawn. B. The fascia iliaca should be separated from the sartorius muscle to obtain a successful block. C. The FICB can be safely performed in patients under general anesthesia. D. The landmark “double pop” technique is a valuable alternative for an ultrasound-guided FICB. 5. A fascia Iliaca block is performed for postoperative pain on a patient undergoing arthroplasty of the hip through an anterior approach. In the PACU the patient complains of pain medial to the operative site due to: A. Continuous pressure on the groin by the center post of the surgical table B. Medial capsular pain from innervation by the anterior branch of the obturator nerve C. Natural anatomic variation causing sparing of the lateral femoral cutaneous nerve D. Pressure from a large volume of local anesthetic infiltrated into the fascial layers 6. The documented predictor of success of the fascia iliaca block is: A. Placement of local anesthetic in a concentrated location adjacent to the femoral nerve B. Plantar flexion of the ankle at a current level of 0.4 mA with a pulse width of 0.3 msec with nerve stimulation C. Spread of local anesthetic toward the femoral nerve medially and underneath the sartorius muscle laterally D. Utilizing a low-volume, high-concentration local anesthetic for infiltration 7. Relevant surface landmarks for performing an ultrasound-guided fascia iliaca block include: A. Anterior superior iliac spine (ASIS) and pubic tubercle B. Femoral artery and iliopsoas muscle C. Femoral artery and sartorius muscle D. Sartorius muscle and greater trochanter

195

Hadzic_Ch32B_p195-200.indd 195

19/04/19 11:29 AM

196

PART 3

Clinical Practice of Regional Anesthesia

8. Which statement is true regarding the multiple approaches to the fascia iliaca block? A. All approaches demonstrate local anesthetic spread underneath fascia iliaca proximally toward the lumbosacral plexus. B. The “double pop” technique describes the tactile sensation of passing through the fascia lata and fascia iliaca. C. The suprainguinal approach has been shown to be more consistently reliable for blocking the lateral femoral cutaneous nerve. D. The suprainguinal approach will have a more proximal spread of anesthetic as the inguinal ligament is not compressing the compartment proximally. 9. When considering a fascia iliaca block, A. Only the femoral nerve lies under the fascia iliaca within the pelvis. B. The fascia iliaca is located anterior to the iliacus muscle. C. The lateral femoral cutaneous nerve often needs a supplemental block. D. The psoas muscle is located deep and medial to the femoral artery. 10. The figure below shows an ultrasound image of the fascia iliaca (arrows). Which letter corresponds to the sartorius muscle?

A

Lateral

C

D

B

A. A B. B C. C D. D 11. A fascia iliaca block would be most useful for which of the following patients? A. A patient who has been hospitalized for medical optimization for 5 days after a hip fracture B. A patient who has been hospitalized for medical optimization for 5 days after a lateral malleolus fracture C. A patient who is 6 hours status post motor vehicle crash with a midshaft tibia fracture requiring open reduction and internal fixation (ORIF) D. A patient who will undergo an elective hip replacement by the lateral approach

Hadzic_Ch32B_p195-200.indd 196

12. A fascia iliaca compartment block: A. Can provide surgical anesthesia for a total hip replacement B. Has a higher likelihood to block the lateral femoral cutaneous nerve than a femoral nerve block C. Reliably blocks the obturator nerve D. Will typically not block the saphenous nerve 13. The suprainguinal approach to the fascia iliaca compartment block: A. Appears to provide better analgesia following hip surgery than the classic fascia iliaca compartment block B. Can provide surgical anesthesia for a total hip replacement C. Efficacy mostly depends on a high concentration of local anesthetic D. Has been demonstrated to provide similar blockade to a posterior lumbar plexus (psoas compartment) block

ANSWERS AND EXPLANATIONS 1. C is correct. The fascia iliaca merges medially with the fascia overlying the psoas muscle. A is incorrect. The deep circumflex artery is located superior to the fascia iliaca and as such can be an important landmark. If spread of local anesthetic is observed around the deep circumflex artery, the deposition of local anesthetic was above the fascia iliaca. B is incorrect. The fascia iliaca covers the entire iliac muscle and attaches cranially to the iliac crest. D is incorrect. The sartorius muscle is located superior to the iliac muscle and superior to the fascia iliaca. 2. B is correct. Numerous studies have investigated the effect of an FICB for perioperative analgesia in patients with hip fractures. Although these studies are very heterogeneous (different patient populations; different FICB techniques; different volumes, concentrations, and types of local anesthetics), the overall conclusion is that the FICB decreases analgesic requirements, pain scores, and the incidence of postoperative complications.1,2 A is incorrect. A recent study by Shariat et al could not demonstrate a beneficial effect of an FICB on postoperative pain after total hip arthroplasty. As the hip joint is innervated by both the lumbar and sacral plexus, it is clear that an FICB will not fully cover postoperative pain.3 C is incorrect. As an FICB will consistently block the femoral nerve with an associated quadriceps weakness, the FICB may impede early mobilization. D is incorrect. All adductor muscles, except the pectineus muscle, are innervated by the obturator nerve. As the obturator nerve is not reliably blocked by an FICB, the FICB is not the block of choice to avoid adduction during transurethral bladder resection.

19/04/19 11:29 AM

CHAPTER 32B

3. D is correct. Releasing the pressure exerted with the ultrasound probe will allow local anesthetics to spread medially and laterally. A is incorrect. The FICB described by Dalens4 was based on the idea that injection under the fascia iliaca would lead to cranial spread toward the lumbar plexus. However, radiological research could not demonstrate spread toward the lumbar plexus.5 B is incorrect. The efficacy of the FICB depends on the injection of an adequate volume of local anesthetics (20–40 mL); the suprainguinal approach might lead to a more proximal spread but even with this approach an adequate volume is necessary.6 C is incorrect. The obturator nerve is not blocked by an FICB; the iliopectineal fascia forms an effective barrier for local anesthetics.5 4. C is correct. The FICB is a “plane block” where local anesthetics are injected between the fascia and the iliac muscle. An injection close to a nerve is not necessary for a successful block; therefore, the risk of intraneural injection is nonexistent. Therefore, the FICB can be safely performed in asleep patients. A is incorrect. High opening injection pressures are indicative for intraneural needle placement. Therefore, the needle should be withdrawn when such a high opening pressure is detected. Gadsden et al demonstrated that needle–fascia contact will also lead to high opening injection pressures. In such cases, when the needle is withdrawn, the injection will occur above the fascia iliaca, leading to an unsuccessful block.7 B is incorrect. An injection separating the fascia iliaca from the iliac muscle is necessary for a successful FICB. An injection between the fascia iliaca and the sartorius muscle will be ineffective. D is incorrect. Dolan et al demonstrated that the ultrasoundguided FICB led to more consistent effects because of a more accurate placement of local anesthetic under the fascia iliaca compared to a landmark technique.8 5. B is correct. The fascia iliaca block does not cover the anterior branch of the obturator nerve, which innervates the medial aspect of the articular capsule and acetabulum of the hip joint as well as (inconstantly) the skin of the medial thigh. A is incorrect. An anterior approach to a hip replacement involves using a table with a center post in the patient’s groin, which causes counter traction when the hip is displaced caudally and externally rotated. There have been multiple reports of postoperative numbness, possibly due to compression of the pudendal or cutaneous branch of the obturator nerves, but not pain. C is incorrect. The fascia iliaca block is performed at a level that is more proximal than the femoral nerve block but distal to a lumbar plexus block. The injection is performed under the fascia iliaca and local anesthetic will diffuse to the femoral nerve sheath and due to its location, to the lateral femoral cutaneous nerve, which is anesthetized

Hadzic_Ch32B_p195-200.indd 197

Ultrasound-Guided Fascia Iliaca Block

197

at least 80% of the time. The lateral femoral cutaneous nerve does not provide sensation to the medial aspect of the hip. D is incorrect. The fascia iliaca is a large-volume block of typically up to 40 mL of local anesthetic used for infiltration. While this may be painful during initial injection, once the nerves are anesthetized, it is unlikely that this would be the cause of pain multiple hours later. 6. C is correct. When performing a fascia iliaca block, it is ideal to see a spread of local anesthetic along the fascial plane both medial and laterally. A is incorrect. While the anesthetic should be seen going in the direction of the femoral nerve, this block is performed within a fascial plane; therefore, adjacent deposition is unlikely to happen and is not ideal. B is incorrect. This block is performed under ultrasound guidance and without nerve stimulation. The goal of the block is to provide spread of local anesthetic along the plane toward the femoral nerve and to not be directly addressing the femoral nerve itself. As such, there is no role for nerve stimulation in this block. Additionally, plantar flexion is caused by stimulation of the sciatic nerve. D is incorrect. The fascia iliaca is a large-volume block of typically up to 40 mL of local anesthetic used for infiltration, as compared to 15–20 mL for most other regional blocks. The amount is typically considered adequate for an adult, with 0.7 mL/kg for a pediatric patient. 7. A is correct. When performing the fascia iliaca block, the ASIS and the pubic tubercle are identified. This is the level of the inguinal ligament. The ultrasound transducer should be placed on this ligament as a starting location for identification of structures. B and C are incorrect. Palpation of the femoral artery can be done, but this does not determine the starting location for placement of the ultrasound for this block. By moving the ultrasound probe caudally from the inguinal ligament toward the femoral crease, the femoral artery should be identified on the image medially, the sartorius muscle laterally, and the iliopsoas muscle deep to the other structures. Separating these structures is the fascia iliaca. D is incorrect. The greater trochanter does not serve as a landmark in this block. 8. B is correct. Prior to ultrasound, or in a setting where ultrasound is unavailable, the “double pop” technique may be used to perform this block. After identifying landmarks, the practitioner inserts the needle, feeling two pops on the way in, the fascia lata, followed by the fascia iliaca. While historically, this was a way to perform this block, it is not reliable as it is dependent on the subjective sensation of the practitioner and many false positives can prevent actual blockade of the nerves. A is incorrect. The theoretical path for the spread of the local anesthetic is that it travels toward the lumbosacral plexus. This has been unable to be reliably demonstrated despite adequate blockade of the femoral nerve.

19/04/19 11:29 AM

198

PART 3

Clinical Practice of Regional Anesthesia

C is incorrect. The suprainguinal approach involves rotating the ultrasound 90 degrees at the inguinal ligament starting point until the sartorius muscle is visualized laterally and caudally, the external oblique laterally and cephalad, and the iliacus muscle medially. D is incorrect. To date, a difference in the reliability of blocking the lateral femoral cutaneous nerve or proximal spread of local anesthetic with this technique has not been demonstrated. 9. B is correct. The fascia iliaca is a fascial plane just superficial and lateral to the femoral artery covering the iliacus muscle. It exits the pelvis at the inguinal ligament, which is the most cephalad border of this block. A is incorrect. Within the pelvis, the femoral nerve and the lateral femoral cutaneous nerve are covered by this fascia. The branch point is outside of the pelvis. C is incorrect. Because this is a more proximal block than the femoral block, the lateral femoral cutaneous nerve is often anesthetized with the same injection and supplemental block is not usually needed. D is incorrect. Deep to the fascia iliaca is the iliacus muscle and the psoas muscle, both of which are lateral and deep to the femoral artery. 10. A is correct. See figure below. The sartorius muscle (A) is lateral to the iliopsoas muscle and the femoral neurovascular bundle (C: femoral nerve; D: femoral artery) when visualized on ultrasound in the inguinal area. It is most often triangular in cross section and it is superficial to the fascia iliaca. Below the fascia iliaca and the sartorius muscle is the iliopsoas muscle (B).

SM

Lateral

FN

FA

Iliopsoas M.

11. A is correct. A fascia iliaca block is appropriate for pain management of a patient who is awaiting surgery for a broken hip. The femoral nerve provides sensory innervation to the hip and knee joints and a fascia iliaca block more than likely also covers the lateral femoral cutaneous nerve, which provides sensation to the lateral aspect of the hip and surrounding muscles. B is incorrect. A fascia iliaca block covers the distribution of the femoral nerve; therefore it would not be adequate for someone awaiting repair of the lateral malleolus. C is incorrect. A fascia iliaca block would not be adequate for the repair of the tibial shaft. Because the femoral nerve

Hadzic_Ch32B_p195-200.indd 198

does not provide sensation to all aspects of the midshaft of the tibia, repair of this bone would also require blockade of the branches of the sciatic nerve. However, and despite the absence of evidence, some authors advise against using regional anesthesia for acute fractures due to the possibility of masking compartment syndrome. D is incorrect. By contrast, the fixation itself, or elective replacement, would also require at least blockade of the sciatic nerve due to the manipulation of tissue posterior to the distribution of the femoral nerve. 12. B is correct. The lateral femoral cutaneous nerve (LFCN) passes under the inguinal ligament lateral to the femoral nerve and anterior to the sartorius muscle as opposed to posterior. It is either outside of the femoral triangle or at the very edge. However, both pass under the fascia iliaca as they continue distally. Because of this, it is less likely that the LFCN will be blocked doing a femoral nerve block as opposed to a fascia iliaca block. A is incorrect. A fascia iliaca block can help with hip pain from a fracture or with postoperative pain due to hip surgery, but the hip joint and surrounding soft tissue has sensory innervation from more than just the LFCN and the femoral nerve. A fascia iliaca block would not be able to provide surgical anesthesia for a hip surgery. C is incorrect. The additional innervation of the hip comes from the superior gluteal nerve, the quadratus femoris nerve, the sciatic nerve, and the obturator nerve, none of which pass under the fascia iliaca. D is incorrect. While the fascia iliaca block can be considered an alternative to the femoral nerve block, the local anesthetic is more likely to spread along the plane and encountering both nerves. Like a femoral nerve block, the sensory blockade will include the distribution of the saphenous nerve. 13. A is correct. The fascia iliaca block can be performed two ways—either above or below the inguinal ligament. Below the inguinal ligament, the fascia iliaca block can be thought of as an alternative to the femoral block with the added benefit of involving the lateral femoral cutaneous nerve. The inguinal ligament, similar to the clavicle in upper extremity blocks, serves as a barrier to prevent spread and contain the local anesthetic. While further studies are needed to definitively prove improved analgesia, it has been seen that the suprainguinal approach provides better postoperative pain relief after hip surgery than the infrainguinal approach. B is incorrect. This is thought to be explained by the increase in proximal spread providing more coverage to branching nerves. While it appears to provide better pain relief, it cannot be used for surgical anesthesia for a hip surgery because the hip is also innervated by the obturator nerve, the superior gluteal nerve, the quadratus femoris nerve, and the sciatic nerve. C is incorrect. As with the infrainguinal approach, the suprainguinal approach to the fascia iliaca is dependent on a large volume of local anesthetic rather than a high concentration of local anesthetic.

19/04/19 11:29 AM

CHAPTER 32B

D is incorrect. The fact that it does not reliably block the obturator nerve means that it is most similar to a femoral nerve block rather than a psoas compartment block.

References 1. Monzon DG, Iserson KV, Vazquez JA. Single fascia iliaca compartment block for post-hip fracture pain relief. J Emerg Med. 2007;32:257-262. 2. Foss NB, Kristensen BB, Bundgaard M, et al. Fascia iliaca compartment blockade for acute pain control in hip fracture patients. Anesthesiology. 2007;106:773-778. 3. Shariat AN, Hadzic A, Xu D, et al. Fascia iliaca block for analgesia after hip arthroplasty. Reg Anesth Pain Med. 2013;38:201-205. 4. Dalens B, Vanneuville G, Tanguy A. Comparison of the fascia iliaca compartment block with the 3-in-1 block in children. Anesth Analg. 1989;69:705–13. 5. Swenson JD, Davis JJ, Stream JO, Crim JR, Burks RT, Greis PE. Local anesthetic injection deep to the fascia iliaca at the level of

Hadzic_Ch32B_p195-200.indd 199

Ultrasound-Guided Fascia Iliaca Block

199

the inguinal ligament: the pattern of distribution and effects on the obturator nerve. J Clin Anesth. 2015;27(8):652-657. 6. Hebbard P, Ivanusic J, Sha S. Ultrasound-guided supra-inguinal fascia iliaca block: a cadaveric evaluation of a novel approach. Anaesthesia. 2011;66(4):300-305. 7. Gadsden J, Latmore M, Levine DM, Robinson A. High opening injection pressure is associated with needle-nerve and needlefascia contact during femoral nerve block. Reg Anesth Pain Med. 2016;41:50-55. 8. Dolan J, Williams A, Murney E, Smith M, Kenny GN. Ultrasound guided fascia iliaca block: a comparison with the loss of resistance technique. Reg Anesth Pain Med. 2008;33:526-531.

Suggested Reading Hadzic A. Ultrasound-guided fascia iliaca block. In: Atchabahian A, Leunen I, Vandepitte C, Lopez AM, eds. Textbook of Regional Anesthesia and Acute Pain Management. 2nd ed. New York, NY: McGraw-Hill Publishing; 2017:chap 33B.

19/04/19 11:29 AM

This page intentionally left blank

Hadzic_FM_p00i-pxx.indd 6

22/04/19 6:40 PM

32C Ultrasound-Guided Lateral Femoral Cutaneous Nerve Block Astrid Van Lantschoot

QUESTIONS DIRECTIONS: Choose the one best response to each question. 1.  Regarding the lateral femoral cutaneous nerve (LFCN), which of the following statements is correct? A. The LFCN is visualized between the tensor fasciae latae and the sartorius muscle. B. The LFCN is 1–2 cm superior to the anterior superior iliac spine (ASIS). C. The LFCN is a hyperechogenic structure with a hypogenic rim. D. The LCFN is easily followed distally. 2.  Regarding the block of the LFCN, which of the following statements is correct? A. The patient is positioned supine with the legs flexed. B. A curved probe is ideal when performing an LFCN block. C. Landmark techniques are very accurate when performing an LFCN block. D. A needle with a length between 3–5 cm is used for an LFCN block. 3.  Regarding the block of the LFCN, which of the following statements is correct? A. Scrotal pain is a good indication for performing block of LFCN. B. Meralgia paresthetica is a good indication for performing block of LFCN. C. In children, a volume of 1.5 mL/kg is necessary. D. The needle is inserted in a medial-to-lateral orientation. 4.  Sensory distribution of a lateral femoral cutaneous block is better described as: A. Anesthesia in the anterolateral thigh B. Anesthesia of the anterior and medial thigh down to and including the knee C. Anesthesia of the lower limb below the knee D. Anesthesia of the skin of the medial aspect of the thigh

5.  Select the correct statement regarding the technique for a lateral femoral cutaneous block: A. The nerve should be identified every time before needle insertion. B. In adults, a minimum volume of 10 mL should be used. C. Transducer is placed just inferior to the anterior superior iliac spine (ASIS), parallel to the inguinal ligament. D. The goal is to inject local anesthetic in the plane between the sartorius muscle and the iliopsoas, underneath the fascia iliaca.

ANSWERS AND EXPLANATIONS 1. A is correct. The LFCN is visualized between the tensor fasciae latae and the sartorius muscle. B is incorrect. The LFCN is 1–2 cm inferior to the ASIS. C is incorrect. The LFCN is a hypoechogenic structure with a hyperechogenic rim. D is incorrect. The LFCN can be followed more proximally; distally it disappears. 2. D is correct. A needle with a length between 3–5 cm is used for an LFCN block. A is incorrect. Patient should have legs extended. B is incorrect. A linear probe is used. C is incorrect. There is a highly variable anatomy, which makes it very challenging for landmark-based technique. 3. B is correct. Meralgia paresthetica is a good indication for performing block of LFCN. A is incorrect. Innervation of LFCN is sensory on the lateral aspect of the thigh. C is incorrect. A volume of 0.15 mL/kg is sufficient in children. D is incorrect. The needle is inserted in a lateral-to-medial orientation. 201

Hadzic_Ch32C_p201-202.indd 201

22/04/19 6:07 PM

202

PART 3

Clinical Practice of Regional Anesthesia

4. A is correct. A block of the lateral femoral cutaneous nerve provides (LFCN) anesthesia or analgesia in the anterolateral thigh. There is a large variation in the area of sensory coverage among individuals because of the highly variable course of the LFCN and its branches. B is incorrect. This sensory distribution corresponds to the femoral nerve. C is incorrect. This sensory distribution corresponds to the popliteal sciatic nerve block. D is incorrect. This sensory distribution corresponds to the obturator nerve. 5. C is correct. With the patient supine, the skin is disinfected and the transducer is placed inferior to the ASIS, parallel to the inguinal ligament. A is incorrect. The correct needle tip position is confirmed by visualizing the spread of local anesthetic in the

Hadzic_Ch32C_p201-202.indd 202

described plane between the tensor fasciae latae muscle (TFLM) and the sartorius muscle (SaM) without necessarily attempting to visualize the nerve. B is incorrect. In an adult patient, 5 mL of local anesthetic is usually sufficient. D is incorrect. The goal is to inject local anesthetic in the plane between the TFLM and the SaM, typically 1–2 cm medial and inferior to the ASIS.

Suggested Reading Hadzic A. Ultrasound-guided lateral femoral cutaneous nerve block. In: Clark TB, Lopez AM, Xu D, Vandepitte C, eds. Textbook of Regional Anesthesia and Acute Pain Management. 2nd ed. New York, NY: McGraw-Hill Publishing; 2017:chap 33C.

22/04/19 6:07 PM

32D Ultrasound-Guided Obturator Nerve Block J. Douglas Jaffe

QUESTIONS DIRECTIONS: Choose the one best response to each question. 1.  Which of the following structures is most likely to remain with intact sensation following the performance of an obturator nerve block under ultrasound? A. Medial thigh skin B. Adductor longus muscle C. Medial hip joint D. Medial knee joint 2.  Between which of the following two muscles is the posterior branch of the obturator nerve most likely to be encountered when performing an ultrasound guided obturator nerve block? A. Adductor longus and pectineus B. Adductor brevis and adductor magnus C. Adductor longus and adductor magnus D. Adductor brevis and pectineus 3.  Between which of the following two muscles is the anterior branch of the obturator nerve most likely to be encountered when performing an ultrasound guided obturator nerve block? A. Adductor longus and pectineus B. Adductor brevis and adductor magnus C. Adductor longus and adductor magnus D. Adductor brevis and pectineus 4.  Under spinal anesthesia, the addition of an obturator nerve block is indicated to prevent involuntary abduction of the thigh during which of the following surgical procedures? A. Transurethral resection of a lateral bladder tumor B. Iliopsoas hematoma evacuation C. Femoral arterial bypass revascularization D. groin exploration

5.  To perform blocks of the anterior and posterior branches of the obturator nerve (obturator block), needle tip should be positioned: A. Between the fascial plane of the pectineus and adductor brevis muscle B. In the fascial plane between the adductor brevis and adductor magnus muscles C. Between the pectineus and adductor brevis muscles, and between the adductor brevis and adductor magnus muscles, respectively D. Between the adductor brevis and adductor magnus to block the 2 branches of the nerve

ANSWERS AND EXPLANATIONS 1. C is correct. During the performance of an ultrasound obturator nerve block, the articular branches are least likely to be blocked as they arise (80% of the time) from the common obturator nerve, which isn’t routinely blocked during the ultrasound guided approach.1-3 A, B, and D are incorrect. These choices are more reliably anesthetized using an ultrasound proximal thigh obturator nerve block. 2. B is correct. The posterior branch lies between the fascial planes of the adductor brevis and adductor magnus muscles.4 A, C, and D are incorrect. These choices are unlikely to contain the posterior branch of the obturator nerve. 3. D is correct. In the thigh, at the level of the femoral crease, the anterior branch is located between the fascia of the pectineus and adductor brevis muscles.4 A, B, and C are incorrect. These choices are unlikely to contain the anterior branch of the obturator nerve.

203

Hadzic_Ch32D_p203-204.indd 203

19/04/19 11:28 AM

204

PART 3

Clinical Practice of Regional Anesthesia

4.  A is correct. A serious complication of transurethral resection of a bladder tumor is bladder perforation, the risk of which is greatly increased in the presence of an “obturator jerk”. Anesthetists should consider the use an obturator nerve block to reduce the incidence of obturator jerk and risk of bladder perforation.1,2,5 B, C, and D are incorrect. These choices are not likely to cause involuntary adductor muscle activation. The lateral bladder tumor excision with cautery often stimulates the obturator nerve as it passes to its distal target structures inducing involuntary adduction of the ipsilateral thigh when neuromuscular blockade is absent (ie, spinal motor block proximal to the adductor muscle group). 5.  C is correct. The block needle is advanced to initially position the needle tip between the pectineus and adductor brevis (to block the anterior branch). At this point, 5–10 mL of local anesthetic solution is injected. The needle is advanced further to position the needle tip between the adductor brevis and adductor magnus muscles (to block the posterior branch), and another 5–10 mL of local anesthetic solution is injected.

2. Sakura S, Hara K, Ota J, Tadenuma S. Ultrasound-guided peripheral nerve blocks for anterior cruciate ligament reconstruction: effect of obturator nerve block during and after surgery. J Anesth. 2010;24:411-417. 3. Anagnostopoulou S, Kostopanagiotou G, Paraskeuopoulos T, et al. Anatomic variations of the obturator nerve in the inguinal region: implications in conventional and ultrasound regional anesthesia techniques. Reg Anesth Pain Med. 2009 Jan-Feb;34(1):33-9. 4. Sinha SK, Abrams JH, Houle TT, Weller RS. Ultrasound-guided obturator nerve block: an interfascial injection approach without nerve stimulation. Reg Anesth Pain Med. 2009 May-Jun;34(3):261-264. 5. Panagoda PI, Vasdev N, Gowrie-Mohan S. Avoiding the obturator jerk during TURBT. Curr Urol. 2018 Oct;12(1):1-5. doi: 10.1159/000447223. Epub 2018 Jun 30.

Suggested Reading Hadzic A. Ultrasound-guided obturator nerve block. In: Boxstael SV, Vandepitte C, Gautier PE, Jalil H, eds. Textbook of Regional Anesthesia and Acute Pain Management. 2nd ed. New York, NY: McGraw-Hill Publishing; 2017:chap 33D.

A, B, and D are incorrect. See explanation for answer C.

References 1. Akkaya T, Comert A, Kendir S, et al. Detailed anatomy of accessory obturator nerve blockade. Minerva Anestesiol. 2008;74:119-122.

Hadzic_Ch32D_p203-204.indd 204

19/04/19 11:28 AM

32E Ultrasound-Guided Saphenous (Subsartorius/Adductor Canal) Nerve Block Thomas F. Bendtsen and Siska Bjørn

QUESTIONS DIRECTIONS: Choose the one best response to each question. 1.  The saphenous nerve: A. Innervates the lateral aspect of the knee B. Sends infrapatellar branches to the knee joint C. Supplies only skin and subcutaneous structures D. Typically innervates the skin at the entire medial side of the foot to the base of the first toe 2.  Which of the following statements is true regarding the course of the saphenous nerve? A. There is no consistent landmark for the saphenous nerve in the lower leg. B. The saphenous nerve lies in the adductor canal in close proximity to the femoral artery and vein. C. The saphenous nerve lies posterior to the medial malleolus before it terminates in several fine branches. D. The saphenous nerve pierces the fascia lata to become subcutaneous posterior to the tendon of the gracilis muscle. 3.  For which of the following indications may a saphenous nerve block be omitted? A. Forefoot surgery B. Major ankle surgery C. Saphenous vein stripping D. Total knee arthroplasty 4.  Which statement is true regarding a saphenous nerve block? A. A large volume of local anesthetic is necessary to provide sufficient anesthesia of the saphenous nerve. B. A proximal approach provides better anesthesia of the saphenous nerve and quicker onset of the block compared to a more distal approach. C. Minor quadriceps weakness due to anesthesia of the medial vastus nerve is a potential consequence of a proximal approach.

D. The saphenous nerve must always be visualized before injection of local anesthetic. 5.  A saphenous nerve block requires: A. An appropriate motor response with electrical nerve stimulation B. A proximal midthigh approach for pain relief after knee surgery C. High concentration of local anesthetic D. Injection of at least 20 mL of local anesthetic

ANSWERS AND EXPLANATIONS 1. B is correct. The saphenous nerve sends infrapatellar branches to the knee joint. A is incorrect. The saphenous nerve does not innervate the lateral aspect of the knee. The infrapatellar branches innervate the anteromedial part of the knee. Anterolaterally the knee is innervated by the lateral femoral cutaneous nerve, the common peroneal nerve, and the femoral nerve branch to the lateral vastus muscle.1 C is incorrect. The saphenous nerve also supplies deeper structures. Cadaver studies have verified nerve endings form the saphenous nerve in the medial and ventral parts of the joint capsules of the ankle and the talocalcaneonavicular joint and the periosteum of the medial malleolus.2–4 D is incorrect. In the majority of cases, the saphenous nerve extends only to the midfoot.4,5 2. B is correct. The saphenous nerve lies in the adductor canal in close proximity to the femoral artery and vein. A is incorrect. The saphenous nerve lies in close proximity to the great saphenous vein in the lower leg. For the paravenous approach at the level of the tibial tuberosity the great saphenous vein serves as an important landmark.6 C is incorrect. The saphenous nerve follows the great saphenous vein in its entire course and therefore also courses anteriorly to the medial malleolus. 205

Hadzic_Ch32E_p205-206.indd 205

19/04/19 11:28 AM

206

PART 3

Clinical Practice of Regional Anesthesia

D is incorrect. The saphenous nerve consistently pierces the fascia lata and becomes subcutaneous between the tendons of the sartorius and gracilis muscles. On the other hand, the infrapatellar branch from the saphenous nerve shows much more variation in relation to the distal part of the sartorius muscle by passing either anterior, posterior, or through it.7,8 3. A is correct. Forefoot surgery is not mentioned as an indication. For forefoot surgery the saphenous nerve block can be omitted in 97% of cases.5 B is incorrect. A saphenous nerve block should always be included as a supplement to a sciatic nerve block for major ankle surgery. The saphenous nerve innervates the medial and ventral parts of the joint capsules of the ankle and the talocalcaneonavicular joint and the periosteum of the medial malleolus.2–4,9 C is incorrect. The great saphenous vein lies in the innervation area of the saphenous nerve, and a saphenous nerve block is therefore indicated for great saphenous vein stripping or harvesting. D is incorrect. A saphenous nerve block should be included for pain relief after total knee arthroplasty. A proximal approach is necessary to anesthetize the infrapatellar nerve before it branches off the saphenous nerve, and to anesthetize the medial vastus nerve, which is also most likely involved in pain after total knee arthroplasty.1 4. C is correct. Minor quadriceps weakness due to anesthesia of the medial vastus nerve is a potential consequence of a proximal approach. A is incorrect. A volume of 5–10 mL of local anesthetic is sufficient. For the adductor canal block a larger volume can be used.6 B is incorrect. A study in volunteers comparing the perifemoral, transsartorial, below the knee field block, medial femoral condyle block, and medial malleolus block does not have results consistently in favor of any of the block types.10 The onset of sensory blockade in the medial aspect of the lower leg was quicker for the transsartorial approach compared to the block at the medial femoral condyle. The onset of the sensory blockade in the medial foot was quicker for the transsartorial and medial malleolus approaches compared to the perifemoral approach.10 D is incorrect. Direct visualization of the saphenous nerve is not always necessary. The saphenous nerve can be very difficult to visualize, and therefore landmarks such as the femoral artery, the descending genicular artery, or the great saphenous vein can be used instead.6 5. B is correct. A midthigh approach for the saphenous nerve block is recommended for pain relief after total knee arthroplasty.11 A proximal approach is required to anesthetize the infrapatellar nerve before it branches of the saphenous nerve and to anesthetize the medial vastus nerve, which is also most likely involved in pain after total knee arthroplasty.1

Hadzic_Ch32E_p205-206.indd 206

A is incorrect. The saphenous nerve is a purely sensory nerve, and therefore no motor response is elicited. Instead the appropriate response when using electrical nerve stimulation for the saphenous nerve block is a paresthesia in the saphenous nerve distribution.6 C is incorrect. The saphenous nerve is a purely sensory nerve and high concentrations of local anesthetic are not required. High concentrations may instead delay patient ambulation if the local anesthetic spreads to motor branches of the femoral nerve innervating the quadriceps muscle.6 D is incorrect. The volume used for the saphenous nerve block is typically 5–10 mL. For the adductor canal block a larger volume of 20–30 mL may be used.6

References 1. Bendtsen TF, Moriggl B, Chan V, Borglum J. The optimal analgesic block for total knee arthroplasty. Reg Anesth Pain Med. 2016;41:711-719. 2. Eglitis N, Horn JL, Benninger B, Nelsen S. The importance of the saphenous nerve in ankle surgery. Anesth Analg. 2016;122:1704-1706. 3. Clendenen SR, Whalen JL. Saphenous nerve innervation of the medial ankle. Local Reg Anesth. 2013;6:13016. 4. Marsland D, Dray A, Little NJ, Solan MC. The saphenous nerve in foot and ankle surgery: its variable anatomy and relevance. Foot Ankle Surg. 2013;19:76-79. 5. López AM, Sala-Blanch X, Magaldi M, Poggio D, Asuncion J, Franco CD. Ultrasound-guided ankle block for forefoot surgery: the contribution of the saphenous nerve. Reg Anesth Pain Med. 2012;37(5):554-557. 6. Hadzic A. Textbook of Regional Anesthesia and Acute Pain Management. 2nd ed. New York, NY: McGraw-Hill Publishing; 2017. 7. Henry BM, Tomaszewski KA, Pekala PA, et al. The variable emergence of the infrapatellar branch of the saphenous nerve. J Knee Surg. 2017;30(6):585-593. 8. Kalthur SG, Sumalatha S, Nair N, Pandey AK, Sequeria S, Shobha L. Anatomic study of infrapatellar branch of saphenous nerve in male cadavers. Ir J Med Sci. 2015;184:201-206. 9. Chen J, Lesser J, Hadzic A, Resta-Flarer F. The importance of the proximal saphenous nerve block for foot and ankle surgery. Reg Anesth Pain Med. 2013;38:372. 10. Benzon HT, Sharma S, Calimaran A. Comparison of the different approaches to the saphenous nerve block. Anesthesiology. 2005;102(3):663-668. 11. Kopp SL, Børglum J, Buvanendran A, et al. Anesthesia and analgesia practice pathway options for total knee arthroplasty: an evidence-based review by the American and European Societies of Regional Anesthesia and Pain Medicine. Reg Anesth Pain Med. 2017;42(6):683-697.

Suggested Reading Hadzic A. Ultrasound-guided saphenous (subsartorius/adductor canal) nerve block. In: Bendtsen TF, Lopez AM, Clark TB, eds. Textbook of Regional Anesthesia and Acute Pain Management. 2nd ed. New York, NY: McGraw-Hill Publishing; 2017:chap 33E.

19/04/19 11:28 AM

32F Ultrasound-Guided Sciatic Nerve Block J. Douglas Jaffe, Arthur Atchabahian, and Jennifer L. Cowell

QUESTIONS DIRECTIONS: Choose the one best response to each question. 1. When compared to the posterior approach, what is the disadvantage of the anterior approach to a sciatic nerve block? A. It may be difficult to position a trauma patient in order to perform this block anteriorly. B. It cannot be performed using ultrasound guidance, but is done with landmarks and nerve stimulation. C. It is not anatomically well-suited for a placement of continuous infusion catheter. D. It provides a smaller area of sensory blockade than a block done at the subgluteal level by a posterior approach. 2. Which statement is true when comparing the anterior approach to the transgluteal posterior approach for a sciatic nerve block? A. The anterior approach is better suited than the transgluteal approach for placement of a continuous infusion catheter. B. The transgluteal approach is done using a linear array probe while the anterior approach uses a curvilinear probe. C. The transgluteal approach is better suited than the anterior approach for a tibial osteotomy. D. The transgluteal approach provides more cutaneous coverage than the anterior approach. 3. Which statement is true regarding performing an anterior sciatic block? A. A linear array probe should be placed on the medial aspect of the thigh.

B. An out-of-plane technique is practical due to the steep angle of insertion. C. Nerve stimulation should yield a patellar twitch at 1.0 mA, 0.1 msec. D. The patient should be supine with the leg extended and in adduction. 4. Which statement is true when deciding between a transgluteal and subgluteal approach to the sciatic block? A. A linear ultrasound probe can be used for both the transgluteal or the subgluteal approach in an adult patient. B. The location of the incision or injury does not change the choice of posterior approaches, only the anterior vs. posterior approach. C. The preference of one approach over the other is based on the patient’s anatomical characteristics and the operator’s personal preference. D. The use of out-of-plane technique is not advised for either approach due to the increased risk of intravascular injection. 5. Which statement is true regarding identifying the sciatic nerve in a transgluteal approach on an adult patient? A. The sciatic nerve is located immediately deep to the gluteus maximus muscle. B. The sciatic nerve is located immediately deep to the quadratus femoris muscle. C. The sciatic nerve’s location can be approximated by palpating the greater trochanteric and sacral hiatus. D. The sciatic nerve’s location is identified on ultrasound using a linear array probe.

207

Hadzic_Ch32F_p207-212.indd 207

19/04/19 11:28 AM

208

PART 3

Clinical Practice of Regional Anesthesia

6. When doing an ultrasound-guided sciatic nerve block by the posterior approach, the location marked with an arrow in the following image corresponds to which structure?

C. Subgluteal D. Popliteal 11. Successful blockade of the sciatic nerve is most likely to leave which of the following anatomic regions with intact sensation? A. Dorsum of the foot B. Plantar surface of the foot C. Lateral ankle D. Medial ankle 12. Which of the following approaches to anesthetizing the sciatic nerve is most likely to leave the posterior femorocutaneous nerve of the thigh with intact sensation? A. Anterior B. Transgluteal/Labat C. Parasacral D. Subgluteal

A. Femur B. Greater trochanter C. Ischial tuberosity D. Sciatic nerve 7. A posterior sciatic nerve block: A. Requires use of a curved transducer regardless of the approach B. Requires visualization of the osseous landmarks on ultrasound when doing a subgluteal injection C. Allows visualization of osseous structures if using the transgluteal approach with a curved probe D. Will provide the same area of anesthesia distribution regardless of the approach 8. A sciatic nerve block through the anterior approach: A. Can provide analgesia for hip surgery B. Is convenient for patients who cannot easily be positioned prone or lateral C. Is easier to perform than the subgluteal approach D. Requires the needle to contact the greater trochanter 9. Which statement is true regarding sciatic nerve block? A. Depending on the approach, the innervation to the skin of the posterior thigh is inconstantly blocked. B. Sciatic nerve block is commonly used to provide analgesia following total knee arthroplasty. C. Sciatic nerve block is typically performed using a lower volume of local anesthetic than for a femoral nerve block. D. Sciatic nerve block will provide surgical anesthesia of the lower leg below the knee. 10. Which of the following approaches to indwelling sciatic perineural catheter placement is least likely to be recommended secondary to infectious and technical challenges of performing the procedure? A. Anterior B. Transgluteal/Labat

Hadzic_Ch32F_p207-212.indd 208

13. Between which of the following two muscles is the sciatic nerve most likely to be found during the performance of a transgluteal sciatic nerve block? A. Gluteus maximus and quadratus femoris B. Gluteus minimus and adductor longus C. Gluteus maximus and adductor longus D. Gluteus minimus and quadratus femoris 14. Which of the following approaches to the sciatic nerve is most likely to spare motor blockade of the hamstring muscles (biceps femoris, semitendinosis, and semimembranosis)? A. Transgluteal B. Parasacral C. Subgluteal D. Popliteal 15. During the performance of an anterior approach to the sciatic nerve, the target nerve is most likely to be encountered between which of the following muscle pairs? A. Adductor magnus and hamstring B. Rectus femoris and piriformis C. Sartorius and adductor longus D. Pectineus and adductor magnus

ANSWERS AND EXPLANATIONS 1. C is correct. Many layers of muscle and soft tissue need to be penetrated with an introducer and this leads to an increased risk of hematoma. It is in the medial thigh, which may be uncomfortable for patients and is likely to be dislodged by patient movement. A is incorrect. The anterior approach to the sciatic nerve block has the advantage of easy positioning for patients who are unable to lie in the lateral or prone position due to trauma or pain. However, leaving a continuous infusion catheter by this approach is not advised.

19/04/19 11:28 AM

CHAPTER 32F

B is incorrect. The catheter would have to make a 90 degree (or more) turn from the direction of insertion of the introducer to the path of the nerve. The anterior approach uses the nerve stimulator as well as a curvilinear probe for the ultrasound due to the increased level of depth needed for visualization. D is incorrect. The sciatic nerve block done at the level of the subgluteal posterior approach as well as the anterior approach both miss the posterior femoral cutaneous nerve, which provides sensation to the posterior thigh. 2. D is correct. The anterior vs. posterior approaches to a sciatic nerve block come down to clinical judgment by the anesthesia provider based on the comfort level of the provider and the status of the patient who will be having surgery. The three most common approaches provide comparable blocks with a few exceptions. A is incorrect. A continuous infusion catheter can be used with the subgluteal approach, but it is not well-suited for the transgluteal or anterior approach due to the number of muscles that the large introducer needle would have to penetrate and have the potential to harm. B is incorrect. All of the approaches can be performed under ultrasound guidance; however, a linear probe can be used only for the subgluteal approach due to the level of depth needed for the transgluteal and the anterior approach. C is incorrect. The anterior approach and subgluteal posterior approach will both provide the same coverage area; however, the transgluteal approach will also include sensation to the posterior thigh as it is performed at a level more cephalad than the branch point of the posterior femoral cutaneous nerve from the sciatic nerve. All of the approaches would provide sufficient coverage for procedures that are performed below the knee. 3. B is correct. The anterior approach to the sciatic nerve block is a structurally deep block. The needle must pass through at least 6–8 cm of muscle and soft tissue. Because of this, attempting to visualize the needle in-plane may be very difficult even with a curvilinear probe. Accessing the nerve via an out-of-plane technique with hydrodissection for guidance is a more practical approach. A is incorrect. The structure is so deep that a curvilinear probe must be used as opposed to a linear probe. C is incorrect. If nerve stimulation is used to help guide injection, a response should be seen in the gastrocnemius or the foot. A patellar twitch can be seen when the femoral nerve is stimulated. D is incorrect. It is placed at the medial thigh with the thigh in abduction and slight external rotation. 4. C is correct. The subgluteal and transgluteal approaches to the sciatic block yield comparable results and the choice between the two is generally dependent only on the level of comfort of the practitioner.

Hadzic_Ch32F_p207-212.indd 209

209

Ultrasound-Guided Sciatic Nerve Block

A is incorrect. The same positioning and landmarks are used for both blocks; however, a linear array probe is unlikely to provide enough depth of field to be used on an adult patient in a transgluteal sciatic block. B is incorrect. Additionally, like the anterior approach, the subgluteal approach does not cover the posterior femoral cutaneous nerve, which innervates the posterior thigh. The transgluteal approach does cover this cutaneous area and therefore the location of the injury or incision factor in to the choice of block. D is incorrect. The block can be done using an out-ofplane technique, again based on the comfort of the practitioner and the patient’s body habitus and there is little risk of intravascular injection due to the fact that the sciatic nerve is not part of a neurovascular bundle. 5. A is correct. The gluteus maximus is superficial to the nerve and the quadratus femoris muscle is deep to the nerve. B is incorrect. The nerve can be located between the gluteus maximus muscle and the quadratus femoris muscle. C is incorrect. The sciatic nerve block can be performed using an anterior or posterior approach. When done posteriorly, it can either be a transgluteal or subgluteal. The choice of approach is dependent on the anesthesia provider as they all accomplish the same goal. When the transgluteal approach is chosen, the patient is positioned laterally with the legs flexed, and the greater trochanter and ischial tuberosity are palpated with the nerve running between the two. The sacral hiatus is also a surface landmark, but while it can be useful for other sciatic nerve block techniques such as Labat’s approach, it is not useful for an ultrasound-guided sciatic nerve block. D is incorrect. After approximating the location, a curvilinear probe is placed on the skin. The linear probe will not provide enough depth of field on an average adult patient. 6. C is correct. This is an ultrasound image of the transgluteal approach to the sciatic nerve using a curved ultrasound transducer probe. It can be identified as transgluteal due to the presence of the osseous landmarks. The sciatic nerve is pictured in the center and is flanked by the osseous structures. On the medial side is the ischial tuberosity. See Figure 32F–1. A and B are incorrect. On the lateral side is the greater trochanter, which is part of the femur. D is incorrect. At this level, the sciatic nerve is covered superficially by the gluteus maximus muscle, which is what is traversed with the needle when performing this block. 7. C is correct. When using the curved probe at this location, the wide field of view will allow the osseous structures to be visualized flanking the nerve. A is incorrect. When performing a transgluteal sciatic nerve block, the greater trochanter and ischial tuberosity

19/04/19 11:28 AM

210

PART 3

Clinical Practice of Regional Anesthesia

Lateral

D is incorrect. The greater trochanter is the lateral aspect of the femur and the anterior approach to the sciatic nerve is on the medial thigh. The needle should not be near the greater trochanter.

Sciatic nerveblock-posterior approach

FIGURE 32F–1.  Ultrasound image demonstrating the sonoanatomy of the sciatic nerve (ScN). The ScN often assumes an ovoid or triangular shape and is positioned deep to the gluteus maximus muscle (GMM) between the ischial tuberosity (IT) and femur. (Reproduced with permission from Hadzic A: Hadzic’s Peripheral Nerve Blocks and Anatomy for Ultrasound-Guided Regional Anesthesia, 2ed. New York, NY: McGraw-Hill Education; 2011.)

are the osseous landmarks to start locating the nerve. Depending on the patient’s body habitus, a linear or curved probe can be used for the transgluteal or subgluteal locations. B is incorrect. Because you can use the linear probe at this level, which will not show you the osseous landmarks, it is not necessary to visualize them at any level. At the subgluteal level, despite using a curved probe, the location is too distal to see the osseous structures anyway. D is incorrect. The sciatic nerve blocks at any level or approach are going to have similar areas of distribution of anesthesia; however, at the transgluteal level, there is an increased chance of anesthetizing the posterior femorocutaneous nerve, which provides sensory innervation to the back of the thigh. This will not be blocked with the subgluteal approach or anterior approach. 8. B is correct. The anterior approach to the sciatic nerve block is performed with the patient supine, ideally with the slightly abducted and externally rotated. This is useful for patients who are unable to be put lateral or prone due to injury, pain, or fixation devices. A is incorrect. The anterior approach is performed around the level or slightly distal to the subgluteal approach. Both of these are distal to the hip joint and therefore would not be beneficial during a hip surgery. C is incorrect. The anterior approach involves going through several centimeters of adipose and muscle of the anteromedial thigh. Because of the significant depth (7 cm average), it is more technically challenging than the posterior approaches, both visually and mechanically.

Hadzic_Ch32F_p207-212.indd 210

9. A is correct. The sciatic nerve provides sensory innervation of the back of the thigh as well as the majority of the lower leg. However, the posterior femorocutaneous nerve, which is the supply to the posterior thigh, branches off of the sciatic nerve at a fairly proximal location. Because of this, depending on the location of the injection of local anesthetic, the posterior thigh may not be blocked. B is incorrect. In addition to sensory innervation, the sciatic nerve also provides motor innervation to a number of the leg muscles. Because of this, it is not often used for postoperative analgesia in total knee arthroplasty patients as early ambulation has been shown to decrease morbidity and mortality. These patients need to have full strength in their leg. C is incorrect. The sciatic nerve at any location in the thigh is a large nerve. It is recommended that volume of local anesthetic (10–15 mL) similar to the femoral nerve also be used in this location. D is incorrect. The transgluteal approach is more likely to cover the posterior femorocutaneous nerve than the other approaches because it is the most proximal injection point. In the lower leg, the medial aspect from the knee to the ankle is in the saphenous nerve distribution and therefore a sciatic injection alone will not provide surgical anesthesia for the entire lower leg. 10. A is correct. In order to access the sciatic nerve via the anterior approach, the needle must traverse many centimeters of muscle, which predisposes the patient to hematoma formation when a large bore introducer needle is used. Additionally, the angle needed to reach the deep structures will require the catheter to make a 90 degree turn to reach the optimal location. Lastly, the catheter entry point at the skin is near the groin, which could increase the risk of infection. B, C, and D are incorrect. The remaining approaches (choices) are favored over the anterior approach due to proximity to the groin and the angle of approach in the anterior position as well as the mass of tissue through which a larger needle must pass to reach its target. 11. D is correct. Successful blockade of the sciatic nerve results in analgesia/anesthesia of the leg with the exception of the medial aspect of the shin and ankle, with variable blockade of the arch of the foot. A, B, and C are incorrect. These regions would be expected to reliably be blocked with successful anesthetization of the sciatic nerve. 12. D is correct. Performing the subgluteal approach to the sciatic nerve is least likely to contact the femorocutaneous nerve of the thigh prior to its distribution of nerve fibers to the skin of the posterior thigh.

19/04/19 11:28 AM

CHAPTER 32F

A, B, and C are incorrect. The remaining approaches, when successfully performed, are more likely to leave the posterior femorocutaneous nerve of the thigh anesthetized. 13. A is correct. When using ultrasound to visualize the sciatic nerve via the transgluteal approach, the nerve will be found deep to the gluteus maximus muscle and superficial to the quadratus femoris muscle. B, C, and D are incorrect. These muscle pairs are unlikely to contain the sciatic nerve between them. 14. D is correct. Approaching the sciatic nerve through a popliteal approach is distal to the majority of the motor units supplying the innervation to the proximal thigh muscles. A, B, and C are incorrect. The proximal location of the approach of these choices suggests a higher likelihood of affecting motor units to the hamstring muscles.

Hadzic_Ch32F_p207-212.indd 211

Ultrasound-Guided Sciatic Nerve Block

211

15. A is correct. When using ultrasound imaging to visualize the sciatic nerve via the anterior approach, the nerve runs between the adductor magnus anteriorly and the hamstring muscles posteriorly. B, C, and D are incorrect. These choices are unlikely to be locations where, during the performance of an anterior approach to a successful sciatic nerve block, the provider will find the sciatic nerve to be contained.

Suggested Reading Hadzic A. Ultrasound-guided sciatic nerve block. In: Atchabahian A, Vandepitte C, Lopez AM, Lin J-A, eds. Textbook of Regional Anesthesia and Acute Pain Management. 2nd ed. New York, NY: McGraw-Hill Publishing; 2017:chap 33F.

19/04/19 11:28 AM

This page intentionally left blank

Hadzic_FM_p00i-pxx.indd 6

22/04/19 6:40 PM

32G Ultrasound-Guided Popliteal Sciatic Block Ana M. Lopez

QUESTIONS DIRECTIONS: Choose the one best response to each question. 1.  Which statement is true regarding the ultrasound anatomy of the sciatic nerve in the popliteal fossa? A. The separation of the tibial nerve and common peroneal nerve is consistently identified at the level of the popliteal crease. B. The sciatic nerve is visualized at the popliteal fossa deep to the popliteal artery and vein. C. The scanning technique should focus on identifying the level of division of the nerve and the connective tissue sheath containing both components. D. The sciatic nerve is typically seen as a hypoechoic, oval structure with a honeycomb pattern. 2.  If nerve stimulator is used during a popliteal sciatic nerve block (0.5 mA, 0.1 msec), contact of the needle tip with either branch of the nerve is associated with: A. Flexion of the knee with stimulation of the sciatic nerve proximal to the division B. Plantar flexion of the foot with stimulation of the tibial nerve C. Dorsal extension with stimulation of the sural nerve D. Paresthesia on the medial aspect of the leg with stimulation of the peroneal nerve 3.  The popliteal block as a sole technique is usually indicated for the following procedure (considering that the tourniquet is placed below the knee): A. Bunionectomy B. Saphenous vein graft harvesting C. Osteosynthesis D. Ankle arthroscopy 4.  Which statement is true regarding the positioning of the patient and needle approach to perform a popliteal block? A. Prone position is convenient to scan, but does not allow in-plane needle approach.

B. In-plane approach from lateral to medial with the patient in lateral position usually results in incomplete block of the tibial nerve. C. Placement of popliteal catheters using the lateral in-plane approach is not recommended due to poor stability of the catheter. D. Lateral in-plane needle approach with the patient in supine position and the extremity elevated is recommended for patients with limited mobility. 5.  Which statement is true regarding popliteal sciatic block? A. For a consistent and fast-onset block, the injection of local anesthetic must occur around the sciatic nerve sheath that contains both components of the nerve. B. Asking the patient to dorsiflex and plantar flex the ankle makes the two sciatic nerve branches twist or move in relation to each other and helps the identification of the nerve. C. Injection at the level of division of the sciatic nerve is safe as the popliteal vessels are distant, deeper, and medial from the nerve. D. The correct injection is recognized when local anesthetic spreads proximally next to the sciatic nerve, but not distally.

ANSWERS AND EXPLANATIONS 1. C is correct. To perform a popliteal block, the ultrasound imaging should focus on identifying the division of the sciatic nerve and the common sheath (Vloka’s sheath) containing both components (tibial and common peroneal nerves). Successful injection will deposit local anesthetic within the common sheath in between the nerves. A is incorrect. The anatomy of the sciatic nerve in the popliteal fossa is variable, and the division into the tibial nerve (TN) and common peroneal nerve (CPN) occurs at an inconstant distance from the popliteal crease, between 2 and 10 cm. 213

Hadzic_Ch32G_p213-214.indd 213

19/04/19 11:27 AM

214

PART 3

Clinical Practice of Regional Anesthesia

B is incorrect. At the popliteal fossa, the sciatic nerve typically is visualized superficial and lateral to the popliteal vessels. D is incorrect. The tibial nerve is seen as a hyperechoic, oval, or round structure with a honeycomb pattern. 2. B is correct. If nerve stimulation is used (0.5 mA, 0.1 msec), contact of the needle tip with the tibial nerve is associated with a motor response of the calf muscles and plantar flexion of the foot and toes. A is incorrect. The branches of the sciatic nerve supplying the hamstring muscles, responsible for the flexion of the knee, take off proximally in the posterior thigh. Therefore, they are spared by blockade at the popliteal fossa. C is incorrect. Stimulation of the sural nerve will result in paresthesia of the posterior aspect of the leg and lateral side of the foot. D is incorrect. The saphenous nerve innervates the skin of the medial aspect of the leg. The peroneal nerve supplies the lateral aspect of the leg. 3. A is correct. Distal branches of the sciatic nerve provide innervation to the entire foot, except for the skin of the medial side of the foot, which is provided by the saphenous nerve. In the vast majority of patients, the sensory innervation of the saphenous does not extend to the bunion area. B is incorrect. The medial aspect of the leg is innervated by the saphenous nerve, which runs next to the saphenous vein down to the ankle. C is incorrect. Osteosynthesis of the tibia requires an incision on the medial aspect of the leg, which is innervated by the saphenous nerve. D is incorrect. Ankle arthroscopy requires the placement of one trocar on the medial side; therefore, the saphenous nerve block is also required. 4. D is correct. Popliteal block can be performed with the patient in the supine, lateral, and prone position using different needle approaches. For patients with limited mobility, the block can be performed in supine position

Hadzic_Ch32G_p213-214.indd 214

with the extremity elevated or flexed to allow scanning of the popliteal fossa. A is incorrect. The prone position is the most convenient to scan and to perform the block as it admits multiple needle insertion techniques. B is incorrect. Popliteal block success depends on the correct needle tip position and the correct distribution of local anesthetic irrespective of patient position and needle direction. C is incorrect. The lateral approach may have some advantages with regard to catheter placement. First, the biceps femoris muscle tends to stabilize the catheter and decrease the chance of dislodgement. Second, if the knee is to be flexed and extended, the side of the thigh is less mobile than the back of the knee. Finally, access to the catheter site is more convenient with the lateral approach. 5. B is correct. If the identification of the sciatic nerve at the level of the popliteal fossa proves difficult, the flexoextension of the ankle results in a twist movement of the tibial and peroneal nerves in relation to each other and the surrounding structures, facilitating their identification. A is incorrect. For a consistent and fast onset block, the injection of local anesthetic must occur inside the sciatic nerve sheath that contains both components of the nerve. C is incorrect. Before injecting the local anesthetic at the level of division of the sciatic nerve, care must be taken to avoid puncture of the popliteal vein, which sometimes lies immediately deep to the nerve. The presence of an epineurial vein inside the nerve is also common. Therefore, releasing the pressure and aspirating before injection is mandatory. D is incorrect. The expected spread after injection into the common sheath of the sciatic nerve at the level of division is around the two components and both proximally and distally along the nerve and its branches.

Suggested Reading Hadzic A. Ultrasound-guided popliteal sciatic block. In: Hadzic A, Lopez AM, Vandepitte C, Sala-Blanch X, eds. Textbook of Regional Anesthesia and Acute Pain Management. 2nd ed. New York, NY: McGraw-Hill Publishing; 2017:chap 33G.

19/04/19 11:27 AM

32H Ultrasound-Guided Ankle Block Sam Van Boxstael

QUESTIONS DIRECTIONS: Choose the one best response to each question. 1.  Which of the following nerves involved in ankle block is a terminal branch of the femoral nerve? A. Deep peroneal nerve B. Saphenous nerve C. Sural nerve D. Tibial nerve 2.  Which of the following is the correct sequence of the relevant structures in the vicinity of the tibial nerve from anterior to posterior? A. Artery/nerve/vein, flexor hallucis longus tendon, flexor digitorum longus tendon, and tibialis posterior tendon B. Flexor digitorum longus tendon, artery/nerve/vein, tibialis posterior tendon, and flexor hallucis longus tendon C. Flexor digitorum longus tendon, tibialis posterior tendon, artery/nerve/vein, and flexor hallucis longus tendon D. Tibialis posterior tendon, flexor digitorum longus tendon, artery/nerve/vein, and flexor hallucis longus tendon 3.  Which of the following areas of the foot is innervated by the tibial nerve? A. Dorsum of the foot B. Heel and sole of the foot C. Lateral margin of the foot and ankle D. Web space between the first and second toes 4.  The superficial peroneal nerve can be identified by placing the transducer: A. Transversely on the leg, approximately 5–10 cm proximal and anterior to the lateral malleolus B. Transversely on the leg, approximately 5–10 cm proximal and anterior to the medial malleolus C. Transversely on the leg, approximately 5–10 cm proximal and posterior to the lateral malleolus D. Transversely on the leg, approximately 5–10 cm proximal and posterior to the medial malleolus

5.  Which structure can be identified in the vicinity of the small saphenous vein as a small hyperechoic structure proximal to the lateral malleolus? A. Deep peroneal nerve B. Saphenous nerve C. Sural nerve D. Superficial peroneal nerve 6.  Which anatomic landmark aids in the location of the saphenous nerve 10–15 cm proximal to the medial malleolus? A. Anterior tibial artery B. Great saphenous vein C. Posterior tibial artery D. Small saphenous vein 7.  Which of the following nerves can be omitted in surgery on the forefoot and toes? A. Deep peroneal nerve B. Saphenous nerve C. Sural nerve D. Tibial nerve

ANSWERS AND EXPLANATIONS 1. B is correct. The saphenous nerve is a sensory branch of the femoral nerve. A, C, and D are incorrect. These nerves are terminal branches of the sciatic nerve. 2. D is correct. A useful mnemonic for the relevant structures in the vicinity is Tom, Dick ANd Harry, which refers to, from anterior to posterior, the tibialis posterior tendon, flexor digitorum longus tendon, artery/nerve/vein, and flexor hallucis longus tendon. A, B, and C are incorrect sequences of the relevant structures in the vicinity of the tibial nerve from anterior to posterior. 3. B is correct. The tibial nerve is the largest of the five nerves at the ankle level and provides innervation to the heel and sole of the foot. A is incorrect. The dorsum of the foot is innervated by the superficial peroneal nerve. 215

Hadzic_Ch32H_p215-216.indd 215

19/04/19 11:27 AM

216

PART 3

Clinical Practice of Regional Anesthesia

C is incorrect. The sural nerve innervates the lateral margin of the foot and ankle. D is incorrect. The web space between first and second toes is innervated by the deep peroneal nerve. 4. A is correct. A transducer placed transversely on the leg, approximately 5–10 cm proximal and anterior to the lateral malleolus, will identify the hyperechoic nerve branches lying in the subcutaneous tissue immediately superficial to the fascia. B, C, and D are incorrect transducer positions to identify the superficial peroneal nerve. 5. C is correct. The sural nerve innervates the lateral margin of the foot and ankle. It is proximal to the lateral malleolus. A, B, and D are incorrect. These nerves have a different location around the ankle. 6. B is correct. The saphenous nerve travels down the medial aspect of the leg below the knee alongside the great saphenous vein. Because it’s a small nerve, it is best

Hadzic_Ch32H_p215-216.indd 216

visualised 10–15 cm proximal to the medial malleolus, using the saphenous vein as a landmark. A is incorrect. The anterior tibial artery crosses the deep peroneal nerve from medial to lateral at the level of the ankle joint. C is incorrect. The posterior tibial artery is situated directly posterior of the tibial nerve. D is incorrect. The sural nerve lies adjacent to the small saphenous vein proximal to the lateral malleolus. 7. B is correct. In 97% of patients, the saphenous nerve innervation does not extend beyond the midfoot. A, C, and D are incorrect. These nerves can’t be omitted in surgery on the forefoot and toes.

Suggested Reading Hadzic A. Ultrasound-guided ankle block. In: Vandepitte C, Lopez AM, Boxstael SV, Jalil H, eds. Textbook of Regional Anesthesia and Acute Pain Management. 2nd ed. New York, NY: McGraw-Hill Publishing; 2017:chap 33H.

19/04/19 11:27 AM

Section 5

Ultrasound-Guided Nerve Blocks for Abdominal and Thoracic Wall Chapter 33

 ltrasound-Guided Transversus Abdominis Plane and U Quadratus Lumborum Blocks  219

Chapter 34

Pectoralis and Serratus Plane Blocks  225

Hadzic_Ch33_p217-224.indd 217

22/04/19 6:31 PM

This page intentionally left blank

Hadzic_FM_p00i-pxx.indd 6

22/04/19 6:40 PM

33 Ultrasound-Guided Transversus Abdominis Plane and Quadratus Lumborum Blocks Hesham Elsharkawy

QUESTIONS DIRECTIONS: Choose the one best response to each question. 1. Which of the following is the most important advantage of quadratus lumborum block (QLB) compared to lateral transversus abdominis plane (TAP) block? A. Suitable for operations below the umbilicus B. Smaller volumes of local anesthetics required C. Wider dermatomal coverage (T7 to L1) D. Safer in patients on anticoagulant medications E. Lower incidence of complications

C. Transversus abdominis muscle D. Spinal nerve E. Middle thoracolumbar fascia 3. In this image (Figure 33–2) obtained during quadratus lumborum block (QLB), identify the following: A. QL lateral approach B. QL posterior approach C. QL anterior (transmuscular) approach

2. In this image (Figure 33–1) from the lateral abdominal and lumbar paravertebral area, identify the following: A. Quadratus lumborum muscle B. Psoas major muscle

FIGURE 33–2  Image obtained during quadratus lumborum block. (Reprinted with permission, Cleveland Clinic Center for Medical Art & Photography © 2014-2018. All Rights Reserved.)

FIGURE 33–1  Image from lateral abdominal and lumbar paravertebral area. (Reprinted with permission, Cleveland Clinic Center for Medical Art & Photography © 2014-2018. All Rights Reserved.)

Hadzic_Ch33_p217-224.indd 219

4. Which statement about the quadratus lumborum (QL) muscle is most likely true? A. The QL muscle originates from the iliac crest and inserts into the last rib. B. The QL muscle is a muscle of the anterior abdominal wall lying dorsal to the iliopsoas. C. The QL assists in producing forward flexion of the lumbar spine. D. The ventral rami pass between the QL and latissimus dorsi muscle.

219

22/04/19 6:31 PM

220

PART 3

Clinical Practice of Regional Anesthesia

5. It is possible to block the visceral pain conduction with all of the following blocks except: A. Thoracic paravertebral blockade B. Anterior quadratus lumborum block C. Epidural analgesia D. Rectus sheath block 6. True or False: The quadratus lumborum block is safe in patients on anticoagulant medications. A. True B. False 7. In this ultrasound image (Figure 33–3) from the lateral abdominal and lumbar paravertebral area, identify the following:

A. Psoas major muscle B. Quadratus lumborum muscle C. Internal oblique muscle D. Erector spinae muscle E. Latissimus dorsi muscle F. Lumbar plexus Directions for Questions 8 through 10: These items refer to the diagnosis, treatment, or management of a single patient.  he patient is a 32-year-old man scheduled for open subtotal T colectomy. The surgery is scheduled to be performed through a midline incision extending from above the umbilicus to the pubic symphysis. The patient refused an epidural catheter.

Hadz Ha dzic ic - Lan ance cea/ a/ NYS YSOR ORA A A

B

FIGURE 33–3  A: Cross section with the ultrasound probe location. B: Ultrasound image of the lateral abdominal wall.

8. Which of the following is the best alternative block to be offered to this patient? A. Unilateral rectus sheath block B. Bilateral quadratus lumborum blocks C. Bilateral subcostal transversus abdominis plane block D. Bilateral fascia transversalis block 9. The patient developed grand mal seizure 10 minutes after receiving bilateral block. What is the most likely cause? A. Systemic local anesthetic toxicity B. Vertebral artery injection C. High epidural injection D. Total spinal anesthesia 10. One day after the surgery, the patient complains of bilateral weakness in the quadriceps muscles. What is the most likely cause?

Hadzic_Ch33_p217-224.indd 220

A. Post-convulsive manifestations B. Possible side effect of the block C. Consequence of kidney puncture D. Epidural spread of the medications E. None of the above

ANSWERS AND EXPLANATIONS 1. C is correct. The QLB produces a more extensive spread of local anesthetic. QLB results in a wider sensory blockade compared to TAP block (T7–L1 for QLB vs. T10–T12 for the lateral TAP block). A, B, D, and E are incorrect. Ultrasound-guided TAP blocks are not able to produce a sensory level above the umbilicus consistently unless you add a subcostal injection.

22/04/19 6:31 PM

CHAPTER 33

Ultrasound-Guided Transversus Abdominis Plane and Quadratus Lumborum Blocks

Ultrasound-guided QLB has been introduced and shown to result in consistent coverage of at least T8 rostral, and L1 caudally. Moreover, QLB has the potential to provide some visceral analgesia considering its potential spread to paravertebral space especially with the anterior approach. 2. See Answer Figure 33–1 for locations of QL muscle, psoas major muscle, transversus abdominis muscle, spinal nerve, and middle thoracolumbar fascia.

221

The quadratus lumborum (QL) muscle is a muscle of the posterior abdominal wall lying deep inside the abdomen and dorsal to the iliopsoas.  The thoracolumbar fascia (TLF) consists of both aponeurotic as well as fascial connective tissue. Its most important function is providing a retinaculum for paraspinal musculature in the lumbar region. It consists of three layers: anterior, middle, and posterior layers.

Middle thoracolumbar fascia (E) Transversus abdominis (C) Quadratus lumborum (A) Spinal nerve (D) Psoas major (B)

ANSWER FIGURE 33–1  Image showing location of items A–E from Question 2. (Reprinted with permission, Cleveland Clinic Center for Medical Art & Photography © 2014-2018. All Rights Reserved.)

3. See Answer Figure 33–2 for locations of QL lateral approach, QL posterior approach, and QL anterior (transmuscular) approach.

This block can be performed via three different approaches. Answer Figure 33–2 shows all three common approaches with the needles. Posterior (B) Lateral (A)

Anterior (C)

ANSWER FIGURE 33–2  Image showing location of items A–C from Question 3. (Reprinted with permission, Cleveland Clinic Center for Medical Art & Photography © 2014-2018. All Rights Reserved.)

Hadzic_Ch33_p217-224.indd 221

22/04/19 6:31 PM

222

PART 3

Clinical Practice of Regional Anesthesia

3A. QLB lateral: After observing the tapering of the abdominal muscle layers under ultrasound, the needle can be directed from anterior to posterior toward the junction of tapered abdominal muscle layers and QL muscle; local anesthetic will then be deposited lateral to QL muscle and superficial to the fascia transversalis. 3B. QLB posterior: The needle can be advanced more posteriorly, and local anesthetic can be deposited posterior to the QL muscle, between the QL muscle and the (erector spinae, latissimus dorsi muscle, and serratus posterior inferior muscle). 3C. QLB anterior (transmuscular): The needle can be advanced from posterior to anterior through erector spinae muscle or through QL described by BØrglum et al to deposit the local anesthetic at the space between the fascial layers of the QL and psoas major muscles.      4. A is correct. Quadratus lumborum muscle originates from the medial half of the iliac crest and inserts into the lower medial border of the last rib (12th), and by four small tendons from the apices of the transverse processes of the upper four lumbar vertebrae. B is incorrect. The QL muscle is a muscle of the posterior abdominal wall lying deep inside the abdomen and dorsal to the iliopsoas. C is incorrect. The QL muscle assists in producing lateral flexion of the lumbar spine.

D is incorrect. The ventral rami pass between the QL and its anterior fascia. 5. D is correct. It is not possible to block visceral pain conduction with a rectus sheath block. A, B, and C are incorrect. Epidural and paravertebral blocks are known to provide visceral analgesia. QL block offers high dermatomal coverage (up to T7), blocks subcostal (T12) and iliohypogastric (L1) nerves, and covers the visceral component due to thoracic paravertebral spread with the anterior approach, and offers analgesia laterally over the iliac crest as far as the greater trochanter. 6. B (False) is correct. The risks of bleeding complications are not known and there are no specific recommendations. Due to the vascularity of the area, retroperitoneal spread of hematoma, proximity of the transmuscular approach to the paravertebral area and the lumbar plexus, the American Society of Regional Anesthesia guidelines should be implemented in patients on anticoagulants who are receiving QLB either single-shot or catheter. Risk versus benefits should be carefully considered. 7. See Answer Figure 33–3 for location of psoas major muscle, QL muscle, internal oblique muscle, erector spinae muscle, latissimus dorsi muscle, and lumbar plexus. 8. B is correct. QLB indications share the same indications as the transversus abdominis plane (TAP) block in

Hadz Ha dzic ic - Lan ance cea/ a/ NYS YSOR ORA A A

B

ANSWER FIGURE 33–3  Image showing location of items A–F from Question 7.

Hadzic_Ch33_p217-224.indd 222

22/04/19 6:31 PM

CHAPTER 33

Ultrasound-Guided Transversus Abdominis Plane and Quadratus Lumborum Blocks

addition to some surgeries with the incision above the umbilicus, and as a component of multimodal postoperative analgesia for a wide variety of abdominal procedures. Complications of the QLB are related to a lack of anatomical understanding and needle expertise. It is possible to puncture intra-abdominal structures such as kidneys, liver, and spleen. Extra caution should be taken especially for a right-sided block as the right kidney is slightly lower than the left and smaller to be seen with ultrasound. 9. A is correct. In bilateral blocks with high-volume systemic local anesthetic, toxicity should be considered.

223

Suggested Readings Blanco R, Ansari T, Riad W, Shetty N. Quadratus lumborum block versus transversus abdominis plane block for postoperative pain after Cesarean delivery, a randomized controlled trial. Reg Anesth Pain Med. 2016;41:757-762. Elsharkawy H. Ultrasound-Guided Quadratus Lumborum Block: How Do I Do It? American Society of Regional Anesthesia and Pain Medicine Newsletter, November 2015;36-42. Hadzic A. Ultrasound-guided transversus abdominis plane and quadratus lumborum blocks. In: Hesham Elsharkawy H, Bendtsen TF, eds. Textbook of Regional Anesthesia and Acute Pain Management. 2nd ed. New York, NY: McGraw-Hill Publishing; 2017:chap 34.

10. B is correct. Transient femoral nerve palsy has been noticed in some patients and attributed to the spread of medication to the lumber plexus and tracking of medication under fascia iliaca.

Hadzic_Ch33_p217-224.indd 223

22/04/19 6:31 PM

This page intentionally left blank

Hadzic_FM_p00i-pxx.indd 6

22/04/19 6:40 PM

34 Pectoralis and Serratus Plane Blocks Gary Kao, Chad Lee, and Jung H. Kim

QUESTIONS DIRECTIONS: Choose the one best response to each question. 1. Which of the following is not an advantage of performing a pectoralis nerve (Pecs) block over thoracic paravertebral block (TPVB) for breast surgery such as a modified radical mastectomy? A. Local anesthetic requirement is lower in Pecs block B. Longer duration of action for the Pecs block C. Lower consumption of opioids in the postoperative 24-hour period D. Reduced postoperative pain scores in patients receiving Pecs block 2. Compared to a thoracic paravertebral block (TPVB), which of the following complications is unique to a Pecs block? A. Bradycardia B. Hypotension C. Injury to the thoracodorsal or long thoracic nerve D. Pleural puncture 3. Which is true concerning Pecs I versus Pecs II blocks? A. No difference in technique is noted by performing the procedure preoperatively or postoperatively. B. Pecs I block is preferred for modified radical mastectomy. C. Pecs II block does not provide adequate coverage for insertion of a pacemaker. D. Pecs II block should be performed prior to Pecs I in order to preserve anatomy visualized under ultrasound.

5. While performing blocks for first-case starts, you realize in your rush in blocking a patient scheduled for radical mastectomy that you only injected in the fascial layer above the muscle that is directly overlaying the 4th rib. Which of the following is a nerve branch that was missed that normally is covered when the block is performed completely? A. Long thoracic nerve B. Intercostobrachial nerve C. Medial pectoral nerve D. Thoracodorsal nerve 6. While you are on call, a chronic pain patient with permanent atrial fibrillation on rivaroxaban who is recently postoperative from mastectomy is brought to the perioperative area in preparation for urgent washout of infected dehisced incisional wound. Assuming no other bleeding diathesis, in considering regional supplementation as part of the patient’s multimodal analgesic plan, which of the following regional blocks would be optimal in this situation? A. Pecs I block B. Thoracic conventional epidural infusion C. Thoracic paravertebral block D. Serratus anterior block 7. Which of the transducer positions illustrated below, is the correct one for needle insertion to perform a PEC II block? A. B only B. B and C C. A and C D. A only

4. How many of the four potential compartments created by injection at the pectoral level communicate with the axilla? A. One B. Two C. Three D. Four

225

Hadzic_Ch34_p225-230.indd 225

22/04/19 12:11 PM

226

PART 3

Clinical Practice of Regional Anesthesia

C5 C4 T2 T3 T4 T5

Hadz Ha adz dziicc - Lancea/ NYSORA A

B

8. The best evidence in studies supports what volume of local anesthetic to be injected for a Pecs I block for optimal spread and analgesia? A. 0.2 mL/kg B. 10 mL up to 20 mL as patient tolerates C. 0.4 mL/kg D. No consensus for evidence-based recommendation 9. Which of the following statements is true regarding serratus plane block? A. Injection deep, rather than superficial, to serratus muscle results in a wider local anesthesia spread and more dermatomal anesthesia coverage. B. It provides axillary region anesthesia by anesthetizing long thoracic nerve. C. It is important to visualize latissimus dorsi muscle in addition to serratus anterior muscle. D. It is not indicated in patients with rib fracture because it anesthetizes only the muscles and skin overlying the ribs. 10. Which of the following statements is the correct description of the nerves? A. The long thoracic nerve is a branch from intercostal nerves T3–T6. It travels along the posterior thoracic cavity and terminates into serratus anterior muscle, which it innervates. B. The medial pectoral nerve arises from upper trunk of brachial plexus C5–C6 and supplies the skin over the medial aspect of anterior chest wall. C. The lateral pectoral nerve emerges from cervical nerves C7–C8, and mostly supplies pectoralis major. D. The intercostobrachialis nerve is the lateral cutaneous branch of the second and third intercostal nerves. It courses over the axilla to medial upper arm and supplies skin innervation to the areas.

Hadzic_Ch34_p225-230.indd 226

C

ANSWERS AND EXPLANATIONS 1. A is not an advantage, so option A is correct. In a randomized control trial by Kulhari et al, patients undergoing modified radical mastectomy were allocated to two groups. One group received a Pecs block while the other underwent a TPVB. Both groups received the same amount of local anesthetic, 25 mL of 0.5% ropivacaine. B and C are advantages, so options B and C are incorrect. The study demonstrated a longer duration of analgesia postoperatively as well as a lower consumption of morphine in the immediate postoperative 24-hour period. D is an advantage, so option D is incorrect. Postoperative pain scores were lower in the Pecs group in the immediate 2 hours after surgery. 2. C is correct. Injury to the thoracodorsal and long thoracic nerve is unique to the Pecs block. A, B, and D are incorrect. The Pecs block avoids the possibility of midline spread that can occur with TPVB and prevents complications associated with sympathetic blockade that can be seen with TPVB. Both procedures can cause pneumothorax and pleural puncture. 3. D is correct. When a Pecs I block is made before a Pecs II, it makes harder to visualize the anatomy pertinent to a Pecs II block because you are required to inject medication at a deeper fascial plane for the Pecs II block. A Pecs II block should be performed to augment the Pecs I block in those cases. A is incorrect. Although a Pecs I and II blocks can be performed either preoperatively or postoperatively, there is a chance that air will be trapped in the subcutaneous tissue postoperatively obscuring views under ultrasound. B is incorrect. A Pecs I block does not provide adequate coverage of the axilla necessary for a modified radical mastectomy.

22/04/19 12:11 PM

CHAPTER 34

C is incorrect. A Pecs II block provides adequate coverage for surgeries limited to the Pec major distribution such as pacemaker insertion. 4. B is correct. There are four potential compartments that can be injected at the pectoral level: 1. Between the superficial and deep pectoral fascial layers 2. Between the pectoral fascia and the clavipectoral fascia 3. Between the clavipectoral fascia and the superficial border of the serratus anterior muscle 4. Between the serratus anterior muscle and the exothoracic fascia Two compartments, the third and fourth compartments, communicate with the axillary region. A, C, and D are incorrect. See explanation for answer B. 5. C is correct. The Pecs II block requires two injections two fascial compartments. The pectoral compartment is accessed at the midaxillary line and covers both the medial and lateral pectoral nerves that travel between the pectoralis minor (deep) and pectoralis major (superficial) and is usually performed with the transducer overlaying the 2nd rib. A, B, and D are incorrect. All run in the target plane of the serratus anterior block (between the latissimus dorsi and serratus anterior). 6. D is correct. Given this was not a radical mastectomy with associated lymph node dissection, the midaxillary anatomy can be surmised to be relatively preserved even in the setting of infection in a nearby area. Serratus anterior plane block would theoretically provide coverage for the intercostobrachial nerve (T2/3) as well as lateral cutaneous branches of the intercostal nerves (T3–T9) of the chest wall that would be affected by postoperative pain. This is still considered a peripheral nerve block (see Table 34–1 from the most recent guidelines, albeit for Interventional Spine and Pain Procedures rather than Regional Anesthesia as such a version does not exist in the most recent 2010 guidelines for anticoagulation published by the American Society of Regional Anesthesia), which is considered a low-risk procedure as opposed to paravertebral block (C) or interlaminar epidural access (B) as it pertains to risk of bleeding. As this patient is still on oral anticoagulants, the high risk of bleeding in the sensitive neuraxial space precludes the option of central regional blocks. Serratus anterior block, as a peripherally located block, can be argued to additionally have mitigated risk of known bleeding risk since the site is easily accessed and compressed should bleeding at injection site become an issue. In reality, the risks or performing any elective regional blockade would have to be balanced with the anticipated postoperative complications including prolonged hospitalization due to poorly controlled pain and hypoventilation and worsened postoperative atelectasis due to splinting from chest wall pain. A is incorrect. Pecs I is performed at the midclavicular line around the 2nd rib, which when compared to the injection site of the other blocks provided in the answer choices, would most likely be in the surgical field or

Hadzic_Ch34_p225-230.indd 227

Pectoralis and Serratus Plane Blocks

227

TABLE 34–1  Procedure-related risk. High-Risk Procedures

Intermediate-Risk Procedures

Low-Risk Procedures

SCS trial and implant Intrathecal catheter and pump implant Vertebral augmentation (vertebroplasty and kyphoplasty) Epiduroscopy and epidural decompression

Interlaminar ESIs (C, T, L, S) Transforaminal ESIs (C, T, L, S) Facet MBNB and RFA (C, T, L) Paravertebral block (C, T, L) Intradiscal procedures (C, T, L) Sympathetic blocks (stellate, thoracic, splanchnic, celiac, lumbar, hypogastric) Peripheral nerve stimulation trial and implant Pocket revision and IPG/ITP replacement

Peripheral nerve blocks Peripheral joints and musculoskeletal injections Trigger point injections including piriformis injection Sacroiliac joint injection and sacral lateral branch blocks

*Patients with high risk for bleeding undergoing low- or intermediate-risk procedures should be treated as intermediate or high risk, respectively. Patients with high risk for bleeding may include old age, history of bleeding tendency, concurrent uses of other anticoagulant/antiplatelets, liver cirrhosis or advanced liver disease, and advanced renal disease. C indicates cervical; L, lumbar; MBNB, medial branch nerve block; RFA, radiofrequency ablation; S, sacral; T, thoracic.

be complicated by the localized infection described in the question, which would be a contraindication to this block. Additionally, the coverage of the overall chest wall along the anterior rami of thoracic spinal nerves would not be well-covered in this situation where pain from localized inflammation from both surgery and infection would spread beyond regions covered by blocking the pectoralis muscles’ innervation from lateral/medial pectoral nerves. B and C are incorrect. See explanation for answer D. 7. A is correct. The PEC II is a modified Pecs I block and can be achieved with one needle insertion point. Start with the transducer positioned as for Pecs I (mid-clavicular level and angled inferolaterally), and then slide laterally to identify the 4th rib, serratus anterior, pec minor and pec major muscles (image B). At this position, local anesthetic can be deposited between the pec minor and pec major muscles (Pecs I), and then between the pec minor and serratus anterior muscle (Pecs II). B and C are incorrect. In the illustration, C is the position to perform a serratus plane block. D is incorrect. Image A corresponds to the transducer position to perform a PEC I block. 8. D is correct. The literature thus far is sparse regarding optimal volume of injectate for adequate spread of local anesthetic in targeted fascial plane for any of the thoracic

22/04/19 12:11 PM

228

PART 3

Clinical Practice of Regional Anesthesia

TABLE 34–2  Summary of published controlled clinical trials and case eports. Author, year

Study Type

Surgery/ Indication

Block type

N

Injectate

Outcome

Blanco et al., 2013

Volunteer study



Serratus plane

4

0.4 mL/kg levobupivacaine 0.125% and gadolinium

Mean duration of paresthesia in the intercostal nerve distribution T2–T9, was 752 minutes (injection superficial to serratus anterior)

Wahba and Kamal, 2014

Randomized controlled trial

Mastectomy

Pecs II versus PVB

60

0.25% levobupivacaine: 15–20 mL T4 PVB, 10 mL Pecs I block

Pecs blocks reduced postoperative morphine consumption (first 24 h) and pain scores (first 12 h) in comparison with PVB following mastectomy

Fujiwara et al., 2014

Case report

Insertion of cardiac resynchronization device

Intercostal at first and second interspace, Pecs I block

1

0.375% ropivacaine: 4 mL intercostal block, 10 mL Pecs I block

Surgery performed under intercostal/Pecs I blocks and dexmedetomidine

Kunhabdulla et al., 2014

Case report

Analgesia for rib fracture

Serratus plane

1

20 mL bolus 0.125% bupivacaine, then infusion of 0.0625% bupivacaine at 7–12 mL/h

Effective analgesia to enable physiotherapy and ambulation

Madabushi et al., 2015

Case report

Analgesia for thoracotomy

Serratus plane

1

6 mL bolus 1% lignocaine, then infusion of bupivacaine 0.1% at 7 mL/h

Improvement in pain and ventilation

Murata et al., 2015

Case report

Breast surgery

Pecs II

2

35 mL 0.2% ropivacaine (mastectomy); 45 mL 0.2% ropivacaine (lumpectomy)

Mastectomy performed under Pecs II block and supplemental infiltration

Ueshima, 2015

Case report

Segmental breast resection

TTP combined with Pecs II

1

0.15% levobupivacaine: 15 mL TTP, 10 mL Pecs I, 20 mL Pecs II

Surgery performed under TTP and Pecs II blocks

Bashandy and Abbas, 2015

Randomized controlled trial

Mastectomy

Pecs II

120

0.25% bupivacaine: 10 mL Pecs I, 20 mL Pecs II

Lower visual analog scale pain scores and opioid requirements in the Pecs group compared to control group

Kulhari, 2016

Randomized controlled trial

Radical mastectomy

Pecs II versus PVB

40

25 mL 0.5 % ropivacaine

Duration of analgesia increased in Pecs block compared to PVB group (4.9 versus 3.3 hours)

Hetta, 2016

Randomized controlled trial

Radical mastectomy

Serratus plane

64

30 mL 0.25 % bupivacaine, Serratus plane; 15 mL 0.25% bupivacaine, PVB

Increased opioid consumption in the serratus plane compared to the PVB group

PVB, paravertebral block; TTP, transversus thoracic muscle plane.

Hadzic_Ch34_p225-230.indd 228

22/04/19 12:11 PM

CHAPTER 34

wall blocks. As with all regional anesthetic blocks, the trend has been to minimize volume of injectate with application of ultrasound guidance for precisely targeted deposition of injectate under live surveillance when compared to a landmark and/or nerve stimulator technique. A, B, and C are incorrect. In a collection of case reports and studies (Table 34–2), one can appreciate the variation in not only the volume but also concentration and type of local anesthetic utilized by the various authors that include all of the above choices. 9. C is correct. The described technique for a serratus plane block indicates injection of the local anesthetic in the fascial plane between the latissimus dorsi and the serratus anterior muscle or deep to the serratus. During the scanning technique, it is necessary to identify these two muscles to locate the correct plane for injection. Additionally, it is important to identify the pleura and ribs to reduce the risk of complications. A is incorrect. The study by Blanco et al. showed that the area of sensory loss to pinprick was consistent whether the injection was superficial or deep underneath the muscle. Also, the MRI scan with gadolinium noted a tendency for the spread to be wider, into the more posterior compartment, after the superficial injection. B is incorrect. Nerves of the axillary region are the intercostobrachial, intercostal from T3–T9, long thoracic, and thoracodorsal. The intercostobrachial nerve is the lateral cutaneous branch of the second, and in some cases the third intercostal nerves. It supplies the sensory innervation of the axilla, and it’s important to consider when analgesia of the axilla is required. D is incorrect. While serratus plane block is performed superficial to intercostal nerves, multiple case reports show efficacy in analgesia for rib fractures. Skin anesthesia is due to direct local anesthesia effect on lateral cutaneous branch of intercostal nerve. Somatic pain arising from rib fracture requires anesthesia of main intercostal nerve and it is thought that local anesthesia penetrates into intercostal space. 10. D is correct. The intercostobrachialis nerve is the lateral cutaneous branch of the second and third intercostal nerves in 67% and 33% of cases, respectively. It crosses the serratus anterior muscle in the midaxillary line to innervate the axilla. The intercostobrachialis nerve is a vital nerve if regional anesthesia of the axilla is required. A is incorrect. The long thoracic nerve arises from cervical nerves C5–C7. It is in the axillary compartment close to the lateral thoracic branch of the thoracoacromial artery

Hadzic_Ch34_p225-230.indd 229

Pectoralis and Serratus Plane Blocks

229

and travels down the lateral aspect of the serratus anterior muscle. Damage to this nerve can occur during breast surgery and axillary lymph node dissection and causes winged scapula. B is incorrect. The medial pectoral nerve arises from the medial cord fibers from C8–T1, behind the axillary artery at the level below the clavicle, and passes through the deep surface of the pectoralis minor, which it perforates and then enters and innervates pectoralis major. Both pectoral nerves enter the deep surface of the pectoralis major, and neither has a cutaneous branch. C is incorrect. The lateral pectoral nerve arises from the lateral cord of the brachial plexus. It crosses the axillary artery anteriorly and pierces the clavipectoral fascia in close relationship with the thoracoacromial artery on the undersurface of the upper portion of the pectoralis major muscle, which it supplies. The lateral pectoral nerve is medial to the pectoralis minor before entering the pectoralis major muscle; it communicates across the axillary artery with the medial pectoral nerve and supplies the pectoralis minor.

Suggested Readings Blanco R, Parras T, McDonnell JG, Prats-Galino A. Serratus plane block: a novel ultrasound-guided thoracic wall nerve block. Anaesthesia. 2013;68:1107-1113. Durant E, Dixon B, Luftig J, Mantuani D, Herring A. Ultrasoundguided serratus plane block for ED rib fracture pain control. Am J Emerg Med. 2017;35:197.e3-197.e6. Gonzales J. PECS versus PVBS for perioperative analgesic management in breast surgery. ASRA News. August 2016;16(3):41-44. Hadzic A. Pectoralis and serratus plane blocks. In: Rafael Blanco R, Barrington MJ, eds. Textbook of Regional Anesthesia and Acute Pain Management. 2nd ed. New York, NY: McGraw-Hill Publishing; 2017:chap 35. Kulhari S, Bharti N, Bala I, Arora A, Singh G. Efficacy of pectoral nerve block versus thoracic paravertebral block for postoperative analgesia after radical mastectomy: a randomized controlled trial. Br J Anaesth. September 2016;117(3):382-386. Naja Z, Lönnqvist PA. Somatic paravertebral nerve blockade. Incidence of failed block and complications. Anaesthesia. 2001;56:1181-1201. Narouze S, Benzon HT, Provenzano DA, et al. Interventional Spine and Pain Procedures in Patients on Antiplatelet and Anticoagulant Medications: Guidelines From the American Society of Regional Anesthesia and Pain Medicine, the... Reg Anesth Pain Medicine. May/June 2015;40(3):182-212. Wahba SS, Kamal SM. Thoracic paravertebral block versus pectoral nerve block for analgesia after breast surgery. Egypt J Anaesth. 2013;30:129-135.

22/04/19 12:11 PM

This page intentionally left blank

Hadzic_FM_p00i-pxx.indd 6

22/04/19 6:40 PM

PART 3E

Local and Regional Anesthesia for Oral and Maxillofacial Surgery Chapter 35

Hadzic_Ch35_p231-236.indd 231

Oral and Maxillofacial Regional Anesthesia  233

18/04/19 6:14 PM

This page intentionally left blank

Hadzic_FM_p00i-pxx.indd 6

22/04/19 6:40 PM

35 Oral and Maxillofacial Regional Anesthesia Malikah Latmore and Ali Shariat

QUESTIONS DIRECTIONS: Choose the one best response to each question. 1. The maxillary division (V2) of the trigeminal nerve exits the cranium through the: A. Foramen ovale B. Incisive foramen C. Foramen rotundum D. Infraorbital foramen 2. Hematoma formation due to accidental puncture of the maxillary artery is a possible complication of which of the following blocks? A. Posterior superior alveolar nerve block B. Anterior superior alveolar (infraorbital) nerve block C. Greater palatine nerve block D. Nasopalatine nerve block 3. For the nasopalatine nerve block, the injection site is located: A. In the area of the incisive foramen, directly lateral to the incisive papillae B. At the height of the mucobuccal fold just distal to the maxillary second molar C. Three-fourths of the anteroposterior distance between the coronoid notch and the pterygomandibular raphae D. The height of the mucobuccal fold over the mental foramen 4. Advantages of the Vazirani-Akinosi closed-mouth mandibular block technique include: A. A low complication rate B. Minimal risk of trauma to the inferior alveolar nerve, artery, and vein C. Minimal discomfort on injection D. All of the above

5. Which block anesthetizes the maxillary central and lateral incisors and canine, surrounding hard and soft tissue on buccal aspect, and mesiobuccal root of the maxillary first molar? A. Greater palatine nerve block B. Anterior superior alveolar (infraorbital) nerve block C. Maxillary nerve block D. Nasopalatine nerve block 6. Which area does the incisive nerve block anesthetize? A. Buccal soft tissue of the molar region B. Mandibular teeth to midline, hard and soft tissue of buccal aspect, anterior two-thirds of the tongue, floor of the mouth C. Mandibular teeth on side of injection, buccal and lingual hard and soft tissue, lower lip D. Premolars, canine, incisors, lower lip, skin over chin, buccal soft tissue anterior to mental foramen 7. Just distal to the maxillary second molar at the level of the mesiolingual cusp is the insertion point for which nerve block? A. Vazirani-Akinosa closed-mouth mandibular block B. Gow-Gates technique C. Incisive nerve block D. Buccal nerve block 8. Which block anesthetizes the mesiobuccal root of maxillary first molar (in some cases), premolars, and surrounding hard and soft tissue on the buccal aspect? A. Nasopalatine nerve block B. Mental nerve block C. Greater palatine nerve block D. Middle superior alveolar nerve block 9. Which nerve originates from the mandibular division? A. Nasopalatine nerve B. Anterior superior alveolar nerve C. Buccal nerve D. Posterior ethmoid

233

Hadzic_Ch35_p231-236.indd 233

18/04/19 6:14 PM

234

PART 3

Clinical Practice of Regional Anesthesia

10. Which of the following injection techniques cannot be utilized for anesthetizing individual teeth? A. Supraperiosteal injection B. Periodontal ligament injection C. Intraseptal injection D. Intrapulpal injection

ANSWERS AND EXPLANATIONS 1. C is correct. The maxillary division (V2) of the trigeminal nerve leaves the cranium through the foramen rotundum, located in the greater wing of the sphenoid bone. A is incorrect. The sensory root of the mandibular branch (V3) of the trigeminal nerve passes through the foramen ovale almost immediately after coming off the trigeminal ganglion. B is incorrect. The nasopalatine nerve enters the oral cavity through the incisive foramen to supply the palatal mucosa of the premaxilla. D is incorrect. The infraorbital nerve divides into three terminal branches after emerging through the infraorbital foramen onto the face. 2. A is correct. The posterior superior alveolar nerve block is used to achieve anesthesia of the maxillary molar teeth up to the first molar, with the exception of the mesiobuccal root in some cases. A possible complication of the posterior superior alveolar nerve block is hematoma formation from injection of anesthetic into the pterygoid plexus of veins or from accidental puncture of the maxillary artery necessitating aspiration before injection. B is incorrect. The anterior superior alveolar nerve block is a technique for achieving anesthesia of the maxillary central and lateral incisors and canine as well as the surrounding soft tissue on the buccal aspect. Accidental maxillary artery puncture is not a complication. C is incorrect. The greater palatine nerve block targets the palatal aspect of the maxillary premolar and molar dentition. D is incorrect. For the nasopalatine nerve block, local anesthetic is deposited in the area of the incisive foramen to anesthetize the nasopalatine nerves bilaterally. 3. A is correct. For the nasopalatine nerve block, anesthetic solution is deposited in the area of the incisive foramen. B is incorrect. The mucobuccal fold is the site of injection for the high tuberosity approach to the maxillary nerve block. C is incorrect. The inferior alveolar nerve block is performed three-fourths of the anteroposterior distance between the coronoid notch and the pterygomandibular raphae and approximately 6–10 mm above the occlusal plane. D is incorrect. The target area of the mental nerve block is the height of the mucobuccal fold over the mental foramen.

Hadzic_Ch35_p231-236.indd 234

4. D is correct. The Vazirani-Akinosa closed-mouth mandibular block technique is useful in patients with limited mouth opening due to trismus or ankylosis of the temporomandibular joint. Other advantages include minimal discomfort on injection, a low complication rate, and minimal risk of trauma to the inferior alveolar nerve, artery, and vein, as well as the pterygoid muscle. 5. B is correct. Anterior superior alveolar (infraorbital) nerve block anesthetizes the maxillary central and lateral incisors and canine, surrounding hard and soft tissue on buccal aspect, and mesiobuccal root of the of maxillary first molar. A is incorrect. Greater palatine nerve block anesthetizes the palatal mucosa and hard palate from first premolar anteriorly to posterior aspect of the hard palate and to midline anteriorly. C is incorrect. Maxillary nerve block anesthetizes the hemimaxilla on the side of injection (teeth; hard and soft, buccal, and lingual tissue). D is incorrect. Nasopalatine nerve block anesthetizes the hard and soft tissue of the lingual aspect of the maxillary anterior teeth from distal of canine on one side to distal of canine on the contralateral side. 6. D is correct. The incisive nerve block anesthetizes the premolars, canine, incisors, lower lip, skin over chin, and buccal soft tissue anterior to mental foramen. A is incorrect. The buccal nerve block anesthetizes the buccal soft tissue of the molar region. B is incorrect. Vazirani-Akinosa closed-mouth block anesthetizes the mandibular teeth to midline, hard and soft tissue of buccal aspect, anterior two-thirds of the tongue, and floor of the mouth. C is incorrect. The inferior alveolar nerve block anesthetizes mandibular teeth on side of injection, buccal and lingual hard and soft tissue, and lower lip. 7. B is correct. The Gow-Gates technique is an alternative to the inferior alveolar nerve block, providing a lower failure rate and a lower incidence of positive aspiration. The Gow-Gates technique anesthetizes the auriculotemporal, inferior alveolar, buccal, mental, incisive, mylohyoid, and lingual nerves. A is incorrect. The insertion point for the VaziraniAkinosa closed-mouth mandibular block is the gingival margin above the maxillary second and third molars. C is incorrect. The insertion point for the incisive nerve block is the height of the mucobuccal fold over the mental foramen. D is incorrect. The insertion point for the buccal nerve block is just distal and buccal to the last molar tooth on the side to be anesthetized. 8. D is correct. The middle superior alveolar nerve block anesthetizes the mesiobuccal root of maxillary first molar (in some cases), premolars, and surrounding hard and soft tissue on the buccal aspect.

18/04/19 6:14 PM

CHAPTER 35

A is incorrect. The nasopalatine nerve block anesthetizes the hard and soft tissue of lingual aspect of maxillary anterior teeth from distal of canine on one side to distal of canine on the contralateral side. B is incorrect. The mental nerve block anesthetizes the buccal soft tissue anterior to mental foramen, lower lip, and chin. C is incorrect. The greater palatine nerve block anesthetizes the palatal mucosa and hard palate from the first premolar anteriorly to the posterior aspect of the hard palate to the midline medially. 9. C is correct. The buccal nerve originates from the anterior mandibular division. A is incorrect. Nasopalatine nerve originates from pterygopalatine branch of the maxillary division. B is incorrect. Anterior superior alveolar nerve originates from the infraorbital branch of the maxillary division.

Hadzic_Ch35_p231-236.indd 235

Oral and Maxillofacial Regional Anesthesia

235

D is incorrect. The posterior ethmoid nerve originates from the nasociliary branch of the ophthalmic division. 10. C is correct. Intraseptal injection cannot be utilized for anesthetizing individual teeth. It only anesthetizes localized soft tissue. A is incorrect. Supraperiosteal injection can anesthetize individual teeth with associated buccal soft tissue. B is incorrect. Periodontal ligament injection can anesthetize individual teeth and associated buccal soft tissue. D is incorrect. An intrapulpal injection can anesthetize an individual tooth.

Suggested Reading Hadzic A. Oral & maxillofacial regional anesthesia. In: Preziosi BD, Hershkin AT, Seider PJ, Casey GM, eds. Textbook of Regional Anesthesia and Acute Pain Management. 2nd ed. New York, NY: McGraw-Hill Publishing; 2017:chap 36.

18/04/19 6:14 PM

This page intentionally left blank

Hadzic_FM_p00i-pxx.indd 6

22/04/19 6:40 PM

PART 3F

Local and Regional Anesthesia for the Eye Chapter 36

Hadzic_Ch36_p237-242.indd 237

Local and Regional Anesthesia for Ophthalmic Surgery  239

19/04/19 11:34 AM

This page intentionally left blank

Hadzic_FM_p00i-pxx.indd 6

22/04/19 6:40 PM

36 Local and Regional Anesthesia for Ophthalmic Surgery Stavros Prineas

QUESTIONS DIRECTIONS: Choose the one best response to each question. 1. In 2012, the most popular form of anesthesia employed by US cataract surgeons was: A. Retrobulbar block B. Sub-Tenon’s block C. Topical anesthesia alone D. Topical anesthesia with supplemental intracameral lidocaine E. Peribulbar block 2. Regarding the anatomy of the orbit, which of the following is true? A. There is no free communication between intraconal and extraconal spaces. B. The lateral walls of the orbits are roughly perpendicular to each other. C. The average axial (anteroposterior) length of the adult male globe is almost invariably 24 mm. D. All the nerves supplying the extraocular muscles of the eye pass through the muscular cone (or annulus of Zinn). E. The Tenon’s capsule extends from the conjunctival fornix anteriorly to the optic nerve posteriorly. 3. Important considerations in determining the type of anesthetic technique for a given ophthalmic procedure include: A. The preferences of the surgeon B. The need for akinesia of the eye C. Whether or not the patient is on anticoagulants D. Only all of the above E. At least all of the above 4. What is the most commonly used quadrant for a sub-Tenon’s block? A. Superotemporal B. Superonasal C. Inferotemporal

D. Inferonasal E. All quadrants are used equally. 5. Which of the following is not a recognized variation of the sub-Tenon’s technique? A. Ultrasound-guided technique using a Seldinger wire B. Nonsurgical technique with a blunt metal cannula C. Nonsurgical technique with a sharp metal needle D. Surgical (“snip”) technique with a blunt metal cannula E. Improvised technique using an intravenous cannula 6. Which statement is true regarding the recognized complications of retrobulbar and peribulbar blocks? A. While a number of complications may result in loss of vision in the injected eye, none of them is life-threatening. B. The oculocardiac reflex is partly mediated by the oculomotor (third) nerve. C. Perforation of the globe has a generally good prognosis. D. Inadvertent intra-arterial or subarachnoid injections that cause seizures or respiratory arrest are usually fatal even when managed appropriately. E. Sneezing occurs in up to one-fifth of patients receiving a local anesthetic injection under sedation. 7. Regarding local anesthetic agents in eye blocks, which of the following is not true? A. Current evidence suggests all agents cause reversible damage to the corneal epithelium, which usually goes undetected and resolves spontaneously. B. Current evidence suggests all agents cause reversible damage to extraocular muscles, which usually goes undetected and resolves spontaneously. C. Drugs that can demonstrably modify the action of local anesthetics in eye blocks include hyaluronidase, clonidine, and epinephrine. D. Because the dose of local anesthetic given in routine eye blocks is so small (3–11 mL), central nervous system toxicity is not a major concern. E. The most commonly used agents are lidocaine, bupivacaine, ropivacaine, and mepivacaine. 239

Hadzic_Ch36_p237-242.indd 239

19/04/19 11:34 AM

240

PART 3

Clinical Practice of Regional Anesthesia

8. Which statement is true regarding the use of sedation for eye blocks? A. For most eye procedures, heavy sedation is preferable during the performance of both the block and the surgery. B. An empathic and informative preoperative briefing can help allay anxiety and enlist the cooperation of the patient. C. Dexmedetomidine is not an acceptable sedative agent for eye surgery. D. Adequate sedation abolishes the oculocardiac reflex. E. The addition of midazolam to propofol sedation reduces the risk of sneezing during an eye block injection. 9. Which statement is true regarding eye blocks and the anticoagulated patient? A. Retrobulbar and peribulbar blocks are not considered high-risk techniques. B. Sight-threatening complications frequently occur with patients on aspirin who undergo a sub-Tenon’s block. C. Choosing the appropriate management plan for a given procedure requires a case-by-case evaluation of the patient and his or her medications against surgical requirements, rather than the application of a formula. D. It is easier to choose a relatively noninvasive technique (eg, topical or subconjunctival anesthesia) if akinesia is required. E. Dabigatran is a novel oral anticoagulant that can be reversed with vitamin K. 10. Which statement is true regarding blocks for adnexal (oculoplastic) surgery? A. A supratrochlear or supraorbital block can be used to prevent blinking during eye surgery. B. Globe perforation is a recognized complication of infraorbital nerve block. C. Tear duct surgery cannot be performed without a general anesthetic. D. Injecting the skin around the eye is not usually painful. E. Variations on a facial nerve block include the Van Sant, O’Blarney, and Atlee blocks.

ANSWERS AND EXPLANATIONS 1. D is correct. This answer is based on the 2012 survey of the American Society of Cataract and Refractive Surgeons. Sub-Tenon’s blocks are more popular in the UK and Australia. A, B, C, and E are incorrect. Based on the same US survey, these options were less popular. 2. B is correct. The cavity of each orbit is a truncated pyramid, with a flattened apex posteriorly and a trapezoidal base facing anterolaterally. The medial (nasal) walls of each orbit are parallel to each other, while the lateral (temporal) walls are offset approximately 45 degrees to the medial walls, making them perpendicular to each other.

Hadzic_Ch36_p237-242.indd 240

A is incorrect. The retrobulbar cone is not sealed by any intermuscular membrane, so there is free communication between the intraconal and extraconal spaces. C is incorrect. The average axial length of the adult eye is 24 mm but varies considerably across the human population, especially in myopic and hypermetropic individuals; that is why it needs to be individually measured prior to surgery. D is incorrect. The abducens nerve (the sixth cranial nerve) usually travels outside the muscular cone. E is incorrect. The Tenon’s capsule extends from the corneal limbus to the optic nerve. 3. E is correct. Other considerations include the cooperativity of the patient, ocular comorbidities (eg, infection or inflammation of the eye), whether the eye being operated on is the only functioning eye, and the volume effect of injections behind the eye. 4. C is correct. While any quadrant can theoretically be used, the most common is the inferotemporal. Other quadrants tend to be used if the inferotemporal cannot for some reason (eg, scarring in that quadrant from multiple previous blocks or pterygium repair, or the presence of a retinal buckle). A, B, D, and E are incorrect. While all quadrants can theoretically be used, the inferonasal (C) quadrant is by far the most commonly used. 5. A is correct. This is not a recognized variation of the sub-Tenon’s technique. Ultrasound-guided eye blocks have been reported but not with a Seldinger technique, which is usually reserved for intravascular or airway access. B, C, D, and E are incorrect. All of these are recognized variations of the sub-Tenon’s technique. 6. E is correct. Sneezing is more likely to occur in patients who are male, have a history of photic sneezing (sneezing with sudden exposure to bright light or sunshine), are under deeper sedation, or who are given concurrent midazolam. It does not appear to occur in patients undergoing blocks with remifentanil sedation or with no sedation at all. A and D are incorrect. Inadvertent intra-arterial injections can cause seizures; subarachnoid injections can cause respiratory arrest. Both are life-threatening; however, when diagnosed promptly and with appropriate resuscitation, they do not usually lead to death. B is incorrect. The oculocardiac reflex is mediated by afferent branches of the trigeminal (fifth) nerve and efferent branches of the vagus (tenth) nerve. C is incorrect. Perforation of the globe usually carries a poor prognosis for sight in the affected eye. 7. D is not true, so option D is correct. All anesthetic agents appear to be myotoxic and corneotoxic to a greater or lesser degree, especially in higher concentrations. A, B, C, and E are true, so options A, B, C, and E are incorrect. While systemic local anesthetic toxicity is not a major concern given the relatively small doses given with

19/04/19 11:34 AM

CHAPTER 36

eye blocks, direct intra-arterial or subarachnoid central nervous system toxicity have both been described and are life-threatening. 8. B is correct. Patients should be given a clear explanation of what will be done to them, what they are likely to experience, and what they may be asked to do during the operation, in language appropriate to their level of understanding. A is incorrect. Heavy sedation while having limited access to the patient and his or her airway invites the risk of restlessness, snoring, or respiratory depression with desaturation, or abrupt awakening during the procedure. C is incorrect. Dexmedetomidine is gaining popularity as a sedative agent; however, it requires a 20 minute priming time to prevent undesirable side effects. D is incorrect. Neither sedation nor even general anesthesia can guarantee to abolish the oculocardiac reflex, which can only be mitigated by prophylactic anticholinergic agents. E is incorrect. The addition of midazolam to propofol sedation actually increases the likelihood of sneezing during an eye block injection. 9. C is correct. Nowadays there is not only a diversity of anticoagulant therapies (several of which defy easy assessment), but also a range of anesthetic management options. Moreover, the risks of discontinuing anticoagulant therapy in some patients may outweigh the risks of perioperative bleeding on full anticoagulation. Consequently, there is

Hadzic_Ch36_p237-242.indd 241







241

Local and Regional Anesthesia for Ophthalmic Surgery

no clear algorithm for managing patients undergoing eye surgery who are on anticoagulants. A is incorrect. Techniques that involve the passage of a sharp needle blindly into the orbit are considered high risk in anticoagulated patients. B is incorrect. Limited studies suggest that sub-Tenon’s blocks in patients on aspirin, warfarin, or clopidogrel may cause subconjunctival hematoma but not sight-threatening complications. D is incorrect. Akinesia is not possible with relatively non-invasive techniques. E is incorrect. There is no antidote for dabigatran.

10. B is correct. Globe perforation is a recognized complication of infraorbital nerve block. A is incorrect. A facial nerve block is required to reduce excessive blinking during surgery. C is incorrect. Tear duct surgery can be done under local using a combination of a nasociliary block, infraorbital block, and a band of infiltration between the two. D is incorrect. Blocks around the eye are usually quite painful, requiring transient deep sedation. E is incorrect. Facial nerve block variants include the Van Lint, O’Brien, and Atkinson blocks.

Suggested Reading Hadzic A. Local and regional anesthesia for ophthalmic surgery. In: Prineas S, ed. Textbook of Regional Anesthesia and Acute Pain Management. 2nd ed. New York, NY: McGraw-Hill Publishing; 2017:chap 37.

19/04/19 11:34 AM

This page intentionally left blank

Hadzic_FM_p00i-pxx.indd 6

22/04/19 6:40 PM

PART 4 Ultrasound Imaging of Neuraxial and Perivertebral Space Chapter 37

Sonography of the Lumbar Paravertebral Space and Considerations for Ultrasound-Guided Lumbar Plexus Block 245

Chapter 38

Lumbar Paravertebral Sonography and Considerations for Ultrasound-Guided Lumbar Plexus Block 249

Chapter 39

Spinal Sonography and Applications of Ultrasound for Central Neuraxial Blocks 255

Hadzic_Ch37_p243-248.indd 243

22/04/19 2:02 PM

This page intentionally left blank

Hadzic_FM_p00i-pxx.indd 6

22/04/19 6:40 PM

37 Sonography of the Lumbar Paravertebral Space and Considerations for UltrasoundGuided Lumbar Plexus Block Hiroaki Murata, Tatsuo Nakamoto, and Takayuki Yoshida

QUESTIONS DIRECTIONS: Choose the one best response to each question. 1. Which of the following statements is correct regarding lumbar plexus block (LPB)? A. LPB is a superficial block. B. LPB is accomplished by a small amount (less than 5 mL) of local anesthetic solution. C. Traditionally, LPB has been performed under computed tomography or fluoroscopy guidance. D. Ultrasound can monitor the needle path and final needle tip placement during LPB. 2. Where are the roots of the lumbar plexus located at the level of L3–L4? A. Between the erector spinae and psoas major muscles B. In the fascial plane within the psoas major muscle C. Lateral to the quadratus lumborum muscle D. Posterior to the transverse process 3. Which of the following is located lateral to the psoas major muscle? A. Erector spinae muscle B. Quadratus lumborum muscle C. Vertebral body D. Transverse process 4. Which of the following is not included in the lumbar plexus? A. Femoral nerve B. Lateral femoral cutaneous nerve C. Obturator nerve D. Sciatic nerve

5. During the ultrasound-guided lumbar plexus block in the transverse oblique view on the left side of the body, which of the following structures can be seen deep to the psoas major muscle and adjacent to the vertebral body? A. Abdominal aorta B. Erector spinae muscle C. Inferior vena cava D. Kidney 6. Which of the following statements is true regarding the lumbar plexus block using a transverse in-plane approach? A. A linear transducer is positioned approximately 4 cm lateral from the midline at the level just cephalad to the iliac crest with the depth at 4 to 6 cm. B. Once the needle reaches the appropriate position, higher injection pressure is required to obtain sufficient local anesthetic spread. C. The dip between the sacrum identified as a flat hyperechoic structure with a large acoustic shadow and the L5 transverse process recognized by its crescent-shaped hyperechoic reflections is useful to identify each transverse process. D. The transverse process should be clearly visualized to obtain adequate images of the psoas major muscle and lumbar plexus. 7. In terms of the approaches for ultrasound-guided lumbar plexus block, what does the “shamrock” consist of? A. Erector spinae, latissimus dorsi, and quadratus lumborum muscles B. Erector spinae, psoas major, and quadratus lumborum muscles C. Latissimus dorsi, psoas major, and quadratus lumborum muscles D. Psoas major, quadratus lumborum, and transversus abdominis muscles

245

Hadzic_Ch37_p243-248.indd 245

22/04/19 2:02 PM

246

PART 4

Ultrasound Imaging of Neuraxial and Perivertebral Space

8. Which of the following statements is true regarding the lumbar plexus block (LPB) using the shamrock method? A. Needle trajectory of the shamrock method is almost the same as in the traditional landmark-guided LPB. B. Needle visualization is difficult because the needle insertion angle is steep during the shamrock method. C. The shamrock method is performed with the patient in supine position. D. The transverse process is a key landmark for needle advancement during the shamrock method. 9. Which of the following signs is visualized in the parasagittal longitudinal ultrasound image at the level of the lumbar transverse process? A. Camel hump sign B. Frog eye sign C. Horse head sign D. Trident sign 10. Which of the following statements is true regarding the ultrasound-guided lumbar plexus block using the parasagittal longitudinal approach? A. Both out-of-plane and in-plane approaches can be employed. B. Ipsilateral dorsiflexion of the foot should be confirmed before injecting the local anesthetic. C. The quadratus lumborum muscle is a key structure to identify the lumbar plexus. D. The vertebral bodies are imaged deep to the transverse processes.

ANSWERS AND EXPLANATIONS 1. D is correct. Ultrasound (US) is increasingly being used to guide peripheral nerve blocks, and it is only logical that ultrasound-guided LPB is of interest because of the ever-increasing availability of US machines that produce high-quality images in the operating room. US has been used to preview the relevant anatomy, measure the depth to the transverse process, guide the block needle to the posterior aspect of the psoas muscle or the lumbar plexus in real time, and monitor needle–nerve contact or spread of local anesthetic during an LPB. Understanding the sonoanatomy of the lumbar paravertebral region is a prerequisite for US-guided LPB. A is incorrect. The lumbar plexus lies deep to and within the psoas muscles. B is incorrect. Because the roots of the lumbar plexus are located in the fascial plane within the posterior aspect of the psoas major muscle, an injection of a sufficient volume of local anesthetic (approximately 20–30 mL) to fill the compartment is required. C is incorrect. Traditionally, LPB has been performed using surface anatomical landmarks to identify the site for needle insertion and eliciting quadriceps muscle contraction in response to nerve stimulation. The main

Hadzic_Ch37_p243-248.indd 246

challenges with accomplishing LPB with anatomical landmarks and peripheral nerve stimulation relate to the depth at which the lumbar plexus is located. Small errors in estimation of landmark or angle of needle insertion can lead to the block needle being directed away from the plexus, resulting in inadvertent deep needle insertion or renal or vascular injury. Therefore, real-time monitoring of the needle and local anesthetic injection during an LPB is desirable and may improve the accuracy and safety of the technique. Although computed tomography and fluoroscopy can be used to increase precision during LPB, these technologies are impractical in terms of costand time-effectiveness, and more importantly, exposure to radiation. 2. B is correct. The lumbar plexus is located in an intramuscular fascial plane or “compartment” formed by the fascia between the anterior two-thirds of the compartment of the psoas major muscle that originates from the anterolateral aspect of the vertebral body and the posterior one-third of the muscle that originates from the anterior aspect of the transverse processes. A and D are incorrect. No components of the roots of the lumbar plexus run between the erector spinae and psoas muscles or posterior to the transverse process. C is incorrect. Some branches of the lumbar plexus pass through the interfascial plane located anteromedial to the quadratus lumborum muscle. 3. B is correct. The quadratus lumborum muscle exists lateral to the psoas major muscle. A and D are incorrect. The erector spinae muscles and the transverse process locate posterior to the psoas major muscle. C is incorrect. The vertebral body locates medial to the psoas major muscle. 4. D is correct. Sciatic nerve is not included in the lumbar plexus. A, B, and C are incorrect. The lumbar plexus is a web of nerves in the lumbar region of the body. It is formed by the anterior rami of L1, L2, L3 and the greater part of L4. It also receives variable contribution of the subcostal nerve (T12). The branches of the lumbar plexus consist of the iliohypogastric, ilioinguinal, genitofemoral, lateral femoral cutaneous, obturator, and femoral nerves. LPB produces anesthesia of the ipsilateral lateral femoral cutaneous, obturator, and femoral nerves. The sciatic nerve is formed from the L4 to S3 segments of the sacral plexus. 5. A is correct. The abdominal aorta is seen deep to the psoas major muscle and adjacent to the vertebral body in the transverse oblique view on the left side of the body. B is incorrect. The erector spinae muscle is seen adjacent to spinous process and lumbar lamina.

22/04/19 2:02 PM

CHAPTER 37

Sonography of the Lumbar Paravertebral Space and Considerations for Ultrasound-Guided Lumbar Plexus Block

C is incorrect. The inferior vena cava is seen deep to the psoas major muscle and adjacent to the vertebral body in the transverse oblique view on the right side of the body. D is incorrect. The kidney is seen lateral to the psoas major muscle and deep to the quadratus lumborum muscle. It locates in the retroperitoneal space, so the image of the kidney is moving with respiration. 6. C is correct. The dip between the sacrum identified as a flat hyperechoic structure with a large acoustic shadow and the L5 transverse process recognized by its crescentshaped hyperechoic reflections is useful to identify each transverse process. A is incorrect. Transverse in-plane approach for lumbar plexus block is performed by placing the curved array transducer approximately 4 cm lateral from the midline at the level just cephalad to the iliac crest with the depth at 9 to 12 cm. B is incorrect. High injection pressure may lead to unwanted epidural spread. D is incorrect. The acoustic shadow provided by transverse process interferes with the image of the psoas major muscle. The window through the lumbar intertransverse space is recommended to obtain clear images of the psoas major muscle and the lumbar plexus. 7. B is correct. “Shamrock” means as three leaves, including the erector spinae, psoas major, and quadratus lumborum muscles. A, C, and D are incorrect. The latissimus dorsi and transversus abdominis muscles, forming parts of the outer wall of the trunk, are also visible using the shamrock method, but not included in the “shamrock.” 8. A is correct. In the shamrock method, the transducer is placed transversely in the abdominal flank adjacent to the iliac crest in a lateral decubitus position patient. The needle insertion point is based on the landmark-guided method; that is, approximately 4 cm lateral to the midline or at the junction of the lateral third and medial twothirds of a line between the spinous process of L3 and a line parallel to the spinal column passing through the posterior superior iliac spine. B is incorrect. The needle can be advanced almost perpendicularly to the ultrasound beam, which allows for a clearer visualization of the needle. C is incorrect. The shamrock method is performed with a patient in a lateral decubitus position. D is incorrect. The acoustic shadow of the lumbar transverse process constitutes the shamrock view. However, with the shamrock view, the transverse process interferes

Hadzic_Ch37_p243-248.indd 247

247

with needle advancement toward the lumbar plexus. By tilting the transducer caudally or cranially from the shamrock view, the transverse process disappears from the ultrasound image, which permits an in-plane posteroanterior needle approach toward the plexus. 9. A is incorrect. The articular processes in a sagittal sonogram produce a sonographic pattern that resembles multiple camel humps, which are referred to as the “camel hump sign.” B is incorrect. The two sacral cornua and the posterior surface of the sacrum in a transverse sonogram produce a characteristic appearance, referred to as the “frog eye sign.” C is incorrect. The sagittal longitudinal sonographic appearance of the lamina produces a pattern that resembles the head and neck of a horse, referred to as the “horse head sign.” D is correct. In the parasagittal longitudinal ultrasound image, the acoustic shadow of the transverse process has a characteristic appearance, referred to as the “trident sign.” 10. A is correct. Both out-of-plane and in-plane approaches can be employed with the parasagittal longitudinal technique. B is incorrect. Electrical nerve stimulation of the lumbar plexus induces ipsilateral quadriceps muscle contractions. Ipsilateral dorsiflexion of the foot is elicited by peroneal nerve stimulation. C is incorrect. The quadratus lumborum muscle is a key structure to identify the lumbar plexus with the shamrock approach, but cannot be seen with the parasagittal longitudinal approach. D is incorrect. The transducer is placed lateral and parallel to the lumbar spine with the parasagittal longitudinal approach. With this transducer position, the psoas major muscle is imaged deep to the transverse processes through the acoustic window of them.

Suggested Readings Hong Kwok W, Karmakar M. Ultrasound-guided spinal and epidural block. NYSORA website. http://www.nysora.com/ spinal-and-epidural-block. Hadzic A. Sonography of the lumbar paravertebral space and considerations for ultrasound-guided lumbar plexus block. In: Murata H, Nakamoto T, Yoshida T, Karmakar MK, eds. Textbook of Regional Anesthesia and Acute Pain Management. 2nd ed. New York, NY: McGraw-Hill Publishing; 2017:chap 38. Sauter AR, Ullensvang K, Niemi G, Lorentzen HT, Bendtsen TF, Borglum J, Pripp AH, Romundstad L. The Shamrock lumbar plexus block: a dose-finding study. Eur J Anaesthesiol. 2015;32:764-70.

22/04/19 2:02 PM

This page intentionally left blank

Hadzic_FM_p00i-pxx.indd 6

22/04/19 6:40 PM

38 Lumbar Paravertebral Sonography and Considerations for Ultrasound-Guided Lumbar Plexus Block Hari Kalagara, Sree Kolli, Vishal Uppal, Christopher B. Robards, and Manoj K. Karmakar

QUESTIONS DIRECTIONS: Choose the one best response to each question. 1. Mark the correct statement regarding the lumbar plexus (LP). A. The LP is formed by contribution from dorsal primary rami of L1–L4. B. The LP lies within the anterior one-third of psoas muscle. C. Nerves originating from the LP follow a relatively consistent pattern with the lateral femoral cutaneous nerve being innermost. D. Psoas muscle contains lumbar nerve roots and branches of the lumbar artery. 2. Lumbar plexus block (LPB) is best suited to provide analgesia following which of the following surgical procedures? A. Achilles tendon repair B. Ankle fusion C. Hallux valgus correction D. Total hip arthroplasty 3. During performance of ultrasound (US)-guided lumbar plexus block (LPB): A. Basic skill is required for interpretation of sonogram and needling. B. A trident sonogram can be visualized in transverse scan. C. A shamrock sonogram can be visualized in parasagittal scan. D. Dual guidance using ultrasound and nerve stimulation is recommended while performing the block. 4. While performing lumbar plexus block (LPB), which of the following muscle twitches is used as an endpoint? A. Biceps femoris B. Semimembranosus

C. Semitendinosus D. Quadriceps femoris 5. Which of the following has not been described as a complication of lumbar plexus block (LPB)? A. Epidural spread B. Foot drop C. Renal injury D. Retroperitoneal hematoma 6. Which statement is true regarding ultrasound (US)guided lumbar plexus block (LPB) techniques? A. During the transverse scan, it is preferable to perform the block at a paramedian oblique scan at transverse process (PMTOS-TP) view rather than PMTOS-AP (articular process) view. B. The lumbar nerve can be easily visualized during a paramedian oblique scan at the transverse process (PMTOS-TP). C. The bony shadow of trident during parasagittal ultrasound scanning represents spinous process. D. With the shamrock approach, visualizing the needle initially can be challenging. 7. Which of the following statements is true regarding performing a lumbar plexus block (LPB) using a lowfrequency curved array ultrasound probe? A. After exiting the intervertebral foramina, the lumbar nerve roots enter the lumbar paravertebral space and continue into the psoas major at the same vertebral level. B. Prone position is the most preferred position as it is easy to visualize the quadriceps twitches on neurostimulation of the lumbar plexus. C. The acoustic shadows of the transverse processes forming the trident sign are used as the landmark in transverse scanning for a LPB. D. The three muscles that form the shamrock during the transverse scan at the flank are quadratus lumborum, psoas major, and the erector spinae. 249

Hadzic_Ch38_p249-254.indd 249

22/04/19 2:04 PM

250

PART 4

Ultrasound Imaging of Neuraxial and Perivertebral Space

8. An 82-year-old man presents for a total hip arthroplasty revision surgery. His past medical history is significant for osteoarthritis, hypertension, and multiple deep vein thrombosis (DVTs) with pulmonary embolism. He is on long-term anticoagulation. Regarding performing a lumbar plexus block (LPB) in this patient for postoperative pain relief, which of the following is true? A. Any hyperechoic shadows or stripes seen within the psoas major muscle are part of the lumbar plexus. B. LPB is safe to perform in this patient as the psoas compartment is avascular and there is minimal risk of bleeding. C. The ultrasound image of skeletal muscle in elderly patients appears brighter and whiter compared to younger individuals, making it difficult to identify the LP. D. When using a dual guidance technique with ultrasound and nerve stimulation, a satisfactory block cannot be achieved if no motor response can be elicited. 9. Which of the following is true regarding the anatomy and appearance of the lumbar plexus (LP) during ultrasound scanning? A. The LP is located between the psoas major muscle and the quadratus lumborum muscle. B. The LP can be identified by using the loss of resistance technique at the level of L1. C. The LP is formed by the union of the anterior primary rami of L1, L2, L3, and L4 with variable contribution from T12 and L5. D. The ultrasound (US)-guided approach to the LPB is a basic-level block that requires minimal training. 10. While performing a lumbar plexus block (LPB), the injection pressure encountered was more than 20 psi. After 30 minutes from the procedure, the patient is unable to move both the lower extremities. Which of the following is the most likely complication in this patient? A. Bilateral lumbar plexus block B. Epidural block C. Retroperitoneal hematoma D. Sciatic nerve block 11. Which of the following movements is preserved following a lumbar plexus block (LPB) via posterior approach? A. Hip adduction B. Hip flexion C. Lower leg extension D. Plantar flexion of the foot 12. Within the substance of the psoas muscle, the lumbar paravertebral space is formed by which of the following combinations? A. Large (fleshy) anterior part of the psoas muscle originating from the anterolateral surface of the vertebral body, and the thinner (accessory) posterior part of the muscle, originating from the anterior aspect of the transverse processes B. Large (fleshy) posterior part of the psoas muscle originating from the anterolateral surface of the vertebral body, and the thinner (accessory) anterior part of the

Hadzic_Ch38_p249-254.indd 250

muscle, originating from the anterior aspect of the transverse processes C. Large (fleshy) anterior part of the psoas muscle originating from the anterolateral surface of the transverse processes and the thinner (accessory) posterior part of the muscle originating from the anterior aspect of the vertebral body D. Thinner (accessory) anterior part of the psoas muscle originating from the anterolateral surface of the vertebral body, and the large (fleshy) posterior part of the muscle, originating from the anterior aspect of the transverse processes 13. Which of the following statements correctly describes the anatomy of the lumbar plexus? A. The nerves exhibit a fanned-out distribution, with the lateral femoral cutaneous (LFC) being outermost, the obturator nerve (OBN) innermost, and the femoral nerve (FN) in between. The positions of the LFC and FN within the psoas compartment are variable and may even lie in a fold of the psoas muscle, but the position of the OBN is relatively consistent. B. The nerves exhibit a fanned-out distribution, with the obturator nerve (OBN) being outermost, the lateral femoral cutaneous (LFC) being innermost, and the femoral nerve (FN) in between. The positions of the LFC and FN within the psoas compartment are variable and may even lie in a fold of the psoas muscle, but the position of the OBN is relatively consistent. C. The nerves exhibit a fanned-out distribution, with the femoral nerve (FN) being outermost, the lateral femoral cutaneous (LFC) being innermost, and the obturator nerve (OBN) in between. The positions of the LFC and FN within the psoas compartment are relatively consistent, but the position of the OBN is variable and may even lie in a fold of the psoas muscle. D. The nerves exhibit a fanned-out distribution, with the lateral femoral cutaneous (LFC) being outermost, the obturator nerve (OBN) innermost, and the femoral nerve (FN) in between. The positions of the LFC and FN within the psoas compartment are relatively consistent, but the position of the OBN is variable and may even lie in a fold of the psoas muscle. 14. Which of the following techniques has been described for scanning of the lumbar plexus? A. Transverse scan B. Sagittal scan (“trident sign”) C. Paramedian transverse oblique scan (PMTOS) D. Transverse flank (“shamrock sign”) E. All the above F. A, B, C 15. Regarding ultrasound imaging of the lumbar plexus (LP), which of the following is/are true? A. The depth of the LP necessitates the use of lowfrequency ultrasound (5–10 MHz) and curved array transducers. B. Low-frequency ultrasound provides good penetration but lacks spatial resolution at the depths (5–9 cm) at which the anatomy relevant for LPB is located.

22/04/19 2:04 PM

CHAPTER 38

Lumbar Paravertebral Sonography and Considerations for Ultrasound-Guided Lumbar Plexus Block

C. Improvements in US technology, the image processing capabilities of ultrasound machines, the availability of compound imaging and tissue harmonic imaging (THI), and the use of new ultrasound scan protocols have all contributed to improved imaging of the lumbar paravertebral region. D. The LP can be adequately imaged in most patients using a high frequency (10–14 MHz) linear array transducer. E. All of the above F. A, B, C 16. The shamrock method of identifying the lumbar plexus (LP) involves positioning the transducer such that which three muscles are visualized in a recognizable pattern? A. External abdominal oblique, internal abdominal oblique, transversus abdominis B. Transversus abdominis, psoas, erector spinae C. Erector spinae, quadratus lumborum, psoas D. Quadratus lumborum, internal abdominal oblique, external abdominal oblique 17. The vascular supply to the lumbar paravertebral region includes which of the following vessels? A. Ascending lumbar veins B. Artery of Adamkiewicz C. Ascending lumbar arteries D. Dorsal branch of the lumbar artery E. A, C, and D F. All of the above 18. Which of the following statements is true regarding ultrasound imaging of the lumbar paravertebral region in the elderly? A. It appears whiter and brighter than in younger patients, and there is also a loss of contrast between the muscle and the adjoining structures, making it more difficult to delineate the lumbar plexus (LP). B. It appears blacker and less bright than in younger patients, and there is also a loss of contrast between the muscle and the adjoining structures, making it more difficult to delineate the LP. C. It appears whiter and brighter than in younger patients, and there is also an increased contrast between the muscle and the adjoining structures, making it easier to delineate the LP. D. It appears blacker and less bright than in younger patients, and there is also an increased contrast between the muscle and the adjoining structures, making it easier to delineate the LP. 19. When performing a lumbar plexus block (LPB), a stimulating needle is observed on ultrasound (US) imaging to be in the posterior aspect of the psoas muscle but fails to elicit a motor response. Why would this be? A. There is a malfunction of the nerve stimulator. B. The stimulating needle may have a manufacturing defect. C. The needle is inserted in the upper lumbar region (L1 and L2).

Hadzic_Ch38_p249-254.indd 251

251

D. The needle is inserted in the middle lumbar region (L3 and L4). E. All of the above F. A, B, and C 20. Which of the following describes the paramedian transverse oblique scan (PMTOS) technique for scanning the lumbar plexus (LP)? A. It is a sagittal scan that was initially described by Karmakar and colleagues. The acoustic shadow of the transverse processes produces a sonographic pattern referred to as the “trident sign.” B. It is a transverse scan that was the first described by Kirchmair and colleagues and they were able to describe the detailed transverse sonoanatomy of the lumbar paravertebral region relevant for LPB. C. It is a transverse flank technique that was described by Sauter and colleagues as an alternative approach for USG LPB. As described, a transverse scan is performed at the flank and immediately above the iliac crest, with the patient in the lateral position and with the side to be blocked uppermost. D. It is a paramedian transverse technique first described by Karmakar. The inferior vena cava (IVC; on the right side) and the aorta (on the left side) are identified anterior to the vertebral body and are useful landmarks to look out for while performing this technique.

ANSWERS AND EXPLANATIONS 1. D is correct. Psoas muscle does contain lumbar nerve roots and branches of the lumbar artery. A is incorrect. The LP is formed by ventral (or anterior) primary rami of L1–L4 (not dorsal rami). B is incorrect. The LP lies within the posterior one-third of psoas. C is incorrect. The nerves that originate from the LP exhibit a somewhat consistent distribution, with the lateral femoral cutaneous nerve being outermost, the obturator nerve innermost, and the femoral nerve in between. The positions of the lateral femoral cutaneous nerve and femoral within the psoas compartment are relatively consistent, but the location of the obturator nerve is variable. 2. D is correct. The hip joint is supplied by the femoral and obturator nerves that arise from the lumbar plexus. Other nerves that supply the hip joint such as sciatic, and superior gluteal nerves, as well as the nerve to the quadratus femoris arise from sacral plexus. A, B, and C are incorrect. Hip and knee joints get a significant innervation from branches of the lumbar plexus. On the contrary, the foot and ankle are mainly supplied by the sciatic nerve with a minor contribution by saphenous nerve. 3. D is correct. It is not always possible to accurately delineate the lumbar plexus nerves within the psoas muscle during

22/04/19 2:04 PM

252

PART 4

Ultrasound Imaging of Neuraxial and Perivertebral Space

ultrasound scans. Therefore, the use of peripheral nerve stimulation in addition to ultrasound imaging (dual guidance) is recommended during the performance of LPB. A is incorrect. USG LPB is considered an advanced-skilllevel block and should be performed only after one has acquired the appropriate level of training and skill. B is incorrect. The trident sonogram can be visualized in the parasagittal scan. C is incorrect. The shamrock sonogram can be visualized in the transverse scan. 4. D is correct. Motor innervation of quadriceps femoris is from branches of the lumbar plexus. A, B, and C are incorrect. The biceps femoris, semimembranosus, and semitendinosus muscles are supplied by branches of the sciatic nerve. 5. B is correct. Foot drop has not been described as complication of lumbar plexus block (LPB). It is a complication of damage to the fibular branch of the sciatic nerve, which is not a part of lumbar plexus. A, C, and D are incorrect. Retroperitoneal hematoma, epidural spread, and renal injury have been reported as complications of LPB. 6. D is correct. With the shamrock method, visualizing the needle can be quite challenging initially, because the sites for the US scan and needle insertion are separated by considerable distance. A is incorrect. It is preferable to use a PMTOS-AP with the patient positioned in the lateral position because it provides better visualization of the anatomy relevant for LPB. B is incorrect. During a PMTOS-TP the lumbar plexus is not sonographically visualized in all patients but, when visualized, the lumbar plexus is seen as a hyperechoic structure in the posterior part of the psoas muscle. C is incorrect. During a parasagittal US scan, the bony shadows of trident are produced by transverse process shadows. 7. D is correct. The shamrock method is a transverse scan performed at the flank and above the iliac crest, with the patient in the lateral position. The three muscles that form the shamrock are quadratus lumborum at the apex, psoas major anteriorly, and the erector spinae posteriorly. A is incorrect. The lumbar nerve roots enter the lumbar paravertebral space after exiting the intervertebral foramen. From the lumbar paravertebral space, they do not enter psoas muscle at the same vertebral level, as they take a steep caudal course and enter the psoas compartment at the vertebral level below. This explains why the L3 contribution to the lumbar plexus lies opposite the L4 intervertebral foramen and the L4 nerve root. B is incorrect. Lumbar plexus can be scanned with the patient in lateral, sitting, or prone positions. Prone position impairs visualization of the quadriceps muscle contraction that is used as an endpoint for needle placement. C is incorrect. The trident sign is formed by the acoustic shadow of the transverse processes while scanning in the

Hadzic_Ch38_p249-254.indd 252

sagittal plane parallel to the midline. The psoas muscle is seen through the acoustic windows between the shadows of transverse processes. 8. C is correct. The echo intensity (EI) of skeletal muscles is significantly increased in the elderly, and therefore the lumbar paravertebral region in the elderly appears whiter and brighter than in younger patients. There is also a loss of contrast between the muscle and the adjoining structures in elderly patients, making it more difficult to delineate the LP. A is incorrect. Psoas muscle contains intramuscular tendons, which also produce hyperechoic shadows. The nerves of the LP can be differentiated from the intramuscular tendons as they are thicker than the muscle fibers, take an oblique course through the psoas muscle, and are better visualized after local anesthetic injection. B is incorrect. The lumbar paravertebral region is highly vascular and contains the ascending lumbar veins and lumbar arteries. The dorsal branch of the lumbar artery is close to the LP and the psoas major muscle itself has a rich network of blood vessels. LPB in patients with mild to moderate coagulopathy or in patients receiving thromboprophylaxis should be performed with caution. D is incorrect. Sometimes no motor response is elicited even though the needle tip is close to the hyperechoic shadows in the posterior aspect of the psoas muscle. This may not be an uncommon phenomenon, considering that it is commonly seen during upper extremity blocks. Care should be taken not to insert the needle in the upper lumbar region because the high lumbar nerves (L1 and L2) contribute predominantly to sensory nerves, and stimulating these nerves may not elicit a motor response. 9. C is correct. The LP is formed by the union of the anterior primary rami of L1, L2, and L3 and the greater part of L4 within the psoas muscle. It also receives a variable contribution from T12 (subcostal nerve) and L5. A is incorrect. The LP is located in an intramuscular fascial plane or compartment also referred to as the psoas compartment, within the posterior one-third of the psoas muscle and is very closely related to the lumbar transverse processes. B is incorrect. The LP is located within the psoas muscle, and the local anesthetic is injected into a fascial plane within the posterior aspect of the psoas muscle during an LPB and cannot be identified by loss of resistance. The idea of loss of resistance to perform LPB was from the wrong belief of Chayen and colleagues that branches of the lumbar plexus and parts of the sacral plexus were located between the psoas major and quadratus lumborum muscles at the level of the L4 vertebra and could be identified using “loss of resistance.” D is incorrect. Interpretation of the LP sonoanatomy and the actual performance of this deeper block require a higher level of training and skill. Performing the LP ultrasound scanning and the block should be considered an advanced-skill-level block.

22/04/19 2:04 PM

CHAPTER 38

Lumbar Paravertebral Sonography and Considerations for Ultrasound-Guided Lumbar Plexus Block

10. B is correct. Injection of local anesthetic under high pressure (> 20 psi) during an LPB results in the unwanted bilateral sensory motor blockade and a high incidence of epidural spread. Therefore, one must ensure that the injection pressure is low (< 15 psi) during ultrasound-guided LPB. A, C, and D are incorrect. See explanation for answer B. 11. D is correct. LPB spares the sciatic nerve, which is responsible for innervation of the muscles that allow plantar flexion of the foot (gastrocnemius, soleus, plantaris, and digital flexors). The posterior approach for lumbar plexus block reliably blocks the femoral, lateral femoral cutaneous, and obturator nerves. A is incorrect. Hip adduction is achieved by contraction of the adductor muscles. The adductor muscles are innervated by the obturator nerve. The obturator nerve is blocked during an LPB. B is incorrect. Hip flexion is achieved by contraction of the iliacus and psoas muscles, which are innervated by the anterior rami of L1–L3, which are blocked during an LPB. C is incorrect. Lower leg extension is achieved by contraction of the quadriceps femoris and is innervated by the femoral nerve. The femoral nerve is blocked during an LPB. 12. A is correct. The sentence describes the correct anatomy of the psoas muscle, having a large fleshy anterior portion originating from vertebral body and a thinner accessory portion originating from the transverse process. B is incorrect. The large (fleshy) part of the psoas is its anterior part, not its posterior part, and the thinner (accessory) part is posterior and not anterior. C is incorrect. The large (fleshy) part originates from the anterolateral portion of the vertebral body, not the transverse process, and the thinner (accessory) part originates from the transverse process, not the vertebral body. D is incorrect. The thinner (accessory) part of the muscle is posterior and originates from the transverse process. 13. D is correct. The listing of nerves from lateral to medial is correct. The LFC is most lateral, the OBN is most medial, and the FN rests in between. The LFC and FN are consistent in their orientation while the OBN is variable. A is incorrect. The listing of nerves from lateral to medial is correct; however, the positions of the LFC and FN are consistent, not variable. The course of the obturator nerve is variable and may lie in a fold of the psoas muscle. B is incorrect. The listing of nerves lateral to medial is incorrect. The LFC is most lateral and OBN is most medial. In addition, the positions of the LFC and FN are consistent, not variable. The course of the obturator nerve is variable and may lie in a fold of the psoas muscle. C is incorrect. Listing of nerves lateral to medial is incorrect. The LFC is most lateral and OBN is most medial. 14. E is correct. All of the listed scanning techniques (A, B, C, and D) have been described.

Hadzic_Ch38_p249-254.indd 253

253

The transverse scan was the first described technique, by Kirchmair and colleagues, who were able to describe the detailed transverse sonoanatomy of the lumbar paravertebral region relevant for LPB. The sagittal scan (“trident sign”) was described by Karmakar and colleagues. The acoustic shadow of the transverse process produces a sonographic pattern referred to as the “trident sign.” The paramedian transverse oblique scan (PMTOS) was described by Karmakar. The inferior vena cava (IVC; on the right side) and the aorta (on the left side) are also identified anterior to the vertebral body and are useful landmarks to look out for while performing a PMTOS. The transverse flank (“shamrock sign”) was described by Sauter and colleagues as an alternative approach for USG LPB. As described, a transverse scan is performed at the flank and immediately above the iliac crest, with the patient in the lateral position and with the side to be blocked uppermost. F is incorrect. This option is incorrect because the transverse flank (“shamrock sign”) has been described. 15. A, B, and C are all true, so option F is correct. The depth of the LP necessitates the use of lowfrequency ultrasound (5–10 MHz) and curved array transducers. Low-frequency ultrasound provides good penetration but lacks spatial resolution at the depths (5–9 cm) at which the anatomy relevant for LPB is located. Improvements in US technology, the image processing capabilities of ultrasound machines, the availability of compound imaging and tissue harmonic imaging (THI), and the use of new ultrasound scan protocols have all contributed to improved imaging of the lumbar paravertebral region. D and E are incorrect. The deep location of the LP necessitates the use of a curvilinear transducer with lowfrequency output. 16. C is correct. The erector spinae, quadratus lumborum, and psoas muscles make up the leaves of a three-leaf clover, also known as the “shamrock sign.” A is incorrect. These are the three abdominal wall muscles and have little to do with locating the LP. B is incorrect. Psoas and erector spinae are important muscles to identify, but the transversus abdominis is not. D is incorrect. While the quadratus lumborum muscle is important in localizing the LP, the internal and external abdominal oblique muscles are not. 17. E is correct. A, C, and D are all correct answers. The ascending lumbar veins are located in the lumbar paravertebral region. The ascending lumbar arteries are located in the lumbar paravertebral region. The dorsal branch of the lumbar artery is located in the lumbar paravertebral region and closely related to the transverse process and posterior aspect of the psoas muscle.

22/04/19 2:04 PM

254

PART 4

Ultrasound Imaging of Neuraxial and Perivertebral Space

B is incorrect. The artery of Adamkiewicz, also known as arteria radicularis magna, connects the anterior spinal artery to the posterior intercostal artery, is usually located in the thoracic region, and is centrally located as opposed to paravertebrally. F is incorrect. This answer is incorrect as the artery of Adamkiewicz is both centrally located and typically in the thoracic region. 18. A is correct. This is the correct description of the LP’s appearance in elderly patients. B is incorrect. The LP appears whiter and brighter, not blacker and less bright in elderly patients. C is incorrect. There is a loss of contrast between the muscle and adjoining structures, making the LP more difficult to delineate in elderly patients. D is incorrect. This sentence is the complete opposite of the correct sentence, answer A. See above answer explanations for details. 19. F is correct. A malfunctioning nerve stimulator (A), defective needle (B), and insertion site at L1–L2 (C) are all possible causes of a failure to elicit a motor response during LPB. Malfunction of a nerve stimulator can be a reason for a failure to elicit a motor response. Some causes include faulty wiring, low battery, desiccated skin electrode, and incorrect pulse duration and amplitude settings. Manufacturing defects including faulty wiring may be a cause of failure to elicit a motor response. Inserting the stimulating needle in the upper lumbar region can result in stimulation of predominantly sensory nerves that do not elicit a motor response. D is incorrect. Inserting the stimulating needle in the middle lumbar region should result in a motor response assuming a correctly functioning nerve stimulator and needle.

Hadzic_Ch38_p249-254.indd 254

E is incorrect. This answer is incorrect because answer D should result in an elicited motor response. 20. D is correct. This describes the paramedian transverse oblique scan (PMTOS) technique. A is incorrect. This describes the trident sign technique. B is incorrect. This describes the transverse scan technique. C is incorrect. This describes the transverse flank or “shamrock sign” technique.

Suggested Readings Hadzic A. Lumbar paravertebral sonography and considerations for ultrasound-guided lumbar plexus block. In: Karmakar MK, ed. Textbook of Regional Anesthesia and Acute Pain Management. 2nd ed. New York, NY: McGraw-Hill Publishing; 2017:chap 39. Karmakar MK, Li JW, Kwok WH, Soh E, Hadzic A. Sonoanatomy relevant for lumbar plexus block in volunteers correlated with crosssectional anatomic and magnetic resonance images. Reg Anesth Pain Med. 2013;38:391-397. Karmakar MK, Li JW, Kwok WH, Hadzic A. Ultrasound-guided lumbar plexus block using a transverse scan through the lumbar intertransverse space: a prospective case series. Reg Anesth Pain Med. 2015;40:75-81. Kirchmair L, Entner T, Kapral S, Mitterschiffthaler G. Ultrasound guidance for the psoas compartment block: an imaging study. Anesth Analg. 2002;94:706-10; table of contents. Simons MJ, Amin NH, Cushner FD, Scuderi GR. Characterization of the neural anatomy in the hip joint to optimize periarticular regional anesthesia in total hip arthroplasty. J Surg Orthop Adv. 2015;24:221-4.

22/04/19 2:04 PM

39 Spinal Sonography and Applications of Ultrasound for Central Neuraxial Blocks Ki Jinn Chin and Pascal A. Ramsodit

QUESTIONS DIRECTIONS: Choose the one best response to each question. 1. What is the best choice for ultrasound (US) transducer and frequency setting in ultrasound imaging of the spine for central neuraxial blocks (CNBs) in the adult population? A. High-frequency linear-array transducer B. Low-frequency linear-array transducer C. High-frequency curved-array (convex) transducer D. Low-frequency curved-array (convex) transducer 2. Which two scan planes (axis) complement each other to obtain anatomical information during an ultrasound (US) examination of the spine with the patient in the sitting, lateral decubitus, or prone position? A. Median plane and transverse plane B. Coronal plane and sagittal plane C. Transverse plane and sagittal plane D. Coronal plane and transverse plane 3. Three different paramedian sagittal views of the spine can be obtained. What is the sequence in which these views are obtained when shifting the probe from medial to lateral? A. Paramedian sagittal transverse process view – paramedian sagittal articular process view – paramedian sagittal lamina view B. Paramedian sagittal lamina view – paramedian sagittal articular process view – paramedian sagittal transverse process view C. Paramedian sagittal articular process view – paramedian sagittal transverse process view – paramedian sagittal lamina view D. Paramedian sagittal lamina view – paramedian sagittal transverse process view – paramedian sagittal articular process view

4. Ultrasound (US) visibility of neuraxial structures can be further improved when the spine is imaged in the paramedian sagittal oblique plane. During a paramedian sagittal oblique scan (PMSOS), the transducer is positioned 2–3 cm lateral to the midline (paramedian) and over the laminae in the sagittal axis. It is tilted slightly. In what direction is it tilted and why? A. The transducer is tilted lateral to ensure the US signal enters the spinal canal through the widest part of the interlaminar space. B. The transducer is tilted medial to ensure the US signal enters the spinal canal through the narrowest part of the interlaminar space. C. The transducer is tilted medial to ensure the US signal enters the spinal canal through the widest part of the interlaminar space. D. The transducer is tilted lateral to ensure the US signal enters the spinal canal through the narrowest part of the interlaminar space. 5. The vertebral canal cannot be imaged if the ultrasound (US) transducer is positioned in a transverse orientation over the spinous process (transverse spinous process view). However what potentially useful information can be obtained from this view? A. It is useful for identifying the midline. B. It is useful for identifying the interlaminar space. C. It is useful for identifying the level of the lumbar/ thoracic spine. D. Is is useful for identifying the depth of the spinal canal. 6. What deformity of the vertebral column should be suspected if the laminae and the articular processes on either side are not symmetrically located on a transverse interspinous view? A. Scoliosis B. Kyphosis C. Lordosis D. Flatback syndrome

255

Hadzic_Ch39_p255-260.indd 255

22/04/19 2:06 PM

256

PART 4

Ultrasound Imaging of Neuraxial and Perivertebral Space

7. On a transverse sonogram of the sacrum at the level of the sacral hiatus, the sacral cornua are seen as two hyperechoic reversed U-shaped structures, one on either side of the midline. Connecting the two sacral cornua, and deep to the skin and subcutaneous tissue, is a hyperechoic band. What does this hyperechoic band represent? A. The posterior surface of the sacrum B. The sacrococcygeal ligament C. The caudal epidural space D. The ligamentum flavum 8. Which intervertebral level usually offers the largest interlaminar space? A. L2–L3 B. L3–L4 C. L4–L5 D. L5–S1 9. What do you have to take into account when estimating actual needle insertion depth from measurement of the depth with ultrasound (US)? A. The US-measured depth usually overestimates actual needle insertion depth. B. The US depth is not important. C. The US-measured depth usually underestimates actual needle insertion depth. D. Scanning movements of the US transducer may shift the skin relative to the spine, making surface markings inaccurate. 10. What was the percentage reduction in the risk of a traumatic procedure associated with the use of ultrasound (US) as reported in the 2013 meta-analysis by Shaikh et al? A. 43% B. 53% C. 63% D. 73% 11. In addition to reducing the technical difficulty of central neuraxial block (CNB) (as measured by needle insertion attempts and passes), using ultrasound (US) imaging to guide CNB offers what additional advantage that has been reported in the literature? A. US reduces the risk of serious complications. B. US increases the clinical efficacy of labor epidural analgesia. C. US has no other beneficial effect. D. US decreases procedural pain.

Hadzic_Ch39_p255-260.indd 256

12. What is the positive predictive value for absence of technical difficulty if both posterior and anterior complexes are visible ? A. 10% B. 75% C. 50% D. 85% 13. In the paramedian sagittal oblique sonogram of the lumbar spine, the posterior complex and anterior complex are seen. What structure does the anechoic space in between represent? A. Thecal sac B. Cauda equina C. Intervertebral disc D. Ligamentum flavum

ANSWERS AND EXPLANATIONS 1. D is correct. US imaging of the spine typically requires the use of low-frequency US (2–5 MHz) and curved-array (convex) transducers. Low-frequency US is required for adequate tissue penetration at the depth (5–7 cm) at which the neuraxial structures are located in adults. In addition to offering lower frequencies and better penetration, curved-array transducers are also preferred over lineararray transducers as they offer a wider field of view, which aids recognition of sonoanatomical landmarks. It may be possible to use a linear-array transducer in children, or in very thin adults where the depth to the vertebral canal is less than 5 cm. 2. C is correct. The anatomical information obtained from the transverse scan and the sagittal scan (longitudinal axis) complement each other during an US examination of the spine. The paramedian sagittal scan (see Figure 39–1) allows identification of vertebral levels, location of paramedian interlaminar spaces, and estimation of depth to the epidural and intrathecal space. The transverse scan can be performed over the spinous process (see Figure 39–2A) or through the interspinous/ interlaminar space (see Figure 39–2B). The former produces the transverse spinous process view, which clearly indicates the position of the neuraxial midline, whereas the latter produces the transverse interspinous view, which precisely identifies the interlaminar window through which the needle may be advanced, the depth to the epidural space, and canal.

22/04/19 2:06 PM

CHAPTER 39

A. Paramedian sagittal scan - lamina

Spinal Sonography and Applications of Ultrasound for Central Neuraxial Blocks

B. Paramedian sagittal scan - articular process

257

C. Paramedian sagittal scan - transverse process

FIGURE 39–1  Axis of scan: paramedian sagittal scan (A) at the level of the lamina; (B) at the level of the articular process; and (C) at the level of the transverse process.

A. Transverse scan - spinous process

B. Transverse scan - interspinous space

FIGURE 39–2  Axis of scan: transverse scan (A) at the level of the spinous process; and (B) at the level of the interspinous space.

Hadzic_Ch39_p255-260.indd 257

22/04/19 2:06 PM

258

PART 4

Ultrasound Imaging of Neuraxial and Perivertebral Space

3. B is correct. The different parts of the vertebra that can be imaged starting in the midline and moving laterally are: the lamina, the articular processes, and the transverse processes. 4. C is correct. The purpose of the medial tilt is to ensure that the US signal enters the spinal canal through the widest part of the interlaminar space and not the lateral sulcus of the spinal canal. 5. A is correct. The transverse spinous process view is not suitable for imaging the neuraxial structures but can be useful for identifying the midline when the spinous

Sacral cornua

process cannot be palpated (eg, in obese patients). It may also be used to estimate the depth of the vertebral canal by adding 3–4 cm (the average height of an adult lumbar spinous process) to the depth of the tip of the spinous process. 6. A is correct. If there is asymmetry, a rotational scoliotic deformity of the vertebral column should be suspected and the needle trajectory altered accordingly. 7. B is correct. See Figure 39–3.

Sacrococcygeal ligament

B

Sacral hiatus C Sacrum A

D

FIGURE 39–3  Transverse sonogram of the sacrum at the level of the sacral hiatus. Note the two sacral cornua and the hyperechoic sacrococcygeal ligament that extends between the two sacral cornua. (A) The hypoechoic space between the sacrococcygeal ligament and the posterior surface of the sacrum is the sacral hiatus. The image in (B) shows the sacral cornua from the water-based spine phantom; the image in (C) shows a three-dimensional (3D) reconstructed image of the sacrum at the level of the sacral hiatus from a 3D CT dataset from the author’s archive; and the image in (D) shows a transverse CT slice of the sacrum at the level of the sacral cornua.

8. D is correct. The lumbosacral junction (L5–S1 gap) is the largest interlaminar space and must not be overlooked in patients with difficult spinal anatomy, as it may be the only accessible site for central neuraxial blocks. 9. C is correct. The difference between US-measured depth and needle insertion depth in most trials is quite small (approximately 0.5 cm or less), with US usually underestimating needle depth. This difference is commonly attributed to soft-tissue compression by the US transducer during the scan. Skin movement is not relevant to depth measurement but can significantly affect the accuracy of markings used to guide needle insertion. 10. D is correct. The 2013 meta-analysis by Shaikh et al1 found a 73% reduction in the risk of traumatic procedure with the use of US compared to non–US-guided procedures.

Hadzic_Ch39_p255-260.indd 258

11. B is correct. US may also increase the efficacy of labor epidural analgesia. In two separate randomized controlled trials by Grau et al,2,3 there was a significant reduction in the rate of incomplete analgesia (2% vs. 8%) in one study3 and epidural failure (0% vs. 5.6%) in the other.3 In addition, a small but statistically significant decrease in postblock pain scores was noted in the US-assisted groups compared to the surface landmark-guided groups. These findings may be partially explained by observed reductions in the incidences of asymmetric and patchy blocks. It is notable that the more recent study by Vallejo et al,4 involving multiple operators, observed a similarly impressive reduction in the epidural failure rate in the US-assisted group. 12. D is correct. This is based on a study by Chin et al.5 The ability to see both anterior and posterior complexes signifies the presence of a good soft-tissue window into the

22/04/19 2:06 PM

CHAPTER 39

Spinal Sonography and Applications of Ultrasound for Central Neuraxial Blocks

13. A is correct. The thecal sac with the cerebrospinal fluid is the anechoic space anterior to the posterior dura. The cauda equina, which is located within the thecal sac, is often seen as multiple horizontal, hyperechoic shadows within the anechoic thecal sac. Pulsations of the cauda equina are identified in some patients. See Figure 39–4.

vertebral canal, which will permit passage of the spinal or epidural needle. Difficulty with locating the intrathecal or epidural space can nevertheless be encountered if (1) the location of the midline and interlaminar space are inaccurately marked on the skin, or (2) needle insertion is handled poorly, leading to deviation from the intended trajectory.

ILS

259

Lamina ESM LF

ES PD

L3

L5

L4

ITS CE

Posterior

AC

IVD

Cranial

Caudal

Anterior

FIGURE 39–4  Paramedian sagittal oblique sonogram of the lumbar spine at the level of the lamina showing the L3–4 and L4–5 interlaminar spaces. Note the hypoechoic epidural space (a few millimeters wide) between the hyperechoic ligamentum flavum and the posterior dura. The intrathecal space is the anechoic space between the posterior dura and the anterior complex. The cauda equina nerve fibers are also seen as hyperechoic longitudinal structures within the thecal sac. The hyperechoic reflections seen in front of the anterior complex are from the intervertebral disc (IVD). The inset image shows a matching computed tomography (CT) scan of the lumbosacral spine in the same anatomical plane as the US scan. The CT slice was reconstructed from a three-dimensional CT dataset from the author’s archive. AC, anterior complex; CE, cauda equina; ES, epidural space; ESM, erector spinae muscle; ILS, interlaminar space; ITS, intrathecal space; IVD, intervertebral disc; L3, lamina of L3 vertebra; L4, lamina of L4 vertebra; L5, lamina of L5 vertebra; LF, ligamentum flavum; PD, posterior dura.

References 1. Shaikh F, Brzezinski J, Alexander S, et al. Ultrasound imaging for lumbar punctures and epidural catheterisations: systematic review and meta-analysis. BMJ. 2013;346:f1720. 2. Grau T, Leipold RW, Conradi R, Martin E. Ultrasound control for presumed difficult epidural puncture. Acta Anaesthesiol Scand. 2001;45:766-771. 3. Grau T, Leipold RW, Conradi R, Martin E, Motsch J. Efficacy of ultrasound imaging in obstetric epidural anesthesia. J Clin Anesth. 2002;14:169-175.

5. Chin KJ, Ramlogan R, Arzola C, Singh M, Chan V. The utility of ultrasound imaging in predicting ease of performance of spinal anesthesia in an orthopedic patient population. Reg Anesth Pain Med. 2013;38:34-38.

Suggested Reading Hadzic A. Spinal sonography and applications of ultrasound for central neuraxial blocks. In: Karmakar MK, Chin KJ, eds. Textbook of Regional Anesthesia and Acute Pain Management. 2nd ed. New York, NY: McGraw-Hill Publishing; 2017:chap 40.

4. Vallejo MC, Phelps AL, Singh S, Orebaugh SL, Sah N. Ultrasound decreases the failed labor epidural rate in resident trainees. Int J Obstet Anesth. Oct 2010;19(4):373-8.

Hadzic_Ch39_p255-260.indd 259

22/04/19 2:06 PM

This page intentionally left blank

Hadzic_FM_p00i-pxx.indd 6

22/04/19 6:40 PM

PART 5 Obstetric Anesthesia

Chapter 40

Hadzic_Ch40_p261-270.indd 261

Obstetric Regional Anesthesia  263

18/04/19 6:19 PM

This page intentionally left blank

Hadzic_FM_p00i-pxx.indd 6

22/04/19 6:40 PM

40 Obstetric Regional Anesthesia Jason Choi, Justin Morello, and Liane Germond

QUESTIONS DIRECTIONS: Choose the one best response to each question. 1. All of the following are normal physiologic changes of pregnancy except: A. Minute ventilation increased by 50% B. Total blood volume increased by 40% C. Functional residual capacity (FRC) decreased by 30% D. Tidal volume decreased by 25% 2. Which statement is true regarding anesthetic requirements in the parturient patient? A. Lower doses of local anesthetics are required due to estrogen-mediated increased neural sensitivity. B. The minimum alveolar concentration (MAC) for inhalational agents is increased. C. Epidural venous engorgement has no effect on subarachnoid spread of local anesthetics in parturients. D. Overall, lower doses of local anesthetics are needed per dermatomal segment in pregnant patients. 3. Which statement is true regarding placental transfer of local anesthetics to the fetus? A. The nonionized forms of local anesthetics are better able to cross the placenta into the fetus. B. Acidosis in the mother increases ion trapping of local anesthetic in the fetus. C. Due to lack of systemic uptake, intrathecal administration of local anesthetics results in negligible levels in the mother and fetus. D. High protein binding of bupivacaine prevents transfer to and accumulation of bupivacaine in the fetus. 4. Pain during the second stage of labor: A. Is due to cervical dilation and uterine distention B. Is innervated by C fibers C. Is transmitted by the pudendal nerve D. Involves lower thoracic and lumbar dermatomes

5. Neuraxial anesthesia is contraindicated in the parturient for all of the following reasons except: A. Severe coagulopathy B. Infection at the lower lumbar area C. Chorioamnionitis D. Severe hypovolemia 6. Paracervical blocks: A. Are a common form of labor analgesia in the United States B. Block pain during the second stage of labor C. Are associated with high incidence of fetal bradycardia D. Are more difficult to place than paravertebral lumbar sympathetic blocks 7. Epidural analgesia during labor has been definitively shown to: A. Prolong the first stage of labor B. Prolong the second stage of labor C. Increase the risk of needing cesarean delivery D. Be more effective using higher concentrations of local anesthetic 8. After routine placement of an epidural catheter, a test dose of 3 mL lidocaine 1.5% with epinephrine 1:200,000 is administered. Which of the following does not rule out intravascular or intrathecal epidural placement? A. Negative aspiration of heme or clear fluid B. No change in heart rate and blood pressure above baseline 30 to 60 seconds after test-dose administration C. No sensory or motor deficit of lower extremities within 3 minutes after administration D. Patient complaining of ringing of the ear and facial numbness 9. Which is an advantage of epidural analgesia over combined spinal analgesia for labor? A. Less motor block B. Decreased spinal headache risk C. Faster onset D. Less pruritus

263

Hadzic_Ch40_p261-270.indd 263

18/04/19 6:19 PM

264

PART 5

Obstetric Anesthesia

10. A 30-year-old parturient for elective repeat cesarean delivery receives a spinal anesthesia. Hyperbaric bupivacaine 15 mg are injected intrathecally. Which of the following is most concerning for possible high spinal anesthesia? A. Complaint of difficulty breathing, despite pulse oximetry reading 100% on 2 L per minute oxygen by nasal cannula B. Difficulty phonating and speaking C. Complaint of numbness and tingling of hands but is able to squeeze hands D. Hypotension responsive to vasopressor 11. A healthy normal parturient labors for several hours with lumbar epidural analgesia but with continued complaints of labor pain. She continues to complain of pain despite bupivacaine boluses via an epidural catheter, loses consciousness, and finally has convulsions and cardiac arrest. All of the following steps should be taken except: A. Administer benzodiazepines B. Administer 20% lipid emulsion bolus of 1.5 mL/kg IV C. Deliver baby within 5 minutes D. Administer cardiac resuscitation and 1 mg dose of epinephrine 12. The following steps should be considered to treat hypotension in a preeclamptic patient after epidural anesthesia, except: A. Increase the dose of vasopressors B. Administer additional fluid boluses C. Ensure left uterine displacement D. Avoid repeated administration of amide local anesthetics 13. While intrathecal opioids are effective at reducing visceral pain (ie, exteriorization of the uterus and abdominal viscera traction), the following are complications and side effects, except: A. Respiratory depression B. Pruritus C. Hypotension D. Urinary retention 14. What dermatome levels must be covered for adequate analgesia of the laboring patient? A. T10–L1 and S2–S4 B. T8–L1 and S1–S4 C. T4–L2 and S1–S4 D. T8–L2 and S2–S4 15. A 32-year-old G3P2 woman presents for her third elective cesarean delivery. During her second pregnancy she failed a vaginal birth after cesarean (VBAC) and had to be urgently brought to the operating room due to significant fetal heart rate decelerations. Recent ultrasound showed breech positioning. You perform spinal analgesia and note that there is poor block progression after 15 minutes.

Hadzic_Ch40_p261-270.indd 264

What is the best next step in the management of this patient? A. Consider general anesthesia B. Consider continuous spinal anesthesia C. Repeat spinal D. Wait 30 minutes 16. A 28-year-old G1P0 woman presents for an elective cesarean delivery due to breech presentation. Spinal anesthesia is performed. She begins complaining of difficulty breathing. Which maneuver is useful to rule out the possibility of high neuraxial anesthesia? A. Allen test B. Durkan test C. Phalen maneuver D. The ability of the patient to squeeze the anesthesia provider’s hand 17. A 23-year-old G1P0 woman at 24 weeks and 4 days gestation presents to the emergency department with a chief complaint of a severe headache. She is found to have systolic and diastolic blood pressure 25% above baseline, proteinuria, and oliguria. What is the next best step in management to prevent seizures in this patient? A. Bupivacaine B. Hydralazine C. Magnesium sulfate D. Phenobarbital 18. An 18-year-old pregnant woman currently in the second trimester presents to the emergency department after having fallen. X-ray imaging shows an open oblique displaced radius and ulna. Orthopedic surgery evaluated the patient and plans on urgently taking the patient to the operating room for internal fixation and repair. What type of anesthesia should be provided? A. Delay the case until after delivery B. General anesthesia C. Monitored anesthesia care (MAC) D. Axillary brachial plexus block 19. A 32-year-old pregnant woman undergoes an elective cesarean due to breech presentation. Spinal anesthesia is performed without difficulties. After the patient is laid supine, what is the next best step in her management? A. Check anesthesia level B. Give phenylephrine C. Left uterine displacement D. Provide supplemental oxygen 20. A 37-year-old pregnant woman undergoes a high-risk cesarean delivery due to twin pregnancy, breech presentation, and three prior cesarean deliveries with a history of failed trial of labor after cesarean (TOLAC). What is an advantage of neuraxial anesthesia when compared to general anesthesia? A. Decreased awareness B. Decreased risk of gastric aspiration C. Decreased severe hypotension D. Increased use of anesthetic drugs

18/04/19 6:19 PM

CHAPTER 40

21. Successful spinal anesthesia is performed for a 25-yearold patient undergoing elective cesarean delivery due to breech presentation. Hyperbaric bupivacaine was injected into the intrathecal space. Despite an adequate dermatomal level, the patient reports discomfort particularly during exteriorization of the uterus. What could have been added to the anesthetic to reduce this discomfort? A. Bicarbonate B. Epinephrine C. Lidocaine D. Preservative-free morphine 22. Epidural analgesia is requested by a 23-year-old G1P0 woman. She is morbidly obese. She has no other significant past medical history. Her pregnancy has been uneventful. During the procedure loss of resistance to saline occurs at 9 cm. The epidural catheter is threaded to 13 cm. A test dose is administered and a transient increase in heart rate and blood pressure is noted. The patient was not having a uterine contraction at the time of administration. What is the likely location of the epidural catheter? A. Adipose tissue B. Epidural space C. Intrathecal space D. Intravascular 23. Epidural analgesia is requested by an 18-year-old G1P0 woman. She has no significant past medical history. Her pregnancy has been uneventful. During the procedure, loss of resistance to saline occurred at 5 cm. The epidural catheter is threaded to 9 cm. A test dose is administered and no change in heart rate and blood pressure is observed. The patient reports suddenly that her lower extremities feel numb and she is unable to adequately ambulate them. What is the likely location of the epidural catheter? A. Adipose tissue B. Epidural space C. Intrathecal space D. Intravascular

ANSWERS AND EXPLANATIONS 1. D is not a normal physiologic change, so option D is correct. Tidal volume increases 40%. A is a normal physiologic change, so option A is incorrect. Minute ventilation increases up to 50% of normal in parturients by term. This increase is due primarily to increases in tidal volume up to 40% and a lesser increase in respiratory rate. B is a normal physiologic change, so option B is incorrect. Total blood volume increases by up to 40% by term and partially accounts for increased cardiac output along with increased HR.

Hadzic_Ch40_p261-270.indd 265

Obstetric Regional Anesthesia

265

C is a normal physiologic change, so option C is incorrect. Due to an enlarged uterus elevating the diaphragm, FRC decreases. 2. D is correct. Due to progesterone-mediated increase neural sensitivity and greater spread of local anesthetic in epidural and subarachnoid space, smaller doses of local anesthetic are needed for epidural and spinal blocks for pregnant patients. A is incorrect. Increased neural sensitivity during pregnancy is progesterone-mediated, not estrogen-mediated. Levels of both hormones increase steadily throughout pregnancy. B is incorrect. MAC is decreased in parturients starting at 8–12 weeks gestation and is thought to be progesterone-related. C is incorrect. Spread of local anesthetic in both epidural and subarachnoid spaces is enhanced due to epidural venous engorgement. 3. A is correct. Local anesthetics cross the placenta via simple diffusion. Local anesthetics are weak bases and exist as a cation or nonionized form. Highly lipid soluble drugs are better able to cross biological membranes as well as nonionized forms of drugs, which have no charge and more lipophilic than their ionized states. Being weak bases, most local anesthetics exist in greater nonionized states in physiologic pH. B is incorrect. Ion trapping is a state that results in greater accumulation of local anesthetic in the fetus than the mother. It can occur with acidosis of the fetus, which causes ionization of local anesthetics and prevents diffusion of the drug back to maternal circulation and tissue. C is incorrect. Surprisingly, intrathecal administration of local anesthetics still results in systemic uptake in the mother. A study showed similar plasma levels of lidocaine for both epidural and spinal injections. Significant levels of local anesthetic were also measured from the umbilical vein at birth after spinal anesthesia. D is incorrect. Although greater protein binding of local anesthetics has been shown to decrease transfer rate across the placenta, it has been shown that prolonged administration of bupivacaine, which is highly protein-bound, results in substantial fetal accumulation of the drug. 4. C is correct. The second stage of labor, which lasts from complete cervical dilation to delivery of baby, is due to distention of the vagina and perineum. Pain signals are transmitted via the pudendal nerve, which arises from the S2–S4 nerve roots. A is incorrect. Pain in the first stage of labor is characterized by cervical dilation and uterine distention. B is incorrect. During the first stage of labor, pain is signaled through C fibers innervating the cervix and uterus. The dermatomes involved are T11–L1. D is incorrect. Pain during the second stage of labor involves the lower sacral dermatomes, whereas pain in the first stage of labor involves the lower thoracic and lumbar dermatomes.

18/04/19 6:19 PM

266

PART 5

Obstetric Anesthesia

5. C is correct. Chorioamnionitis is not an absolute contraindication for neuraxial anesthesia. If the patient has documented sepsis from chorioamnionitis, neuraxial blockade would only be relatively contraindicated out of concern for possible risk of central nervous system infection. A is incorrect. Severe coagulopathy is an absolute contraindication due to risk of epidural hematoma or subarachnoid bleeding. This may be seen in the parturient with HELLP syndrome or if a consumptive coagulopathy develops. Often, there is some degree of thrombocytopenia in the parturient. However, the thrombocytopenia is not commonly significant enough to contraindicate neuraxial block. B is incorrect. Infection at the site of needle insertion is contraindicated due to risk of infection at epidural space or central nervous system. D is incorrect. Severe hypovolemia is a relative contraindication for neuraxial anesthesia due to sympathetic block from the neuraxial block. The neuraxial block worsens the hypovolemia by increasing venous pooling and exacerbating any hypotension. Even in a euvolemic patient, increasing preload through intravenous fluid bolus helps to blunt the hypotension often seen after neuraxial block. 6. C is correct. Due to high rates of fetal bradycardia associated with the block and bupivacaine, paracervical blocks are now rarely performed in the United States. Fetal bradycardia may be related to uterine artery constriction or increased uterine tone from placement of the block. A is incorrect. In a survey from 2001, only 2% to 3% of parturients in the United States received paracervical blocks. B is incorrect. Paracervical blocks are effective in blocking pain from the first stage of labor. D is incorrect. Paravertebral lumbar sympathetic blocks may be an alternative to neuraxial blocks. They also treat pain from the first stage of labor. However, they are not commonly performed due to technical difficulty of placement, need for bilateral placement, and risk of intravascular injection. Paracervical blocks require simple technique—a submucosal injection at the vaginal fornix. 7. B is correct. Use of epidural analgesia has been shown to prolong the second stage of labor in nulliparous patients. This effect is thought to be due to decrease in expulsive forces from the epidural block or to fetal malpositioning. A is incorrect. Overall, epidural analgesia does not prolong the first stage of labor, provided that aortocaval compression is avoided. C is incorrect. The incidence of cesarean delivery is not affected by epidural analgesia. The incidence of cesarean delivery in nulliparous women was similar when compared to rates in nulliparous women using IV PCA only. In addition, timing of epidural analgesia initiation (latent vs. active phase) had no effect on cesarean delivery rates. D is incorrect. Low concentrations of local anesthetics in combination with opioids are recommended for epidural infusion in order to minimize motor block and

Hadzic_Ch40_p261-270.indd 266

prolongation of second stage of labor. Commonly, concentrations of 0.625% to 0.125% bupivacaine, levobupivacaine, or ropivacaine are used. 8. A is correct. Negative aspiration of blood or cerebrospinal fluid through an epidural catheter cannot rule out possible catheter misplacement. For this reason, a test dose is commonly administered to detect intrathecal or intravascular placement. B is incorrect. Epinephrine 15 mcg injected intravascularly or with rapid vascular uptake should transiently increase heart rate and blood pressure. However, there is a high degree of false positives from this test due to labor pain from contractions transiently elevating heart rate and blood pressure. C is incorrect. Lidocaine 45 mg injected intrathecally will result in a detectable motor and sensory block. D is incorrect. Tinnitus, facial numbness, and perioral tingling are common signs of elevated systemic levels of local anesthetics. It is unlikely that an intravascular injection of the test dose will cause such symptoms; it is possible to see after prolonged administration of local anesthetic through infusion or high doses for cesarean delivery. 9. D is correct. Incidence of pruritus is less with epidural than intrathecal opioid injection. A is incorrect. Despite common misconceptions, combined spinal–epidural (CSE) for labor analgesia has been shown to cause less motor block than epidural-only technique. Despite a lack of motor block, there is profound and rapid-onset analgesia.1 B is incorrect. Surprisingly, the incidence of postdural puncture headache is no greater with CSE compared to epidural-only technique. Overall, pregnant women have higher risks of developing postdural puncture headache.2 C is incorrect. Meta-analysis has shown significantly faster onset of analgesia with CSE vs. epidural-only technique (2–5 minutes vs. 10–15 minutes).3 10. B is correct. Difficulty speaking or phonating indicates an inability to pass air through vocal cords and likely involves loss of diaphragm function. The phrenic nerves innervate the diaphragm and originate from C3–C5 nerve roots. With a high spinal anesthetic, rapid control of the airway is paramount and will require endotracheal intubation. A is incorrect. A complaint of difficult breathing is common after spinal anesthesia cesarean delivery due to block of abdominal and thoracic levels. An inability to sense stretch of thoracic levels often causes the patient to complain of dyspnea. Such a complaint may signal an impending high spinal anesthesia and require careful examination and observation. C is incorrect. Parasthesia of the hands occurs with anesthetization of C6–T1 levels and may signal impending high spinal anesthesia. If the patient is able to squeeze her hands, this indicates a high but incomplete block level. D is incorrect. Hypotension is common after spinal anesthesia in general and often requires fluid administration

18/04/19 6:19 PM

CHAPTER 40

and transient vasopressor support. High spinal anesthesia can result in refractory hypotension, bradycardia, asystole, and even cardiac arrest. 11. D should not be done, so option D is correct. Cardiovascular collapse in the setting of local anesthetic systemic toxicity (LAST) requires prompt initiation of advanced cardiac life support. However, a modification of epinephrine doses is required. The 2010 ASRA guidelines recommend use of low-dose (10–100 mcg bolus in adults) epinephrine and avoidance of vasopressin during ACLS treatment of cardiac arrest due to LAST.4 A should be done, so option A is incorrect. Seizures due to LAST should be treated with benzodiazepines. Propofol should be avoided, especially in the setting of cardiovascular collapse. B should be done, so option B is incorrect. Lipid emulsion administration is paramount in treating cardiac arrest from LAST, although the mechanism of efficacy is not completely understood. Initial bolus of 1.5 mL/kg over 1 minute should be followed by an infusion of 0.25 mL/kg/min. Repeated bolus doses and increasing infusion rate may be required. C should be done, so option C is incorrect. In the setting of cardiac arrest, the fetus should be delivered within 5 minutes if resuscitation attempts are unsuccessful. 12. A is correct. There is an increased sensitivity to vasopressors in preeclampsia; therefore, lower doses are usually required to correct hypotension. B is incorrect. Although preeclampsia is accompanied by exaggerated retention of water and sodium, the shift of fluid and proteins from the intravascular into the

Obstetric Regional Anesthesia

267

extravascular compartment may result in hypovolemia. Additional fluid boluses are indicated. C is incorrect. Left uterine displacement is vital for the management of hypotension in the pregnant patient to minimize aortacaval compression, and also improve placental perfusion. D is incorrect. The total maternal body clearance of amide local anesthetics is prolonged in preeclampsia, and repeated administration of these drugs can lead to higher blood concentrations than in normotensive patients. 13. C is correct. Hypotension is not a common complication related to the use of intrathecal opioids. A is incorrect. If a rostral spread of intrathecal opioids result, delayed respiratory depression could occur, usually within 30 minutes of injection. The effect is rare with fentanyl and sufentanil. B and D are incorrect. The most common side effects of intrathecal opioids are pruritus, nausea, vomiting, and urinary retention. 14. A is correct. Visceral pain is primarily experienced during the first stage of labor and is largely due to cervical dilation and uterine contractions. Pain is transmitted by the autonomic nervous system through sympathetic fibers that pass through the paracervical tracks and enter the CNS at T10–L1. During the second stage of labor, pudendal nerve fibers that innervate the birthing canal enter the CNS at S2–S4 and are responsible for transmitting somatic pain. Therefore to provide adequate analgesia to the laboring patient, one must cover the T10–L1 and S2–S4 dermatomes. See Figure 40–1. B, C, and D are incorrect. See explanation for answer A.

T10-T11

Hypogastric nerve Uterus S2-S4

Bladder Rectum Pelvic nerve Pudendal nerve Clitoris Anus Vagina

FIGURE 40–1  Pain pathways in a parturient.

Hadzic_Ch40_p261-270.indd 267

18/04/19 6:19 PM

268

PART 5

Obstetric Anesthesia

15. D is correct. The patient in the above question presents for nonemergent elective cesarean section. According to the management algorithm for an obstetric patient with inadequate neuraxial anesthesia, the best next step in the management of this patient is to wait 30 minutes. A, B, and C are incorrect. If there continues to be no block one should repeat the spinal, epidural, CSE, or consider general anesthesia. If there is a low-level block one could continue epidural anesthesia with slow incremental dosing or perform CSE with low-dose spinal (one-half original dose). Lastly, if a full block has resulted after waiting 30 minutes, one should proceed with surgery. In emergent clinical situations, one could consider general anesthesia or continuous spinal anesthesia if time allows and the patient’s condition is favorable. 16. D is correct. The ability of the patient to squeeze the provider’s hand means that the block level is below the level of the brachial plexus C6–T1. A is incorrect. The Allen test is used to assess arterial blood flow to the hands. B and C are incorrect. The Durkan test and Phalen maneuver are used to evaluate carpal tunnel syndrome. 17. C is correct. In the United States, magnesium sulfate is the first-line agent used to prevent seizures in a pregnant patient with signs and symptoms of severe preeclampsia. It is highly effective at preventing seizures. A is incorrect. Bupivacaine is a local anesthetic. If injected intravascularly, it could lead to seizures. B is incorrect. Hydralazine is used as an antihypertensive agent. Hydralazine is most commonly used as a vasodilator because it can increase uteroplacental and renal blood flow. D is incorrect. Phenobarbital is not a first-line agent to prevent seizures in the preeclamptic patient with severe features. 18. D is correct. An axillary brachial plexus block is indicated for forearm surgery. In this patient, it is the type of anesthesia that should be provided. A is incorrect. Local anesthetics have not been shown to be teratogenic. The case cannot be delayed until after delivery given this is an open displaced fracture. B is incorrect. If possible general anesthesia should be avoided in preference for regional techniques. Several major studies have shown that there is an increased incidence of fetal death when performing general anesthesia on pregnant patients. C is incorrect. The patient would not tolerate this procedure under MAC alone. 19. C is correct. Compression of the lower aorta can occur in the supine position. In addition, significant hypotension can occur in pregnant patients undergoing neuraxial anesthesia. This necessitates immediate left uterine

Hadzic_Ch40_p261-270.indd 268

displacement after placing the patient in the supine position. A is incorrect. Spinal anesthesia takes several minutes to set up; it should not be checked immediately after the patient is laid supine. B is incorrect. Phenylephrine is the first-line agent to treat hypotension in the pregnant patient undergoing cesarean delivery. D is incorrect. Pregnant patients generally do not require supplemental oxygen unless specific comorbidities necessitate this. 20. B is correct. Regional anesthesia prior to cesarean delivery has several advantages to general anesthesia. Gastric aspiration risk is reduced. A is incorrect. Regional anesthesia allows the mother to remain awake during the delivery. C is incorrect. Severe hypotension is more common in spinal anesthesia when compared to general anesthesia due to the sympathectomy that occurs. D is incorrect. Anesthetic drug use is decreased. 21. D is correct. Visceral discomfort is commonly experienced by patients undergoing cesarean delivery especially during exteriorization of the uterus and traction on abdominal viscera. This can occur despite an adequate dermatomal level. Improved analgesia can be provided by added fentanyl or preservative-free morphine to the local anesthetic solution. A and B are incorrect. Bicarbonate and epinephrine can be added to local anesthetics to decrease vascular uptake and hasten the onset time, respectively. C is incorrect. Lidocaine would not improve visceral discomfort. 22. D is correct. A test dose is administered after placement of an epidural catheter. Aspiration alone is not reliable enough for detecting catheter misplacement. In this case transient increases in heart rate and blood pressure are noted. This means that there was likely inadvertent intravascular catheter placement. It is important to make sure that the patient is not having uterine contractions when the test dose is administered, because a false positive result can occur. A is incorrect. There would be no increase in heart rate or blood pressure if the catheter was threaded into adipose tissue. B is incorrect. There would be no increase in heart rate or blood pressure if the catheter was threaded into the epidural space. C is incorrect. There would be no increase in heart rate or blood pressure if the catheter was threaded into the intrathecal space, rather, the patient’s legs would become heavy. 23. C is correct. A test dose is administered after placement of an epidural catheter. A small dose of local anesthetic is added to a test dose. In this case a sensory and motor

18/04/19 6:19 PM

CHAPTER 40

block is quickly observed indicating likely intrathecal catheter placement. A is incorrect. Placement of the catheter in the adipose tissue would not generate symptoms. B is incorrect. An epidural placement of the catheter would not generate symptoms after administering the test dose. D is incorrect. In the absence of uterine contraction, increased heart rate and blood pressure indicate an intravascular placement of the catheter.

References 1. Campbell DC, Camann WR, Datta S. The addition of bupivacaine to intrathecal sufentanil for labor analgesia. Anesth Analg. 1995;81:305.

Hadzic_Ch40_p261-270.indd 269

Obstetric Regional Anesthesia

269

2. Norris MC, Grieco WM, Borkowski M, et al. Complications of labor analgesia: epidural versus combined spinal epidural techniques. Anesth Analg. 1995;79:529. 3. Simons SW, Cyna AM, Dennis AT, Hughes D. Combined spinalepidural versus epidural analgesia in labour. Cochrane Database Syst Rev. 2007;3:CD003401. 4. Neal JN, Bernards CM, Butterworth JF, et al. ASRA Practice Advisory on local anesthetic systemic toxicity. Reg Anesth Pain Med. 2010;35:152-161.

Suggested Reading Hadzic A. Obstetric regional anesthesia. In: Choi J, Germond L, Santos AC, eds. Textbook of Regional Anesthesia and Acute Pain Management. 2nd ed. New York, NY: McGraw-Hill Publishing; 2017:chap 41.

18/04/19 6:19 PM

This page intentionally left blank

Hadzic_FM_p00i-pxx.indd 6

22/04/19 6:40 PM

PART 6 Pediatric Anesthesia

Chapter 41

Regional Anesthesia in Pediatric Patients: General Considerations  273

Chapter 42

Pediatric Epidural and Spinal Anesthesia and Analgesia  277

Chapter 43

Peripheral Nerve Blocks for Children  283

Chapter 44

Acute and Chronic Pain Management in Children  285

Hadzic_Ch41_p271-276.indd 271

18/04/19 6:25 PM

This page intentionally left blank

Hadzic_FM_p00i-pxx.indd 6

22/04/19 6:40 PM

41 Regional Anesthesia in Pediatric Patients: General Considerations Steve Roberts and Will Gauntlett

QUESTIONS DIRECTIONS: Choose the one best response to each question. 1. The benefits of regional anesthesia to pediatric surgical patients: A. Cause an increased length of inpatient stay B. Include improved postoperative gastric function C. Are outweighed by the need for more intensive postoperative nursing observations D. Are limited to older children 2. When managing a neonatal epidural, which of the following increases safety? A. A maximum of 3 days infusion to prevent local anesthetic (LA) accumulation B. Operator experience is not important. C. The decreased risk of toxicity with levobupivacaine D. Siting the epidural while the patient is awake 3. An understanding of the difference in drug handling in the pediatric population is important when considering which local anesthetic (LA) and what dose are required for regional techniques. Which of the following is correct? A. Neonates and infants show reduced ability to clear LAs from their systems. B. The volume of distribution in an infant is significantly lower than in an adult. C. Metabolism of amide LAs is increased in the neonatal system. D. Reduced amounts of plasma proteins, including albumin, result in a lower proportion of unbound LA. 4. During inguinal hernia repair for a 10 kg infant, the heart rate increases and you are concerned that your caudal block is not providing adequate analgesia. You used 10 mL of 0.25% levobupivacaine to perform the block and are considering asking the surgeon to

infiltrate more local anesthesia (LA) locally. Which of the following statements is true about LA toxicity? A. The safe dose of levobupivacaine is 3 mg/kg if used with dilute adrenaline. B. Peripheral nerve blocks are not associated with LA toxicity. C. Further LA could safely be given if 2 or more hours have passed since the block was administered. D. General anesthesia may mask the signs of LA toxicity. 5. The effects of local anesthetic (LA) toxicity on pediatric patients include: A. Increased firing of the sinoatrial node leading to dysrhythmias B. Calcium efflux leading to reduced cardiac contraction C. Seizures that cannot be managed by simple benzodiazepines D. Vasodilatation in higher concentration 6. The management of suspected local anesthetic (LA) toxicity in children includes the following: A. Initial bolus of 15 mg/kg of lipid emulsion B. Up to two repeated lipid emulsion boluses of 1.5 mg/kg C. Prolonged CPR at 30:2 compressions to ventilation if cardiovascular collapse occurs D. Commencement of lipid emulsion infusion at 15 mL/kg/hr if cardiovascular stability has not been achieved 7. You have chosen to use topical local anesthetic (LA) while performing a lumbar puncture for a 9-year-old girl. She is very frightened about the needle being painful. When considering topical LA: A. LMX-4 has the shortest onset and offset time of the three most common topical LAs. B. Eutectic mixture of local and anesthetics (EMLA) is a mixture of amethocaine and prilocaine. C. EMLA should be applied 30 minutes before the procedure starts. D. Ametop can cause irritation and erythema on the skin.

273

Hadzic_Ch41_p271-276.indd 273

18/04/19 6:25 PM

274

PART 6

Pediatric Anesthesia

8. You are anesthetizing for an orthopedic list and begin your preassessment of the patients. When considering regional techniques, which of the following is true? A. Regional techniques should not be undertaken in children with established chronic pain conditions. B. Regional blocks should be performed as proximally as possible to provide the best analgesic cover and minimize risk of nerve damage. C. Conversations around regional blocks are best had with the parents alone as they may distress the child. D. Clinical examination is an important part of preassessment for regional technique. 9. For safe postoperative care of patients with regional blocks, which of the following is true? A. Children who have had regional techniques are not suitable for day case surgery. B. For “single shot” regional blocks there are no special nursing considerations. C. Even when pain-free, the motor weakness and reduced sensation of successful regional techniques can be anxiety-provoking for patients and parents postoperatively. D. If a child with a nerve catheter describes pain postoperative, the priority should be to examine the catheter and pump. 10. When considering training and development of regional anesthetic skills: A. Training is essential for safe and effective use of regional anesthetics. B. Regional techniques are best learned on patients in situ. C. Excellent ultrasound (US) technique negates the need for in-depth anatomical knowledge. D. Anesthetic departments should focus training around the most challenging regional techniques.

ANSWERS AND EXPLANATIONS 1. B is correct. The use of regional techniques in children is associated with preserved peristalsis and improved splanchnic perfusion. This is especially important for intestinal pathology such as necrotizing enterocolitis (NEC) and gastroschisis. Improved gastric function may also result in early return to enteral feeding. A is incorrect. The use of regional anesthesia in pediatric practice is associated with a reduced length of hospital stay. This is likely due to a combination of improved analgesia, and a faster return to normal feeding and mobility. C is incorrect. The superior analgesia provided by working regional analgesia makes postoperative management of pediatric patients less challenging. Benefits include reduced need for supplemental analgesia, parental support, and also less exposure to medication that might induce delirium or dependency. D is incorrect. Whilst neonatal physiology demands close attention to calculations of safe local anesthetic doses,

Hadzic_Ch41_p271-276.indd 274

regional anesthesia can be safely and effectively used in children of all ages. 2. C is correct. While all LAs require care in administration, levobupivacaine has a less cardiotoxic profile than the other amide LAs. Animal models show reduced myocardial dysfunction and incidence of dysrhythmia with levobupivacaine compared to bupivacaine. A is incorrect. The maximum recommended duration is 48 hours for a neonatal epidural due to the risk of LA accumulation secondary to neonatal physiology. B is incorrect. Most complications occur with inexperienced operators; this procedure should be performed only by an anesthetist who is experienced in pediatric regional techniques, supported by departmental guidelines and training. D is incorrect. While this remains the approach for most adult epidural insertions, this does not apply to pediatric practice. Insertion under general anesthesia or deep sedation facilitates a safer insertion as it guarantees patient cooperation and minimizes movement (recommended by ESRA and ASRA). 3. A is correct. Renal maturation can take up to 8 years. This results in reduced clearance of LAs from young children. This coupled with changes in drug metabolism and extracellular volumes can make estimating drug concentrations and duration of action challenging. B is incorrect. The volume of distribution is greater in neonates and infants as their percentage of body water is greater. This is most pronounced in the premature neonate. C is incorrect. Hepatic metabolism of amide LAs is impaired due to the immaturity of the cytochrome system in neonates and infants. D is incorrect. Reduced numbers of alpha-1-acid glycoproteins and albumin result in an increased amount of unbound, free drug in neonates. 4. D is correct. General anesthesia may mask the early signs of LA toxicity. Central nervous system signs may not be identifiable due to hypnosis while myocardial depression may initially be confused with the depressant effects of anesthetic agents. A is incorrect. The safe maximum dose is 2.5 mg/kg for levobupivacaine. B is incorrect. LA toxicity is a risk for all regional techniques regardless of site of administration. C is incorrect. Caution should be given to repeated doses in the first 48 hours after a block. There are several physiological variations in pediatric populations, including drug handling and percentage of active drug that may maintain plasma concentrations of LA for many hours. 5. D is correct. During LA toxicity cardiovascular collapse can be exacerbated by reduced vascular tone secondary to high LA concentrations.

18/04/19 6:25 PM

CHAPTER 41

A is incorrect. Sinoatrial activity is reduced during LA toxicity and it is this that can result in malignant rhythms. B is incorrect. Calcium influx results in reduced strength of myocardial muscle contraction and cardiac contractility. C is incorrect. CNS effects of LA toxicity are varied and can include generalized seizures as a result of sodium channel blockade and interruption of normal neuronal communication. These can be managed as with other seizures using benzodiazepines as a first line. Use of general anesthesia to terminate refractory seizures may be needed if initial management is unsuccessful. 6. B is correct. A maximum of two further lipid emulsion boluses can be given after the initial bolus if there is still cardiovascular instability or deterioration. These should all be of 1.5 mg/kg and at least 5 minutes should be left between each administration. A is incorrect. Initial bolus of lipid emulsion should be 1.5 mg/kg i.v. C is incorrect. Prolonged CPR may be needed in LA toxicity but in this population the Resuscitation Council recommends 15:2 compressions to breaths. D is incorrect. An infusion of 15 mL/kg/hr of lipid emulsion should be started after the initial bolus regardless of cardiovascular stability. This infusion may be increased to 30 mL/kg/hr after 5 minutes if cardiovascular stability has not been achieved or if there is deterioration. 7. D is correct. Ametop can cause local skin erythema. This can negate the advantages for cannulation brought about by Ametop’s vasodilatational properties. A is incorrect. LMX-4 (4% liposomal lidocaine solution) has the shortest onset time of the three at 30 minutes but can be left on for 3–4 hours. This makes it a good choice for emergency procedures where rapid onset is needed but exact timing of procedures can often be uncertain. B is incorrect. EMLA is a mix of lidocaine and prilocaine. C is incorrect. EMLA must be administered at least 1 hour before the procedure. 8. D is correct. Careful assessment of the patient’s clinical history and any behavioral challenges is important in planning if and how to proceed with a block. Examination of the proposed site of block may identify possible contraindications to a block or difficulties in insertion. Children with neurological deficits should have them recorded prior to any regional technique. A is incorrect. Regional techniques, including nerve catheters, may be excellent choices for postoperative analgesia in chronic pain patients. Indeed, they may make postoperative management significantly less challenging.

Hadzic_Ch41_p271-276.indd 275

Regional Anesthesia in Pediatric Patients: General Considerations

275

B is incorrect. Regional blocks should be performed as peripherally as possible while still maintaining effectiveness. This should ensure a safer block with fewer side effects. C is incorrect. Conversations should be had with parents and the child whenever appropriate. Forewarning the child around paresthesia and heaviness in the blocked limb can prevent anxiety in the recovery period. Furthermore, it may be appropriate to engage the child in giving consent for the procedure if the child shows understanding. 9. C is correct. Parents and, where possible, children, should be counseled preoperatively as to what to expect from their blocks. Heavy limbs that the child cannot feel can cause worry and upset. A is incorrect. Children who are ambulatory after regional techniques can be discharged on the same day of surgery if they meet institutional day case criteria. B is incorrect. All patients with regional blocks should be cared for on designated wards where staff have the appropriate training and skill mix to safely care for them. D is incorrect. The first priority should be to alleviate pain in the child. Interrogation of the nerve catheter should come after this. For orthopedic surgery also exclude compartment syndrome. 10. A is correct. Practice and refinement of regional skills are vital in providing the most effective and safest outcomes for patients. All practitioners should ensure that they maintain and develop their regional skills as part of their clinical development. B is incorrect. The techniques needed to perform safe and effective blocks should be practiced and refined (eg, needling on phantoms and scanning on adult volunteers) before moving to patients. Initially these patients should be older children and under supervision of more experienced colleagues. C is incorrect. US provides benefits to the range of blocks possible as well as aiding efficacy. However, it does not replace the need for the operator to understand the anatomy associated with the blocks performed. D is incorrect. Departments should provide regular, structured training on the common regional techniques encountered in pediatric practice. They should ensure that members of the department are able to provide safe and effective care around these procedures.

Suggested Reading Hadzic A. Regional anesthesia in pediatric patients: general considerations. In: Roberts S, ed. Textbook of Regional Anesthesia and Acute Pain Management. 2nd ed. New York, NY: McGraw-Hill Publishing; 2017:chap 42.

18/04/19 6:25 PM

This page intentionally left blank

Hadzic_FM_p00i-pxx.indd 6

22/04/19 6:40 PM

42 Pediatric Epidural and Spinal Anesthesia and Analgesia Belen De Jose Maria and Luc Tielens

QUESTIONS DIRECTIONS: Choose the one best response to each question. 1. Anatomical differences in children compared with adults that should be considered when using neuraxial anesthesia are: A. The conus medullaris is located higher in the spinal column. B. The sacral hiatus is located more cephalad than in older children. C. Below 1 year of age, the dural sac is more cephalad. D. The epidural fat is densely packed in small children. 2. Which of the following statements is true regarding caudal local anesthetics (LAs) in children? A. The optimum concentration of bupivacaine is 0.125%–0.175%. B. Patient height provides a better correlation than body weight in predicting spread of LA after a caudal block. C. Fentanyl 1–2 mcg/mL is the recommended adjuvant in neonates. D. A caudal single-injection bolus of 0.5 mL/kg will reach a high lumbar level. 3. Severe complications of regional anesthesia in children are rare. However: A. Cardiovascular instability must always be considered, as in adults. B. The use of epidural blood patch to treat postdural puncture headache (PDPH) is contraindicated in children. C. Neonates and very young infants are at greater risk for local anesthetic systemic toxicity (LAST) than older children. D. Caudal catheters, if placed under strict aseptic conditions, are not at a higher risk of colonization. 4. A 6-month-old boy, body weight of 5 kg, is scheduled for bilateral inguinal hernia repair. A caudal block under general anesthesia is performed with a 25-gauge

Tuohy needle at a 45-degree angle. After placement and stylet withdrawal, a cerebrospinal fluid leakage appears. What could have been done to prevent this situation? A. A 22-gauge short-bevel Tuohy needle should have been used. B. The patient should have been placed in Trendelenburg position. C. The insertion angle of the needle should have been shallower. D. Ultrasound (US) assessment of the caudal anatomy could have been done. 5. Which of the following clinical cases is not a straightforward indication for a single-shot caudal block? A. Urologic surgery lasting 6 hours B. Short orthopedic surgery on both limbs C. Bilateral hernia repair in a 4-year-old girl D. Unilateral incarcerated hernia repair 6. You plan to perform an ultrasound (US)-assisted caudal block. Which of the following approaches would you choose (SAX: short-axis, LAX: long-axis, IP: in-plane, OOP: out-of-plane)? A. SAX and LAX preprocedural assessment of caudal anatomy; SAX and IP needle insertion B. SAX and LAX preprocedural assessment of caudal anatomy; LAX and IP needle insertion C. LAX and SAX preprocedural assessment of caudal anatomy; SAX and IP needle insertion D. LAX and SAX preprocedural assessment of caudal anatomy; LAX and OOP needle insertion 7. A 1-day-old girl is scheduled for a Kasai surgical repair of a biliary atresia. There is no available experienced anesthesiologist who can perform lumbar epidurals in newborns. The anesthesia team decides to use a continuous caudal catheter. Which of the following is true? A. The catheter will find a strong resistance to advancement. B. The catheter will easily be infected. C. Only a Tuohy needle should be used to place the catheter. D. The catheter’s tip localization should be followed by ultrasound (US) assistance.

277

Hadzic_Ch42_p277-282.indd 277

22/04/19 6:13 PM

278

PART 6

Pediatric Anesthesia

8. An 8-year-old boy is diagnosed with a left tibial osteosarcoma. The planned surgery will be resection of the tumor and left tibia plus a transfer of the right fibula to the left leg. Which of the following is true if a lumbar epidural block is planned? A. The epidural needle should not be placed above L4–L5. B. Ultrasound (US) assistance should be used. C. The epidural space will be best found by means of loss of resistance to air. D. The block should be a single-shot block at L1–L2. 9. Next Tuesday morning your first patient in the surgical list is a Nuss procedure to repair a pectus excavatum in a 13-year-old girl. Which of the following is true regarding an epidural block for analgesia? A. High thoracic catheter advancement from a lumbar insertion site is frequently successful. B. Epidural needle insertion in pediatric patients can be performed at any thoracic interspace using either a midline or paramedian approach; however, a paramedian approach is often preferred. C. Preprocedural US imaging is performed with the probe in three planes: transverse (SAX), median sagittal (median LAX), and paramedian oblique sagittal (paramedian oblique LAX). D. The epidural space is found by ultrasound (US) guidance instead of loss of resistance (LOR) to saline. 10. A 34-week-old premature infant, body weight of 2.1 kg, is scheduled for left hernia repair. A spinal anesthesia is planned. Which of the following is true? A. Bupivacaine 0.25% or ropivacaine 0.2% are the most common local anesthetics (LAs) used in spinal anesthesia in children. B. The usual dose of LA varies between 0.5 and 1 mg/kg. C. Using a high volume of LA per kg of weight enables a long-lasting block. D. The patient’s appropriate neck flexion is important to allow best interspinous space opening.

ANSWERS AND EXPLANATIONS 1. B is correct. In young children, the sacral hiatus is located more cephalad than in older children. The sacrum of children is also more flat and narrow compared with the adult population. At birth, the sacral plate, which is formed by five sacral vertebrae, is not completely ossified and continues to fuse until approximately 8 years of age (although it may take until 21 years of age). There is a 6% incidence of sacral atresia. The incomplete fusion of the sacral vertebral arch forms the sacral hiatus. The caudal epidural space can be accessed easily in infants and children through the sacral hiatus. Because of the continuous development of the sacral canal roof, there is considerable variation in the sacral hiatus. A is incorrect. There are significant anatomical differences in children compared with adults that should be considered when using neuraxial anesthesia. For instance, in neonates and infants, the conus medullaris is located lower in the spinal column (at approximately the L3 vertebra) compared with that in adults, in whom it is situated at approximately

Hadzic_Ch42_p277-282.indd 278

the L1 vertebra. This is a result of different rates of growth between the spinal cord and the bony vertebral column in infants. However, at approximately 1 year of age, the conus medullaris reaches the L1 level similar to that in an adult. C is incorrect. The dural sac may end more caudally: at S4 in infants younger than 1 year and at S2 in older children. Therefore, because of the increased risk of accidental dural puncture, caution is warranted when placing caudal blocks in infants. D is incorrect. The loosely packed epidural fat may facilitate the spread of local anesthetic and help achieve a quicker block onset. It may also allow the unimpeded advancement of epidural catheters from the caudal epidural space to the lumbar and thoracic levels. 2. A is correct. As a general rule, high concentrations of LAs, such as 0.5% bupivacaine or 0.5% ropivacaine, are seldom used in epidural blocks in children. Instead, larger volumes of more dilute LA are more commonly used to cover multiple dermatomes. The maximal safe dose of bupivacaine is 2.5 mg/kg. For caudal use, the optimum concentration of bupivacaine is 0.125%–0.175%. B is incorrect. Body weight is usually a better correlation than patient age and height in predicting spread of local anesthetic after a caudal block. C is incorrect. The use of opioids as an adjuvant in neonates is not recommended because of the very unpredictable action and reaction in these patients. D is incorrect. A simple rule for a caudal single injection in children under 20 kg is to use bupivacaine 0.125%– 0.175% or ropivacaine 0.2% and give a bolus of one of the following: 0.5 mL/kg to achieve a sacral surgical level; 1.0 mL/kg to achieve a high lumbar surgical level; 1.25 mL/kg to achieve a low thoracic surgical level. 3. C is correct. For continuous epidural infusion, neonates and very young infants are at greater risk for LAST than older children. A is incorrect. Significant changes in blood pressure are uncommon in pediatric patients after the accurate administration of epidural analgesia. B is incorrect. The use of epidural blood patch to treat PDPH has been used with success in adults since 1960. There are now many reports of its successful use in children as well.1,2 D is incorrect. Although studies have not found clinical evidence of higher infection rates with the caudal approach, bacterial colonization has been reported to be higher. 4. D is correct. US can be used to assess the caudal anatomy prior to the landmark-based technique or to guide needle placement. Preassessment is particularly useful in screening children with cutaneous stigmata of spinal dysraphism. A is incorrect. Short-bevel Tuohy or Crawford needles (5 cm in length) with stylets offer a better tactile sensa­tion when the sacrococcygeal ligament is punctured. For children aged 1 year or older, a 22-gauge needle is used; for children younger than 1 year of age, a 25-gauge needle may be used. The use of a styletted needle may reduce the risk

22/04/19 6:13 PM

CHAPTER 42

Pediatric Epidural and Spinal Anesthesia and Analgesia

279

of introducing a dermal plug into the caudal space,3 although an epidermal cell graft tumor in the epidural space has yet to be reported. Some authors advocate the use of a 22-gauge angiocath, suggesting that it is easier to detect intravascular placement and intraosseous placement with this needle. B is incorrect. The patient is placed in a lateral decubitus position with the neck flexed and the knees drawn up to the chest. C is incorrect. The insertion angle of the needle should not be shallower. The needle is inserted and advanced into the sacral hiatus at approximately a 70-degree angle. 5. A is correct. Single-injection caudal epidural blockade is widely used to provide perioperative analgesia in pediatric practice. As a single injection, it offers a reliable and effective block for patients undergoing urologic, general, and orthopedic surgery involving the lower abdomen and lower limbs. A single-injection caudal epidural may not be suitable for every case because it has a limited dermatomal distribution and a short duration of action. New local anesthetics and adjuvants, as well as continuous catheter approaches, may overcome these limitations.

FIGURE 42–2  US image in the short-axis view showing the sacral cornea and sacrococcygeal membrane.

B, C, and D are incorrect. All of these are straightforward indications for a single shot caudal block. 6. B is correct. The probe is positioned first in a transverse plane at the sacrum level; this is called the short-axis view (SAX). The exact positions of the cornua and the sacrococcygeal membrane (SCM) are defined (Figure 42–1). With the sacrococcygeal membrane in the middle of the image (Figure 42–2), the probe is then rotated 90 degrees into a midline sagittal position over the lower sacrum, which is called the long-axis view (LAX; Figure 42–3). The SCM and the ventral and dorsal layer of the os sacrum, with the caudal epidural space in between, are easily identified. In small children, the dural sac may be visible in this position, although in older children you need to scan in a more cephalad direction (Figure 42–4). These movements can be

FIGURE 42–3  US probe in the long-axis view.

FIGURE 42–1  US probe in the short-axis view.

Hadzic_Ch42_p277-282.indd 279

FIGURE 42–4  US image in the long-axis view showing the sacrococcygeal membrane and caudal epidural space in the os sacrum.

22/04/19 6:13 PM

280

PART 6

Pediatric Anesthesia

performed in the opposite order, but a complete exploration of the space in both axes is recommended. After this preprocedural US assessment of the caudal anatomy, the needle can be introduced into the sacral hiatus with the probe in either axis. If the probe is held in the SAX, the needle insertion will be automatically in the out-of-plane (OOP) approach (Figure 42–5). If the probe is held in the LAX, the needle insertion will then be in the in-plane (IP) approach (Figure 42–6). A, C, and D are incorrect. See explanation for B.

FIGURE 42–5  US probe in the short-axis view and out-of-plane needle insertion.

FIGURE 42–6  US probe in the long-axis view and in-plane needle insertion.

7. D is correct. It is extremely important to know where the tip of the catheter is finally located: too low a catheter tip level will result in poor analgesia; too high may cause respiratory depression. However, because catheters don’t

Hadzic_Ch42_p277-282.indd 280

travel linearly in the epidural space, measuring the length of the catheter against the patient’s back does not accurately determine the targeted surgical level. Therefore, the location of the catheter tip should be verified using an objective test (ie, radiography, nerve stimulation, electrocardiography, or, preferably, US). These techniques may be considered cumbersome or redundant, and in children above 1 year of age, the development of a lumbar curve during infancy might prevent easy cephalad advancement of the catheter. Therefore, some have suggested that caudal catheter placement should be limited to patients younger than 1 year of age. It is the opinion of the authors that, at least in children older than 1 year of age, catheters should be placed as close to the surgical dermatome as possible. Appropriate pediatric and US training is therefore recommended in all cases and mandatory prior to performing lumbar or thoracic epidurals in young children. A is incorrect. Minor resistance to the passage of the catheter can usually be overcome by simple flexion or extension of the patient’s vertebral column and/or by simultaneously injecting normal saline through the catheter. Some authors use a specialized stimulation epidural catheter (the Epidural Positioning System using the Tsui test, Arrow International Inc., Reading, PA). B is incorrect. The overall infection rate associated with caudal epidural catheters appears to be low. Compared with lumbar epidural catheters, there is some concern regarding infection with the prolonged use of caudally placed catheters owing to the proximity of the sacral hiatus to the rectum. To reduce the risk of contamination, strict aseptic techniques, catheter tunneling and fixing with occlusive dressing in a cephalad direction can be used. C is incorrect. Continuous caudal epidural anesthesia can be done with an IV catheter (an 18-gauge angiocatheter for a 20-gauge epidural catheter or a 16-gauge angiocatheter for a 19-gauge epidural catheter) or with an 18-gauge Crawford or Tuohy needle inserted through the sacrococcygeal membrane, as described for the single-injection technique. The epidural catheter is then advanced carefully from the caudal space to the target level. 8. B is correct. Today, the best available method to assess the epidural depth is US imaging. With the probe in a paramedian oblique sagittal plane, the distance from the skin to the ligamentum flavum is measured with the US machine’s caliper. This measurement provides a good estimate of the depth at which the LOR is going to be felt. Therefore, a preprocedural US assessment of each individual patient is highly recommended. A is incorrect. Children should be positioned in the lateral decubitus position for direct lumbar epidural placement. In children over 1 year of age, an 18-gauge, 5-cm Tuohy needle, marked every 0.5 cm, with a 20-gauge epidural catheter is often used. In children under 1 year of age, a 20-gauge, 5-cm Tuohy needle, marked every 0.5 cm, with a 22-gauge catheter should be considered. However, these narrower catheters get kinked, occlude, and leak more often. Although identification of the intervertebral space and ligamentum flavum in most pediatric patients is easy, the ligamentum flavum

22/04/19 6:13 PM

CHAPTER 42

Pediatric Epidural and Spinal Anesthesia and Analgesia

281

TABLE 42–1  Formulas to determine depth of epidural space from skin. Note: An individual preprocedural ultrasound assessment is the preferred method of determining the depth of epidural space from the skin. 1. Rough estimate: 1 mm/kg body weight 2. Depth (cm) = 1 + (0.15 × age in years) 3. Depth (cm) = 0.8 + (0.05 × weight in kilograms)

can be less tensile in children; hence a distinctive “pop” may not be easily felt when penetrating this layer. In addition, the distance from the skin to the epidural space can be very superficial. Formulas for estimating the distance from skin to epidural space have been proposed (see Table 42–1). However, formulas are only a guideline and change according to the angle of epidural needle placement. C is incorrect. A midline approach to lumbar epidural needle placement is preferred. Identification of the epidural space is commonly achieved by loss of resistance (LOR) to saline. LOR to air should be avoided due to the risk of introducing a venous air embolism, particularly in neonates and infants. D is incorrect. Lumbar epidural analgesia is commonly used for continuous infusions and is rarely used as a single-injection technique. A direct lumbar approach is indicated primarily for pain control during and after lower extremity surgery. Lumbar epidural placement, particularly in young children, is performed after the induction of general anesthesia. However, this approach may also be performed awake in a select group of cooperative children and adolescents. Caution should be exercised whenever performing lumbar epidural analgesia above the level of spinal cord end to avoid direct needle trauma. 9. C is correct. Preprocedural US imaging is performed with the probe in three planes: transverse (SAX), median sagittal (median LAX), and paramedian oblique sagittal (paramedian oblique LAX). The distance from the patient’s skin to the epidural space can thus be measured. In the SAX (Figure 42–7), the window between two spinous processes must be found, and the anterior

FIGURE 42–8  US image in the short-axis view, showing the posterior complex with the dura (white arrow).

complex of the vertebral column (posterior longitudinal ligament, anterior dura, and vertebral body) in the depth of the image will be the first structures to be identified. The posterior complex (ligamentum flavum and posterior dura) may be more difficult to visualize but will be approximately at the level of the lamina (Figure 42–8). A is incorrect. Intended high thoracic catheter advancement from a lumbar insertion site is rarely successful. B is incorrect. Using the midline approach, insertion of the needle is easier at the lower thoracic level (T10–T12) than at the midthoracic (T4–T7) level. D is incorrect. The thoracic epidural space is identified with LOR to saline; air is not recommended in children. 10. B is correct. See Table 42–2. A is incorrect. Children require a higher dose of local anesthetic drug due to higher total cerebrospinal fluid (CSF), and spinal CSF volumes. Bupivacaine 0.5%, 0.5–1 mg/kg, is generally used for spinal anesthesia in children weighing less than 10 kg. A dose toward the higher end of the range is preferred for smaller children. TABLE 42–2  Spinal anesthesia dosage in children. Local anesthetic solution: ropivacaine or bupivacaine 0.5% 0.5–1 mg/kg. An easy way to calculate the dose for a single hernia repair in infants is as follows:

FIGURE 42–7  Preprocedural scanning with nonsterile set-up for teaching purposes; the US probe is in the short-axis view.

Hadzic_Ch42_p277-282.indd 281

Age (mos)

Weight (kg)

Dose bupivacaine 0.5%

1

3

1 mg/kg

2

4

0.8 mg/kg

3

5

0.6 mg/kg

>4

6

0.4 mg/kg

Possible additives: • Epinephrine wash • Clonidine 1 mcg/kg • Morphine 10 mcg/kg only for cardiac surgery to facilitate earlier extubation)

22/04/19 6:13 PM

282

PART 6

Pediatric Anesthesia

C is incorrect. CSF volume is higher on a milliliter-per-kilogram basis in infants and neonates (4 mL/kg) compared with adults (2 mL/kg). In addition, CSF in infants is distributed relatively more in the spinal canal than in the head, as opposed to the distribution in adults. This may, in part, account for the higher LA dose requirements and shorter duration of action of spinal anesthesia in infants. The high cardiac output characteristic of the pediatric population shortens the duration of spinal blocks in children still further. D is incorrect. Spinal anesthesia is customarily administered in the lateral or sitting position in children. If the lateral position is preferred, the patient is positioned at the very border of the surgical table and held firmly by an assistant. Otherwise, the patient can be placed in the middle of the surgical table but on top of several blankets; these will give the necessary height for the anesthesiologist’s hands to be placed comfortably while performing the block (Figure 42–9). If the sitting position is preferred, special attention must be paid in infants to ensure that the neck is not flexed, as this could result in airway obstruction (Figure 42–10). It is essential to vigilantly monitor oxygen saturation in infants while performing the spinal to ensure adequacy and patency of the airway. Moreover, neck flexion is not necessary because it does not facilitate the performance of the block in small children. In older children, an assistant should be present to maintain proper positioning and to reassure and distract the child while the block is being performed. The use of a pacifier while the block is being performed in a nonsedated infant is usually helpful.

FIGURE 42–10  Spinal anesthesia in the sitting position. Head flexion must be avoided to prevent airway obstruction.

References 1. Liley A, Manoharan M, Upadhyay V. The management of a postdural puncture headache in a child. Paediatr Anaesth. 2003;13(6):534-537. 2. Janssens E, Aerssens P, Alliet P, et al. Post-dural puncture headaches in children. A literature review. Eur J Pediatr. 2003;162:117-121. 3. Baris S, Guldogus F, Baris YS, et al. Is tissue coring a real problem after caudal injection in children? Paediatr Anaesth. 2004;14:755-758.

Suggested Reading

FIGURE 42–9  Spinal anesthesia in the lateral position. The child is placed on top of several blankets to gain height for the anesthesiologist’s hands while performing the block.

Hadzic_Ch42_p277-282.indd 282

Hadzic A. Pediatric epidural and spinal anesthesia & analgesia. In: De Jose Maria B, Tielens L, Roberts S, eds. Textbook of Regional Anesthesia and Acute Pain Management. 2nd ed. New York, NY: McGraw-Hill Publishing; 2017:chap 43.

22/04/19 6:13 PM

43 Peripheral Nerve Blocks for Children Franklin Chiao

QUESTIONS DIRECTIONS: Choose the one best response to each question. 1.  Levobupivacaine 0.2% at 1 mL/kg in a single shot caudal without any additive is used for hypospadias surgery. What advantage did the 0.2% concentration have over the 0.25% concentration? A. Higher level of block B. Shorter discharge time C. Lower incidence of cardiac toxicity D. Decreased motor weakness 2.  Neonatal spinal anesthesia supplemented with dexmedetomidine most often results in: A. Less conversion to general anesthesia B. Hemodynamic instability C. Slower discharge from PACU D. Shorter surgery duration 3.  A 1-month-old former 32-week gestation male infant is scheduled for an inguinal hernia repair. How does practitioner level (attending/CRNA/resident) matter in terms of success/failure of placement of an infant spinal? A. Spinal success rate is similar for all practitioner types. B. Attendings have as much as 10–15 times lower failure rate than residents. C. In order of highest success rate: attending > resident > CRNA. D. Ability to obtain CSF flow is similar with all practitioners. 4.  Local anesthetic drug dosage in neonates compared to adults should be: A. Increased due to higher glomerular filtration B. Adjusted downward due to lower α1-acid glycoprotein levels

C. Increased because of larger weight-based blood volume D. Kept the same 5.  Penile block is a valid block technique for which surgery? A. Scrotoplasty B. Proximal hypospadia C. Redo circumcision D. Hidden penis 6.  Which of the following statements is correct regarding an infraorbital nerve block in an infant? A. It provides effective analgesia for cleft lip repair at a volume of 1 mL/kg. B. It is opioid sparing for cleft palate repair. C. It provides more analgesia for tonsillectomy than cleft palate repair. D. It does not benefit patients having endoscopic sinus surgery. 7.  Which of the following is true regarding greater occipital nerve block in pediatrics? A. It is a treatment for cluster headaches. B. It is an effective technique for migraine headaches. C. Posterior fossa surgery is a relative contraindication. D. It reduces opioid use when utilized for cervical neuralgia. 8.  Which of the following statements is true regarding complication rates among pediatric central and peripheral nerve blocks? A. Complication rates are less for central nerve blocks than peripheral blocks only under general anesthesia. B. Complication rates are the same. C. Complication rates are less for central nerve blocks than peripheral blocks. D. Complication rates for central nerve blocks are six times as great as peripheral nerve blocks.

283

Hadzic_Ch43_p283-284.indd 283

18/04/19 6:30 PM

284

PART 6

Pediatric Anesthesia

ANSWERS AND EXPLANATIONS

References

1. D is correct. When caudal is used for analgesic purposes, motor block is not necessary and may worry caregivers in the post-anesthesia care unit even though risk of complications is extremely low.1

2. Chiao F, Boretsky K. Dexmedetomidine as a supplement to spinal anesthesia block: a case report of three infants. A A Case Rep. 2017 Aug 15;9(4):127-128.

2. A is correct. Dexmedetomidine enhances regional anesthesia blockade and deepens anesthesia, so when the block begins to wear off, sedative effects remain.2 3. B is correct. Infant spinal requires training and experience. It is more difficult than performing an adult spinal anesthetic and the perioperative management is very different. Consideration for who does the first attempt should be made if spinal is particularly important in the infant.3,4 4. B is correct. Peripheral nerve block dose should be adjusted to account for hepatorenal differences in neonates.5 5. C is correct. Penile block covers the distal two-thirds of the penis; therefore, it covers circumcision well. It can be used as a supplemental block for various other penile procedures as well.6 6. A is correct. Infraorbital nerve block has been shown in several studies to provide good analgesia for cleft lip surgery.7 7. B is correct. This type of block may be helpful for chronic pain patients with migraines.8 8. D is correct. Both central and peripheral nerve blocks have a long safety record, but in self-reported databases, peripheral nerve blocks have a lower rate of complications. In the future, as equipment for central and peripheral blocks improves, the relative rates may change.9

Hadzic_Ch43_p283-284.indd 284

1. Ivani G, De Negri P, Lonnqvist PA, et al. A comparison of three different concentrations of levobupivacaine for caudal block in children. Anesth Analg. 2003;97(2):368-371.

3. Davidson AJ, Disma N, De Graaff JC, et al. Neurodevelopmental outcome at 2 years of age after general anaesthesia and awakeregional anaesthesia in infancy (GAS): An international multicentre, randomised controlled trial. Lancet. 2016;387(10015):239-250. 4. Williams RK, Adams DC, Aladjem EV, et al. The safety and efficacy of spinal anesthesia for surgery in infants: the Vermont Infant Spinal Registry. Anesth Analg. 2006;102(1):67-71. 5. NS Morton, FRCA FRCPCH. Local and regional anaesthesia in infants. Continuing Education in Anaesthesia Critical Care & Pain. 2004;4(5):148-151. 6. Soh CR, Ng SB, Lim SL. Dorsal penile nerve block. Paediatr Anaesth. 2003 May;13(4):329-333. 7. Bouattour L, Smaoui M, Belhaj S, Khemakhem K, Chikhrouhou H. Infraorbital nerve block for cleft lip surgery: 8AP7-8. European Journal of Anaesthesiology. 2007;24(39):100. 8. Yongguo Tang, Junfang Kang, Yu Zhang, Xuejun Zhang. Influence of greater occipital nerve block on pain severity in migraine patients: A systematic review and meta-analysis. Am J Emerg Med. Nov 2017;35(11):1750-1754. 9. Walker BJ, Long JB, Sathyamoorthy M, et al. Complications in pediatric regional anesthesia: an analysis of more than 100,000 blocks from the pediatric regional anesthesia network. Anesthesiology. 2018;129(4):721-732.

Suggested Reading Hadzic A. Peripheral nerve blocks for children. In: Roberts S, ed. Textbook of Regional Anesthesia and Acute Pain Management. 2nd ed. New York, NY: McGraw-Hill Publishing; 2017:chap 44.

18/04/19 6:30 PM

44 Acute and Chronic Pain Management in Children Franklin Chiao

QUESTIONS DIRECTIONS: Choose the one best response to each question. 1. Which of the following statements is true regarding neonatal pain infrastructure? A. The sensation for pain is not present. B. Significant pain structures exist as early as the second trimester. C. Hydromorphone is not an effective drug for immature receptors. D. Newborn circumcision is most easily performed with supplemental opioids. 2. Neonatal pain management frequently suffers from: A. Undertreatment B. Lack of multimodal analgesia options C. Limited opioid choices D. Malfunctioning epidural catheters 3. How often can peripheral nerve blockade be performed in children in community hospitals? A. Frequently B. Rarely C. Not commonly D. Nearly always 4. Inadequate analgesia for newborn circumcision may result in: A. Minimal hemodynamic changes B. Long-term behavioral changes and short-term physiological disturbances C. Increased bleeding from blood pressure increases D. Limited changes in behavior

6. The most commonly used nonopioid pharmacologic drug in pediatrics is: A. Acetaminophen B. Ibuprofen C. Clonidine D. Ketorolac 7. COX-1 isoenzymes are involved with: A. Ovulation B. Control of cell growth C. Inflammatory prostaglandins D. Preservation of the stomach lining 8. Oxycodone and hydrocodone have: A. Less nausea than morphine B. 80% oral bioavailability C. Peak affect at 30 minutes D. Plasma half-life of 90 minutes 9. Tramadol has effects through multiple mechanisms, including: A. a2 agonism B. N-methyl-D-aspartate (NMDA) receptor agonism C. Norepinephrine reuptake inhibition D. COX-2 receptor 10. Gabapentin benefits in pediatrics include: A. Positive mood effects B. Reduction of opioid-related side effects in posterior spinal fusion C. Minimal sedative effects D. Reduction of perioperative opioid use for posterior spinal fusion

5. Which is part of the CRIES pain scale? A. Requires oxygen < 30% B. Legs drawn up C. Inconsolable D. Heart rate increase > 20 bpm 285

Hadzic_Ch44_p285-286.indd 285

18/04/19 6:31 PM

286

PART 6

Pediatric Anesthesia

ANSWERS AND EXPLANATIONS

References

1. B is correct. The functional spinal reflex is present by 19 weeks. Sensory fibers are abundant by 20 weeks, and connections to subplate neurons have differentiation by 25 weeks.1,2

2. Lee SJ, Ralston HJ, Drey EA, et al. Fetal pain: a systematic multidisciplinary review of the evidence. JAMA. 2005;294:947-954.

1. Fitzgerald M. Neurobiology of fetal and neonatal pain. In: Wall PD, Melzack R, eds. Textbook of Pain. 3rd ed. London: Churchill Livingstone; 1994:153-164.

2. A is correct. Historically, neonates were not given analgesic medications for surgery. Although this is no longer the case, pain is still undertreated, and multimodal analgesic methods are underutilized.3,4

3. Anand KJ, Hickey PR. Pain and its effects in the human neonate and fetus. N Engl J Med. 1987;317:1321-1329.

3. C is correct. Rural and nonacademic centers are lacking in pediatric pain management options. In particular, peripheral nerve blocks and pain-controlled analgesia (PCA) devices are often not provided.5

5. Chiao FB, Wang A. An examination of disparities in pediatric pain management centered on socioeconomic factors and hospital characteristics. J Racial Ethn Health Disparities. 2018 Feb;5(1):73-77.

4. B is correct. As early as birth, lack of adequate analgesia for neonates can have long-term effects.6,7 5. A is correct. CRIES is a well-known pain scale and has both observed and physiologic components, including crying, requires O2, increased vital signs, expression, and sleeplessness. 6. A is correct. Acetaminophen has a long history of use in pediatrics, is comfortably used by many pediatricians, and has few major side effects.8,9 7. D is correct. Nonsteroidal antiinflammatory drugs (NSAIDs) work through COX mechanisms. Pathways are frequently analyzed during drug development because of their impact on specific daily functions.10,11 8. A is correct. Both oxycodone and hydrocodone cause nausea but less than morphine. This has led to its frequent use in ambulatory settings. 9. C is correct. Tramadol has atypical opioid activity as it also works through the serotonergic and noradrenergic systems. 10. D is correct. Gabapentin has an opioid-sparing effect for spinal fusion. It thereby may have other notable benefits as well for these and other surgeries.12

Hadzic_Ch44_p285-286.indd 286

4. Stevens B, Gibbins S, Franck LS. Treatment of pain in the neonatal intensive care unit. Pediatr Clin North Am. 2000;47:633-650.

6. Taddio A, Katz J, Ilersich AL, Koren G. Effect of neonatal circumcision on pain response during subsequent routine vaccination. Lancet. 1997;349(9052):599-603. 7. Taddio A, Katz J. The effects of early pain experience in neonates on pain responses in infancy and childhood. Paediatr Drugs. 2005;7(4):245-257. 8. Birmingham PK, Tobin MJ, Henthorn TK, et al. Twenty-fourhour pharmacokinetics of rectal acetaminophen in children: an old drug with new recommendations. Anesthesiology. 1997;87(2):244-252. 9. Rusy LM, Houck CS, Sullivan LJ, et al. A double-blind evaluation of ketorolac tromethamine versus acetaminophen in pediatric tonsillectomy: analgesia and bleeding. Anesth Analg. 1995;80(2):226-229. 10. Vane JR, Botting RM. Mechanism of action of nonsteroidal anti-inflammatory drugs. Am J Med. 1998;104(3A):2S-8S; discussion 21S-22S. 11. Vane JR, Bakhle YS, Botting RM. Cyclooxygenases 1 and 2. Annu Rev Pharmacol Toxicol. 1998;38:97-120. 12. Rusy LM, Hainsworth KR, Nelson TJ, et al. Gabapentin use in pediatric spinal fusion patients: a randomized, double-blind, controlled trial. Anesth Analg. 2010;110(5):1393-1398.

Suggested Reading Hadzic A. Acute and chronic pain management in children. In: Diwan RM, ed. Textbook of Regional Anesthesia and Acute Pain Management. 2nd ed. New York, NY: McGraw-Hill Publishing; 2017:chap 45.

18/04/19 6:31 PM

PART 7 Anesthesia in Patients with Specific Considerations Chapter 45

Perioperative Regional Anesthesia in the Elderly  289

Chapter 46

Regional Anesthesia and Cardiovascular Disease  295

Chapter 47

Regional Anesthesia and Systemic Disease  299

Chapter 48

Regional Anesthesia in the Patient with Preexisting Neurologic Disease  303

Chapter 49

Acute Compartment Syndrome of the Limb: Implications for Regional Anesthesia  307

Chapter 50

Peripheral Nerve Blocks for Outpatient Surgery  309

Chapter 51

Neuraxial Anesthesia and Peripheral Nerve Blocks in Patients on Anticoagulants  313

Chapter 52

Regional Analgesia in the Critically Ill  317

Chapter 53

Acute Pain Management in the Opioid-Dependent Patient  319

Chapter 54

Regional Anesthesia in Patients with Trauma  325

Chapter 55

Regional Anesthesia for Cardiac and Thoracic Anesthesia  329

Chapter 56

Regional Anesthesia in Austere Environment Medicine  333

Chapter 57

Anesthesia for Humanitarian Relief Operations  337

Hadzic_Ch45_p287-294.indd 287

18/04/19 6:32 PM

This page intentionally left blank

Hadzic_FM_p00i-pxx.indd 6

22/04/19 6:40 PM

45 Perioperative Regional Anesthesia in the Elderly Thomas M. Halaszynski

QUESTIONS DIRECTIONS: Choose the one best response to each question. 1. How have the United States elderly patient population demographics changed over the last 100 years and what impact will the changing demographics have on perioperative services? A. Geriatric population growth impacts surgical treatment services through increased number of surgical procedures along with increased utilization of perioperative services. B. Patients over age 65 years represent approximately 12% of the population, account for 33% of health care costs, 38% of hospital bed stays, and 21% of inpatient surgical procedures. C. Of Medicare in-patients, 40% suffer a minor or major medical, surgical, or anesthesia-related complication(s) during hospitalization for noncardiac surgery; 8% experience a “failure to rescue” event; and 4% die in the perioperative period. D. All of the above 2. Compared to middle-aged patients (< 50 years), what volume of local anesthetic (LA) should be administered to elderly patients (> 65 years) to achieve an adequate ultrasound-guided supraclavicular brachial plexus block (US-SCB)? A. The same volume as the middle-aged patient population B. Half of the volume of LA C. One-eighth the volume of LA D. Double the volume of LA 3. What type of change(s) to the autonomic nervous system (ANS) are seen in the elderly patient population as a result of the “human aging process”? A. An increased parasympathetic drive and a decreased sympathetic drive B. An increased sympathetic drive and a decreased parasympathetic drive

C. Decreases in both sympathetic and parasympathetic activity D. Increases in both sympathetic and parasympathetic activity 4. When compared to middle-aged adults, elderly patients process pain differently due to changes in aging human physiology such that: A. Elderly patients have a higher threshold for thermal pain stimuli. B. Elderly patients are able to tolerate higher levels of pain. C. Elderly patients have a lower threshold for thermal pain stimuli. D. Elderly patients require higher perioperative opioid rescue. 5. What has been determined to be the minimum effective intrathecal morphine dose when combined with bupivacaine for postoperative analgesia in elderly patients undergoing lower extremity surgery? A. 300 mcg B. 50 mcg C. 100 mcg D. 200 mcg 6. Compared to epidural analgesia, paravertebral blockade (PVB) in elderly patients for thoracotomy surgery has been associated with which of the following? A. Greater incidence of hypotension B. Increased IV or PO analgesic requirements after surgery C. Increased incidence of pruritus D. Equally effective analgesia

289

Hadzic_Ch45_p287-294.indd 289

18/04/19 6:32 PM

290

PART 7

Anesthesia in Patients with Specific Considerations

7. Anatomic changes associated with aging can make performing neuraxial blockade difficult due to degenerative disk and vertebral joint changes, distortion and compression of intervertebral and epidural spaces, along with calcified ligaments. Therefore, which of the following vertebral spaces has been found to be most accessible when performing neuraxial blockade in the elderly? A. L2–L3 B. L3–L4 C. L4–L5 D. L5–S1 8. In elderly patients undergoing knee surgery, which perioperative anesthetic modality resulted in decreased need for rescue analgesics, decreased rates of complications, early mobilization, improved patient participation with physical therapy, and highest patient satisfaction? A. General anesthesia with IV patient-controlled analgesia (PCA) for postoperative analgesia B. General anesthesia with peripheral nerve blockade C. General anesthesia with epidural analgesia D. Spinal anesthesia with IV PCA for postoperative analgesia 9. Perception of pain and evaluation of pain management therapies in the elderly present problems arising from differences in: A. Cognitive functional abnormalities B. Variations in medication tolerance and abuse C. End-organ impairment/compromise (affecting medication metabolism and excretion) D. All of the above 10. Up to what percentage of elderly patients can experience perioperative delirium? A. 10% B. 40% C. 80% D. 100% 11. Which of the following regional anesthetic techniques would have the least chance of affecting the pulmonary physiology of an 89-year-old man with end-stage chronic obstructive pulmonary disease (COPD)? A. Thoracic epidural B. Lumbar epidural C. Interscalene brachial plexus nerve block D. General anesthesia

ANSWERS AND EXPLANATIONS 1. D is correct. Demographics of the elderly patient population in the United States (US) have changed over the last 100 years. The entire US population almost tripled during the 20th century and the geriatric segment of the population grew tenfold alone. Only 4% of the population

Hadzic_Ch45_p287-294.indd 290

(less than 5 million individuals) was over 65 years old at the turn of the century, but the number of people aged 65 and older now constitutes 12% of the population. It is estimated that the number of elderly individuals will double again by 2040. The oldest old, those greater than 85 years of age, represent the fastest growing segment of the population. Currently octogenarians account for 12% of all elderly; however, they are predicted to constitute almost 20% of the elderly population by 2040. The number of centenarians is increasing even faster—from 57,000 in 1996 to an estimated/predicted 447,000 individuals by 2040. Thus, not only is the absolute number of older Americans increasing but the overall US population is also becoming older with proportionally fewer individuals under 65 years of age. 2. B is correct. The minimum effective anesthetic volume (MEAV) of LA required to achieve surgical anesthesia in 50% or 95% of the elderly population (MEAV50 and MEAV95, respectively) was studied. Authors of one study found a nearly 50% decrease in the cross-sectional area (CSA) of the brachial plexus at the level of the first rib in the elderly patients compared to middle-aged patients. In addition, results showed for the MEAV50 for US-SCB in middle-aged versus elderly patients that the elderly require a 50% dose reduction to achieve a successful block and believed that this may be due to the smaller CSA found in the elderly population compared to the middle-aged population as well as increased sensitivity to LAs among the elderly. A and C are incorrect. The likelihood of LA complications will depend on the elderly patient’s underlying functional reserve and comorbidities. Both age and comorbid disease contribute to the loss of functional reserve of all organs and most importantly, the heart, lungs, brain, and kidneys. Decreased functional organ reserve diminishes the ability of older patients to restore homeostasis that is distorted by physiologic changes caused by disease. In addition, LA administered during surgery and anesthesia can also increase perioperative risk if organ function is compromised. D is incorrect. It is known that the body changes (ie, physiological along with organ reserve and alterations of drug metabolism) with age and, as such, the elderly have increased sensitivity to many medications, including LAs. Therefore, doubling the typical dose of LA during an US-SCB could predispose elderly patients toward LA toxicity issues. 3. B is correct. An increased sympathetic drive and a decreased parasympathetic drive occur as a result of the “human aging process.” The autonomic nervous system in the elderly is characterized by a net activation of sympathetic stimulation and a relative decrease in basal parasympathetic activity.2 A, C, and D are incorrect. Aging of the ANS is also characterized by decreased baroreflex sensitivity, a reason that elderly patients have autonomic dysreflexia and a propensity to develop orthostatic hypotension upon standing

18/04/19 6:32 PM

CHAPTER 45

from the seated position. Parashar et al have found that recovery of heart rate after exercise is significantly affected in the elderly, secondary to a sluggish vagal response and increased sympathetic drive, with many patients becoming symptomatic to increased heart rate and sudomotor activity to the point of having to cease activity.4 Seals et al have shown that an exercise program in the elderly can be advantageous to lower resting heart rate as well as basal serum catecholamine levels and can improve elderly patients’ exercise performance by decreasing fatigability and increasing left ventricular function.5 Regional anesthetic interventions can mitigate cardiac morbidity resulting from these physiological derangements. This has been demonstrated in a study involving patients who received epidural anesthesia for aortic aneurysm repair who experienced, compared to a control group, improved myocardial oxygen delivery. The epidural group also was shown to have improved ventricular function during periods of surgical stress, most likely secondary to blunting some of the high basal sympathetic outflow through anesthetizing the thoracic sympathetic chain fibers.6–8 4. A is correct. Elderly patients have been found to have a higher threshold for thermal pain stimuli. No significant change compared to nonelderly patients has been found with respect to mechanical or electrical stimuli.9 B is incorrect. With respect to “tolerance,” when elderly patients do register a sensation as painful, they do not tolerate it as well as the nonelderly, and they are not as apt to localize the source.2 This is likely a result of decreases in noradrenergic and serotonergic neurons that contribute to the impairment of descending inhibitory mechanisms.2 D is incorrect. There is an age-related decrease in opioid requirements after surgery with elderly patients.10 One study in particular has found that, in looking at elderly patients undergoing specific surgeries, older patients reported 10% to 20% less pain per decade after age 60.11 This is likely a manifestation of anatomical differences in the elderly nervous systems, including a decrease in the density of unmyelinated nerve fibers, a slowing of nerve conduction velocity, and reductions in hormone mediators of pain conduction such as substance P.12–14 5. C is correct. In a study of elective hip arthroplasty patients comparing multiple doses of intrathecal morphine combined with bupivacaine, it was found that 100 mcg was effective, although safe administration of up to 200 mcg is commonly recommended in the literature as well. 100 mcg of morphine has been found to provide effective analgesia while minimizing untoward adverse effects such as pruritus, nausea, and respiratory depression.15 Groups in a dose-response study by Murphy et al randomized to receive 50, 100, or 200 mcg of morphine intrathecally had no significant differences in the incidence of nausea or respiratory depression. Use of neuraxial morphine has been found to improve immediate postoperative pain scores as well as reduce intravenous morphine requirements after surgery.16 A, B, and D are incorrect. See explanation for answer C.

Hadzic_Ch45_p287-294.indd 291

Perioperative Regional Anesthesia in the Elderly

291

6. D is correct. Paravertebral blockade has been found to be at least as effective as epidurals for post-thoracotomy pain control, and one study found patients to have improved pain scores at rest after surgery.17 The PVB procedure, unlike neuraxial procedures, can be safely performed under sedation, an important advantage for patients unable to sit comfortably or lie on their side. A, B, and C are incorrect. Epidural dosing can frequently result in hypotension if not treated, as well as an increased incidence of pruritus from epidural opioid additives when compared to paravertebral analgesia with local anesthetics alone. Thoracic epidurals can also have unwanted physiological effects in the pulmonary system, such as contributing to atelectasis and decreased peak expiratory flow rates. PVB patients have been found to have decreased incidences of these side effects, as well as decreased incidences of urinary retention and nausea when compared to matched epidural groups.18 7. D is correct. Accessing the epidural or intrathecal space in elderly patients can pose a unique challenge. The ligamentum flavum, with age, becomes more calcified, altering the normal tactile changes a proceduralist appreciates during loss of resistance techniques to access the desired space. Osteophytes, osteoporotic changes, and pronounced scoliotic changes can also decrease intervertebral space size, limiting needle access. In addition, obtaining the ideal patient position during procedures can be limited by pain or cognitive impairment. The L5–S1 interspace is typically the largest intervertebral space and, therefore, the most easily accessed in the elderly. If a higher level of neuraxial blockade is desired, it is recommended to attempt a paramedian approach in patients with difficult vertebral anatomy for the aforementioned reasons.2 A, B, and C are incorrect. See explanation for answer D. 8. B is correct. Patient satisfaction after knee surgery has been shown to be highest, in the referenced studies, in groups receiving peripheral nerve blocks, such as sciatic and femoral blockade. Peripheral nerve blockade offers elderly patients several benefits: decreased need for supplemental analgesics that have their own side effect profiles (nausea, constipation, etc.), decreased rates of complications secondary to neuraxial techniques (urinary retention, hemodynamic variability), and improved participation with physical therapy and early mobilization.15,19,20 A and D are incorrect. PCA use can be associated in the elderly population with increased rates of sedation, nausea, and vomiting when compared to epidural or peripheral nerve blockade.21 C is incorrect. Epidural analgesia has been shown to offer superior analgesia at rest and with movement when compared to IV PCA, although it can be associated with a higher rate of motor blockade, pruritus (when opioid additives are used), hemodynamic changes, and urinary retention. For patients specifically undergoing knee surgery, motor blockade can preclude participation with physical therapy and can delay early mobilization.22

18/04/19 6:32 PM

292

PART 7

Anesthesia in Patients with Specific Considerations

9. D is correct. Inadequate treatment of acute pain is more likely to occur in older patients, especially those who are cognitively impaired. Even though cognitively impaired patients are just as likely as others of the same age and younger to experience painful conditions, the number of pain complaints, along with reported pain intensity, has been shown to be inversely related to the degree of cognitive impairment (due to diminished memory, impairment of capacity to report, or because less pain is experienced). Physiologic changes associated with aging vary markedly among individuals. The administration of pain medications warrants a decrease in dose (maintenance and/or bolus) of drug required for analgesia to avoid the risk of increased plasma drug accumulation and accumulation of active metabolites. Age-related changes in the body are responsible for alterations seen with systemic absorption, distribution, and clearance of medications that may result in an increased sensitivity, decreased dose requirement, and change in the onset and duration of action in elderly patients. 10. C is correct. Delirium can occur in up to 80% of elderly patients after surgery. It is more common amongst patients undergoing cardiac and major noncardiac surgery, as well as emergency and trauma cases. A, B, and D are incorrect. Increasing age, use of narcotics, and the presence of preoperative chronic pain syndromes are all positively associated with a higher risk for the development of delirium after surgery.25-29 Postoperative cognitive dysfunction (POCD) is a sequela of postoperative delirium and is also common among the elderly surgical population with the aforementioned risk factors.30 Regional anesthesia is beginning to be recognized for its potential role in decreasing postoperative delirium and POCD.31,32 In a study of elderly patients undergoing surgery for hip fracture, patients who received femoral nerve blocks were found to have less incidence of delirium after surgery and required significantly less, or even no, supplemental narcotic analgesic agents.33 11. B is correct. Lumbar epidural anesthesia typically does not affect diaphragmatic excursion or the accessory muscles of breathing that some elderly patients with preexisting respiratory conditions may rely upon for adequate ventilation. Functional residual capacity (FRC) does not vary appreciably from baseline during spinal and lumbar epidural anesthesia.2 A is incorrect. Thoracic epidural and paravertebral anesthesia has been shown to be associated with varying degrees of lung volume reductions secondary to intercostal muscle relaxation. C is incorrect. An interscalene and/or supraclavicular brachial plexus block can frequently be associated with phrenic nerve blockade as well, causing a hemidiaphragmatic paralysis for part of the duration of the plexus blockade. This block likely would not be tolerated well in a patient with limited pulmonary reserve if the phrenic nerve should be blocked during upper brachial plexus blockade.

Hadzic_Ch45_p287-294.indd 292

As the supraclavicular approach to the brachial plexus targets an area further removed anatomically from the phrenic nerve, which lies close to the C5 nerve root, it is often thought of as having a much lower chance of phrenic nerve involvement. It has been shown, though, that up to two-thirds of supraclavicular block patients will have some degree of diaphragmatic paralysis as measured by ultrasonography.34 This is usually not clinically significant, but for patients with poor pulmonary function it can pose a risk. For these patients, an axillary approach to upper extremity nerve branches may be a good alternative for anesthesia and analgesia for the distal upper limbs. Lower extremity nerve blocks and abdominal regional anesthetics have been shown, however, to benefit elderly patients. They experience fewer hypoxic events with regional and epidural anesthesia and have fewer respiratory complications.2

References 1. White PF, White LM, Monk T, et al. Perioperative care for the older outpatient undergoing ambulatory surgery. Anesth Analg. 2012;114:1190-1215. 2. Hadzic A. Textbook of Regional Anesthesia and Acute Pain Management. 2nd ed. New York, NY: McGraw-Hill Publishing; 2017:865-880. 3. Pavicic Saric J, Vidjak V, Tomulic K, Zenko J. Effects of age on minimum effective volume of local anesthetic for ultrasoundguided supraclavicular brachial plexus block. Acta Anaesthiesiol Scand. 2013;57(6):761-766. 4. Parashar R, et al. Age-related changes in autonomic functions. J Clin Diagn Res. 2016 Mar;10(3):CC11-CC15. 5. Seals DR, Taylor JA, Ng AV, Esler MD. Exercise and aging: autonomic control of the circulation. Med Sci Sports Exerc. 1994;26(5):568-576. 6. Schmidt C, Hinder F, Van Aken H, et al. The effect of high thoracic epidural anesthesia on systolic and diastolic left ventricular function in patients with coronary artery disease. Anesth Analg. 2005;100:1561-1569. 7. Jakobsen CJ, Nygaard E, Norrild K, et al. High thoracic epidural analgesia improves left ventricular function in patients with ischemic heart. Acta Anaesthesiol Scand. 2009;53:559-564. 8. Lagunilla J, Garcia-Bengochea JB, Fernandez AL, et al. High thoracic epidural blockade increases myocardial oxygen availability in coronary surgery patients. Acta Anaesthesiol Scand. 2006;50:780-786. 9. Gagliese L, Weizblit N, Ellis W, Chan VW. The measurement of postoperative pain: a comparison of intensity scales in younger and older surgical patients. Pain. 2005;117:412-420. 10. Fjell AM, Walhovd KB. Structural brain changes in aging: courses, causes and cognitive consequences. Rev Neurosci. 2010;21:187-221. 11. Thomas T, Robinson C, Champion D, McKell M, Pell M. Prediction and assessment of the severity of post-operative pain and of satisfaction with management. Pain. 1998;75:177-185. 12. Sato A, Sato Y, Suzuki H. Aging effects on conduction velocities of myelinated and unmyelinated fibers of peripheral nerves. Neurosci Lett. 1985;53:15-20.

18/04/19 6:32 PM

CHAPTER 45 13. Rivner MH, Swift TR, Malik K. Influence of age and height on nerve conduction. Muscle Nerve. 2001;24:1134-1141. 14. Gagliese L, Ferrell M. The neurobiology of aging, nociception and pain: an integration of animal and human experimental evidence. In: Gibson SJ, Weiner DK, eds. Progress in Pain Research and Management: Pain in the Older Person. Seattle: IASP; 2005:25-44. 15. Murphy PM, Stack D, Kinirons B, Laffey JG. Optimizing the dose of intrathecal morphine in older patients undergoing hip arthroplasty. Anesth Analg. 2003;97:1709-1715. 16. Sultan P, Gutierrez MC, Carvalho B. Neuraxial morphine and respiratory depression: finding the right balance. Drugs. 2011;71:1807-1819. 17. Beaussier M, Weickmans H, Parc Y, et al. Postoperative analgesia and recovery course after major colorectal surgery in elderly patients: a randomized comparison between intrathecal morphine and intravenous PCA morphine. Reg Anesth Pain Med. 2006;31:531-538. 18. Davies RG, Myles PS, Graham JM. A comparison of the analgesic efficacy and side effects of paravertebral vs epidural blockade for thoracotomy—a systematic review and meta-analysis of randomized trials. Br J Anaesth. 2006;96:418-426. 19. Rebel A, Sloan P, Andrykowski M. Retrospective analysis of high-dose intrathecal morphine for analgesia after pelvic surgery. Pain Res Manag. 2011;16:19-26.

293

Perioperative Regional Anesthesia in the Elderly

25. Bitsch MS, Foss NB, Kristensen BB, Kehlet H. Acute cognitive dysfunction after hip fracture: frequency and risk factors in an optimized, multimodal, rehabilitation program. Acta Anaesthesiol Scand. 2006;50:428-436. 26. Fong HK, Sands LP, Leung JM. The role of postoperative analgesia in delirium and cognitive decline in elderly patients: a systematic review. Anesth Analg. 2006;102:1255-1266. 27. Greene NH, Attix DK, Weldon BC, Smith PJ, McDonagh DL, Monk TG. Measures of executive function and depression identify patients at risk for postoperative delirium. Anesthesiology. 2009;110:788-795. 28. Vaurio LE, Sands LP, Wang Y, Mullen EA, Leung JM. Postoperative delirium: the importance of pain and pain management. Anesth Analg. 2006;102:1267-1273. 29. Morimoto Y, Yoshimura M, Utada K, Setoyama K, Matsumoto M, Sakabe T. Prediction of postoperative delirium after abdominal surgery in the elderly. J Anesth. 2009;23:51-56. 30. Newman S, Stygall J, Hirani S, Shaefi S, Maze M. Postoperative cognitive dysfunction after noncardiac surgery: a systematic review. Anesthesiology. 2007;106:572-590. 31. Gerbershagen HJ, Aduckathil S, van Wijck AJ, Peelen LM, Kalkman CJ, Meissner W. Pain intensity on the first day after surgery: a prospective cohort study comparing 179 surgical procedures. Anesthesiology. 2013;118:934-944.

20. Bujedo BM, Santos SG, Azpiazu AU. A review of epidural and intrathecal opioids used in the management of postoperative pain. J Opioid Manag. 2012;8:177-192.

32. Papaioannou A, Fraidakis O, Michaloudis D, Balalis C, Askitopoulou H. The impact of the type of anaesthesia on cognitive status and delirium during the first postoperative days in elderly patients. Eur J Anaesthesiol. 2005;22:492-499.

21. Foss NB, Kristensen MT, Kristensen BB, Jensen PS, Kehlet H. Effect of postoperative epidural analgesia on rehabilitation and pain after hip fracture surgery: a randomized, double-blind, placebo-controlled trial. Anesthesiology. 2005;102:1197-1204.

33. Mason SE, Noel-Storr A, Ritchie CW. The impact of general and regional anesthesia on the incidence of post-operative cognitive dysfunction and post-operative delirium: a systematic review with meta-analysis. J Alzheimers Dis. 2010;22:67-79.

22. Zaric D, Boysen K, Christiansen C, Christiansen J, Stephensen S, Christensen B. A comparison of epidural analgesia with combined continuous femoral-sciatic nerve blocks after total knee replacement. Anesth Analg. 2006;102:1240-1246.

34. Mak PH, Irwin MG, Ooi CG, Chow BF. Incidence of diaphragmatic paralysis following supraclavicular brachial plexus block and its effect on pulmonary function. Anaesthesia. 2001;56(4):352-356.

23. Gibson SJ, ed. Pain and Aging: The Pain Experience Over the Adult Life Span. Seattle: IASP; 2003. Dostrovsky JO, Carr DB, Koltzenburg M, eds. Proceedings of the 10th World Congress on Pain. Progress in Pain Research and Management. 24. Onur OA, Piefke M, Lie CH, Thiel CM, Fink GR. Modulatory effects of levodopa on cognitive control in young but not in older subjects: a pharmacological fMRI study. J Cog Neurosci. 2011;23:2797-2810.

Hadzic_Ch45_p287-294.indd 293

Suggested Reading Hadzic A. Perioperative regional anesthesia in the elderly. In: Dominguez JE, Halaszynski TM, eds. Textbook of Regional Anesthesia and Acute Pain Management. 2nd ed. New York, NY: McGraw-Hill Publishing; 2017:chap 46.

18/04/19 6:32 PM

This page intentionally left blank

Hadzic_FM_p00i-pxx.indd 6

22/04/19 6:40 PM

46 Regional Anesthesia and Cardiovascular Disease Christiana Burt

QUESTIONS DIRECTIONS: Choose the one best response to each question. 1.  The cardiovascular effects of high thoracic epidural anesthesia (TEA) (T1–T5) include the following except for: A. An increase in cardiac output B. Reduction in heart rate and systemic arterial pressure C. Blunting of the sympathetic response to laryngoscopy D. Reduction in chest pain in patients with severe coronary artery disease and unstable angina pectoris 2.  An 85-year-old man with severe long-standing hypertension presents with a hip fracture requiring surgical intervention. Which of the following statements is true regarding anesthesia in this context? A. Hypertension is not a significant problem as regional anesthesia will prevent the blood pressure rising. B. Lumbar epidural anesthesia may reduce the risk of coronary ischemia. C. Severe mitral regurgitation would be an indication for emergency cardiac surgery to be undertaken first. D. Hypotension is less likely to occur as a result of peripheral vasodilation due to the likely presence of left ventricular hypertrophy and increased force of ventricular contraction. 3.  Which of the following congenital cardiac conditions carries the greatest potential risks for noncardiac surgery with a central neuraxial regional anesthetic technique? A. “Hole in the heart” atrial septal defect (ASD) corrected as a child B. Hypoplastic right heart syndrome palliated with a Fontan circulation C. Aortic stenosis as a result of a congenital bicuspid valve operated on 6 months ago with a bioprosthetic valve replacement D. Congenital mitral valve prolapse awaiting surgical correction

Hadzic_Ch46_p295-298.indd 295

4.  Intercostal blockade (ICB) in cardiac surgery: A. Is a useful adjunct to postoperative analgesia B. Provides inferior analgesia postoperatively compared to intravenous opiate C. Carries a high risk of cardiovascular instability D. Is more effective than intrathecal opiate 5.  Considering cardiovascular function, a patient with poor left ventricular ejection fraction: A. Undergoing carotid endarterectomy surgery should have an isolated regional anesthetic technique without general anesthesia B. Will usually have coexisting coronary artery disease C. May benefit from increased diastolic perfusion time as a result of bradycardia D. May benefit from an anesthetic technique that reduces systemic vascular resistance 6.  Regarding central neuraxial blockade in cardiac surgery, which of the following statements is correct? A. Cardiac surgery may only be performed with general anesthesia. B. Thoracic epidural analgesia is recommended by the American College of Cardiology for use in coronary artery bypass grafting surgery. C. The estimated risk of neurological complication after a central neuraxial technique in cardiac surgery is 1 in 50,000. D. The use of thoracic epidural analgesia may reduce the risk of atrial fibrillation after cardiac surgery. 7.  Which of the following statements is true regarding regional analgesia in a patient with severe coronary artery disease? A. Regional anesthesia and analgesia should be avoided due to the relationship between hypotension and myocardial ischemia. B. A greater dose of local anesthetic should be used in order to maximize the benefits from pain reduction. C. Lower extremity peripheral nerve blockade carries a low risk of cardiovascular complications. D. Tachycardia is beneficial as the increased systemic pressure increases coronary perfusion.

295

22/04/19 2:07 PM

296

PART 7

Anesthesia in Patients with Specific Considerations

8.  The pathophysiological consequences of severe aortic stenosis: A. Are generally insignificant if regional anesthesia is planned B. Mean that aortic valve surgery or valvuloplasty should take place before elective noncardiac surgery C. Respond well to systemic vasodilation D. Mean that a central neuraxial technique should never be used 9.  A 30-year-old woman in her third trimester of pregnancy is incidentally found to have a congenitally bicuspid aortic valve with moderate aortic regurgitation. You are asked to give an opinion on the suitability of regional analgesia/anesthesia for labor and delivery. Which of the following statements regarding this situation is true? A. Care will need to be taken with a central neuraxial technique as this is likely to worsen the regurgitation; hence lumbar epidural analgesia is not recommended for labor. B. The patient should be referred for urgent cardiac surgery as the risk of progression to severe regurgitation would threaten the pregnancy. C. Her cardiac disease is low risk and shouldn’t have a large influence on her care, but lumbar epidural analgesia in labor would probably be beneficial. D. The advice would be the same regardless of the cause of the moderate aortic regurgitation.

perfusion if blood pressure is allowed to drop too low. Measures to prevent large decreases in blood pressure should be taken, with regard to the patient’s baseline blood pressure. C is incorrect. Severe mitral regurgitation may be an indication for urgent cardiological assessment and treatment, particularly if it were causing symptoms and signs of heart failure, but would not in itself be an indication for urgent cardiac surgery. D is incorrect. Left ventricular hypertrophy is indeed likely to be present in this patient, but this increases the risk of hypotension as a result of peripheral vasodilation due to the greater likelihood of subendocardial ischemia in response to compromised coronary perfusion and the possibility of left ventricular outflow tract obstruction in response to decreased LV cavity size.

1. A is correct. An increase in cardiac output is not observed with high thoracic epidural anesthesia due to the reduction in chronotropic and inotropic drive to the myocardium. B is incorrect. A reduction in heart rate and systemic arterial pressure is commonly seen due to reduced sympathetic outflow. C is incorrect. Licker et al in 1995 reported that patients who received TEA in addition to general anesthesia (GA) had smaller increases in mean arterial pressure and heart rate during laryngoscopy and tracheal intubation than those who received GA only, suggesting that high TEA affords some hemodynamic protection during these maneuvers. D is incorrect. Blomberg et al in 1989 showed that high TEA relieved chest pain in patients with unstable angina pectoris.

3. B is correct. Ideally surgery should take place in a specialist facility with support from clinicians familiar with the patient and his/her current medical status. Potential preexisting problems may include dysrhythmias, anticoagulant medication, poor left ventricular function, and thrombosed blood vessels. Cardiac output is dependent on passive venous return to the pulmonary circulation and pulmonary vascular resistance. Sedation must be used with extreme caution due to the risk of hypoxia and hypercarbia, which may precipitate an acute rise in pulmonary vascular resistance and corresponding reduction in cardiac output. Blood pressure monitoring will need to take into account any surgical shunts (eg, Blalock-Taussig) or thrombosed arteries. A sudden reduction in systemic vascular resistance should be avoided. A is incorrect. A simple ASD corrected fully as a child should pose little additional risk to an adult presenting for noncardiac surgery. Problems may arise if there is a significant residual leak across the patch causing right heart overload, particularly if accompanied by increased pulmonary vascular resistance. C is incorrect. As long as the new valve is functioning well and the patient has recovered from his/her surgery this should cause little additional risk for further noncardiac procedures. Anticoagulant medications in addition to aspirin would normally not be necessary for a bioprosthetic valve but would need to be taken into consideration if a mechanical valve was used. D is incorrect. This lesion would usually result in mitral regurgitation, which would be considered for surgical correction once severe or once the patient has significant symptoms. Reduction in systemic vascular resistance is usually well tolerated by patients with isolated regurgitant valvular lesions as long as the coronary arteries are patent.

2. B is correct. A reduction in ischemic episodes has been demonstrated in elderly patients undergoing surgery for hip fracture who received continuous lumbar epidural analgesia perioperatively. A is incorrect. Severe long-standing hypertension is associated with a shift in the autoregulatory curve for many vascular beds, resulting in a decrease in end-organ

4. A is correct. Several studies have reported a benefit in postoperative analgesia with ICB compared to opiate administration or placebo. B is incorrect. A direct comparison in patients undergoing minimally invasive direct coronary artery bypass (MIDCAB) surgery by Behnke et al showed better pain relief in the group who received ICB.

ANSWERS AND EXPLANATIONS

Hadzic_Ch46_p295-298.indd 296

22/04/19 2:07 PM

CHAPTER 46

C is incorrect. ICB carries significantly less risk of cardiovascular complications than central neuraxial blockade. D is incorrect. There is no evidence to suggest that ICB provides better analgesia than intrathecal opiate. 5. D is correct. A reduction in systemic vascular resistance will improve ejection fraction and reduce the workload on the left ventricle. Care must be taken to ensure this doesn’t happen at the expense of coronary artery perfusion. A is incorrect. The recent GALA trial showed no conclusive difference in outcome between patients undergoing carotid endarterectomy with regional anesthesia compared to general anesthesia alone, although the study may have excluded patients at highest risk for adverse cardiovascular outcomes. In the absence of clear guidelines, the decision on which technique to use should be made using a patient-centered approach. B is incorrect. Poor left ventricular function can be the end result of a variety of pathology including isolated valvular disease and inherited and acquired myocardial muscle disorders. Coexisting coronary artery disease should be considered and investigated on the basis of patient history and risk factors. C is incorrect. A patient with poor left ventricular ejection fraction has limited reserve to increase contractility in response to increased filling or decreased heart rate. Cardiac output and aortic root pressure will therefore be likely to drop in response to bradycardia, limiting any advantage in increased diastolic perfusion time. Finding the right balance may be difficult particularly if there is coexisting coronary artery disease. 6. D is correct. Several teams have shown a reduction in supraventricular dysrhythmias after cardiac surgery in groups that received thoracic epidural analgesia in addition to general anesthesia for cardiac surgery (mostly coronary artery bypass grafting) as compared to general anesthesia with systemic opiate analgesia. A is incorrect. Several centers around the world perform cardiac surgery using central neuraxial regional techniques without general anesthesia. The practice is currently rare in North America, Australasia, and Western Europe. B is incorrect. Thoracic epidural analgesia is listed as a class IIb recommendation (= “may be considered”) in the 2011 ACCF/AHA guidelines on coronary artery bypass grafting. C is incorrect. A systematic review and meta-analysis from 12 published cohorts, including more than 14,000 patients, suggested the maximum risk for transient neurological injury following the use of thoracic epidural to be 1 in 1700 for cardiac and vascular surgery. 7. C is correct. Uncomplicated lower extremity peripheral nerve blocks are associated with minimal hemodynamic disturbances. A is incorrect. There is a relationship between hypotension and myocardial ischemia but this does not preclude the

Hadzic_Ch46_p295-298.indd 297

Regional Anesthesia and Cardiovascular Disease

297

use of regional anesthesia and analgesia as long as large drops in systemic blood pressure are avoided. Poor-quality pain relief may increase the risk of myocardial ischemia due to tachycardia and catecholamine-mediated coronary vasospasm. B is incorrect. Patients with ischemic heart disease may experience a range of complications perioperatively including myocardial infarction and dysrhythmias. If anything, a lower dose of local anesthetic should be considered in order to minimize the risk of toxic effects. D is incorrect. Tachycardia increases myocardial oxygen demand while also reducing the time available for diastolic coronary artery perfusion. This will offset any advantage resulting from increased aortic root pressure, increasing the risk of myocardial ischemia in a patient with severe coronary artery disease. 8. B is correct. Severe aortic stenosis is a life-threatening condition and should be treated before nonurgent elective surgery. In an emergency situation, the risks vs. benefits of proceeding with noncardiac surgery would need to be considered and treatment planned on an individual basis. A is incorrect. Severe aortic stenosis usually coexists with left ventricular hypertrophy, resulting in diastolic dysfunction with preserved systolic function up until the point of decompensation. A combination of tachycardia, dehydration, and vasodilation can lead to hypotension resulting in cardiac arrest unresponsive to resuscitation. As such even minor procedures planned with peripheral nerve blockade would need to be undertaken with care and preferably delayed until treatment of the aortic valve disease. C is incorrect. In severe aortic stenosis, coronary artery perfusion pressure is dependent on maintenance of systemic vascular resistance. Profound vasodilation is poorly tolerated. D is incorrect. There are multiple case reports in the literature of successful management of emergency surgery with central neuraxial anesthesia in patients with severe aortic stenosis. A catheter technique with slow-onset of blockade, careful titration of local anesthetic agents, and close cardiovascular monitoring to optimize filling status and vasoconstrictor administration can be effective. 9. C is correct. Vasodilation is generally well tolerated in patients with isolated regurgitant valvular disease with a lessening of regurgitant fraction. Vasoconstriction leads to a worsening of regurgitant fraction; hence good-quality pain relief during labor would probably be beneficial. A is incorrect. Vasodilation is usually well tolerated by patients with normal coronary arteries and regurgitant valvular lesions. Lumbar epidural analgesia is not contraindicated and has been used successfully in obstetric patients with a variety of types of cardiac disease. B is incorrect. Moderate aortic regurgitation is not an indication for cardiac surgery, particularly while pregnant. The patient should be followed up in order to determine the optimal time for surgery if needed in the future.

22/04/19 2:07 PM

298

PART 7

Anesthesia in Patients with Specific Considerations

D is incorrect. If the patient had a severely dilated ascending aorta as the cause of her aortic regurgitation then advice in this circumstance should focus on avoidance of hypertension and straining during labor. Good-quality regional analgesia/anesthesia could be extremely useful to facilitate safe obstetric care.

Suggested Readings

Hadzic A. Regional anesthesia & cardiovascular disease. In: Burt CC, Littwin SM, Adebayo J, Mallavaram NA, Thys DM, eds. Textbook of Regional Anesthesia and Acute Pain Management. 2nd ed. New York, NY: McGraw-Hill Publishing; 2017:chap 47. Hillis LD, Smith PK, Anderson JL, et al. 2011 ACCF/AHA Guideline of for Coronary Artery Bypass Graft Surgery: A report of the American College of Cardiology Foundation/American Heart Association Task Force on Practice Guidelines. Circulation. 2011;124(23):e652-e735.

Goldszmidt E, Macarthur A, Silversides C, Colman J, Sermer M, Siu S. Anesthetic management of a consecutive cohort of women with heart disease for labor and delivery. Int J Obstet Anesth. 2010;19:266-272.

Hadzic_Ch46_p295-298.indd 298

22/04/19 2:07 PM

47 Regional Anesthesia and Systemic Disease Matt Levine

QUESTIONS DIRECTIONS: Choose the one best response to each question. 1. Lumbar neuraxial anesthesia results in the following pulmonary changes: A. Bronchoconstriction is frequently seen. B. Effort-dependent pulmonary function tests (eg, FEV1) are decreased. C. Partial pressure of CO2 rises and partial pressure of O2 falls. D. Tidal volume decreases with a compensatory rise in respiratory rate. 2. With regard to brachial plexus block and phrenic nerve blockade, which of the following is true? A. Phrenic nerve blockade is poorly tolerated by most patients, and if it occurs patients must remain in hospital for observation. B. Regional anesthesia for shoulder surgery is impossible without interscalene block, and therefore a high risk of phrenic nerve block. C. The risk of phrenic nerve block reduces with more distal approaches to brachial plexus blockade. D. Unilateral phrenic nerve blockade results in a 50% decrease in FEV1 and FVC. 3. Patients with chronic renal disease, when compared to healthy patients, have: A. Prolonged duration of peripheral nerve blocks, due to reduced clearance of local anesthetics by the kidney B. Reduced protein binding of bupivacaine, which may put them at increased risk of systemic toxicity C. Reduced response to lipid emulsion in the event that local anesthetic toxicity does occur D. Reduced risk of cardiovascular toxicity, due to hypokalemia and alkalosis

4. In patients with diabetic peripheral neuropathy: A. Current required to obtain muscle twitches is always higher than in healthy patients. B. Neurological injury following peripheral nerve block is common. C. Sensory blockade tends to be prolonged, but motor blockade shorter when compared to healthy patients. D. Sensory and motor blockade both tend to be prolonged when compared to healthy patients. 5. With regard to perioperative glucose homeostasis: A. Perioperative hyperglycemia is caused solely by reduced insulin release from the pancreas. B. Perioperative hyperglycemia is common, and of no clinical significance. C. Regional anesthesia can cause hyperglycemia, and is best avoided in diabetics. D. Regional anesthesia may blunt the stress response to surgery, and prevent the associated hyperglycemia. 6. Regional anesthesia in the obese patient: A. Can be challenging due to difficult landmark identification B. Has no advantages over general anesthesia C. Is contraindicated due to the high rate of serious complications D. Is no more likely to fail than in normal-weight patients 7. Apnea following high neuraxial anesthesia: A. Is of theoretical concern only, and never occurs in practice B. Is typically caused by brainstem hypoperfusion C. Necessitates intubation and mechanical ventilation D. Usually results from bilateral phrenic nerve block

299

Hadzic_Ch47_p299-302.indd 299

19/04/19 10:51 AM

300

PART 7

Anesthesia in Patients with Specific Considerations

8. Pneumothorax as a complication of brachial plexus block: A. Always requires intercostal drain insertion to prevent tension pneumothorax from developing B. Has disappeared since the introduction of ultrasound guidance C. Is likely underreported in the literature D. Is more common with the interscalene approach than other approaches 9. In patients with severe chronic liver disease and hepatic coagulopathy: A. Bleeding risk is the same for all approaches to the brachial plexus. B. Epidural placement is safe as long as the INR is < 1.5. C. Neuraxial anesthesia is absolutely contraindicated. D. Platelet abnormalities may exist in addition to clotting factor deficiency. 10. Thoracic epidural anesthesia in patients with severe chronic obstructive pulmonary disease (COPD): A. Is associated with a high risk of pneumothorax, which is likely to be poorly tolerated in this population B. Is contraindicated because intercostal muscle weakness will lead to respiratory failure C. May be used as an alternative to general anesthesia for mastectomy and open abdominal surgery D. May impair the cough reflex and lead to sputum retention and atelectasis following thoracotomy

ANSWERS AND EXPLANATIONS 1. B is correct. Effort-dependent pulmonary function tests such as FEV1, FVC, and PEFR are modestly decreased due to weakness of abdominal and chest wall muscles that are used during forced expiration. A is incorrect. Bronchoconstriction is a theoretical concern due to reduced sympathetic tone; however, this does not occur to any significant degree. C is incorrect. Partial pressure of CO2 and O2 are essentially unchanged. D is incorrect. Lumbar epidural anesthesia does not affect resting minute ventilation, tidal volume, or respiratory rate. 2. C is correct. The risk of phrenic nerve block is as high as 100% with the interscalene approach, although the risk may be reduced with low volumes of local anesthetic. The supraclavicular approach is typically associated with a 50% risk of phrenic nerve block, although this may be reduced significantly with ultrasound guidance. Infraclavicular (coracoid) and axillary approaches are sufficiently distal to avoid the phrenic nerve altogether. A is incorrect. Phrenic nerve blockade is of questionable significance in patients with good respiratory function,

Hadzic_Ch47_p299-302.indd 300

most of whom will remain asymptomatic or complain only of a mild, subjective sensation of dyspnea. B is incorrect. Interscalene block is typically used for patients undergoing shoulder surgery as it provides excellent analgesia. However, in patients who may not tolerate phrenic nerve blockade, alternative approaches can be used such as axillary nerve or infraclavicular brachial plexus blocks combined with suprascapular nerve block. D is incorrect. Unilateral phrenic nerve block results in paralysis of half of the diaphragm, which is the main muscle used in respiration. However, the reduction in FEV1 and FVC is only 27% because the other muscles involved (eg, intercostals) are not affected. 3. B is correct. Acidemia and hyperkalemia are frequently present in patients with chronic renal disease. Acidemia decreases the protein binding of bupivacaine, leaving a greater proportion of the drug unbound and therefore able to cause systemic toxicity. In addition, hyperkalemia reduces the amount of bupivacaine required to induce cardiovascular (but not neurological) toxicity. A is incorrect. Chronic renal disease does not appear to affect the latency, duration, or quality of peripheral nerve blocks. Local anesthetics are primarily cleared by the liver, not the kidney. C is incorrect. Lipid emulsion has been used successfully to treat local anesthetic toxicity in patients with chronic renal disease. There is no data to suggest that it will be less effective in these patients, and no dose adjustment is recommended. D is incorrect. Patients with chronic renal failure tend to have hyperkalemia and acidosis (not hypokalemia and alkalosis), which increases their risk of toxicity as described above. 4. D is correct. Prolonged sensory and motor blockade has been demonstrated in diabetics when compared to nondiabetics, and the duration of blockade increases as glycemic control worsens. A is incorrect. Although patients with diabetic neuropathy typically have reduced conduction velocity and amplitude in motor and sensory nerves, in clinical practice most patients have similar current intensity thresholds when compared to nondiabetics without neuropathy. B is incorrect. The true incidence of neurological injury in these patients is unknown, but thought to be rare due to the absence of reports in the literature. C is incorrect. Both sensory and motor blockade tend to be prolonged as described above. 5. D is correct. The so-called stress response to surgery is a complex phenomenon, one aspect of which is an increase in plasma glucose levels. By preventing nociceptive input to the central nervous and neuroendocrine systems, regional anesthesia can blunt this stress response and the associated hyperglycemia.

19/04/19 10:51 AM

CHAPTER 47

A is incorrect. The stress response to surgery and its effect on glucose homeostasis is complex, and involves numerous hormones and counterregulatory mechanisms. Insulin release from the pancreas may be reduced, but numerous other factors are at play, including increased cortisol, catecholamine, and glucagon secretion, all of which contribute to hyperglycemia.1 B is incorrect. Perioperative hyperglycemia is indeed common, but studies have shown that it can be of great clinical concern, leading to increased infection risk, prolonged hospital length of stay, and increased mortality. C is incorrect. Regional anesthesia does not cause hyperglycemia; rather it can prevent perioperative hyperglycemia as described above. 6. A is correct. Obesity has been shown to make it more difficult to identify lumbar spinal interspaces, palpate landmarks, and position patients, making regional anesthesia challenging in this patient population.2 B is incorrect. Although regional anesthesia may be challenging in this population group, it is an attractive alternative to general anesthesia as it may reduce the risk of many airway and cardiopulmonary complications. C is incorrect. A review of over 9000 blocks found that the complication rate was higher in obese patients than nonobese patients, but that the actual rate was very low in all groups regardless of BMI. D is incorrect. Obesity is a risk factor for block failure due to the challenges mentioned above. 7. B is correct. Brainstem hypoperfusion is the most common cause, and spontaneous respiration typically resumes following correction of hypotension with IV fluid and vasopressors. A is incorrect. Apnea is commonly reported as a consequence of high spinal anesthesia. C is incorrect. Spontaneous respiration often resumes following correction of hypotension, so intubation may not be required. Ventilation should be supported until this occurs. D is incorrect. Bilateral phrenic nerve block does not seem to be part of the mechanism; if this were the case, spontaneous respiration would not resume until the block had receded. 8. C is correct. Many pleural punctures will result in small pneumothoraces, which resolve spontaneously. A is incorrect. If the pneumothorax is small and asymptomatic it may be managed conservatively. However, large or symptomatic pneumothoraces may require drainage. B is incorrect. Ultrasound guidance seems to have reduced, but not eliminated, the risk of pneumothorax. A review by Gauss et al of over 6000 supraclavicular and infraclavicular blocks found a 0.06% incidence of symptomatic pneumothorax, although no effort was made to look for asymptomatic pneumothoraces.3

Hadzic_Ch47_p299-302.indd 301

Regional Anesthesia and Systemic Disease

301

D is incorrect. Pneumothorax is more common when the puncture site overlies the pleura (eg, supraclavicular block). 9. D is correct. Hepatic coagulopathy is associated with a complex range of abnormalities including reduced synthesis of procoagulant and anticoagulant proteins, thrombocytopenia, platelet dysfunction, and fibrinolysis. A is incorrect. The risk of bleeding is greater when the needle must be placed in the vicinity of a major blood vessel, particularly when that vessel is noncompressible (eg, subclavian artery). B is incorrect. The American Society of Regional Anesthesia guidelines state, “an INR less than 1.5 during initiation of warfarin therapy should be associated with normal hemostasis.” However, this may not be the case in the more complex setting of hepatic coagulopathy. Additional laboratory studies may be required to fully delineate the nature of the coagulopathy. In addition, platelet number and function must be considered.4 C is incorrect. Hepatic coagulopathy is not an absolute contraindication to neuraxial anesthesia. A careful assessment of risks and benefits must be made before deciding whether or not to perform a neuraxial block in this setting. 10. C is correct. Thoracic epidural anesthesia can be used as an alternative to general anesthesia for a variety of procedures, including mastectomy and various open abdominal procedures. This could be advantageous in patients with severe COPD who are at risk from pulmonary complications associated with general anesthesia. A is incorrect. Although pneumothorax has been reported in the literature as a complication of thoracic epidural placement, it appears to be very rare.5 B is incorrect. Intercostal muscle weakness often occurs as a result of thoracic epidural anesthesia. However, it has only a very mild impact on pulmonary dynamics and is well tolerated in this patient population. D is incorrect. Epidural anesthesia does not impair the cough reflex. Moreover, by providing good-quality analgesia, it can improve the ability of the postthoracotomy patient to breathe deeply, cough, clear secretions, and reduce atelectasis.6

References 1. Desborough, JP. The stress response to trauma and surgery. Br J Anaesth. July 2000;85(1):109-117. 2. Nielsen KC, Guller U, Steele SM, Klein SM, Greengrass RA, Pietrobon R. Influence of obesity on surgical regional anesthesia in the ambulatory setting: an Analysis of 9,038 blocks. Anesthesiology. 2005;102(1):181-187. 3. Gauss A, Tugtekin I, Georgieff M, Dinse-Lambracht A, Keipke D, Gorsewski G. Incidence of clinically symptomatic pneumothorax in ultrasound-guided infraclavicular and supraclavicular brachial plexus block. Anaesthesia. 2014 Apr;69(4):327-336. doi:10.1111/ anae.12586.

19/04/19 10:51 AM

302

PART 7

Anesthesia in Patients with Specific Considerations

4. Horlocker T, Wedel DJ, Rowlingson JC, et al. Regional anesthesia in the patient receiving antithrombotic or thrombolytic therapy: American Society of Regional Anesthesia and Pain Medicine Evidence-Based Guidelines (Third Edition). Reg Anesth Pain Med. 2010 Jan-Feb;35(1):64-101. 5. Zaugg M, Stoehr S, Weder W, Zollinger A. Accidental pleural puncture by a thoracic epidural catheter. Anaesthesia. 1998;53:69-78. 6. Mahon SV, Berry PD, Jackson M, Russell GN, Pennefather SH. Thoracic epidural infusions for post-thoracotomy pain: a comparison

Hadzic_Ch47_p299-302.indd 302

of fentanyl-bupivacaine mixtures vs. fentanyl alone. Anaesthesia. 1999;54:641-646.

Suggested Reading Hadzic A. Regional anesthesia & systemic disease. In: Latmore M, Levine M, Gadsden J, eds. Textbook of Regional Anesthesia and Acute Pain Management. 2nd ed. New York, NY: McGraw-Hill Publishing; 2017:chap 48.

19/04/19 10:51 AM

48 Regional Anesthesia in the Patient with Preexisting Neurologic Disease Evan Sutton

QUESTIONS DIRECTIONS: Choose the one best response to each question. 1. Which of the following describes the condition in which patients with preexisting neural compromise may become more susceptible to injury at another site when exposed to a secondary insult? A. Acquired peripheral neuropathy B. Central nervous system neuropathy C. Double-crush phenomenon D. Hereditary peripheral neuropathy 2. Which group of disorders is the most common inherited peripheral neuropathy? A. Charcot-Marie-Tooth disease B. Diabetic polyneuropathy C. Entrapment neuropathy D. Hereditary neuropathy with liability to pressure palsy 3. Which of the following is the most common cause of systemic polyneuropathy? A. Diabetic polyneuropathy B. Entrapment neuropathy C. Guillain-Barré syndrome D. Multiple sclerosis 4. Which disorder presents with distal symmetric sensorimotor polyneuropathy and is characterized by axonal degeneration, multifocal fiber loss, and often autonomic dysfunction? A. Chemotherapy-induced peripheral neuropathy B. Diabetic polyneuropathy C. Postpolio syndrome D. Postsurgical inflammatory neuropathy 5. Which acquired neurologic disorder is characterized by areflexia and diffuse ascending neuromuscular paralysis? A. Chemotherapy-induced peripheral neuropathy B. Diabetic polyneuropathy

C. Guillain-Barré syndrome D. Postsurgical inflammatory neuropathy 6. Which of the following is an inflammatory autoimmune disease characterized by focal demyelination within the central nervous system? A. Amyotrophic lateral sclerosis B. Diabetic polyneuropathy C. Multiple sclerosis D. Postpolio syndrome 7. Which of the following methods of regional anesthesia should be avoided in a patient with multiple sclerosis (MS) presenting for a unilateral total knee arthroplasty (TKA)? A. Adductor canal block for postoperative pain control B. Epidural analgesia for postoperative pain control C. Femoral nerve block for postoperative pain control D. Spinal anesthesia for surgical analgesia 8. Which of the following disorders presents with predominant motor dysfunction? A. Amyotrophic lateral sclerosis B. Multiple sclerosis C. Postpolio syndrome D. Spinal cord injury 9. True or False: Postpolio syndrome is an absolute contraindication to neuraxial anesthesia. A. True B. False 10. Patients with spinal stenosis or compressive lumbar disk disease may be at increased risk of neurologic complications following neuraxial blockade. The proposed mechanism(s) of injury include all of the following except: A. Ischemia B. Local anesthetic neurotoxicity C. Local anesthetic systemic toxicity D. Mechanical trauma

303

Hadzic_Ch48_p303-306.indd 303

19/04/19 10:52 AM

304

PART 7

Anesthesia in Patients with Specific Considerations

11. Which of the following anesthetics is best for preventing episodes of autonomic dysreflexia in patients with spinal cord injury? A. General anesthesia B. Monitored anesthesia care C. Spinal anesthesia D. Total intravenous anesthesia

ANSWERS AND EXPLANATIONS 1. C is correct. The “double-crush phenomenon” describes the condition in which patients with preexisting neural compromise are exposed to a secondary insult resulting in additive damage from each isolated injury. Unfortunately there is paucity of data to support the association of preexisting neurologic disease and postregional anesthesia dysfunction. A is incorrect. Acquired peripheral neuropathies are associated with trauma, toxins, or disease-related processes that result in inflammation and/or impaired blood flow. B is incorrect. Central nervous system neuropathies include a group of disorders that affect the neurological function of the brain and/or spinal cord. D is incorrect. Hereditary peripheral neuropathies represent a group of nerve disorders with a wide range of genotypes that typically present with gradual sensorimotor deficits over years to decades. 2. A is correct. Charcot-Marie-Tooth (CMT) neuropathies are caused from mutations in more than 30 genes responsible for manufacturing neurons or the myelin sheath. CMT disorders affect approximately 1 in 2500 people making it the most common inherited neuropathy. B is incorrect. Diabetic polyneuropathy is an acquired peripheral neuropathy that results in axonal degeneration secondary to the reduced delivery of blood, oxygen, and other essential nutrients. C is incorrect. Entrapment neuropathy occurs when a single nerve is chronically compressed or mechanically injured at a specific location. D is incorrect. Hereditary neuropathy with liability to pressure palsy (HNPP) is a rare inherited demyelinating peripheral neuropathy in which individuals suffer from repeated motor and sensory neuropathies following brief nerve compression or mild trauma (ie, pressure palsies). 3. A is correct. Diabetic polyneuropathy (DPN) is the most common cause of systemic polyneuropathy, affecting 4% to 8% of newly diagnosed diabetics, and more than 50% of patients with long-standing diabetes. B is incorrect. Entrapment neuropathy is one of the most prevalent disorders of the peripheral nervous system, and occurs when a single nerve is chronically compressed or mechanically injured at a specific location.

Hadzic_Ch48_p303-306.indd 304

C is incorrect. Guillain-Barré syndrome (GBS) is a rare inflammatory demyelinating polyneuropathy that is thought to occur from infection, pregnancy, vaccinations, immunosuppression, systemic illness, and/or transfusion of blood products. D is incorrect. Multiple sclerosis is an inflammatory autoimmune disorder of the central nervous system with a lifetime risk of 1 in 400 adults. 4. B is correct. Distal symmetric sensorimotor polyneuropathy is synonymous with diabetic polyneuropathy. The pathophysiology is thought to be due to axonal degeneration of small and eventually large nerve fibers secondary to the reduced delivery of essential nutrients, oxygen, and blood. The autonomic nervous system can also be affected. A is incorrect. Chemotherapy-induced peripheral neuropathy (CIPN) is a frequent side effect of several chemotherapeutic agents. Although CIPN can present in a “glove and stocking” distribution, the mechanism of injury is thought to involve damage to microtubules, mitochondrial disruption, and cytotoxic effects on DNA. C is incorrect. Postpolio syndrome (PPS) refers to new-onset neurologic symptoms that develop years after an acute poliomyelitis infection. The pathophysiology behind PPS is thought to be due to an ongoing process of denervation and reinnervation of peripheral motor nerves. Because sensory nerves are usually spared, initial symptoms of PPS include muscle weakness, fatigue, gait instability, and muscle atrophy. D is incorrect. Postsurgical inflammatory neuropathy (PSIN) describes an autoimmune or inflammatory process that may cause severe postoperative neurologic deficits. PSIN may present as focal, multifocal, or diffuse neurologic deficits after a surgical procedure and improves with intravenous immunoglobulin or high-dose corticosteroid immunosuppression. 5. C is correct. Guillain-Barré syndrome is an acute, inflammatory demyelinating polyneuropathy characterized by areflexia and diffuse ascending neuromuscular paralysis. The degree and distribution of paralysis is variable and can include sensory nerve, cranial nerve, and autonomic nervous system involvement. A is incorrect. Chemotherapy-induced neuropathy is a dose-limiting side effect of several commonly used chemotherapeutic agents. Symptoms are often in the “gloveand-stocking” distribution and consist of pain and/or paresthesias. B is incorrect. Distal symmetric sensorimotor polyneuropathy is synonymous with diabetic polyneuropathy. D is incorrect. Postsurgical inflammatory neuropathy is believed to be an idiopathic, immune-mediated response to a physiologic stress such as an infectious process, vaccination, or a surgical procedure. Pain and sensory deficits often improve before the resolution of motor deficits with immunosuppression therapy.

19/04/19 10:52 AM

CHAPTER 48

Regional Anesthesia in the Patient with Preexisting Neurologic Disease

6. C is correct. Multiple sclerosis (MS) is an inflammatory autoimmune disorder of the central nervous system (CNS) with a lifetime risk of 1 in 400, making it the most common debilitating neurologic disease in young adults. MS is characterized by focal demyelination within the CNS that causes a classic “waxing and waning” of symptoms including sensorimotor deficits, diplopia and/or vision loss, bowel and/or bladder dysfunction, and ataxia. A is incorrect. Amyotrophic lateral sclerosis is a progressive degenerative disease of both upper and lower motor neurons. The cause is unknown, but theories include glutamate excitotoxicity, oxidative stress, mitochondrial dysfunction, paraneoplastic tumors, autoimmune disease, and viral infection. B is incorrect. Diabetic polyneuropathy (DPN) is a disease of the peripheral and autonomic nervous systems. The pathophysiology behind DPN is thought to be due to axonal degeneration of small and large nerve fibers secondary to the reduced delivery of essential nutrients, oxygen, and blood. D is incorrect. Postpolio syndrome (PPS) develops several years after an acute poliomyelitis infection and affects the anterior horn cells of the spinal cord. PPS is considered a lower motor neuron disorder. The motor effects of PPS are thought to be related to an ongoing process of denervation ad reinnervation that ultimately ends when denervation is no longer compensated for by reinnervation. 7. D is correct. It is generally believed that spinal anesthesia can precipitate worsening neurologic deficits in patients with multiple sclerosis due to the neurotoxic effects of local anesthetics and the lack of myelin in the spinal cord. A is incorrect. The adductor canal block has been successfully used for postoperative pain control following total knee arthroplasty (TKA) for many years.1 Peripheral nerve blockade has not been definitively shown to be harmful in the setting of multiple sclerosis (MS). B is incorrect. The concentration of local anesthetic in the white matter of the spinal cord is four times higher after intrathecal injection compared to epidural injection. Consequently, epidural anesthesia and analgesia has traditionally been recommended over spinal anesthesia in MS patients. C is incorrect. The femoral nerve block (FNB), like the adductor canal block, has also been successfully used for postoperative pain control following TKA.1 As a peripheral nerve block, a FNB is considered a safe and effective way to provide analgesia to patients with multiple sclerosis. 8. C is correct. Postpolio syndrome (PPS) refers to new-onset neurologic symptoms that develop several years after an acute poliomyelitis infection. PPS affects the anterior horn cells of the spinal cord. Initial symptoms include muscle weakness, fatigue, gait instability, joint pain, and muscle atrophy. Sensory deficits are generally not characteristic of the syndrome and are only observed if a secondary disorder is present (eg, compressive radiculopathy or disk herniation). The motor effects of PPS are thought to be

Hadzic_Ch48_p303-306.indd 305

305

related to an ongoing process of denervation and reinnervation that ultimately ends when denervation is no longer compensated for by reinnervation. A is incorrect. Amyotrophic lateral sclerosis (ALS) is a progressive degenerative disease of upper and lower motor neurons. ALS can involve motor, sensory, and autonomic nerve dysfunction. B is incorrect. Multiple sclerosis is an inflammatory autoimmune disorder of the central nervous system characterized by focal demyelination within the spinal cord and brain. Signs and symptoms include motor, sensory, and autonomic nerve dysfunction. D is incorrect. Spinal cord injuries can result in complete or incomplete neurologic deficits involving motor and sensory nerves as well as the autonomic nervous system. 9. B (False) is correct. Although patients with postpolio syndrome (PPS) have fewer motor neurons than normal, there have been no reports of worsening neurologic status following neuraxial anesthesia with normal doses of tetracaine and bupivacaine. Importantly, the paucity of clinical data on this topic prevents clear recommendations from being made regarding the safety of neuraxial anesthesia or peripheral nerve blockade in patients with PPS. 10. C is correct. Local anesthetic systemic toxicity (LAST) is not a proposed mechanism of injury in this case. LAST is a manifestation of intravascular injection of large doses of local anesthetics with morbidity extending beyond a localized/compressive effect in patients with preexisting spinal stenosis or compressive lumbar disk disease. A is incorrect. Patients with spinal stenosis have a reduction in the diameter of the spinal canal resulting in less anatomic space for fluid collections. Consequently, small quantities of fluid may result in significant increases in pressure around the neuraxis that would have no clinical effect in a widely patent spinal canal. In large retrospective reviews, the majority of cauda equina cases involved epidural analgesia, which suggests an ischemic etiology secondary to mechanical compression of the spinal cord by an infusing local anesthetic. Moreover, patients with preexisting spinal stenosis or lumbar disk disease were found to have an increased risk for the development or worsening of neurologic deficits when compared to the general population undergoing a neuraxial technique. B is incorrect. Direct neurotoxicity from local anesthetic is another proposed mechanism of injury to patients with preexisting spinal stenosis and/or compressive lumbar disk disease. D is incorrect. Direct mechanical trauma to central nervous system tissue and/or surrounding vascular structures may result in a compressive hematoma and development or worsening of neurologic deficits in patients with preexisting spinal stenosis and/or compressive lumbar disk disease. 11. C is correct. Spinal (or epidural) anesthesia techniques can be valuable adjuncts in the management of patients

19/04/19 10:52 AM

306

PART 7

Anesthesia in Patients with Specific Considerations

undergoing surgical procedures below the level of the spinal cord injury. Numerous case reports and case series have demonstrated that neuraxial techniques are safe and effective in preventing episodes of autonomic dysreflexia in spinal cord injury patients, even those with high cord lesions. A is incorrect. General anesthesia with low-concentration volatile anesthetic does not offer protection against autonomic dysreflexia. Although higher concentrations of volatile anesthetic may be effective, anesthesia-related hemodynamic instability may not be well tolerated within this patient population. B is incorrect. Autonomic dysreflexia is an exaggerated sympathetic response to a surgical stimulus below the level of the spinal cord injury. Monitored anesthesia care (MAC) allows for the administration of a spectrum of anesthesia ranging from mild to deep sedation with preparedness to manage the hemodynamic and respiratory effects of general anesthesia. MAC is unlikely to achieve the appropriate depth of anesthesia required to blunt autonomic dysreflexia in response to a surgical stimulus. D is incorrect. Total intravenous anesthesia (TIVA) is a technique used to achieve general anesthesia using a combination of intravenous agents in the absence of

Hadzic_Ch48_p303-306.indd 306

inhalation agents. Although large doses of intravenous agents are capable of inducing deep general anesthesia and potentially preventing autonomic dysreflexia, this method may be associated with hemodynamic instability, which may not be tolerated in patients with spinal cord injury. Overall, general anesthesia (by way of TIVA or inhalation delivery) is not as reliable in preventing the exaggerated sympathetic response to surgical stimulation when compared to spinal anesthesia.

Reference 1. Kim DH, Lin Y, Goytizolo EA, et al. Adductor canal block versus femoral nerve block for total knee arthroplasty: a prospective, randomized, controlled trial. Anesthesiology. March 2014;120:540-550. doi:10.1097/ALN.0000000000000119

Suggested Reading Hadzic A. Regional anesthesia in the patient with preexisting neurologic disease. In: Jacob AK, Kopp SL, Hebl JR, eds. Textbook of Regional Anesthesia and Acute Pain Management. 2nd ed. New York, NY: McGraw-Hill Publishing; 2017:chap 49.

19/04/19 10:52 AM

49 Acute Compartment Syndrome of the Limb: Implications for Regional Anesthesia Sam Van Boxstael

QUESTIONS DIRECTIONS: Choose the one best response to each question. 1.  In which anatomic location does acute compartment syndrome (ACS) occur most often? A. Hand B. Lower leg and forearm C. Thigh D. Upper arm 2.  What is the only objective and accurate test to diagnose acute compartment syndrome? A. Arterial duplex of the affected limb B. Elevated serum creatinine kinase C. Computed tomography angiography of the affected limb D. Direct compartment pressure measurement in the suspected compartment 3.  From which compartmental pressure in a normotensive patient is emergent intervention indicated? A. 10 mm Hg B. 20 mm Hg C. 30 mm Hg D. 40 mm Hg 4.  Which of the following is a possible factor leading to compartment syndrome? A. Tight circumferential dressings B. Open fractures C. Reperfusion after prolonged periods of ischemia D. All of the above 5.  Which of the following is the most important clinical symptom in acute compartment syndrome (ACS)? A. Pain out of proportion to the injury B. Pallor C. Paresis D. Pulselessness

6.  Age is a strong predictor of the development of a compartment syndrome in patients with a proximal tibial shaft fracture. Which age group has the highest prevalence? A. 12–29 years of age B. 30–49 years of age C. 50–69 years of age D. > 70 years of age 7.  Which statement is true regarding the use of regional anesthesia and acute compartment syndrome (ACS)? A. It is safe to perform regional blocks in patients at risk of ACS. B. If ACS is clinically suspected, compartment pressure measurement without delay is mandatory. C. Patients are less likely to develop ACS when they have received regional anesthesia. D. None of the above

ANSWERS AND EXPLANATIONS 1. B is correct. ACS is most common in the lower leg and forearm. Upper leg muscles are at a lower risk for injury than are the smaller muscles of the lower leg, because the muscles of the thigh can dissipate the large forces of direct trauma. 2. D is correct. Changes in compartment pressures can precede the clinical signs of compartment syndrome. Compartment pressure measurement is the only objective and accurate test to diagnose and record compartment syndrome. 3. C is correct. Compartmental pressures greater than 30 mm Hg require emergent intervention because ischemia is imminent. 4.  A, B, and C are all possible factors, so option D is correct. See Table 49–1 for factors leading to compartment syndrome. 307

Hadzic_Ch49_p307-308.indd 307

22/04/19 6:14 PM

308

PART 7

Anesthesia in Patients with Specific Considerations

TABLE 49–1  Factors leading to compartment syndrome. Conditions that Increase the Compartment Volume • Direct soft tissue trauma with or without long bone fracture (10%–20% incidence after closed fracture) • Closed tibial shaft fractures (40%) and closed forearm fractures (12%) • Soft tissue crush injuries without fractures in 23% of cases of compartment syndrome1,2 • Open fractures, which should theoretically decompress the adjacent compartments, may lead to compartment syndrome3 • Hemorrhage: Vascular injury, coagulopathy • Anticoagulation therapy4 • Revascularization of limb after ischemia • High-energy trauma, as from high-speed motor vehicle accident or crush injury • Increased capillary permeability after burns (especially circumferential) • Infusions or high-pressure injections (eg, regional blocks, paint guns) • Extravasations of arthroscopic fluid (eg, after routine knee arthroscopy5) • Reperfusion after prolonged periods of ischemia • Anabolic steroid use, resulting in muscle hypertrophy6 • Decreased serum osmolarity (eg, nephritic syndrome7) • Strenuous exercise, especially in previously sedentary people Conditions that Lead to a Reduction in Volume of Tissue Compartments • Tight circumferential dressings (eg, can occur with cotton cast padding alone) • Closure of fascial defects8 • Cast or splint, especially if placed before removal of surgical tourniquet • Prolonged limb compression, as in Trendelenburg and lateral decubitus positions2,9 or in patients obtunded from alcohol or drug abuse • Excessive traction to fractured limbs10

5. A is correct. Pain out of proportion to the injury, especially with passive stretch of the muscles in the suspicious compartment or limb, is one of the most significant indicators of ACS. The other classic Ps of pallor, paresis, and pulselessness have very poor predictive value. Pallor and pulselessness rarely present in ACS. By the time paresis manifests, the damage is largely irreversible. 6. A is correct. The highest prevalence of ACS is between 12 and 29 years of age. 7. B is correct. If ACS is clinically suspected, compartment pressure measurement without delay is mandatory.

Hadzic_Ch49_p307-308.indd 308

Performing regional blocks in patients at risk of a compartment syndrome remains controversial. Several case reports and case series suggest that regional anesthesia may have delayed the diagnosis of ACS. In contrast, a number of cases and reviews suggest regional anesthesia may not mask timely diagnosis and, in fact, may even facilitate detection of ACS. Therefore, regional blocks should be performed in consultation with the patient and the surgical team. Astute management, compartment tissue monitoring, and perhaps low concentrations and volumes of local anesthetics should be considered.

References 1. Matsen F. Compartmental syndrome. A unified concept. Clin Orthop. 1975;113:8-14. 2. Botte M, Santi M, Prestianni C, Abrams R. Ischemic contracture of the foot and ankle: Principles of management and prevention. Orthopedics. 1996;19:235-244. 3. Ziv I, Mosheiff R, Zeligowski A, et al. Crush injuries of the foot with compartment syndrome: Immediate one-stage management. Foot Ankle. 1989;9:185-189. 4. Whitesides T, Harada H, Morimoto K. The response of skeletal muscle to temporary ischemia: An experimental study. J Bone Joint Surg. 1971;53A:1027-1028. 5. McQueen M, Gaston P, Court-Brown C. Acute compartment syndrome: Who is at risk? J Bone Joint Surg. 2000;82B:200-203. 6. Ulmer T. The clinical diagnosis of compartment syndrome of the lower leg: Are clinical findings predictive of the disorder? J Orthop Trauma. 2002;16:572-577. 7. Shuler FD, Dietz MJ. Physician’s ability to manually detect isolated elevations in leg intracompartmental pressure. J Bone Joint Surg Am. 2010;92:361-367. 8. McQueen MM. Acute compartment syndrome. In: Bucholz RW, Court-Brown CM, Heckman JD, Tornetta P 3rd (eds). Rockwood and Green’s Fractures in Adults, 7th ed. Philadelphia: Lippincott Williams & Wilkins; 2010:689-708. 9. Tighe PJ, Elliott CE, Lucas SD, et al. Noninvasive tissue oxygen saturation determined by near-infrared spectroscopy following peripheral nerve block. Acta Anesth Scand. 2011;55:1239-1246. 10. Elliott KG. Intramuscular pH as a novel diagnostic tool for acute compartment syndrome: A prospective clinical study [dissertation]. Aberdeen, Scotland: University of Aberdeen; 2007. uk.bl.ethos.485671.

Suggested Reading Hadzic A. Acute compartment syndrome of the limb: implications for regional anesthesia. In: Sala-Blanch X, de Andrés JA, Dewaele S, eds. Textbook of Regional Anesthesia and Acute Pain Management. 2nd ed. New York, NY: McGraw-Hill Publishing; 2017:chap 50.

22/04/19 6:14 PM

50 Peripheral Nerve Blocks for Outpatient Surgery Thibaut Vanneste

QUESTIONS DIRECTIONS: Choose the one best response to each question.

A. Heavy sedation is necessary. B. Premedicating patients judiciously while maintaining a meaningful patient contact may improve the patient’s experience. C. Sedation is cost effective. D. Sedation allows faster recovery.

1. With the use of regional anesthesia, what should be the first priority? A. Safety B. Efficiency C. Minimizing opioid and general anesthesia side effects D. Economics

6. A block team can increase: A. Operating room (OR) turnover time B. Block success C. Patient safety D. Recovery time

2. For what reason is multimodal analgesia important in ambulatory surgery? A. Avoiding side effects B. Prolonging hospital stay C. Additive positive effects D. Slow recovery

7. Regional anesthesia has been demonstrated to improve outcomes such as: A. Renal replacement therapy B. Bladder dysfunction C. Seizures D. Mortality

3. Concerning multimodal analgesia, which statement is true?1 A. High-dose opioids are a cornerstone in the multimodal treatment strategy. B. Locoregional anesthesia is a cornerstone in the multimodal treatment strategy. C. The use of locoregional anesthesia will not reduce the postoperative need for opioids. D. Locoregional anesthesia is not a part of the multimodal treatment strategy.

8. Which of the following is not a symptom of local anesthetic systemic toxicity (LAST)? A. Cardiac dysrhythmias occurring 4 hours after injection B. Tinnitus C. Seizures D. Coma

4. Which of the following adjuvants enhances and prolongs analgesia when injected intravenously? A. Buprenorphine B. Clonidine C. Dexamethasone D. Midazolam 5. Regarding sedation for ambulatory peripheral nerve blocks (PNBs), which statement is true?

9. Treatment of local anesthetic systemic toxicity (LAST) includes all except: A. Airway management B. Hyperventilation C. Treating seizures D. Intravenous lipid emulsion therapy 10. Complications of regional anesthesia can be avoided by using: A. Nerve stimulation B. Injection pressure monitoring C. Ultrasound D. All of the above

309

Hadzic_Ch50_p309-312.indd 309

19/04/19 10:53 AM

310

PART 7

Anesthesia in Patients with Specific Considerations

ANSWERS AND EXPLANATIONS 1. A is correct. With the use of regional anesthesia techniques, the first priority should be on safety. B, C, and D are incorrect. The second priority should be reducing patient pain and minimizing opioid and general anesthesia (GA) side effects. The third priority should be economics, and the fourth efficiency.2,3 2. A is correct. Well-planned multimodal analgesia is important in the ambulatory surgery setting for several reasons. First, patients prefer to avoid side effects such as nausea, vomiting, and somnolence.4,5 B is incorrect. Uncontrolled side effects can result in unplanned hospital admission, which is costly and inconvenient for all parties. C is incorrect. The use of multimodal analgesia has the potential for real synergistic positive effects while also reducing the likelihood of serious side effects.6 D is incorrect. Multimodal analgesic regimens improve the likelihood of early recovery.7 3. B is correct. In an outpatient setting, regional anesthesia with peripheral nerve blocks (PNBs) is a cornerstone of multimodal analgesia and opioid-sparing strategies. A is incorrect. High-dose opioids can cause hyperalgesia by activating neurons and glial cells.8 C is incorrect. Potent opioids, such as remifentanil and fentanyl, can cause hyperalgesia and rapid tolerance in humans.9 Other undesirable side effects are nausea, somnolence, and respiratory depression, all of which are highly undesirable in the outpatient setting. D is incorrect. In an outpatient setting, regional anesthesia with peripheral nerve blocks (PNBs) is a cornerstone of multimodal analgesia and opioid-sparing strategies. 4. C is correct. Dexamethasone is an adjuvant that can enhance or prolong the duration of analgesia when injected perineurally or intravenously.10-15 A, B, and D are incorrect. Other adjuvants can prolong analgesia duration when injected perineurally with single-injection peripheral nerve blocks (PNBs). 5. B is correct. Many patients undergoing surgical procedures experience preoperative anxiety.16-18 In an effort to improve the perioperative experience, short-acting anxiolytic agents and/or other medications (eg, midazolam, opioids, ketamine, gabapentinoids) are often given before a PNB is placed.19,20 Premedicating patients judiciously while maintaining a meaningful patient contact may improve the patient’s experience and allow for beneficial patient feedback during the procedure. A is incorrect. Heavy premedication may lead to loss of airway tone and subsequent hypoxia and hypercapnia.21 C is incorrect. From a cost perspective, premedicated patients require bedside monitoring, and more personnel may be required depending on the physical layout, surgical case workload, and current staffing levels.

Hadzic_Ch50_p309-312.indd 310

D is incorrect. Heavily premedicated patients may take longer to recover and need to remain within the surgical center longer,22 resulting in patient dissatisfaction. 6. C is correct. A multidisciplinary group at Duke University initiated a “block nurse” team in 2010. Block nurses completed a focused training program that emphasized patient flow, educating and preparing patients, assisting anesthesiologists, monitoring patients, and enhancing safety during the pre, intra-, and post-procedure periods. Within a year, rapid OR turnover time increased by 26%, OR on-time starts increased by 7%, orthopedic cases receiving preoperative blocks increased by 19%, and patient safety (eg, no wrong-sided blocks) improved.23 A, B, and D are incorrect. See explanation for answer C. 7. D is correct. Improved outcomes from neuraxial use in the outpatient surgery setting include improved shortterm and long-term pain control, reduced surgical stress, improved gastrointestinal function, lower rates of postoperative nausea and vomiting, and fewer unplanned admissions or readmissions.24-27 More physiologically significant outcomes include reductions in perioperative myocardial infarction, pulmonary complications, and mortality.28,29 A, B, and C are incorrect. See explanation for answer D.



8. A is correct. Cardiac dysrhythmias occurring 4 hours after injection is not a symptom of local anesthetic systemic toxicity (LAST). Cardiac dysrhythmias can occur as late as 30–75 minutes after the injection of local anesthetics and may be intractable despite adherence to advanced cardiac life support guidelines.30 This delayed response highlights the need for continuous monitoring of patients in the block area as well as the operating room. B, C, and D are incorrect. Symptoms of LAST can be tinnitus, perioral numbness, metallic taste, mental status changes or anxiety, visual changes, muscle twitching, and ultimately, seizures. Although uncommon, LAST can be a life-threatening complication for patients. 9. B is correct. Hyperventilation is not part of the treatment of LAST. Guidelines on the care of patients with LAST include airway management, prevent hyperventilation (to avoid increase in seizure threshold), treating a seizure if it occurs, intravenous lipid emulsion therapy, and transfer to a tertiary care setting with cardiopulmonary bypass capability.31 Ambulatory facilities should consider creating LAST kits with checklists that are housed in the areas where blocks are performed.32 A, C, and D are incorrect. Airway management, treating seizures, and intravenous lipid emulsion therapy are part of the treatment of LAST. 10. D is correct. The development of more objective methods of monitoring needle placement and administering local anesthetics, such as ultrasound guidance, low-current nerve stimulation, and opening injection pressure monitoring are likely to even further decrease the risk of intrafascicular injection and neurologic injury.33-35 When

19/04/19 10:53 AM

CHAPTER 50

an intrafascicular injection is present there will be a high opening injection pressure and patients often report pain. Also, logically, the risk of permanent neural damage is higher when injecting higher volumes of local anesthetics. Confirming the absence of motor response to low-current nerve stimulation gives the additional security of avoiding intrafascicular injection.

References 1. Kettner SC, Willschke H, Marhofer P. Does regional anaesthesia really improve outcome? Br J Anaesth. 2011;107(Suppl 1): i90-i95. 2. Boezaart AP, Tighe PJ. The progression of regional anesthesia into acute and perioperative pain medicine. Int Anesthesiol Clin. 2011;49:104-109. 3. Lee F. If Disney ran your hospital: 9-1/2 things you would do differently. Swanson G, ed. Bozeman, MT: Second River Healthcare; 2004:28. 4. Williams BA, Kentor ML, Vogt MT, et al. Potential hospital cost savings via associated postanesthesia care unit bypass and sameday discharge. Anesthesiology. 2004;100:697-706. 5. Macario A, Weinger M, Truong P, et al. Which clinical anesthesia outcomes are both common and important to avoid? The perspective of a panel of expert anesthesiologists. Anesth Analg. 1999;88:1085-1091. 6. Joshi GP. Multimodal analgesia techniques for ambulatory surgery. Int Anesthesiol Clin. 2005;43:197-204. 7. Young A, Buvanendran A. Multimodal systemic and intraarticular analgesics. Int Anesthesiol Clin. 2011;49:117-133. 8. Richebé P, Rivat C, Laulin JP, et al. Ketamine improves the management of exaggerated postoperative pain observed in perioperative fentanyl-treated rats. Anesthesiology. 2005;102:421-428. 9. Chia YY, Liu K, Wang, JJ, et al. Intraoperative high dose fentanyl induces postoperative fentanyl tolerance. Can J Anaesth. 1999;46:872-877. 10. Rasmussen SB, Saied NN, Bowens C, et al. Duration of upper and lower extremity peripheral nerve blockade is prolonged with dexamethasone when added to ropivacaine: a retrospective database analysis. Pain Med. 2013;14:1239-1247. 11. Desmet M, Braems H, Reynvoet M, et al. I.V. and perineural dexamethasone are equivalent in increasing the analgesic duration of a single-shot interscalene block with ropivacaine for shoulder surgery: a prospective, randomized, placebo-controlled study. Br J Anaesth. 2013;111:445-452. 12. Parrington SJ, O’Donnell D, Chan VW, et al. Dexamethasone added to mepivacaine prolongs the duration of analgesia after supraclavicular brachial plexus blockade. Reg Anesth Pain Med. 2010;35:422-426. 13. Vieira PA, Pulai I, Tsao GC, et al. Dexamethasone with bupivacaine increases duration of analgesia in ultrasound-guided interscalene brachial plexus blockade. Eur J Anaesthesiol. 2010;27:285-288. 14. Cummings KC III, Napierkowski DE, Parra-Sanchez I, et al. Effect of dexamethasone on the duration of interscalene nerve blocks with ropivacaine or bupivacaine. Br J Anaesth. 2011;107:446-453. 15. Abdallah FW, Johnson J, Chan V, et al. Intravenous dexamethasone and perineural dexamethasone similarly prolong the

Hadzic_Ch50_p309-312.indd 311

311

Peripheral Nerve Blocks for Outpatient Surgery

duration of analgesia after supraclavicular brachial plexus block: a randomized, triple-arm, double-blind, placebo-controlled trial. Reg Anesth Pain Med. 2015;40:125-132. 16. Lichtor JL, Johanson CE, Mhoon D, et al. Preoperative anxiety: Does anxiety level the afternoon before surgery predict anxiety level just before surgery? Anesthesiology. 1987;67:595-598. 17. Brand LR, Munroe DJ, Gavin J. The effect of hand massage on preoperative anxiety in ambulatory surgery patients. AORN J. 2013;97:708-717. 18. Norris W, Baird WLM. Pre-operative anxiety: A study of the incidence and aetiology. Br J Anaesth. 1967;39:503-509. 19. White PF, Tufanogullari B, Taylor J, et al. The effect of pregabalin on preoperative anxiety and sedation levels: a dose-ranging study. Anesth Analg. 2009;108:1140-1145. 20. Shafer A, White PF, Urquhart ML, et al. Outpatient premedication: use of midazolam and opioid analgesics. Anesthesiology. 1989;71:495-501. 21. Eichhorn V, Henzler D, Murphy MF. Standardizing care and monitoring for anesthesia or procedural sedation delivered outside the operating room. Curr Opin Anaesthesiol. 2010;23:494-499. 22. Viitanen H, Annila P, Viitanen M, et al. Premedication with midazolam delays recovery after ambulatory sevoflurane anesthesia in children. Anesth Analg. 1999;89:75. 23. Cullen KA, Hall MJ, Golosinskiy A. Ambulatory surgery in the United States, 2006. Natl Health Stat Report. 2009;28:1-25. 24. Williams BA, Kentor ML, Williams JP, et al. PACU bypass after outpatient knee surgery is associated with fewer unplanned hospital admissions but more phase II nursing interventions. Anesthesiology. 2002;97:981-988. 25. Carli F, Kehlet H, Baldini G, et al. Evidence basis for regional anesthesia in multidisciplinary fast-track surgical care pathways. Reg Anesth Pain Med. 2011;36:63-72. 26. Liu SS, Strodtbeck WM, Richman JM, et al. A comparison of regional versus general anesthesia for ambulatory anesthesia: A meta-analysis of randomized controlled trials. Anesth Analg. 2005;101:1634-1642. 27. Williams B, Bottegal M, Kentor M, et al. Rebound pain scores as a function of femoral nerve block duration after anterior cruciate ligament reconstruction: retrospective analysis of a prospective, randomized clinical trial. Reg Anesth Pain Med. 2007;32:186-192. 28. Rodgers A, Walker N, Schug S, et al. Reduction of postoperative mortality and morbidity with epidural or spinal anaesthesia: results from overview of randomised trials. BMJ. 2000;321:1493-1504. 29. Bonnet F, Maret E. Influence of anaesthetic and analgesic techniques on outcome after surgery. Br J Anaesth. 2005;95:52-58. 30. Dix SK, Rosner GF, Nayar M, et al. Intractable cardiac arrest due to lidocaine toxicity successfully resuscitated with lipid emulsion. Crit Care Med. 2011;39:872-874. 31. Neal JM, Mulroy MF, Weinberg GL. American Society of Regional Anesthesia and Pain Medicine checklist for managing local anesthetic systemic toxicity: 2012 version. Reg Anesth Pain Med. 2012;37:16-18. 32. Ben-David B. Complications of regional anesthesia: an overview. Anesthesiol Clin North America. 2002;20:665-667. 33. Ip VH, Tsui BC. Practical concepts in the monitoring of injection pressures during peripheral nerve blocks. Int Anesthesiol Clin. 2011;49:67-80.

19/04/19 10:53 AM

312

PART 7

Anesthesia in Patients with Specific Considerations

34. Tsui BC, Knezevich MP, Pillay JJ. Reduced injection pressures using a compressed air injection technique (CAIT): an in vitro study. Reg Anesth Pain Med. 2008;33:168-173. 35. Gadsden JC, Choi JJ, Lin E, Robinson A. Opening injection pressure consistently detects needle-nerve contact during ultrasound-guided interscalene brachial plexus block. Anesthesiology. 2014;120:1246-1253.

Hadzic_Ch50_p309-312.indd 312

Suggested Reading Hadzic A. Peripheral nerve blocks for outpatient surgery. In: Spofford CM, Foldes P, Laur J, eds. Textbook of Regional Anesthesia and Acute Pain Management. 2nd ed. New York, NY: McGraw-Hill Publishing; 2017:chap 51.

19/04/19 10:53 AM

51 Neuraxial Anesthesia and Peripheral Nerve Blocks in Patients on Anticoagulants Tom C. Van Zundert

QUESTIONS DIRECTIONS: Choose the one best response to each question. 1. In which subgroup of patients is the risk of neuraxial hematoma formation after neuraxial blockade the lowest? A. Elderly patients B. Obstetric patients C. Patients using aspirin D. Patients with abnormal coagulation 2. Which of the following medications exerts its anticoagulant properties by blocking the P2Y12 receptor on platelets? A. Aspirin B. Celecoxib C. Ticlopidine D. Tirofiban

6. Time from last heparin dose to save removal of an epidural catheter in patients on heparin therapy is: A. 12 hours when low-molecular-weight heparin (LMWH) is used prophylactically B. 12 hours when low-molecular-weight heparin (LMWH) is used therapeutically C. 1 hour when heparin is used intravenously D. Not applicable, because heparin therapy is a contraindication for having an epidural catheter in situ 7. Which of the direct oral anticoagulants (DOACs) has the highest renal elimination? A. Apixaban B. Dabigatran C. Rivaroxaban D. The above DOACs are hardly, if at all, eliminated by the kidneys.

4. Which of the following platelet function tests has good screening capabilities for von Willebrand defects as well as for antiplatelet medication? A. Multiple-platelet aggregometry test (Multiplate) B. Platelet function analyzer (PFA) C. Thromboelastography (TEG) platelet mapping D. VerifyNow

8. Managing patients on direct oral anticoagulant (DOAC) therapy who could benefit from a neuraxial technique can be challenging for what reason? A. Because of the DOAC use, a neuraxial technique should not be employed within 5 days of last use. B. In low-risk venous thromboembolism (VTE) or stroke patients, withholding of the DOAC for a maximum of two to three half-life intervals is advised. C. Restarting DOAC therapy after neuraxial catheter removal, is allowed after 8 hours (the time for a clot to stabilize). D. Stopping five half-life intervals and bridging with low-molecular-weight heparin (LMWH) therapy is a good method in most patients.

5. The effect of vitamin K antagonists (such as warfarin) on coagulation can best be measured using the: A. Activated partial thromboplastin time (aPTT), which is prolonged due to early inhibition of factor VII synthesis B. Activated partial thromboplastin time (aPTT), which is prolonged due to early inhibition of factor X synthesis C. Prothrombin time (PT), which is prolonged due to early inhibition of factor VII synthesis D. Prothrombin time (PT), which is prolonged due to early inhibition of factor X synthesis

9. Which statement is true concerning the management of neuraxial hematoma? A. A watchful waiting strategy is advised, as normally spontaneous recovery occurs. B. In suspected cases, CT is the preferred imaging modality. C. Occurrence of hematoma after the first 24 hours of neuraxial blockade is rare. D. Return of motor weakness and/or sensory deficit after complete block resolution is highly suggestive of a neuraxial hematoma.

3. Concerning the stopping of antiplatelet medication before neuraxial blockade: A. Clopidogrel is recommended to be stopped for 3 days. B. Prasugrel is recommended to be stopped for 5 days. C. Ticagrelor is recommended to be stopped for 7 days. D. Ticlopidine is recommended to be stopped for 10-14 days.

Hadzic_Ch51_p313-316.indd 313

313

22/04/19 2:10 PM

314

PART 7

Anesthesia in Patients with Specific Considerations

10. When performing peripheral nerve blocks (PNBs) in anticoagulated patients: A. All the three international guidelines recommend to follow the same precautions as with neuraxial blockades. B. The American Society of Regional Anesthesia and Pain Medicine (ASRA) (American guidelines) recommend to follow the same precautions as with neuraxial blockades. C. The ESA (European guidelines) recommend to follow the same precautions as with neuraxial blockades. D. The ÖGARI (Austrian guidelines) recommend to follow the same precautions as with neuraxial blockades.

ANSWERS AND EXPLANATIONS 1. B is correct. Obstetric patients have the lowest risk of neuraxial hematoma formation following neuraxial blockade. Spinal hematoma is seen in 1:200,000 cases. Protective mechanisms include: hypercoagulable state of pregnancy, wider capacity of epidural space, and higher intra-epidural pressure. A is incorrect. Elderly have increased risk of degenerative spine abnormalities, osteoporosis, and peripheral vascular disease. Risk is estimated at 1:3800. C is incorrect. Although in most cases aspirin use is not a contraindication for neuraxial blockade, the risk is evidently higher in this subgroup when compared to overall risk of hematoma formation after neuraxial blockade (ranging from 1:2700 to 1:19,505). D is incorrect. Patients with abnormal coagulation profiles have a high risk of hematoma formation (estimated at 1:315). 2. C is correct. By blocking the P2Y12 receptor with ticlopidine, adenosine diphosphate (ADP) is not able to activate platelets through this receptor. Other drugs in this category are clopidogrel, prasugrel, ticagrelor, and cangrelor. A is incorrect. Aspirin blocks platelet function by inhibiting COX-1 and subsequent TXA2 formation. B is incorrect. Celecoxib is a selective COX-2 inhibitor, which has no antiplatelet effects. D is incorrect. Tirofiban blocks the αIIbβ3 receptor on platelets. Platelet-to-platelet aggregation mediated by fibrinogen is hereby abolished. 3. D is correct. Ticlopidine is recommended to be stopped for 10 to 14 days by several international societies. Ticlopidine exhibits delayed reactions on platelet function. Platelet function appears to be inhibited after 24 to 48 hours of ingestion. Its effect can last up to 2 weeks after reaching a steady state. A is incorrect. Clopidogrel should be stopped for preferably 7 days; however 5 days is deemed an acceptable alternative if platelet function testing shows absence of platelet inhibition. Inhibition rates are around 60%-70%.

Hadzic_Ch51_p313-316.indd 314

B is incorrect. Recommended duration of prasugrel withdrawal before neuraxial injection is 7 to 10 days. This is mainly based on the irreversible effect of prasugrel together with a high rate of inhibition (90%) of platelets. C is incorrect. Ticagrelor also exhibits 90% inhibition of platelet function. However, because of the reversible binding of ticagrelor, a shorter interval of 5 days is recommended between drug intake and neuraxial blockade. 4. B is correct. PFA is the only test of the four that incorporates flow in its measuring principle. This flow results in high shear stress and will unravel the von Willebrand protein. Thereby the von Willebrand protein is opened up and it will be possible to bind and activate platelets via the GPIb-IX-V receptor. A is incorrect. Multiplate does not use flow in its test. Platelets are activated on metal rods using specific agonists for the various platelet receptors in a cuvette with a magnetic stirrer. C is incorrect. TEG is insensitive for von Willebrand defects as no flow is present in the method. Viscoelastic methods such as TEG first initiate the coagulation cascade. The formed thrombin will then activate platelets via protease-activated receptors (PARs). D is incorrect. In the VerifyNow assay, blood is mixed and activated in specific cartridges. Platelet clumping occurs, which changes the optical density of the blood. This change in optical density is a measure of platelet inhibition by antiplatelet medication. 5. C is correct. Warfarin inhibits the vitamin K–dependent clotting factors (II, VII, IX, X) as well as the anticoagulant proteins C and S. The first factor to be affected by warfarin is factor VII (and protein C) because of its short half-life time. Factor VII is activated in the extrinsic pathway by adding tissue factor to the PT assay. A is incorrect. Although factor VII is the first attenuated coagulation protein after warfarin use, the aPTT is designed for monitoring of heparin and not of vitamin K antagonists. B is incorrect. Both statements are wrong. However, factor X is also a vitamin K–dependent coagulation factor. It is part of the final common pathway that can be activated via both the intrinsic as well as the extrinsic pathway. Note that, severe deficiencies in factor X (or in factor V, which is the cofactor of factor X) will lead to prolongations of both PT and aPTT. D is incorrect. It is correct that the PT should be used for warfarin monitoring, but factor X is not the first factor responsible for this effect. 6. A is correct. It is recommended to stop prophylactic LMWH 12 hours and therapeutic LMWH 24 hours before epidural catheter insertion. Before removing the catheter, the same intervals apply. Resuming LMWH therapy after removal of the epidural catheter can be done after 4 hours.

22/04/19 2:10 PM

CHAPTER 51

B is incorrect. Time from last LMWH dose when used therapeutically is 24 hours. Resuming LMWH therapy after removal of the epidural catheter is 4 hours for both prophylactic and therapeutic dosages. C is incorrect. Removal of the catheter can be done after stopping the intravenous therapy for 4 hours. Intravenous heparinization can be safely employed when waiting 1 hour after uncomplicated neuraxial catheter placement. Monitoring of the aPTT or activated clotting time (ACT) is advised in order not to overdose patients. D is incorrect. Heparin therapy is not a contraindication in patients with indwelling epidural catheters. Careful follow-up of the neurological state is advised if patients remain on LMWH therapy while the catheter is still in situ. 7. B is correct. Dabigatran is primarily dependent on renal elimination. At 80%, it has the highest renal clearance. A is incorrect. Apixaban is eliminated by the kidneys by 24% to 29%. C is incorrect. Rivaroxaban is eliminated by the kidneys by one-third. D is incorrect. All the three DOACs are eliminated to a certain extent by the kidneys. Thus in patients with renal impairment, dose and/or time adjustments need to be made. 8. D is correct. This seems to be a good alternative in most, if not all, patients on DOACs. Laboratory monitoring can help decide when it is the safest moment to perform neuraxial techniques. A is incorrect. Recommendations for last DOAC dose and neuraxial injection are based on five half-life intervals and not days. For the three DOACs, these five half-life intervals are 85 hours (dabigatran), 65 hours (rivaroxaban), and 75 hours (apixaban). B is incorrect. When withholding DOACs for two to three half-life intervals, the risk of neuraxial hematoma may be too high. Only in high-risk VTE or stroke patients, the thromboembolic risk may outweigh the neuraxial hematoma risk. C is incorrect. The statement that a clot needs 8 hours to stabilize is correct. However, DOACs can be given earlier, because the time to reach peak effect is around 2 hours. The safer option, however, is waiting 24 hours, but this might give higher VTE or stroke risk. 9. D is correct. Warning signs include: sudden, severe, constant back pain, which might aggravate with sneezing or straining. Other signs are loss of bladder or bowel function. Therefore, drugs that minimize sensory and motor blockade should be used. A is incorrect. Recovery without surgical intervention is rare. Functional recovery is better when time between onset of symptoms and intervention is shorter. Surgical evacuation within 12 hours from the onset of motor deficit might lead to the best outcome. B is incorrect. CT scanning can be used in patients with contraindications for MRI. But preferably an MRI should be performed as soon as possible in suspected cases.

Hadzic_Ch51_p313-316.indd 315

315

Neuraxial Anesthesia and Peripheral Nerve Blocks in Patients on Anticoagulants

C is incorrect. Indeed, most of the cases will be apparent in the first 24 hours. However, in approximately 25% of the cases, symptoms occur between 24 and 72 hours. In the remainder this can be up to weeks or months. 10. B is correct. It is correct that the current ASRA guidelines recommend to use to same guidelines in PNBs as in neuraxial blocks. This third guideline dates from 2010. The next revision might be less conservative. A is incorrect. Some guidelines distinguish between deep and superficial blocks. Deep blocks, such as lumbar plexus and visceral sympathetic blocks, carry the risk of retroperitoneal hematoma and should follow neuraxial rules. Superficial nerve blocks, especially when ultrasound-guided is used, probably are safe in case of residual anticoagulation. C is incorrect. It is stated by the ESA that the recommendations for neuraxial blocks do not routinely apply to PNBs. D is incorrect. The Austrian guidelines state that superficial nerve blocks can be safely performed in the presence of anticoagulants.

Suggested Readings Connolly G, Spyropoulos AC. Practical issues, limitations, and periprocedural management of the NOACs. J Thromb Thrombolysis. 2013;36:212-222. FDA report on ticlopidine. https://www.accessdata.fda.gov/ drugsatfdadocs/nda/2001/19-979S018Ticlidprntlbl.pdf. Accessed September 26, 2017. Gremmel T, Frelinger AL 3rd, Michelson AD. Platelet physiology. Semin Thromb Hemost. 2016 Apr;42(3):191-204. Hadzic A. Neuraxial anesthesia & peripheral nerve blocks in patients on anticoagulants. In: Benzon HT, Jabri RS, Van Zundert TC, eds. Textbook of Regional Anesthesia and Acute Pain Management. 2nd ed. New York, NY: McGraw-Hill Publishing; 2017:chap 52. Horlocker TT, Wedel DJ, Rowlingson JC, et al. Regional anesthesia in the patient receiving antithrombotic or thrombolytic therapy: American Society of Regional Anesthesia and Pain Medicine Evidence-Based Guidelines. 3rd ed. Reg Anesth Pain Med. 2010 Jan-Feb;35(1):64-101. Lagerkranser M, Lindquist C. Neuraxial blocks and spinal haematoma: review of 166 cases published 1994-2015. Part 2: diagnosis, treatment, and outcome. Scand J Pain. 2017 Apr;15():130-136. Liew A, Douketis J. Perioperative management of patients who are receiving a novel oral anticoagulant. Intern Emerg Med. 2013;8:477-484. Narouze S, et al. Interventional spine and pain procedures in patients on antiplatelet and anticoagulant medications: guidelines from the American Society of Regional Anesthesia and Pain Medicine, the European Society of Regional Anaesthesia and Pain Therapy, the American Academy of Pain Medicine, the International Neuromodulation Society, the North American Neuromodulation Society, and the World Institute of Pain. Reg Anesth Pain Med. 2015 May-Jun;40(3):182-212. Paniccia R, Priora R, Liotta AA, Abbate R. Platelet function tests: a comparative review. Vasc Health Risk Manag. 2015 Feb 18; 11():133-148.

22/04/19 2:10 PM

This page intentionally left blank

Hadzic_FM_p00i-pxx.indd 6

22/04/19 6:40 PM

52 Regional Analgesia in the Critically Ill Sebastian Schulz-Stübner

QUESTIONS DIRECTIONS: Choose the one best response to each question.

D. Patient-controlled regional analgesia is not feasible in the ICU. E. Large amounts of data from registry studies in the critically ill prove the risk-free use of regional analgesia catheters.

1.  Which statement is true regarding epidural catheter use in critically ill patients? A. The most common side effects of an epidural are hypertension and tachycardia. B. Neurologic assessment always needs to be performed by evoked potentials. C. Continuous infusion of local anesthetics causes more pronounced hemodynamic changes than intermittent bolus doses. D. Coagulopathy or anticoagulation causes a serious bleeding risk while an epidural is in place. E. Major indications for epidural catheters are chest trauma, thoracic and abdominal surgery, and paralytic ileus.

4. Which statement is true regarding peripheral nerve block for the lower extremities in the critically ill patients? A. Femoral nerve catheters are helpful in the management of acute pain from femoral neck fractures in the period between injury to shortly after surgical stabilization of the fracture. B. A continuous femoral catheter in combination with a sciatic block provides analgesia for the entire lower extremity. C. Postoperative care personnel should be specifically trained in handling regional analgesia catheters and must be aware of the potential complications and their early warning signs. D. All the above are true statements.

2.  Which statement is true regarding peripheral nerve catheters in critically ill patients? A. Continuous interscalene or cervical paravertebral blocks provide excellent analgesia for rib fractures. B. An interscalene brachial plexus block results in the loss of hemidiaphragmatic function. C. Guidance by eliciting paresthesia is the safest way for nerve blocks in the ICU. D. Central neuraxial blocks in heavily sedated patients are good for junior resident training because of less anxiety. E. A femoral nerve block provides good analgesia for a broken pelvis.

5. Which of the following is not an adverse effect of local anesthetics to take into account in critically ill patients? A. Neurotoxicity and myotoxicity B. Central nervous system excitation or depression C. Coagulopathy D. Cardiotoxicity

3.  What is important about the management of catheters for regional analgesia in the ICU? A. Accidentally disconnected catheters can be reconnected under defined circumstances. B. Critical care nurses are well trained in the management of every kind of catheter, so no special training for regional analgesia catheters is required. C. The risk of drug errors in critically ill patients is negligible.

ANSWERS AND EXPLANATIONS 1. E is correct. Additional important indications are pancreatitis and intractable angina pain. A is incorrect. The most common side effects of epidural blocks are bradycardia and hypotension related to sympathetic block. B is incorrect. Discontinuation of continuous infusion allows neurologic assessment when necessary. C is incorrect. Hemodynamic changes can be more pronounced with intermittent bolus dosing, in patients with hypovolemia, or in patients with reduced venous return secondary to high positive end-expiratory pressure ventilation. 317

Hadzic_Ch52_p317-318.indd 317

19/04/19 10:57 AM

318

PART 7

Anesthesia in Patients with Specific Considerations

D is incorrect. There is no hard evidence of increased risks of epidural bleeding with developing coagulopathy or therapeutic anticoagulation while an epidural catheter is in place. Nevertheless, the benefits of epidural analgesia should be weighed against the risk of this serious complication. 2. B is correct. Although phrenic nerve blockade has negligible effects in mechanically ventilated patients, it may impair weaning from mechanical ventilation in high-risk patients. A is incorrect. In patients with altered mental status in whom opioid-based analgesic regimens might make neurologic evaluation difficult, excellent analgesia can be achieved for the shoulder or upper limb with continuous interscalene, cervical paravertebral, or infraclavicular approaches to the brachial plexus. C is incorrect. Real-time ultrasound guidance for peripheral catheter placement might be especially beneficial in the sedated critically ill patient. D is incorrect. Performance of blocks anatomically close to the centroneuraxis can carry a higher risk of spinal cord needle or injection injury. In heavily sedated critically ill patients, such blocks should be performed only by clinicians with adequate experience. E is incorrect. A femoral nerve catheter provides good analgesia for hip fractures. 3. A is correct. A study by Langevin1 suggests that if catheters become disconnected when the fluid in the catheter is static, the proximal 25 centimeters of the catheter may be immersed in a disinfectant, cut, and reconnected to a sterile connector. This technique is feasible only for catheters in which the fluid column can be observed. Stimulating catheters should never be cut because of the danger of unwinding the internal metal spiral wire, which conducts electrical current. No study has examined the risk of reconnecting these catheters after thorough disinfection of the outer surface, which is likely a common practice in many institutions. B is incorrect. Critical care nursing personnel should be specifically trained in handling regional analgesia catheters and must be aware of the potential complications and their early warning signs. C is incorrect. Because of the frequently large and confusing numbers of various infusion catheters in critically ill patients, the risk of drug errors and incorrect

Hadzic_Ch52_p317-318.indd 318

administration of drugs through continuous regional analgesia catheters may be higher in these patients. D is incorrect. If the patient is cooperative enough, a patient-controlled regional anesthesia (PCRA) regimen is preferable2 and such systems can also be used in a nursecontrolled fashion for intermittent bolus application without the need for additional manipulation of the infusion system. E is incorrect. The overall risk of permanent neurologic damage (from direct trauma, bleeding, or serious infection) or death from regional anesthesia and analgesia seems to be low in the perioperative setting, as shown by large surveys by Auroy and coworkers3,4 and Moen and associates.5 Although the studies certainly include critically ill patients, no specific subgroup data are available. 4. D is correct. A, B, and C are true statements regarding peripheral nerve blocks for the lower extremities in the critically ill patient. 5. C is correct. Coagulopathy is not an adverse effect of local anesthetics. A, B, and D are incorrect. All other adverse effects have been reported in the literature.

References 1. Langevin PB, Gravenstein N, Langevin SO, et al. Epidural catheter reconnection. Safe and unsafe practice. Anesthesiology. 1996;85:883-888. 2. Savoia G, Alampi D, Amantea B, et al. Postoperative pain treatment SIAARTI recommendations 2010. Short version. Minerva Anestesiol. 2010;76:657-667. 3. Auroy Y, Benhamou D, Bargues L, et al. Major complications of regional anesthesia in France: the SOS Regional Anesthesia Hotline Service. Anesthesiology. 2002;97:1274-1280. 4. Auroy Y, Narchi P, Messiah A, et al. Serious complications related to regional anesthesia: results of a prospective survey in France. Anesthesiology. 1997;87:479-486. 5. Moen V, Dahlgren N, Irestedt L. Severe neurological complications after central neuraxial blockades in Sweden 1990–1999. Anesthesiology. 2004;101:950-959.

Suggested Reading Hadzic A. Regional analgesia in the critically ill. In: Schulz-Stübner S, ed. Textbook of Regional Anesthesia and Acute Pain Management. 2nd ed. New York, NY: McGraw-Hill Publishing; 2017:chap 53.

19/04/19 10:57 AM

53 Acute Pain Management in the Opioid-Dependent Patient Thibaut Vanneste

QUESTIONS DIRECTIONS: Choose the one best response to each question. 1. Which statement is true regarding opioid-dependent patients? A. Diarrhea is not a symptom of withdrawal. B. Tolerance is a physiologic adaptation in which decreasing amounts of a drug are required to achieve the same pharmacologic effects after prolonged use. C. Withdrawal refers to physiologic symptoms resulting from the abrupt discontinuation of chronically administered opioids. D. Withdrawal is often life threatening. 2. Tolerance does not develop to which effect of opioids? A. Analgesia B. Respiratory depression C. Nausea D. Constipation 3. Which opioid has the longest half-life? A. Heroin B. Morphine C. Methadone D. Remifentanyl 4. Withdrawal symptoms can be elicited by: A. Abrupt discontinuation B. Slow dose reduction C. Administration of an agonist D. Dose augmentation 5. Which statement is true regarding opioid-dependent patients? A. Dependence and substance abuse are two separate disorders. B. Addiction is defined as the continued use of a substance despite adverse consequences.

C. Maintenance therapy aims to prevent withdrawal symptoms by using short-acting substitute agents. D. Tolerance develops to all effects of opioids. 6. Which statement is true regarding acute pain management in opioid-dependent patients? A. The use of opioid treatment for chronic noncancer pain is questioned because of the inconclusive evidence of long-term efficacy and possible adverse consequences. B. Informed consent is not necessary when prescribing opioids for chronic pain. C. Opioid abusers represent the larger part of the opioid-dependent population. D. Chronic pain in an opioid-dependent patient is never the result of opioid-induced hyperalgesia. 7. Which statement is true? A. Tolerance causes a leftward shift in the dose-response curve. B. Opioid-induced hyperalgesia (OIH) causes an upward shift in the dose-response curve. C. Tolerance causes a rightward shift in the dose-response curve. D. Tolerance is the result of upregulation of the opioid receptor. 8. The initial clinical strategy for the treatment of tolerance and opioid-induced hyperalgesia (OIH) includes: A. High-dosed opioids B. No usage of nonopioids C. Regional anesthesia techniques D. No treatment of withdrawal symptoms 9. Identifying tolerance in a given patient is possible at which moment? A. Preoperative B. Intraoperative C. Postoperative D. All of the above

319

Hadzic_Ch53_p319-324.indd 319

19/04/19 10:58 AM

320

PART 7

Anesthesia in Patients with Specific Considerations

10. Which statement is true regarding medications? A. Opioid antagonists are preferred during the perioperative period. B. Antidepressants should be stopped preoperative. C. Pain medication should be stopped preoperative. D. Document all medications the patient is taking preoperative, along with the medication doses. 11. Regarding methadone, which statement is true? A. The use of methadone once daily has a good analgesic effect. B. Methadone only binds to opioid receptors. C. Methadone has a biphasic elimination. D. Methadone shortens QT interval.

15. Proposed mechanisms for the efficacy of opioid rotation do not include: A. Complete opioid cross-tolerance B. Active metabolites C. Different opioid receptor affinity D. Different nonopioid receptor affinity

ANSWERS AND EXPLANATIONS 1. C is correct. Withdrawal refers to physiologic symptoms resulting from the abrupt discontinuation of chronically administered opioids.

12. Regarding buprenorphine, which statement is true? A. Buprenorphine has a low affinity for the mu receptor. B. Buprenorphine has a low half-life. C. Buprenorphine is a partial mu-agonist. D. Buprenorphine shortens QT interval.

A is incorrect. The symptoms of opioid withdrawal include abdominal cramps, anxiety, diarrhea, disturbed sleep, irritability, dysphoria, nausea and vomiting, rhinorrhea, urinary frequency, twitching, lacrimation, and increased muscle spasms.

13. Intraoperative management of chronic opioid-dependent patients includes: A. Management of undesired opioid effects such as respiratory depression and nausea B. Lower doses of opioids because preoperative opioids are continued C. Avoiding the use of nonopioids, as they are not necessary anymore D. Avoiding regional anesthesia

B is incorrect. Tolerance is a physiologic adaptation in which increasing amounts of a drug are required to achieve the same pharmacologic effects after prolonged use.

14. Which is not an advantage of intravenous patientcontrolled analgesia (PCA)? A. Decreasing the risk of under-medication B. Increasing patient satisfaction C. Reducing demands on nursing staff D. Less side effects compared with oral medication

D is incorrect. Although withdrawal from opioids is rarely life threatening, it can be very uncomfortable. 2. D is correct. Tolerance does not develop to miosis or constipation.1 A, B, and C are incorrect. Tolerance develops to most effects of opioids, including analgesia, euphoria, sedation, respiratory depression, and nausea. 3. C is correct. The onset and time course of opioid withdrawal are determined by the half-life of the drug (Figure 53–1).

Severity of signs and symptoms

Withdrawal from heroin Onset: 8–24 hours Duration: 4–10 days

0

Withdrawal from methadone Onset: 12–48 hours, sometimes more Duration: 10–20 days, sometimes more

10 Days

20

FIGURE 53–1  Symptoms and duration of heroin and methadone withdrawal.

Hadzic_Ch53_p319-324.indd 320

19/04/19 10:58 AM

CHAPTER 53

A, B, and D are incorrect. Withdrawal symptoms of short-acting opioids such as heroin and morphine typically begin more rapidly than long-acting agonist opioids such as methadone. The half-life of remifentanyl, heroine, morphine and methadone is respectively 3 to 10 minutes, 7 to 8 minutes, 2 to 6 hours, and 15 to 60 hours (www.bcfi.be).2 4. A is correct. Physical dependence is recognized as a state of adaption manifested by withdrawal symptoms that can be elicited by abrupt discontinuation. B, C, and D are incorrect. Withdrawal symptoms can be elicited by rapid dose reduction or the administration of an antagonist.3 5. B is correct. Addiction is defined as the continued use of a substance despite adverse consequences. It is a behavioral syndrome characterized by evidence of psychological dependence or craving, use despite harmful adverse effects, and other drug-related aberrant behavior (eg, altering prescriptions, drug hoarding or sales, unsanctioned dose escalation). A is incorrect. Before the release of the fifth edition of the Diagnostic and Statistical Manual of Mental Disorders in 2013, substance abuse and dependence were considered two separate disorders. They are now considered to be a single disorder measured on a continuum from mild to severe. Substance abuse and dependence are now recognized as the recurrent maladaptive use of a substance that leads to clinically significant impairment or distress personally, professionally, or socially.4 Physical dependence is recognized as a state of adaption manifested by withdrawal symptoms that can be elicited by abrupt discontinuation, rapid dose reduction, or the administration of an antagonist.3 C is incorrect. Maintenance therapy aims to prevent the craving and withdrawal symptoms of opioid-dependent individuals by substituting substances with long-acting and less euphoric opioids such as methadone and buprenorphine.5 D is incorrect. Tolerance can develop after the prolonged use of opioids. It is a phenomenon that is described as a decrease in the effect of a drug after repeated administration, which can be overcome by increasing the dose of the drug. Tolerance develops to most effects of opioids, including analgesia, euphoria, sedation, respiratory depression, and nausea; however, tolerance does not develop to miosis or constipation.1 6. A is correct. The estimated prevalence of chronic noncancer pain is predicted to increase as the population ages. The use of opioids in chronic noncancer pain has become increasingly common, and guidelines have been set forth by the American Pain Society, the American Academy of Pain Medicine, and the American Society of Interventional Pain Physicians for this purpose.6,7 However, evidence for the efficacy of long-term opioid use in noncancer pain has been inconclusive.

Hadzic_Ch53_p319-324.indd 321

Acute Pain Management in the Opioid-Dependent Patient

321

B is incorrect. With the increasing awareness of the potential adverse consequences of long-term opioid use, a “universal precautions” approach with appropriate risk assessment and management has been recommended. This approach includes proper diagnosis for the etiology of pain; informed consent; the use of treatment agreements; assessment of analgesia, activity, adverse effects, and aberrant behavior; and careful documentation.8 C is incorrect. The smaller portion of the opioid-dependent population includes opioid abusers. Heroin is the most common illicit opioid that is abused, although its use has plateaued over the past 10 years. However, prescription opioids are increasingly being abused. Prescription opioid sales quadrupled between 1999 and 2010.9 In 2010, the number of Americans reporting nonmedical use of prescription opioids reached 12 million.10 Since 2003, more deaths due to opioid overdose have involved prescription opioid analgesics than heroin and cocaine combined.11 Further complicating the picture is that opioid abuse and misuse may also be seen in patients prescribed opioids for chronic pain. In a review examining the prevalence of addiction in chronic pain patients, prevalence varied from 0%–31%.12 Pain is prevalent in patients with a history of opioid abuse. In studies of patients receiving methadone maintenance treatment, 61% report chronic pain, and 37% report severe chronic pain.13,14 D is incorrect. Increased reports of pain may occur as a result of opioid-induced hyperalgesia (OIH), which occurs when the long-term use of opioids causes a hypersensitivity to painful stimuli.15 7. C is correct. Tolerance can develop after the prolonged use of opioids. It is a phenomenon that is described as a decrease in the effect of a drug after repeated administration, which can be overcome by increasing the dose of the drug. A is incorrect. It is important to recognize that while OIH and tolerance are distinctly different processes, they are related and have overlapping mechanisms. While tolerance causes a rightward shift in the dose-response curve, OIH leads to a downward shift in the dose-response curve (Figure 53–2). The precise molecular mechanisms underlying tolerance and OIH are not fully understood and are still being investigated. B is incorrect. OIH is a mechanism by which the analgesic properties of a drug are decreased secondary to increased pain sensitivity. As a result of OIH and tolerance, opioiddependent patients may have a lower pain threshold and require higher doses of opioids to achieve adequate pain relief. D is incorrect. Tolerance is classically thought to be due to the downregulation of opioid receptors. However, other mechanisms likely play a role, as well, such as adaptations downstream of receptor activation.16 OIH has many proposed mechanisms, including those involving the central glutaminergic system, spinal dynorphins, descending facilitation, genetic mechanisms, and enhanced nociceptive responses.17

19/04/19 10:58 AM

322

PART 7

Anesthesia in Patients with Specific Considerations

Pain threshold

Opioid dose

3

2

1

Opioid naïve Opioid tolerant Opioid-induced hyperalgesia Pain tolerance

FIGURE 53–2  In opioid-naïve patients, increasing the opioid dose results in increased analgesia (1). In opioid-tolerant patients, the baseline threshold remains the same, but an increased dose of opioid is required to reach the same analgesic effect (2). Note that in opioid-induced hyperalgesia, maximal analgesic effect is not reached, baseline pain tolerance is reduced, and there is a downward shift from the opioidtolerant patient (3).

8. C is correct. The initial clinical strategy for the treatment of tolerance and OIH are similar: regional anesthesia techniques, optimize nonopioids, control withdrawal, and adjust opioid doses. A, B, and D are incorrect. See explanation for answer C. 9. D is correct. Opioid-tolerant patients can often be identified during preoperative assessment. In some patients, tolerance may not become evident until a diminished response to intraoperative opioids is noted or when postoperative pain control becomes challenging. 10. D is correct. A crucial first step in the management of the perioperative pain in opioid-dependent patients is identifying the extent of opioid tolerance in a given patient. In some patients, the diagnosis can be made by the preoperative assessment, whereas in others, tolerance may not become evident until a diminished response to intraoperative opioids is noted or when postoperative pain control is found to be difficult. Document all medications the patient is taking, along with the medication doses. Confirm medications and doses with the original provider. A is incorrect. Opioid antagonists such as naltrexone and naloxone are not advised in the perioperative period for an opioid-dependent patient. The postoperative administration of these drugs can bring upon withdrawal symptoms in this patient population, which may place

Hadzic_Ch53_p319-324.indd 322

the patient at medical risk, especially in the presence of comorbidities. If the administration of an opioid antagonist is needed for respiratory depression, it should be administered carefully in titrated doses. B is incorrect. It is important to remember that patients who are being treated for chronic pain often exhibit poor coping and have a psychosocial aspect to their pain that is compounded by anxiety and depression. For this reason, antidepressants and antianxiety medications should be continued during the perioperative period. In patients with opioid abuse, it is also important to note that the perioperative period is not an optimal time to detoxify the patient. C is incorrect. Patients with chronic pain on opioids should be advised to continue taking their medications as prescribed, including on the morning of surgery. 11. C is correct. Methadone has a biphasic elimination curve with a rapid alpha-elimination phase followed by a much longer beta-elimination phase. A, B, and D are incorrect. Thus, for maintenance treatment of opioid addiction, the use of methadone once daily prevents withdrawal. However, the analgesic effects of methadone require more frequent dosing.18 Methadone is a long-acting opioid agonist that also has activity as an NMDA antagonist. Methadone may prolong the QT

19/04/19 10:58 AM

CHAPTER 53

interval, which can lead to fatal cardiac arrhythmias. Caution should be used when methadone is combined with other drugs that may also prolong the QT interval. 12. C is correct. Buprenorphine is a partial mu-opioid agonist. A is incorrect. Buprenorphine has great affinity for the mu-opioid receptor, approximately 1000 times that of morphine.19 For that reason, it may precipitate withdrawal in patients already on opioids because its high receptor affinity displaces other opioids. However, in a patient who has stopped using opioids and is in withdrawal, buprenorphine will ease symptoms of withdrawal through its partial agonist effect.20 B is incorrect. Buprenorphine has a long half-life of approximately 37 hours. Thus, for the treatment of addiction it can be used once daily. D is incorrect. It may also be associated with a prolongation of the QT interval.20 13. A is correct. A multimodal approach for pain control including regional techniques and nonopioid analgesics should be considered for opioid-dependent patients. Patients who are opioid-dependent should continue their baseline opioid dose during the perioperative period prior to the induction of anesthesia with additional analgesics provided as needed. B, C, and D are incorrect. Opioid-dependent patients may require larger intraoperative opioid doses than opioid-naïve patients. Respiratory rate can be used as a method to appropriately dose opioids intraoperatively. If the patient is spontaneously breathing either throughout the case or at the end after reversal of muscle relaxant, the opioid dose may be titrated to a respiratory rate of 8–12 breaths per minute. Opioid effects can also occur in the opioid-tolerant patient, including nausea, vomiting, pruritus, and respiratory depression. Therefore, it is essential to monitor vital signs and sedation level in the perioperative period while opioids are being administered even in this group of patients. 14. D is correct. Less side effects compared with oral medication is not an advantage of intravenous patient-controlled analgesia (PCA). Intravenous PCA is an effective option for postsurgical analgesia especially in the opioid-dependent patient. PCA allows the patient to administer pain medication almost instantly on an as-needed basis, decreasing the risk of under-medication and breakthrough pain and increasing patient satisfaction. By allowing patients to administer their own pain medication, demands on nursing staff are also reduced. A, B, and C are incorrect. All of these are advantages of intravenous patient-controlled analgesia (PCA). 15. A is correct. Opioid rotation or switching between different opioids is an option to improve analgesia while decreasing the overall daily opioid dose and opioidrelated side effects for individual compound failures despite titration.

Hadzic_Ch53_p319-324.indd 323

Acute Pain Management in the Opioid-Dependent Patient

323

B, C, and D are incorrect. Proposed mechanisms for the efficacy of opioid rotation include incomplete opioid cross-tolerance, active metabolites, and different opioid and nonopioid receptor affinity.21 When rotating opioids, the equianalgesic dose should be reduced to account for incomplete cross-tolerance.

References 1. Gustin HB, Alik H. Opioid analgesics. In: Hardman JG, Limbird LE, eds. Goodman and Gilman’s The Pharmacological Basis of Therapeutics. New York: McGraw-Hill; 2001:569-619. 2. Rook EJ, Huitema AD, van den Brink W, et al. Population pharmacokinetics of heroin and its major metabolites. Clin Pharmacokinet. 2006;45(4):401-417. 3. Stein C, Kopf A. Anesthesia and treatment of chronic pain. In: Miller RD, Eriksson LI, Fleisher LA, Wiener-Kronish JP, Young WL, eds. Miller’s Anesthesia. 7th ed. Philadelphia: Churchill Livingstone; 2009:1803-1804. 4. American Psychiatric Association. Diagnostic and Statistical Manual of Mental Disorders. 5th ed. Arlington, VA: American Psychiatric Publishing; 2013. 5. Mitra S, Sinatra RS. Perioperative management of acute pain in the opioid-dependent patient. Anesthesiology. 2004;101:212-227. 6. Chou R, Fanciullo GJ, Fine PG, et al. Clinical guidelines for the use of chronic opioid therapy in chronic noncancer pain. J Pain. 2009;10:113-130. 7. Manchikanti L, Salahadin A, Atluri S, et al. American Society of Interventional Pain Physicians (ASIPP) guidelines for responsible opioid prescribing in chronic non-cancer pain: part 2—guidance. Pain Physician. 2012;15:S67-S116. 8. Gourlay DL, Heit HA, Almahrezi A. Universal precautions in pain medicine: a rational approach to the treatment of chronic pain. Pain Med. 2005;6:107-112. 9. Substance Abuse and Mental Health Services Administration. Results from the 2010 National Survey on Drug Use and Health: detailed table, table 7.1.a. http://www.samhsa.gov/data/ NSDUH/2k10NSDUH/tabs/Sect7peTabs1to45.htm#Tab7.1A. Accessed June 11, 2013. 10. Centers for Disease Control and Prevention. Prescription painkiller overdose in the U.S. http://www.cdc.gov/features/vitalsigns/painkilleroverdoses/. Accessed June 1, 2013. 11. Centers for Disease Control and Prevention. Vital signs: overdoses of prescription opioid pain relievers. http://www.cdc.gov/ mmwr/preview/mmwrhtml/mm6043a4.htm. Accessed June 30, 2013. 12. Minozzi S, Amato L, Davoli M, et al. Development of dependence following treatment with opioid analgesics for pain relief: a systematic review. Addiction. 2013;108:688-698. 13. Jamison RN, Kauffman J, Katz NP. Characteristics of methadone maintenance patients with chronic pain. J Pain Symptom Manage. 2000;19:53-62. 14. Rosenblum A, Joseph H, Fong C, et al. Prevalence and characteristics of chronic pain among chemically dependent patients in methadone maintenance and residential treatment facilities. JAMA. 2003;289:2370-2378. 15. Compton P, Athanasos P, Elashoff D. Withdrawal hyperalgesia after acute opioid physical dependence in nonaddicted humans: a preliminary study. J Pain. 2003;4:511-519.

19/04/19 10:58 AM

324

PART 7

Anesthesia in Patients with Specific Considerations

16. Bailey CP, Connor M. Opioids: cellular mechanisms of tolerance and physical dependence. Curr Opin Pharmacol. 2005;5:60-68. 17. Lee M, Silverman SM, Hansen H, et al. A comprehensive review of opioid-induced hyperalgesia. Pain Physician. 2011;14:145-165. 18. Ferrari A, Coccia CP, Bertolini A, et al. Methadone—metabolism, pharmacokinetics and interactions. Pharmacol Res. 2004;50: 551-559. 19. Bryson EO, Lipson A, Gevirtz C. Anesthesia for patients on buprenorphine. Anesthesiol Clin. 2010;28:611-617.

21. Huxtable CA, Roberts LJ, Somogyi AA, et al. Acute pain management in opioid-tolerant patients: a growing challenge. Anaesth Intensive Care. 2011;39:804-823.

Suggested Reading Hadzic A. Acute pain management in the opioid-dependent patient. In: Doan L, Largi J, Choi L, Gharibo C, eds. Textbook of Regional Anesthesia and Acute Pain Management. 2nd ed. New York, NY: McGraw-Hill Publishing; 2017:chap 54.

20. Bart G. Maintenance medication for opiate addiction: the foundation of recovery. J Addict Dis. 2012;31:207-225.

Hadzic_Ch53_p319-324.indd 324

19/04/19 10:58 AM

54 Regional Anesthesia in Patients with Trauma Brendan Keen, Kasra Razmjou, and Jeff Gadsden

QUESTIONS DIRECTIONS: Choose the one best response to each question. 1. Which of the following is true regarding the use of continuous perineural catheters after trauma? A. Continuous catheters should be removed at 48 hours to prevent infection. B. Catheters can be used for both background analgesia and for surgical anesthesia. C. Infection rates for catheters may depend on the type of catheter used. D. The maximum combined rate of 0.2% ropivacaine for all catheter infusions should not exceed 8 mL/hr. 2. The femoral head is innervated by all of the following except: A. Femoral nerve B. Obturator nerve C. Sacral plexus D. Lateral femoral cutaneous nerve 3. The benefits of neuraxial anesthesia for operative fixation of a hip fracture include all of the following except: A. Reduced early mortality B. Decreased incidence of deep vein thrombosis C. Decreased pulmonary complications D. Increased risk of myocardial events 4. Regarding rib fractures, all of the following are true except: A. The number of rib fractures correlates with mortality. B. The most common cause of mortality is atelectasis. C. Rib fractures lead to shallow breathing due to pain on inspiration. D. Epidural analgesia may decrease the number of days requiring mechanical ventilation by half. 5. A patient with tibial fracture develops acute compartment syndrome (ACS) of the anterior compartment of

the lower extremity. Which nerve provides sensation to this compartment? A. Deep peroneal nerve B. Femoral nerve C. Superficial peroneal nerve D. Tibial nerve 6. Which of the following is a benefit of interscalene nerve block over intravenous sedation for reduction of shoulder dislocation? A. Absence of need for cardiorespiratory monitoring during procedure B. Decreased need for one-to-one nursing care in the emergency department setting C. Decreased risk for redislocation following reduction D. Shoulder girdle relaxation from anesthetizing the inferior trunk of the brachial plexus 7. Regarding regional anesthesia in burn patients, which of the following is most likely true? A. It is acceptable to place a peripheral nerve block catheter through an area of burn. B. Levels of alpha-1 acid glycoprotein are usually elevated in burn patients, which may be protective against local anesthetic toxicity. C. Neural blockade often exaggerates local vasospasm at areas of skin graft and can be detrimental to skin graft longevity. D. Neuraxial anesthesia is typically contraindicated as burn patients tend to be hypocoaguable. 8. Which of the following is not a benefit of peripheral nerve blockade during digit reimplantation? A. Decreased stress response leading to less release of prothrombotic mediators B. Improved postoperative pain scores in those with peripheral nerve blockade C. Muscle relaxation resulting in decreased chance of inadvertent patient movement intraoperatively D. Parasympathetic blockade resulting in vasodilatation

325

Hadzic_Ch54_p325-328.indd 325

19/04/19 3:13 PM

326

PART 7

Anesthesia in Patients with Specific Considerations

9. Which fracture places the patient at highest risk for developing acute compartment syndrome (ACS)? A. Distal tibia B. Femoral neck C. Proximal tibia D. Ulnar 10. Which of the following has clearly been demonstrated to be associated with the development of chronic pain following trauma? A. Absence of any psychiatric disorder at time of injury B. High intensity of acute pain at the time of injury C. Male gender D. Younger age at time of injury

ANSWERS AND EXPLANATIONS

catheter. In trauma patients, the risk for infection starts to rise at about 48 hours, but the overall risk is still low with only 0%–3% of all catheters showing evidence of infection. C is incorrect. Catheter type may play a role in the development of infection. Lai et al reported a case series of two superficial and four deep infections, in which the deep infections requiring operative incision and drainage were associated with stimulating catheter use. The authors hypothesized that repetitive movements of a catheter with an internal metal coil could result in microhematoma formation, providing a rich culture medium for hematogenously spread bacteria. In trauma patients it may be prudent to use catheter kits that lack a coiled metal tip. D is incorrect. Common clinical practice, supported by long-term studies in military patients receiving multiple infusions, is to limit the overall rate of 0.2% ropivacaine to approximately 12–14 mL/hr. This is clearly patient- and clinician-dependent and individual circumstances will dictate clinical practice, but 14 mL/hr appears to be a safe overall limit.

1. B is correct. Continuous perineural catheters are an excellent technique for analgesia following polytrauma. They can provide targeted analgesia that is virtually devoid of side effects for days to weeks. In patients who are returning to the operating room every 2–3 days for repeated procedures (eg, debridements), the catheter(s) can be maintained with a background infusion of a low- concentration solution for analgesia (eg, 0.1%–0.2% ropivacaine), and then bolused with a higher-concentration solution (eg, 0.5% ropivacaine) for surgical anesthesia.

2. D is correct. The femoral head is not innervated by the lateral femoral cutaneous nerve. The lateral femoral cutaneous nerve provides sensation to the lateral thigh. Unlike the femoral nerve, obturator nerve, and sacral plexus (Figure 54–1), the lateral femoral cutaneous nerve does not innervate the femoral head. Understanding this osteotomal innervation is important in block planning.

A is incorrect. Catheters may be left in place for days to weeks, depending on the indication, the anatomical site, and the estimated risk of infection vs. benefit of the

A, B, and C are incorrect. As evident from Figure 54–1, the femoral nerve, obturator nerve, and sacral nerve innervate the proximal femur.

Obturator nerve Femoral nerve Sacral plexus

Anterior

Posterior

Hadzic - Lancea/ NYSORA FIGURE 54–1  Osteotomal innervation of the proximal femur.

3. D is correct. Increased risk of myocardial events is not a benefit of neuraxial anesthesia for operative fixation of a hip fracture. Neuraxial anesthesia has multiple benefits including reduced early mortality, incidence of deep vein thrombosis, postoperative confusion, and pulmonary complications. According to a meta-analysis by Luger

Hadzic_Ch54_p325-328.indd 326

et al, there were no differences between groups in the rates of arrhythmias, myocardial events, congestive heart failure, intraoperative blood loss, renal failure, or stroke. A, B, and C are incorrect. All of these are benefits of neuraxial anesthesia for operative fixation of a hip fracture.

19/04/19 3:13 PM

CHAPTER 54

4. B is not true, so option B is correct. Rib fractures are the most common injury associated with chest trauma. The cause of mortality is primarily related to pulmonary injury, such as lung contusion and pneumothorax, and delayed pulmonary processes, such as pneumonia and acute respiratory distress syndrome. Although atelectasis is common in patients with rib fractures secondary to shallow breathing, atelectasis is not the most common cause of mortality. A, C, and D are true, so options A, C, and D are incorrect. Mortality is linked to the number of ribs that are fractured. One to two fractured ribs carries a risk of mortality of 5%; 15% for 3–5 ribs; and 34% for 6 or more ribs. This is especially true if elderly. Rib fractures have a large effect on pulmonary mechanics, as the pain from inspiration leads to shallow breathing, predisposing to V/Q mismatch, hypoxemia, atelectasis, pneumonia, and acute respiratory distress syndrome. Regional techniques that target the thoracic spinal nerves (eg, epidural, paravertebral, or intercostal nerve blockade) are very effective at relieving this pain and decreasing the need for, or duration of, mechanical ventilation. 5. A is correct. Tibial fracture is the most common injury leading to compartment syndrome. The anterior compartment of the lower extremity contains the tibialis anterior, extensor hallucis longus, extensor digitorum longus, and peroneus tertius muscles and is at risk for developing compartment syndrome following tibial fracture. The deep peroneal nerve provides sensation to the anterior compartment. B is incorrect. The femoral nerve does not supply sensation to any of the four compartments of the lower extremity; thus, femoral nerve block is unlikely to mask any pain from compartment syndrome, allowing proper diagnosis to ensue. C is incorrect. The superficial peroneal nerve supplies sensation to the lateral compartment. D is incorrect. The tibial nerve supplies sensation to the deep posterior compartment and the superficial posterior compartment. 6. B is correct. Interscalene nerve block provides an attractive alternative over IV sedation for shorter procedures taking place in the emergency department. IV sedation with propofol or ketamine may cause acute cardiorespiratory changes in the patient and one-to-one nursing may be needed to monitor these patients more closely following the procedure. Patients also would typically be required to be NPO for at least 6 hours prior to the procedure, which may delay treatment. A is incorrect. Cardiorespiratory monitoring should still be employed during peripheral nerve block procedures. C is incorrect. There is no evidence that interscalene nerve block decreases the incidence of redislocation. D is incorrect. Shoulder girdle relaxation is accomplished by anesthesia of the superior trunk of the brachial plexus, not the inferior trunk. 7. B is correct. Levels of alpha-1 acid glycoprotein (AAG), an acute phase reactant and one of the principal plasma

Hadzic_Ch54_p325-328.indd 327

Regional Anesthesia in Patients with Trauma

327

proteins that binds local anesthetics, are typically elevated for 20 days in burn patients. This theoretically provides a larger safety margin against local anesthetic systemic toxicity. A is incorrect. Placing a peripheral nerve block catheter through an area of burn is contraindicated. C is incorrect. Neural blockade decreases stress response and stress-related mediators at area of skin graft, leading to decreased vasospasm and local thrombosis during skin grafting procedures. D is incorrect. Burn patients tend toward a hypercoagulable state. Neuraxial anesthesia is typically not contraindicated unless the patient has developed coagulation abnormalities from sepsis or profound blood loss without factor replacement. 8. D is correct. Peripheral nerve blockade typically results in significant vasodilatation but this occurs from sympathetic, not parasympathetic, blockade. A is incorrect. Peripheral nerve blockade decreases stress response, leading to a lower amount of proinflammatory mediators being released. B is incorrect. Pain scores have been shown to be significantly reduced in patients who have received a peripheral nerve block. C is incorrect. Muscle relaxation and decreased patient movement is another added benefit of peripheral nerve blockade performed preoperatively. 9. C is correct. Over one-third of all cases of acute compartment syndrome are associated with fracture of the tibia. Fractures including the proximal and middle thirds of the diaphysis place the patient at particularly high risk due to bulkier muscle mass in these areas compared to the ankle. A is incorrect. Fractures of the distal tibia do not place patients at as high of a risk for compartment syndrome as fractures of the proximal and middle thirds of the tibia. B is incorrect. Femoral neck fractures do not place the patient at a high risk for compartment syndrome. D is incorrect. Fractures of the forearm place the patient at some risk for compartment syndrome, but tibial fractures account for the highest percentage of compartment syndrome cases. 10. B is correct. A high intensity of acute pain at the time of injury is closely associated with the development of chronic pain following this injury. Regional anesthesia has been shown to significantly decrease the intensity of acute pain following injury, despite this evidence for peripheral nerve blocks resulting in decreased chronic pain is very weak and more studies are currently needed to evaluate this. A is incorrect. Patients with untreated anxiety or depression at time of injury are at higher risk for developing chronic pain. C is incorrect. Female gender, not male gender, is associated with a higher incidence of chronic pain following injury. D is incorrect. Older patients, not younger, are at higher risk for developing chronic pain following injury.

19/04/19 3:13 PM

328

PART 7

Anesthesia in Patients with Specific Considerations

Suggested Readings Carrier FM, Turgeon AF, Nicole PC, et al. Effect of epidural analgesia in patients with traumatic rib fractures: a systematic review and meta-analysis of randomized controlled trials. Can J Anaesth. 2009;56:230-242. doi:10.1007/s12630-009-9052-7. Clay FJ, Watson WL, Newstead SV, McClure RJ. A systematic review of early prognostic factors for persisting pain following acute orthopedic trauma. Pain Res Manag. 2012;17:35-44. Gadsden J. Regional Anesthesia in Trauma. Cambridge: Cambridge University Press, 2012.

Hadzic_Ch54_p325-328.indd 328

Hadzic A. Regional anesthesia in patients with trauma. In: Gadsden J, Lin E, Warlick AL, eds. Textbook of Regional Anesthesia and Acute Pain Management. 2nd ed. New York, NY: McGraw-Hill Publishing; 2017:chap 55. Luger TJ, Kammerlander C, Gosch M, et al. Neuroaxial versus general anaesthesia in geriatric patients for hip fracture surgery: does it matter? Osteoporos Int. 2010;21(Suppl 4):S555-S572. doi:10.1007/ s00198-010-1399-7. Ulmer T. The clinical diagnosis of compartment syndrome of the lower leg: are clinical findings predictive of the disorder? J Orthop Trauma. 2002;16:572-577.

19/04/19 3:13 PM

55 Regional Anesthesia for Cardiac and Thoracic Anesthesia Daryl Steven Henshaw

QUESTIONS DIRECTIONS: Choose the one best response to each question. 1. The use of high thoracic epidural analgesia/anesthesia (hTEA) for cardiac surgery results in a segmental thoracic sympathectomy. This sympatholysis results in which of the following physiological effects? A. Decreased stability of intraoperative hemodynamics B. Dilation of the coronary and internal mammary arteries C. Increased frequency of cardiac arrhythmias D. Increased heart rate 2. The most recently published estimation of the risk of epidural hematoma formation related to thoracic epidural analgesia (TEA) in cardiac surgery patients is: A. 1:5000 B. 1:12,000 C. 1:40,000 D. 1:75,000 3. When performing an awake cardiac surgery using high thoracic epidural anesthesia (hTEA), which of the following is a major disadvantage of this technique? A. Can only be utilized for off-pump coronary artery bypass surgery B. Does not allow for the performance of transesophageal echocardiography (TEE) C. Has a low patient acceptance rate D. Is associated with increased costs 4. While there are many potential advantages of spinal analgesia for cardiac surgery, what is the main reason why this technique has not gained widespread acceptance? A. A potentially increased risk of neuraxial hematoma B. An increased rate of side effects including pruritus, nausea, vomiting, and urinary retention

C. It does not reliably attenuate sympathetic activity. D. The analgesic benefit does not reliably extend past 24 hours. 5. The use of intrathecal opioids and/or local anesthetics in cardiac surgery has been shown to result in which of the following? A. Decreased rate of myocardial infarction B. Decreased rate of perioperative arrhythmias C. Decreased perioperative mortality D. Improved postoperative analgesia 6. The use of thoracic epidural analgesia (TEA) for thoracic surgery results in sympathetic blockade. Which of the following is one of the beneficial effects of this sympatholysis? A. Decreased coronary blood flow and a resultant increase in myocardial ischemia B. Decreased gastric motility and perfusion C. Decreased systemic stress response D. Increased bronchial tone and reactivity as a result of unopposed vagal tone 7. A 65-year-old patient with chronic obstructive pulmonary disease (COPD) undergoes a thoracotomy and receives general anesthesia as well as thoracic epidural analgesia (TEA). Which of the following is true in regards to the patient’s pulmonary function following surgery? A. The patient will have a reduced vital capacity (VC) compared to a patient that receives general anesthesia alone. B. The patient will have a reduced forced expiratory volume in one second (FEV1) compared to a patient that receives general anesthesia alone. C. The patient will have an increased functional residual capacity (FRC) compared to a patient that receives general anesthesia alone. D. The patient will have increased airway resistance compared to a patient that receives general anesthesia alone.

329

Hadzic_Ch55_p329-332.indd 329

22/04/19 6:16 PM

330

PART 7

Anesthesia in Patients with Specific Considerations

8. Which of the following correctly describes the anatomy of the paravertebral space? A. The pleura forms the ventral border, the vertebrae form the medial border, and the costotransversal processes form the dorsal border. B. The lung parenchyma forms the ventral border, the vertebrae form the medial border, and the pleura forms the dorsal border. C. As the spinal nerves travel through the paravertebral space they are surrounded by a thick layer of fascia. D. The paravertebral space is an isolated compartment and does not reliably communicate with the intercostal space of the epidural space. 9. A 75-year-old man is undergoing thoracoscopic surgery for a wedge resection. You plan to perform a singleinjection paravertebral block for postoperative analgesia using a landmark-based technique. Which of the following is true regarding the approach? A. The needle insertion site is 1.5 cm lateral to midline. After contacting the transverse process, the needle is redirected either caudal of cephalad to “walk off ” the transverse process and then advanced 1.5–2.0 cm. B. The needle insertion site is 1.5 cm lateral to midline. After contacting the transverse process, the needle is redirected either caudal of cephalad to “walk off ” the transverse process and then advanced 1.0–1.5 cm. C. The needle insertion site is 2.5 cm lateral to midline. After contacting the transverse process, the needle is redirected either caudal of cephalad to “walk off ” the transverse process and then advanced 1.0–1.5 cm. D. The needle insertion site is 2.5 cm lateral to midline. After contacting the transverse process, the needle is redirected either caudal or cephalad to “walk off ” the transverse process and then advanced 1.5–2.0 cm. 10. Which of the following is true regarding thoracic paravertebral blockade (tPVB)? A. Given the distance from the neuraxis the risk of spinal hematoma is not significant and therefore there are no concerns regarding periprocedural coagulation or thromboprophylaxis. B. Multiple studies have established that there is a reliable relationship between the volume of injectate and the extent of spread. C. Most experts still consider thoracic epidural analgesia (TEA) to be the gold standard for analgesia after thoracic surgery. However, the available literature supports that tPVB is a reasonable alternative when TEA is not feasible. D. Studies have demonstrated that for patients undergoing thoracic procedures, tPVB provides superior postoperative analgesia when compared to thoracic epidural analgesia (TEA), but results in a higher rate of side effects.

ANSWERS AND EXPLANATIONS 1. B is correct. The sympatholysis that results from high thoracic epidural analgesia (hTEA) results in dilation of both

Hadzic_Ch55_p329-332.indd 330

the coronary and internal mammary arteries and therefore improves perfusion to the respective vessels. A is incorrect. The sympathectomy that results from hTEA typically results in improved stability of intraoperative hemodynamics. C is incorrect. Because of the improved hemodynamics and perfusion that result from the sympathectomy after hTEA, the frequency of cardiac dysrhythmias is decreased. D is incorrect. Heart rate is typically decreased after hTEA as a result of the sympathectomy and the resultant blockade of cardiac accelerator fibers. 2. B is correct. The most recently published risk estimation of hematoma as a result of epidural blockade is 1:12,000 with a previously reported 95% confidence interval of 1:150,000 to 1:1500. A, C, and D are incorrect. As stated above, the estimated risk of epidural hematoma following epidural hematoma placement in cardiac surgery is 1:12,000. 3. B is correct. Intraoperative transesophageal echocardiography (TEE) has become a standard procedure for patients undergoing cardiac surgery, but unfortunately employing a regional anesthesia technique that allows for awake cardiac surgery does not allow for the use of TEE as it cannot be performed on an awake patient. A is incorrect. There have been both case reports and case series that have described the successful performance of awake cardiac surgery for patients undergoing a variety of valve replacement surgeries requiring the use of cardiopulmonary bypass. C is incorrect. As shown in Table 55–1, awake cardiac surgery has been well received by patients and has a high acceptance rate. At least one publication reported that patients who have undergone cardiac surgery awake after having had a prior cardiac surgery under general anesthesia all preferred the experience of awake cardiac surgery. D is incorrect. Table 55–1 also shows that one potential benefit of awake cardiac surgery is decreased costs. This may be a result of decreased intensive care unit (ICU) length of stay (or avoidance of the ICU entirely), and a decreased overall hospital length of stay.

TABLE 55–1  Potential benefits of awake cardiac surgery. Excellent postoperative pain relief Bypass of ICU or short-term ICU stay Avoidance of endotracheal intubation No weaning Faster mobilization Shorter hospital stay Decreased costs High patient acceptance ICU, intensive care unit.

22/04/19 6:16 PM

CHAPTER 55

4. A is correct. The main reason why spinal analgesia has not gained widespread acceptance is likely the concern for neuraxial hematoma formation given the need for systemic anticoagulation (heparin) during cardiac surgery. While the risk of neuraxial hematoma is also presumably increased following thoracic epidural analgesia for the same reason, roughly 50% of neuraxial hematomas following thoracic epidural analgesia occur during catheter removal. B is incorrect. While the administration of intrathecal opioids would be expected to increase the rate of side effects including pruritus, nausea, vomiting, and urinary retention following surgery, these are usually both easily treated and self-limited. They are not the main reason why spinal analgesia for cardiac surgery has not gained widespread acceptance. C is incorrect. Spinal analgesia that consists of intrathecal opioids alone does not attenuate the sympathetic response to cardiac surgery and cardiopulmonary bypass. Reliable attenuation is only achieved when local anesthetics are included. While this statement is true, it is not the main reason why spinal analgesia has not gained widespread acceptance. D is incorrect. Without the ability to continuously infuse local anesthetics or local anesthetic and opioid mixtures, the analgesic benefits of intrathecal opioids is limited to 24 hours. While this statement is true, it is not the main reason why spinal analgesia has not gained widespread acceptance. 5. D is correct. Multiple meta-analyses have demonstrated that the use of spinal analgesia does improve postoperative analgesia as demonstrated by a reduction in opioid use and/or reported pain scores. A, B, and C are incorrect. The use of spinal analgesia has not been shown to result in a decreased rate of myocardial infarction, a decreased rate of perioperative arrhythmias, or reduced perioperative mortality. 6. C is correct. The use of thoracic epidural analgesia (TEA) does result in a decrease in the sympathetic stress response as a result of sympathetic blockade. A is incorrect. As a result of the sympathetic blockade that results from TEA, coronary blood flow is improved and there is a reduction in myocardial ischemia. B is incorrect. The sympathectomy that results from TEA increases gastrointestinal motility and perfusion. D is incorrect. It is true that the sympathetic blockade provided by TEA may result in increased bronchial tone and reactivity as a result of unopposed vagal tone. However, this would not be expected to be a beneficial effect of TEA. 7. C is correct. Functional residual capacity (FRC) would be expected to increase in a patient that receives both thoracic epidural analgesia (TEA) and general anesthesia when compared to a patient that receives general anesthesia alone.

Hadzic_Ch55_p329-332.indd 331

Regional Anesthesia for Cardiac and Thoracic Anesthesia

331

A is incorrect. The use of TEA in healthy volunteers has been shown to reduce vital capacity. However, when vital capacity (VC) in patients undergoing general anesthesia alone is compared to patients receiving TEA and general anesthesia, TEA attenuates the reduction in VC. B is incorrect. Forced expiratory volume in one second (FEV1) in a patient that has received TEA in addition to general anesthesia would be expected to increase compared to a patient that has received general anesthesia alone. D is incorrect. Airway resistance is not increased by the use of TEA in patients with obstructive pulmonary disease. This is shown in Table 55–2. TABLE 55–2  Pulmonary effects of thoracic epidural anesthesia.1–3 Decreased bronchial reactivity as a result of systemic effect of local anesthetic No increased airway resistance in patients with severe obstructive pulmonary disease No influence on hypoxic pulmonary vasoconstriction (HPV)

8. A is correct. The paravertebral space is delineated by the pleura ventrally, the vertebral column medially, and the costotransversal processes dorsally. B is incorrect. The anatomic boundaries mentioned here describe the pleural space rather than the paravertebral space. C is incorrect. As the spinal nerves travel through the paravertebral space they are not surrounded by a thick layer of fascia; rather they are embedded in paravertebral adipose tissue. D is incorrect. The paravertebral space is not an isolated compartment. It is continuous laterally with the intercostal space and medially with the epidural space by way of the intervertebral foramen. 9. C is correct. The correct needle insertion site is 2.5 cm lateral to midline. After the transverse process is contacted, the needle is redirected either caudal or cephalad to “walk off ” the transverse process and then advanced 1.0–1.5 cm into the paravertebral space. A is incorrect. A needle insertion site of 1.5 cm lateral to the midline would be too medial and likely would contact the articular processes. In addition, advancing the needle 1.5–2.0 cm past the transverse process would potentially puncture the pleura. B is incorrect. Again, a needle insertion site 1.5 cm lateral to midline would be too medial. D is incorrect. While this answer correctly describes the distance from midline that the needle should be inserted, the distance that the needle is advanced after “walking off ” either the cephalad or caudal edge of the transverse process is too deep. Advancing the needle 1.5–2.0 cm would likely puncture the pleura.

22/04/19 6:16 PM

332

PART 7

Anesthesia in Patients with Specific Considerations

10. C is correct. While systemic reviews and a meta-analyses have concluded that thoracic paravertebral blockade (tPVB) is at least as effective as thoracic epidural analgesia (TEA) and is associated with a better side effect profile, there are concerns regarding the methodology used in these comparative studies. As optimally conducted studies comparing TEA to tPVB are lacking, most experts still consider TEA to be the gold standard for analgesia after thoracic surgery. However, when TEA is not feasible, tPVB is viewed as a reasonable alternative. A is incorrect. Given the vicinity of the paravertebral space to the neuraxis, the risk of spinal hematoma is a concern and therefore the same recommendations regarding safe time intervals between anticoagulation and the performance of a procedure should be observed. B is incorrect. To date, no reliable relationship between the volume of injectate and the extent of spread has been established. D is incorrect. As stated above, the comparative studies between tPVB and TEA have only demonstrated noninferiority (the two techniques are at least as effective) for patients undergoing thoracic surgery. Studies have not

Hadzic_Ch55_p329-332.indd 332

been able to demonstrate that tPVB provides superior postoperative analgesia. In addition, in these comparative studies, tPVB has been linked to lower rates of side effects.

References 1. Chow MY, Goh MH, Boey SK, et al. The effects of remifentanil and thoracic epidural on oxygenation and pulmonary shunt fraction during one-lung ventilation. J Cardiothorac Vasc Anesth. 2003;17:69-72. 2. Groeben H, Schäfer B, Pavlakovic G, et al. Lung function under high thoracic segmental epidural anesthesia with ropivacaine or bupivacaine in patients with severe obstructive pulmonary disease undergoing breast surgery. Anesthesiology. 2002;96:536-541. 3. Groeben H, Schwalen A, Irsfeld S, et al. High thoracic epidural anesthesia does not alter airway resistance and attenuates the response to an inhalational provocation test in patients with bronchial hyperreactivity. Anesthesiology. 1994;81:868-874.

Suggested Reading Hadzic A. Regional anesthesia for cardiac and thoracic anesthesia. In: Kessler P, ed. Textbook of Regional Anesthesia and Acute Pain Management. 2nd ed. New York, NY: McGraw-Hill Publishing; 2017:chap 56.

22/04/19 6:16 PM

56 Regional Anesthesia in Austere Environment Medicine M. Kwesi Kwofie

QUESTIONS DIRECTIONS: Choose the one best response to each question. 1.  Peripheral nerve blocks (PNBs) are well suited for anesthesia in austere environments over general anesthesia because: A. They allow optimal use of scarce resources. B. Equipment for PNBs is always available in developing countries. C. Routine monitoring is not required during PNB procedures. D. PNBs administered outside the operating room do not require the availability of airway equipment. 2.  General anesthesia with anesthetic vapors can be challenging in austere environments because: A. It provides poor operating conditions. B. It allows more patients to be treated with fewer resources. C. Draw-over anesthesia is widely used by most anesthesiologists. D. High-pressure oxygen may be required. 3.  Logistic challenges that may affect safe anesthesia plans include: A. Weather B. Terrain C. Availability of electricity D. All of the above 4.  Transportation of modern anesthesia infrastructure to austere environments is most often an effective approach for: A. Humanitarian medical mission planners B. Disaster medical mission planners C. Medical teaching missions D. Military combat support missions 5.  Benefits of continuous peripheral nerve blocks (CPNBs) in austere environments include: A. Improved weight bearing on a blocked limb B. Preventing further injury to the blocked limb

Hadzic_Ch56_p333-336.indd 333

C. No requirement for follow-up of patients with CPNBs D. Facilitating dressing changes 6.  Which of the following is the best course of action if a continuous peripheral nerve block (CPNB) site infection is suspected? A. Remove the catheter and resite 2 cm distally. B. Remove the catheter only if blood cultures are positive. C. Remove the catheter, culture the tip, and start antibiotics. D. Consult an infectious disease (ID) specialist to see if the catheter should be removed. 7.  Which of the following is true about continuous peripheral nerve blocks (CPNBs)? A. They need to be replaced at each repeated operation. B. They are the gold standard mode of anesthesia in developing countries. C. Their use is often limited by lack of training. D. The use of CPNBs has been limited by the advancement of technologies such as ultrasound. 8.  Which statement is true regarding opioids? A. They can be easily transported internationally. B. They play a major role in pain management in austere environment anesthesiology (AEA). C. They are routinely added to continuous peripheral nerve block (CPNB) infusions. D. Their use in austere environments is safer than in large developed health systems.

ANSWERS AND EXPLANATIONS 1. A is correct. Whenever possible, regional anesthesia is the preferred technique for most cases during an austere environment anesthesiology (AEA) mission because it allows conservation of resources for general anesthesia that may be required for more complicated surgeries or emergencies. Unfortunately, the technology and personnel support needed to employ general anesthesia successfully can exhaust the resources available during AEA. 333

19/04/19 11:06 AM

334

PART 7

Anesthesia in Patients with Specific Considerations

B is incorrect. Austere environments such as disaster zones, developing countries, and war zones can frequently suffer from an inability to acquire and maintain a consistent supply of any type of medicines and equipment. C is incorrect. Whether in or out of the OR, locations where PNBs are performed should have basic monitors, including ECG, blood pressure, and pulse oximetry. D is incorrect. Any anesthetizing location where PNBs are performed, whether in or out of the OR, should be equipped with airway equipment, suction, supplemental oxygen, and medications in the event of an airway emergency. 2. D is correct. Most modern anesthesia machines and workstations require the use of high-pressure oxygen (> 40–45 psi). A is incorrect. General anesthesia is the gold standard for anesthesia in both austere environments and developed modern healthcare systems. Operating conditions for most procedures are very good. B is incorrect. Regional anesthesia is the preferred technique for most cases during an AEA mission because it allows conservation of resources that may be required for more complicated surgeries, emergencies, or other cases not amenable to regional anesthesia. C is incorrect. Most anesthesia workstations in modern developed healthcare systems employ an anesthesia machine requiring high-pressure medical gas. One solution to this problem is a draw-over anesthesia vaporizer breathing system. Draw-over systems require no electricity or high-pressure gas for the function of the machine or the circuit.1 However, they use much larger flows and volumes of anesthetic vapors. Pressurized gas or electricity may be required for ventilator function. Unfortunately, training on these machines and vaporizers in modern training programs and knowledge of their use by the average anesthesia provider is often limited. 3 D is correct. Severe winds, rain, and floods can limit the ability to access necessary medicines and equipment. The presence of terrain or roads that are difficult to pass can cause difficulty with the supply chain of equipment and medicines in austere environments. Developing countries with poor transportation infrastructure as well as previously effective transportation infrastructure that has been damaged by natural and human-made disasters are common ways that terrain can limit anesthesia options in austere environments. Electricity is often required for the use of routine monitors and anesthesia equipment. Inconsistent electrical power delivery is a frequent problem in many developing countries and can be worsened by disaster environments complicating safe anesthesia delivery. 4. D is correct. One approach to overcoming the realities of austere environment anesthesiology (AEA) is to surmount local resource limitations by transporting the

Hadzic_Ch56_p333-336.indd 334

modern anesthesia infrastructure to the AEA location. This approach is best illustrated by the US military in caring for wounded soldiers on the battlefields of Iraq and Afghanistan through the fielding of combat support hospitals (CSHs) that have capabilities similar to civilian hospitals in developed countries. While the CSH is a successful approach to caring for injured patients anywhere in the world, the logistics required to place and support such a facility are well beyond the capability of most humanitarian medical mission planners or disaster medicine planners. An exception outside of the realm of the military is the medical mission, Mercy Ships, which has “floating hospitals” with modern equipment for medical missions that can be deployed around the world. A and B are incorrect. See explanation for answer D. C is incorrect. Medical teaching missions are often focused on training local practitioners to practice more effectively and safely use the equipment and medicines that are available to them, rather than transporting infrastructure at great cost to try to mimic modern well-funded and developed healthcare systems. Furthermore, such a budget is rarely available to medical teaching missions. 5. D is correct. CPNBs can extend the benefits of regional anesthesia for days into a patient’s recovery from surgery. This can have a profoundly positive influence on the care of patients who require evacuation over long distances or have injuries that necessitate frequent surgical interventions or dressing changes. The CPNB can be bolused with local anesthetic to reestablish a surgical-level block for dressing changes or surgery. A is incorrect. Patients receiving regional anesthesia as part of their care should be warned to avoid weight bearing on blocked extremities, which may increase the risk of falls. B is incorrect. Patients should be advised to take special precautions to avoid further injury to the blocked region of the body because normal protective sensations and proprioception will be diminished or absent. C is incorrect. Anesthesiologists must have an established plan for a follow-up of CPNB patients. Follow-up can be as simple as a telephone call, but CPNB patients and their catheters should be evaluated daily. Patients treated with CPNB catheters should also be educated on signs and symptoms of local anesthetic toxicity. The patient must have 24-hour access to an anesthesia provider should problems occur during the infusion. 6. C is correct. If a CPNB site infection is suspected, then the CPNB catheter should be removed, considerations given to culturing the catheter tip, and blood test/cultures ordered if appropriate. If an abscess is suspected or neurological dysfunction is present, consider prompt imaging studies and consultation with other appropriate specialties. Appropriate antibiotics should be administered and consultation with an infectious disease specialist should be considered.

19/04/19 11:06 AM

CHAPTER 56

A is incorrect. Placement of a catheter through infected tissue would be at least relatively contraindicated. B and D are incorrect. See explanation for answer C. 7. C is correct. Despite the excellent safety profile of PNB, the use of single-injection and catheter techniques can be limited by a lack of local proficiency.2 This may be improved by local training. A is incorrect. One of the benefits of CPNBs is that patients having repeated procedures or dressing changes may be able to have their CPNB bolused to reestablish surgical anesthesia if required. B is incorrect. General anesthesia remains the gold standard for anesthesia care throughout the world because of its versatility. However, when amenable, regional anesthesia in AEA may have a better safety profile, decreasing morbidity and mortality.2 D is incorrect. The wide availability and decrease in cost of portable ultrasound technologies have revolutionized the clinical practice of single-shot and continuous peripheral nerve blockade. 8. B is correct. Despite their risks and side effects, opioids remain a cornerstone of acute pain management in and out of austere environments. The use of regional anesthesia may decrease opioid requirements, which would decrease dose-related opioid requirements, such as respiratory depression. A is incorrect. International regulations concerning the transportation of opioids can complicate the availability of medications in AEA. C is incorrect. The addition of common opioids (eg, morphine, hydromorphone, fentanyl, sufentanil) to perineural infusions has not demonstrated benefit over systemic administration.3 Therefore, addition of opioids to perineural infusion may accumulate and potentiate opioid-related systemic side effects, so their perineural use should be discouraged.

Hadzic_Ch56_p333-336.indd 335

Regional Anesthesia in Austere Environment Medicine

335

D is incorrect. AEA may involve situations with limited patient access and monitoring. Noisy, low-light situations on an aircraft or absent electrical service after a disaster are frequent examples during AEA when patients may be at increased risk for unrecognized opioid-induced respiratory depression due to monitoring challenges.

References 1. Eltringham R, Sprenker C, Camporesi E. Anesthesia in difficult locations and in developing countries. In: Ehrenwerth J, Eisenkraft JB, Berry JM, eds. Anesthesia Equipment: Principles and Applications. 2nd ed. Philadelphia: Elsevier Saunders; 2013: 581-583. 2. Ariyo P, Trelles M, Helmand R, et al. Providing anesthesia care in resource-limited settings: a 6-year analysis of anesthesia services provided at Médecins Sans Frontières facilities. Anesthesiology. 2016 Mar;124(3):561-569. 3. Murphy DB, McCartney CJ, Chan VW. Novel analgesic adjuncts for brachial plexus block: a systematic review. Anesth Analg. 2000 May;90(5):1122-1128.

Suggested Reading Hadzic A. Regional anesthesia in austere environment medicine. In: Buckenmaier III CC, ed. Textbook of Regional Anesthesia and Acute Pain Management. 2nd ed. New York, NY: McGraw-Hill Publishing; 2017:chap 57.

19/04/19 11:06 AM

This page intentionally left blank

Hadzic_FM_p00i-pxx.indd 6

22/04/19 6:40 PM

57 Anesthesia for Humanitarian Relief Operations Steve Coppens and Leen Govaers

QUESTIONS DIRECTIONS: Choose the one best response to each question. 1.  Which of the following is an advantageous reason to choose ketamine during disaster situations? A. Extensive surgery is possible, and multiple doses can be given. B. Its use has been extensively proven in mass-casualty scenarios. C. Ketamine provides both anesthesia and analgesia; however, it requires a secured airway. D. Ketamine does not require a secured airway and laryngospasms do not occur during the use of ketamine. 2.  Propofol is advantageous over ketamine because: A. Anesthetists from high-resource settings are more familiar with giving propofol total intravenous anesthesia (TIVA) than using ketamine. B. Propofol is much more forgiving of temperature extremes during transport and storage compared to ketamine. C. Propofol TIVA 2% comes in high concentration and doesn’t require large volumes to be transported to the disaster area. D. Working alongside local anesthetic nurses and officers who have extensive experience with TIVA propofol improves safety and speeds turnover of cases. 3.  Regional anesthesia techniques are the preferred way to provide anesthesia in disaster situations because they: A. All provide stable hemodynamics requiring a minimum of resuscitation B. Are easy to perform, requiring minimal portable equipment C. Have minimal side effects D. Provide full anatomic coverage

4.  Regarding peripheral nerve blocks (PNB), which statement is true? A. They increase the burden of the limited drug supply in disaster situations when used as single-shot technique. B. They should be placed using indwelling catheters to reduce the opioid need in the postoperative phase and decrease the medical supply need. C. They should have a bolus and continuous infusion regiment when placed for postoperative care. This ensures adequate analgesia and again decreases medical supply need. D. Using indwelling catheters is a risk for infection in difficult environmental conditions. 5.  Regarding inhalational anesthesia, which statement is true? A. Requires oxygen cylinders to run a Boyle-type anesthetic device B. Needs oxygen which can be easily produced through the use of an oxygen concentrator providing oxygen at high bar pressures to drive a Boyle-type anesthetic device C. The use of a draw-over type anesthetic device can give hypoxic mixtures. D. The use of a draw-over type anesthetic device requires careful volatile agent monitoring. 6.  Which statement is true regarding patient injury patterns following earthquakes? A. Mostly limb injuries with a predominance of lower limb B. Mostly limb injuries with a predominance of upper limb C. Predominantly thoracic-abdominal traumas D. Show a strong link between extremity fractures and any other fractures 7.  Effective anesthesia response plan during disasters: A. Incorporates a plan for provider safety in the field B. Incorporates having at least five-patient capacity per day per provider C. Involves working under direct control of a central commanding field hospital at distance D. Should include rapid field deployment within 72 hours

337

Hadzic_Ch57_p337-340.indd 337

19/04/19 11:07 AM

338

PART 7

Anesthesia in Patients with Specific Considerations

8.  A portable regional anesthesia disaster kit should be equipped with what monitoring device? A. Finger pulse oximeter B. Automatic blood pressure (BP) measurement device C. Nerve stimulator D. Portable lightweight ECG monitor

ANSWERS AND EXPLANATIONS 1. B is correct. The use of ketamine under disaster conditions has been extensively described by Mulvey et al.1,2 During the Haiti earthquake disaster in 2010 and the tsunami in 2004 it has proven its usefulness with a low complication rate associated with the use of the drug. A is incorrect. Ketamine in repeat doses can cause tachyphylaxis. Surgery requiring more than 1 hour of work should probably not be performed and repeat doses are tricky to calculate. C is incorrect. Although it is a very potent drug, a secured airway is not necessary. The drug allows for spontaneous breathing in correctly administered doses. D is incorrect. Ketamine in combination with the placement of a laryngeal mask airway can sometimes produce laryngospasm.3 This could also be because ketamine causes extensive saliva secretion. Atropine may sometimes even be recommended. As seen in the explanation for answer C, it may be more advantageous to let the patient breathe spontaneously without securing the airway at all. 2. A is correct. Indeed most aid workers who work in modern facilities will be much more familiar with propofol TIVA than with the use of ketamine. However, ketamine has huge advantages; it’s both anesthetic and analgesic, it can be given IV and IM, and can be given while maintaining spontaneous breathing.4 B is incorrect. Ketamine comes in highly concentrated doses, which allow for multiple uses. Storage is much easier and is not influenced by temperatures during transport or storage. C is incorrect. Propofol even in 2% is not a high concentration and requires a huge stockpile. Propofol also does not have analgesic effect, so it must be combined with opioids. Propofol is a drain on transport logistics. Ketamine is much more forgiving in that way. D is incorrect. Most local anesthetic helpers will have much more experience with ketamine than they have with propofol. 3. B is correct. Most single-shot regional techniques require a minimum of portable equipment; a neurostimulator and needle set should be enough. There are even portable small ultrasound devices nowadays. For neuraxial techniques, only the needle set is needed. This is all in stark contrast with ventilators, monitoring equipment, tubing, and so on, needed for full anesthesia.

Hadzic_Ch57_p337-340.indd 338

A is incorrect. Although peripheral nerve blocks do provide stable hemodynamics, neuraxial techniques do not and in combination with potential loss of blood and sympathectomy, extensive fluid management is often needed. C is incorrect. Peripheral nerve blocks indeed have minimal side effects but again neuraxial techniques have been associated with severe hypotension and adverse outcome.5 D is incorrect. Regional anesthesia has limited anatomic coverage. Peripheral nerve blocks do not always cover all areas and neuraxial techniques are mainly helpful for lower limb injuries. 4. D is correct. Although a few have reported the use of CPNB in war or disaster zones, the chances of infection are much higher even if there is something similar to a field hospital. The few studies reporting on those indwelling catheters all rapidly transported their patients to abroad hospitals or highly equipped military field hospitals. A is incorrect. Of course the insertion of single-shot will require local anesthetic. It will cover analgesia for the first 12 hours and will instead decrease the opioid and multimodal analgesia need. B is incorrect. Using CPNB will increase the chances of infection and requires additional equipment and supplies, such as infusion pumps and tubing. Of course in scenarios where serial wound debridements or repeat surgery is necessary, they still could still be advantageous. Nonetheless, CPNBs are not the standard and will increase the burden of medical supply need. SS techniques are the standard and repeat SS nerve block is also a possibility without the need for extra medical supplies. C is incorrect. When using CPNB it is best to use bolusonly techniques to reduce demands on limited supplies and personnel. 5. A is correct. Oxygen cylinders at high bar pressure are needed to run our familiar Boyle-type anesthetic device. Oxygen cylinders are bulky, difficult, and unsafe to transport and therefore impractical to use. Especially in the first weeks after a disaster when field hospitals are still being built, electricity is scarce and means of transportation is limited (Figure 57–1).4 B is incorrect. Oxygen concentrators are sometimes the only way to reliably produce oxygen in disaster areas. However the oxygen they produce is only at 1 bar and is not enough to drive our Boyle anesthetic devices. It can be easily used for draw-over-type devices, however. Oxygen concentrators also need electricity (Figure 57–1). C is incorrect. Hypoxic mixtures cannot be given using a draw-over device; monitoring inspired oxygen content is therefore not vital, which is an added bonus in disaster areas. It’s also very economical in its oxygen usage (Figure 57–2). D is incorrect. Draw-over devices are nonrebreathing circuits. Agent monitoring is not absolutely essential (Figure 57–2).

19/04/19 11:07 AM

CHAPTER 57

Anesthesia for Humanitarian Relief Operations

Craniofacial

Thoracic

Abdomen

Limbs

339

17.73%

10.31%

7.12%

63.01%

FIGURE 57–1  Continuous-flow anesthetic apparatus.

Oxford inflating bellows

EMO

Oxygen

OMV

Valve

FIGURE 57–2  Draw-over apparatus.

6. A is correct. As seen in several analyzed studies, 51%–63% of all injuries due to earthquakes are traumatic limb injuries. Lower limb involvement seems invariably to exceed 60% of those cases (Figure 57–3).6–8 B is incorrect. Limb injuries seen in earthquake disasters are predominantly lower limb traumas. C is incorrect. Only 15%–20% of all injuries postearthquake are thoracic and abdominal cases. Thoracic trauma is usually severe and has a high mortality rate. D is incorrect. There is a very distinct link between skull, thorax, and spine fractures; however, there is a very weak link between extremity fractures and other fractures.

Hadzic_Ch57_p337-340.indd 339

Hadzic - Lan ncea/ NYSORA FIGURE 57–3  Anatomic distribution pattern of traumatic injuries immediately following major earthquakes.

7. A is correct. Safety in the field is one of the biggest priorities in any rapid anesthesia/surgery team deployment. If safety cannot be provided, care providers cannot perform their duties. B is incorrect. An effective plan should have as goal to be able to deal with at least 20–30 patients a day per provider. C is incorrect. With communication lines severed, and electricity and transportation limited or nonexistent, being able to work autonomously is a must. D is incorrect. Time equals lives. The first 24 hours are crucial and having a plan for a very rapid response team to be able to set up an advance post and take care of casualties before field hospitals can be deployed (which can take a few weeks) is crucial. 8. A is correct. A finger pulse oximeter is very compact and lightweight and also measures heart rate. It is essential in disaster areas and also in humanitarian aid programs. B is incorrect. Automatic BP measurement devices are bulky, use up battery power fast, and are not essential. Instead, carry manual BP cuffs in your kit. C is incorrect. Ultrasound devices will usually not be in the kit, so the neurostimulator is not used as a monitoring device. Instead it is the essential nerve block equipment. D is incorrect. ECG monitors are very bulky, use up battery power, and provide little information that small finger pulse oximeters cannot give. They should not fill up space in your kit.

19/04/19 11:07 AM

340

PART 7

Anesthesia in Patients with Specific Considerations

References 1. Mulvey JM, Qadri AA, Maqsood MA. Earthquake injuries and the use of ketamine for surgical procedures: the Kashmir experience. Anaesth Intensive Care. 2006;34:489-494. 2. Mulvey JM, Awan SU, Qadri AA, Maqsood MA. Profile of injuries arising from the 2005 Kashmir earthquake: the first 72 h. Injury. 2008;39:554-560. 3. Green SM, Roback MG, Krauss B. Laryngospasm during emergency department ketamine sedation: a case-control study. Pediatr Emerg Care. 2010 Nov;26(11):798-802. 4. Craven RM. Managing anaesthetic provision for global disasters. BJA: British Journal of Anaesthesia. 2017 Dec;119(1):i126–i134. https://doi.org/10.1093/bja/aex353 5. Jiang J, Xu H, Liu H, et al. Anaesthetic management under field conditions after the 12 May 2008 earthquake in Wenchuan, China. Injury. 2010;41:1-3.

Hadzic_Ch57_p337-340.indd 340

6. Malish R, Oliver DE, Rush RM Jr, et al. Potential roles of militaryspecific response to natural disasters—analysis of the rapid deployment of a mobile surgical team to the 2007 Peruvian earthquake. Prehosp Disaster Med. 2009;24:3-8. 7. Goncharov SF. Medical consequences of earthquake disasters in Russia. In: World Health Organization Centre for Health Development: Earthquakes and people’s health: proceedings of a WHO symposium, part 2. Kobe, Japan. January 27–30, 1997. 8. Missair A, Pretto EA, Visan A, et al. A matter of life or limb? A review of traumatic injury patterns and anesthesia techniques for disaster relief after major earthquakes. Anesth Analg. 2013;117:934-941.

Suggested Reading Hadzic A. Anesthesia for humanitarian relief operations. In: Missair A, ed. Textbook of Regional Anesthesia and Acute Pain Management. 2nd ed. New York, NY: McGraw-Hill Publishing; 2017:chap 58.

19/04/19 11:07 AM

PART 8 Regional Anesthesia in the Emergency Department Chapter 58

Hadzic_Ch58_p341-346.indd 341

 egional Anesthesia and Acute Pain Management in the R Emergency Department  343

22/04/19 2:11 PM

This page intentionally left blank

Hadzic_FM_p00i-pxx.indd 6

22/04/19 6:40 PM

58 Regional Anesthesia and Acute Pain Management in the Emergency Department Sam Van Boxstael and Pascal Vanelderen

QUESTIONS DIRECTIONS: Choose the one best response to each question. 1. Which nerve needs to be blocked to produce an excellent anesthesia and analgesia to remove foreign bodies from the sole of the foot? A. Tibial nerve B. Superficial peroneal nerve C. Deep peroneal nerve D. Saphenous nerve 2. Which of the following statements is true regarding ketamine? A. Ketamine has a unique dissociative anesthetic effect at low doses (0.1–0.3 mg/kg IV). B. Ketamine is an NMDA receptor agonist. C. Ketamine is a potent analgesic at low doses (0.1–0.3 mg/kg IV). D. S-ketamine is not suitable for use in the ED due to substantially increased risk of psychotropic side effects. 3. Which of the following agents is suitable for emergency topical anesthesia in nonintact skin? A. Eutectic 1:1 mixture of 2.5% lidocaine and 2.5% prilocaine (EMLA) B. 4% lidocaine cream in a liposomal matrix (LMX) C. vapocoolant spray D. lidocaine 4%, epinephrine 0.1%, and tetracaine 0.5% (LET) 4. Which of the following statements is true regarding nitrous oxide sedation? A. Frequently observed side effects are nausea and vomiting, euphoria, and dizziness. B. The odds of vomiting increase with concomitant administration of opioids or when clear fluids were ingested < 2 h before N2O administration.

C. Oxygen desaturation and generalized tonic-clonic seizures are increasingly seen with higher N2O concentrations and prolonged administration. D. All of the above 5. A 30-year-old patient presents to the ED with a dislocated elbow without neurovascular lesions. A reduction of the dislocation needs to be performed. Which regional anesthetic technique that most reliably covers the elbow and distal structures of the arm, would you suggest? A. Supraclavicular brachial plexus block B. Infraclavicular brachial plexus block C. Interscalene brachial plexus block D. Superficial cervical plexus block 6. Which statement is true regarding regional anesthesia for rib fractures? A. Epidural analgesia is associated with a higher risk of pneumonia and an increase in ventilator days compared to systemic opioid analgesia. B. Intercostal blocks are seldom associated with local anesthetic toxicity. C. In patients with multiple rib fractures, intercostal nerve blocks provide more consistent analgesia with lower volumes of local anesthetics compared to epidural analgesia. D. Paravertebral nerve block is an alternative for patients with contraindications for epidural analgesia. 7. Which of the following applications of the superficial cervical plexus block in the ED will result in insufficient analgesia? A. Laceration repair in the neck B. Clavicle fracture analgesia C. Ear lobe laceration repair D. Incision and drainage of a deltoid abscess

343

Hadzic_Ch58_p341-346.indd 343

22/04/19 2:11 PM

344

PART 8

Regional Anesthesia in the Emergency Department

8. An 80-year-old woman is brought to the ED with hip fracture. Your colleague suggests a fascia iliaca block to alleviate her pain. Which of the following nerves does a fascia iliaca block target? A. Femoral nerve, sciatic nerve, obturator nerve B. Femoral nerve, obturator nerve, lateral femoral cutaneous nerve C. Obturator nerve D. All of the above

TABLE 58–1  Emergency topical anesthesia.a

9. Which peripheral nerve block is most suited for reduction of shoulder dislocations? A. Supraclavicular brachial plexus block B. Infraclavicular brachial plexus block C. Interscalene brachial plexus block D. Superficial cervical plexus block 10. Oligoanalgesia in the emergency department is caused by which of the following factors? A. Difficulty in adequately assessing pain B. Apprehension about opioid dependence C. Patient ethnicity D. All of the above

ANSWERS AND EXPLANATIONS 1. A is correct. Tibial nerve needs to be blocked to produce an excellent anesthesia and analgesia to remove foreign bodies from the sole of the foot. B is incorrect. The dorsum of the foot is innervated by the superficial peroneal nerve. C is incorrect. The webspace between first and second toes is innervated by the deep peroneal nerve. D is incorrect. The saphenous nerve innervates the medial malleolus and a variable portion of the medial side of the leg below the knee. 2. C is correct. Ketamine is a potent analgesic at low doses (0.1–0.3 mg/kg IV). A is incorrect. Ketamine is an NMDA receptor antagonist with a unique dissociative anesthetic effect at higher doses (1–2 mg/kg IV). B is incorrect. Ketamine is an NMDA receptor antagonist. D is incorrect. While not available in the USA, S-ketamine is uniquely suitable for use in the ED due to substantially reduced psychotropic side effects. 3. D is correct. Lidocaine 4%, epinephrine 0.1%, and tetracaine 0.5% (LET), is suitable for emergency topical anesthesia in non-intact skin. A, B, and C are incorrect. These agents are suitable for emergency topical anesthesia in the intact skin. See Table 58–1.

Hadzic_Ch58_p341-346.indd 344

Agent

Action

Intact Skin

 

Eutectic 1:1 mixture of 2.5% lidocaine and 2.5% prilocaine (EMLA)

Onset is 60 min; occlusive dressing is required; methemoglobinemia is a rare complication of prilocaine

4% lidocaine cream in a liposomal matrix (LMX)

Onset 30 min, and no occlusive dressing is required

Vapocoolant spray

Ethyl chloride is the most commonly used agent. Spray until skin blanches for up to 10 s 3–9 in. from skin surface; duration is approximately 1 min after blanching

Nonintact Skin

 

Lidocaine 4%, epinephrine 0.1%, and tetracaine 0.5% (LET)

Onset 30 min after direct application to open wound

Other techniques for delivering lidocaine topical anesthesia such as iontophoresis and needle-free jet delivery remain uncommon in the emergency department and are under study. Agents containing cocaine and benzocaine are less commonly used owing to concerns about systemic toxicity (cocaine) and methemoglobinemia (benzocaine). a

4. A, B, and C are all true, so option D is correct. N2O frequently induces vomiting, euphoria and dizziness. The odds of vomiting also increase with concomitant administration of opioids or when clear fluids were ingested 20 psi followed by injection pressure < 5 psi is a sign of extraneural injection. B. Nerve stimulation at < 0.2 mA is the most sensitive sign of intrafascicular needle-tip placement. C. Pressure monitoring can be used to detect intravascular injection. D. Ideally, local anesthetic spread is visualized in the desired tissue plane with opening pressure < 15 psi, and an absence of an evoked motor response at 0.5 mA. 9. Key components of block documentation include: A. Anticipated sensory changes from nerve block B. Duration of nerve block C. Indication for nerve block D. Rate of injection in mL/sec

363

Hadzic_Ch62_p363-366.indd 363

22/04/19 4:24 PM

364

PART 9

Complications of Local and Regional Anesthesia

10. Reasons to include an image or video clip of a peripheral nerve block in a patient’s chart include all of the following except: A. Billing for the nerve block B. Billing for the use of ultrasound guidance C. Demonstrating pertinent findings experienced during block placement D. Evidence in medicolegal cases

ANSWERS AND EXPLANATIONS 1. C is correct. 10 mcg. Intravenous injection of 10–15 mcg of epinephrine reliably increases the systolic blood pressure greater than 15 mm Hg even in the presence of beta blockade and sedation. This has become the standard dose for detection of intravascular injectate in regional anesthetic procedures. A and B are incorrect. Though a typical concentration of epinephrine used is 2.5–5 mcg/mL (1:200,000 and 1:400,000, respectively), an amount of 2.5–5 mcg of epinephrine is insufficient to produce a reliable blood pressure increase. D is incorrect. 50 mcg of epinephrine would certainly result in a hemodynamic response in most patients. However, in the concentrations typically used (2.5–5 mcg/mL) this would mean administering 10–20 mL of local anesthetic to reach a dose of 50 mcg. This essentially negates the purpose of the test dose, which is to minimize the volume accidentally injected into the vasculature. Increasing the concentration of epinephrine in the block solution above 2.5–5 mcg/mL is not recommended as there is a risk of vasoconstriction and vascular compromise to the nerve and adjacent tissues. 2. B is correct. Numerous animal and human studies have suggested that 0.2 mA is a current intensity threshold below which intimate needle-nerve contact can be expected. When an evoked motor response is present at less than 0.2 mA, there is a high likelihood of intraneural needle-tip placement and intraneural inflammation after injection. A is incorrect. Voelckel et al examined sciatic nerve blocks with evoked motor response < 0.2 mA and at 0.3–0.5 mA. A motor response at a current of < 0.2 mA was always either intraneural or very close to the epineurium. This was further supported by Wiesmann et al, who demonstrated similar findings in a brachial plexus model. C and D are incorrect. The 0.3–0.5 mA cohort in the Voelckel et al study showed no evidence of nerve tissue inflammation. 3. C is correct. An analysis of more than 25,000 block procedures by Barrington and Kluger revealed that incidence of LAST was reduced by 65%–80% (depending on the propensity analysis used) when ultrasound guidance was used compared to blocks done without ultrasound. The

Hadzic_Ch62_p363-366.indd 364

reasons cited for this finding include the ability to avoid vascular trauma, the possible early detection of intravascular injection, and the reduction in overall volumes of local anesthetic required to achieve a sufficient block when ultrasound guidance is used. A is incorrect. Current ultrasound technology cannot reliably differentiate between intrafascicular and extrafascicular nerve injections. This is important as intrafascicular injections result in clinical nerve damage, whereas extrafascicular injections are not likely to be associated with damage. B is incorrect. UGRA has not been shown to decrease the risk or the incidence of nerve damage. Reasons for this are multiple, but may relate to the wide variation in user ability, patient anatomy, lack of recognition of intraneural injection, and inadequate ultrasound resolution. D is incorrect. Due to close proximity of pleura to brachial plexus in the supraclavicular region, this block was previously unpopular despite being highly effective for upper extremity surgery. Supraclavicular block has enjoyed a resurgence in popularity due to its relatively straightforward sonoanatomical features, and reliability. 4. B is correct. Several studies have demonstrated that OIP > 15–20 is associated with poor neurologic outcome and/or directly observed intraneural injection using ultrasound. A is incorrect. 5 psi is a safe OIP, as pressures below 15 psi are consistently associated with safe needle-nerve conditions. However, limiting pressure to that low of a value may be excessively restrictive and there is no evidence that 5 psi as an upper limit is “more” safe than 15 psi. C and D are incorrect. Hadzic et al showed intrafascicular canine sciatic nerve injections consistently had OIP > 20 psi while both perineural and intraneural, extrafascicular injections had low OIP. Krol et al studied median, ulnar, and radial nerve injections in cadavers and showed intraneural injection was always > 15 psi while extraneural OIP was < 10 psi. 5. D is correct. In-line manometry is a simple way to continuously monitor injection pressure. These monitors function via a spring-loaded membrane that connects between the syringe and tubing system, detecting pressure in real-time. Some display a pressure range (0–15 psi, 15–20 psi, and greater than 20 psi), while others simply have a hydraulic valve that shuts off flow when pressure reaches 15 psi. A is incorrect. The compressed-air injection technique is another way to monitor injection pressure. B is incorrect. There is no reliable way to detect injection pressure via ultrasound. Doppler ultrasound imaging shows flow toward or away from the ultrasound transducer. C is incorrect. “Hand feel” is not a reliable or reproducible method of pressure monitoring. Experienced anesthesiologists appear to be unable to distinguish between intraneural injection and injection into other soft tissues during in vitro studies.

22/04/19 4:24 PM

CHAPTER 62

Monitoring, Documentation, and Consent for Regional Anesthesia Procedures

6. A is correct. Boyle’s law states at a fixed temperature, pressure and volume are inversely proportional. In the compressed injection technique, compressing 10 mL of air in a syringe to 5 mL (ie, halving the volume) doubles the pressure in the system. Because atmospheric pressure is 14.7 psi, the pressure in the system is an additional 14.7 psi, which is close to the recommended safe upper limit of opening injection. B is incorrect. Charles’ law states if a gas is held at a fixed pressure, temperature and volume will be directly related. C is incorrect. Gay-Lussac’s law states the pressure of a fixed volume of gas is dependent upon its temperature. D is incorrect. Henry’s law states at a constant temperature, the amount of gas dissolved in a volume of liquid is directly proportional to the partial pressure of the gas. 7. A is correct. An inflammatory response is seen histologically, along with disruption of nerve architecture after forceful needle-nerve contact, even without injection of local anesthetic. B is incorrect. Intrafascicular injection is associated with clinical and histologic damage and should be avoided. C is incorrect. An evoked motor response below 0.5 mA signifies close needle-nerve proximity and the needle should be slightly withdrawn in this case. D is incorrect. Ultrasound cannot be used to reliably show needle-nerve contact. Studies have shown that intraneural injection does not always result in a visible increase in nerve area and once nerve swelling is visible on ultrasound, damage to nerves may have already occurred. 8. D is correct. Ideally, all three monitors are used in a complementary fashion: local anesthetic spread is visualized in the desired plane on ultrasound, opening injection pressure is below 15 psi, and there is no evoked motor response at a low-current (< 0.5 mA) intensity. A is incorrect. This pressure pattern of excessively high followed by low injection pressure is suggestive of intrafascicular injection and rupture of the fascicle. B is incorrect. In a study of ultrasound-guided interscalene brachial plexus blocks, the most sensitive sign of intrafascicular injection was opening injection pressure of greater than 15 psi. This was more sensitive than nerve stimulation below 0.2 mA and presence of paresthesia. C is incorrect. Injection pressure monitoring will not reliably indicate intravascular injection of local anesthetic. 9. C is correct. Indication or reason for performing a nerve block, whether for surgical anesthesia or for postoperative pain, should be clearly documented in the block note.

Hadzic_Ch62_p363-366.indd 365

365

A is incorrect. Anticipated sensory changes that are not yet present on exam do not need to be documented in the block procedure note. B is incorrect. Duration of the nerve block is not a necessary component of the block procedure note. Furthermore, the block note should be completed at the time of the procedure, and not hours/days afterward when the block has resolved. D is incorrect. Rate of injection may be related to injection pressure, but this is often difficult to estimate or measure directly. More important than the overall rate is the avoidance of a forceful injection that results in high injection pressure. 10. A is correct. In the United States, an image of the nerve block is not required to meet compliance standards for billing Medicare or third-party insurers for the placement of the nerve block. However, billing for ultrasound guidance for nerve blocks does require saving an image of the procedure. B is incorrect. In order to bill for use of ultrasound guidance, an electronic or printed copy of the image needs to either be placed in the medical record, or be archived in a location that can be accessible should an insurer or CMS audit the use of those billing codes. C is incorrect. Including imaging of the nerve block allows documentation of pertinent findings such as spread of local anesthetic around nerve bundle. This becomes especially useful if an unexpected event occurs such as prolonged block or a postoperative paresthesia. The image can be used to better understand the relationship between the nerve/plexus, local anesthetic, and needle. D is incorrect. Video evidence of the block can be used for defense in medicolegal cases if the imaging is part of the medical record. This is probably most useful if the video clip contains the final approach of the needle toward the nerve/plexus as well as the initial injection and expansion of local anesthetic adjacent to the neural and/or vascular structures.

Suggested Readings Hadzic A. Monitoring, documentation, and consent for regional anesthesia procedures. In: Gadsden J, ed. Textbook of Regional Anesthesia and Acute Pain Management. 2nd ed. New York, NY: McGraw-Hill Publishing; 2017:chap 63. Gadsden JC, Choi JJ, Lin E, Robinson A. Opening injection pressure consistently detects needle–nerve contact during ultrasoundguided interscalene brachial plexus block. Anesthesiology. 2014;120:1246-1253. Sala-Blanch X, Ribalta T, Rivas E, et al. Structural injury to the human sciatic nerve after intraneural needle insertion. Reg Anesth Pain Med. 2009;34:201-205.

22/04/19 4:24 PM

This page intentionally left blank

Hadzic_FM_p00i-pxx.indd 6

22/04/19 6:40 PM

63 Diagnosis and Management of Spinal and Peripheral Nerve Hematoma Thomas F. Bendtsen and Siska Bjørn

QUESTIONS DIRECTIONS: Choose the one best response to each question. 1.  Spinal epidural hematoma: A. Is an accumulation of blood between the dura and the arachnoid of the spinal canal B. Is always spontaneous C. Incidence is decreasing D. Is typically asymptomatic 2.  Which of the following statements is correct regarding the risks factors of spinal epidural hematoma? A. Anticoagulation therapy is a risk factor. B. Female gender is not a risk factor. C. Increasing age is not a risk factor. D. Race is a risk factor. 3.  Which of the following statements is true regarding diagnosing spinal epidural hematoma? A. Acute axial back pain is a typical presenting symptom. B. A complete blood cell count is relevant to diagnose spinal epidural hematoma. C. CT myelography is the gold standard diagnostic measure. D. Spinal epidural hematoma typically appears within 3 hours after the procedure. 4.  In case of epidural anesthesia followed by back pain, which statement arouses the strongest suspicion of spinal epidural hematoma? A. Sensory or motor deficit returns several hours after the epidural block has initially worn off. B. The last epidural dose was given less than 3 hours ago. C. The patient has moderate to severe back pain without any neurologic deficits. D. Valsalva maneuver and palpation of the procedural site do not aggravate the pain.

5.  Which statement is true regarding imaging methods for diagnosis of spinal epidural hematoma? A. Conventional CT is a reliable diagnostic method regardless of the location of the hematoma. B. CT myelography is more sensitive and specific compared to MRI. C. MRI can be used to assess the age of the hematoma. D. On MRI a spinal epidural hematoma will typically be seen located ventral to the thecal sac. 6.  Which of the following statements is correct regarding neurologic recovery after spinal epidural hematoma? A. Complete neurologic recovery can be achieved when surgery is performed within 36 hours for complete motor deficit and within 48 hours for incomplete motor deficit. B. Surgery is always indicated, as complete neurologic recovery can never occur with conservative treatment. C. Functional recovery has never been reported when surgery was performed more than 72 hours after onset of symptoms. D. The prognosis is not related to the level of preoperative neurologic deficit. 7.  Peripheral hematoma after nerve blocks: A. Is most commonly diagnosed with a CT scan B. Is not associated with deep plexus blocks and anticoagulation therapy C. Typically presents with neurologic dysfunction D. Typically requires invasive therapy

ANSWERS AND EXPLANATIONS 1. D is correct. Spinal epidural hematoma is typically asymptomatic. A is incorrect. An accumulation of blood between the dura and the arachnoid is a spinal subdural hematoma. A spinal epidural hematoma is defined as an accumulation of blood between the vertebrae and the dura of the spinal canal. 367

Hadzic_Ch63_p367-370.indd 367

19/04/19 11:15 AM

368

PART 9

Complications of Local and Regional Anesthesia

B is incorrect. Spontaneous spinal epidural hematoma can occur, but it is a rare condition with an estimated incidence of 0.1 patients per 100,000 patients per year.1 Spinal epidural hematoma can also be caused by invasive procedures in or near the epidural space or by a traumatic injury. C is incorrect. Previously, the reported incidence of spinal epidural hematoma was less than 1:150,000 for epidural and less than 1:220,000 for spinal anesthesia.2 However, newer studies show higher incidences of spinal epidural hematoma after neuraxial anesthesia. In elderly, female patients undergoing total knee arthroplasty an incidence of 1:3600 has been reported.3,4 2. A is correct. Anticoagulation therapy is a risk factor. B is incorrect. Female gender is a risk factor. An explanation could be the higher prevalence of osteoporosis in females. Osteoporosis causes narrowing of the epidural space, thereby increasing the risk of a spinal epidural hematoma.3-5 C is incorrect. With increasing age the epidural space becomes narrower and less compliant. Osteoporosis may account for the association to both female gender and increasing age.3-5 D is incorrect. Race is not a known risk factor of spinal epidural hematoma. 3. A is correct. Acute axial back pain is a typical presenting symptom.6-8 B is incorrect. A complete blood cell count is not relevant to diagnose spinal epidural hematoma, but it should be performed to assess the extent of the bleeding, if an infection is present and to check for bleeding diathesis.9 C is incorrect. CT myelography was previously the gold standard for diagnosing spinal epidural hematoma. It has now been replaced by MRI, which is noninvasive and more specific compared to CT myelography.10,11 D is incorrect. Spinal epidural hematoma typically presents within the first 24–48 hours after a procedure performed in or near the epidural space. If the last epidural dose was given less than 3 hours ago it will lessen the suspicion of spinal epidural hematoma.9 4. A is correct. Return of sensory or motor deficit after the epidural block has initially worn off should arouse the strongest suspicion of spinal epidural hematoma. B is incorrect. Spinal epidural hematoma typically presents within the first 24–48 hours after a procedure performed in or near the epidural space. If the last epidural dose was given less than 3 hours ago, a spinal epidural hematoma is not the primary suspicion.9 C is incorrect. Acute axial back pain is a typical presenting symptom, but the absence of any neurologic deficits will lessen the suspicion of spinal epidural hematoma. In case of new or progressive neurologic symptoms, immediate clinical evaluation and diagnostic workup are always mandated to rule out spinal epidural hematoma.9

Hadzic_Ch63_p367-370.indd 368

D is incorrect. If a spinal epidural hematoma is present, palpation of the area will typically worsen the pain. Additionally, actions such as Valsalva maneuver that increase the intraspinal pressure will aggravate the pain.9 5. C is correct. MRI can be used to assess the age of the hematoma. A is incorrect. Conventional CT can be used for diagnosis in the lumbar and cervical spine, but there is a greater risk of false negative results compared to CT myelography or MRI. In the thoracic spine, resolution is poorer and conventional CT cannot be used as a reliable diagnostic measure.10 B is incorrect. CT myelography was previously the gold standard, but it has been replaced by MRI, which is more sensitive and specific.10 D is incorrect. Ventrally in the spinal canal, the dura is firmly adherent to the posterior longitudinal ligament, thereby preventing the accumulation of blood. Consequently, a spinal epidural hematoma is typically located posterior or posterolateral to the thecal sac.9,12 6. A is correct. Complete neurologic recovery can be achieved when surgery is performed within 36 hours for complete motor deficit and within 48 hours for incomplete motor deficit. B is incorrect. There have been case reports describing successful nonoperative treatment of spinal epidural hematoma.13-15 Several factors are important, and complete recovery without surgery is most likely in cases with only mild neurologic deficit, and in cases where the hematoma is localized at the cauda equina level.16 C is incorrect. Functional recovery has been reported when surgery was performed after more than 72 hours of symptoms.17,18 However, the prognosis is worse when there is a delay between injury and surgical intervention. Therefore it is critical to be aware of and react to the symptoms, as the process of obtaining an MRI and consulting with a spine surgeon will delay the time to intervention. D is incorrect. The preoperative level of neurologic deficit and the operative interval are the most critical factors affecting recovery after spinal epidural hematoma.19,20 7. A is correct. Peripheral hematoma after nerve blocks is most commonly diagnosed with a CT scan.21 B is incorrect. Deep plexus blocks with poorer ultrasound visibility and a longer needle path compared to superficial blocks increase the risk of blood vessel puncture. In case of puncture of deep vessels, compression is complicated and therefore deep blocks are less suitable for anticoagulated patients compared to superficial nerve blocks.22 Reports of severe hemorrhagic complications including death after deep plexus blockade in anticoagulated patients exist.23,24 For patients undergoing deep plexus or peripheral block, the American Society of Regional Anesthesia and Pain Medicine recommends that the guidelines regarding neuraxial techniques be similarly applied.23

19/04/19 11:15 AM

CHAPTER 63

369

Diagnosis and Management of Spinal and Peripheral Nerve Hematoma

C is incorrect. In contrast to spinal epidural hematoma, the presenting symptom is typically not neurologic dysfunction but instead tenderness or pain in the area, bruising, and swelling. More extensive peripheral hematomas can present with hypotension due to blood loss, which is especially seen with hematomas located in the retroperitoneal space.9,22 D is incorrect. The strategy is typically expectant management. The patient is observed carefully and blood transfusion can be given as needed. Invasive procedures such as surgical drainage are considered only in case of clinical deterioration.9

References 1. Holtås S, Heiling M, Lönntoft M. Spontaneous spinal epidural hematoma: findings at MR imaging and clinical correlation. Radiology. 1996;199(2):409-413. 2. Tryba M. Epidural regional anesthesia and low molecular heparin: pro [in German]. Anasthesiol Intensivmed Notfallmed Schmerzther. 1993;28:179-181.

infection on postoperative MRI. Clin Spine Surg. 2016;29(9): E471-E474. 12. Fukui M, Swarnkar A, Williams R. Acute spontaneous spinal epidural hematomas. Am J Neuroradiol. 1999;20:1365-1372. 13. Pahapill PA, Lownie SP. Conservative treatment of acute spontaneous spinal epidural hematoma. Can J Anaesth. 1998;25:159-163. 14. Schwarz SK, Wong CL, McDonald WN. Spontaneous recovery from a spinal epidural hematoma with atypical presentation in a nonagenarian. Can J Anesth. 2004;51:557-561. 15. Tailor J, Dunn IF, Smith E. Conservative management of spontaneous spinal epidural hematoma associated with oral anticoagulant therapy in a child. Childs Nerv Syst. 2006;22: 1643-1645. 16. Johnston RA. The management of acute spinal cord compression. J Neurol Neurosurg Psychiatr. 1993;56:1046-1054. 17. Enomato T, Maki Y, Nakagawa K, et al. Spontaneous spinal epidural hematoma: report of a case. Neurol Surg. 1980;8:875-880. 18. Yao YX, Li MX, Sun LJ. Good outcomes after the delayed removal of an epidural hematoma. A case report. Medicine (Baltimore). 2018;87(14).

3. Moen V, Dahlgren N, Irestedt L. Severe neurological complications after central neuraxial blockades in Sweden in 1990–1999. Anesthesiology. 2004;101:950-959.

19. Foo D, Rossier A. Preoperative neurological status in predicting surgical outcome of spinal epidural hematomas. Surg Neurol. 1981;15:389-340.

4. Popping DM, Zahn PK, Van Aken HK, Dasch B, Boche R, Pogatzki-Zahn EM. Effectiveness and safety of postoperative pain management: a survey of 18,925 consecutive patients between 1998 and 2006 (2nd revision): a database analysis of prospectively raised data. Br J Anaesth. 2008;101:832-840.

20. Groen RT, Van Alphen HA. Operative treatment of spontaneous spinal epidural hematomas: a study of the factors determining postoperative outcome. Neurosurgery. 1996;39:494-508. 21. Monib S, Ritchie A, Thabet E. Idiopathic retroperitoneal hematoma. J Surg Tech Case Rep. 2011;3:49-51.

5. Cummings SR, Nevitt MC, Browner WS, et al. The study of osteoporotic fractures research group: risk factors for hip fracture in white women. N Engl J Med. 1995;332:767-773.

22. Enneking FK, Chan V, Greger J, Hadzic A, Lang SA, Horlocker TT. Lower-extremity peripheral nerve blockade: essentials of our current understanding. Reg Anesth Pain Med. 2005;30:4-35.

6. Mattle H, Sieb J, Rohner M, et al. Nontraumatic spinal epidural and subdural hematomas. Neurology. 1987;37:1351-1356.

23. Horlocker T, Wedel DJ, Rowlingson JC, et al. Regional anesthesia in the patient receiving antithrombotic or thrombolytic therapy: American Society of Regional Anesthesia and Pain Medicine evidence-based guidelines (third edition). Reg Anesth Pain Med. 2010;35:64-101.

7. Cooper DW. Spontaneous spinal epidural hematoma. Case report. J Neurosurg. 1967;26:343-345. 8. Figueroa J, DeVine JG. Spontaneous spinal epidural hematoma: literature review. J Spine Surg. 2017;3(1):58-63. 9. Hadzic A. Textbook of Regional Anesthesia and Acute Pain Management. 2nd ed. New York, NY: McGraw-Hill Publishing; 2017. 10. Avrahami E, Tadmor R, Ram Z, et al. MR demonstration of spontaneous acute epidural hematoma of thoracic spine. Neuroradiology. 1989;31:89-92. 11. Radcliff K, Morrison WB Kepler C, Moore J, Sidhu GS, Gendelberg D. Miller, Songali MA, Vaccaro AR. Distinguishing pseudomeningocele, epidural hematoma, and postoperative

Hadzic_Ch63_p367-370.indd 369

24. Maier C, Gleim M, Weiss T, Stachetzki U, Nicolas V, Zenz M. Severe bleeding following lumbar sympathetic blockade in two patients under medication with irreversible platelet aggregation inhibitors. Anesthesiology. 2002;97:740-743.

Suggested Reading Hadzic A. Diagnosis and management of spinal and peripheral nerve hematoma. In: Nelson A, Benzon HT, Jabri RS, eds. Textbook of Regional Anesthesia and Acute Pain Management. 2nd ed. New York, NY: McGraw-Hill Publishing; 2017:chap 64.

19/04/19 11:15 AM

This page intentionally left blank

Hadzic_FM_p00i-pxx.indd 6

22/04/19 6:40 PM

PART 10 LAST: Local Anesthetic Systemic Toxicity

Chapter 64

Hadzic_Ch64_p371-376.indd 371

Local Anesthetic Systemic Toxicity 373

22/04/19 4:29 PM

This page intentionally left blank

Hadzic_FM_p00i-pxx.indd 6

22/04/19 6:40 PM

64 Local Anesthetic Systemic Toxicity Steve Coppens and Leen Govaers

QUESTIONS DIRECTIONS: Choose the one best response to each question. 1. The pillars of local anesthetic systemic toxicity (LAST) treatment consist of: A. Administration of propofol induction dose, for counteracting seizures B. Airway management and circulatory support C. Avoiding further alkalosis until initiation of lipid emulsion therapy D. Prompt administration of a 2% lipid emulsion 2. Patient-related risk factors for local anesthetic systemic toxicity (LAST) are: A. Extreme muscle mass, because skeletal muscle acts as a depot for local anesthetic (LA) B. Having epilepsy and seizures as a comorbidity C. Malnutrition D. Site of injection 3. Lipid resuscitation therapy (LRT): A. And propofol together worsen lipid accumulation B. Has clinical side effects such as bronchospasm, laboratory interferences, and hyperamylasemia C. Has side effects such as transaminitis, hepatosplenomegaly, and bacterial contamination D. Works solely as a “lipid sink” 4. What statement is true regarding the incidence of local anesthetic systemic toxicity (LAST)? A. The incidence of LAST has not decreased over the last years, due to the rise of cases in regional anesthesia. B. The incidence of LAST is higher in lower extremities due to the richly vascular area. C. The incidence of LAST is highest in patients with glutamine deficiency. D. The incidence of LAST is highest with paravertebral blocks.

5. Which statement is true regarding the prevention of local anesthetic systemic toxicity (LAST)? A. Complete ASA monitoring is the key. B. Frequent aspiration is one of the key elements. C. The ultrasound’s main advantage is the ability to use incremental doses. D. An intravascular marker such as 1–2 µg of epinephrine can be used as a test dose. 6. Which statement is true regarding local anesthetic systemic toxicity (LAST)? A. LAST after inferior alveolar blocks is rare due to the injection next to the vessel-poor area of the pterygomandibular space. B. LAST can even occur with topical use of LAs for awake intubations. C. LAST from a retrobulbar block is caused by epidural spread of the anesthetic causing brainstem anesthesia. D. LAST occurrences after transverse abdominis plane (TAP) blocks for cesarean delivery are due to higher serum albumin levels of pregnant patients. 7. Cardiac toxicity during local anesthetic systemic toxicity (LAST): A. Can even occur at low serum concentrations because LAs accumulate in mitochondria and cardiac tissue at a ratio of 1:6 relative to plasma B. Is based on high concentrations of LA facilitating the carnitine-acylcarnitine translocase in mitochondria C. Is caused by electrophysiologic and contractile dysfunction D. Is higher with bupivacaine because of its low lipophilicity and strong affinity for the voltage-gated sodium channel 8. Why are children more susceptible to local anesthetic systemic toxicity (LAST)? A. Because most children are anesthetized when a block is placed B. Because they are more vessel-rich and accidental intravascular injection and absorption are more likely C. Because they have decreased serum levels of albumin D. Because they may have diminished muscle mass

373

Hadzic_Ch64_p371-376.indd 373

22/04/19 4:29 PM

374

PART 10

LAST: Local Anesthetic Systemic Toxicity

9. The typical presentation of local anesthetic systemic toxicity (LAST): A. Can have a delayed onset of no more than 30 minutes B. Can have a delayed onset of up to 1 hour C. Manifests itself as an isolated cardiovascular (CV) dysfunction D. Manifests itself as an isolated central nervous system (CNS) dysfunction 10. What is true regarding the management of cardiovascular rescue during local anesthetic systemic toxicity (LAST)? A. Avoid epinephrine if possible; use vasopressin instead. B. Avoid lidocaine and amiodarone as antiarrhythmic drugs. C. Calcium channel blockers are the drug of choice to be used as antiarrhythmic drugs. D. Reduce the amount of epinephrine to < 1 g IV to avoid arrhythmogenic effects and avoid beta blockers to avoid hypotension.

ANSWERS AND EXPLANATIONS 1. B is correct. LAST typically starts with prodromal neurological symptoms. Patient conditions can rapidly deteriorate from that point on to seizures and coma. Accompanying cardiovascular symptoms can also progress in a fast manner and can eventually lead to arrhythmias, hypotension, and asystole. Airway management and circulatory support are therefore the cornerstones of LAST treatment. Starting intralipid infusion is important but it is meaningless if Airway, Breathing, and Circulatory support is not initiated as swiftly as possible. A is incorrect. Although propofol can be used in sedative or antiseizure doses, full induction doses of propofol are not recommended. Some people may be fooled into thinking propofol can be a substitute for lipid emulsion 20% and in the same way act as antiseizure medication. However, high doses of propofol have a severe negative inotrope effect. Benzodiazepines (midazolam) remain the best option to counter seizures without compromising cardiovascular stability. C is incorrect. Rapid degradation of cardiovascular stability and hypercapnia with lactate development will lead to acidosis, not alkalosis. Avoiding further acidification, hyperventilation, and supportive measures will help bridge the time gap between administrating the lipid emulsion and its effect. D is incorrect. Prompt administration of 20% lipid emulsion is one of the cornerstones in the treatment of LAST. A bolus of 1.5 mg/kg should be administered right after cardiopulmonary life support is initiated. A continuous infusion of 0.25 mL/kg/min should be prepared by helpers and started as quickly as possible. 2. C is correct. Malnutrition leads to several conditions that can greatly increase the risk for the occurrence of LAST. For one, malnutrition involves muscle wasting. Skeletal

Hadzic_Ch64_p371-376.indd 374

muscle acts as a depot for systemically absorbed LA, which acts as a buffer. Frail patients who are malnourished will also have a lower serum albumin level; again this will increase the chance of free LA in the bloodstream. Usually the diminutive patient will have other conditions such as liver or kidney failure/problems, which also are risk factors. A is incorrect. Strong muscular people may cause problems with depth of needling and difficult block procedures. But, muscle mass acts as a buffer for LA and is protective, certainly not a risk factor. B is incorrect. Having epilepsy does not influence the risk for LAST to occur. D is incorrect. The site of injection is a risk factor; however, it is not a patient-related one. Intercostal, paravertebral, and upper limb blocks increase the chance of LAST, but are patient-independent risk factors. 3. B is correct. Potential side effects of lipid emulsion therapy can include allergenic reactions, nausea/emesis, dyspnea, and chest pain. Nonetheless actual reported side effects are usually limited to bronchospasm, hyperamylasemia, and interference of laboratory tests. A is incorrect. There is insufficient lipid content in sedative or antiseizure doses, and even propofol induction doses, to worsen lipid accumulation. Propofol can therefore never be seen as an alternative to LRT. C is incorrect. Although transaminitis, hepatosplenomegaly, and bacterial contamination are in theory possible, they are only seen with prolonged use of lipid emulsion, never in the short-term acute administration in case of LAST treatment. D is incorrect. Although still not completely understood, it is now thought LRT has a dual effect. Previously only the lipid sink, or lipid shuttle effect of the emulsion, was thought to remove the LA from its negative action point and transferring it to sites of storage and detoxification. Now the multimodal theory teaches us that the lipid emulsion also improves cardiac output. Thus, the lipid emulsion works both through a scavenging effect (the aforementioned “lipid sink”) and a direct cardiotonic effect. 4. D is correct. The classical teaching that the vascular absorption of LAs is highest with intercostal blocks followed by epidural and brachial plexus injections corresponds to the clinical data demonstrating that the highest incidence of LAST occurs with paravertebral blocks, followed by upper extremity and trunk/lower blocks.1 A is incorrect. LAST has dramatically decreased over the years. Peripheral nerve block (PNB) LAST has decreased from 1.6–2/1000 in the 1990s to 0.08–0.98/1000 between 2003 and 2013, even though the total amount of PNBs being done thanks to ultrasound and teaching/fellowships is on the rise. Likewise, EA and LAST has gone down. This probably has to do with greater awareness of LAST, the use of test doses (in the case of EA), and the use of ultrasound, which has allowed providers to lower volumes in PNB as well as monitor the spread and possible intravascular injection.

22/04/19 4:29 PM

CHAPTER 64

B is incorrect. LAST incidence is higher in vessel-rich areas. However, the upper extremities are much denser in vessel-rich vascular areas as opposed to the lower extremities. C is incorrect. LAST incidence is higher with carnitine deficiency. Carnitine is an essential amino acid required for the transport of fatty acids into the mitochondria. Patients with a carnitine deficiency are at risk to have increased susceptibility to LA cardiotoxicity. 5. B is correct. None of the measures to prevent LAST is in itself precise or perfect. It is the combination of several steps to heighten security in which lies the key to success. Frequent aspiration is one of those elements. The use of ultrasound, injecting incremental doses, the use of test doses, and using the lowest effective dose all have a place in this algorithm toward safer practice. A is incorrect. Complete ASA monitoring is mandatory when performing regional anesthesia. Monitoring the patient closely will help detect the first signs of LAST; however, this does not help PREVENT the occurrence of LAST. C is incorrect. The rise of the ultrasound (US)-guided regional nerve blocks has indeed been a key element in patient safety. The ultrasound’s main advantage is the ability to see actual spread and to decrease the amount of LA injected when spread is sufficient. Incremental dosing was already incorporated in safe practice before US use became widely popular. D is incorrect. Epinephrine can be used as an intravascular marker. It is widely used for testing epidural-placed catheters, less frequently for PNB. However it requires 10–15 µg to have a reasonable sensitivity and positive predictive value. 6. B is correct. An awake fiberoptic intubation is always stressful, for both the patient and even the most experienced anesthesiologist. A generous amount of LAs will usually be given to make sure the procedure is as painless and smooth as possible for the patient. Couple this with the nasal, oropharyngeal, and tracheal mucosa that is extremely vessel-rich and one can imagine the uptake of LAs can be very high. A is incorrect. LAST is indeed rare for an inferior alveolar block, although the pterygomandibular space is extremely vessel-rich. Even experienced oral surgeons have a high incidence of intravascular needle placement in that exact anatomical location. However, most inferior alveolar blocks require only low to modest doses of LAs to work; hence, the low incidence of LAST. C is incorrect. The subarachnoid spread is responsible for this rare complication. D is incorrect. The incidence of LAST reported after bilateral TAP for cesarean delivery is high, but it is likely caused because of the high dosage needed to make this volume block work and the repeat doses the patients may already have had during labor from epidural or even top ups and spinals. Several other factors can increase LAST chances in pregnancy. Hormonal

Hadzic_Ch64_p371-376.indd 375

Local Anesthetic Systemic Toxicity

375

changes may increase sensitivity of neural tissue and cardiac toxicity. Finally, reduced levels of albumin may increase the free fraction of some LAs in pregnant patients. 7. C is correct. LAs will reduce the sodium ion flux through voltage-gated sodium channels by a combination of an increased energy barrier and steric hindrance. When the nonspecific sodium channels in the heart are blocked by high concentrations of LAs, electrophysiological and contractile dysfunction of the heart will follow. A is incorrect. Even at lower than expected LA serum concentrations, toxicity can occur, but because of the high 6:1 ratio accumulation in mitochondria and cardiac tissue. B is incorrect. LA toxicity is based on the sodium channel block abilities. LAs also inhibit calcium channels and carnitine-acylcarnitine translocase. They do not facilitate them. D is incorrect. LA cardiac toxicity is higher with bupivacaine, but because of its high lipophilicity and hence very strong affinity for the voltage-gated sodium channel. 8. D is correct. The pediatric population frequently has lower muscle mass, leaving them more vulnerable to LAST. Skeletal muscle acts as a depot and as such also as a buffer.2 A is incorrect. Sedation can indeed mask the first prodromal phase of LAST. However, this does not mean that chances of having LAST are increased due to sedation (or even general anesthesia; see reference on safety with blocks under GA in pediatric population), just that it may be missed or lately diagnosed. B is incorrect. Children are not more vessel-rich. No evidence supports the fact that there are more accidental intravascular punctures in children. C is incorrect. In normal pediatric populations, the serum levels of albumin are not lower in comparison to adults. 9. B is correct. Contrary to most beliefs, the onset time of LAST is actually more than 30 minutes and can have a delay of up to 1 hour. Monitoring patients after a block should therefore be at least 1 hour and not the usually cited 30 minutes. A is incorrect. Most standard practices use the 30-minute rule for monitoring after a regional block has been performed. This may be inadequate. C is incorrect. Typical presentation is prodromes of CNS dysfunction followed in fast succession with CV dysfunction. Isolated CV dysfunction is of course a possible variant; however, it is not the “typical” onset of LAST. D is incorrect. Typical presentation is prodromes of CNS dysfunction followed in fast succession with CV dysfunction. Isolated CNS dysfunction is of course a possible variant, where only seizures, convulsions, and maybe coma is seen.

22/04/19 4:29 PM

376

PART 10

LAST: Local Anesthetic Systemic Toxicity

10. D is correct. It is important to note that in our model, the higher doses of epinephrine (25 mcg/kg) are actually much lower than what is traditionally considered high-dose epinephrine treatment (0.1 mg/kg or 100 mcg/ kg) as defined by the 1992 American Heart Association guidelines for cardiopulmonary resuscitation and found in several studies to impair outcomes in clinical resuscitation. Beta blockers cause hypotension and counteract the working of epinephrine.3 A is incorrect. Vasopressin should be avoided. It leads to worse outcomes, and the hemodynamic effects are also much worse in comparison with low-dose epinephrine use.4 B is incorrect. Lidocaine should always be avoided! So should other LAs or antiarrhythmic drugs from class 1 antiepileptic drugs. C is incorrect. Calcium channel blockers can cause serious hypotension and exacerbate the already precarious situation. They are absolutely to be avoided.

2. Ivani G, Suresh S, Ecoffey C, et al. The European Society of Regional Anaesthesia and Pain Therapy and the American Society of Regional Anesthesia and Pain Medicine Joint Committee Practice Advisory on Controversial Topics in Pediatric Regional Anesthesia. Reg Anesth Pain Med. 2015 Sep-Oct;40(5):526-532. 3. Luo M, Yun X, Chen C, et al. Giving priority to lipid administration can reduce lung injury caused by epinephrine in bupivacaine-induced cardiac depression. Reg Anesth Pain Med. 2016;41(4):469. 4. Di Gregorio G, Schwartz D, Ripper R, et al. Lipid emulsion is superior to vasopressin in a rodent model of resuscitation from toxin-induced cardiac arrest. Crit Care Med. 2009;37(3):993.

Suggested Reading Hadzic A. Local anesthetic systemic toxicity. In: Gitman M, Fettiplace M, Weinberg G, eds. Textbook of Regional Anesthesia and Acute Pain Management. 2nd ed. New York, NY: McGraw-Hill Publishing; 2017:chap 65.

References 1. Barrington MJ, Kluger R. Ultrasound guidance reduces the risk of local anesthetic systemic toxicity following peripheral nerve blockade. Reg Anesth Pain Med. 2013;38:289-299.

Hadzic_Ch64_p371-376.indd 376

22/04/19 4:29 PM

PART 11 Perioperative Outcome and Economics of Regional Anesthesia Chapter 65

Regional Anesthesia, Cost, Operating Room, and Personnel Management  379

Chapter 66

Regional Anesthesia and Perioperative Outcome  381

Chapter 67

The Effects of Regional Anesthesia on Functional Outcome After Surgery  383

Hadzic_Ch65_p377-380.indd 377

19/04/19 11:18 AM

This page intentionally left blank

Hadzic_FM_p00i-pxx.indd 6

22/04/19 6:40 PM

65 Regional Anesthesia, Cost, Operating Room, and Personnel Management Franklin Chiao

QUESTIONS DIRECTIONS: Choose the one best response to each question. 1.  Which situation results in the shortest presurgical operating room time? A. Peripheral nerve block placed in the operating room B. Peripheral nerve block placed in a block room C. General anesthesia only D. Peripheral nerve block in combination with general anesthesia 2.  Anesthesia-controlled time is shortest in which case? A. General anesthesia alone B. Peripheral nerve block placed in the preoperative holding area C. Combination of general anesthesia and peripheral nerve block D. No significant differences in the groups 3.  Anesthesia induction time is shortest for which group? A. General anesthesia  B. Neuraxial block C. Peripheral nerve block  D. General anesthesia combined with neuraxial block 4.  Turnover time is longest in which group? A. Peripheral nerve block  B. General anesthesia  C. No significant differences in the groups D. Combination of general and regional anesthesia 5.  Which is the greatest cost in the post-anesthesia care unit (PACU)? A. Personnel costs B. Drugs C. Materials D. Monitors

6.  How does regional anesthesia most impact PACU costs? A. It impacts nausea and vomiting. B. It decreases drug usage in the PACU. C. It has a better recovery profile. D. It allows for bypass of phase I of PACU. 7.  Which of the following is true regarding anesthesiarelated costs? A. General anesthesia is significantly more costly compared to peripheral nerve blocks. B. Epidurals are significantly less costly compared to general anesthesia. C. Spinal is significantly less costly compared to general anesthesia. D. Neuraxial is significantly less costly than peripheral nerve blocks. 8.  Which is true regarding PACU bypass and regional anesthesia? A. Research shows it can occur in over 80% of regional anesthesia cases. B. The bypass rate is the same when compared to general anesthesia. C. There are fewer unexpected admissions in the general anesthesia PACU bypass group. D. Overall PACU time was similar in the bypass group.

ANSWERS AND EXPLANATIONS 1. B is correct. Blocks performed in the preoperative area reduce time in the operating room before surgery starts. It is even shorter than inducing general anesthesia.1,2 2. B is correct. Peripheral nerve blocks placed in the preoperative area reduce anesthesia-controlled time.2,3 3. A is correct. General anesthesia induction is quicker than placing a peripheral nerve block in the operating room.4

379

Hadzic_Ch65_p377-380.indd 379

19/04/19 11:18 AM

380

PART 11

Perioperative Outcome and Economics of Regional Anesthesia

4. C is correct. Turnover time for all combinations of cases was the same and appears standardized for all cases.3,5,6 5. A is correct. Staffing costs are the largest expense in the PACU.7,8 6. D is correct. Passing phase I is an advantage in terms of cost and with regional anesthesia this is more likely.7,9 7. C is correct. Additional cost savings accrue from performing cases with spinal anesthesia compared to general anesthesia.5 8. A is correct. Bypassing phase I of the PACU can occur at a very high rate when regional anesthesia is used. This is a major benefit and economic advantage.7,10

References 1. Armstrong KP, Cherry RA. Brachial plexus anesthesia compared to general anesthesia when a block room is available. Can J Anaesth. 2004;51(1):41-44. 2. Mariano ER, Chu LF, Peinado CR, Mazzei, WJ. Anesthesiacontrolled time and turnover time for ambulatory upper extremity surgery performed with regional versus general anesthesia. J Clin Anesth. 2009;21(4):253-257. 3. Williams BA, Kentor ML, Williams JP, et al. Process analysis in outpatient knee surgery: effects of regional and general anesthesia on anesthesia-controlled time. Anesthesiology. 2000;93(2):529-538. 4. Liu SS, Strodtbeck WM, Richman JM, Wu CL. A comparison of regional versus general anesthesia for ambulatory anesthesia: a meta-analysis of randomized controlled trials. Anesth Analg. 2005;101(6):1634-1642.

Hadzic_Ch65_p377-380.indd 380

5. Schuster M, Gottschalk A, Berger J, Standl T. A retrospective comparison of costs for regional and general anesthesia techniques. Anesth Analg. 2005;100(3):786-794. 6. Mazzei WJ. Operating room start times and turnover times in a university hospital. J Clin Anesth. 1994;6(5):405-408. 7. Dexter F, Macario A, Manberg PJ, Lubarsky DA. Computer simulation to determine how rapid anesthetic recovery protocols to decrease the time for emergence or increase the phase I postanesthesia care unit bypass rate affect staffing of an ambulatory surgery center. Anesth Analg. 1999;88(5):1053-1063. 8. Swenson JD, Davis JJ. Getting the best value for consumable supplies in regional anesthesia. Int Anesthesiol Clin. 2011;49(3):94-103. 9. Williams BA, Kentor ML, Vogt MT, et al. Economics of nerve block pain management after anterior cruciate ligament reconstruction: potential hospital cost savings via associated postanesthesia care unit bypass and same-day discharge. Anesthesiology. 2004;100(3):697-706. 10. Williams BA, DeRiso BM, Figallo CM, et al. Benchmarking the perioperative process: III. Effects of regional anesthesia clinical pathway techniques on process efficiency and recovery profiles in ambulatory orthopedic surgery. J Clin Anesth. 1998;10(7):570-578.

Suggested Reading Hadzic A. Regional anesthesia, cost, operating room, and personnel management. In: Laur J, Dexter F, eds. Textbook of Regional Anesthesia and Acute Pain Management. 2nd ed. New York, NY: McGraw-Hill Publishing; 2017:chap 67.

19/04/19 11:18 AM

66 Regional Anesthesia and Perioperative Outcome Franklin Chiao

QUESTIONS DIRECTIONS: Choose the one best response to each question. 1. One outcome advantage of regional anesthesia over general anesthesia involves: A. Higher billing compliance B. 10 times lower rate of prostate cancer C. Elimination of anesthesia-induced metastatic brain cancer D. Reducing the hypothalamic-pituitary-adrenal axis activation and subsequent immune suppression 2. Pain itself has what impact on the immune system? A. Counteracts opioid-induced immune suppression B. Suppresses natural killer (NK) cell function C. Reduces immune-related cancer metastasis D. Possesses cytotoxic properties 3. Epidural for thoracotomy surgery has what affect post-procedure? A. Significantly lower rates of post-thoracotomy pain at 6 months post-procedure B. Slightly lower rates of post-thoracotomy pain at 6 months post-procedure C. Thoracotomy pain is not affected after 1 month post-procedure. D. Thoracotomy pain decreases initially in the first week and then remains the same. 4. Some economic benefits of regional anesthesia are: A. Larger bundled payments B. Higher billing compliance C. Both shorter length of hospital stay and lower resource utilization D. Quicker time to rehabilitation facilities and shorter turnover time 5. Major perioperative outcome improvements related to regional anesthesia include: A. Decreased rate of mechanical ventilation and blood transfusion B. 10 times lower rate of deep vein thrombosis

C. Shorter stay in rehabilitation facilities D. Decreased number of aneurysm ruptures 6. Which of the following statements is true regarding short-term benefits from neuraxial versus general anesthesia for noncardiac surgery? A. Overall mortality was reduced and some studies note significant differences were found when analyzing data by surgery type. B. Deep vein thrombosis rates were lower and pneumonia rates were the same. C. Pulmonary embolism rates were lower and renal failure rates were the same. D. Myocardial infarction rates were lower and mortality did not vary significantly based on surgery type. 7. For total knee arthroplasty, an analysis of the American College of Surgeons National Surgical Quality Improvement Program (ACS NSQIP) database from 2005 to 2010 revealed that in spinal versus general anesthesia, there was: A. Lower rate of pneumonia and less use of nasal cannula B. Shorter surgery duration, and lower overall complications C. Similar myocardial infarction rates and lower ASA status D. Quicker room turnover rates, and nonsignificantly shorter duration of hospital stay 8. For intrathoracic surgery, regional anesthesia results in: A. Lower rate of arrhythmias and similar risk of myocardial infarction B. Paravertebral block and epidural with the same rates of pneumothorax C. Faster rehabilitation and pain scores lower by 10% D. Similar length of stay and stroke risk 9. Epidural use for major abdominal surgery: A. Reduces postoperative ileus but not nausea and emesis B. Improves functional residual capacity after surgery and does not affect wound infection rates C. Reduces nausea and infectious complications D. Lowers risk of pulmonary embolism by 75% 381

Hadzic_Ch66_p381-382.indd 381

19/04/19 11:19 AM

382

PART 11

Perioperative Outcome and Economics of Regional Anesthesia

10. Epidural placement for labor when compared to systemic opioid administration resulted in: A. An increased, but not statistically significant, risk for instrumental vaginal delivery B. A higher risk of cesarean delivery C. The same length of labor D. A higher rate of learning disabilities

2. Shavit Y, Martin FC, Yirmiya R, et al. Effects of a single administration of morphine or footshock stress on natural killer cell cytotoxicity. Brain Behav Immun. 1987;1(4):318-328. 3. Andreae MH, Andreae DA. Local anaesthetics and regional anaesthesia for preventing chronic pain after surgery. Cochrane Database Syst Rev. 2012;10:CD007105. 4. Lenart MJ, Wong K, Gupta RK, et al. The impact of peripheral nerve techniques on hospital stay following major orthopedic surgery. Pain Med. 2012;13(6):828-834.

ANSWERS AND EXPLANATIONS

5. Memtsoudis SG, Sun X, Chiu Y-L, et al. Perioperative comparative effectiveness of anesthetic technique in orthopedic patients. Anesthesiology. 2013;118(5):1046-1058.

1. D is correct. General anesthesia is worse from an oncologic standpoint when compared to sedation or regional anesthesia.1

6. Rodgers A, Walker N, Schug S, et al. Reduction of postoperative mortality and morbidity with epidural or spinal anaesthesia: results from overview of randomised trials. BMJ. 2000;321(7275):1493.

2. B is correct. Pain suppresses NK cell function.2

7. Wijeysundera DN, Beattie WS, Austin PC, Hux JE, Laupacis A. Epidural anaesthesia and survival after intermediate-to-high risk non-cardiac surgery: a population based cohort study. Lancet. 2008;372(9638):562-569.

3. A is correct. Epidural has a both a short-term and long-lasting advantage when used for thoracotomy.3 4. C is correct. Hospitals and patients benefit from regional anesthesia as length of stay and resource utilization goes down.4 5. A is correct. With regional anesthesia, decreased blood loss from a sympatholytic effect occurs, and there is better pulmonary function, which leads to lower rates of mechanical ventilation.5 6. A is correct. Mortality was improved with neuraxial anesthesia; this could be due to many benefits including decreased blood loss.6,7 7. B is correct. Spinal anesthesia for total knee arthroplasty was more time efficient and improved outcomes compared to general anesthesia. It has become a standard of care.8 8. C is correct. Regional anesthesia benefits for thoracic surgery extend into areas of physical rehabilitation and have mild reductions in pain scores.9,10 9. A is correct. Epidurals reduce postoperative ileus through a sympatholytic effect on the bowels. Nausea and emesis were not impacted significantly. Epidurals can cause hypotension in general, which may have weakened opioid sparing antiemetic effects.6,11–13 10. A is correct. This was likely due to some minor motor weakness with an epidural.14

References 1. Kurosawa S, Kato M. Anesthetics, immune cells, and immune responses. J Anesth. 2008;22(3):263-277.

Hadzic_Ch66_p381-382.indd 382

8. Pugely AJ, Martin CT, Gao Y, Mendoza-Lattes S, Callaghan JJ. Differences in short-term complications between spinal and general anesthesia for primary total knee arthroplasty. J Bone Joint Surg Am. 2013;95(3):193-199. 9. Ballantyne JC, Carr DB, deFerranti S, et al. The comparative effects of postoperative analgesic therapies on pulmonary outcome: cumulative meta-analyses of randomized, controlled trials. Anesth Analg. 1998;86(3):598-612. 10. Davies RG, Myles PS, Graham JM. A comparison of the analgesic efficacy and side-effects of paravertebral vs epidural blockade for thoracotomy—a systematic review and meta-analysis of randomized trials. Br J Anaesth. 2006;96(4):418-426. 11. Wahba WM, Don HF, Craig DB. Post-operative epidural analgesia: effects on lung volumes. Can Anaesth Soc J. 1975; 22(4):519-527. 12. Kabon B, Fleischmann E, Treschan T, Taguchi A, Kapral S, Kurz A. Thoracic epidural anesthesia increases tissue oxygenation during major abdominal surgery. Anesth Analg. 2003;97(6):1812-1817. 13. Jorgensen H, Wetterslev J, Moiniche S, Dahl JB. Epidural local anaesthetics versus opioid-based analgesic regimens on postoperative gastrointestinal paralysis, PONV and pain after abdominal surgery. Cochrane Database Syst Rev. 2000;(4):CD001893. 14. Liu EHC, Sia ATH. Rates of caesarean section and instrumental vaginal delivery in nulliparous women after low concentration epidural infusions or opioid analgesia: systematic review. BMJ. 2004;328(7453):1410.

Suggested Reading Hadzic A. Regional anesthesia and perioperative outcome. In: Stundner O, Suzuki S, Memtsoudis SG, eds. Textbook of Regional Anesthesia and Acute Pain Management. 2nd ed. New York, NY: McGraw-Hill Publishing; 2017:chap 68.

19/04/19 11:19 AM

67 The Effects of Regional Anesthesia on Functional Outcome After Surgery Franklin Chiao

QUESTIONS DIRECTIONS: Choose the one best response to each question. 1.  Which of the following is true of chronic pain after surgery? A. After thoracotomy or limb amputation, 40% of patients suffer from persistent pain. B. After cesarean delivery, 25% of patients develop persistent pain. C. Risk of chronic pain after inguinal hernia surgery is about 33%. D. Chronic pain is defined as pain beyond 8 months. 2.  Which of the following is true regarding regional anesthesia after major joint replacement surgery? A. Short-term gains in function are usually sustained in the long run. B. Early mobilization is helpful but not a central part of long-term joint function. C. Gains in passive joint range of motion due to regional anesthesia last for several months after surgery. D. Anesthesia that lasts more than 24 hours worsens outcome due to stretch injury. 3.  Regional anesthesia may impact postoperative cognitive dysfunction (POCD). Which of the following statements is true about POCD? A. It independently predicts adverse outcomes but not mortality increases. B. It independently predicts long-term cognitive impairment. C. It is associated with similar health care costs compared to patients without it. D. Morbidity increases but mortality does not.

4.  Research papers comparing regional versus general anesthesia for postoperative cognitive dysfunction (POCD) have limitations and show what findings? A. Long-term cognitive function was not significantly different. B. Midazolam does not have affects on POCD. C. Short-term cognitive impairment was similar in both groups. D. Regional benefits on POCD depend the most on gender. 5.  Which of the following is false regarding regional anesthesia for thoracic and cardiac surgeries? A. After intrathoracic procedures, pulmonary mechanics were reported to be better in patients receiving regional anesthesia. B. In cardiac surgery, the addition of thoracic epidural anesthesia to general anesthesia was associated with improved coronary perfusion and myocardial oxygen supply. C. Controversy for the use of epidural anesthesia for cardiac surgery is related with the increased risk for epidural hematoma in this population. D. Paravertebral blocks were reported to be more effective than epidural anesthesia for intrathoracic surgery.

ANSWERS AND EXPLANATIONS 1. A is correct. Post-thoracotomy pain and phantom limb pain still occur frequently despite multimodal analgesic techniques.1,2 2. C is correct. Regional anesthesia for joint replacement allows for better joint flexibility. It could be due to improved rehabilitation, lower pain scores, or better operating conditions.3,4

383

Hadzic_Ch67_p383-384.indd 383

19/04/19 11:20 AM

384

PART 11

Perioperative Outcome and Economics of Regional Anesthesia

3. B is correct. POCD is very debilitating and is a risk factor for long-term cognitive deterioration.5 4. A is correct. Evidence is weak when examining regional anesthesia and POCD. Current evidence does not find a long-term cognitive difference for POCD affects for patients undergoing general or regional anesthesia.6 5. D is false, so option D is correct. Paravertebral block is not more effective than epidural analgesia. A, B, and C are incorrect. All of these are true statements regarding regional anesthesia for thoracic and cardiac surgeries.

References 1. Sng BL, Sia AT, Quek K, Woo D, Lim Y. Incidence and risk factors for chronic pain after caesarean section under spinal anaesthesia. Anaesth Intensive Care. 2009;37(5):748-752. 2. Jung BF, Ahrendt GM, Oaklander AL, Dworkin RH. Neuropathic pain following breast cancer surgery: proposed classification and research update. Pain. 2003;104(1-2):1-13.

Hadzic_Ch67_p383-384.indd 384

3. Singelyn FJ, Deyaert M, Joris D, Pendeville E, Gouverneur JM. Effects of intravenous patient-controlled analgesia with morphine, continuous epidural analgesia, and continuous three-in-one block on postoperative pain and knee rehabilitation after unilateral total knee arthroplasty. Anesth Analg. 1998;87(1):88-92. 4. Capdevila X, Barthelet Y, Biboulet P, Ryckwaert Y, Rubenovitch J, d’Athis F. Effects of perioperative analgesic technique on the surgical outcome and duration of rehabilitation after major knee surgery. Anesthesiology. 1999;91(1):8-15. 5. Marcantonio ER, Flacker JM, Michaels M, Resnick NM. Delirium is independently associated with poor functional recovery after hip fracture. J Am Geriatr Soc. 2000;48(6):618-624. 6. Guay J: General anaesthesia does not contribute to long-term postoperative cognitive dysfunction in adults: a meta-analysis. Indian J Anaesth. 2011;55(4):358-363.

Suggested Reading Hadzic A. The effects of regional anesthesia on functional outcome after surgery. In: Atchabahian A, Andreae MH, eds. Textbook of Regional Anesthesia and Acute Pain Management. 2nd ed. New York, NY: McGraw-Hill Publishing; 2017:chap 69.

19/04/19 11:20 AM

PART 12 Acute Pain Management

Chapter 68

Intravenous Patient-Controlled Analgesia  387

Chapter 69

Continuous Peripheral Nerve Blocks  389

Chapter 70

 rganization of an Acute Pain Management Service Incorporating Regional O Anesthesia Techniques  391

Chapter 71

 ultimodal Analgesia: Pharmacologic Interventions and Prevention of M Persistent Postoperative Pain  393

Chapter 72

The Role of Nonopioid Analgesic Infusions in the Management of Postoperative Pain  397

Hadzic_Ch68_p385-388.indd 385

19/04/19 3:11 PM

This page intentionally left blank

Hadzic_FM_p00i-pxx.indd 6

22/04/19 6:40 PM

68 Intravenous Patient-Controlled Analgesia Astrid Van Lantschoot and Thibaut Vanneste

QUESTIONS DIRECTIONS: Choose the one best response to each question. 1.  Which statement is correct regarding the site of action of opioids? A. The mechanism of action for opioids is only via the mu-receptors of the central nervous system. B. Pure agonists are the most useful for acute pain. C. Agonist-antagonist opioids act as a kappa antagonist and a mu-receptor agonist. D. Partial agonists have a major role for analgesia. 2.  Which statement is correct regarding patient-controlled analgesia (PCA) for acute post-operative management? A. Morphine is the best analgesic for PCA. B. In at least 0.5% of the cases, PCA gives respiratory depression. C. The fentanyl patch has a comparable effect to PCA, and therefore is a good choice in acute pain. D. PCA is cost-effective through savings on nursing costs. 3.  Which statement is true regarding improving patient-controlled analgesia (PCA) effect? A. Nonsteroidal anti-inflammatory drugs (NSAIDs), cyclo-oxygenase (COX)-2 inhibitors, and acetaminophen all have an opioid-sparing effect. B. NSAIDs, COX-2 inhibitors, and acetaminophen reduce opioid-induced side effects. C. Ketamine infusions should be avoided perioperatively because of the increased risk of hallucinations. D. Association of lidocaine has an opioid-sparing effect. 4.  Which statement is correct regarding opioid monitoring? A. Respiratory frequency is the most sensitive parameter to assess respiratory depression. B. End-tidal carbon dioxide (EtCO2) is the most sensitive parameter to assess respiratory depression.

C. Background infusion has no significant effect on respiratory depression. D. Nursing and follow-up of the patient and the patient-controlled analgesia (PCA) can be regulated “on indication.” 5.  Which statement is correct regarding fentanyl use in patient-controlled analgesia (PCA)? A. Fentanyl is very lipophilic and thus has a slow onset time when used in PCA. B. Fentanyl causes less respiratory depression as compared to morphine and hydromorphone. C. Fentanyl is a very good choice for patients with renal insufficiency. D. Fentanyl does not have a significant rise in contextsensitive half-time. 6.  Which of the following statements is true? A. Hydromorphone is 5–6 times as potent as morphine and produces more active metabolites as compared to morphine. B. Morphine undergoes hepatic glucuronidation and is excreted through the kidneys. C. Tramadol can cause seizures because of its mu activity. D. Methadone has a short half-life. 7.  Which of the following statements is true? A. In an opioid-tolerant patient, multimodal analgesia should be considered only when basic analgesia regimen (IV PCA) fails. B. In obese patients, the dosing regimen of the PCA needs to be adjusted for the corrected body weight. C. The elderly have an increased risk of delirium and postoperative cognitive dysfunction, which can be aggravated by poor pain control. D. IV PCA should not be used in labor analgesia because it causes significantly lower Apgar scores.

387

Hadzic_Ch68_p385-388.indd 387

19/04/19 3:12 PM

388

PART 12

Acute Pain Management

ANSWERS AND EXPLANATIONS 1. B is correct. Pure agonists are the most useful for acute pain because of their complete mu binding. A is incorrect. They also act via receptors in the peripheral nervous system. C is incorrect. Agonist-antagonist opioids act as a kappa agonist and mu-receptor antagonist. D is incorrect. Because they only have partial mu-receptor binding, pure agonists have a significant ceiling effect. 2. D is correct. It does reduce demand on the nursing staff, who are required for delivery of IV or IM PRN analgesics. This reduced workload of the nursing staff may indeed have a cost benefit. There is no apparent decrease in cost. A is incorrect. There is no standard opioid. Patient characteristics will influence choice of opioid. B is incorrect. All side effects together are less than 0.5%. C is incorrect. Patches have a good indication for chronic cancer pain but none in acute pain. 3. A is correct. Several meta analyses suggest that the use of nonsteroidal anti-inflammatory drugs (NSAIDs), cyclo-oxygenase (COX)-2 inhibitors, and acetaminophen all have an opioid-sparing effect. B is incorrect. NSAIDs, COX-2 inhibitors, and acetaminophen do not have an influence on opioid side effects. C is incorrect. Ketamine infusions do not produce a significant increase in hallucinations. D is incorrect. Association of lidocaine does not change opioid dose, pain level, or side effects. 4. B is correct. End-tidal carbon dioxide (EtCO2) is the most sensitive parameter to assess respiratory depression. A is incorrect. Respiratory frequency alone is not a sensitive enough parameter. C is incorrect. Accumulation augments the risk of respiratory arrest. D is incorrect. Tight follow-up schedules should be foreseen.

Hadzic_Ch68_p385-388.indd 388

5. C is correct. Fentanyl is a very good choice for patients with renal insufficiency because of its hepatic metabolization. A is incorrect. Fentanyl has a fast onset due to its lipophilic nature. B is incorrect. Fentanyl causes the same rate of respiratory depression as compared to morphine and hydromorphone. D is incorrect. Fentanyl has a significant rise because of its long elimination half-life and high distribution volume. 6. B is correct. Morphine undergoes hepatic glucuronidation and is excreted through the kidneys. A is incorrect. Hydromorphone produces morphine3-glucuronide (M3G) and significantly less morphine6-glucuronide (M6G). C is incorrect. Tramadol can cause epilepsy by inhibiting reuptake of serotonin and norepinephrine. D is incorrect. Methadone has a long and unpredictable elimination half-life. 7. C is correct. The elderly have an increased risk of delirium and postoperative cognitive dysfunction, which can be aggravated by poor pain control. A is incorrect. From the outset, multimodal analgesia and an analgesia plan should be considered. Opioid-tolerant patient often are a challenge. B is incorrect. Dosing should be the same, regardless of body weight because obese patients have an increased risk for apnea and opioid-related side effects. D is incorrect. There is no evidence that Apgar scores are significantly different with IV PCA vs. patient-controlled epidural analgesia (PCEA).

Suggested Reading Hadzic A. Intravenous patient-controlled analgesia. In: Hanna MN, Ahmed O, Hall S, eds. Textbook of Regional Anesthesia and Acute Pain Management. 2nd ed. New York, NY: McGraw-Hill Publishing; 2017:chap 70.

19/04/19 3:12 PM

69 Continuous Peripheral Nerve Blocks Admir Hadzic, Alexander Vloka, Angela Lucia Balocco, and Samantha Kransingh

QUESTIONS DIRECTIONS: Choose the one best response to each question. 1.  Regarding continuous nerve blocks, all of the following statements are true except: A. The best infusion rate is 5 mL/hr. B. Typically, a lower concentration of local anesthetics is used (eg, 0.125% bupivacaine). C. Femoral catheters in mobile patients can become displaced from the therapeutic position after 6 hours in 25% of cases. D. Most common systems are catheter-through-needle designs. E. Their clinical efficacy is well-documented in clinical trials. 2.  Well-documented indications for perineural infusions of local anesthetics through catheters include: A. Sympathectomy to improve circulation B. Treatment of Raynaud’s phenomenon C. Cancer pain D. Complex regional pain syndrome E. All of the above 3.  Of the options below, which is not a well-documented benefit of perineural infusions of local anesthetics through catheters? A. Control of postoperative pain B. Decreased use of opioids when treating acute postoperative pain C. Advancement of physical therapy goals D. Complete post operative analgesia for knee arthroplasty, via a femoral catheter. E. Earlier readiness and discharge from the hospital 4.  Risks of perineural infusion of local anesthetics through catheters include: A. Inadvertent intravascular placement B. Inadvertent intrathecal placement C. Neurologic complications D. Infection E. All of the above

5.  Regarding perineural infusion of local anesthetics through catheters, which of the following is false? A. Decreased pain and opioid consumption is the most consistent benefit of perineural catheters. B. Infraclavicular catheters require a relatively large volume of local anesthetic to be successful. C. Because multiple nerves innervate knee or hip joints, additional analgesics are typically required, even with fully functional perineural catheters. D. Interscalene catheters may result in side effects similar to those of a single-injection block. E. Boluses of local anesthetics are not required. 6.  Which of the following is an absolute contraindication to continuous peripheral nerve blocks? A. Patient refusal/inability to cooperate B. Systemic infection C. Preexisting neuropathy D. Coagulopathy

ANSWERS AND EXPLANATIONS 1. A is not true, so option A is correct. The best infusion rate is as yet undetermined as there are varying influences that affect infusion rate, eg, anatomic site. B, C, D and E are incorrect. All of these are true statements regarding continuous peripheral nerve blocks. 2.  A, B, C, and D are all indications for perineural infusions, so option E is correct. The reported indications for perineural infusions of local anesthetics include sympathectomy/vasodilation after vascular accidents or embolism, digit reimplantation, limb salvage, and treatment of Raynaud phenomenon. Continuous infusions have also been described for painful chronic conditions such as phantom limb pain, complex regional pain syndrome, cancer pain, and preoperative pain control. 3. D is correct. This is not a well-documented benefit of perineural infusions because a femoral catheter does not provide complete postoperative analgesia for knee arthroplasty. Femoral catheters have been studied 389

Hadzic_Ch69_p389-390.indd 389

22/04/19 4:20 PM

390

PART 12

Acute Pain Management

extensively in the setting of knee arthroplasty, and continuous femoral infusions have been validated with multiple documented in-hospital benefits including improved analgesia and earlier attainment of range-ofmotion. However, multiple nerves innervate surgical sites such as the knee or hip. Thus, even with a functional continuous peripheral nerve block, additional analgesics are typically required. A, B, C and E are incorrect. All of these are welldocumented benefits of perineural infusions of local anesthetics through catheters. 4.  A, B, C, and D are all risks, so option E is correct. Perineural catheters may be unintentionally inserted into the intravascular, epidural, intrathecal, or intraneural spaces. Infectious risks specific to perineural catheters have been investigated in multiple studies. Neurologic complications are a rare but serious complication of any regional technique. It is difficult to attribute the contribution of the surgical procedure, patient positioning, the single-injection block or catheter/infusion to the neurological complication. 5. E is false, so option E is correct. Evidence suggests that providing a basal infusion rate with the addition of patient-controlled boluses optimizes analgesia, and minimizes breakthrough pain and supplemental analgesic requirements. Adding patient-controlled boluses usually decreases the required basal infusion rate, the incidence of

Hadzic_Ch69_p389-390.indd 390

an insensate extremity, and local anesthetic consumption, the last allowing for longer infusion duration in the ambulatory setting. A, B, C and D are incorrect. All of these are true statements regarding perineural infusion of local anesthetics through catheters. 6. A is correct. Commonly described absolute contraindications include infection at the catheter-insertion site, allergy to local anesthetics, and patient refusal/inability to cooperate. B, C, and D are incorrect. Relative contraindications have been described, including coagulopathy/anticoagulation, systemic infection/bacteremia, preexisting neuropathy, desire for postoperative neurovascular examinations, and elevated fall risk/inability to comply with activity restrictions.

Suggested Readings Hadzic A. Continuous peripheral nerve blocks. In: Monahan AM, Ilfeld BM, eds. Textbook of Regional Anesthesia and Acute Pain Management. 2nd ed. New York, NY: McGraw-Hill Publishing; 2017:chap 71. Marhofer D, Marhofer P, Triffterer L, et al. Dislocation of perineural catheters: a volunteer study. Br J Anaesth. 2013;111(5):800-8006. Monahan AM, Ilfeld BM. Continuous peripheral nerve blocks: local anesthetic solutions and infusion strategies. In: Hadzic A. Textbook of Regional Anesthesia and Acute Pain Management. 2nd ed. New York, NY: McGraw-Hill Publishing; 2017.

22/04/19 4:20 PM

70 Organization of an Acute Pain Management Service Incorporating Regional Anesthesia Techniques Astrid Van Lantschoot and Thibaut Vanneste

QUESTIONS DIRECTIONS: Choose the one best response to each question. 1.  Which statement is true regarding intravenous patientcontrolled analgesia (IV-PCA)? A. Without front loading, it takes at least three elimination half-lives before a minimum effective analgesic concentration (MEAC) is reached. B. Addition of a basal infusion in IV PCA improves analgesia significantly. C. When a patient is somnolent, nursing staff or family caregivers can make use of the IV PCA. D. When converting opioids, one should always use the exact same dose. 2.  Which statement is correct regarding peripheral nerve blocks and catheters? A. All patients can be sent home in a day-care setting with a continuous peripheral nerve block catheter. B. The peripheral infusion catheter can be removed only in a hospital environment. C. Patients should be warned about local anesthetic systemic toxicity (LAST) and trained to recognize the symptoms (eg, dizziness, metal taste, tinnitus). D. In case of catheter migration, the catheter can be left in place until the next planned visit. 3.  Which statement is correct? A. Chronic pain is a result of poorly managed acute pain. B. ASA guidelines recommend at least two nonopioid analgesics as cornerstone for analgesia before adding an opioid. C. Neuropathic pain can be easily managed with gabapentinoids. D. Large doses of ketamine are necessary for a significant potentiation of opioids.

4.  Which statement is correct regarding continuous epidural analgesia? A. Coagulation should only be checked before placement of an epidural catheter. B. The level should be chosen at the highest dermatome point of the surgical trauma. C. The epidural catheter should advance the epidural space between 3 and 5 cm. D. An epidural infusion should only contain local anesthetic and/or opioid. 5.  Which statement is true regarding acute pain management? A. Infiltration of the wound has no added value for postoperative pain control. B. For opioid-tolerant patients, their daily opioid dosing before surgery should not be given in case epidural or regional anesthesia is scheduled. C. Tricyclic antidepressants cannot be used in neuropathic pain. D. Opioids with a high intrinsic efficacy are more effective in controlling postoperative pain. 6.  Which statement is true regarding acute pain management from a nursing perspective? A. Even when epidural catheter is placed lumbar of thoracic level, the same local anesthetic dosing can be used. B. Acute pain service is mainly managed by the nurses under the authority of a doctor. C. Nurses can choose their pain assessment tool depending on their experience. D. Epidural solutions contain preservatives to prevent microbial contamination.

391

Hadzic_Ch70_p391-392.indd 391

22/04/19 2:15 PM

392

PART 12

Acute Pain Management

ANSWERS AND EXPLANATIONS 1. A is correct. Before starting PCA, analgesia must be established with an initial loading dose of opioid. Without front loading, MEAC is not achieved for at least three elimination half-lives. PCA is intended to maintain a level of pain control, not to initiate satisfactory analgesia. Therefore, if the PCA process is interrupted by pump failure, a faulty intravenous line, or inadequate patient dosing, the patient will require bolus titration to achieve comfort before re-initiating PCA. B is incorrect. Addition of a basal infusion in IV PCA does not add analgesic proportions; it does add side effects. C is incorrect. Adverse events are correlated with inappropriate use of IV PCA, such as use by proxy. D is incorrect. There is no uniform converting dose. A safety margin should be used when converting opioids because of incomplete cross-tolerance. Converting to 75% of the dose is advised. 2. C is correct. Patients should be warned about local anesthetic systemic toxicity (LAST) and trained to recognize the symptoms (eg, dizziness, metal taste, tinnitus). A is incorrect. Not all patients can be sent home in daycare setting with a continuous peripheral nerve block catheter. Careful patient selection excludes high-risk patients (eg, hepatic or renal failure, coagulopathies). B is incorrect. Depending on the circumstances, a peripheral infusion catheter can be removed at home by a trained nurse or by an educated caregiver. D is incorrect. In case of catheter migration, the catheter should be removed promptly because of the increased risk of toxicity with migration near vascular spaces. 3. B is correct. ASA guidelines recommend at least two nonopioid analgesics as cornerstone for analgesia before adding an opioid. A is incorrect. Chronic pain can be but is not always a result of poorly managed acute pain.

Hadzic_Ch70_p391-392.indd 392

C is incorrect. Neuropathic pain can be very reluctant, though it can be treated with gabapentinoids. D is incorrect. Small doses (0.5–0.15 mg/kg IV) of ketamine are sufficient for a significant potentiation of opioids. 4. C is correct. The epidural catheter should advance the epidural space between 3 and 5 cm. A is incorrect. Coagulation should be checked before placement of an epidural catheter and before its removal. B is incorrect. The level should ideally be chosen mid dermatome of the anticipated trauma. D is incorrect. An epidural infusion can contain local anesthetic, opioid, and/or other additives (eg, clonidine, dexmedetomidine). 5. D is correct. Opioids with a high intrinsic efficacy are more effective in controlling postoperative pain. A is incorrect. Infiltration of the wound does help for postoperative pain. B is incorrect. The basic analgesic regime should be continued even for opioid-tolerant patients. C is incorrect. Tricyclic antidepressants in adjunct to gabapentinoids can help neuropathic pain. 6. B is correct. Acute pain service is mainly managed by the nurses under the authority of a doctor. A is incorrect. Solutions and the pump settings can vary. C is incorrect. A uniform pain assessment tool should be selected for the whole staff. Depending on the specific patient (eg, child vs elderly), a different tool can be used. D is incorrect. Epidural solutions do not contain preservatives, and thus have an increased risk.

Suggested Reading Hadzic A. Organization of an acute pain management service incorporating regional anesthesia techniques. In: Lukof A, Viscusi ER, Schechter L, Lenart S, Colfer K, Witkowski T, eds. Textbook of Regional Anesthesia and Acute Pain Management. 2nd ed. New York, NY: McGraw-Hill Publishing; 2017:chap 72.

22/04/19 2:15 PM

71 Multimodal Analgesia: Pharmacologic Interventions and Prevention of Persistent Postoperative Pain Adam C. Young and Asokumar Buvanendran

QUESTIONS DIRECTIONS: Choose the one best response to each question. 1. A 36-year-old woman with history of Crohn’s disease is complaining of pain in the post-anesthesia recovery unit following ileocecal resection for small bowel obstruction. Which agent would be most appropriate to treat her pain symptoms? A. Oral hydrocodone/acetaminophen 10/325 mg B. Rectal acetaminophen 650 mg C. Intravenous acetaminophen 1000 mg D. Intravenous ketorolac 30 mg 2. When combined with acetaminophen, which of the following substances has been proven to have a synergistic effect? A. Gabapentin B. Lidocaine C. Ketamine D. Ibuprofen 3. Nociceptor sensitization occurs following tissue damage and cyclooxygenase (COX) production of which ligand? A. Prostaglandins (PEs) PGE1 and PGE2 B. Histamine and bradykinin C. Substance P D. Calcitonin gene-related peptide 4. Despite escalating the dose of gabapentin from 300 mg three times daily to 600 mg three times daily, a 56-yearold man fails to endorse analgesia following total knee arthroplasty. What statement best explains this? A. Neuropathic pain does not occur in patients following total knee arthroplasty. B. The patient did not have hyperalgesia, and therefore did not meet indications for anticonvulsant therapy.

C. The dose of gabapentin was escalated too quickly. D. Gabapentin is actively transported intracellular and becomes saturated, preventing further absorption. 5. Central sensitization is modulated through which receptors? A. Voltage-dependent calcium channels B. Transient receptor potential cation channel, subfamily V C. N-type calcium channels D. Voltage-gated sodium channels 6. A 47-year-old woman with a history of prior spine surgery is undergoing revision lumbar spinal fusion to extend her fusion. Preoperatively she takes oxycodone 60 mg/daily. Which medication could reduce her pain in the short- and long-term following this surgery? A. Ketamine B. Celecoxib C. Pregabalin D. Dexmedetomidine 7. Which steroid has been shown to have the ability to reduce the incidence of chronic pain? A. Dexamethasone B. Prednisone C. Hydrocortisone D. Methylprednisolone 8. A 72-year-old man is prescribed tramadol 50 mg four times daily as needed following inguinal hernia repair. Despite taking his medication as prescribed, he fails to endorse analgesia. What laboratory test might elucidate the underlying reason? A. Plasma-free metanephrine level B. Genetic testing for methylenetetrahydrofolate reductase mutation C. Cytochrome 2D6 activity levels D. Urine drug screen

393

Hadzic_Ch71_p393-396.indd 393

19/04/19 11:28 AM

394

PART 12

Acute Pain Management

9. Which of the following measures has been shown to prevent the development of chronic pain following abdominal surgery? A. Intravenous lidocaine infusion B. Epidural analgesia C. Continuous wound infiltration with a subcutaneous catheter D. Transversus abdominis plane blocks 10. A 46-year-old woman presents to the emergency department 4 months following right mastectomy for breast cancer. She is admitted and further workup demonstrates no abnormalities. Perioperative use of which of the following blocks may have reduced her chances of developing chronic pain? A. Pecs 1 and 2 blocks B. Thoracic epidural analgesia C. Paravertebral blocks D. Serratus plane block

ANSWERS AND EXPLANATIONS 1. C is correct. Intravenous acetaminophen has a rapid time to meaningful analgesia less than 30 minutes following administration. A, B, and D are incorrect. An intravenous medication would be appropriate following abdominal surgery. NSAIDs can increase the risk of anastomotic leak and would best be avoided. 2. D is correct. The synergism between nonsteroidal antiinflammatory agents and acetaminophen is well-described. A, B, and C are incorrect. There is no such description of synergism between local anesthetics, anticonvulsants, or ketamine with NSAIDs. 3. A is correct. PGE1 and PGE2 are the byproducts of cell membrane metabolism by cyclooxygenase. They are responsible for sensitizing nociceptors to painful and nonpainful stimuli. B is incorrect. Bradykinin is an inflammatory mediator involved in pain signaling and can cause vasodilation. Histamine is a molecule that acts as a neurotransmitter in the central nervous system and peripherally it is involved in a number of other physiologic processes. C is incorrect. Substance P is an excitatory neurotransmitter involved in the transmission of signals; it is thought to be one of the first chemicals released in response to pain. D is incorrect. Calcitonin gene-related peptide is released in the dorsal horn of the spinal cord and is involved in the pain pathway. 4. D is correct. Escalating the dose of gabapentin further saturates this transporter system and overall absorbed fraction decreases. A is incorrect. Neuropathic pain following total knee arthroplasty is common.

Hadzic_Ch71_p393-396.indd 394

B is incorrect. This is reflected in studies that demonstrate improved analgesia with the addition of anticonvulsants to multimodal protocols. C is incorrect. Gabapentin is actively transported and its bioavailability can vary based on the number of receptors an individual possesses. 5. A is correct. Voltage-dependent calcium channels are upreg­ ulated in the dorsal root ganglia following surgery and are part of the process that leads to central sensitization. B, C, and D are incorrect. The other receptors listed are involved in the pain pathway but have not been linked to the mechanism of central sensitization. 6. A is correct. Opioid-experienced patients present a particularly difficult challenge with acute pain management following surgery. The NMDA receptor has been indicated in chronic pain leading to the use of ketamine as a therapeutic agent. Ketamine has been shown to have beneficial effects in the immediate and delayed postoperative period in spine surgery. B is incorrect. Celecoxib would be relatively contraindicated given the concerns for poor bone healing with the use of NSAIDs. C is incorrect. Pregabalin is a useful medication as part of a perioperative multimodal regimen; however, it has not been studied in this specific population. D is incorrect. Use of dexmedetomidine has not been proven to provide relief from acute or chronic pain. 7. C is correct. Hydrocortisone has been shown in cardiac surgery to reduce rates of chronic pain. A is incorrect. Meta-analyses have shown improved pain management with high dose dexamethasone, but only immediately post-op. B is incorrect. Prednisone is used chronically for inflammatory conditions, which may include those with pain symptoms. However, prednisone has not been studied in the perioperative phase. D is incorrect. Methylprednisolone is a common agent used to treat acute pain in the form of an oral steroid taper. Its use in the perioperative period is unstudied. 8. C is correct. Certain medications, including opioids, require metabolism to an active form in order to exert their desired effects. Cytochrome polymorphisms can lead to reduced levels of active drug, resulting in lack of effect. Up to 15% of the general population may have reduced CYP 2D6 activity. A is incorrect. Plasma-free metanephrines is a useful study to investigate presence of excess catecholamines, as in a pheochromocytoma. B is incorrect. The MTHFR gene has been implicated as one of the many genes that may be involved with addiction, including that of opioids. It has no effect on metabolism. D is incorrect. A urine drug screen is a tool used to monitor compliance during opioid therapy.

19/04/19 11:28 AM

CHAPTER 71

395

Multimodal Analgesia: Pharmacologic Interventions and Prevention of Persistent Postoperative Pain

9. B is correct. Epidural analgesia has been shown to reduce chronic pain in both thoracic surgery and abdominal surgery. A is incorrect. Intravenous lidocaine infusions have demonstrated similar efficacy to epidural analgesia in patients undergoing abdominal surgery; however, there are no studies examining long-term effects. C and D are incorrect. Although both have been shown to have impact on acute pain, neither has demonstrated a long-term effect on chronic post-operative pain following abdominal surgery. 10. C is correct. Of the regional techniques discussed, paravertebral block is the only to reduce pain at 4 months. A and D are incorrect. The Pecs and serratus plane blocks are newer techniques that are useful and may have longterm benefit once investigated for this specific benefit. B is incorrect. Thoracic epidural analgesia is an excellent adjunct for thoracic surgery.

Kairaluoma PM, Bachmann MS, Rosenberg PH, et al. Preincisional paravertebral block reduces the prevalence of chronic pain after breast surgery. Anesth Analg. 2006;103:703-708. Lavand’homme P, De Kock M, Waterloos H. Intraoperative epidural analgesia combined with ketamine provides effective preventive analgesia in patients undergoing major digestive surgery. Anesthesiology. 2005;103:813-820. Loftus RW, Yeager MP, Clark JA, et al. Intraoperative ketamine reduces perioperative opiate consumption in opiate-dependent patients with chronic back pain undergoing back surgery. Anesthesiology. 2010;113:639-646. Neurontin Package Insert, Pfizer. Accessed April 8, 2014. http://www. pfizer.com/products/product-detail/neurontin Ong CK, Seymour RA, Lirk P, et al. Combining paracetamol (acetaminophen) with nonsteroidal antiinflammatory drugs: a qualitative systematic review of analgesic efficacy for acute postoperative pain. Anesth Analg. 2010;110:1170-1179.

Suggested Readings

Rasmussen ML, Mathiesen O, Dierking O, et al. Multimodal analgesia with gabapentin, ketamine and dexamethasone in combination with paracetamol and ketorolac after hip arthroplasty: a preliminary study. Eur J Anaesthesiol. 2010;27:324-330.

Gan SH, Ismail R, Wan Adnan WA, Zumi W. Impact of CYP2D6 genetic polymorphism of tramadol pharmacokinetics and pharmacodynamics. Mol Diag Ther. 2007;11:171-181.

Rushfeldt CF, Sveinbjornsson B, Soreide K, Vonen B. Risk of anastomotic leakage with use of NSAIDs after gastrointestinal surgery. Int J Colorectal Dis. 2011;26:1501-1509.

Hadzic A. Multimodal analgesia: pharmacologic interventions and prevention of persistent postoperative pain. In: Young AC, Buvanendran A, eds. Textbook of Regional Anesthesia and Acute Pain Management. 2nd ed. New York, NY: McGraw-Hill Publishing; 2017:chap 75.

Sinatra RS, Jahr JS, Reynolds LW, et al. Efficacy and safety of single and repeated administration of 1 gram of intravenous acetaminophen injection (paracetamol) for pain management after major orthopedic surgery. Anesthesiology. 2005;102:822-831.

Ibarra MM, S-Carralero GC, Vicente GU, et al. Chronic postoperative pain after general anesthesia with or without a single-dose preincisional paravertebral nerve block in radical breast cancer surgery. Rev Esp Anestesiol Reanim. 2011;58:290-294. Issioui T, Klein KW, White PF, et al. The efficacy of premedication with celecoxib and acetaminophen in preventing pain after otolaryngologic surgery. Anesth Analg. 2002;94:1188-1193.

Hadzic_Ch71_p393-396.indd 395

Tiippana EM, Hamunen K, Kontinen VK, et al. Do surgical patients benefit from perioperative gabapentin/pregabalin? A systematic review of efficacy and safety. Anesth Analg. 2007;104:1545-1556. Weis F, Kilger E, Roozendaal B, et al. Stress doses of hydrocortisone reduce chronic stress symptoms and improve health- related quality of life in high-risk patients after cardiac surgery: a randomized study. J Thorac Cardiovasc Surg. 2006;131:277-282.

19/04/19 11:28 AM

This page intentionally left blank

Hadzic_FM_p00i-pxx.indd 6

22/04/19 6:40 PM

72 The Role of Nonopioid Analgesic Infusions in the Management of Postoperative Pain James K. Kim

QUESTIONS DIRECTIONS: Choose the one best response to each question. 1. Perioperative IV ketamine in patients undergoing cervical and lumbar spine surgery has been shown to: A. Have no effect on pain scores B. Increase perioperative analgesic requirements C. Improve patient satisfaction D. Affect bone healing after spine surgery 2. Perioperative IV ketamine has not been shown to be beneficial in the setting of: A. Postamputation pain B. Epidural analgesia C. Opioid tolerance D. Burn patients 3. In the perioperative setting, IV lidocaine infusion has been shown to increase: A. Inflammatory activity B. Opioid requirement C. Hospital length of stay D. Rate of bowel function return 4. IV ketamine has not been shown to be beneficial in the setting of: A. Pelvic surgery (gynecologic, urologic) B. Spine surgery C. Total intravenous anesthesia with propofol and remifentanil D. Abdominal surgery 5. IV lidocaine infusion has been shown to be beneficial in the setting of: A. Total hip replacement B. Coronary artery bypass surgery

C. Spine surgery D. Total intravenous anesthesia 6. A high-dose naloxone infusion may lead to hyperalgesia and increased opioid requirement by: A. Promoting endorphin release B. Displacing endorphins from receptor sites (not related to analgesia) C. Blocking the action of the released or displaced endorphin at the postsynaptic receptor D. Augmenting the activity of opioid receptors 7. Which of the following is not an N-methyl-D-aspartate (NMDA) antagonist? A. Ketamine B. Dexmedetomidine C. Naloxone D. Magnesium 8. An intraoperative esmolol infusion has been shown to: A. Increase inhalation anesthetic requirement B. Decrease hemodynamic responses C. Prolong discharge time D. Increase opioid requirement 9. Dexmedetomidine has all of the following properties except: A. Anxiolytic B. Sedative C. Analgesic D. Antiemetic 10. Which of the following nonopioid analgesic infusions is most likely to lead to respiratory depression? A. Lidocaine B. Ketamine C. Dexmedetomidine D. Naloxone

397

Hadzic_Ch72_p397-400.indd 397

22/04/19 2:16 PM

398

PART 12

Acute Pain Management

ANSWERS AND EXPLANATIONS 1. C is correct. Perioperative IV ketamine in patients undergoing spine surgery has been shown to improve overall patient satisfaction. A is incorrect. IV ketamine has been shown to decrease overall pain scores. B is incorrect. IV ketamine decreases perioperative analgesic requirements, including opioids. D is incorrect. Ketamine has not been shown to have any effect on bone healing. 2. A is correct. IV ketamine infusion for 72 hours was not effective in reducing morphine consumption or in decreasing the incidence of stump allodynia. At 6-month follow-up, the incidences of phantom pain and stump pain did not show any statistical difference. Overall, ketamine is not known to reduce acute central sensitization or the incidence and severity of postamputation pain. B is incorrect. The addition of IV ketamine to epidural analgesia has been shown to result in less morphine requirements and reduced area of hyperalgesia. C is incorrect. The addition of IV ketamine in the perioperative setting appears to be beneficial in opioid-tolerant patients by decreasing both opioid requirements and pain scores. D is incorrect. IV ketamine has demonstrated its analgesic efficacy in burns, with a reduction in secondary hyperalgesia, in comparison with opioid monotherapy. The combination of ketamine plus morphine resulted in the abolition of hyperalgesia in a study of 67 burn patients. 3. D is correct. IV lidocaine infusion has been shown to improve recovery by promoting a shorter duration of postoperative ileus. A is incorrect. IV lidocaine has been shown to reduce surgery-induced immune changes and to modify the antiinflammatory activity, which modulates the surgeryinduced response. B is incorrect. IV lidocaine has been shown to decrease opioid requirements in the perioperative setting. C is incorrect. One study showed that IV lidocaine accelerated return of bowel function and shortened the length of hospital stay by 1 day. 4. C is correct. No beneficial effect of a ketamine infusion was noted when the general anesthetic consisted of total intravenous anesthesia with remifentanil and propofol infusions. The absence of beneficial effect may be related to the generous use of intraoperative opioids. A, B, and D are incorrect. IV ketamine has been shown to be beneficial in the setting of pelvic surgery (gynecologic, urologic), spine surgery, and abdominal surgery. 5. C is correct. Intraoperative lidocaine infusion has been shown to reduce postoperative pain after lumbar surgery.

Hadzic_Ch72_p397-400.indd 398

A is incorrect. In the setting of total hip replacements, IV lidocaine infusion did not affect postoperative pain scores, opioid consumption, or range of motion in hip flexion. B is incorrect. Lidocaine infusion was noted to be ineffective in reducing the supplemental fentanyl, midazolam, or propranolol postoperative requirements in patients who underwent coronary artery bypass. D is incorrect. IV lidocaine has shown no added benefit in the setting of total intravenous anesthesia. 6. C is correct. Naloxone has been shown to initially produce analgesia in a dose-dependent manner and then cause hyperalgesia. At higher doses, naloxone blocks the action of the released or displaced endorphins at the postsynaptic receptor. A, B, and D are incorrect. Promoting endorphin release, displacing endorphins from receptor sites, and augmenting the activity of opioid receptors may explain the mechanisms by which low-dose naloxone infusions produce analgesia. 7. B is correct. Dexmedetomidine is an α2 agonist with properties including anxiolysis, sedation, and hypnosis. It does not have any effect on NMDA receptors. A is incorrect. Ketamine is an NMDA antagonist. C is incorrect. Naloxone is an opioid receptor antagonist that is also an antagonist of NMDA receptors. D is incorrect. Magnesium is the second most common intracellular ion that also acts as an antagonist of NMDA receptors, minimizing the perception and duration of pain. 8. B is correct. Esmolol is a cardioselective β1-receptor blocker used to blunt the adrenergic response caused by various stimuli during surgery. It has a very rapid onset and short duration of action, making it an ideal drug to titrate. A is incorrect. Esmolol infusion has been shown to reduce the intraoperative use of inhalation anesthesia. C is incorrect. In the setting of ambulatory surgery, esmolol infusion has been shown to decrease the time to discharge. D is incorrect. Esmolol infusion has been shown to decrease intraoperative and postoperative fentanyl requirements. 9. D is correct. Dexmedetomidine has not been shown to have antiemetic properties. A, B, and C are incorrect. Dexmedetomidine has anxiolytic, sedative, hypnotic, and analgesic properties. 10. B is correct. Although ketamine causes less respiratory depression than other sedatives such as propofol and opioids, it is an NMDA antagonist that may cause mild respiratory depression. A is incorrect. Lidocaine is a sodium channel blocker that has no effect on respiratory drive.

22/04/19 2:16 PM

CHAPTER 72

The Role of Nonopioid Analgesic Infusions in the Management of Postoperative Pain

C is incorrect. Dexmedetomidine is a sedative that is able to achieve its effects without causing respiratory depression. D is incorrect. Naloxone is an opioid receptor antagonist that can be used to reverse opioid-induced respiratory depression.

399

Kim KT, Cho DC, Sung JK, et al. Intraoperative systemic infusion of lidocaine reduces postoperative pain after lumbar surgery: a double-blinded, randomized, placebo-controlled clinical trial. Spine J. 2014;8:1559-1566. McGuinness SK, Wasiak J, Cleland H, et al. A systematic review of ketamine as an analgesic agent in adult burn injuries. Pain Med. 2011;12:1551-1558.

Suggested Readings Hadzic A. The role of nonopioid analgesic infusions in the management of postoperative pain. In: De Oliveira, Jr GS, Benzon HT, White PF, eds. Textbook of Regional Anesthesia and Acute Pain Management. 2nd ed. New York, NY: McGraw-Hill Publishing; 2017:chap 76.

Hadzic_Ch72_p397-400.indd 399

22/04/19 2:16 PM

This page intentionally left blank

Hadzic_FM_p00i-pxx.indd 6

22/04/19 6:40 PM

PART 13 Education in Regional Anesthesia

Chapter 73

Hadzic_Ch73_p401-406.indd 401

Teaching Regional Anesthesia  403

22/04/19 6:32 PM

This page intentionally left blank

Hadzic_FM_p00i-pxx.indd 6

22/04/19 6:40 PM

73 Teaching Regional Anesthesia Alwin Chuan and Bernard Roach

QUESTIONS DIRECTIONS: Choose the one best response to each question. 1.  Which of the following is the minimum number of regional anesthesia procedures as stipulated by the current Anesthesiology Residency Review Committee (RRC) program? A. 100 epidurals, 50 spinals, and 40 peripheral nerve blocks B. 50 epidurals, 50 spinals, and 40 peripheral nerve blocks C. 50 epidurals, 75 spinals, and 50 peripheral nerve blocks D. 50 epidurals, 100 spinals, and 50 peripheral nerve blocks 2.  How many epidural blocks are required for residents to achieve clinical competence with an 80% success rate? A. 15–30 B. 30–45 C. 45–60 D. 60–90 3.  Which type of block is the most frequently performed in residency training programs? A. Spinal B. Combined spinal-epidural C. Epidural D. Peripheral nerve 4.  According to Neal, to what extent is ultrasound guidance used in comparison to peripheral nerve stimulation? A. Less by a wide margin B. Less by a short margin C. More by a short margin D. More by a wide margin 5.  The worst consequence of inadequate training for residents in regional anesthesia is: A. Reluctance to attempt a block when clinically indicated B. Attempting a block without the appropriate skills C. Inability to train others in nerve blocks D. Poor block selection for a given type of surgery 6.  A survey by Smith et al showed the greatest lack of confidence in CA-3 residents in performing regional anesthesia was for which block?

A. Interscalene B. Epidural (thoracic) C. Sciatic D. Retrobulbar 7.  Rosenblatt showed more than how many interscalene blocks were required to show 50% autonomy? A. 5 B. 10 C. 15 D. 20 8.  According to a national survey, which is the most infrequently performed upper limb block? A. Interscalene B. Supraclavicular C. Snfraclavicular D. Axillary

ANSWERS AND EXPLANATIONS 1. B is correct. The current RRC program requirement state that residents must perform 50 epidural, 50 spinal, and 40 peripheral nerve blocks plus an additional 25 nerve blocks for pain management.1 2. D is correct. Studies of clinical competence show it takes between 60 and 90 epidural blocks to reach at least 80% success.2-4 3. C is correct. See Figure 73–1. 4. D is correct. According to Neal, ultrasound-guidance as a nerve localization tool now surpasses by a wide margin peripheral nerve stimulation. 5. B is correct. In the year 2000, approximately 40% of residents reported inadequate experience with peripheral nerve blocks, which had both educational and safety implications.5 Residents not adequately trained in a particular block are unlikely to use that block in practice6; or worse, they may try to use techniques without the proper skills.7,8 6. D is correct. See Figure 73–2. 403

Hadzic_Ch73_p401-406.indd 403

22/04/19 6:32 PM

404

PART 13

Education in Regional Anesthesia

18

12

6

0

1990 Spinal

2000

Epidural

Nerve Block

FIGURE 73–1  Distribution of types of regional anesthesia in residency training programs did not change significantly between 1990 and 2000 (P = .75).

CA1

CA2

CA3 91%

Retrobulbar 75%

Sciatic 62%

Femoral 51%

Brachial-Interscalene 25%

Epidural-Thoracic

20%

Intravenous Regional

14%

Ankle 7%

Brachial-Axillary Epidural-Lumbar Spinal

0% 0%

0%

100%

FIGURE 73–2  The percentage of residents, per training year, who categorized themselves as “not confident” in performing a particular block. Exact percentages are listed for the CA-3 resident class. Residents felt least confident with peripheral nerve blocks, and no CA-3 resident admitted to being “not confident” for spinal or lumbar epidural anesthesia.

IV Regional Intercostal Cervical Upper extremity Axillary Interscalene Wrist Supraclavicular Elbow Infraclavicular Lower extremity Ankle Femoral Sciatic Popliteal 0

100%

FIGURE 73–3  Percentage of peripheral nerve blocks performed in practice in the United States, as reported by practitioners. (Reproduced with permission from Hadzić A, Vloka JD, Kuroda MM, et al: The practice of peripheral nerve blocks in the United States: a national survey [p2e comments]. Reg Anesth Pain Med. 1998 May-Jun;23(3):241–246.)

Hadzic_Ch73_p401-406.indd 404

22/04/19 6:32 PM

CHAPTER 73

7. B is correct. Rosenblatt et al showed that more than 10 interscalene blocks are necessary before the resident attains at least 50% autonomy.9 8. C is correct. See Figure 73–3.

References 1. Accreditation Council for Graduate Medical Education. Program requirements for graduate medical education in anesthesiology. July 2016. Accessed August 29, 2016. 2. Konrad C, Schuepfer G, Wietlisbach M, Gerber H. Learning manual skills in anesthesiology. Is there a recommended number of cases for anesthetic procedures? Anesth Analg. 1998;86:635-639. 3. Kopacz DJ, Neal JM, Pollock JE. The regional anesthesia “Learning Curve”: What is the minimum number of epidural and spinal blocks to reach consistency? Reg Anesth. 1996;21:182-190. 4. Schuepfer G, Konrad C, Schmeck J, et al. Generating a learning curve for pediatric caudal epidural blocks: an empirical evaluation of technical skills in novice and experienced anesthetists. Reg Anesth Pain Med. 2000;25:385-388.

Hadzic_Ch73_p401-406.indd 405

Teaching Regional Anesthesia

405

5. Kopacz DJ, Neal JM. Regional anesthesia and pain medicine: residency training—the year 2000. Reg Anesth Pain Med. 2002;27:9-14. 6. Buffington CW, Ready LB, Horton WG. Training and practice factors influencing the use of regional anesthesia: implications for resident education. Reg Anesth. 1986;11:2-6. 7. Smith MP, Sprung J, Zura A, et al. A survey of exposure to regional anesthesia techniques in American anesthesia residency training programs. Reg Anesth Pain Med. 1999;24:11-16. 8. Hadzic A, Vloka JD, Kuroda MM, et al. The practice of peripheral nerve blocks in the United States: a national survey. Reg Anesth Pain Med. 1998;23:241-246. 9. Rosenblatt MA, Fishkind D. Proficiency in interscalene anesthesia—how many blocks are necessary? J Clin Anesth. 2003;15:282-288.

Suggested Reading Hadzic A. Teaching regional anesthesia. In: Niazi AU, Neal JM, eds. Textbook of Regional Anesthesia and Acute Pain Management. 2nd ed. New York, NY: McGraw-Hill Publishing; 2017:chap 77.

22/04/19 6:32 PM

This page intentionally left blank

Hadzic_FM_p00i-pxx.indd 6

22/04/19 6:40 PM

PART 14 Statistics and Principles of Research Design in Regional Anesthesia and Acute Pain Medicine Chapter 74

Hadzic_Ch74_p407-412.indd 407

Principles of Statistical Methods for Research in Regional Anesthesia   409

19/04/19 11:33 AM

This page intentionally left blank

Hadzic_FM_p00i-pxx.indd 6

22/04/19 6:40 PM

74 Principles of Statistical Methods for Research in Regional Anesthesia Maxine M. Kuroda

QUESTIONS DIRECTIONS: Choose the one best response to each question. 1. What is statistics? A. The field of applied mathematics that analyzes data to prove a specific hypothesis B. The study of variability C. The field of applied mathematics that analyzes data to know what the minimum sample size is to enroll in research studies D. The field of applied mathematics that should only be used to analyze data collected in large surveys and research studies 2. Which statement is true regarding basic study designs? A. The randomized clinical trial (RCT) is the gold standard of research designs, and thus should be used whenever possible. B. When deciding on which study design to use, choose the one that involves the most treatment arms as this helps the statistician to stratify the outcome data to control for confounding. C. When deciding on which study design to use, choose the one that collects the most variables as this enables the statistician to analyze the data with the more powerful, next-generation statistical approaches. D. None of the above 3. Which statement is correct regarding descriptive vs. inferential statistics? A. Descriptive statistics are used for case reports, whereas inferential statistics are used for randomized clinical trials. B. If time is limited, go directly to the inferential statistical analyses because they directly answer the research question. C. Distributional properties (such as measures of central tendency and variability) are more important than how the data were collected. D. A variable’s descriptive characteristics impact choice of statistical approach.

4. What is variance? A. Variance is the standard deviation squared, and can be expressed as a range, standard deviation, standard error of the mean, error bars on a graph, the 95% confidence interval, or margins of error in surveys. B. Variability itself varies in the eyes of the beholder. C. Analysis of variance produces an F ratio whose numerator represents variation between sample means and whose denominator represents variation within the samples. D. All of the above 5. What statement is true regarding type I and type II errors? A. The type I error can be considered a “false alarm” while the type II error can be considered a “missed opportunity.” B. For stating the null and alternative (research) hypotheses, the type II error is more important to consider than the type I error. C. Negative studies mean that no type I or type II errors were committed. D. Positive studies mean that the type I error was much smaller than the type II error. 6. Which statement is true regarding null and alternative (research) hypotheses? A. Failure to reject the null hypothesis is more likely to occur if its “address” on the sampling distribution is in the tails of the distribution. B. The alternative (research) hypothesis may be a nondirectional, a unidirectional (upper tail critical), or a unidirectional (lower tail critical) statement. C. For studies that are in a new field of research endeavor, try to choose one of the unidirectional alternative hypotheses as they help to focus the research direction for studies in a field. D. The null and alternative hypotheses can be modified provided interim analyses have been conducted on data from the first few subjects by a qualified statistician.

409

Hadzic_Ch74_p407-412.indd 409

19/04/19 11:33 AM

410

PART 14

Statistics and Principles of Research Design in Regional Anesthesia and Acute Pain Medicine

7. Which statement is true regarding sample size estimation? A. With sample sizes that are overly large, it is possible for differences that are small and perhaps even clinically unimportant to be declared statistically significant. B. It is most economical to use smaller sample sizes when searching for smaller differences, and to reserve the larger sample sizes when searching for larger differences. C. Power is lower with larger sample sizes because type II error (β) is increased. D. For sample size estimation, the type I error is more important to consider than the type II error. 8. Which statement is true regarding P-values vs. confidence intervals? A. A P-value < 0.05 indicates that the effect is real. B. Trials that can show P-values < 0.05 have at least 95% power to detect a difference. C. Confidence intervals (CI) are preferable to P-values as they indicate the range of possible effect sizes compatible with the data. D. A trial that results in a P-value < 0.001 has proven that a difference truly exists because the chances of an error are so small. 9. Which statement is true regarding bias and confounding? A. Positive studies are those with a confounding variable. B. Negative studies do not have any evidence of confounding or bias. C. Bias is systematic error. D. It is often possible to adjust for confounding and bias by statistical methods (eg, stratification on levels of the confounder, regression modeling). 10. What statement is true regarding prospectively collected data? A. Data that are prospectively collected are not immune to cohort effects. B. Because data collected prospectively must be on a continuous scale, measures of central tendency (mean, median, mode), spread and distributional shape can be evaluated. C. Serial measures on the same subjects can be analyzed by the Student t test as long as only two groups of subjects are included in the analysis. D. Error variance cannot change over time so prospectively collected data are more reliable than retrospectively collected data. 11. What statement is true regarding types of data? A. Consult a statistician at the planning stage of a research study. B. Investigators should keep a data dictionary that includes variable name, variable label and definition, variable type, value categories and labels, and any comments that should be considered in data collection or analysis. C. Statisticians can discuss choice of the statistical tests that are appropriate to address the research hypotheses,

Hadzic_Ch74_p407-412.indd 410

whether data transformations are needed (and, if so, which ones), and applicable multiple comparisons. D. All of the above 12. Which statement is true regarding interpreting results? A. It is difficult to mislead the readership when findings are presented graphically. B. Excluding error bars is one way of showing the readership that there must have been very little error in the measurements. C. If the statistical tests and associated P-values are presented, it is not necessary to include the 95% confidence intervals. D. None of the above

ANSWERS AND EXPLANATIONS 1. B is correct. It is seldom feasible to study an entire population, so samples taken from the population are studied. But each sample will be slightly, moderately, or vastly different from the next sample. A is incorrect. We may find a result that is anticipated (given current knowledge in the field)—or—we may not (a surprise for everyone, including ourselves)! We do not set out on a research study with the intention of “proving” that we are correct. C is incorrect. Estimates of a feasible and reasonable sample size are needed for inferential studies that test hypotheses. Nonetheless, there are descriptive studies in which sample size calculations are not needed. D is incorrect. While data from large surveys (eg, national political opinion polls) are frequently collected and analyzed—particularly, in an election year—small surveys and research studies can also be important to evaluate. 2. D is correct. Every study design has its pros and cons. Study designs that are overly complex are difficult to conduct and their data may be too entangled for clear interpretation. A is incorrect. The hallmark of the RCT is that the investigator manipulates the exposure or treatment. Clearly, this is not always ethical—even if subjects are randomly allocated to exposure or treatment “arms.” Women who are pregnant cannot be forced to smoke or, conversely, not to smoke, after they have been enrolled in a study of the effects of smoking on the fetus! B is incorrect. The inclusion of unnecessary “arms” in the trial escalates costs, and is confusing to everyone involved—ethics committees, research assistants, statisticians, and journal editors/readers. C is incorrect. Questionnaires, instruments, and variables are not chosen for the quantity of data that they collect; they are chosen for their ability to tap the information needed to address the research objectives. They should be accurate and reliable (ie, have good psychometric properties and a good track record of use in the field), but they

19/04/19 11:33 AM

CHAPTER 74

Principles of Statistical Methods for Research in Regional Anesthesia

should also have other desirable features (eg, ease of using the transducer on an ultrasound machine and clarity of ultrasound pictures). 3. D is correct. Descriptive characteristics must be considered when choosing the inferential approach! A variable that has a bell-shaped distribution may not be treated in the same way as a variable with a highly skewed distribution. A is incorrect. Descriptive statistics are useful for all study designs. B is incorrect. Descriptive statistical analyses should not be overlooked! Spreadsheets must be checked for data entry errors; outlying and missing values must be identified. C is incorrect. Loosely controlled data collection (eg, not adhering to protocol assessment times, differences in interview environment or research assistant) can result in unreliable and inaccurate data. 4. A, B, and C are all true, so option D is correct. Simply put, no matter how it is sliced and diced, variability is the substance of statistics. It is useful to know the different ways that variability is expressed in order to use them appropriately. For instance, in certain contexts, the range may be more meaningful than the standard error of the mean; the coefficient of variation cannot be used for measures where zero is arbitrary. What may be worrisome variability to one individual, may be acceptable variability to another. If there is no treatment effect (that is, H0 is true), the F ratio will be ∼1.0 because numerator and denominator variations are similar. 5. A is correct. The type I error is the probability of claiming a statistically significant difference when, in truth, there is none. The type II error is the probability of claiming no statistically significant difference when, in truth, there is one. B is incorrect. Type I and II error rates are not stated with the null and alternative (research) hypotheses. C is incorrect. Negative studies are those whose results do not reject the null hypothesis. They are frequently underappreciated and often not even published. D is incorrect. Positive studies are those whose results do reject the hypothesis. However, these results may be overstated, and the studies may not be adequately examined for factors that could have resulted in spuriously “significant” findings. 6. B is correct. An example of a nondirectional hypothesis is: sensory-motor block duration differs among subjects given catheters A or B during shoulder surgery. An example of a unidirectional hypothesis is: time to return to walking is greater in subjects given local anesthetic A than in subjects given local anesthetic B for total knee arthroplasty.

Hadzic_Ch74_p407-412.indd 411

411

A is incorrect. The null hypothesis is more likely to be rejected if its address is in one or the other tail of the sampling distribution. This is because rare occurrences reside in the tails of the sampling distribution. C is incorrect. For research questions in a new field, it is typically too early to entertain unidirectional alternative hypotheses. D is incorrect. Once stated, hypotheses cannot be changed even if interim analyses have been planned. 7. A is correct. Larger sample sizes decrease β, thereby increasing power (1-β). Larger sample sizes also decrease the pooled variance (σ). Together, statistically significant but clinically unimportant differences may be identified. B is incorrect. The size of the difference that is important to detect should not be tweaked to save on costs as this jeopardizes interest among the medical community. C is incorrect. In fact, larger sample sizes tend to decrease the type II error (β), and since power is 1-β, power would increase. D is incorrect. Investigators must weigh the risks and benefits of both type I and type II errors for their studies. When estimating sample size, the investigator sets the type I error (typically 5%), but has only indirect control over the type II error. That is, s/he can decrease the type II error by increasing sample size, and can increase the type II error by decreasing sample size. 8. C is correct. Confidence intervals indicate statistical significance, provide an estimate of the size of the difference, and the likely variability in the estimated size of the difference. A is incorrect. By chance alone, about 1 in 20 “significant” findings will be spurious. B is incorrect. Although the type II error does tend to go up or down depending on whether the type I error is decreased or increased (respectively), the type I and II error probabilities do not sum to 1.0. Thus, while α might be 0.05, power is not necessary 95%. D is incorrect. It takes more than a few studies “to prove” anything. For instance, there could have been uncontrolled confounders in the trial; the trial could have been biased. A trial with a P-value < 0.001 simply indicates that there is evidence that a difference exists. 9. C is correct. Subjects who are enrolled in research studies are aware that they are being studied (Hawthorne effect), resulting in biased responses that may influence research results in unknown ways. A is incorrect. Positive studies are those whose results are in the expected direction. They may or may not include potential confounders. B is incorrect. No study is completely immune to bias. D is incorrect. In general, bias cannot be adjusted by statistical methods. The best approach is to use past experiences (even those of colleagues) to help anticipate certain features of a study that could make it vulnerable to bias.

19/04/19 11:33 AM

412

PART 14

Statistics and Principles of Research Design in Regional Anesthesia and Acute Pain Medicine

10. A is correct. Such data could still be biased (eg, data on intensity of postoperative pain that are collected before ultrasound-guided nerve blocks may not be comparable to data on pain that are collected after ultrasound-guided nerve blocks have been established). B is incorrect. Data collected prospectively may be on any metric (eg, continuous, ordinal, or nominal). C is incorrect. Since serial measures are inherently linked, the data should be analyzed by procedures that can account for changes over time. Even if averaged separately for each of the two groups, differences between the groups may be masked if the data were averaged over the period of study. D is incorrect. Testing measurements may vary in their responsiveness to changes over time.

12. D is correct. A, B, and C are false statements regarding interpreting results. A is incorrect. It is very easy to create misleading graphics by creative manipulation of the X and Y axes. B is incorrect. Error bars are informative to readers. Some measures have little variability; others have substantial variability (or may include outlying values that are suspect). C is incorrect. P-values indicate whether trial results are likely to have occurred simply through chance (given the null hypothesis is true). Confidence intervals indicate statistical significance, provide an estimate of the size of the difference, and the likely variability in the estimated difference.

11. A, B and C are all true, so option D is correct. “To call in the statistician after the experiment is done may be no more than asking him to perform a post-mortem examination: He may be able to say what the experiment died of.” Sir Ronald A. Fisher

Suggested Reading

Hadzic_Ch74_p407-412.indd 412

Hadzic A. Principles of statistical methods for research in regional anesthesia. In: Kuroda MM, ed. Textbook of Regional Anesthesia and Acute Pain Management. 2nd ed. New York, NY: McGraw-Hill Publishing; 2017:chap 79.

19/04/19 11:33 AM

Index Note: Page numbers followed by f or t refer to the page location of figures and tables, respectively. A Aα fibers, 11–12 Aβ fibers, 11–12 Aγ fibers, 11–12 AAG. See Alpha-1 acid glycoprotein Abdominal aorta, 245–246 Abdominal surgery chronic pain after, 394–395 epidural anesthesia for, 381–382 thoracic epidural anesthesia for, 109, 111, 114, 117 Abducens nerve, 137, 140 Absorption, 143–144 Accidental total spinal anesthesia, 100–101 Acetaminophen in children, 285–286 ibuprofen and, 393–394 nonsteroidal anti-inflammatory drugs and, 393–394 for persistent postoperative pain, 393–394 Acidemia, in chronic renal disease, 299–300 Acoustic impedance, 143–145 Acoustic velocity, 143–144 Acquired peripheral neuropathies, 303–304 Action potential, 53, 55 Acute compartment syndrome predisposing factors, 308t regional anesthesia and, 307–308 tibial fracture as cause of, 325–327 Acute pain in children, 285–286 in elderly, 292 in emergency department, 343–345 general questions about, 391–392 natural killer cells and, 72 in opioid-dependent patients, 319–323 regional anesthesia in, 391–392 Addiction definition of, 319, 321 opioid, 319, 321 Adductor brevis, 203–204 Adductor canal, 205 Adductor magnus, 190f, 203–204, 208, 211 Adductor pollicis muscle, 186 Adnexal surgery, eye blocks for, 240–241 Aging. See also Elderly autonomic nervous system changes secondary to, 289–291 pain management affected by, 290, 292 peripheral nerves affected by, 12, 14

Hadzic_Index_p413-428.indd 413

Akinesia, 240–241 Alcohol and chlorhexidine gluconate, for infection control, 64 Alcoholism, 356 Allergic reactions to lidocaine, 87, 89 to local anesthetics, 24–25 Allogeneic blood transfusion, 65, 67 Alpha-1 acid glycoprotein, 325, 327 Alternative hypothesis, 409, 411 Ambiance, 147–148 Ambulatory surgery multimodal analgesia in, 309–310 peripheral nerve blocks for, 309–310 American Academy of Pain Medicine, 321 American College of Surgeons National Surgical Quality Improvement Program database, 381–382 American Pain Society, 321 American Society of Anesthesiologists (ASA), 63–64 American Society of Interventional Pain Physicians, 321 American Society of Regional Anesthesia and Pain Medicine, 314–315 Ametop, 273, 275 Amides anaphylaxis to, 24–25 in children, 273–274 history of, 23–24 tumor necrosis factor-α-induced Src activation blocked by, 66, 68 A-mode, 143–144 Amyotrophic lateral sclerosis, 303, 305 Anal sphincter tone, 123, 125 Analgesia epidural. See Epidural anesthesia and analgesia local infiltration. See Local infiltration analgesia postoperative, liposomal bupivacaine for, 28–29 spinal. See Spinal analgesia Analgesic adjuvants infusate addition of, 37–39 in peripheral nervous system, 31–32 Anaphylaxis lidocaine as cause of, 87, 89 local anesthetics as cause of, 24–25

Anesthesia costs associated with, 379–380 epidural. See Epidural anesthesia and analgesia general. See General anesthesia for humanitarian relief operations, 337–339 induction time for, 379 regional. See Regional anesthesia spinal. See Spinal anesthesia tumescent, 3–4 turnover time for, 379–380 Anesthesia response plan, in disaster conditions, 337, 339 Anesthetic agents cancer metastasis and, 65, 67 local. See Local anesthetic(s) Anesthetic pathway benefits of, 59–60 goals of, 59–60 Angle, 147–148 Ankle block, ultrasound-guided, 147, 149, 215–216 Anterior approach to quadratus lumborum block, 219, 221f, 222 to sciatic nerve block, 207–211 Anterior longitudinal ligaments, 94, 97 Anterior meningo-vertebral ligament, 17, 19 Anterior rami, 7, 9 Anterior superior alveolar nerve anatomy of, 233–235 block of, 233–234 Anterior superior iliac spine, as ultrasoundguided fascia iliaca block landmark, 195, 197 Anterior tibial artery, 215–216 Anticoagulants low-molecular-weight heparin, 313–315 neuraxial anesthesia in, 313–315 peripheral nerve blocks in, 313–315 quadratus lumborum block and, 220, 222 warfarin, 313–314 Anticoagulation, eye blocks and, 240–241 Antiplatelet medications, before neuraxial blockade, 313–314 Anxiety peripheral nerve injury and, 356–357 preoperative, 310

19/04/19 1:40 PM

414

Index

Aortic regurgitation, maternal, 296–298 Aortic stenosis description of, 296–297 epidural anesthesia and analgesia in, 110, 115 Apixaban, 313, 315 Apnea, after high neuraxial anesthesia, 299, 301 Arachnoid cells, 138f Arachnoid mater, 137–138 Arachnoid sheath, trabecular, 18–19 Arachnoiditis, 100–101, 104–105 Artery of Adamkiewicz, 251, 254 Artifacts, ultrasound, 147–149 ASD. See Atrial septal defect Aspirin, neuraxial anesthesia and, 313–314 ASRA. See American Society of Regional Anesthesia and Pain Medicine Atrial septal defect, 295–296 Austere environmental medicine anesthesia infrastructure transport in, 333–334 continuous peripheral nerve blocks in, 333–335 general anesthesia with vapors in, 333–334 logistical challenges in, 333–334 opioids in, 333, 335 regional anesthesia in, 333–335 Autonomic dysreflexia, in spinal cord injuries, 304–306 Autonomic nervous system, in elderly, 289–291 Awake cardiac surgery benefits of, 330t high thoracic epidural analgesia for, 329–330 Awake endotracheal intubation regional anesthesia for, 81–83 remifentanil for sedation during, 81, 84 topical anesthesia for, 81–83 Awake fiberoptic intubation methylene blue administration during, 81, 83–84 regional anesthesia during, 82, 85 Awake nasal intubation, gag reflex in, 81, 84 Axial resolution, 143–144 Axillary artery, 174–175, 179 Axillary brachial plexus block description of, 264, 268 perivascular approach of, 177–178 ultrasound-guided, 177–179 Axillary nerve anatomy of, 9 ultrasound-guided block of, 167–168 Axonotmesis, 11–12, 356–357 Axons, 7, 15–16 B Back pain, postepidural placement of, 113, 119 Barker, Arthur, 4 Basal infusion rate, of continuous local anesthetics, 37–38 Beach-chair positioning, 359–360

Hadzic_Index_p413-428.indd 414

Betamethasone, 124–125 Bias, 410–411 Bicarbonate, 265, 268 Bicep muscles, 177–178 Biceps femoris, 190f Bier, August, 3, 24 Bier block, 87–90 Bladder tumor, lateral excision of, 204 Block cart for continuous peripheral nerve blocks, 49, 51 medications on, 49, 51 Block documentation, 363, 365 Block team, 309–310 Blood pressure, systolic, epinephrine effects on, 363–364 Blood transfusion, immunomodulatory effect of, 65, 67 Blood–nerve barrier, 15–16 Bloody tap, 113, 118 B-mode, 143–144 Body mass index, peripheral nerve injury and, 356–357 Bonica, John, 4 Boyle’s law, 363, 365 Brachial plexus anatomy of, 7, 9, 164f at costoclavicular space, 174, 176 Brachial plexus block general questions about, 299–300 history of, 23–24 infraclavicular. See Infraclavicular brachial plexus block interscalene block, 167–168 pneumothorax as complication of, 300–301 supraclavicular. See Supraclavicular brachial plexus block ultrasound-guided, 151–152 Brachialis, 190f Bradycardia, fetal, 263, 266 Bradykinin, 393–394 Brainstem hypoperfusion, 299, 301 Braun, Heinrich, 3–4 Bronchoconstriction, 299–300 BSmart™, 44, 46 Buccal nerve, 233, 235 Bunionectomy, 28 Bupivacaine cardiotoxicity of, 24–25 caudal epidural block, 123, 125 in children, 277–278 chondrotoxic effect of, 77–78 dilution of, 27–28 general questions about, 18–19, 35–36, 82, 85, 111, 117, 129, 131 hyperbaric, 264, 266 latency with, 36 liposomal, 27–29 morphine versus, for postoperative analgesia in elderly, 289, 291 peak plasma concentration of, 27, 29 pharmacokinetics of, 27, 29 repeat injections of, 27, 29 ropivacaine versus, 37–38

for sciatic nerve block using Labat’s posterior approach, 11–12 seizures and, 264, 268 Tmax of, 28–29 Bupivacaine-collagen implants, 28–29 Buprenorphine, 31–32, 320, 323 Burn patients, regional anesthesia in, 325, 327 C C7, 94, 95f C fibers, 11–12 CAIT. See Compressed air injection technique Calcitonin gene-related peptide, 393–394 Calcium channel blockers, for local anesthetic systemic toxicity, 374, 376 Calcium currents, 31–32 Camel hump sign, 246–247 Cancer anesthetic technique considerations in, 71–72 deaths related to, 71–72 epidural anesthesia and analgesia and, 109, 115 inflammation and, 66, 68 metastasis of anesthetic agents and, 65, 67 factors involved in, 71–72 inflammation and, 66, 68 intercellular adhesion molecule-1 in, 66, 68 local anesthetics and, 66, 68 opioids and, 66–67, 71–72 perioperative factors in, 71–72 in perioperative period, 65, 67 regional anesthesia and, 66, 68, 71–72 Src protein tyrosine kinase in, 66, 68 Th1 to Th2 ratio in, 65, 67 recurrence of deaths caused by, 71–72 regional anesthesia and, 71–73 Cancer surgery anesthetic technique considerations in, 71–72 perioperative care, 72–73 Cannula-over-needle device, general questions about, 50, 52 Capacitance, 53, 55 Cardiac surgery awake benefits of, 330t high thoracic epidural analgesia for, 329–330 central neuraxial blockade in, 295, 297 intercostal blockade in, 295–297 local anesthetics in, 329, 331 regional anesthesia for, 329–332 spinal analgesia for, 329, 331 transesophageal echocardiography for, 329–330 Cardiotoxicity, 24–25, 317

19/04/19 1:40 PM

Index Cardiovascular disease, regional anesthesia and, 295–298 Carpal tunnel release, 177–178 Cataract surgery, anesthesia for, 239–240 Catheter(s) for continuous femoral nerve block, 189, 192 epidural. See Epidural catheters infections associated with, 325–326 intrathecal, general questions about, 104–105 local infiltration analgesia delivery using, 77, 79 occlusive dressing over, 50, 52 on ultrasound, 147–148 Catheter site infections, 63–64 Catheter-associated bloodstream infections, 64 Catheter-based infusion, local infiltration analgesia via, 77, 79 Catheter-over-needle device general questions about, 50, 52 perineural catheter versus, 44, 46 Cauda equina nerve roots, 94, 97 Cauda equina syndrome, 17–19, 99, 101 Caudal anesthesia, 123–125 Caudal block in children, 277, 279 ultrasound-assisted, 277, 279–280, 279–280f Caudal catheter, continuous, 277, 280 Caudal local anesthetics, in children, 277–278 Celecoxib, 313–314, 393–394 Cell adhesion molecules, in inflammation, 66, 68 Central nerve blocks in children, 283–284 complication rates for, 283–284 Central nervous system demyelination of, 303, 305 mechanical trauma to, 303, 305 neuropathies, 303–304 Central neuraxial blocks, ultrasound for, 255–259 Central neuraxial regional anesthesia, and congenital heart conditions, 295–296 Central sensitization, 393–394 Cerebrospinal fluid composition of, 94, 96 general questions about, 94, 96, 137, 138, 140 leakage of, 103–104 lumbosacral, 103–104 spinal needle aspiration of, 103–104 volume of, in infants and neonates, 281 Cervical plexus anatomy of, 163, 163f–164f, 165 deep, 163, 165 questions regarding, 7, 9 sensory distribution of, 164–165 sono-anatomy of, 164f, 165 superficial, 164

Hadzic_Index_p413-428.indd 415

Cervical plexus block in emergency department, 343–344 general questions about, 343–344 for shoulder dislocation, 344–345 ultrasound-guided, 163–166 Cervical spine surgery, ketamine in, 397–398 Cesarean delivery combined spinal-epidural anesthesia for, 130, 132 high spinal anesthesia for, 264, 266 regional anesthesia before, 264, 268 spinal anesthesia for, 3, 17–18, 264, 266 visceral discomfort during, 265, 268 Charcot-Marie-Tooth neuropathies, 303–304 Charles’ law, 363, 365 Chemical arachnoiditis postepidural anesthesia, 113, 119 Chemical mediators of tissue damage/ inflammation, in peripheral nerves, 31–32 “Chemical tourniquet,” 4 Chemotherapy-induced peripheral neuropathy, 303–304 Children. See also Infants; Neonates acetaminophen in, 285–286 acute pain management in, 285–286 cerebrospinal fluid volume in, 281 chronic pain management in, 285–286 epidural anesthesia in, 277–282 gabapentin in, 285–286 greater occipital nerve block in, 283–284 local anesthetic systemic toxicity in, 373, 375 lumbar epidural block in, 278, 280–281 neuraxial anesthesia in, 277–278 pain management in, 285–286 penile block in, 283–284 peripheral nerve blocks in, 283–284, 356–357 postdural puncture headache in, 277–278 regional anesthesia in, 273–275 renal maturation in, 274 spinal anesthesia in, 277–282, 282f Chlorhexidine gluconate, 52, 64 Chloroprocaine caudal epidural block, 123, 125 general questions about, 82, 85, 87–88, 112, 117 Chorioamnionitis, 263, 266 Chronaxy, 53, 55 Chronic liver disease, 300–301 Chronic noncancer pain, opioids for, 321 Chronic obstructive pulmonary disease, thoracic epidural anesthesia in, 300–301 Chronic pain after abdominal surgery, 394–395 in children, 285–286 corticosteroids for, 393–394 opioid addiction in, 321 paravertebral block for, 394–395 after surgery, 383 after trauma, 326–327

415

Chronic renal disease, 299–300 CIPN. See Chemotherapy-induced peripheral neuropathy Circumcision analgesia for, 285–286 general questions about, 123, 125 Clavipectoral fascia, 229 Clinical pathways developmental sequence for, 59–61 meta-analysis of, 59–60 Clonidine lidocaine and, 87, 89 to local anesthetics, 112, 117 perineural, 31–32 Clopidogrel, 313–314 Coagulation, vitamin K antagonist effects on, 313–314 Coagulopathy, 263, 266, 317–318 Cobb angle, 96 Cobo, Bernabé, 3–4 Coca, 3 Cocaine history of, 3, 23–24 vasoconstriction caused by, 25 Color flow Doppler, 143–145 Combat support hospitals, 334 Combined spinal-epidural analgesia, 263, 266 Combined spinal-epidural anesthesia for cesarean delivery, 130, 132 complication rate in, 130, 132 epidural analgesia versus, for labor, 129–130 epidural anesthesia in, 129, 131 epidural volume extension with, 129, 131 failed spinal component of, 130, 132 for labor, 129–132 needle-through-needle technique, 129, 131 postdural puncture headache in, 130, 132 separate needle technique for, 129, 131 single-shot spinal versus, 129–132 for total knee arthroplasty, 130, 132 Common peroneal nerve, 9 Community hospitals, peripheral nerve blocks in children performed in, 285–286 Compartment pressure, 307 Complete blood cell, for spinal epidural hematoma, 367–368 Complex regional pain syndrome, 360–361 Complications, extra treatment added to reduce risk of, 59–60 Compound imaging, 147–148 Compressed air injection technique, 44, 46, 363, 365 Compressive lumbar disk disease, local anesthetic systemic toxicity in, 303, 305 Computed tomography myelography, for spinal epidural hematoma, 367–368 Confidence intervals, 410–412

19/04/19 1:40 PM

416

Index

Confounding, 410–411 Congenital heart conditions, central neuraxial regional anesthesia and, 295–296 Connective tissue layers of, 15–16 of peripheral nerves, 11, 13, 15–16 Consent, for regional anesthesia, 363–365 Continuous caudal catheter, 277, 280 Continuous epidural analgesia, 391–392 Continuous epidural infusions, 63–64 Continuous infusion catheter, for sciatic nerve block, 207, 209 Continuous nerve blocks, 389–390 Continuous perineural catheters, in trauma patients, 325–326 Continuous peripheral nerve blocks in austere environments, 333–335 benefits of, 49–51, 333–334 in disaster conditions, 337–338 equipment for, 49–52 general questions about, 333–335 infection risks with, 50, 52 local anesthetic solutions for, 37–39 local anesthetics for, 37–39 needle localization techniques, 50–52 needles for, 50–51 patient refusal/inability to cooperate, 389–390 site infections from, 333–335 stimulating catheters for, 50–51 Continuous regional anesthesia, history of, 3 Continuous-flow anesthetic device, 337–338, 339f Continuous-infusion catheter, for femoral nerve block, 190–191, 193 Controlled-release local anesthetics, 27–29 Conus medullaris, 123–124, 277–278 Corning, James Leonard, 3–4, 23–24, 99, 101 Coronary artery disease, regional anesthesia in, 295, 297 Coronary artery perfusion pressure, 297 Corticosteroids for acute traumatic spinal cord injury, 356, 358 chronic pain managed with, 393–394 perineural, 32 Costoclavicular space, brachial plexus at, 174, 176 Costs anesthesia-related, 379–380 in post-anesthesia care unit, 379–380 Cough reflex, 300–301 COX-1 isoenzymes, 285–286 COX 2, 65, 67 Crawford needle, in children, 277–278 Cricoid cartilage, 85 Cricoid membrane, 85 CRIES pain scale, 285–286 Critically ill patients, regional anesthesia in, 317–318 CRPS. See Complex regional pain syndrome CSF. See Cerebrospinal fluid Curbelo, Manuel, 3–4 Cushing, Harvey, 4

Hadzic_Index_p413-428.indd 416

Cutting needles, 17–18 Cystectomy, epidural anesthesia and analgesia for, 109, 114 Cytochrome polymorphisms, 393–394 D Dabigatran, 313, 315 Data collection, 410, 412 Decadron, 37–38 Deep cervical plexus, 7 Deep circumflex artery, 195–196 Deep fascial connective tissue, 352 Defense and Veterans Pain Rating Scale, 43, 45 Delayed type hypersensitivity, 89 Delirium, in elderly, 290, 292 Dendrites, 7 DepoFoam delivery system, 28 Dermatomes of femoral nerve, 190, 193 general questions about, 94, 96, 100–101 of inguinal ligament, 111, 116 surface landmarks correlated with, 116t of thumb, 111, 116 Descriptive statistics, 409, 411 Desmosomes, 138f Device pump, for local anesthetics, 38–39 Dexamethasone, 309–310 chronic pain and, 393–394 perineural, 31–32 Dexketoprofen, 87, 89 Dexmedetomidine general questions about, 240–241, 393–394 lidocaine and, 87–88 neonatal spinal anesthesia supplemented with, 283–284 properties of, 397–398 respiratory depression and, 397, 399 Dextrose 5%, 53, 55 Diabetic nephropathy, 27–28 Diabetic peripheral neuropathy, 299–300 Diabetic polyneuropathy, 303–305 Differential blockade, 111, 116 Digit reimplantation, peripheral nerve block for, 325, 327 Dilution, of liposomal bupivacaine, 27–28 Direct oral anticoagulants, 313, 315 Disaster conditions anesthesia response plan in, 337, 339 continuous peripheral nerve blocks in, 337–338 earthquakes, 337, 339 inhalational anesthesia in, 337–338 ketamine applications in, 337–338 peripheral nerve blocks in, 337–338 portable regional anesthesia kit for, 338–339 regional anesthesia in, 337–338 Distal symmetric sensorimotor polyneuropathy, 303–304 DOACs. See Direct oral anticoagulants Dogliotti, Achille, 4 Doppler mode, 143–144 Dorsal rami, 7

Double-crush phenomenon, 303–304 “Double pop” technique, for fascia iliaca block, 196–197 Draw-flow anesthetic device, 337–338, 339f Droperidol, 114, 120 Dropfoot, 189, 192 Drug abuse, 356 Dry tap, 103–104 Dura mater, 137–138 Dural fibers, 138f Dural flap, U-shaped, 17–18 Dural puncture, 17–18 Dural sac general questions about, 277–278 level of termination, 93–94 DVPRS. See Defense and Veterans Pain Rating Scale Dynamic range, 144 Dyspnea, after supraclavicular brachial plexus block, 169, 171 E Ear, regional blocks of, 157, 159 Earthquakes, patient injury patterns after, 337, 339, 339f E-cath, 46 Edema, postoperative, 359, 361 Effect size, 59–60 Elastomeric pumps, for regional anesthetic delivery, 38–39 Elbow arthroscopy of, 177–178 dislocation of, regional anesthesia for reduction of, 343–344 flexion of, 9 innervation of, 8, 10 peripheral nerves of, 181–182 ultrasound-guided nerve blocks at, 181–183 Elderly. See also Aging autonomic nervous system in, 289–291 bupivacaine in, 289, 291 delirium in, 290, 292, 387–388 demographics of, 289–290 epidural space in, 291 human aging process in, 289–291 intrathecal space in, 291 local anesthetics in, 289–290 lumbar paravertebral space in, 251, 254 minimum effective anesthetic volume in, 290 morphine in, 289, 291 pain management in, 290, 292 pain processing in, 289, 291 paravertebral blockade in, 289, 291 perioperative delirium in, 290, 292 perioperative regional anesthesia in, 289–292 population growth for, 289–290 postoperative analgesia in, 289, 291 postoperative cognitive dysfunction in, 387–388 regional anesthesia in, 289–292 ultrasound-guided supraclavicular brachial plexus block in, 289–290

19/04/19 1:40 PM

Index Electrical impedance, 53–55 Electromyography, 351, 361 Emergency department acute pain management in, 343–345 oligoanalgesia in, 344–345 regional anesthesia in, 343–345 topical anesthesia in, 343–344, 344t EMLA, 273, 275, 343, 344t Endoneurium, 7–8, 15–16, 94, 96–97 End-tidal carbon dioxide, 387–388 Enhancement artifact, 148–149 Entrapment neuropathy, 303–304 Epidural anesthesia and analgesia for abdominal surgery, 381–382 in aortic stenosis, 110, 115 back pain, 113, 119 cancer recurrence and, 109, 115 cardiovascular effects of, 111, 116 caudal, 123–124 chemical arachnoiditis, 113, 119 in children, 277–282 combined spinal-epidural anesthesia versus, 129–130 continuous, 391–392 continuous caudal, 277, 280 for cystectomy, 109, 114 effects of, 109, 115 high thoracic cardiovascular effects of, 295–296 sympatholysis caused by, 329–330 hypotension after, 264, 267 hypotension in, 109, 111, 114, 116 in hysteroscopic procedures, 109, 114 for labor and delivery, 263, 266, 382 local infiltration analgesia versus, 77–78 for lower limb vascular surgery, 109, 114 lumbar, 278, 280–281 in myasthenia gravis, 109, 114 placement of, 111, 115 postdural puncture headache, 113–114, 119 side effects of, 114, 119 speeding up, 112, 117 spinal epidural hematoma after, 367–368 testing of, 43, 46 thoracic for abdominal surgery, 109, 111, 114, 117 epidural hematoma formation associated with, 329–330 for thoracotomy surgery, 381–382 in total hip arthroplasty, 109, 114 total knee arthroplasty under, 111, 117 Epidural block caudal, 123–124 in neonates, 273–274 Epidural blood patch general questions about, 113, 119, 137, 138 for postdural puncture headache, 277–278 Epidural catheters in critically ill patients, 317 heparin and, 313, 315

Hadzic_Index_p413-428.indd 417

in immune thrombocytopenic purpura, 110, 115 infections associated with, 63–64 intrathecal placement of, 263, 266 intravascular placement of, 263, 266 in lateral decubitus position, 112, 117 in sitting position, 112, 117 test dose after placement of, in pregnancy, 265, 268–269 unsafe placement of, 110, 115 Epidural fat in children, 277–278 description of, 18–19 general questions about, 111, 116 Epidural hematoma description of, 114, 119 regional anesthesia as cause of, 355–356 spinal diagnosis of, 367–368 general questions about, 367–368 imaging of, 367–368 incidence of, 368 neurologic recovery after, 367–368 onset of, 367–368 risk factors of, 367–368 thoracic epidural anesthesia and, 329–330 Epidural infusions, 63–64 Epidural lipomatosis, 18–19 Epidural space, 4 in elderly, 291 formulas for determining, 281t local anesthetics in, 111, 117 paramedian approach to, 112, 117 sagittal view of, 96f Epidural stimulation test, 44, 47 Epidural test dose, 112, 118 Epidural volume extension, 129, 131 Epinephrine for cesarean delivery, 265, 268 general questions about, 49, 51 intravascular injection of, 123, 125, 266 as intravascular marker, 373, 375 local anesthetic and, 100,102 for local anesthetic systemic toxicity, 374, 376 test dosing of, for systolic blood pressure effects, 363–364 Epineurium, 11, 13, 15–16, 94, 96–97, 350–352 Equipment. See also specific equipment for continuous peripheral nerve blocks, 49–52 for regional anesthesia, 43–47 Erector spinae muscle, 220, 222f, 245–246 Error bars, 410, 412 Escherichia coli, 99, 101 Esmarch tourniquet, 4 Esmolol, intraoperative infusion of, 397–398 Esters anaphylaxis to, 24–25 in myasthenia gravis, 109, 114 in pregnancy, 23–24 ETCO2. See End-tidal carbon dioxide

417

Eutectic mixture of local anesthetics. See EMLA EVE. See Epidural volume extension Evoked motor response, for peripheral nerve block, 363–364 Exsanguination, 88, 90 External laryngeal nerve, 81, 83 Extrafascicular injection, in peripheral nerve block, 11, 13 Extrinsic pathway, of inflammation, 66, 68 Eye anesthetic block of, 239–241 axial length of, 239–240 F Face superficial nerve blocks of, 157–158 ultrasound-guided nerve blocks of, 157–160 Face pain scale, 43, 45 Facial artery vasospasm, 158 Factor VII, 314 Failed spinal anesthesia, 103–106 Fascia iliac, 195–196, 198 Fascia iliac block general questions about, 195–197 for hip fractures, 344 ultrasound-guided, 195–199 Fascia iliaca, 189, 192, 192f Fascial click technique, 50–51 Fascicles general questions about, 96 nerve, 7–8, 15 Fascicular bundles, 7–8 Femoral artery, 195, 197 Femoral catheters, 389–390 Femoral head, 325–326 Femoral nerve anatomy of, 326f dermatomes of, 190, 193 fascia iliaca and, 189, 192, 192f general questions about, 250, 253 innervation by, 325, 327 peripheral nerve stimulation of, 53–54 stimulation of, 189, 192 Femoral nerve block continuous, 37–38 ultrasound-guided, 189–194 Femoral nerve palsy, 220, 223 Fentanyl in children, 277–278 in combined spinal-epidural anesthesia, 129–132 intrathecal, 101 natural killer cell activity affected by, 67 patient-controlled analgesia use of, 387–388 Fetus bradycardia in, 263, 266 placental transfer of local anesthetic to, 263, 265 FICB. See Fascia iliac compartment block Filum terminale, 94, 97, 123–124 Finger pulse oximeter, 338–339

19/04/19 1:40 PM

418

Index

Fluoroscopy, 50, 52 Foot foreign objects in, anesthesia for removal of, 343–344 tibial nerve innervation of, 215–216 Forced expiratory volume in one second, 329, 331 Forearm fractures, 327 Forefoot surgery, nerves omitted in, 215–216 Foreign objects in foot, anesthesia for removal of, 343–344 Fractures forearm, 327 rib, 325, 327 tibial, acute compartment syndrome secondary to, 325–327 Frequency, 143–145 Frog eye sign, 246–247 Functional outcome after surgery, 383 Functional residual capacity, 329, 331 G Gabapentin in children, 285–286 dosing of, 393–394 Gabapentinoids, 391–392 Gag reflex, in awake nasal intubation, 81, 84 Ganglion cyst excision, 87, 89 Gartner, Joseph, 24 Gay-Lussac’s law, 363, 365 GBS. See Guillain-Barré syndrome General anesthesia in austere environments, 333–334 autonomic dysreflexia and, 304, 306 circumcision under, 123, 125 host immunity affected by, 72 induction time for, 379 local and regional anesthesia versus, 59–60 neuraxial anesthesia versus, for noncardiac surgery, 381–382 opioids for, 66–67 postoperative cognitive dysfunction and, 383 regional anesthesia versus, 381–382 thoracic epidural anesthesia and, 329, 331 Genitofemoral nerve, 8–9 Genitourinary surgery, caudal block in, 123, 125 Geriatrics. See Elderly Globe perforation, 239–241 Glossopharyngeal nerve anatomy of, 81–82 gag reflex triggered by, 81, 84 Glossopharyngeal nerve block external approach to, 82, 85 peristyloid, 81, 83 Glucose, perioperative homeostasis of, 299–301 Gluteus maximus, 207, 209 Gow-Gates technique, 233–234 Great saphenous vein stripping, 205–206 Greater occipital nerve block in children, 283–284 general questions about, 157, 159

Hadzic_Index_p413-428.indd 418

Greater palatine nerve block, 233–234 Greene needle, 18–19 Guillain-Barré syndrome, 303–304 H Halothane, natural killer cell activity affected by, 65, 67 Halsted, William, 3 Hand(s) nerves of, 181–182 paresthesia of, 264, 266 “Hand feel,” for injection pressure monitoring, 363–364 Hanging drop technique, 112, 117 Hawthorne effect, 411 Head, nerve supply to, 165f Headache benign, 137, 139 postdural puncture. See Postdural puncture headache Hearing loss, after spinal anesthesia, 99, 101 Heart rate, after high thoracic epidural analgesia, 329–330 Heeling, 151–152 HELLP syndrome, 266 Hematoma, 233–234 epidural, regional anesthesia as cause of, 355–356 nerve injury caused by, 355–356 neuraxial, 313–315 Hemidiaphragmatic paresis, 169, 171 Hemorrhoidectomy, liposomal bupivacaine for, 27–28 Henry’s law, 363, 365 Heparin, 313–315 Hepatic coagulopathy general questions about, 300–301 neuraxial anesthesia and, 299, 301 Hepatic dysfunction, liposomal bupivacaine considerations in, 27–28 Hereditary neuropathy with liability to pressure palsy, 303–304 Hereditary peripheral neuropathies, 303–304 Hernia repair, liposomal bupivacaine for, 27–28 Heroin half-life of, 321 withdrawal from, 320f, 321 High neuraxial anesthesia apnea after, 299, 301 general questions about, 266, 268 High spinal anesthesia, for cesarean delivery, 264, 266 High thoracic epidural analgesia awake cardiac surgery using, 329–330 heart rate after, 329–330 sympatholysis caused by, 329–331 High thoracic epidural anesthesia, 295–296 Hildebrandt, August, 3 Hip fractures of fascia iliac block for, 344 neuraxial anesthesia for, 325–326 innervation of, 8, 10 Hip pain, fascia iliaca block for, 196, 198

Histamine, 393–394 HNPP. See Hereditary neuropathy with liability to pressure palsy Horse head sign, 246–247 Human aging process, 289–291 Humanitarian relief operations, anesthesia for, 337–339 Hydralazine, 264, 268 Hydrocodone, 285–286 Hydrocortisone, 393–394 chronic pain and, 393–394 Hydrodissection, 151–153 Hyoid bone, 81–83, 85 Hyperalgesia, 310 naloxone as cause of, 397–398 opioid-induced, 319, 321, 322f Hyperbaric local anesthetic solutions, 103–105 Hyperbaric therapy, 81, 84 Hyperkalemia, in chronic renal disease, 299–300 Hypertension, 295–296 intraoperative, 359, 361 Hypotension, 65, 67 in epidural anesthesia and analgesia, 109, 111, 114, 116 lidocaine as cause of, 87, 89 in preeclampsia, 264, 267 after spinal anesthesia, 99, 101 Hypothermia, natural killer cell activity affected by, 65, 67 Hypovolemia, 65, 67, 263, 266 Hysteroscopic procedures, epidural anesthesia and analgesia in, 109, 114 I Ibuprofen, acetaminophen and, 393–394 ICAM-1. See Intercellular adhesion molecule-1 Ileus, postoperative, epidural anesthesia effects on, 381–382 Iliac crests, 93–94, 95f Iliopsoas muscle, 198 Immobilization, postoperative, 359, 361 Immune modulation, perioperative period and, 65, 67 Immune system opioid effects on, 66–67 pain effects on, 381–382 Immune thrombocytopenic purpura, epidural catheter in, 110, 115 Immunologic responses, to local anesthetics, 24–25 Impedance, 53–55 Incisive nerve block, 233–234 Indwelling sciatic perineural catheter, 208, 210 Infants. See also Children; Neonates cerebrospinal fluid volume in, 281 infraorbital nerve block in, 283–284 spinal anesthesia in, 278, 281–282, 283–284 Infection(s) catheter site, 63–64 catheter-related, 325–326

19/04/19 1:40 PM

Index in continuous peripheral nerve blocks, 50, 52 epidural catheter-associated, 63–64 in regional anesthesia, 63–64 Infection control, 63–64 Inferential statistics, 409, 411 Inferior alveolar block, local anesthetic systemic toxicity after, 373, 375 Inferior angle of scapula, 93–94, 95f, 110, 115 Inflammation cancer, 66, 68 cell adhesion molecules in, 66, 68 nerve injury and, 355–356 Inflammatory lumbar plexopathy, 361 Inflammatory neuropathies description of, 350, 353 postsurgical, 355, 357, 359–360 Infraclavicular brachial plexus block for elbow dislocation, 343–344 indications for, 176 level of, 173, 175 nerve stimulator for, 173, 175 for open reduction and internal fixation of elbow, 173, 175 for shoulder dislocation, 344–345 ultrasound-guided, 173–176 Infrainguinal approach, for fascia iliaca block, 198 Infraorbital nerve block, in infants, 283–284 Infrared thermal imaging, 43, 45 Infusion pump, 37–38 Inguinal ligament, dermatomes of, 111, 116 Inhalational anesthesia, in disaster conditions, 337–338 Injection pressure BSmart™ for, 44, 46 fascia iliac compartment block, 195, 197 for lumbar plexus block, 250, 253 measurement of, 44, 46 monitoring of, 11, 13, 363–365 opening. See Opening injection pressure in peripheral nerve block, 11, 13 In-line manometry, for injection pressure monitoring, 363–364 In-line pressure manometer, 44, 46 Inner epineurium, 11, 13 In-plane approach, 167–168 Intensive care unit catheter management in, 317–318 regional anesthesia in, 317–318 Intercellular adhesion molecule-1, 66, 68 Intercostal blockade, in cardiac surgery, 295–297 Intercostobrachial nerve, 170 Intercostobrachialis nerve, 229 Interfascicular epineurium, 7–8, 15–16 Interlaminar space, 256, 258, 259f Internal laryngeal nerve, 81, 83 Internal oblique muscle, 220, 222f Interscalene brachial plexus block nerve injury rates with, 11, 13 pulmonary physiology affected by, 290, 292

Hadzic_Index_p413-428.indd 419

for shoulder dislocation, 344–345 shoulder surgery uses of, 299–300 ultrasound-guided, 167–168 Interscalene catheter, continuous, 37–38 Interscalene nerve blocks intravenous sedation versus, for shoulder dislocation, 325, 327 resident autonomy in performing, 403 ultrasound-guided, 151–152 Interspinous ligament, 94–95, 97, 112, 117 Intra-arterial injections, 239–240 Intra-articular injections, for postoperative analgesia, 77–78 Intrafascicular injection, in peripheral nerve block, 11 Intralipid 20%, 43, 45, 49–51, 81, 84 Intraneural injection, nerve injury and, 11, 13–14 Intraneural plexus, 15 Intrapulpal injection, 234–235 Intrascalene brachial plexus block for elbow dislocation, 343–344 ultrasound-guided, 44 Intraseptal injection, 234–235 Intrathecal catheters, general questions about, 104–105 Intrathecal ligament, 94, 97Intrathecal space, in elderly, 291 Intrathoracic surgery, regional anesthesia for, 381–382 Intravenous infusions, lidocaine, 397–398 Intravenous ketamine, 397–398 Intravenous patient-controlled analgesia, 387–388, 391–392 Intravenous regional anesthesia, 87–90 Intravenous sedation, interscalene nerve block versus, for shoulder dislocation, 325, 327 Intrinsic pathway, of inflammation, 66, 68 Ischemic heart disease, 297 Isoflurane, natural killer cell activity affected by, 65, 67 IV-PCA. See Intravenous patient-controlled analgesia IVRA. See Intravenous regional anesthesia J Joint innervation, 8, 10 Joint replacement surgery, regional anesthesia for, 383 K Kennedy, Foster, 4 Ketamine advantages of, 337–338 in cervical spine surgery, 397–398 in disaster conditions, 337–338 general questions about, 65, 67, 343–344 intravenous, 397–398 in lumbar spine surgery, 397–398 morphine and, 398 opioid potentiation by, 391–392 postoperative pain managed with, 393–394

419

propofol versus, 337–338 respiratory depression caused by, 397–399 Knee innervation of, 8, 10 total arthroplasty of. See Total knee arthroplasty Koller, Carl, 4, 23–24 Kyphoscoliosis, 19 L L3-L4 roots, of lumbar plexus, 245–246 L4-5, epidural fat, 18–19 L5–S1 in elderly, 290–291 general questions about, 256, 258 Labat, Gaston, 4, 43–44 Labat’s posterior approach to sciatic nerve block, 11–12 Labor. See also Pregnancy combined spinal analgesia for, 263, 266 combined spinal-epidural anesthesia for, 129–132 dermatome levels of, 264, 267 epidural analgesia in, 263, 266 epidural anesthesia for, 382 opioids for, 382 pain in, 263, 265 second stage of, 263, 265 visceral pain in, 264, 267 Laminectomy, lumbar pain syndrome after, 124–125 Larynx innervation, 81–82, 82f LAST. See Local anesthetic systemic toxicity Lateral approach, to quadratus lumborum block, 219, 221f, 222 Lateral bladder tumor excision, 204 Lateral decubitus position, epidural catheter in, 112, 117 Lateral femoral cutaneous nerve anatomy of, 201 description of, 196, 198, 250, 253 femoral head innervation by, 325–326 ultrasound-guided block of, 201–202 Lateral malleolus, 215–216 Lateral meningo-vertebral ligament, 17, 19 Lateral pectoral nerve, 229 Latissimus dorsi muscle, 220, 222f Left ventricular ejection fraction, 295, 297 Left ventricular hypertrophy, 295–296 LET. See Lidocaine, epinephrine, tetracaine Levobupivacaine in children, 283–284 general questions about, 103–105, 111, 117, 274 LIA. See Local infiltration analgesia Lidocaine cardiotoxicity of, 24–25 caudal epidural block, 123, 125 dexketoprofen added to, 87, 89 dexmedetomidine added to, 87–88 general questions about, 17, 82, 85 history of, 3–4 hypotension caused by, 87, 89 intrathecal administration of, 266

19/04/19 1:40 PM

420

Index

Lidocaine (Cont.): intravenous infusion of, 397–398 intravenous regional anesthesia using, 87–88 liposomal bupivacaine dilution with, 27–28 for local anesthetic systemic toxicity, 374, 376 recurrent laryngeal nerve block using, 81, 83 Lidocaine, epinephrine, tetracaine, 343, 344t Ligamentum flavum in cervical region, 93, 95 definition of, 97 general questions about, 17, 19, 111, 115, 116t, 129, 131, 291 in lumbar region, 93, 95 morphologic variations of, 95f supraspinous ligament versus, 93, 95 Limb occlusion pressure, 359–361 Lipid emulsion for local anesthetic systemic toxicity, 299–300, 373–374 side effects of, 374 Lipid resuscitation therapy, 373–374 Lipid solubility, local anesthetic potency affected by, 23–24, 35–36 Liposomal bupivacaine, 27–29 Liposomes, 27–28 Liver disease, chronic, 300–301 LMWH. See Low-molecular-weight heparin LMX-4, 273, 275 Local and regional anesthesia, general anesthesia versus, 59–60 Local anesthesia history of, 3–4 for ophthalmic surgery, 239–241 Local anesthetic(s) advancements in, 3 advancing age for, 112, 117 allergic reactions to, 24–25 anaphylaxis with, 24–25 axons and, 15–16 basal infusion rate of, 37–38 cancer metastasis and, 66, 68 in cardiac surgery, 329, 331 caudal, in children, 277–278 caudal epidural block, 123, 125 in children, 273–274, 277–278 clonidine to, 112, 117 continuous, 37–38 controlled-release, 27–29 deposition of, optimization techniques for, 152–153 device pump for, 38–39 diffusion of, over nerve sheath, 24 duration of effect, 3 effectiveness of, 24 in elderly, 289–290 encapsulation of, 27–28 in epidural space, 111, 117 epinephrine and, 100,102 in eye blocks, 239–241 for femoral nerve block, 191, 193–194 history of, 3–4, 23–24

Hadzic_Index_p413-428.indd 420

immunologic responses to, 24–25 intraneural injections of, 11, 13–14 lipid solubility effects on potency of, 23–24, 35–36 long-acting, 49, 51 minimum effective anesthetic volume, in elderly, 290 mixing of, 35–36 myotoxicity, 317 in neonates, 283–284 nerve fibers blocked by, 11–12 neurotoxicity of, 11, 13–14, 24–25, 317 for pectoralis nerve block, 226–227, 229 perineural injection/infusion of, 16, 389–390 pharmacology of, 23–25 pKa value of, 23–24, 35–36 placental transfer of, to fetus, 263, 265 potency of, 23–24 in pregnancy, 23–25 properties of, 35–36 repeat injections of, 27, 29 rostral spread of, 100,102 seizures caused by, 24–25 short-acting, 49, 51 side effects of, 273–274 sodium bicarbonate added to, 112–113, 117–118 speed of onset, 23–24 spread of, 147–148 topical, in children, 273, 275 toxicity of. See also Local anesthetic systemic toxicity cardiotoxicity, 24–25 in children, 273–275 intralipid 20% for, 43, 45, 49–51, 81, 84 after intravenous regional anesthesia, 88–90 lipid emulsion for, 299–300 uptake of, 100–101 vasoconstrictive properties of, 82, 85 Local anesthetic mixtures for peripheral nerve blocks, 35–36 for postoperative analgesia, 77–78 Local anesthetic solutions accidental subdural injection of, 103–104 for continuous peripheral nerve blocks, 37–39 epinephrine added to, 49, 51 hyperbaric, 103–105 subdural injection of, 103–104 Local anesthetic systemic toxicity cardiac collapse in, 267 cardiac toxicity during, 373, 375 cardiovascular rescue during, 374, 376 in children, 277–278, 373, 375 clinical presentation of, 374–375 description of, 24–25, 27, 35–36 incidence of, 373–375 lipid emulsion for, 373–374 management of, 374, 376 patient-related risk factors for, 373–374 prevention of, 373, 375 seizures caused by, 267

in spinal stenosis, 303, 305 symptoms of, 113, 118–119, 309–310 treatment of, 309–310, 373–374 ultrasound guidance effects on, 363–364 Local infiltration analgesia bupivacaine for, 77–78 catheter delivery of, 77, 79 catheter-based infusion for, 77, 79 characteristics of, 77–78 epidural analgesia versus, 77–78 peripheral nerve block versus, 77–78 for total hip arthroplasty, 77–79 for total knee arthroplasty, 77–79 Long thoracic nerve, 229 Long-acting local anesthetics, 49, 51 LOP. See Limb occlusion pressure Loss of resistance to air, 112, 117 Lower extremity, cutaneous innervation of, 8–9 Lower limb vascular surgery, epidural anesthesia and analgesia for, 109, 114 Low-molecular-weight heparin, 313–315 LRT. See Lipid resuscitation therapy Lumbar disk disease, local anesthetic systemic toxicity in, 303, 305 Lumbar epidural analgesia, in elderly, 290, 292 Lumbar epidural block, in children, 278, 280–281 Lumbar neuraxial anesthesia, pulmonary changes caused by, 299–300 Lumbar pain syndrome after laminectomy, 124–125 Lumbar paravertebral space anatomy of, 250, 253 in elderly, 251, 254 sonography of, 245–247, 249–254 vascular supply of, 251, 253 Lumbar plexus, 220, 222f anatomy of, 245–246, 249–252 electrical nerve stimulation of, 247 L3-L4 roots of, 245–246 nerves of, 249, 251 scanning of, 250, 253 ultrasound imaging of, 250, 253 Lumbar plexus block, ultrasound-guided complications of, 249, 252 considerations for, 249–254 indications for, 249, 251 injection pressure in, 250, 253 landmarks for, 245–246 low-frequency curved array ultrasound probe for, 249, 252 muscle twitches in, 249, 252 paramedian oblique scan at transverse process, 249, 252 parasagittal longitudinal approach, 246–247 posterior approach for, 250, 253 “shamrock” method for, 245–247, 249, 251–253 transverse in-plane approach to, 245, 247

19/04/19 1:40 PM

Index Lumbar radiculopathy, 124–125 Lumbar spine surgery, ketamine in, 397–398 Lumbar vertebral canal, epidural fat distribution in, 18–19 Lumbosacral junction, 256, 258 M MAC. See Monitored anesthesia care Magnesium sulfate, for seizure prevention, 264, 268 Malignant hyperthermia, epidural anesthesia and analgesia in, 109, 114–115 Mandibular nerve block, ultrasound-guided, 158–159 Mastectomy, pectoralis nerve block for, 225–226 Maxillary artery, accidental puncture of, 233–234 Maxillary first molar, 233–235 Maxillary incisors, regional anesthesia of, 233–234 Maxillary nerve block, ultrasound-guided, 158–159 Maxillary second molar, 233–234 Maxillofacial regional anesthesia. See Oral and maxillofacial regional anesthesia MEAC. See Minimum effective analgesic concentration MEAV. See Minimum effective anesthetic volume Medial malleolus, 215–216 Medial pectoral nerve, 229 Median nerve general questions about, 177–179 muscles innervated by, 8–9 neurostimulation of, 185 recognition of, 185 ultrasound-guided block of, 182–183 wrist block, 185–186 Medical teaching missions, 333–334 Meningitis description of, 114, 119 after spinal anesthesia, 99, 101 Meningo-vertebral ligament, 17, 19 Mental nerve block, 233, 235 Meperidine, 130, 132 Mepivacaine general questions about, 36, 82, 85 for sciatic nerve block using Labat’s posterior approach, 11–12 Mesoneurium, 11, 13 Meta-analysis, of clinical pathways, 59–60 Methadone general questions about, 320, 322–323 half-life of, 321 withdrawal from, 320f, 321 Methemoglobin, 88–89 Methemoglobinemia, 83, 87–89 Methylene blue, 81, 83–84, 87–88 Methylprednisolone for acute traumatic spinal cord injury, 356, 358 chronic pain managed with, 393–394

Hadzic_Index_p413-428.indd 421

Midazolam, 65, 67 propofol and, 240–241 for seizures, 25 Middle superior alveolar nerve block, 233–234 Middle thoracolumbar fascia, 219, 221f Miller Fisher test, 46 Minimally invasive direct coronary artery bypass surgery, 296 Minimum effective analgesic concentration, 391–392 Minimum effective anesthetic volume, in elderly, 290 Minimum local anesthetic dose, 129, 131 Mitral regurgitation, 295–296 Monitored anesthesia care, 304, 306 Monitoring injection pressure, 11, 13, 363–365 opioid, 387–388 for regional anesthesia, 363–365 Monro-Kellie hypothesis, 140 Morphine bupivacaine versus, for postoperative analgesia in elderly, 289, 291 in elderly, 289, 291 general questions about, 130, 132, 387–388 half-life of, 321 immunosuppression induced by, 66–67 ketamine and, 398 Motor fibers, chronaxy of, 53, 55 Motor response, 53–55 Motor stimulation, 151, 153 MTHFR gene, 394 Multimodal analgesia, 393–394 in ambulatory surgery, 309–310 Multiplate, 313–314 Multiple sclerosis general questions about, 303–305 total knee arthroplasty in, regional anesthesia for, 303, 305 Multivesicular liposome, 27–28 m-opioid receptors, 66–67 Muscular twitches, peripheral nerve stimulator detection of, 53 Musculocutaneous nerve, 177, 179 Myasthenia gravis, epidural anesthesia and analgesia in, 109, 114 Myelinated fibers, sodium channels in, 23–24 Myotoxicity, of local anesthetics, 317 N Nail beds, 186 Nalbuphine, 114, 120 Naloxone, 322, 397–398 respiratory depression and, 397, 399 Naltrexone, 322 Nasopalatine nerve block, 233–234 National Surgical Quality Improvement Program, 381–382 Natural killer cells acute pain and, 72 cancer metastasis and, 65, 67 morphine effects on, 66–67 opioid effects on, 72 pain effects on, 381–382

421

Nausea and vomiting, intraoperative, 99, 101 Needle(s) for continuous peripheral nerve blocks, 50–51 cutting, 17–18 localization techniques for, 50–52 obstruction of, 103–104 pencil-point, 17–18, 44 peripheral nerve injury and, 351 postdural puncture headache risks and, 43–44 for superficial cervical plexus block, 165–166 Needle gauge, postdural puncture headache and, 137–138 Needle trauma, 11, 13 Needle–nerve contact evoked response motor value indicating, 363–364 general questions about, 363, 365 Needle-through-needle technique, 129, 131 Negative studies, 409, 411 Neonates. See also Children; Infants cerebrospinal fluid volume in, 281 circumcision in, analgesia for, 285–286 epidural block in, 273–274 local anesthetics in, 283–284 neuraxial anesthesia in, 278 pain management in, 285–286 pain structure in, 285–286 regional anesthesia in, 273–275 spinal anesthesia in, with dexmedetomidine, 283–284 Nerve blocks. See specific block Nerve conduction, theories involved in, 4 Nerve fascicles, 7–8, 15 Nerve fibers, local anesthetic block of, 11–12 Nerve stimulation for femoral nerve block, 191, 193 peripheral algorithm for, 54f of femoral nerve, 53–54 popliteal sciatic nerve block use of, 213–214 Nerve stimulation threshold, 44, 46 Nerve stimulators. See Peripheral nerve stimulators Nerve-mapping pens, 53, 55 Neuraxial anesthesia in anticoagulant patients, 313–315 antiplatelet agent cessation before, 313–314 for autonomic dysreflexia prevention in spinal cord injuries, 304–306 benefits of, 310 in children, 277–278 general anesthesia versus, for noncardiac surgery, 381–382 hematoma formation after, 313–314 hepatic coagulopathy and, 299, 301 high, 266, 268 apnea after, 299, 301 for hip fracture, 325–326 outcomes improved using, 310 postpolio syndrome and, 303, 305

19/04/19 1:40 PM

422

Index

Neuraxial anesthesia (Cont.): in pregnancy, 24, 263–264, 266, 268 reasonability assessments for, 109, 115 spinal cord dysfunction and, 355–356 Neuraxial blockade, lumbosacral cerebrospinal fluid volume effects on, 103–104 Neuraxial blocks, 63–64 Neuraxial hematoma, 313–315 spinal analgesia and, 329, 331 Neuraxial regional anesthesia, congenital heart conditions and, 295–296 Neurologic disease, regional anesthesia in, 303–306 Neuromuscular paralysis, 303–304 Neuron, 7–8 Neuropathic pain, 391–392 Neuropraxia, 11–12 Neurotmesis, 11–12 Neurotoxicity, of local anesthetics, 24–25, 317 Nitrous oxide sedation, general questions about, 343–344 NMDA antagonists description of, 343–344 general questions about, 397–398 Nociceptive pain, in peripheral nerves, 31–32 Nociceptor sensitization, 393–394 Nodes of Ranvier, 24, 53, 55 Nonsteroidal anti-inflammatory drugs acetaminophen and, 393–394 description of, 286 Nose, regional blocks of, 157–159 NRS. See Numeric rating scale Nuclear factor-κB, 66, 68 Null hypothesis, 409, 411 Numeric rating scale, 43, 45 O Obesity, regional anesthesia in, 299, 301 Obstetric patients neuraxial hematoma formation in, 313–314 spinal anesthesia in, 102 Obturator nerve, 250, 253, 326f anterior branch of, 8–9 fascia iliac compartment block and, 195, 197 Obturator nerve block, ultrasound-guided, 203 Occlusive dressings, 50, 52 Oculocardiac reflex, 239–240 OIH. See Opioid-induced hyperalgesia Oligoanalgesia, 344–345 Open reduction and internal fixation of elbow, infraclavicular brachial plexus block for, 173, 175 Opening injection pressure determinants of, 44–46 measurement of, 44, 46 upper limit of, for safe peripheral nerve block, 363–364 Ophthalmic nerve, branches of, 157–158

Hadzic_Index_p413-428.indd 422

Ophthalmic surgery, local and regional anesthesia for, 239–241 Opioid(s) in austere environments, 333, 335 cancer metastasis and, 66–67, 71–72 in children, 277–278 for chronic noncancer pain, 321 deaths caused by, 321 for general anesthesia, 66–67 general questions about, 333, 335 half-life of, 319–321 hyperalgesia caused by, 310 immune system affected by, 66–67 intrathecal in cardiac surgery, 329, 331 general questions about, 264, 267 ketamine potentiation of, 391–392 for labor and delivery, 382 monitoring of, 387–388 naloxone effects on, 397–398 natural killer cell inhibition by, 72 perineural administration of, 31–32 prescription sales of, 321 pruritus associated with, 99, 101 pruritus caused by, 114, 120 rotation of, 320, 323 side effects of, 264, 267 site of action for, 387–388 switching of, 320, 323 tolerance to, 319–322 withdrawal from duration of, 320f symptoms of, 320 Opioid receptors, 321 Opioid-dependent patients acute pain management in, 319–323 general questions about, 319–321 Opioid-induced hyperalgesia, 319, 321, 322f Opponens pollicis muscle, 186 Oral and maxillofacial regional anesthesia, 233–235 Orbit, 239–240 Osteoporosis, 368 Outpatient surgery, peripheral nerve blocks for, 309–310 Oxycodone, 285–286 Oxygen concentrators, in disaster conditions, 337–338 Oxygen cylinders, in disaster conditions, 337–338 P PACSLAC. See Pain Assessment Checklist for Seniors with Limited Ability to Communicate Pain acute. See Acute pain assessment of, 43, 45 chronic. See Chronic pain chronic noncancer, opioids for, 321 immune system effects of, 381–382 management of. See Pain management neuropathic, 391–392 phantom limb, 383, 398 in pregnancy, 113, 118, 267f

Pain Assessment Checklist for Seniors with Limited Ability to Communicate, 43, 45 Pain fibers chronaxy of, 53, 55 general questions about, 53, 55 Pain management in children, 285–286 in elderly, 290, 292 fascia iliaca block for, 196, 198 femoral nerve block for, 191, 193 after lumbar plexus block, 250, 252 multimodal approach, 323 saphenous nerve block for, after total knee arthroplasty, 205–206 theories involved in, 4 Pain processing, in elderly, 289, 291 Pain rating scales in children, 285–286 CRIES, 285–286 description of, 43, 45 Palatoglossal arch, 81, 83 Palatopharyngeal arch, 81–83, 85 Paracervical blocks, 263, 266 Paralytic ileus, 111, 117 Paramedian approach, 100–101, 112, 117 Paramedian oblique scan at transverse process, 249, 252 Paramedian sagittal oblique scan, 255–259, 257f Paramedian transverse oblique scan, 250–251, 253–254 Paraneural connective tissue, 352 Paraneurium, 8, 15–16 Parasagittal longitudinal approach, to ultrasound-guided lumbar plexus block, 246–247 Paravertebral nerve block chronic pain managed with, 394–395 in elderly, 289, 291 general questions about, 330–331 needle insertion site for, 330–331 for rib fractures, 343–344 Paravertebral space, 330–331 Pathways anesthetic, 59–61 clinical, 59–61 complication occurrence while following, 59–60 for regional anesthesia, 59–61 Patient positioning, for popliteal sciatic nerve block, 213–214 Patient-controlled analgesia fentanyl in, 387–388 intravenous, 320, 323, 387–388 after total knee arthroplasty, in elderly, 290–291 Patient-controlled boluses, 37–38 Patient-controlled regional anesthesia, 318 PCA. See Patient-controlled analgesia PCRA. See Patient-controlled regional anesthesia PDPH. See Postdural puncture headache Pectoralis nerve block complications of, 225–226

19/04/19 1:40 PM

Index serratus plane block versus, 229 studies of, 228t thoracic paravertebral block versus, 225–226 Pediatric patients. See Children; Infants; Neonates Pencil-point needles, 17–18, 44 Penile block, in children, 283–284 Percutaneous epidural neuroplasty, 124–125 Peribulbar block, 239–240 Perineum analgesia, 100,102 Perineural catheter, catheter-over-needle versus, 44, 46 Perineural infusions indications for, 389 risks of, 389–390 Perineurium, 7–8, 11, 13, 15–16, 94, 96–97, 351–352 Periodontal ligament injection, 234–235 Perioperative analgesia, fascia iliaca block for, 195–196 Perioperative delirium, in elderly, 290, 292 Perioperative period cancer metastasis in, 65, 67 immune modulation and, 65, 67 Peripheral catheters, general questions about, 391–392 Peripheral nerve(s) age-related changes in, 12, 14 anatomy of, 7–8, 349, 351–352 connective tissue of, 11, 15–16 excitatory influences on, 32f identification of, during block, 11–13 inhibitory influences on, 32f injury to, 11, 13–14 intrafascicular injection into, 12 nociceptive pain in, 31–32 pia mater and, 94, 96–97 structure of, 7–8, 15–16 Peripheral nerve blocks adjuvant agents for perineural use in, 31–32 for ambulatory surgery, 309–310 in anticoagulant patients, 313–315 in austere environmental medicine, 333–335 in children, 283–284, 285–286, 356–357 commonly performed type of, 403, 404f in community hospitals, 285–286 complications of, 14, 283–284 compressed air injection technique for, 44, 46 continuous, 37–39 deep, 315 digit reimplantation use of, 325, 327 in disaster conditions, 337–338 general questions about, 15–16, 391–392 high injection pressure during, 11 images of, 364–365 intrafascicular injection in, 11 intraneural contact in, evoked motor response value for, 363–364 local anesthetic mixtures for, 35–36 local infiltration analgesia versus, 77–78

Hadzic_Index_p413-428.indd 423

needle–nerve contact in, evoked motor response value for, 363–364 needle–nerve proximity, 350, 353 nerve identification during, 11–13 nerve injury after epidemiology of, 349–350 etiology of, 349, 351 incidence of, 349–350 needle characteristics’ effect on, 349, 351 pathophysiology of, 350, 352 preexisting disease and, 350, 353 prevention of, 349, 351 nerve injury during, 11, 45 neurostimulation for placement of, 363–364 for outpatient surgery, 309–310 peripheral nerve lesions in, 11–12 presurgical operating room time for, 379 for shoulder dislocation, 344–345 superficial, 315 ultrasound-guided, 15–16, 35–36, 43–46 video evidence of, 364–365 Peripheral nerve catheters, in critically ill patients, 317–318 Peripheral nerve complexes, 351–352 Peripheral nerve hematoma diagnosis of, 367–369 management of, 367–369 Peripheral nerve injury body mass index and, 356–357 epidemiology of, 349–350 etiology of, 349, 351 general questions about, 355, 357 incidence of, 349–350 needle characteristics’ effect on, 349, 351 pathophysiology of, 350, 352 patient-related risk factor associated with, 356–357 perioperative, 359–361 preexisting disease and, 350, 353 prevention of, 349, 351 risk factors for, 356–357 tourniquet-related, 359–361 Peripheral nerve lesions, 11–12 Peripheral nerve stimulation of femoral nerve, 53–54 ultrasound guidance versus, 403 Peripheral nerve stimulators amplitude range of, 55 current range of, 54–55 general questions about, 53–55 for infraclavicular brachial plexus block, 173, 175 malfunctioning, 251, 254 muscular twitch detection using, 53 needle tip polarity with, 44, 47 nerve identification using, 11–12 popliteal sciatic nerve block use of, 213–214 settings for, 53–55 Peripheral nervous system analgesic adjuvants in, 31–32 general questions about, 351–352

423

Peripheral neuropathies, hereditary, 303–304 Peristyloid approach, 81, 83 Peroneal nerve, superficial, 215–216 Persistent postoperative pain, 393–395 Pethidine, 100,102 PFA. See Platelet function analyzer Phantom limb pain, 383, 398 Pharmacology, of local anesthetics, 23–25 Pharyngeal nerve, 81, 83 Phenobarbital, 264, 268 Phrenic nerve, 266 anatomy of, 7 general questions about, 167–168 palsy of, 171 Phrenic nerve block diaphragmatic paralysis caused by, 299–300 general questions about, 299–300 Physical dependence, 321 Pia mater, peripheral nerves and, 94, 97 pKa, 23–24, 35–36 Placenta, local anesthetic transfer to fetus via, 263, 265 Plasma free metanephrines, 393–394 Platelet function analyzer, 313–314 PMTOS. See Paramedian transverse oblique scan Pneumatic tourniquets nerve injury caused by, 359, 361 tissue damage caused by, 359, 361 Pneumocephalus, 112, 117 Pneumothorax brachial plexus block as cause of, 300–301 general questions about, 169, 171 PNI. See Peripheral nerve injury POCD. See Postoperative cognitive dysfunction Polyneuropathies, 303–304 Popliteal approach, to sciatic nerve block, 208, 211 Popliteal nerve block catheter, 44, 46 Popliteal sciatic block, ultrasound-guided, 213–214 Portable regional anesthesia kit, for disasters, 338–339 Positive studies, 409, 411 Post-anesthesia care unit bypassing of, 379–380 costs in, 379–380 regional anesthesia effects on, 379–380 Postdural puncture headache in children, 277–278 in combined spinal-epidural anesthesia, 130, 132 diagnosis of, 137, 139 duration of, 137, 138 epidural blood patch for, 277–278 features of, 137, 138 general questions about, 17–19, 43–44, 99, 102, 113–114, 119, 137–140, 138f needle gauge and, 137–138 pain associated with, 137, 140 risk for, 137–138

19/04/19 1:40 PM

424

Index

Posterior approach for lumbar plexus block, 250, 253 for quadratus lumborum block, 219, 221f, 222 for sciatic nerve block, 207–209, 208f Posterior cervical triangle, 9 Posterior epidural fat, 18–19 Posterior femoral cutaneous nerve, 9 Posterior femorocutaneous nerve, 208, 210–211 Posterior longitudinal ligaments, 97 Posterior meningo-vertebral ligament, 17, 19 Posterior superior alveolar nerve block, 233–234 Posterior tibial artery, 215–216 Postoperative analgesia liposomal bupivacaine for, 28–29 morphine versus bupivacaine for, in elderly, 289, 291 Postoperative cognitive dysfunction, 383, 387–388 Postoperative ileus, epidural anesthesia effects on, 381–382 Postoperative nausea, 78 Postoperative pain dexketoprofen added to lidocaine for, 87, 89 ketamine for, 393–394 management of, 393–395 persistent, management of, 393–395 Postoperative period edema in, 359, 361 evaluation in, 360–361 immobilization in, 359, 361 Postpolio syndrome, 303–305 Postsurgical inflammatory neuropathies, 303–304, 355, 357, 359–360 Prasugrel, 313–314 Prednisone, chronic pain and, 393–394 Preeclampsia, hypotension in, 264, 267 Pregnancy. See also Labor anesthetic requirements in, 263, 265 epidural catheter placement in, test dose for, 265, 268–269 local anesthetics in, 23–25 maternal aortic regurgitation, 296–298 neuraxial anesthesia in, 263–264, 266, 268 pain in, 113, 118 pain pathways in, 267f physiologic changes of, 263, 265 regional anesthesia in, 296–298 spinal anesthesia in, 264, 268 Premature infants, spinal anesthesia in, 278, 281–282 Preservative-free 2% lidocaine, 124–125 Pressure palsy, 350, 353 Presurgical operating room time, 379 Prilocaine general questions about, 3–4 intravenous regional anesthesia using, 87–88 Procaine, 3–4 Proinflammatory cytokines, 67 Pronator teres, 9

Hadzic_Index_p413-428.indd 424

Propofol COX 2 affected by, 65, 67 ketamine versus, 337–338 midazolam added to, 240–241 for seizures, 25, 373–374 Prostaglandin E1, 393–394 Prostaglandin E2, 393–394 Protein binding, in pregnancy, 23, 25 Proximal femur, 326f Pruritus opioid-related, 114, 120 after spinal anesthesia, 99, 101 PSIN. See Postsurgical inflammatory neuropathies Psoas major, 219, 220, 221f, 222f, 245–246, 249, 251 Pterygopalatine ganglion, 81, 84 Pubic tubercle, as ultrasound-guided fascia iliaca block landmark, 195, 197 Pulmonary physiology, 290, 292 Pulmonary system, lumbar neuraxial anesthesia effects on, 299–300 Pulse oximeter in disaster conditions, 338–339 finger, 338–339 Pump local anesthetic, 38–39 regional anesthetic, 38–39 P-values, 410–412 Q QSART. See Quantitative sudomotor axon reflex test Quadratus, innervation of, 9 Quadratus femoris, 207, 209 Quadratus lumborum, 219, 222, 246–247 Quadratus lumborum block complications of, 223 transversus abdominis plane block versus, 219–221, 223 ultrasound-guided, 219–223 Quantitative sudomotor axon reflex test, 360–361 Quincke, Heinrich, 3 Quincke spinal needle, 18–19, 43–44 R Radial nerve anatomy of, 181–182 general questions about, 177–178 questions regarding, 8–9 tourniquet-related injury to, 359, 361 ultrasound-guided block of, 182–183 Radical mastectomy, 225–227 Randomized clinical trial, 409–410 Rectus sheath block, 220, 222 Recurrent laryngeal nerve lidocaine block of, 81, 83 vocal cord innervation by, 82, 84 Refraction, 143–144 Regional anesthesia acute compartment syndrome and, 307–308 in acute pain management, 391–392 assessments for, 274–275

in austere environmental medicine, 333–335 for awake endotracheal intubation, 81–83 during awake fiberoptic intubation, 82, 85 beneficial effects of, 4 benefits of, 309–310 in burn patients, 325, 327 cancer metastasis and, 66, 68, 71–72 cancer recurrence and, 71–73 for cardiac surgery, 329–332, 383–384 cardiovascular disease and, 295–298 catheters for, in intensive care unit, 317–318 central neuraxial, congenital heart conditions and, 295–296 before cesarean delivery, 264, 268 complications of in children, 277–278 prevention of, 309–310 consent for, 363–365 continuous, 3 in coronary artery disease, 295, 297 in critically ill patients, 317–318 in disaster conditions, 337–338 documentation for, 363–365 economic benefits of, 381–382 for elbow dislocation reduction, 343–344 in elderly, 289–292 in emergency department, 343–345 epidural hematoma caused by, 355–356 equipment for, 43–47 functional outcome after surgery affected by, 383 general anesthesia versus, 381–382 infections related to, 63–64 in intensive care unit, 317–318 for intrathoracic surgery, 381–382,383–384 intravenous, 87–90 for joint replacement surgery, 383 local anesthesia and, 59–60 monitoring for, 363–365 nerve injury after, 355–356 neurologic complications of, 355–358 in neurologic disease, 303–306 in obese patients, 299, 301 for ophthalmic surgery, 239–241 oral and maxillofacial, 233–235 outcomes improved using, 309–310 pathways for, 59–61 patient-controlled, 318 in pediatric patients, 273–275 perioperative outcome improvements related to, 381–382 post-anesthesia care unit costs affected by, 379–380 for postoperative care in children, 274–275 postoperative cognitive dysfunction and, 383 in pregnancy, 296–298 priorities with, 309–310

19/04/19 1:40 PM

Index resident confidence in, 403, 404f for rib fractures, 343–344 spinal epidural abscesses secondary to, 355–356 statistical methods for research in, 409–412 in systemic disease, 299–302 teaching of, 403, 404f for thoracic surgery, 329–332, 383–384 training and development of skills, 274–275 in trauma patients, 325–327 ultrasound-guided, 151–153, 363–364 writings about, 4 Regional anesthetic pump, 38–39 Regulatory T cells, 65, 67 Remifentanil, 240 half-life of, 321 for sedation during awake endotracheal intubation, 81, 84 Renal disease, chronic, 299–300 Res ipsa loquitur, 351 Residency Review Committee, minimum regional anesthesia procedures stipulated by, 403 Residents epidural block clinical competence by, 403 inadequate training of, 403 nerve block training in, 403, 404f training programs for, 403 Resolution, ultrasound, 143, 145 Respiratory depression fentanyl-induced, 387–388 ketamine-induced, 397–399 Retrobulbar block, 239–240 Reverberation artifact, 148–149 Rheobase, 53, 55 Rib fractures general questions about, 325, 327 regional anesthesia for, 343–344 Rivaroxaban, 225, 227, 313, 315 Ropivacaine bupivacaine versus, 37–38 caudal epidural block, 123, 125 in children, 278 general questions about, 112, 117, 123–125 history of, 3–4 optimal concentration of, 37–38 for sciatic nerve block using Labat’s posterior approach, 11–12 S SABER-bupivacaine, 28–29 Sacral atresia, 278 Sacral canal, 123–124 Sacral cornea, 279f Sacral curvature, 123–124 Sacral foramina, 124 Sacral hiatus in children, 277–278 general questions about, 207, 209, 256, 258f Sacral plexus, 326f

Hadzic_Index_p413-428.indd 425

Sacrococcygeal membrane, 279f Saddle block, 100,102 Sample size, 410–411 Saphenous nerve anatomy of, 8–9, 205, 215 localization landmarks for, 215–216 Saphenous nerve block sciatic nerve block and, 205–206 ultrasound-guided, 205–206 Saphenous vein, 215–216 Sartorius muscle, 190f, 192, 195–196, 198 Scalp block, 157, 159 SCANNING, 148–149 Scapula inferior angle of, 93–94, 95f, 110, 115 spine of, 110, 115 Scapular plane, 94 Scattering, 143–144 Schleich, Karl Ludwig, 4 Sciatic nerve anatomy of, 192, 210f, 213–214 innervation by, 208, 210 popliteal sciatic nerve block of, 213–214 questions regarding, 8–9 tourniquet-related injury to, 359, 361 ultrasonography of, 210f Sciatic nerve block anterior approach, 207–211 Labat’s posterior approach to, 11–12 popliteal approach, 208, 211 posterior approach, 207–209, 208f saphenous nerve block with, 205–206 subgluteal approach, 208–210 transgluteal approach, 207, 208–209, 211 ultrasound-guided, 207–211 Scoliosis, 255, 258 neuraxial procedure in, 94, 96 Sedation for eye blocks, 240–241 nitrous oxide, 343–344 remifentanil for, during awake endotracheal intubation, 81, 84 Seizures local anesthetic systemic toxicity as cause of, 267 local anesthetics as cause of, 24–25 management of, 24–25 prevention of, 264, 268 propofol for, 373–374 Semimembranosus, 190f Separate needle technique, 129, 131 Serratus anterior block, 225–229, 228t Serum tryptase, 113, 119 Sevoflurane, natural killer cell activity affected by, 65, 67 Shadowing artifacts, 147–149 “Shamrock” method, for lumbar plexus block, 245–247, 249, 251–253 Shear stress, 359, 361 Shivering, after spinal anesthesia, 99, 101 Short-acting anxiolytic agents, 310 Short-acting local anesthetics, 49, 51 Shoulder abduction of, 9 dislocation of

425

interscalene nerve block versus intravenous sedation for, 325, 327 peripheral nerve block for, 344–345 innervation of, 8, 10 Single-shot spinal anesthesia, combined spinal-epidural anesthesia versus, 129–132 Sise, Lincoln, 3–4 Site infections, from continuous peripheral nerve blocks, 333–335 Sitting position, epidural catheter placement in, 112, 117 Small saphenous vein, 215–216 Sneezing, 239–240 Sodium bicarbonate, local anesthetics and, 112–113, 117–118 Sodium channels structure of, 23–24 voltage-gated, 23–24 Sodium currents, 31–32 Soma, 7 Spinal analgesia for cardiac surgery, 329, 331 neuraxial hematoma formation risks, 329, 331 Spinal anesthesia accidental total, 100–101 for cesarean delivery, 3, 17–18 in children, 277–282, 282f complications of, 3 dosage of, 281t failed, 103–106 general questions about, 100–102 hearing loss after, 99, 101 history of, 99, 101 hypotension after, 99, 101 in infants, 283–284 in lateral position, 282f meningitis after, 99, 101 nausea and vomiting during, 99, 101 in neonates, 283–284 in obstetric patients, 101 obturator nerve block added to, 203 paramedian approach to, 100–101 patient positioning for, 282f in pregnancy, 264, 268 in premature infants, 278, 281–282 sensory block of, 100–101 in sitting position, 282f for total knee arthroplasty, 381–382 Spinal block at fourth interspace, 105f inadequate duration of, 104–105 insufficient height for surgery, 104–105 patchy, 104–105 Spinal cord anatomy of, 100–101 blood supply to, 111, 116 level of termination, 93–94 needle trauma to, 355–357 vascular system of, 99, 101 Spinal cord dysfunction, neuraxial anesthesia and, 355–356

19/04/19 1:40 PM

426

Index

Spinal cord injuries autonomic dysreflexia in, 304–306 corticosteroids for, 356, 358 general questions about, 303, 305 traumatic, corticosteroids for, 356, 358 Spinal dura mater, 139 Spinal epidural abscesses general questions about, 114, 119 regional anesthesia as cause of, 355–356 Spinal epidural hematoma diagnosis of, 367–368 general questions about, 367–368 imaging of, 367–368 incidence of, 368 neurologic recovery after, 367–368 onset of, 367–368 risk factors of, 367–368 Spinal meninges, 17–19, 123–124 Spinal needles cerebrospinal fluid aspiration through, 103–104 design of, 18–19, 43–44 manufacturers of, 139f Spinal nerves anatomy of, 7, 9 general questions about, 219, 221f thoracic, 7, 9 Spinal puncture, 17–18 Spinal stenosis general questions about, 18–19, 350, 353 local anesthetic systemic toxicity in, 303, 305 Spinal subdural hematoma, 367 Spine, paramedian sagittal views of, 255–259, 257f Spine of scapula, 110, 115 Sprotte needle, 18–19, 113, 119 Src protein tyrosine kinase, 66, 68 Statistics, 59–60, 409–412 Sterile gown, 63–64 Sternocleidomastoid muscle, 165–166 Stimulating catheters, for continuous peripheral nerve blocks, 50–51 Stimulation current, 53, 55 Stress response general questions about, 299–301 peripheral nerve block and, 325, 327 Stump allodynia, 398 Subarachnoid injections, 239–240 Subarachnoid space, local anesthetic spread in, 100–101 Subdural block, 18–19, 100,102 Subdural hematoma, spinal, 367 Subdural space, 18–19, 93, 96 Subgluteal posterior approach, to sciatic nerve block, 208–210 Substance P, 31–32, 393–394 Sub-Tenon’s block, 239–240 Sufentanil, 129, 131 Superficial cervical plexus block of, 163–165 description of, 7

Hadzic_Index_p413-428.indd 426

Superficial peroneal nerve innervation by, 325, 327 ultrasound identification of, 215–216 Superior laryngeal nerve block illustration of, 83f landmarks for, 81, 83 ultrasound-guided, 81, 83 Supraclavicular brachial plexus block assessment of, 43, 45 dyspnea after, 169, 171 for elbow dislocation, 343–344 in elderly, 289–290 equipment for, 169–170 general questions about, 43, 45 multiple injections in, 169, 171 nerve injury rates with, 11, 13 patient positioning for, 169, 171 pulmonary physiology affected by, 290, 292 for shoulder dislocation, 344–345 ultrasound-guided, 169–171, 170f Suprainguinal approach, for fascia iliaca block, 196, 198–199 Supraperiosteal injection, 234–235 Suprascapular nerve block, ultrasoundguided, 167–168 Supraspinous ligament general questions about, 112, 117 ligamentum flavum versus, 93, 95 Sural nerve, 8–9, 213–216 Surgery chronic pain after, 383 functional outcome after, 383 Sympathectomy, from thoracic epidural anesthesia, 329, 331 Sympatholysis benefits of, 329, 331 high thoracic epidural analgesia as cause of, 329–331 “Syringe swap,” 104–105 Systemic disease, regional anesthesia and, 299–302 Systemic toxicity, to local anesthetics, 24–25, 27, 35–36 Systolic blood pressure, epinephrine effects on, 363–364 T Tachycardia, 297 TAM. See Tumor-associated macrophages Teeth, regional anesthesia of, 233–234 TEG. See Thromboelastography Tetracaine, 4 Th2 cells, 65, 67 Thoracic epidural analgesia high awake cardiac surgery using, 329–330 heart rate after, 329–330 sympatholysis caused by, 329–331 thoracic paravertebral block versus, 330, 332 Thoracic epidural anesthesia for abdominal surgery, 109, 111, 114, 117

in chronic obstructive pulmonary disease, 300–301 epidural hematoma formation associated with, 329–330 general anesthesia and, 329, 331, 383–384 high, 295–296 pulmonary effects of, 329, 331, 331t sympathectomy from, 329, 331 Thoracic paravertebral block general questions about, 330, 332 pectoralis nerve block versus, 225–226 thoracic epidural analgesia versus, 330, 332 Thoracic spinal nerves, 7, 9 Thoracic surgery, regional anesthesia for, 329–332, 383–384 Thoracotomy surgery, epidural anesthesia for, 381–382 Thromboelastography, 313–314 Th1/Th2 cell ratio in cancer metastasis, 65, 67 propofol effects on, 65, 67 Thumb adduction of, 9 dermatomes of, 111, 116 Tibial fracture, acute compartment syndrome secondary to, 325–327 Tibial nerve anatomy of, 9 block of, for foreign objects in foot, 343–344 foot innervation by, 215–216 innervation by, 325, 327 Tibial osteosynthesis, 213–214 Ticagrelor, 313–314 Ticlopidine, 313–314 Tight junctions, 15–16, 138f Time-gain compensation, 143, 145 Tirofiban, 313–314 Tissue harmonic imaging, 147–148 TIVA. See Total intravenous anesthesia Tmax, of SABER-bupivacaine, 28–29 Toe surgery, nerves omitted in, 215–216 Tolerance definition of, 319–320 development of, 319–321 opioid, 319–322 Topical anesthesia for awake endotracheal intubation, 81–83 emergency, 343–344, 344t Topical local anesthetics, in children, 273, 275 Total hip arthroplasty anterior approach to, 195, 197 epidural anesthesia and analgesia for, 109, 114 local infiltration analgesia for, 77–79 lumbar plexus block for, 249, 251 Total intravenous anesthesia, 304, 306 Total knee arthroplasty combined spinal-epidural anesthesia for, 130, 132 epidural anesthesia for, 111, 117

19/04/19 1:40 PM

427

Index femoral catheters in, 389–390 femoral nerve block for, 189, 192 local infiltration analgesia in, 77–79 in multiple sclerosis patient, 303, 305 patient satisfaction after, 290–291 regional anesthesia for, 303, 305 saphenous nerve block for pain management after, 205–206 spinal anesthesia for, 381–382 Tourniquet paralysis, 361 Tourniquets description of, 4, 87–89 pneumatic, tissue damage caused by, 359, 361 Toxicity, of local anesthetics, 24–25 Trabecular arachnoid sheath, 18–19 Training programs, for residents, 403 Tramadol general questions about, 31–32, 67, 387–388, 393 mechanism of action, 285–286 Transducers, ultrasound general questions about, 147, 149 for pectoralis nerve block, 225, 227 for superficial cervical plexus block, 165–166 Transesophageal echocardiography, for cardiac surgery, 329–330 Transgluteal posterior approach, to sciatic nerve block, 207, 208–209, 211 Transient facial paralysis, 158–159 Transient femoral nerve palsy, 220, 223 Transient root irritation syndrome, 17–19 Transmuscular approach, to quadratus lumborum block, 219, 221f, 222 Transverse abdominis plane blocks, local anesthetic systemic toxicity after, 373, 375 Transverse in-plane approach, for lumbar plexus block, 245, 247 Transverse interspinous, 256 Transverse scan, of lumbar plexus, 250, 253 Transverse spinous process view, 255–256, 258 Transversus abdominis plane block quadratus lumborum block versus, 219–221, 223 ultrasound-guided, 219–223 Trauma chronic pain after, 326–327 hip fractures, 325–326 regional anesthesia in, 325–327 rib fractures, 325, 327 Treg cells. See Regulatory T cells Triamcinolone, 124–125 Triceps, 190f Tricyclic antidepressants, 391–392 Trident sign, 246–247, 250–251, 253–254 Trigeminal ganglion, 157–158 Trigeminal nerve mandibular division of, 233, 235 maxillary division of, 233–234 Trigeminal nerve block, 157–158 Tsui test, 46 Tuffier’s line, 94

Hadzic_Index_p413-428.indd 427

Tumescent anesthesia, 3–4 Tumor necrosis factor-α-induced Src activation, 66, 68 Tumor-associated macrophages, 65, 67 Tuohy, Edward, 3 Tuohy needle, in children, 277–278 Turnover time, 379–380 Type I error, 409, 411 Type II error, 59–60, 409, 411 U UGRA. See Ultrasound-guided regional anesthesia Ulnar nerve general questions about, 177–178 injury to, 359–360 questions regarding, 8–9 ultrasound-guided block of, 182–183 wrist block, 185, 186 Ultrasound anterior complexes, 256, 258–259 B-mode, 143–144 catheters on, 147–148 for central neuraxial blocks, 255–259 coupling medium for, 143–144 definition of, 143–144 focus adjustments, 143, 145 local anesthetic systemic toxicity reductions using guidance by, 363–364 long-axis view, 277, 279–280f monitoring uses of, 363, 365 needle localization using, 50, 52 nerve identification and preservation using, 11, 13 neuraxial blockade uses of, 103–104 peripheral nerve stimulation versus, 403 physics of, 143–145 poor resolution on, 12 posterior complexes, 256, 258–259 propagation speed of, 143–144 resolution of, 143, 145 SCANNING acronym for, 148–149 short-axis view, 277, 279–280f transducer for, 147, 149 traumatic procedures associated with, 256, 258 Ultrasound gel, 143–144 Ultrasound image artifacts on, 147–149 of femoral nerve, 190, 193 optimization of, 147–149, 151–152 PART acronym for, 151–152 shadowing artifacts on, 147–149 Ultrasound-assisted caudal block, 277, 279–280, 279–280f Ultrasound-guided eye blocks, 239–240 Ultrasound-guided nerve blocks algorithm for, 54f ankle, 215–216 ankle block, 147, 149 axillary brachial plexus block, 177–179 axillary nerve block, 167–168 benefits of, 151–152, 153 bioeffect caused by, 143–144

brachial plexus block, 151–152 cervical plexus block, 163–166 of ear, 157, 159 at elbow, 181–183 of face, 157–160 fascia iliac block, 195–199 femoral nerve block, 189–194 greater occipital nerve, 157, 159 heat generation in, 143–144 infection control during, 63–64 infraclavicular brachial plexus block, 173–176 interscalene brachial plexus block, 44, 151–152, 167–168 lateral femoral cutaneous nerve block, 201–202 lumbar plexus block. See Lumbar plexus block mandibular nerve, 158–159 maxillary nerve, 158–159 nerve injury and, 349–350 of nose, 157–159 obturator nerve, 203–204 peripheral nerve blocks, 15–16, 35–36, 43–45, 49, 51 popliteal sciatic, 213–214 quadratus lumborum, 219–223 saphenous nerve, 205–206 scalp block, 157, 159 sciatic nerve, 207–211 short-axis, in-plane approach for, 151–152 superior laryngeal nerve, 81, 83 supraclavicular brachial plexus block description of, 169–171, 170f in elderly, 289–290 suprascapular nerve block, 167–168 transversus abdominis plane, 219–223 trigeminal nerve block, 157–158 at wrist, 185–186 Ultrasound-guided regional anesthesia, 363–364 Unmyelinated fibers, sodium channels in, 23–24 Upper extremity, intravenous regional anesthesia of, 87–88 Upper trunk, 7, 9 Urinary retention, postoperative, 99, 101 U-shaped dural flap, 17–18 V Vagus nerve, 81, 84 Variance, 409, 411 Varicella zoster virus, epidural catheter placement in, 110, 115 VAS. See Visual analog scale Vascular smooth muscle, dilatation of, 24–25 Vascular system, of spinal cord, 99, 101 Vasoconstriction cocaine-induced, 25, 82, 85 epinephrine-induced, 51 Vasodilation, 297 Vasopressin, for local anesthetic systemic toxicity, 374, 376

19/04/19 1:40 PM

428

Index

Vastus medialis, 190f Vazirani-Akinosi closed-mouth mandibular block technique, 233–234 Velocity error artifact, 148–149 VerifyNow, 313–314 Vertebra prominens, 110, 115 Vertebral canal, 255, 258 Vertebral column deformities, 255, 258 Visceral pain general questions about, 220, 222 in labor, 264, 267 Visual analog scale, 45 Vital capacity, 329, 331

Hadzic_Index_p413-428.indd 428

Vitamin K antagonists, coagulation effects of, 313–314 Vloka’s sheath, 213 Vocal cords, anesthetic block of, 81–82 Voltage-dependent calcium channels, 393–394 Voltage-gated sodium channels, 23–24 Vomiting, nitrous oxide sedation as cause of, 343–344 von Willebrand defects, 313–314

W Warfarin, coagulation effects of, 313–314 Whitacre needle, 18–19 Withdrawal definition of, 320 opioid, 319–321, 320f Wrist, 181–182 Wrist block median nerve transducer, 185–186 nerve injury rates with, 11, 13 ulnar nerve, 185, 186 ultrasound-guided, 185–186

19/04/19 1:40 PM